Download as pdf or txt
Download as pdf or txt
You are on page 1of 346

T SCH TON HC TR TH K 21

001

L THUT S CP
CU CC S
TC GI: W. SIERPINSKI
Bin tp: A. Schinzel
Khnh Nguyn dch t bn in ln hai nm 1988 cun Elementary theory of
numbers ca Sierpinski. Bn tho hon thnh ln th nht thng 10/2012 ti
Si Gn Ch Ln. In th nghim 300 cun trn giy thng theo bn chnh sa
thng 12/2012. Ngoi ra c in thm 30 bn trn giy trng. In xong thng 1/2013.
Bn dch (mang m s 001) thuc chng trnh xy dng t sch ton hc tr th
k 21 ch trng bi {K@} v do dch gi nm bn quyn

LU HNH NI B

T SCH TON HC TR TH K 21
001

L THUT S CP
CU CC S
TC GI: W. SIERPINSKI
Bin tp: A. Schinzel
Khnh Nguyn dch t bn in ln hai nm 1988 cun Elementary theory of
numbers ca Sierpinski. Bn tho hon thnh ln th nht thng 10/2012 ti
Si Gn Ch Ln. In th nghim 300 cun trn giy thng theo bn chnh sa
thng 12/2012. Ngoi ra c in thm 30 bn trn giy trng. In xong thng 1/2013.
Bn dch (mang m s 001) thuc chng trnh xy dng t sch ton hc tr th
k 21 ch trng bi {K@} v do dch gi nm bn quyn

LU HNH NI B

L thuyt s cp ca cc s c l l mt trong nhng ch


tt nht xy dng nhng hiu bit ton hc u
tin. N yu cu rt t cc kin thc m u v cc ch
ca n l rt quen thuc v r rng. Cc lp lun c s
dng cng rt n gin v khng qu nhiu. Hn na n
l ch duy nht trong ton hc c hnh thnh mt
cch t nhin bi s t m ca con ngi. - G.H.Hardy

LI GII THIU CA TC GI
Waclaw Sierpinski
Ngy nay cc nhnh mi pht trin trong ton hc thng c t tn theo nhng cch gi
truyn thng tr nn quen thuc trc . Tuy nhin nhng tn gi nh vy nhiu khi khng
thc s cho bit mt cch chnh xc s pht trin cng nh cc ch m n cp ti. iu ny
cng xy ra vi l thuyt ca cc s. L thuyt ca cc s (cng vi nhng s lin h vi cc ngnh
khoa hc khc ca n) l mt lnh vc cha ng nhng ch v phng php c v tr c bit
trong rt nhiu nhnh ton hc khc nhau.
Tn gi L thuyt ca cc s ph hp vi mt l thuyt i cng nghin cu v cc s v cc
dng m rng ca n. Chng hn bt u t s nguyn, ta c cc s hu t, s thc v s phc. T
cc loi s khc nhau ta xy dng nhng php ton (cc ton t) trn cc s . Tuy nhin y
ng ra l S hc cao cp. Nguyn nhn l v L thuyt ca cc s thng ch lin quan ti tnh
cht ca cc s nguyn trong khi S hc cao cp s dng ti c cc l thuyt i s v cc ton t.
Tt nhin l thuyt ca cc s s khng ch xoay quanh cc s nguyn v trn thc t c rt nhiu
tnh cht ca cc s nguyn c pht hin v chng minh da trn s tm hiu cc s v t v cc
s phc. Hn na c rt nhiu cc nh l v cc s nguyn c th c chng minh theo cch
n gin nu ta khng ch s dng cc s v t v cc s phc m cn s dng ti gii tch v l
thuyt v cc hm. L thuyt ca cc s vi s kt hp vi mt s ch ca gii tch hnh thnh
nn b mn S hc gii tch. B mn ny c s khc bit vi L thuyt s cp ca cc s im
cn bn l n s dng ti khi nim gii hn. Tuy vy mc d ch chnh ca cun sch ny l
L thuyt s cp ca cc s nhng vn s c mt s ng dng ca S hc gii tch c xt ti.
Cun sch c xy dng da trn hai cun sch khc ca ti trong nhng nm 1914 v 1959 l
1. Teoria Liczb (L thuyt cc s), n bn ln th nht, Warszawa 1914; n bn ln th
hai, Warszawa 1925; n bn ln th ba, Warszawa-Wroclaw 1950 (544 trang)
2. Teoria Liczb, Phn II, Warszawa 1959 (487 trang).
minh ha cho s pht trin L thuyt ca cc s trong mt thp k va qua ch cn nhc li
rng s nguyn t ln nht c tm ra vo nm 1950 l s 2127 1 (s ny c 39 ch s) trong khi
ngy nay s nguyn t ln nht tm c l s 211213 1 (s ny c 8376 ch s). Vo nm 1950
ta mi ch bit 12 s hon ho trong khi ngy nay ta tm c 23 s nh th.
Trong cun sch ny ti s trnh by rt nhiu kt qu c bit ca L thuyt s cp ca cc s
c cng b trong nhng nm gn y bi cc nh ton hc ti t rt nhiu quc gia khc nhau.
Tin s A.Hulanicki l ngi dch bn tho cun sch sang ting Anh. Tin s A.Schinzel l
ngi chun b ph lc v thm vo rt nhiu ngh v ghi ch lin quan ti cc kt qu c
cng b gn y. Tin s A.Makowski l ngi c cc chng minh. Ti c bit cm n cc ng
nghip ni trn. Ti cng cm n Bin tp vin L.Izertowa ti t Nh xut bn khoa hc Ba Lan,
ngi chun b rt nhiu cho bn in ca cun sch ny.
Ba Lan
1963

LI NI U CA NGI BIN TP
CHO BN IN LN TH HAI

Andrzej Schinzel
Trong qu trnh bin tp cun sch Elementary theory of numbers ca Sierpinski chun b
cho ln in th hai, ti (Schinzel) gi nguyn cc ch v th t trnh by m tc gi la
chn. Trong khong 20 nm k t khi bn in ln u ra i th c rt nhiu cng trnh nghin
cu mi c thc hin. Cc cng trnh cho nhiu cu tr li cho cc cu hi c t ra
trong bn in ln th nht. V vy ti cho rng nhim v ca mnh l b sung v hon chnh li mt
s mc v lm y hn cc trch dn, ng thi sa li mt s li sai.
thc hin cng vic ny ti nhn c s h tr ca cc ng nghip Jerzy Browkin v
Andrzej Makowski. Ti cm n s cng tc ca h. Ti cng nhn c nhng gi v nhng s
chnh sa t cc nh ton hc khc l cc Gio s John Brillhart, Eckford Cohen, Tin s
Waldemar Gorzkowski, cc Gio s Erich Michalup, M.V.Subbarao, Antoni Wakulicz v Gio s
Gregory Wulczyn. Bin tp vin Krystyna Regulska ti t Nh xut bn khoa hc Ba Lan kim
tra cc yu t k thut ca bn tho, trong c bng tra cu danh sch cc nh ton hc c
trch dn.
Ba Lan
Thng 2 nm 1985

LI GII THIU CA NGI DCH


Khnh Nguyn
Ti cn nh nhng ngy hc u tin thi cp 2 khi bt u tm hiu v phng trnh nghim
nguyn th mt ai ti cng cc bn hc gii phng trnh x n y n z n vi n 2 v ni
rng ngi no gii c s c coi l mt nh ton hc thc th. S n gin ca phng
trnh v danh hiu nh ton hc thc th khin chng ti cm thy v cng ho hng. Chng ti
trn thc t s dng rt nhiu giy nhp v nhiu bui tnh ton trin min m khng dn ti
kt qu. S ho hng ban u nhanh chng chuyn thnh s tht vng nng n. Tnh trng ny
cn tr nn t hn khi chng ti thm ch cn khng gii quyt c trng hp ring khi n 3 .
Trong trng hp da vo phn tch quen thuc x3 y3 ( x y)( x 2 xy y 2 ) z 3 c th suy
ra mi nhn t trong biu thc gia u l ly tha bc ba vi cc iu kin b sung v chng ti
cm gic mnh c th xy dng c mt nghim mi nh hn nghim ban u (nu c) v c
nh th. Tuy nhin cui cng th cc tnh ton chi tit vn khng c hon thin. S tht vng
khin chng ti chn nn v thm ch cn khng tr li nghin cu g thm v phng trnh
Pythagoras x 2 y 2 z 2 v coi rng y l mt trng hp tm thng khng cn xt ti. Mt khc
cc ti liu v ton nhng nm trong trng hc l khng phong ph do chng ti c cm
gic mnh ging nh vn ang chi vi nhng bi ton n l v khng c g c bit.
Ch ti khi ln cp 3 th ti mi c tip cn vi th vin thc s ca mt trng i hc (ti hc
lp 10 ti khi chuyn Ton Tin i hc Khoa hc t nhin v do c s dng gn nh ton
b h thng th vin ca trng i hc Quc Gia H Ni). Sau khi tra cu trong hp phch th ti
chn mt cun sch tng i dy v c ta ting Vit tng i d hiu l L thuyt s cp
ca cc s. Cng cn ni thm l ti c chn cun sch c ta n gin v ngy ti cha
c tho sch ton bng ting Anh.
Tuy nhin rt bt ng l mt cun sch c ta c v s cp nh vy li c ring mt mc ni
v phng trnh x3 y 3 z 3 m chng ti loay hoay ht c thi cp 2. Theo th phng
trnh ny l khng c nghim no ngoi cc nghim tm thng v cun sch thm ch cho ti
hai chng minh cho kt qu . Trong mt chng minh da trn tnh ton v bin i s cp
cng vi phng php xung thang, chng minh kia da trn cc s nguyn phc. S ho hng
nhng ngy cp 2 thc s tr li v hai chng minh ny rt gn gi vi nhng tng ban u
m chng ti c gng pht trin nhng khng em li kt qu. Tt nhin ngay sau ti
nhn ra tuy tng ban u l ging nhau nhng chng ti khng c nhng pht trin mang
tnh quyt nh. Khong cch gia cc tnh ton khng c kt qu v mt chng minh trn vn
trong trng hp ny nm cc nim v cc s nguyn phc, v cc chun ca s nguyn phc,
tnh chia ht ca s nguyn phc (nhng tng ca Gauss) ch khng ch n thun l mt vi
ng thc mang tnh cht k thut no .
Mt im th v l ngay sau th ti nhanh chng b cun ht bi mt vn khc. Cun sch
ny thc s l mt ti liu rt c gi tr vi v s cc nh l, cc kt qu, cc chng minh, trch dn
cc nh ton hc v mi lin kt gia cc bi ton. T vic c v cc phng trnh Diophante c
dng quen thuc mt cch c h thng ti chuyn qua c v cc s nguyn phc v nhanh chng
tip xc vi chng minh lut tng h bc hai. Sau l cc m u v l thuyt ng d v cc
nh l cng chng minh p ca Jacobi v tng bn bnh phng. Nhng n tng nht c l
l cc nghin cu v s xut hin cc s nguyn t trong mt cp s cng cho trc. Cc c
lng v s lng cc s nguyn t c bit n tng. S phong ph trong cc nh l cng vi
bng danh sch dy c cc nh ton hc c trch dn khin ti ln u tin c cm gic rng
ton hc l rt rng ln, xuyn sut v c ngha hn mt phng trnh ring r rt nhiu.
Sau ny trong qu trnh tip tc c v hc ln ti bit rng phng trnh x n y n z n v nh
l cui cng ca Fermat mi ti vi nm sau (k t khi chng ti nhn c cu ) mi c gii
bi Andrew Wiles. Chng minh hon thin c Wiles cng b nm 1995 v ti i hi Ton hc
th gii 1998 th Wiles c trao huy chng danh d cho chng minh (huy chng Fields

gii hn tui nhn gii l 40). Hn na gi tr ca vic gii phng trnh ny khng thc s
nm kt qu m li chnh l nhng l thuyt p m trong qu trnh tm li gii cho n cc
nh ton hc xy dng nn. l cc l thuyt v cc dng modular, l thuyt v phng trnh
elliptic v cc ngnh khoa hc hin i m chng ti thi cha h nghe ni ti v cng khng
th hnh dung ni, chng hn l hnh hc i s s hc.
Ti cho rng cun sch ny l mt ti liu tt m ngay c cc bn hc sinh cp 2 cng c th bt
u c m khng cn mt s chun b no trc v mt kin thc. Hn na tinh thn ct li
trong cc php chng minh cng chnh l du vt ca s p ca ton hc m cc bn nn tip
xc cng sm cng tt. Theo , sau mt thi gi chun b th cui cng th ti dch ton b cun
sch ny sang ting Vit. V y l bn dch cun sch . Tc l cun Elementary theory of
numbers ca nh ton hc Wacaw Sierpinski (1882-1969). Cun sch ny c in ln th nht
vo nm 1964 (ngha l vi nm trc khi tc gi qua i) v c in ln th hai nm vo nm
1988 vi s bin tp ca nh ton hc Andrzej Schinzel. Bn dch ny da trn bn in ln th hai.
Theo ti cc bn hc sinh cp 2 v cp 3 s c th c ton b cun sch ny mt cch tng i
thoi mi. Hn na trong cun sch ny th ngoi s phong ph v cc kt qu th cc kin thc s
cp v l thuyt s cng c trnh by y vi trnh t rt hin i. Do cng c th s dng
cun sch nh l mt gio trnh nng cao v s hc dnh cho cc bn hc sinh kh gii.
Chng trnh by v cc phng trnh Diophante l mt chng tuyt hay v trong cc phng
php v tng c cha ng ngay trong cc li gii v cc bi th c sp xp theo trnh
t c tnh gn kt rt cao. Tuy nhin trong cun sch ny li khng cp ti chng minh ca
Matijasevich v vic khng tn ti phng php tng qut gii cc phng trnh Diophante
tng qut (bi ton Hilbert s 10). iu ny cng d hiu v nh l ny c trnh by nm 1970,
ngha l mt nm sau khi Sierpinski qua i.
Sierpinski c bit ti vi nhng cng hin xut sc trong l thuyt tp hp, c bit l v tin
chn v gi thuyt continuum. C th ng chng minh c trong h tin Zermelo-Fraenkel
th t gi thuyt continuum dng m rng c th suy ra tnh ng n ca tin chn. Bn cnh
mc d Cantor l cha ca l thuyt tp hp nhng Sierpinski li l ngi u tin ging dy
v l thuyt tp hp bc i hc (1909). ng cng b 724 bi bo v 50 cun sch. C ba hnh
fractal c t theo tn ng l tam gic Sierpinski, thm Sierpinski v ng cong Sierpinski.
ng cong Sierpinski c ng dng quan trng trong vic gii quyt bi ton ngi a th v l
c s xy dng ng cong lin tc ph kn hnh vung n v. Sierpinski ging dy ti Lww
t nm 1908 ti 1914. Lww l ni (sau vi nm) trng phi Banach ni ting ra i. Trng
phi Banach ra i nm 1920 l mt trong mt s trng phi quan trng i vi vic pht trin
v hon thin gii tch hm hin i vo nm 1932.
Si Gn
Thng 12 nm 2012

MC LC
CHNG 1
TNH CHIA HT V PHNG TRNH BT NH BC MT
1.
2.
3.
4.
5.
6.
7.

Tnh chia ht
Bi s chung nh nht
c s chung ln nht
Cc s nguyn t cng nhau
Quan h gia c s chung ln nht v bi s chung nh nht
nh l c bn ca s hc
Cc cng thc a1 , a2 ,..., an1 at , a2 ,..., an , an1

1
3
3
4
5
5

v a1 , a2 ,..., an1 a1 , a2 ,..., an , an1

8. Quy tc tnh cc c s chung ln nht ca hai s


9. Biu din s hu t thnh lin phn s
10. Dng tuyn tnh ca c s chung ln nht
11. Phng trnh bt nh m bin bc 1
12. nh l s d Trung Hoa
13. nh l Thue
14. Cc s khng c c s chnh phng

9
11
12
14
17
18
19

CHNG 2
GII TCH DIOPHANTE BC HAI V CAO HN
1. Gii tch Diophantine mt bin
2. Cc phng trnh Diophante nhiu bin
2
2
2
3. Phng trnh x y z

21
22
22

4. Nghim t nhin ca phng trnh x y z vi x y 1


5. Cc tam gic Pythagoras c cng din tch
6. V cc bnh phng c tng v hiu u l bnh phng
4
4
2
7. Phng trnh x y z
8. V ba bnh phng c tng i mt l bnh phng ng
9. Cc s iu ha
2
2
2
2
10. Phng trnh x y z t
11. Phng trnh xy zt

26
29
32
36
38
40
42
44

12. Phng trnh x x y y z

47

14. Phng trnh x y 2 z


3

13. Phng trnh x 9 x y 27 y z


4

48
49

15. Phng trnh x y az vi a 2


16. S tam gic
17. Phng trnh x2 Dy 2 1

52
53
56

18. Phng trnh x k y vi k nguyn


19. Mt s phng trnh m

62
67

CHNG 3
S NGUYN T
1. S nguyn t v phn tch s t nhin thnh tch cc s nguyn t
2. Sng Eratosthenes v bng cc s nguyn t
3. Hiu ca cc s nguyn t lin tip

71
73
74

4.
5.
6.
7.
8.
9.

Gi thuyt Goldbach
Cc s nguyn t lp thnh cp s cng
Cc s nguyn t trong mt cp s cng cho trc
Tam thc Euler x 2 x 41
Gi thuyt H
Hm s x

76
78
79
80
82
84

10. Chng minh nh Bertrand (nh l Tchebycheff)


11. nh l H.F.Scherk
12. nh l H.E.Richert
13. Gi thuyt v cc s nguyn t
14. Bt ng thc ca hm x

85
91
93
94
96

20. nh l s nguyn t v cc h qu

99

CHNG 4
S CC C S V TNG CA CHNG
1. S cc c s
2. Cc tng d (1) d (2) ... d (n)

101
103

3. Cc chui vi cc h s d n

105

4.
5.
6.
7.

106
111
114
114

Tng cc c s
Cc s hon ho
Cc s bn b
Tng 1 2 ... n

8. Cc chui vi h s n

115

9. Tng ca cc hng t xc nh bi
cc c s t nhin ca mt s t nhin n
10. Hm Mobius
11. Hm Liouville n

116
117
119

CHNG 5
NG D
1.
2.
3.
4.
5.
6.
7.
8.
9.

ng d v cc tnh cht
Nghim ca cc ng d thc v h thng d y
Nghim ca a thc v nghim ca ng d thc
ng d thc bc mt
nh l Wilson v nh l Fermat nh
Cc s idonei
Cc s gi nguyn t v gi nguyn t tuyt i
nh l Lagrange
ng d thc bc hai

121
123
125
127
128
140
141
144
147

CHNG 6
HM CH EULER V NH L EULER
1.
2.
3.
4.
5.
6.
7.
8.

Hm ch Euler
Cc tnh cht ca hm ch Euler
nh l Euler
Cc s vi s m cho trc theo mt modulo cho trc
S tn ti v hn cc s nguyn t trong cp s cng nk 1
S tn ti cn nguyn thy ca s nguyn t
Thng d bc n ca mt s nguyn t theo modulo p
Cc tnh cht v ng dng ca hm ch s

151
160
161
164
165
170
174
175

CHNG 7
BIU DIN H C S TY
1.
2.
3.
4.
5.
6.

Biu din ca s t nhin trong c s ty


Biu din trong h c s m
Phn s v hn trong h c s cho trc
Biu din ca cc s hu t
S chun tc v s chun tc tuyt i
Phn s thp phn trong c s bin thin

179
182
183
185
187
188

CHNG 8
LIN PHN S
1.
2.
3.
4.
5.

Lin phn s v s hi t ca chng


Biu din mt s v t thnh lin phn s
Lut xp x tt nht
Lin phn s biu din cc cn bc hai
S dng lin phn s D gii cc
phng trnh x 2 Dy 2 1 v x 2 Dy 2 1
6. Lin phn s dng phc

191
192
195
196
205
208

CHNG 9
K HIU LEGENDRE V K HIU JACOBI

D
v cc tnh cht
p

1. K hiu Legendre

213

2.
3.
4.
5.

217
220
220
222

Lut tng h bc hai


Tnh ton k hiu Legendre
K hiu Jacobi v cc tnh cht
Lut Eisenstein

CHNG 10
CC S MERSENNE V CC S FERMAT
1.
2.
3.
4.
5.

Mt s tnh cht ca cc s Mersenne


nh l ca E.Lucas v D.H.Lehmer
S nguyn t ln nht tm c
c s nguyn t ca cc s Fermat
iu kin cn v mt s Fermat l s nguyn t

227
228
231
233
237

CHNG 11
BIU DIN CC S T NHIN THNH TNG CC LY THA BC k KHNG M
1. Tng ca hai bnh phng
2. S cch biu din thnh tng hai bnh phng
3. Tng ca hai bnh phng cc s t nhin
4. Tng ca ba bnh phng
5. Biu din bi tng bn bnh phng
6. Tng ca bn bnh phng cc s t nhin
7. Tng ca m 5 bnh phng dng
8. Hiu ca hai bnh phng
9. Tng ca hai lp phng
10. Phng trnh x3 y 3 z 3
11. Tng ca ba lp phng
12. Tng ca bn lp phng
13. Mt s tng cc lp phng c gi tr bng nhau
14. Tng ca cc trng phng

239
241
245
247
251
255
258
260
261
262
265
267
268
269

15. nh l Waring

270

CHNG 12
MT S BI TON CA L THUYT CNG TNH CA CC S
1.
2.
3.
4.
5.

Phn hoch dng tng


Biu din thnh tng ca n hng t khng m
Ma phng
nh l Schur v cc h qu
Cc s l khng c dng 2k p vi p nguyn t

273
274
274
277
281

CHNG 13
S NGUYN PHC
1. Chun ca s nguyn phc. Cc s lin kt
2. Thut ton Euclid v c s chung ln nht
ca cc s nguyn phc
3. Bi s chung nh nht ca cc s nguyn phc
4. Cc s nguyn t phc
5. Phn tch ca s nguyn phc thnh cc c s nguyn t phc
6. S cc s nguyn phc vi chun cho trc
7. nh l Jacobi v tng bn bnh phng

285
287
290
290
293
294
297

TI LIU THAM KHO

305

DANH SCH TRA CU CC NH TON HC

323

TRA CU NHANH CC CH

327

CHNG 1
TNH CHIA HT V PHNG TRNH BT NH BC MT
1. Tnh chia ht
Cc s t nhin l cc s 1, 2,... . Cc s nguyn l cc s t nhin, s 0 v cc s m 1, 2, 3,... .
S nguyn a chia ht cho s nguyn b nu tn ti s nguyn c m a bc . Khi ta vit b a v
ni b l c s ca a , a l bi s ca b . Ta vit b | a nu b khng l c s ca a . V vi mi s
nguyn b ta c 0 0.b nn mi s nguyn u l c s ca 0 . V vi mi s nguyn a ta c
a a.1 nn 1 l c s ca mi s nguyn.
Gi s x, y, z l cc s nguyn tha mn
(1)

x | y v y | z

Khi tn ti cc s nguyn t v u tha mn y xt v z yu . S v tu l mt s nguyn (v n


l tch ca hai s nguyn). V vy t z xv suy ra x | z . Vy t (1) suy ra x | z . Do c s ca
c s ca mt s nguyn th cng l c s ca s nguyn . Quan h chia ht l quan h c tnh
bc cu. Do nu x | y th x | ky vi mi s nguyn k .
D dng chng minh c s chung ca hai s nguyn cng l c s ca tng v hiu cc s .
Hn na nu d | a v d | b th vi mi s nguyn x v y ta c d | ax by. Tht vy, v d | a v

d | b suy ra tn ti cc s nguyn k v l m a kd , b ld , suy ra ax by kx ly d v lu


kx ly l s nguyn suy ra d | ax by . Quan h chia ht l quan h c tnh kt hp.
Cc cng thc a bc, a b c , a b c , a b c l tng ng. V vy cc cng thc

b | a , b | a , b | a , b | a cng tng ng vi nhau. Do nghin cu tnh chia ht gia cc


s nguyn ta ch cn nghin cu tnh chia ht gia cc s t nhin.
T nh ngha b | a ta nhn thy nu 0 | a th a 0 . Tuy nhin nu a 0 th mi c s b ca a
l khc 0 v b cng l c s ca a . V vy vi mi s nguyn a 0 th cc c s b ca a c
th sp xp thnh cc cp b, b . Do tm tt c cc c s ca mt s nguyn ta ch cn tm
cc c s t nhin ca s v b sung thm cc s i ca cc s va tm c.
Nh vy tp hp cc c s v cc bi s ca mt s l cc tp hp i xng. Mt khc vic tm cc
c s ca mt s cho trc l kh hn vic tm tt c cc bi s ca s . Tht vy, tt c cc bi
s ca s nguyn a l cc s nguyn c dng ka vi k l s nguyn ty . Cc bi s ny c sp
xp thnh dy v hn v c hai pha ..., 2a, a,0, a, 2a,... . Trong khi vic tm tt c cc c s
ca a l khng n gin. iu ny c v c bit v tp hp cc c s ca mt s nguyn cho
trc l hu hn trong khi tp hp cc bi s ca s nguyn l v hn.
Nu s t nhin a chia ht cho s t nhin d th d a . V vy tm tt c cc c s dng ca
s nguyn a th ta ch cn chia a ln lt cho cc s t nhin 1, 2,..., a v chn ra cc s m
thng s l s nguyn (php chia khng c d). Do cc php tnh ton theo cch ny l hu hn
nn v l thuyt ta c mt phng php tm tt c cc c s ca mt s nguyn cho trc. Tuy
nhin c nhng kh khn khi tin hnh tnh ton c th. Chng hn thi gian thc hin phng
293
php ny i vi s a 2 1 (c 89 ch s) l rt ln ngay c vi cc my tnh in t. Tuy
nhin ta c th tm tt c cc c s ca s 2293 (ln hn a ). S ny c ng 294 c s lp thnh
mt cp s nhn l 1, 2, 22 , 23 ,..., 2293 . Ta cng cha tm c bt k c s khng tm thng no
ca s 2163B 4 1. Hn na mc d ta bit rng c nhng c s nh vy (so snh vi Chng 10)
nhng ta cha bit s ny c tt c bao nhiu c s khng tm thng.

2 | Tnh chia ht

Trong mt s trng hp cc c s ca mt s t nhin c tm ra bng cch s dng cc my


tnh in t. Chng hn vi s 18!1 :59 108514808571661 . S dng my tnh SWAC,
D.H.Lehmer ch ra s ny c ng bn c s t nhin l 1, 226663, 478749547 v chnh n
(Gabard [1] trang 218-220). Trong Chng 4 ta s nghin cu s cc c s ca mt s t nhin.
Vn nghin cu xem mt s cho trc c phi l c s ca mt s cho trc khc hay khng l
thc s kh khn. Trong mt s trng hp ta cn ti s tr gip ca my tnh. Chng hn s
65536
1 chia ht cho m 825753601 . Trng hp ny c bit th v
dng my tnh ta bit s a 2
(xem Chng 10 mc 4). S a c 19729 ch s v vy vic vit c th s di dng thp phn
l khng kh thi. Tuy nhin ta s khng em a chia cho m quyt nh xem a c chia ht cho
m hay khng. Ta cn mt cch bin dch khc my tnh c th tnh ton c. Mt v d khc l
23471
23473
1 . S th nht c hn 107064 ch s trong khi s
s chia ht ca s 22 1 i vi s 5 2
th hai c 7067 ch s. Ta s tr li bi ton ny trong Chng 10 mc 4.
Bi tp. 1. Chng minh rng nu a v b l cc s t nhin th a !b !| a b ! .
Chng minh. Tnh cht ny l hin nhin ng nu t nht mt trong cc s a v b bng 1 v vi
mi s t nhin b ta c (b 1)! b!(b 1) suy ra 1!b!| 1 b ! . Do bi ton ng vi a b 3 .
Gi s n l s t nhin ln hn 2 v bi ton ng vi mi cp hai s t nhin c tng khng ln
hn n . Xt hai s t nhin a v b c tng bng n 1 . Ta bit bi ton ng nu t nht mt
trong hai s a v b bng 1 . Gi s a 1 v b 1 . Do bi ton ng vi mi cp hai s t nhin c
tng bng n v a 1 b n, a b 1 n suy ra a 1!b!| a b 1! v do

a !b 1!| a b 1!. Nhng ta c (a b)! (a b 1)!(a b) (a b 1)!a (a b 1)!b , v

(a 1)!b!| (a b 1)! v (a 1)!a a ! nn a !b!| (a b 1)!a . Tng t t a !b 1!| a b 1!


suy ra a !b!| (a b 1)!b . Cng li ta c a !b!| (a b)! . Suy ra nh l ng vi cc s t nhin c
tng bng n 1 . Theo nguyn l quy np suy ra bi ton ng vi mi a v b .
2. Chng minh rng vi s t nhin k th tch P a 1 a 2 ... a k chia ht cho k ! .
Chng minh. R rng P a k !/ a ! v vy theo bi tp 1 vi b k ta c iu phi chng minh.
3. Chng minh rng nu a1 , a2 ,..., am l cc s t nhin m 2 th a1 !a2 !...am !| a1 a2 ... am ! .
Chng minh. Theo bi tp 1 th bi ton ng vi m 2 . Gi s bi ton ng vi s t nhin m .
t a1 , a2 ,..., am, am1 l cc s t nhin. Ta c a1 a2 ... am !am 1 !| a1 a2 ... a m a m 1 !, s
dng gi thit quy np, suy ra bi ton ng vi m 1 . iu phi chng minh. Trng hp ring
vi m 3, a1 n, a2 2n, a3 3n v n 1,2,.., ta c n!(2n)!(3n)!| (6n)! vi n 1, 2,... .
4. Chng minh rng nu S l tp hp gm cc s t nhin m tng v hiu ca hai phn t bt k
thuc S cng l phn t thuc S , gi s d l s t nhin nh nht thuc S , th S l tp hp cc
bi s t nhin ca d ( y cc hiu c ly theo hai phn t phn bit v theo th t s ln
tr s b).
Chng minh. Theo gi thit th tng ca hai phn t bt k thuc S cng l mt phn t thuc S
nn bng quy np ta chng minh c tng hu hn cc phn t thuc S cng l mt phn t
thuc S . Trong trng hp c bit khi tt c cc phn t u bng d ta suy ra cc s c dng
nd vi n 1, 2,... u thuc S . Ngha l S cha mi bi s t nhin ca d . Mt khc gi s k l
phn t thuc S nhng khng phi bi s ca d . Th th khi chia k cho d ta nhn c s d
dng r d . Ta c k qd r vi q l s t nhin. Nu q 0 th k r d tc l k d . iu ny
mu thun vi gi thit d l phn t nh nht thuc S . Vy qd l bi s t nhin ca d v do
l mt phn t thuc S . H qu l s t nhin r k qd , l hiu ca hai phn t thuc S , l mt

CHNG 1. TNH CHIA HT V PHNG TRNH BT NH BC MT | 3

phn t thuc S . iu ny khng th c v r d . Vy mi phn t ca S u l bi s t nhin


ca d v ta c iu phi chng minh.
2. Bi s chung nh nht
K hiu a1 , a2 ,..., an l dy hu hn cc s nguyn. Mi s nguyn chia ht cho tt c cc s

ai i 1, 2,..., n c gi l bi s chung ca cc s a1 ,..., an . Mt bi s nh vy l tch ca tt c

cc s a1 , a2 ,..., an . Nu t nht mt trong cc s bng 0 th 0 l bi s chung duy nht ca

chng. Nu tt c cc s ai i 1, 2,..., n u khc 0 th tn ti v hn cc bi s chung ca cc s


chng hn cc s nguyn c dng k a1 a2 ... an , k l s nguyn. Trong trng hp ny cc s
c bi s chung l s t nhin chng hn a1 a2 ... an vi x k hiu gi tr tuyt i ca x . V
trong mi tp hp cc s t nhin u tn ti s nh nht nn trong cc bi s chung t nhin ca
cc s a1 , a2 ,..., an tn ti s nh nht, s ny c gi l bi s chung nh nht ca cc s

a1 , a2 ,..., an v c k hiu l a1 , a2 ,..., an .

nh l 1. Mi bi s chung ca cc s t nhin a1 , a2 ,..., an u chia ht cho bi s chung nh nht


ca cc s .
Chng minh. S dng phn chng. Gi s tn ti bi s chung M ca cc s nguyn a1 , a2 ,..., an
m khng chia ht cho bi s chung nh nht N ca cc s th M qN r vi r l s t nhin
N . V vy r M qN . K hiu i l ch s ty trong cc s 1, 2,..., n. V M v N u l cc bi
s ca ai nn tn ti cc s nguyn xi v yi m M xi ai v N yi ai . Do

r M qN xi qyi ai suy ra ai | r vi mi i 1, 2,..., n suy ra s t nhin r l bi s chung ca

cc s nguyn a1 , a2 ,..., an v nh hn bi s chung nh nht N . V l.


3. c s chung ln nht
K hiu S l tp hp cho trc (hu hn hoc v hn) gm cc s nguyn tha mn t nht mt
trong chng, chng hn a0 , l khc 0 . Mi s nguyn d l c s ca mi phn t thuc S c
gi l c s chung ca cc s nguyn thuc S . R rng 1 l c s chung ca cc s nguyn thuc
S . Mi c s chung d ca cc s nguyn thuc S u l c s ca s t nhin a0 v do n
khng ln hn a0 . T y suy ra s cc c s chung ca cc s nguyn thuc S l hu hn v
do trong cc c s chung tn ti s ln nht. S ny c gi l c s chung ln nht ca
cc s nguyn thuc S v k hiu l d S . R rng d S l s t nhin. By gi k hiu d l c s

chung ty ca cc s nguyn thuc S v t N d , d S . Hn na k hiu a l s nguyn thuc

S . Ta c d | a v d S | a suy ra a l bi s chung ca cc s d v d S nn theo nh l 1 th

d , dS | a . Vy

N d , d S l c s chung ca cc s nguyn thuc S v v d S l c s chung

ln nht ca cc s nguyn nn N d S . Nhng s t nhin N l bi s chung nh nht ca cc


s d v d S nn n chia ht cho d S suy ra N d S . V vy N d S v do d | d S . Ta c nh l
nh l 2. Nu S l tp hp (hu hn hay v hn) cc s nguyn m trong c t nht mt phn
t khc 0 th tn ti c s chung ln nht ca cc s nguyn thuc S . Hn na c s chung ln
nht ny chia ht cho mi c s chung khc ca cc s nguyn thuc S .
C th chng minh rng (Hensel [1]) nu f x l a thc bc n vi h s nguyn v k l s
nguyn ty th c s chung ln nht ca cc s f x khi x nhn mi gi tr nguyn l bng vi
c s chung ln nht ca n 1 s nguyn f k , f k 1 , f k 2 ,..., f k n . V vy chng hn

4 | Bi s chung nh nht v c s chung ln nht. Cc s nguyn t cng nhau


3
vi f x x x th c s chung ln nht ca cc s f x khi x nhn mi gi tr nguyn l

bng vi c s chung ln nht ca cc s nguyn f 1 0, f 0 0, f 1 0, f 2 6, ngha


l bng 6 .
4. Cc s nguyn t cng nhau
Hai s nguyn a v b c c s chung ln nht bng 1 gi l cc s nguyn t cng nhau.
nh l 3. Khi chia cc s nguyn a v b cho c s chung ln nht ca chng th ta nhn c cc
s nguyn t cng nhau.
Chng minh. Gi d l c s chung ln nht ca hai s nguyn a v b . t a1 a d , b1 b d .
Nu cc s nguyn a1 v b1 khng nguyn t cng nhau th c s chung ln nht ca chng l d1
s ln hn 1 v ta c a2 a1 d1 v b2 b1 d1 l cc s nguyn. Nhng khi a dd1 a2 , b dd1 b2
suy ra s nguyn dd1 l c s chung ca cc s nguyn a v b suy ra dd1 d . iu ny khng
th c v d1 1 . Vy a1 v b1 nguyn t cng nhau. nh l 3 c chng minh.
c s chung ln nht ca cc s nguyn a1 , a2 ,..., an c k hiu l a1 , a2 ,..., an .
Lp lun s dng trong chng minh nh l 3 cho ta kt qu sau
nh l 3a . Khi chia cc s nguyn a1 , a2 ,..., an cho c s chung ln nht ca chng th ta nhn
c cc s nguyn c c s chung ln nht bng 1 .
Gi s r l mt s hu t (ngha l t s a b ca hai s nguyn a v b vi b 0 . C th gi s
b 0. Nu (a, b) d th t a d a1 , b d b1 th theo nh l 3 ta nhn c cc s nguyn t
cng nhau a1 v b1 vi b1 0 . Khi ta c r a b a1 b1 . V vy mi s hu t u c th biu
din di dng mt phn s ti gin (ngha l phn s vi t s v mu s nguyn t cng nhau)
vi t s l s nguyn v mu s l s t nhin.
By gi ta chng minh rng nu a, b 1 v c | a th c, b 1. Tht vy, nu (c, b) d th d | b v

d | c m c | a suy ra d | a . H qu l d l c s chung ca cc s nguyn a v b v theo nh l 2


th n l c s ca c s chung ln nht ( 1 ) ca hai s . Suy ra d 1 chng t c, b 1 .
Vi mi dy hu hn cc s t nhin a1 , a2 ,..., an ta d dng tm c s t nhin a nguyn t
cng nhau vi mi phn t ca dy. Chng hn s a a1 a2 ... an 1 . Khi mi c s chung d i
ca cc s nguyn a v ai , vi i l ch s bt k trong cc s 1, 2,..., n, cng l c s ca a1 a2 ...an
nn n cng l c s ca hiu a a1 a2 ...an 1 v do bng 1 .
T nhn xt ny ta kt lun rng tn ti dy v hn cc s t nhin m mi phn t khc nhau
trong dy u nguyn t cng nhau. Tuy nhin cng thc c th cho phn t th n ca mt dy
nh vy l khng n gin. Mt dy n gin nht thuc dng ny l dy cc s i mt nguyn t
k
cng nhau Fk 22 1 k 0,1, 2,... . Tht vy xt cc s nguyn m n 0 . Ta bit vi mi s

nguyn x v s t nhin k th x 1| x k 1 v x k 1 x 1 x k 1 x k 2 ... x 1 . p dng tnh


n1

n1

n1

2
2
cht ny vi x 22 , k 2mn1 ta c 2 1| 2 1 . V Fn 22 1| 22 1 v 22 1 Fm 2, ta
m

c Fn | Fm 2. Do nu d | Fn v d | Fm th d | Fm 2 suy ra d | 2 . Nhng d l c s ca s l

Fm nn bn thn n cng l s l. V vy t d | 2 suy ra d |1 . Chng t Fm , Fn 1 vi m n 0 .

Kt qu tng qut hn cng ng: nu a v b l cc s nguyn t cng nhau v nu 2 | ab th mi


phn t phn bit trong dy a 2 b2 k 0,1, 2,... l nguyn t cng nhau.
k

CHNG 1. TNH CHIA HT V PHNG TRNH BT NH BC MT | 5

C th chng minh nu k l s t nhin 16 th trong k s t nhin lin tip lun tn ti t nht


mt s nguyn t vi k 1 s cn li (Pilai [4]). Mt khc c th chng minh rng vi mi s t
nhin k 17 th tn ti dy k s t nhin lin tip m, m 1,..., m k 1 m khng c phn t no
trong dy nguyn t cng nhau vi tt c cc phn t cn li (Pillai [5],[6] v Brauer [2]). Vi
k 17 th m 2184 tha mn cc iu kin nu trn. Ni cch khc trong cc s t nhin
2184,2185,,2200 khng c s no nguyn t cng nhau vi tt c cc s cn li. Cc s trong dy
trn m chia ht cho mt trong cc s 2,3,5,7 th khng phi l s nguyn t cng nhau vi tt c
cc s cn li v vi mi n 2,3,5,7 th tn ti t nht hai phn t trong dy trn chia ht cho n .
Ngoi ra ch cn li hai phn t khc l 2189 v 2197. Nhng s th nht cng vi 2200 l chia ht
cho 11 cn s th hai cng vi 2184 l chia ht cho 13.

Bi tp. 1. Chng minh rng nu m v n l cc s t nhin, m l, th 2m 1, 2n 1 1 .


Chng minh (J. Browkin). Gi d l c s chung ln nht ca cc s 2m 1 v 2n 1. Khi d l
s l v 2m 1 kd , 2n 1 ld , vi k v l l cc s t nhin. V vy 2m kd 1, 2n ld 1 suy ra

2mn kd 1 td 1, 2mn ld 1 ud 1 vi t v u l cc s t nhin. H qu l t


n

td 1 ud 1 suy ra ta c d | 2 v do d l nn d 1 .
2. Chng minh rng vi mi s t nhin n ta c n! 1, n 1! 1 1.
Chng minh. Nu d | n ! 1 v d | (n 1)! 1 th t ng thc (n! 1)(n 1) (n 1)! n 1 ta thy

d | n 1! n 1, suy ra d | n v v d | n! 1, ta c d |1 .
5. Quan h gia c s chung ln nht v bi s chung nh nht
nh l 4. Tch ca hai s t nhin bng vi tch ca c s chung ln nht v bi s chung nh nht
ca hai s .
Chng minh. Vi hai s t nhin a v b k hiu N a, b . V ab l bi s chung ca a v b nn
t nh l 1 suy ra N | ab . t ab dN vi d l s t nhin. V N l bi s chung ca a v b nn
ta c N ka lb vi k v l l cc s t nhin. T y suy ra ab dN dka dlb v do a dl
v b dk chng t d l c s chung ca a v b . By gi k hiu t l c s chung ty ca a
v b . Ta c a ta1 , b tb1 suy ra ta1 b1 l bi s chung ca cc s a v b . Do t nh l 1 ta c

N | ta1 b1 . V vy vi s nguyn u ta c ta1 b1 Nu . Nhng dN ab t 2 a1 b1 , suy ra tN u dN . H


qu l d tu v t | d . V vy s t nhin d l c s chung ca a v b v hn na mi c s
chung ca cc s u l c s ca d . Vy d l c s chung ln nht ca cc s a v b . T
cng thc ab dN suy ra nh l 4 c chng minh.
Khi a v b l cc s nguyn t cng nhau, ngha l d a, b 1 , th cng thc ab Nd tr thnh

N ab . Ta c h qu sau
H qu. Bi s chung nh nht ca hai s t nhin nguyn t cng nhau chnh l tch ca hai s .
6. nh l c bn ca s hc
Gi s a v b l cc s t nhin nguyn t cng nhau v c l s t nhin m b | ac . S ac chia ht
cho c hai s a v b do theo nh l 1 th n chia ht cho bi s chung nh nht ca cc s .
Bi s ny theo nh l 4 th chnh l tch ab . V vy ac tab vi t l s nguyn. Suy ra c tb v
do b | c . Ta c nh l sau y
nh l 5. S t nhin l c s ca mt tch hai s t nhin v nguyn t cng nhau vi mt trong
hai s s l c s ca s cn li.

6 | Quan h gia c s chung ln nht v bi s chung nh nht. nh l c bn ca s hc

nh l 5 thng c gi l nh l c bn ca s hc. Ta chng minh nh l ny ng vi cc


s t nhin. nh l cng ng vi cc s nguyn v php i du khng nh hng ti tnh chia
ht ca cc s.
H qu. Nu a, b, c l cc s nguyn tha mn a | c, b | c v (a, b) 1 th ab | c.
Chng minh. Nu a | c th c at vi t l s nguyn. V b | c ta c b | at v v a, b 1 , t nh l
5 suy ra b | t ngha l t bu vi u l s nguyn. Do c at abu suy ra ab | c .
T nh l 5 ta chng minh c
nh l 6. Nu a, b, c l cc s nguyn tha mn (a, b) (a, c) 1 th a, bc 1 .
Chng minh. K hiu d a, bc v d1 b, d . Ta c d1 | b v d1 | d . V d | a, d1 | a nn ta thy v

d1 | a , d1 | b v a, b 1 nn d1 1 . Do b, d 1 . Nhng do d a, bc , d | bc , t nh l 5 suy
ra d | c. V d | a v a, c 1 suy ra d 1 ngha l a, bc 1 .
S dng quy np ta c
a
nh l 6 . Gi s n l s t nhin 2 . Nu a1 , a2 ,..., an v a l cc s nguyn tha mn ai , a 1

vi mi i 1, 2,..., n th a1 a2 ... an , a 1.
Ni cch khc nh l 6 ch ra rng mt s nguyn nguyn t cng nhau vi cc s nguyn cho
trc th n cng nguyn t cng nhau vi tch ca cc s .
a

Lp lun trong chng minh nh l 5 cho ta kt qu tng qut hn: nu a, b v c l cc s nguyn


tha mn b | ac th b | a, b b, c .
T nh l 6a ta suy ra

H qu 1. Nu a, b 1 v n l s t nhin th a n , bn 1.

a
Chng minh. Nu a, b 1 th theo nh l 6 (vi a1 a2 ... an a ) ta c a n , b 1 suy ra

(li theo nh l 6 vi a1 a2 ... an b ) ta c a n , bn 1 .


a

T H qu 1 ta c
H qu 2. Vi mi s t nhin a, b, n m a n | bn suy ra a | b .
Chng minh. t a, b d . Ta c a dat , b dbt vi at , b1 1. V vy theo H qu 1 ta c

(a1n , b1n ) 1 . V a n | bn , hoc tng ng a1n d n | b1n d n , ta c a1n | b1n suy ra a1n | a1n , b1n chng t
a1n |1 suy ra at 1, a d v h qu l, v b db1 ab1 , a | b, iu phi chng minh.

Lu rng vi hai s t nhin a v b th t iu kin a a | bb khng suy ra a | b c. Chng hn

44 |1010 nhng 4 |10 v tng t 99 | 2121 nhng 9 | 21 .


Ghi ch. Khi nim v tnh chia ht c th c m rng cho cc s thc theo cch nh sau. Cho
trc hai s thc v khi ta ni l c s ca v vit | nu tn ti s nguyn k
m k . Trong trng hp ny th t 2 | 2 khng suy ra | . Chng hn 2 | 6 nhng ta
khng c

2 | 6 v nu ngc li th tn ti s nguyn k m

6 k 2 suy ra k 3 suy ra

3 k v v vy k 1 tc l k 2 v do 3 k 4 . iu ny khng ng.
2

H qu 3. Vi cc s t nhin a, b v n 1 th t a n | 2bn suy ra a | b .

CHNG 1. TNH CHIA HT V PHNG TRNH BT NH BC MT | 7

Chng minh. t a, b d suy ra a da1 , b db1 vi a1 , b1 1 . V vy theo H qu 1 th v

a , b 1 v t a
n
1

n
1

| 2bn suy ra d n a1n | 2d nb1n do a1n | 2b1n . S dng a1n , b1n 1 v nh l 5 suy

n
ra a1 | 2 m n 1 suy ra a1 1 v do a d chng t a | b .

nh l 7. Nu mt s t nhin l ly tha bc m ca mt s hu t v m l s t nhin th s l


ly tha bc m ca mt s t nhin.
Chng minh. Gi s s t nhin n l ly tha bc m ca s hu t p q . Trong mc 4 ta bit c

a
th gi s p v q l cc s t nhin v p, q 1 . V vy theo nh l 6 suy ra p m , q 1. Mt

khc v n p q ta c nq m p m suy ra q | p m v v vy q | p m , q 1 . Do q 1 (v q l s t
m

nhin) v h qu l n p ngha l n l ly tha bc m ca mt s t nhin.


m

Ta c h qu trc tip ca nh l 7
H qu. Cn bc m ca s t nhin khng phi ly tha bc m ca mt s t nhin l s v t.
c bit cc s

2, 3, 5, 6, 7, 8, 10, 3 2, 3 3, 3 4 u l s v t.

Bi tp. 1. Chng minh rng nu a, b, d l cc s nguyn tha mn a, b 1 v d | a b th

d , a 1 v d , b 1 .
a, b 1 v d | a b . Nu d , a th | d v | a do v
d | a b, | a b nn | a b a suy ra | b . V vy | b do | a, b . Ta c d , a 1 .
Tng t chng minh c d , b 1 .
Chng minh. Gi s

2. Chng minh rng nu n, n1 v n2 l cc s t nhin m n | n1 n2 v cc s n1 , n2 u khng chia


ht cho n th s
(*)

n1
n1 n2
n1 ,

l c s ca n v 1 d n .

n1 n1 n2
n1
n nn
n
n1 ,
l s t nhin v do n1 1 k , 1 2 1 l ,
nn
d
n
d
d
n
d
vi k v l l cc s t nhin nguyn t cng nhau. Ta cng c k d , n2 d nl v v d , l 1, d | n .
Chng minh. Theo (*) ta c

V vy d l c s ca n . Nu d 1, ta c n2 nl v v vy n | n2 , mu thun vi gi thit. Nu

d n th theo (*) suy ra d | n1 , ta c n | n1 , mu thun vi gi thit. Vy d l c s ca n vi


1 d n, iu phi chng minh.
3. Chng minh rng nu a v b l cc s t nhin nguyn t cng nhau v m l s t nhin ty
th trong cp s cng a bk k 0,1, 2,... tn ti v hn s nguyn t cng nhau vi m .
Chng minh. Gi s a, b 1 v m l s t nhin ty . S m c cc c s nguyn t cng nhau
vi a (chng hn c s 1 ). K hiu c l c s ln nht nh vy. Ta s chng minh rng s
a bc nguyn t cng nhau vi m . Ta c (a, b) 1 v theo nh ngha ca c th (a, c) 1 . V vy

a, bc 1 . T bi tp 1 suy ra nu d | a bc th d , a 1 v d , bc 1 do d , c 1. Mt khc
nu d | m th v c | m , d , c 1, h qu nh l 5 suy ra dc | m . Hn na v d , a 1 v a, c 1

8 | Quan h gia c s chung ln nht v bi s chung nh nht. nh l c bn ca s hc

th a, dc 1 . Do dc l c s ca m v nguyn t cng nhau vi a . Nhng v c l c s ln


nht c tnh cht ny nn d 1 . Vy ta chng minh nu d l c s chung ca cc s a bc v
m th d 1 . Suy ra a bc, m 1 . Vy nu l l s t nhin ty th vi k c lm cc s a bk
v m nguyn t cng nhau. iu phi chng minh.
4. Chng minh rng nu a v b l cc s t nhin nguyn t cng nhau th cp s cng
a kb k 0,1, 2,... cha dy con v hn cc phn t thuc dy l i mt nguyn t cng nhau.
Chng minh. Ta xy dng dy con u1 , u2 ,... tha mn yu cu bi ton bng quy np. t u1 a .
Xt n l s t nhin ty . Gi s ta xc nh c dy cc s u1 , u2 ,..., u n i mt nguyn t
cng nhau. Theo bi ton 3 th vi s t nhin u1 u2 ...un tn ti phn t thuc dy

a kb k 0,1, 2,... m nguyn t cng nhau vi u1 u2 ...un . K hiu s nh vy l un 1 . Chng t

dy u1 , u2 ,... xc nh theo cch ny c tnh cht yu cu.


nh l 8. Gi s a v b l cc s t nhin nguyn t cng nhau m tch ca chng l ly tha bc n
n
ca mt s t nhin, ngha l ab c vi n l s t nhin, th cc s a v b cng l cc ly tha bc
n ca cc s t nhin.
Chng minh. t

a, c d

th a da1 , c dc1 vi

da1b d n c1n suy ra a1b d n1c1n . M d | a v a, b 1

n 1

a1 , c1 1 . Theo gi thit ab cn ta c
a
ta c d , b 1 suy ra theo nh l 6 th

n 1 n
n 1 n
n
, b 1 . ng thc a1b d c1 chng t b | d c1 . V vy theo nh l 5 th b | c1 . Mt khc v

a1 , c1 1 , t nh l 6a

n 1 n
n
suy ra a1 , c1n 1 v v a1b d c1 suy ra c1 | a1b do theo nh l 5 ta

n
n
n
n
n 1
n
c c1 | b . Do b | c1 v c1 | b suy ra b c1 nn a1 d
v a da1 d . Chng t cc s a v b
cng l ly tha bc n cc s t nhin.

H qu. Gi s k , c v n l cc s t nhin v a1 , a2 ,..., ak l dy cc s t nhin i mt nguyn t


n
cng nhau v a1 a2 ...ak c . Khi mi s thuc dy a1 , a2 ,..., ak l ly tha bc n cc s t nhin.

7. Cc cng thc a1 , a2 ,..., an1 at , a2 ,..., an , an1 v a1 , a2 ,..., an1 a1 , a2 ,..., an , an1
Ta chng minh cc cng thc
(2)

a1, a2 ,..., an1 a1, a2 ,..., an , an1

(3)

a1 , a2 ,..., an1 a1 , a2 ,..., an , an1

nh l 9. Vi cc s t nhin n 2 v a1 , a2 ,..., an1 th cng thc (2) ng.


Chng minh. t d a1 , a2 ,..., an , an1 th d l c s chung ca cc s a1 , a2 ,..., an v an 1 .
V a1 , a2 ,..., an l c s ca cc s a1 , a2 ,..., an nn d l c s ca cc s a1 , a2 ,..., an , an1 . K
hiu d ' l c s ty ca cc s a1 , a2 ,..., an , an1 . Theo nh l 2 ta c d ' | a1 , a2 ,..., an . V

d ' | an1 nn ta c theo nh ngha ca d v s dng nh l 2 th d ' | d . V vy d l c s chung


ca cc s a1 , a2 ,..., an , an1 v chia ht cho mi c s chung ca cc s . Do d l c s
chung ln nht ca a1 , a2 ,..., an1 . Cng thc (2) c chng minh.
T lp lun trong chng minh ny ta thy tnh c s chung ln nht a1 , a2 ,..., an th ta c th
tnh ln lt d2 a1 , a2 , d3 d2 , a3 , d4 d3 , a4 ,..., dn1 dn1 , an1 , a1 , a2 ,..., an dn1 , an ,

CHNG 1. TNH CHIA HT V PHNG TRNH BT NH BC MT | 9

nh vy php tnh ton c s chung ln nht ca cc s t nhin ty quy v cc php tnh lin
tip cc c s chung ln nht ca hai s t nhin.
nh l 10. Vi cc s t nhin n 2 v a1 , a2 ,..., an1 th cng thc (3) ng.
Chng minh. t N a1 , a2 ,..., an , an1 th N l bi s chung ca cc s a1 , a2 ,..., an v an 1 .

V a1 , a2 ,..., an l bi s ca cc s a1 , a2 ,..., an nn N l bi s ca cc s a1 , a2 ,..., an , an1 . K


hiu M l bi s chung ty ca cc s a1 , a2 ,..., an , an1 . Theo nh l 1 ta c a1 , a2 ,..., an | M .
M an 1 | M nn li s dng nh l 1 v suy ra a1 , a2 ,..., an , an1 | M hoc tng ng N | M .
Vy N l bi s chung ca cc s a1 , a2 ,..., an , an1 v l c s ca mi bi s chung khc ca cc
s ny. Do N l bi s chung nh nht ca cc s d. Cng thc (3) c chng minh.
Vy tnh bi s chung nhr nht a1 , a2 ,..., an ta ln lt tnh bi s chung nh nht ca cc cp

N2 a1 , a2 , N3 N2 , a3 ,..., Nn1 Nn2 , an1 a1 , a2 ,..., an Nn1 , an .


nh l 11. Nu n l s t nhin v cc s a1 , a2 ,..., an i mt nguyn t cng nhau th

a1, a2 ,..., an a1 a2 ...an .

Chng minh. T h qu nh l 4 suy ra bi ton ng vi n 2 . By gi xt n l s t nhin ty


2 . Gi s bi ton ng vi s t nhin n v a1 , a2 ,..., an , an1 l cc s t nhin i mt nguyn

a
t cng nhau. Khi ai , an1 1 vi mi i 1, 2,..., n . Theo nh l 6 v h qu nh l 4 th

a1 a2 ... an an1 a1 a2 ... an an1 . Nhng


a1 a2 ... an a1 , a2 ,..., an v theo (3)

theo gi thit quy np th bi ton ng vi n nn


th a1 a2 ...an an1 a1 , a2 ,..., an , an1 a1 , a2 ,..., an , an1 ,

suy ra bi ton ng vi n 1 v v vy ng vi mi s t nhin.


iu ngc li cng ng: nu vi cc s t nhin n 2 v a1 , a2 ,..., an ta c cng thc

a1, a2 ,..., an a1 a2 ... an th cc s a1 , a2 ,..., an

l i mt nguyn t cng nhau.

Mnh sau y cng ng: tch ca n 2 s t nhin bng vi tch ca c s chung ln nht v
bi s chung nh nht ca chng khi v ch khi cc s l i mt nguyn t cng nhau.
Mnh ny khng ng vi n 2 chng hn 2 4 2, 4 2, 4 .
8. Quy tc tnh cc c s chung ln nht ca hai s
Gi s a v b l hai s t nhin cho trc. Chia a cho b ta nhn c thng s q v s d r
nh hn b . Ta c a qb r . T ng thc ny suy ra mi c s chung ca a v b cng l c s
ca s d r a qb v mi c s chung ca b v r cng l c s ca a . Vy c s chung ca
a v b chnh l cc c s chung ca b v r . Do a, b b, r . Ta k hiu a n0 , b n1 , r n2

v c ng thc n0 , n1 n1 , n2 . Nu n2 0 th n0 , n1 n1 . Nu n2 0 th ta c th chia n1 cho

n2 v k hiu s d l n3 v li c n1 , n2 n2 , n3 . Qu trnh ny lp li v ta thu c

n0 , n1 n1 , n2 ,
n1 , n2 n2 , n3 ,
n2 , n3 n3 , n4 ,
(4)

10 | Quy tc tnh cc c s chung ln nht ca hai s

nk 2 , nk 1 nk 1 , nk ,
nk 1 , nk nk , nk 1
V ni 1 k hiu s d nhn c khi chia ni 1 cho ni i 1, 2,..., k nn ta c ni 1 ni vi i 1, 2,..., k
suy ra dy cc s ni l gim nghim ngt, ngha l n1 n2 n3 ... 0. Dy ny hu hn v ch c
n s nguyn khng m phn bit nh hn n . V vy trong dy (4) tn ti phn t cui cng, ngha
l nk 1 , nk nk , nk 1 . Nu ta c nk 1 0 th ta c th chia nk cho nk 1 v nhn c ng thc

nk , nk 1 nk 1, nk 2 , mu thun vi gi thit ch c k ng thc trong (4). V vy nk 1 0 v do


nk 1 , nk nk . Cc ng thc trong (4) suy ra n0 , n1 n1 , n2 n2 , n3 ... nk 1 , nk nk ,
suy ra n0 , n1 nk . T cc lp lun ny ta nhn c quy tc tnh c s chung ln nht ca hai
s t nhin cho trc: tnh c s chung ln nht ca hai s t nhin n0 v n1 ta chia n0 cho n1
v tm s d n2 . Sau ta chia n1 cho n2 v tm c s d n3 . Tip tc nh vy ta chia n2 cho n3
v c nh th. bc cui cng ta nhn c s d bng 0 . S d nhn c trong bc trc
chnh l c s chung ln nht ca cc s n0 v n1 .
Quy tc va nhn c cng c gi l thut ton chia hoc thut ton Euclid hoc l thut ton
lin phn s. Tn gi cui cng s c trnh by c th trong mc 9.
T thut ton Euclid suy ra c s chung ln nht ca hai s t nhin cho trc c th nhn c
sau hu hn php chia. Tuy nhin s cc php chia li c th ln ty . Ngha l vi mi s t nhin
n th tn ti cc s t nhin an v bn m tm c s chung ln nht ca chng bng thut ton
Euclid th cn ti n php chia. chng minh tnh cht ny ta xt dy

u1 u2 1, un un1 un2 , vi n 3, 4,...

(5)
Ta c
(6)

u1 1, u2 1, u3 2, u4 3, u5 5, u6 8, u7 13, u8 21, u9 34,...

y l dy s Fibonacci: hai phn t u tin ca dy bng 1 v cc phn t tip theo bng tng
ca hai phn t lin trc n. t an un2 , bn un1. p dng thut ton Euclid tm

an , bn un2 , un1 . Ta nhn c dy cc php chia


un 2 1 un 1 un ,
un 1 1 un un 1 ,

u4 1 u3 u2 ,
u3 2 u2 .

R rng c n php chia tt c. Chng hn tm c s chung ln nht ca cc s u12 144 v

u11 89 bng thut ton Euclid th ta cn 10 php chia. C th chng minh rng cc s nh nht
m cn ng n php chia tm c s chung ln nht ca chng bng thut ton Euclid chnh l
un 2 v un 1 . Ta chng minh kt qu sau y
nh l 12. S cc php chia cn thit tm c s chung ln nht ca hai s t nhin bng thut
ton Euclid l khng ln hn nm ln s cc ch s thp phn ca s nh hn (Lame [1]).
Chng minh. u tin ta chng minh tnh cht sau y ca dy s Fibonacci un n 1, 2,... c
nh ngha trn
(7)

un5 10un vi n 2,3,...

CHNG 1. TNH CHIA HT V PHNG TRNH BT NH BC MT | 11

Tnh ton trc tip chng t vi n 2 th cng thc (7) ng (vi u7 13 10u2 10 ). Vi n 3
th theo (5) ta c

un5 un 4 un3 2un3 un 2 3un 2 2un1


5un1 3un 8un 5un 1.
Dy (6) khng gim, un un1 un2 2un1 , 2un 4un1 v un5 8un 5un1 8un 4un1 10un
nn suy ra un5 10un .
T (7), bng quy np ta chng minh c
(8)

un5l 10l un ,

n 2,3,...; l 1, 2,...

K hiu no v n1 n0 l hai s t nhin cho trc. Gi s tm c s chung ln nht n0 , n1


bng thut ton Euclid cn ti k php chia

n0 q1n1 n2 ,
n1 q2 n2 n3 ,
(9)

nk 2 qk 1nk 1 nk ,
nk 1 qk nk .

Ta c qk 2 v nu qk 1 th nk nk 1 , iu ny v l v nk l s d nhn c khi chia nk 2 cho

nk 1 . V vy nk 1 qk nk 2nk 2 u3 nn
nk 2 nk 1 nk u3 u2 u4 , nk 3 nk 2 nk 1 u4 u3 u5 ,..., n1 uk 1.
l
Do nu k 5l , hoc tng ng k 5l 1 , th n1 u5l 2 v theo (8) (vi n 2 ) th n1 10 .

Ngha l n1 c t nht l 1 ch s trong biu din thp phn. V vy nu n1 c l ch s th k 1 .


nh l 12 c chng minh.
T nh l 12 suy ra tm c s chung ln nht ca hai s t nhin cho trc bng thut ton
Euclid m s nh hn c nhiu nht 6 ch s th cn nhiu nht 30 php chia. Trong nh l 12 th
s 5 khng th thay bi 4 v ta cn 10 php chia tm c s chung ln nht ca 144 v 89
(Brown J.L Jr. [1], Dixon [1],[2]).
9. Biu din s hu t thnh lin phn s
K hiu n0 , n1 l cc s t nhin v (9) l dy cc ng thc nhn c bng cch s dng thut
n
n
1
ton Euclid cho cc s n0 v n1 . Vi mi i 1, 2,..., k 1 ta c i 1 qi
v k 1 qk . Do
ni
ni / ni 1
nk

n0
1
q1
1
n1
q2
1
q3
q4
(10)

1
1
qk 1
qk

12 | Biu din s hu t thnh lin phn s. Dng tuyn tnh ca c s chung ln nht

Ta vit gn cng thc ny thnh

n0
1| 1| 1|
1 |
1|
q1

...

. Trong (9) th q1 l s
n1
| q2 | q3 | q4
| qk 1 | qk

nguyn dng l thng s nhn c khi chia s t nhin n0 cho s t nhin n1 , cc s qi vi

i 2,3,..., k l cc s t nhin v ni 1 ni . Biu thc trong v phi ca (10), q1 l s nguyn v


q2 , q3 ,..., qn l cc s t nhin, c gi l lin phn s n (hoc gn hn l lin phn s).
V vy s dng thut ton Euclid th mi s hu t u biu din c thnh mt lin phn s n.
V d. Xt s 314159/100000. p dng thut ton Euclid ta nhn c

314159 3 100000 14159,


100000 7 14159 887,
14159 15 887 854,
887 1 854 33,
854 25 33 29,
33 1 29 4,
29 7 4 1,
4 4 1.

314159
1| 1 | 1| 1 | 1| 1| 1 |
3

. Mt v d khc khi ta xt cc s un1 un trong
100000
| 7 |15 |1 | 25 |1 | 7 | 4
uk k 1, 2,... l cc s Fibonacci (mc 8). T (10) suy ra vi mi s t nhin n ta c
un1
1| 1| 1|
1|
|
1 ... . Trong du
xut hin n 1 ln. V vy chng hn ta c
un
|1 |1 |1
|1
|
u3
u2
u
1| u
1| 1|
1|
1,
1 , 4 1 v c nh th. Ta cng c th vit 4 1 .
u1
u2
|1 u3
|1 |1
u3
|2
V vy

Ta s tm hiu chi tit hn v cc lin phn s trong chng 8.


10. Dng tuyn tnh ca c s chung ln nht
nh l 13. Nu a1 , a2 ,..., am l m 1 s nguyn m t nht c mt s trong chng l khc 0 th tn
ti cc s nguyn t1 , t2 ,..., tm tha mn
(11)

a1, a2 ,..., am a1t1 a2t2 ... amtm .

Chng minh. K hiu D l tp hp cc s t nhin xc nh bi quy tc: s n thuc D khi v ch


khi tn ti cc s nguyn x1 , x2 ,..., xm tha mn
(12)

n a1 x1 a2 x2 ... am xm .

Ni cch khc D l tp hp tt c cc s t nhin c dng a1 x1 a2 x2 ... am xm vi x1 , x2 ,..., xm


l cc s nguyn. Tp hp ny khng rng (ngha l n cha t nht mt phn t) v ak 0 (vi

1 k m ) nn ak thuc D bi v n c dng a1 x1 a2 x2 ... am xm , vi xi 0 vi i k v xk


bng 1 hoc 1 tng ng vi ak 0 hoc ak 0 .
K hiu d l s t nhin nh nht thuc tp hp D (s d tn ti v trong mt tp hp cc s t
nhin lun tn ti phn t nh nht). Nu d thuc D th theo nh ngha tn ti cc s nguyn
t1 , t2 ,..., tm tha mn
(13)

d a1t1 a2t2 ... amtm .

CHNG 1. TNH CHIA HT V PHNG TRNH BT NH BC MT | 13

Nhng v d l phn t nh nht ca D nn vi mi s t nhin n c dng (12) vi x1 , x2 ,..., xm l


cc s nguyn th n d . Ta s chng minh rng vi cc s nguyn x1 , x2 ,..., xm bt k th s

a1 x1 a2 x2 ... am xm chia ht cho d . Gi s phn chng th tn ti cc s nguyn y1 , y2 ,..., ym m


khi chia a1 y1 a2 y2 ... am ym cho d th ta nhn c thng s q v s d dng r . Ta c

a1 y1 a2 y2 ... am ym qd r suy ra theo (13)

r a1 y1 a2 y2 ... am ym q a1 t1 a2 t2 ... am tm a1 x1 a2 x2 ... am xm


Vi xi yi qti l cc s nguyn vi i 1, 2,..., m . V vy s t nhin r c dng (12) suy ra r thuc

D . Nhng mt khc r l s d nhn c khi chia mt s cho d nn n nh hn d , mu thun


vi gi thit d l phn t nh nht ca D . Vy ta chng minh vi cc s nguyn x1 , x2 ,..., xm
bt k th s a1 x1 a2 x2 ... am xm chia ht cho d . c bit d | a1 x1 a2 x2 ... am xm , vi xk 1
v xi 0 vi i k . V vy vi k 1, 2,..., m, d | ak ngha l d l c s chung ca a1 , a2 ,..., am .
K hiu l c s chung ty ca cc s a1 , a2 ,..., am . Khi tn ti cc s nguyn z1 , z2 ,..., zm

m ak zk k 1, 2,..., m . Theo (13) ta c d a1 t1 a2 t2 ... am tm t1 z1 t2 z2 ... tm zm ,


suy ra | d . T y ta kt lun rng c s chung d bng vi a1 , a2 ,..., am bi v n chia ht cho
mi c s chung khc ca cc s a1 , a2 ,..., am . Vy t (13) suy ra (11) v nh l c chng
minh.
Gi s a1 , a2 ,..., am l m 1 s nguyn tha mn a1 , a2 ,..., am 1 . Theo nh l 13 suy ra tn ti
cc s nguyn t1 , t2 ,..., tm tha mn
(14)

a1 t1 a2 t2 ... am tm 1.

Ngc li, gi s vi cc s nguyn cho trc a1 , a2 ,..., am tn ti cc s nguyn t1 , t2 ,..., tm tha


mn (14). V tri ca phng trnh chia ht cho mi c s chung ca cc s a1 , a2 ,..., am . Nhng
v phi ca phng trnh l 1 suy ra a1 , a2 ,..., am 1 . Ta c nh l sau y

nh l 14. Vi m 1 th a1 , a2 ,..., am 1 khi v ch khi tn ti cc s nguyn t1 , t2 ,..., tm tha mn

a1 t1 a2 t2 ... am tm 1 .
H qu. Nu vi cc s nguyn d , k v a1 , a2 ,..., am vi m 1 ta c a1 , a2 ,..., am 1 v d | kai vi

i 1, 2,..., m, th d | k .
Chng minh. Theo nh l 14 th t a1 , a2 ,..., am 1 suy ra tn ti cc s nguyn t1 , t2 ,..., tm tha
mn a1 t1 a2 t2 ... am tm 1 . Nhng v d | kai vi mi i 1, 2,.., m, d | kaiti vi i 1, 2,..., m suy ra

d | k a1 t1 a2 t2 ... am tm v h qu l d | k . iu phi chng minh.

Mnh tng t cc nh l 13 v 14 ng vi a thc mt bin nhng khng ng vi cc a


thc nhiu bin. Tht vy nu f x, y x v g x, y y th c s chung ln nht ca cc a
thc f x, y v g x, y l hng s. Biu thc xp x, y yq x, y khng th l hng s khc 0
vi cc a thc p x, y , q x, y bt k (Bochner [1]).
Tr li vi nh l 13. Mt vn c t ra l vi cc s a1 , a2 ,..., am cho trc lm sao tm c
cc s t1 , t2 ,..., tm tha mn (11). Chng minh trn ca nh l khng cho mt gi no lm
iu ny (ta gi y l mt chng minh s tn ti trn l thuyt m khng c tnh kin thit, tc l
mt chng minh dng n). Tuy nhin ta c th gii quyt vn ny bng cch s dng thut ton

14 | Phng trnh bt nh

bin bc 1

Euclid. Bt u vi trng hp m 2 . B qua trng hp tm thng khi mt trong cc s bng


0. i du nu cn thit cc s t1 v t2 v gi s a1 v a2 l cc s t nhin, k hiu cc s ny l

n0 v n1 . S dng thut ton Euclid ta nhn c cng thc (9). Ta bit nk n0 , n1 . ng thc

p cht trong (9) tng ng vi


(15)

nk nk 2 qk 1 nk 1

Th gi tr ca nk 1 t ng thc trc trong (9) ta c

nk nk 2 qk 1 nk 3 qk 2 nk 2
qk 1 nk 3 1 qk 1 qk 2 nk 2 .
Ta li th gi tr ca nk 2 nhn c t ng thc trc trong (9) v c nh vy. Sau k 2
php th ta nhn c nk n0 x n1 y, vi x v y l cc s nguyn. R rng th tc ny cho ta
cch tnh ton hu hiu cc s x t1 v y t2 .
Trong trng hp tng qut khi m l s t nhin ty 1 ta thc hin bng quy np. Gi s vi
mi s nguyn a1 , a2 ,..., am ta c quy tc tnh t1 , t2 ,..., tm tha mn (11). Gi s a1 , a2 ,..., am , am1
l cc s nguyn cho trc. Theo nh l 9 ta c a1 , a2 ,..., am1 a1 , a2 ,..., am , am1 . Ta bit
t lp lun trn th ta c mt quy tc tm cc s x v y tha mn
(16)

a , a ,..., a , a a , a ,..., a x a
1

m1

m1

y.

t xi ti x vi t 1, 2,..., m v xm1 y . Theo (16) v (11) ta c


(17)

a1, a2 ,..., am1 a1 x1 a2 x2 ... am xm am1 xm1 ,

Vi x1 , x2 ,..., xm1 l cc s nguyn. V vy ta c mt quy tc tm x1 , x2 ,..., xm1 tha mn (17)


da theo quy tc bit tm cc s t1 , t2 ,..., tm . V vy theo quy np chng t: vi mi m 1 v
cc s nguyn a1 , a2 ,..., am m t nht mt trong s chng l khc 0 th tn ti quy tc tm cc s

t1 , t2 ,..., tm tha mn (11).


11. Phng trnh bt nh m bin bc 1
nh l 15. Cho trc m 1 s nguyn a1 , a2 ,..., am m t nht mt s l khc 0 . Phng trnh
(18)

a1 x1 a2 x2 ... am xm b,

c nghim nguyn x1 , x2 ,..., xm khi v ch khi a1 , a2 ,..., am | b.


Chng minh. Gi s tn ti cc s nguyn x1 , x2 ,..., xm tha mn (18). T (18) suy ra c s chung
ca cc s a1 , a2 ,..., am l c s ca b . V vy a1 , a2 ,..., am | b, suy ra iu kin cn.

Mt khc gi s d a1 , a2 ,..., am | b . Khi tn ti s nguyn k m b kd . V t nht mt trong


cc s a1 , a2 ,..., am l khc 0 nn theo nh l 13 th tn ti cc s nguyn t1 , t2 ,..., tm tha mn
(11). t xi kti vi mi i 1, 2,..., m . V d a1 , a2 ,..., am nn theo (11) ta c

a1 x1 a2 x2 ... am xm k a1 t1 a2 t2 ... am tm kd b.
iu kin c chng minh.

CHNG 1. TNH CHIA HT V PHNG TRNH BT NH BC MT | 15

nh l 15 c th pht biu di dng sau y: phng trnh h s nguyn bc 1 vi m 1 bin l


c nghim t nhin khi v ch khi h s t do ca phng trnh ny chia ht cho c s chung ln
nht ca cc h s ca cc bin.
T chng minh nh l 15 v tnh cht vi mi s nguyn cho trc a1 , a2 ,..., am th c th tm cc
s t1 , t2 ,..., tm tha mn (11), ta suy ra nu phng trnh (11) l c nghim nguyn th ta c th tm
cc s nguyn x1 , x2 ,..., xm tha mn (11), ngha l tn ti quy tc tm t nht mt nghim nguyn
ca phng trnh (18). Cu hi t ra l lm th no tm mi nghim nguyn nh vy.
Bt u vi trng hp m 2 . Xt phng trnh
(19)

ax by c

Vi a, b, c l cc s nguyn v a, b | c. C th gi s c a v b u khc 0 v nu ngc li ta c


phng trnh mt bin v d dng tm c nghim ca n. Do a, b | c ta c th tim cc s
nguyn x0 , y0 tha mn
(20)

ax0 by0 c

Gi s x v y l cc s nguyn bt k tha mn phng trnh (19). T (19) v (20) suy ra


(21)

a x x0 b y0 y

V d a, b l c s chung ln nht ca a v b nn ta c a da1 , b db1 vi a1 v b1 l cc s


nguyn nguyn t cng nhau. T (21) ta c
(22)

a1 x x0 b1 y0 y .

Do a1 , b1 1 v theo nh l 6 suy ra b1 | x x0 do x x0 b1 t vi t l s nguyn. Theo (22)


th a1 b1 t b1 y0 y m b1 0 nn y0 y a1 t . T cc ng thc x x0 b1 t , y0 y a1 t suy ra
(23)

x x0 b1 t ,

y y0 a1 t.

Vy nu x, y l nghim nguyn ca (19) th chng c dng (23) vi t l s nguyn.


By gi k hiu t l s nguyn ty . Ta tm x v y t (23) v tnh gi tr ca

ax by a x0 b1 t b y0 a1 t ax0 by0 ab1 ba1 t.


T (20) v ab1 ba1 da1 b1 db1 a1 0 suy ra (19). V vy: cc s nguyn x v y l nghim ca
(19) khi v ch khi vi s t nhin t no ta c cng thc (23) c tha mn.
Vy vi t 0, 1, 2,... cng thc (23) cho tt c cc nghim nguyn ca (19). V t nht mt trong
hai s a1 , b1 l khc 0 , nu (19) c t nht mt nghim nguyn th n c v hn nghim nh vy.
Ta chng minh nh l sau y
nh l 16. Nu a v b l cc s t nhin nguyn t cng nhau th tn ti cc s t nhin u v v
tha mn au bv 1 .
Chng minh. Theo nh l 15 th tn ti cc s nguyn x0 v y0 tha mn ax0 by0 1. Ta chn
s nguyn t0 tha mn t0 x0 b v t0 y0 a , t u x0 bt0 0 v v y0 at0 0 . Khi
u v v l cc s t nhin v au bv ax0 by0 1.

T nh l 16 ta suy ra ba h qu sau y

16 | Phng trnh bt nh

bin bc 1

l
m
H qu 1. Nu cc s t nhin a, b, l , m tha mn a b v l , m l cc s nguyn t cng nhau th

tn ti t nht mt s t nhin n tha mn a nm v b nl .


Chng minh. V l , m 1 nn theo nh l 16 suy ra tn ti cc s t nhin r v s m lr ms 1 .

l
m
V vy t a b suy ra a alr ms alr a ms br a s

. Do a l ly tha bc m ca s hu t

br a s nn theo nh l 7 suy ra n l ly tha bc m ca mt s t nhin n br a s . V vy


a nm suy ra bm al nml chng t b nl . Vy a nm v b nl vi n l s t nhin.
H qu 2. Nu a v b l cc s t nhin nguyn t cng nhau th mi s t nhin n ab u c th
biu din di dng n ax by, vi x, y l cc s t nhin.
Chng minh. Xt a v b l cc s t nhin nguyn t cng nhau v u, v l cc s t nhin tha
mn nh l 16. Khi au bv 1 suy ra vi n ab th anu bnv n ab nn nu b nv a 1 .
Do tn ti s nguyn t tha mn nv a t nu b (y l s nguyn ln nht nh hn nu b ).
t x nu bt , y at nv . Ta c x 0 v y 0 v ax by a nu bt b at nv n suy ra
iu phi chng minh.
Lu rng trong H qu 2 th s ab khng th thay bng mt s nh hn. V nu a, b 1 th s

ab t n khng c biu din dng ax by ab vi x, y l cc s t nhin. Tht vy, gi s


ab ax by th ax a y b suy ra v a, b 1, b | x nn x b . Vy ab ax by ab by ab ,
nhng iu ny l khng th.

H qu 3. Vi cc s t nhin cho trc a 1, m, n th a m 1, a n 1 a m,n 1.


Chng minh. t m, n th m m1 , n n1 vi m1 v n1 l cc s t nhin nguyn t cng
nhau. Theo nh l 16 th tn ti cc s t nhin u, v tha mn m1 u n1 v 1 suy ra mu nv .

t d a m 1, a n 1 . R rng a

m,n

1| a m 1 v a m,n 1| a n 1 suy ra a m,n 1| d . Mt khc ta

c d | a m 1 suy ra d | a mu 1 v d | a n 1 suy ra d | a nv 1 . V vy

d | a mu a nv a nv a mu nv 1 a nv a 1

Do d | a m 1 v a 1 suy ra d , a 1 v v vy d | a 1 . H qu l d | a

m,n

1 m theo cng thc

a m,n 1| d th suy ra a m,n 1 d a m 1, a n 1 , iu phi chng minh.


Vy ta chng minh cc nghim nguyn ca phng trnh (19) c cho bi cng thc (23). By
gi ta xt trng hp tng qut hn dng (18) vi s bin ty m . Phng php tm cc nghim
ca phng trnh (18) trnh by di y l mt trong cc phng php n gin nht.
u tin lu rng ta ch cn xt phng trnh (18) vi ai ( i 1, 2,.., m ) l cc s t nhin bi v
cc h s bng 0 khng nh hng ti cc nghim v nu c h s ai 0 th ta ch cn thay ai bi

ai v i du ca bin s tng ng. Nu c hai h s ai bng nhau chng hn a1 a2 th t

x1 x2 x ta c phng trnh
(24)

a1 x a3 x3 a4 x4 ... am xm b.

T mi nghim nguyn x1 , x2 ,..., xm ca (18) ta c th nhn c nghim x, x3 , x4 ,..., xm ca (24)


bng cch t x x1 x2 . Ngc li t mi nghim nguyn x, x3 , x4 ,... ca (24) ta c th nhn
c nghim ca (18) bng cch t x1 l s nguyn ty v x2 x x1 . V vy vic tm tt c cc

CHNG 1. TNH CHIA HT V PHNG TRNH BT NH BC MT | 17

nghim nguyn ca (18) trong trng hp c hai h s bng nhau c quy v vic tm cc
nghim nguyn ca (24) vi s bin t hn.
Nu c hai h s trong (24) bng nhau th ta li c th tin hnh gim bin nh vy. Vy ta c th
gi s cc h s ca phng trnh (18) l cc s t nhin phn bit. Gi s a1 l s ln nht trong
cc h s . Khi a1 a2 . Gi s khi chia a1 cho a2 ta nhn c thng s k v s d a2 . Khi
ta c a1 a2 k a2 vi k l s t nhin v a2 l s nguyn tha mn 0 a2 a2 . t

x1 kx1 x2 , x2 x1 , a1 a2 . Ta c a1 x1 a2 x2 a2 kx1 x2 a2 x1 a1 x1 a2 x2 . V vy (24) c

th vit li thnh
(25)

a1 x1 a2 x2 a3 x3 ... am xm b.

T mi nghim nguyn x1 , x2 ,..., xm ca (24) ta nhn c nghim nguyn x1, x2 , x3 ,..., xm ca (25)
bng cch t x1 kx1 x2 , x2 x1 . Ngc li t mi nghim x1, x2 , x3 ,..., xm ca (25) ta nhn c
nghim nguyn ca (18) bng cch t x1 x2 , x2 x1 kx2 .
V vy bi ton tm cc nghim t nhin ca (18) c quy v vic gii phng trnh (25) vi s
ln nht trong cc h s ca cc bin l nh hn s ln nht trong cc h s ca (18). Tip tc nh
vy, t phng trnh (25) ta li nhn c phng trnh vi s ln nht trong cc h s ca cc
bin l nh hn s ln nht trong cc h s ca cc bin ca (25). Qu trnh ny dn ti mt
phng trnh mt bin m bit cch gii.
Vy ta chng minh c vi phng trnh tuyn tnh h s nguyn th tn ti phng php tm
tt c cc nghim nguyn ca n. Tuy nhin phng php trnh by trn cha a ra mt quy
tc thun tin nht tm tt c cc nghim ca phng trnh tuyn tnh trong thc t m mi ch
l mt chng minh s tn ti ca cc nghim.
Nu trong (18) c mt h s a1 , a2 ,..., am , chng hn a1 , bng 1 th tt c cc nghim nguyn ca
(18) nhn c bng cch ly x2 , x3 ,..., xm ty v t x1 b a2 x2 a3 x3 ... am xm .
D thy nu phng trnh (18) c nghim nguyn v m 1 th n c v hn nghim nguyn. Tht
vy, nu y1 , y2 ,..., ym l cc s nguyn tha mn a1 y1 a2 y2 ... am ym b th t xi yi am ti
vi i 1, 2,..., m 1 v xm ym a1 t1 ... am1 tm1 vi t1 , t2 ,..., tm1 l cc s nguyn ty ta nhn
c cc s nguyn x1 , x2 ,..., xm tha mn (18).
Cng d dng chng minh rng nu (18) c nghim nguyn x1 , x2 ,..., xm th cc s nguyn

x1 , x2 ,..., xm c th biu din thnh t hp tuyn tnh ca m 1 tham s nguyn.


Tnh cht ny cho php chng ta tm cc nghim nguyn ca h n phng trnh tuyn tnh m
bin. lm iu ta khai trin cc bin ca phng trnh th nht thnh t hp tuyn tnh ca
m 1 tham s vi h s nguyn v th vo n 1 phng trnh cn li. Khi c th coi cc tham s
nh l cc bin s v ta nhn c h n 1 phng trnh m 1 bin. Qu trnh ny lp li v cui
cng ta nhn c hoc l mt phng trnh (mt hoc nhiu bin) m bit cch gii hoc l
mt hoc nhiu phng trnh mt bin.
12. nh l s d Trung Hoa
nh l 17. Gi s m l s t nhin 2, a1 , a2 ,..., am l cc s t nhin i mt nguyn t cng nhau
v r1 , r2 ,..., rm l cc s nguyn ty . Khi tn ti cc s nguyn x1 , x2 ,..., xm tha mn
(26)

a1 x1 r1 a2 x2 r2 ... am xm rm .

Chng minh. nh l ng vI m 2 v nu a1 , a2 l cc s nguyn t cng nhau th phng trnh

a1 x a2 y r2 r1 c nghim nguyn x v y . Gi s m l s t nhin ty 2 . Gi s nh l

18 | nh l s d Trung Hoa. nh l Thue. Cc s khng c c s chnh phng

ng vi m . t a1 , a2 ,..., am , am1 l cc s t nhin i mt nguyn t cng nhau v t

r1 , r2 ,..., rm , rm1 l cc s nguyn ty . T gi thit nh l ng vi m suy ra tn ti cc s nguyn


x1 , x2 ,..., xm tha mn (26). V cc s a1 , a2 ,..., am l nguyn t cng nhau vi am1 nn theo nh l
6a th s a1 a2 ... am nguyn t cng nhau vi am1 v do tn ti cc s nguyn t v u tha mn
a a ...a
a1 a2 ...am t am1 u rm1 a1 x1 r1. t xi 1 2 m t xi , vi i 1, 2,..., m v xm 1 u . R rng
al
cc s x1, x2 ,..., xm 1 nguyn v a1 x1 r1 a2 x2 r2 ... am1 xm 1 rm1 , theo quy np suy ra nh
l c chng minh.
T nh l 17 suy ra nu c hai trong m 2 s t nhin a1 , a2 ,..., am l nguyn t cng nhau v

r1 , r2 ,..., rm l cc s nguyn ty th tn ti s nguyn k tha mn khi chia k cho a1 , a2 ,..., am ta


ln lt nhn c cc s d r1 , r2 ,..., rm . y l l do nh l c gi l nh l cc s d.
Hin nhin khi cng vo k cc bi s ty ca s a1 a2 ...am th ta nhn c s nguyn m khi
chia cho a1 , a2 ,..., am cng ln lt cho cc s d r1 , r2 ,..., rm . Suy ra tn ti v hn cc s nguyn c
tnh cht ny.
Ta trnh by mt ng dng n gin ca nh l 17. Cho trc hai s t nhin m v s . Ta
chng minh trong mc 4 rng dy cc s Fk 22 1 k 0,1, 2,... l i mt nguyn t cng
k

s
nhau. t ai Fi v rl i vi mi i 1, 2,..., m . Vi c a1 x1 r1 cng thc (26) suy ra

Fi s xi ai xi a1 x1 r1 ri c i nn Fi s | c i vi mi i 1, 2,..., m. V Fi 1 vi i 1, 2,..., nn cc
s c 1, c 2,..., c m u chia ht cho ly tha bc s ca mt s t nhin ln hn 1 .
V vy ta c kt qu sau: vi mi s t nhin s tn ti dy di ty cc s t nhin lin tip m mi
s du chia ht cho mt ly tha bc s ca mt s t nhin ln hn 1 .
13. nh l Thue
nh l 18 (Thue [1]). Nu m l s t nhin v a l s nguyn nguyn t cng nhau vi m th tn
ti cc s t nhin x v y u nh hn m v cc s ax y u chia ht cho m vi cc du
thch hp.
Chng minh. nh l ng vi m 1 v trong trng hp ny ta t x y 1 . Gi s m l s t
nhin ln hn 1. K hiu q l s t nhin ln nht nh hn hoc bng

q 1

m . Khi q 1 m v

m. Xt biu thc ax y vi x, y nhn cc gi tr 0,1, 2,..., q . C ng q 1 m biu


2

thc nh vy m ch c th c m s d khc nhau c th nhn c khi chia mt s cho m nn


tn ti hai cp phn bit x1 , y1 v x2 , y2 vi x1 x2 m cc biu thc ax y c cng s d khi chia
cho m . H qu l s ax1 y1 ax2 y2 a x1 x2 y1 y2 chia ht cho m . Ta khng th c

x1 x2 v nu nh th th y1 y2 chia ht cho m m v 0 y1 q m m (v m 1 ) v
0 y2 m nn ta c mu thun v cc cp x1 , y1 v x2 , y2 l phn bit. ng thc y1 y2 cng

khng xy ra v nu nh th th a x1 x2 chia ht cho m m a nguyn t cng nhau vi m nn

m | x1 x2 v v 0 x1 q m, 0 x2 q v x1 x2 suy ra mu thun. Vy ta phi c x1 x2 v

y1 y2 . V x1 x2 , x x1 x2 l s t nhin. S y1 y2 c th l s nguyn m nhng n khc 0


nn y y1 y2 l s t nhin. Ta c x x1 x2 x1 q m , y q m v do chn cc du
+ v thch hp ta c a x1 x2 y1 y2 ax y chia ht cho m . iu phi chng minh.

CHNG 1. TNH CHIA HT V PHNG TRNH BT NH BC MT | 19

Vi mt s thay i t chng minh trn ta nhn c kt qu tng qut hn sau y c trnh


by bi Scholz v Schoenberg ([1] trang 44): nu m, e v f l cc s t nhin tha mn

e m, f m ef th vi s nguyn a m a, m 1 tn ti cc s nguyn x v y m vi cc du +
v thch hp ta c m | ax y v 0 x f , 0 y e . V cc tng qut khc ca nh l Thue c
th xem trong Brauner v Reynolds [1], Modell [6] v Nagell [6].
14. Cc s khng c c s chnh phng
S nguyn khng c c s chnh phng nu n khng chia ht cho mi bnh phng cc s t
nhin 1 . Cc s t nhin khng c c s chnh phng 20 l: 1, 2, 3, 5, 6, 7, 10, 11, 13, 14, 15,
17, 19.
T kt qu c chng minh trong mc 12 suy ra tn ti dy di ty cc s t nhin lin tip
m tt c cc s u c c s chnh phng. Trong bn s t nhin lin tip lun tn ti s c
c s chnh phng (v c t nht mt s chia ht cho 4 22 ). C th chng minh rng tn ti v
hn cc b ba s t nhin lin tip m cc s u khng c c s chnh phng.
C th chng minh rng mi s t nhin 1 u l tng ca hai s khng c c s chnh phng
v c v hn cch biu din s thnh hiu ca cc s nh vy (Nagell [1]. Sierpinski [36]). Hn
na mi s t nhin ln u l tng ca mt s khng c c s chnh phng v mt s t
nhin (Estermann [1], Hooley [1]). Ta chng minh nh l sau y
2
nh l 19. Mi s t nhin n u c biu din duy nht di dng n k l vi k v l l cc s t
nhin v l khng c c s chnh phng.

Chng minh. Vi mi s t nhin n , k hiu k l s t nhin ln nht tha mn k 2 | n . Ta c

n k 2l vi l l s t nhin. Nu l c c s chnh phng th t l r 2 s vi r , s l cc s t


nhin v r 1 . V vy n (kr )2 s v do (kr )2 | n vi kr k , mu thun vi nh ngha ca k .
By gi gi s n k12l1 vi k1 , l1 l cc s t nhin v l1 khng c c s chnh phng. t
2 2
2 2
d k , k1 ta c k dh, k1 dh1 vi h, h1 l cc s t nhin v h, h1 1 . V n d h l d h1 l1 , ta

2
2
2
c h l h1 l1 v v h2 , h12 1 nn theo nh l 5 suy ra h | l1 chng t h 1 v l1 khng c c s

chnh phng. Suy ra k dh d . Nhng v d | k1 ta c k | k1 suy ra k k1 m theo nh ngha ca

k v ng thc n k12l1 suy ra l l1.

20 | nh l s d Trung Hoa. nh l Thue. Cc s khng c c s chnh phng

CHNG 2
GII TCH DIOPHANTE BC HAI V CAO HN
1. Gii tch Diophantine mt bin
Gii tch Diophante l mt nhnh ca l thuyt s tp trung nghin cu cc phng trnh nghim
nguyn. Cc phng trnh ny c gi l phng trnh Diophante. Chng c t theo tn ca
nh ton hc Hy Lp Diophantus, ngi sng trong triu i Alexandria vo th k th 3 trc
Cng Nguyn v nghin cu v nhng phng trnh thuc dng ni trn t rt sm.
Ta bt u vi cc phng trnh mt bin bc ty .
Gi s v tri ca phng trnh l mt a thc vi h s nguyn, ngha l c dng
(1)

a0 x m a1 x m1 ... am1 x am 0,

trong m l s t nhin cho trc v a0 , a1 ,..., am l cc s nguyn vi a0 0 v am 0 . Nu


tn ti s nguyn x tha mn phng trnh (1) th

x m1 a1 x m2 ... am1 x am .

suy ra x phi l c s nguyn ca am . Mt khc v am khc 0 nn n ch c hu hn c s do


tt c cc nghim nguyn ca phng trnh (1) c th tm c bng hu hn cc php th. Ta ch
cn ln lt thay th cc c s (c m v dng) ca am vo phng trnh (1). Nu am 0 th r
rng x 0 l nghim ca phng trnh. Cc nghim khc l nghim ca phng trnh rt gn

a0 x m1 a1 x m2 ... am2 x am1 0


trong cc nghim tm c theo cch tng t cho trung hp am1 0 . Nu am1 0 th
phng trnh c rt gn xung bc m 2 v ta p dng quy np.
Xt v d phng trnh x7 x 2 0. Cc nghim nguyn ca phng trnh ny u l c s
nguyn ca 2 . Th trc tip ta thy ch c 1 tha mn phng trnh vy suy ra l nghim
nguyn duy nht cn tm.
Vi phng php va trnh by ta c th nhn thy v mt k thut th khng kh tm tt c cc
nghim nguyn cho cc phng trnh Diophante dng a thc. Tnh hung ny rt khc vi vic
gii cc phng trnh i s m nh chng ta bit th cng thc nghim tng qut cho cc a
thc bc ba v bn l rt phc tp. Hn na vi mt s a thc bc cao hn bn th cc nghim
khng th tm c bng cc php gii i s.
Tng t nh th, vic tm tt c cc nghim hu t ca cc a thc vi h s nguyn cng khng
phc tp hn. Gi s s hu t r tha mn phng trnh (1) vi h s nguyn a0 , a1 ,..., am . Ta gi
thit a0 0 v nu b qua trng hp r 0 th c th gi thit thm am 0 . Biu din r di
dng r k s , trong s l s t nhin, k l s nguyn v k , s 1. Thay x k s vo (1) ta c

a0 k m a1 k m1 a2 k m2 s ... am s m1 s,
am s m a0 k m1 a1 k m2 s ... am1 s m1 k .
Phng trnh th nht chng t s | a0 k m m k , s 1 suy ra s | a0 . Phng trnh th hai chng t

k | am s m m k , s 1 suy ra k | am . Vy nghim hu t ca phng trnh ban u c th tm c

k
trong k l c
s
s nguyn ca am v s l c s t nhin ca a0 sau tm xem phn s no tha mn.
bng hu hn cc php th nh sau: ln lt thay x bi cc phn s ti gin

22 | Gii phng trnh Diophante mt bin. Cc phng trnh Diophante nhiu bin

2. Cc phng trnh Diophante nhiu bin


i vi cc phng trnh Diophante nhiu bin c mt s vn sau y c t ra: phng
trnh c t nht mt nghim nguyn hay khng? S nghim nguyn ca phng trnh l hu hn hay
v hn? Tm tt c cc nghim nguyn ca phng trnh ?
Cc vn ny c mc kh tng dn theo trnh t c t ra trn. i vi mt s phng
trnh th ta khng c cu tr li cho c ba cu hi nu trn. V d hin nay ta cha bit phng
trnh x3 y3 z 3 30 c nghim hay khng. Nhng ta tm c t nht bn nghim nguyn ca
phng trnh x3 y 3 z 3 3 l x, y, z 1,1,1 , 4, 4, 5 , 4, 5, 4 , 5, 4, 4 . Hn na mc d ta
chng minh c phng trnh ny khng cn nghim no khc trong trng hp
x y z 150000 (Scarowsky v Boyarsky [1]) nhng ta vn khng bit n cn nhng nghim
khc hay khng. Mc phc tp ca php gii phng trnh ny c L.J.Mordell [5] so snh vi
s xut hin ca cc ch s 1,2,,9 trong biu din thp phn ca .
Mt v d khc, ta bit phng trnh x3 y3 z 3 2 c v hn nghim nguyn v ta tm c

mt h nghim v hn ca n l x, y, z 1 6n3 ,1 6n3 , 6n2 vi n l s t nhin ty . Nhng


ta vn cha bit c phi tt c cc nghim ca phng trnh ny hay khng. Mt khc ta c th
chng minh phng trnh x3 y3 z 3 4 khng c nghim nguyn bi v mt lp phng chia
cho 9 s c s d l 0,1 hoc 8, nh th tng ca hai lp phng khi chia cho 9 s d 0,1,2,7,8. Do
tng ca ba lp phng khi chia cho 9 s d 0,1,2,3,6,7,8 nhng khng th l 4 hoc 5. Do c
hai phng trnh x3 y3 z 3 4 v x3 y 3 z 3 5 u khng c nghim nguyn. Tng qut hn,
phng trnh x3 y3 z 3 k l v nghim trong trng hp k chia 9 d 4 hoc 5).
Phng trnh x3 y3 z 3 6 c cc nghim x, y, z 1, 1, 2 , 43, 58, 65 , 55, 235, 236
nhng ta khng bit s nghim ca phng trnh ny l hu hn hay khng.
Trong mt s trng hp kh khn ca vic tm tt c cc nghim nguyn ca mt phng trnh
ch thun ty l vn tnh ton s cp bi v ngay c khi ta bit phng php tm tt c cc
nghim vn c th tn rt nhiu thi gian tnh ton c th.
V d xt phng trnh xy 2293 1. Ta c th chng minh c phng trnh ny c nghim
khng tm thng ( x v y u ln hn 1) nhng ta khng th tnh c c th d cho trn l
thuyt ta c th em 2293 1 chia ln lt cho cc s nh hn 2293 1 tm c cc c s ca
n. Cc tnh ton ny i hi rt nhiu thi gian.
Mt khc ta cng cha bit c mt phng php no m sau mt qu trnh tnh ton hu hn c
th quyt nh rng phng trnh x3 y3 z 3 30 c th gii c hay khng. Tt nhin l c th
chng minh phng trnh ny khng c nghim nguyn dng kh d dng.
3. Phng trnh x 2 y 2 z 2
Ta xt mt phng trnh c bit bc hai vi ba bin: phng trnh Pythagoras
(2)

x2 y 2 z 2

Phng trnh ny c bit quan trng trong lnh vc tam gic lng v hnh hc gii tnh. Hn na
trng hp ring ca n khi x y c lin quan trc tip vi chng minh n gin nht cho s tn
ti ca cc s hu t.
Ta s tm tt c cc nghim hu t ca (2).
B qua cc nghim tm thng khi mt trong cc bin x, y trit tiu, ta ch xt cc nghim t
nhin v du ca bin s khng nh hng ti phng trnh. Nu x, y, z l cc s t nhin tha

CHNG 2. GII TCH DIOPHANTE BC HAI V CAO HN | 23

mn (2) th ta ni x, y, z l mt b s tam gic Pythagoras (xem thm cun sch ni ring v cc


tam gic ny c son bi Sierpinski [35]).
Mt nghim ca (2) c gi l nghim nguyn thy nu x, y, z l cc s t nhin v khng c c
s chung ln hn 1. Nu , , l nghim nguyn thy ca (2) v d l s t nhin ty th

x d ,

(3)

y d ,

z d

cng l nghim ca (2).


Ngc li nu x, y, z l nghim t nhin ca (2) th t x, y, z d ta c x d , y d , z d
trong , , 1 (Chng 1 nh l 3a ). Thay vo (2) ta c d d d . Chia c hai
2

v cho d 2 ta nhn c b s , , l nghim nguyn thy ca (2).


Ta ni rng nghim t nhin x, y, z ca (2) thuc lp th d nu

x, y, z d .

Ta nhn thy

nhn c tt c cc nghim t nhin thuc lp d ta ch cn nhn tt c cc nghim nguyn thy


ca (2) vi d . V vy ta ch cn tm tt c cc nghim nguyn thy ca (2).
Gi s rng x, y, z l nghim nguyn thy ca (2). Ta chng minh rng mt trong cc s x, y l
chn v s cn li l l. Gi s ngc li ngha l c hai s l cng chn hoc cng l. Trong
trng hp u tin th x 2 y 2 z 2 chn do z chn v v vy x, y, z c c chung l 2, mu
thun. ch ra trng hp th hai cng khng xy ra ta chng minh rng bnh phng ca mt
s l chia cho 8 d 1. Tht vy, mt s l bt k s c dng 2k 1 vi k nguyn no . Ta c biu
din 2k 1 4k 2 4k 1 4k k 1 1. Do mt trong hai s k v k 1 l s chn nn n chia
2

ht cho 2 v vy 4k k 1 chia ht cho 8 v do 2k 1 chia 8 d 1. H qu l tng ca hai


2

bnh phng l chia 8 d 2. Suy ra tng ca hai bnh phng l khng th l mt bnh phng l
v cng khng th l mt bnh phng chn v mt bnh phng chn s chia ht cho 4 nn chia 8
ch c th d 0 hoc 4. iu phi chng minh.
Ta c th gi thit y l chn v x l l. Khi z cng l l. Phng trnh (2) c th vit di dng
(4)

y 2 z x z x .

Cc s z x v z x l tng v hiu ca hai s l nn bn thn chng u l s chn. t


(5)

z x 2,

z x 2b,

trong a v b l cc s t nhin. V vy z a b, x a b . Cc ng thc ny suy ra a v b


nguyn t cng nhau v nu ngc li ta gi s chng c c s chung 1 th z k , x l

trong k v l l cc s t nhin. Khi y 2 z 2 x 2 k 2 l 2 2 trong y 2 chia ht cho 2


suy ra y chia ht cho (Chng 1 mc 6 H qu 2). iu ny mu thun vi gi thit x, y, z l
nghim nguyn thy v 1 li tr thnh c s chung ln hn 1 ca x, y, z.
Theo gi thit y chn nn y 2c trong c l s t nhin no . Thay vo (5) v rt gn,
phng trnh (4) tr thnh
(6)

c 2 ab.

Nhng do a, b 1 , p dng nh l 8 Chng 1, ng thc (6) suy ra cc s a, b u l bnh


phng. Ngha l a m2 , b n2 , trong m, n l s t nhin v m, n 1 (do a, b 1 ). V vy

z a b m2 n 2 ,
v do c 2 ab m2 n2 m y = 2c nn y = 2mn.

x a b m2 n 2 ,

24 | Phng trnh Diophante nhiu bin. Phng trnh

x2 y 2 z 2

Vy ta chng minh c nu x, y, z l nghim nguyn thy ca (2) v y l s chn th


(7)

x m2 n 2 ,

y 2mn,

z m2 n 2 ,

trong m, n l s t nhin vi m, n 1 v m n v x l s t nhin. Hn na mt trong cc s


m, n l chn, s cn li l l. Tht vy, chng khng th cng chn v chng nguyn t cng nhau.
Chng cng khng th cng l bi v nu chng cng l th t (7) suy ra tt c cc s x, y, z phi
chn, mu thun vi x, y, z 1 . Vy 2 | mn suy ra y 2mn chia ht cho 4.
Ta chng mnh iu ngc li cng ng, ngha l nu m, n l hai s t nhin nguyn t cng nhau,
m n v mt trong hai s l l, s cn li l chn th cc s x, y, z nhn c t m, n theo cng
thc (7) tr thnh nghim nguyn thy ca (2).
u tin ta ch rng cc s x, y, z thu c bi cng thc (7), m, n l cc s t nhin v m n
tr thnh nghim ca (2). Ta ch cn kim tra rng
(8)

n2 2mn m2 n2 .
2

By gi s dng gi thit m, n nguyn t cng nhau ta chng minh x, y, z 1 . Gi s ngc li


th tn ti c s chung 1 ca cc s x, y, z . S khng th chn v z m2 n2 l tng ca
mt bnh phng l v mt bnh phng chn s l l. Nhng theo (7)
(9)

2m2 x z,

2n2 z x;

Do m 2 v n 2 u chia ht cho . iu ny khng th v m, n 1 suy ra m2 , n2 1 . Cng


thc (9) chng t rng ng vi cc s m, n khc nhau ta c cc nghim x, y, z khc nhau. Cc kt
qu trn dn ta ti vi nh l sau y
nh l 1. Tt c cc nghim nguyn thy ca phng trnh x 2 y 2 z 2 vi y chn c cho bi
(10)

x m2 n 2 ,

y 2mn,

z m2 n 2 ,

vi m, n l cc s t nhin nguyn t cng nhau v trong c mt s chn, mt s l, m > n .


J.Ginsburg [1] lu i vi mt nghim nguyn thy ca phng trnh x 2 y 2 z 2 th lit
k mt cch c h thng cc cp s m, n tha mn iu kin trong nh l 1 (i khi cn gi l
phn t sinh ca nghim) th ch cn xt t s x z y nh l mt phn s ti gin m n l .
lit k mt cch trnh t tt c cc nghim nguyn thy ca (2) ta ln lt xt cc gi tr 2, 3, 4,
ca m v vi mi gi tr ta xt cc s n nguyn t cng nhau vi m , nh hn m v l s
chn nu m l. Theo cch ny ta c bng 20 nghim nguyn thy u tin nh sau

CHNG 2. GII TCH DIOPHANTE BC HAI V CAO HN | 25

Nh bit thu c tt c cc nghim t nhin ca phng trnh (2) ta phi nhn mi nghim
nguyn thy ln lt vi cc s t nhin 1, 2, 3, ri b sung thm nghim bng cch i vai tr
ca x v y . Hn na mi nghim t nhin ca (2) s thu c mt ln duy nht theo cch ny.
Mt khc theo ng thc (8) th khi thay cc s t nhin m, n vi m n vo cng thc (7) ta thu
c nghim t nhin ca (2). Theo cch ny th d cho b sung thm cc nghim bng cch i
vai tr ca x v y ta cng khng thu c tt c cc nghim t nhin ca (2). Chng hn ta khng
th thu c t (7) nghim 9,12,15 do khng tn ti s t nhin m v n m tha mn
15 m2 n2 (bi v khng c s no trong cc s 15 12 14, 15 22 11, 15 32 6 l bnh
phng ng).
Tt c cc nghim ca (2) c cho bi cng thc sau y

x m2 n 2 l ,

y 2mnl ,

z m2 n 2 l ,

trong m, n m v l l s t nhin, sau b sung thm cc nghim thu c bng cch i


vai tr x v y . Tuy nhin cng thc trn li cho cc nghim trng nhau i vi mt s b s
m, n, l khc nhau. Chng hn nghim 12,16,20 thu c khi m 2, n 1, l 4 v
m 4, n 2, l 1. Nghim 48,64,80 thu c khi m 8, n 4, l 1 ; m 4, n 2, l 4 v
m 2, n 1, l 16.
Nghim u tin c lit k trong bng trn l nghim ca phng trnh (2) vi x, y, z l cc s
t nhin nh nht c th. Hn na trong nghim ny th cc s x, y, z cn l s t nhin lin tip.
Khng kh chng minh chnh l nghim duy nht cha cc s t nhin lin tip. Tht vy,
nu ba s t nhin lin tip n 1, n, n 1 tha mn phng trnh n 1 n2 n 1 th n2 4n
2

suy ra n 4 v ta thu li nghim 3,4,5. D dng chng minh phng trnh 3n 4n 5n khng c
nghim t nhin n no tr nghim n 2 . Tht vy, ta c 3 4 5 do n 1 khng phi nghim
cn tm. Hn na 32 42 52 v vi n 2 th

5n 52 5n2 32 5n2 42 5n2 32 3n2 42 4n2 3n 4n.


V vy 3n 4n 5n vi mi n 2 . Tng t ta chng minh c nu a 2 b2 c 2 th a n bn cn
vi mi n 2 . Mt khc phng trnh 3x 4 y 5z ch c nghim t nhin duy nht x y z 2
nhng chng minh iu ny khng d (Sierpinski [17], Nagell [11]).
L.Jesmanowicz [1] chng minh cc phng trnh 5x 12 y 13z , 72 24 y 25z , 9x 40 y 41z ,
11x 60 y 61z ch c nghim t nhin x y z 2 v t t ra cu hi rng c tn ti cc s
t nhin a, b, c tha mn a 2 b2 c 2 m phng trnh a x b y c z c nhiu hn mt nghim
x y z 2 hay khng (Ko Chao [2],[3],[4]).
Ngoi ra ta bit c v hn cc b s Pythagoras nguyn thy

a, b, c

m phng trnh

a x b y c z ch c ng mt nghim t nhin x y z 2 (Lu Wen Twan [1], Jozefiak [2],


Podsypanin [1], Demyanenko [1]).
Ta chng minh rng vi mi nghim nguyn thy ca (2) th s chn trong cc s x, y s chia
ht cho 4. V vy trong mi nghim x, y, z ca (2) th c t nht mt trong hai s x, y chia ht cho
4. Ta s chng minh trong hai s cng c mt s chia ht cho 3. Gi s ngc li ta vit
x 3k 1 , y 3l 1, k v l l s nguyn. V vy x 2 y 2 3 3k 2 3l 2 2k 2l 2 . Nhng y

khng th l bnh phng ng v bnh phng ca mt s ch c th chia ht cho 3 hoc chia 3 d


2
1, iu ny l hin nhin v ta c biu din 3t 1 3 3t 2 2t 1 .
By gi ta s chng minh rng trong mi nghim t nhin ca (2) th t nht mt trong cc s
x, y, z chia ht cho 5. chng minh iu ny ta hy xt mt s nguyn ty m khng chia ht

26 | Nghim t nhin ca phng trnh

x2 y 2 z 2

vi x y 1

cho 5. Ta c m 5k 1 hoc m 5k 2 trong k l s nguyn no . Trong trng hp th


nht th m2 5 5k 2 2k 1 v trong trng hp th hai m2 5 5k 2 4k 4 . Nh th bnh

phng ca mt s khng chia ht cho 5 khi chia cho 5 s c s d l 1 hoc 4. By gi gi s


x, y, z u khng chia ht cho 5 th khi x 2 v y 2 chia 5 d 1 hoc 4 suy ra x 2 y 2 chia 5 d 2,3
hoc 0. Do x 2 y 2 z 2 nn loi tr hai trng hp u tin v z 2 chia 5 khng th c s d l 2
hoc 3. V vy trng hp th ba phi xy ra. Suy ra z chia ht cho 5. iu phi chng minh.
Do 3, 4,5 l tam gic Pythagoras suy ra cc s 1,2,3,4,5 l tt c cc s t nhin n m ta c th
kt lun rng mi b s Pythagoras u c t nht mt cnh chia ht cho n .
By gi ta s tm tt c cc nghim ca (2) m hai trong s cc s x, y, z l s t nhin lin tip. R
rng cc nghim nh th u l nghim nguyn thy. V vy z l s l v z y 1 ch c th xy ra
khi y chn. Do theo (10) ta c m2 n2 2mn z y 1 hoc tng ng m 2 1 trong
2

t m n suy ra m n 1 ngha l m n 1 . V vy x m2 n2 n 1 n 2 2n 1 ,
2

y 2n n 1 , z y 1 2n(n 1) 1 . Do tt c cc nghim ca (2) m z y 1 u c cho


bi cng thc x 2n 1, y 2n(n 1), z 2n(n 1) 1 vi n 1, 2,3, ...
Di y l 10 nghim u tin

V mt s nghim khc thuc dng ny

V cc nghim ny xem thm Willey [1]. Tip theo ta xt trng hp x y 1 .


4. Nghim t nhin ca phng trnh x 2 y 2 z 2 vi x y 1
Trong cc nghim nguyn thy ca (2) c lit k trong mc 3 ta ch hai nghim 3,4,5 v
21,20,29. D dng chng minh rng c v hn nghim tha mn tnh cht trong mc ny. Tht vy,
nu ta c hai s t nhin x v z tha mn x 2 x 1 z 2 th ta c
2

3x 2 z 1 3x 2 z 2
2

4 x 3z 2 .
2

Tht vy, ta c 3x 2 z 1 3x 2 z 2 18 x 2 24 xz 8 z 2 18 x 12 z 5
2

T x 2 x 1 z 2 suy ra 2 x2 2 x 1 z 2 trong
2

CHNG 2. GII TCH DIOPHANTE BC HAI V CAO HN | 27

3x 2 z 1

3x 2 z 2 16 x 2 24 xz 9 z 2 16 x 12 z 4
2

4 x 3z 2 .
2

V vy t mt tam gic Pythagoras cho trc gm hai cnh gc vung l hai s t nhin lin tip
th ta c th thu c mt tam gic Pythagoras khc vi cng tnh cht. Bt u vi tam gic
nguyn thy 3,4,5 ta thu c tam gic mi c cc cnh l 3 3 2 5 1 20 , 21 v
4 3 3 5 2 29 . Tng t t tam gic mi ny ta thu c tam gic vi cc cnh
3 20 2 29 1 119 , 120 v 4 20 3 29 2 169 . Di y l su tam gic u tin thu c
theo cch ny

Khng kh chng minh rng qu trnh ny cho mt dy cc tam gic m cnh gc vung c
di ln hn s ln lt l chn v l. t x1 3 , y1 4 , z1 5 v vi n 1, 2,3,... t
(11)

xn1 3xn 2 zn 1 , yn1 xn1 1 , zn1 4 xn 3zn 2 .

Ta s chng minh xn , yn , zn n 1, 2,... u l cc tam gic Pythagoras vi cc cnh gc vung l


cc s t nhin lin tip.
B . Nu cc s t nhin x, z tha mn phng trnh
(12)

x 2 x 1 z 2
2

v nu x 3 th
(13)

x0 3x 2 z 1 ,

z0 3z 4 x 2

l cc s t nhin tha mn phng trnh


(14)

x02 x0 1 z02 ,
2

v z0 z .
Chng minh. Theo (13) ta c
(15)

x02 x0 1 2 x02 2 x0 1 18x 2 8z 2 24 xz 18x 12 z 5 ,


2

z02 16 x2 9 z 2 24 xz 16 x 12 z 4 .
Theo (2) th z 2 2 x2 2 x 1 . Ta c

16 x2 9 z 2 24 xz 16 x 12 z 4 8z 2 18x2 24 xz 18x 12 z 5
V t (15) suy ra (14).
Theo (13) ta cn chng minh x0 , z0 l cc s t nhin v z0 z .
Tc l cn chng minh 3x 2 z 1 0 v 0 3z 4 x 2 z hoc tng ng
(16)

2 z 3x 1 ,

3z 4 x 2

Do x 3 suy ra x2 3x 2 x 3 . Theo (12) th

z 2x 1.

28 | Nghim t nhin ca phng trnh

x2 y 2 z 2

vi x y 1

4 z 2 8x 2 8x 4 9 x 2 8x 4 x 2

9 x 2 8x 4 2 x 3 9 x 2 6 x 1 3x 1 .
2

Suy ra 2 z 3x 1 v do x 0 , 2 z 4 x 1 ta c z 2 x 1 . S dng (12) v x 0 suy ra

9 z 2 18x 2 18x 9 16 x 2 16 x 4 4 x 2 ,
2

trong 3z 4 x 2 v ta chng minh c (16). B c chng minh.


By gi gi s tn ti cc tam gic Pythagoras x, x 1, z khc vi cc tam gic xn , xn 1, zn
nh ngha trn. Trong s cc tam gic nh vy tn ti tam gic x, y, z vi z nh nht. Khi

x khng th nh hn hoc bng 3 bi v nu ngc li ta c x, y, z 3, 4,5 . t


u 3x 2 z 1 , v 3 z 4 x 2 .

(17)

Theo b th u, u 1, v l tam gic Pythagoras vi v z . Do z l nh nht trong cc tam gic


Pythagoras khng c dng xn , xn 1, zn nn vi n no ta c u xn , v zn v

xn1 3u 2v 1 , yn1 xn1 1 , zn1 4u 3v 2 .


V vy, theo (17)

xn1 3 3x 2 z 1 2 3z 4 x 2 1 x ,
zn1 4 3x 2 z 1 3 3z 4 x 2 2 z .

Do tam gic x, x 1, z li l mt trong s cc tam gic xn , yn , zn . Mu thun. Vy ta chng


minh c cc tam gic xn , xn 1, zn n 1, 2,... l tt c cc tam gic Pythagoras m trong
hai cnh gc vung l cc s t nhin lin tip.
C th chng minh c nu dy v hn u1 , u2 ,... v v1 , v2 ,... c nh ngha nh sau: u0 0 ,

u1 3 , un1 6un un1 2 vi n 1, 2,... v v0 1 , v1 5 , vn1 6vn vn1 vi n 1, 2,..., th


un2 un 1 vn2 vi n 1, 2,..., v un , un 1, vn l tam gic th n trong dy (11).
2

Ta cng c th chng minh rng nu 1 2

2 n 1

an bn 2 vi n 1, 2,..., an v bn l cc s

an 1n an 1n

,
, bn l tam gic th n trong dy (11).
nguyn th

2
2

By gi ta gi s cc s t nhin x v z tha mn (12). Do mt trong hai s x, x 1 l chn, s cn


1
2
li l l, z l l v r rng z x 1 . Hn na z 2 2 x 1 . V th u z x 1 v v 2 x 1 z
2
l cc s t nhin. V vy theo ng thc

z x 1 z x x 1 z
2

v x 2 x 1 z 2 ta thu c
2

(18)

1
u u 1 v 2 .
2

1 2
2
2
z x x 1
4

CHNG 2. GII TCH DIOPHANTE BC HAI V CAO HN | 29

S tu

1
u u 1 vi u l s t nhin c gi l s tam gic (xem mc 16).
2

Cng thc (18) chng t rng s tam gic l mt bnh phng ng.
V vy ng vi mi nghim t nhin ca phng trnh x 2 x 1 z 2 u cho mt nghim t
2

nhin ca (18) bng cch t u z x 1 , v x 1 z / 2 . iu ngc li cng ng: nu nghim


t nhin u v v tha mn (18) th ta t x u 2v , z 2u 2v 1 . S dng ng thc

u 2v u 2v 1 2u 2v 1
2

4 v 2 u u 1
2

1
2 x 1 z . Nh vy
2
2
cc cng thc ny bin i tt c cc nghim t nhin x, z ca phng trnh x 2 x 1 z 2
Ta thu c nghim ca phng trnh x 2 x 1 z 2 v u z x 1 , v
2

thnh tt c cc nghim t nhin u v v ca (18) hoc ni cch khc l thnh tt c cc s tam


gic l bnh phng ng. T y suy ra tnh v hn ca cc s tam gic loi ny. Ta lit k su s
tam gic nh th

t1 12 , t8 62 , t49 352 , t288 2042 , t1681 11892 , t9800 69302 .


Suy ra ng thc
(19)

2 z 2 x 1

2 2 x z 1 1 2 z 2 x 2 x 1
2

Ngha l nu cc s t nhin x , z tha mn (12) th t


(20)

a 2z 2x 1, b 2 x z 1 ,

Ta thu c
(21)

a 2 2b2 1 ,

vi a, b l cc s t nhin v theo (12) ta c z 2 x 1 v 4 z 2 2 x 1 trong 2 z 2 x 1 .


2

Cng thc (20) tng ng vi


(22)

x b

1
a 1 , z a b .
2

Nu a v b l cc s t nhin v tha mn (21) th a l s l ln hn 1 v cc s c cho bi


(22) u l s t nhin. Hn na t (20) suy ra (22) nn theo (21), (20) v (19) ta thy x v z
tha mn (12). Vy t tt c cc nghim t nhin x, z ca (12), s dng cng thc (20), ta thu
c tt c cc nghim t nhin a v b ca (21). Bn nghim u tin ca (12) c lit k
trn cho ta bn nghim a, b ca phng trnh 21 : 3, 2 , 17,12 , 99,70 , 577, 408 .

Ngc li, t tt c cc nghim t nhin ca (21), s dng cng thc (22) ta thu c tt c cc
nghim ca phng trnh (12).
5. Cc tam gic Pythagoras c cng din tch
T danh sch cc tam gic Pythagoras uc lit k trong mc 1 ta nhn thy cc tam gic (21, 20,
29) v (35, 12, 37) c cng din tch (210 n v din tch) v v th c t nht hai tam gic nguyn
thy Pythagoras vi cnh huyn khc nhau c cng din tch.
Xt cc tam gic khng nguyn thy vi cnh huyn 37 ta nhn c 8 tam gic 6,8,10 ,

9,12,15 , 12,16, 20 , 15, 20, 25 , 10, 24, 26 , 18, 24,30 , 30,16,34 , 21, 28,35 vi din tch

30 | Cc tam gic Pythagoras c cng din tch

ln lt l 24, 54, 96, 150, 120, 216, 240, 294. V vy ta nhn thy khng c cp tam gic no trong
cc tam gic Pythagoras vi cnh huyn 37 c cng din tch tr cp 21, 20, 29 , 35,12,37 .

Ta ch rng nu hai tam gic Pythagoras c cng din tch v cc cnh huyn bng nhau th

chng trng nhau. Tht vy, nu a1 , b1 , c1 v a2 , b2 , c2 l cp tam gic nh vy vi a1 b1 ,

a2 b2 th theo gi thit ta c cc ng thc a1 b1 a2 b2 v c1 c2 trong a12 b12 a22 b22 suy


ra

a1 b1

a2 b2 v
2

a1 b1

a2 b2 vi a1 b1 a2 b2 v a1 b1 a2 b2 suy ra
2

a1 a2 v b1 b2 .
Theo danh sch trong mc 3 ta xt tam gic Pythagoras (15, 112, 113). Tam gic ny c din tch l
840 4 210 tc l bn ln ln hn din tch tam gic (21, 20, 29) v (35, 12, 37). Nhn hai v ca
cc tam gic ln hai ln ta ln lt thu c hai tam gic (42, 40, 58) v (70, 24, 74) vi din
tch ng bng 840. V vy ta nhn c ba tam gic Pythagoras (15, 112, 113), (42, 40, 58), (70,
24, 74) u c cng din tch. Tt c cc tam gic ny u khng nguyn thy. Ta bit rng din
tch chung nh nht ca ba tam gic nguyn thy l 13123110 v cc tam gic l (4485, 5852,
7373), (19019, 1390, 19069), (3059, 8580, 9089). Cc phn t sinh tng ng ln lt l (39, 38),
(138, 5), (78, 55). Cu hi t ra l khi no th tn ti mt s lng ln cc tam gic Pythagoras c
cng din tch v c cnh huyn khc nhau. Kt qu cho cu hi ny c trong nh l sau y.
nh l 2 (Fermat). Vi mi s t nhin n u tn ti n tam gic Pythagoras c cng din tch v
cc cnh huyn khc nhau.
nh l c chng minh bng quy np da trn kt qu sau y
B . Cho trc n tam gic Pythagoras c cng din tch v cc cnh huyn khc nhau. Ta c c
th xy dng n 1 tam gic Pythagoras vi cng din tch v cnh huyn khc nhau trong t nht
c mt tam gic c cnh huyn l.
Chng minh. Cho trc s t nhin n . Gi s ak , bk , ck vi ak bk ck , k 1, 2,..., n l n tam
gic Pythagoras vi cng din tch v cc cnh huyn khc nhau, hn na c1 l. t

ak 2c1 b12 a12 ak , bk 2c1 b12 a12 bk ,

(23)

ck 2c1 b12 a12 ck , vi k 1, 2,..., n

an 1 b12 a12 , bn1 4a1b1c12 , cn 1 4a12b12 c14 .


2

(24)

vi k 1, 2,..., n . Khi cc tam gic ak , bk , ck u l tam gic Pythagoras do chng ln lt


ng dng vi cc tam gic ak , bk , ck , k 1, 2,..., n . Hn na an 1 , bn1 , cn 1 cng l tam gic
Pythagoras. iu ny c suy trc tip t (24), phng trnh a12 b12 c12 v ng thc

b2 a2 16a2b2 a2 b2 4a2b2 a2 b2
4

2 2

By gi ta chng minh cc tam gic ak , bk , ck vi k 1, 2,..., n 1 tha mn cc iu kin cn li.


K hiu l din tch ca cc tam gic ak , bk , ck , k 1, 2,..., n . Ta c ak bk 2 vi k 1, 2,..., n .
Din
1
2

tch

ca

cc

ak bk 2c12 b12 a

bng vi

2 2
1

1
2

tam

ak , bk , ck

gic

ak bk 4c12 b12 a

2 2
1

an 1bn1 2 b12 a

2 2
1

vi

k 1, 2,..., n

. Din tch ca tam gic

c12 a1b1 4c12 b12 a

2 2
1

theo

(23)

an 1 , bn1 , cn 1 ,

. V vy cc tam gic

bng

theo (24),

ak , bk , ck

vi

k 1, 2,..., n 1 u c cng din tch. chng minh rng cnh huyn ca cc tam gic ny u

CHNG 2. GII TCH DIOPHANTE BC HAI V CAO HN | 31

khc nhau ta ch rng cc s ck , k 1, 2,..., n, u khc nhau. Mt khc theo (23), ck' k n u
l s chn. Ta li c theo (24) th s cn' 1 l l do c1 l. B c chng minh.
p dng b cho trng hp n 1 . Tam gic Pythagoras nh nht c p dng trong b l
'
'
'
'
'
'
3, 4,5 . Ta c hai tam gic c cng din tch a1 , b1 , c1 v a2 , b2 , c2 th theo (23) ta c

2 b12 a12 c1 2 7 5 70 v do a1' 3 70 210, b1' 4 70 280, c1' 5 70 350 . Nn theo

(24) th a2' 42 32 49, b2' 4 3 4 52 1200, c2' 4 32 42 52 1201. Ta c hai tam gic
2

Pythagoras cn tm l (210, 280, 350) v (49, 1200, 1201) c cng din tch l 29400 v cc cnh
huyn khc nhau, mt trong s l l. p dng b ln na cho hai tam gic ny ta thu c ba
tam gic Pythagoras vi cnh huyn khc nhau v cng din tch nhng cc cnh ca chng u
ln hn 1010 . Mt khc, s dng mt phng php khc ta tm ra ba tam gic nh vy nhng
cc cnh u nh hn 104 . Hn na cng tn ti bn tam gic Pythagoras vi cnh huyn khc
nhau v c cng din tch vi di cc cnh u nh hn 105 . l cc tam gic
518, 1320, 1418 , 280, 2442, 2458 , 231, 2960, 2969 , 111, 6160, 6161 v din tch chung ca
chng l 314880 . Nm tam gic Pythagoras vi cnh nh hn 106 c cnh huyn khc nhau v c
cng din tch l cc tam gic phn bit (2805, 52416, 52491) , (3168, 46410, 46518) ,
(5236,14040, 28564) , (6006, 24480, 25206) , (8580, 17136, 19164) vi din tch chung 73513440.
Hin nhin l ch tn ti hu hn tam gic vi din tch cho trc. Hn na cc cnh gc vung
ca tam gic phi l c s ca 2 .
Mt khc t b d dng suy ra tn ti v hn cc tam gic vung vi cnh hu t v din tch l
6. Tht vy, t chng minh ca b suy ra nu ta c n tam gic Pythagoras c cng din tch
vi cc cnh huyn phn bit v mt trong s chng l l th tn ti n 1 tam gic Pythagoras c
cng din tch d 2 vi cnh huyn phn bit v mt trong s chng l l, trong d l mt s t
nhin. Bt u vi tam gic 3, 4, 5 v s dng b n 1 ln ta thu c n tam gic Pythagoras
vi cnh huyn phn bit v c cng din tch l 6m2 , trong m l s t nhin ph thuc vo n .
Chia tt c cc cnh ca cc tam gic ny cho m ta nhn c n tam gic vung khng trng nhau
vi di cc cnh hu t v din tch u bng 6. Do n l s t nhin ty nn ta suy ra s tam
gic vung c cnh hu t v din tch bng 6 khng th hu hn c.
C th chng minh kh n gin rng vi mi s t nhin n u tn ti n tam gic Pythagoras
khng trng nhau vi cng chu vi. Tht vy, khng c hai tam gic nguyn thy khng trng nhau
no l ng dng, nhng s tam gic nh vy l v hn, ta c th chn n tam gic khng trng
nhau ak , bk , ck k 1, 2,..., n nh vy v t ak bk ck sk vi k 1, 2,..., n .
Vi k 1, 2,..., n t

s s1 s2 ... sn ,

ak'

ak s
bs
cs
, bk' k , ck' k .
sk
sk
sk

Ta c ak' bk' ck' s vi k 1, 2,..., n v hn na khng c cp ak' , bk' , ck'

k 1, 2,..., n

no

ng dng. Do chng cng khng trng nhau. Danh sch tt c cc tam gic Pythagoras nguyn
thy vi cnh nh hn 10000 c dn y bi A.A.Krishnawami trong [1]. Hai tam gic b
thiu c tm ra bi D.H.Lehmer trong [5]. Kim tra trc tip ta thy c 70 tam gic vi chu vi
khng vt qu 1000 v c 703 tam gic c chu vi khng vt qu 10000.
D dng chng minh vi mi s t nhin s u tn ti tam gic nguyn thy m chu vi ca n l
ly tha bc s ca mt s t nhin. Tht vy, xt t l s t nhin s 1 v t
s

1
1
s 1

m 2 t , n 2t 1 m . Do t s ta c 1 1 1
. Hn na t s 1 suy
2s 2
2t 2s
s 1 2

32 | V cc bnh phng c tng v hiu u l bnh phng

ra 2t 1 2s 1 t s . T n l s t nhin nh hn m (v 2t 1 2s t s 2m ). Hin nhin


s

m, n 1 . Tm cc s x, y, z

t cng thc (9) ta thu c tam gic Pythagoras m chu vi ca n l

x y z 2m mm n 2t 2t 1 .
s

Vi s 2 ta c tam gic (63, 16, 65) vi chu vi l 122 .


D dng tm tt c cc tam gic Pythagoras c ln din tch v chu vi bng nhau (Comberousse
[1], trang 190-191). Cc cnh x, y, z ca cc tam gic nh vy tha mn cc phng trnh
1
x 2 y 2 z 2 v x y z xy . Th z vo ta thu c phng trnh
2
(25)

x 4 y 4 8 .

Suy ra x 4 | 8 . Ta khng th c x 4 0 v nu x 4 1 hoc x 4 2 ta s ln lt c


y 4 8 hoc y 4 2 suy ra y 4 hoc y 0 , v l. Nu x 4 4 hoc x 4 8 th
x 0 , v l. Vy x 4 0 v do v x 4 | 8 suy ra x 4 1, 2, 4 hoc 8 v do x 5, 6,8 hoc

12 . T s dng (25) ta thu c y 12,8,6 hoc 5 . T y ta thu c hai tam gic

5,

12, 13 v 6, 8, 10 . R rng hai tam gic ny khng trng nhau. Din tch v chu vi ca tam

gic th nht l 30 v th hai l 24.


D dng chng minh tn ti v hn tam gic Pythagoras c cnh l hu t v din tch ca chng
c ln bng chu vi. C th chng minh rng tt c cc tam gic u, v, w nh vy u c dng

2 m n
u
,
n

4m
v
,
mn

2 m2 n 2

m n n

trong m v n m l cc s t nhin.
6. V cc bnh phng c tng v hiu u l bnh phng
Ta s nghin cu v s tn ti cc s t nhin x, y, z, t tha mn
(26)

x2 y 2 z 2 ,

x2 y 2 t 2

Ni cch khc ta s i tm cc s t nhin x v y tha mn tng v hiu cc bnh phng ca chng


u l bnh phng ng. Ta c nh l Fermat sau y
nh l 3. Khng tn ti cc s t nhin m tng v hiu cc bnh phng ca chng u l bnh
phng ng.
Chng minh. Gi s tn ti cc s t nhin x v y tha mn x 2 y 2 z 2 v x 2 y 2 t 2 vi z v

t l cc s t nhin v z t . Trong tt c cc cp s x, y nh vy tn ti cp s m x 2 y 2 l nh
nht. K hiu x, y l cp s . Ta c x, y 1 v nu d | x v d | y th do x 2 y 2 z 2 , x2 y 2 t 2
ta s suy ra d 2 | z 2 , d 2 | t 2 do d | z v d | t t suy ra phng trnh c th gin c cho d 2 m
ta gi thit x, y l b s m x 2 y 2 nh nht c th, mu thun. Vy d 1 .
Theo (26) suy ra 2x 2 z 2 t 2 . V vy cc s z v t c cng tnh chn l. Suy ra cc s z t v
1
1
z t u chn v v vy z t v z t l s t nhin.
2
2
Nu d |

1
1
z t v d | z t v d ln hn 1 th d | z . Do
2
2

CHNG 2. GII TCH DIOPHANTE BC HAI V CAO HN | 33

z t z t
x2

2 2
2

(27)

suy ra d 2 | x 2 v d | x . T v x 2 y 2 z 2 ta cng c d | y , mu thun vi x, y 1 . V vy

z t z t
,

1
2
2

(28

1
1
z t , z t , x to thnh mt nghim nguyn thy ca
2
2
phng trnh Pythagoras, theo nh l 1 th suy ra tn ti cc s t nhin nguyn t cng nhau
m, n vi m n v mt trong hai s l chn, s kia l, trong 1 z t m2 n2 , 1 z t 2mn
2
2
1
1
hoc z t m2 n 2 , z t 2mn . Do 2y 2 z 2 t 2 nn trong c hai trng hp ta c
2
2
2
2
2
2 y 2 m n 4mn hay y 2 m2 n2 4mn . Do y chn nn y 2k vi k l s t nhin no .
T (28) v (27) ta nhn thy cc s

S dng cng thc ca y 2 ta nhn uc

(29)

n2 mn k 2

Do m, n 1 ta c m n, m 1 v vy m2 n2 , m 1 v m2 n2 , n 1 .
T (29) p dng h qu nh l 8 Chng 1, tt c cc s m2 n2 , m, n l bnh phng ng, v vy

m a 2 , n b 2 , m 2 n 2 c 2 , vi a, b, c l cc s t nhin.
Do m, n 1 v mt trong cc s m, n l chn v s cn li l l nn ta thu c m n, m n 1 .
Tht vy, mi c s chung ca cc s l m n v m n u l, nhng n cng l c ca 2m v

2n v vy t m, n 1 suy ra c s ny bng 1.

T m n, m n 1 v m n m n m n c suy ra m n v m n l cc bnh phng.


2

V vy t m a 2 , n b2 suy ra cc s a 2 b2 v a 2 b2 cng l cc bnh phng.


Nhng a 2 b2 x 2 y 2 , tri vi gi thit trn ca cp x, y . Ta c iu phi chng minh.
Mt khc tn ti v hn cc cp s t nhin x, y m tn ti s t nhin z v t tha mn

x2 y 2 z 2 1 v x 2 y 2 t 2 1 . Chng hn q chn th vi x

q4
1, y q 3 ta c
2

x 2 y 2 q 2 q 4 / 2 1, x2 y 2 (q 4 / 2 q 2 )2 1
2

2n 2n

Ta cng c 2n2

Tn ti cp s t nhin

1 1 vi n 1, 2,...
2

x, y tha mn vi cc s t nhin z, t no ta c

x y z 1, x y t 1 , chng hn 212 122 142 1,212 122 102 1.


2

Khng kh tm cc s t nhin x, y m tn ti cc s t nhin z , t tha mn x 2 y 2 z1 1 v

x2 y 2 t 2 1, chng hn
132 112 172 1 , 132 112 72 1, 892 792 1192 1,892 792 412 1.
T nh l (3) suy ra h phng trnh sau

34 | V cc bnh phng c tng v hiu u l bnh phng

(*)

x2 y 2 u 2

x2 2 y 2 v2

khng c nghim t nhin x, y, u, v .


Tht vy nu tn ti cc s t nhin x, y, u, v tha mn (*) th u 2 y 2 v 2 , u 2 y 2 x 2 , mu thun
vi nh l 3.
H qu 1. Khng tn ti cc s t nhin a, b, c tha mn a 4 b4 c.
Chng minh. Nu tn ti cc s t a, b, c nh vy th ta gi s a, b 1 v nu a, b d 1 th
t a da1, b bd1 ta c d 4 (a14 b14 ) c 2 suy ra d 2 | c 2 v vy c d 2c1 v ta c a14 b14 c12 vi

a1, b1 1. Gi s a, b 1, suy ra a 2 , b2 1 v vy phng trnh tr b

c2 a 4 . Cc s b2 , c, a 2

to thnh mt nghim nguyn thy ca phng trnh Pythagoras. Khi theo nh l 1 ta suy ra
tn ti cc s t nhin m, n, m n, tha mn a 2 m2 n2 v b2 m2 n2 hoc b2 2mn. Trng
hp th nht l khng th xy ra v n mu thun vi nh l 3. Trong trng hp th hai ta c

a 2 b2 m n 2 v a 2 b2 m n cng mu thun vi H qu 1.
2,

T y suy ra khng tn ti cc s t nhin m tng v hiu cc bnh phng ca chng u l bi


s th k ca bnh phng cc s t nhin no v nu ngc li ta s c a 4 b4 kuv , mu
2

thun vi H qu 1.
Theo H qu 1 th hiu ca hai ly tha bc bn ca cc s t nhin khng phi bnh phng ng.
Nhng tch ca hai hiu nh vy vn c th l bnh phng ng. Chng hn

3
5

24 114 24 9752 , 24 14 234 74 20402 ,

44 214 204 35672 , 94 74 114 24 78002 ,

H qu 2. Khng tn ti nghim t nhin ca phng trnh x 4 y 4 z 4 (y l nh l cui cng


ca Fermat trong trng hp bc 4, xem thm mc 18).

Chng minh. Nu x, y, z tha mn phng trnh th z 4 y 4 x 2

, mu thun vi H qu 1.

H qu 2 ni rng khng tn ti tam gic Pythagoras vi cc cnh l bnh phng ng.


K.Zarankiewicz t ra cu hi c tn ti hay khng cc tam gic Pythagoras vi di cc cnh
l cc s tam gic (ngha l cc s dng tn n n 1 / 2 ). C th kim tra cc s tam gic

t132 8778, t143 10296, t164 13530 lp thnh mt tam gic Pythagoras. Ngoi ra ta khng bit
thm nghim no khc. Tuy nhin tn ti v hn cc tam gic Pythagoras m cc cnh gc vung l
cc s tam gic lin tip. Trong mc 4 ta chng minh phng trnh x 2 x 1 z 2 c v hn
2

nghim t nhin x, z . Vi mi nghim x, z nh vy th t22x t22x1 2 x 1 z . V d ta c


2

2
2
t62 t72 352 , t40
t41
41.29 . Hn na cng tn ti v hn cc tam gic Pythagoras nguyn thy
2

m cc cnh gc vung l cc s tam gic. Trong s c tam gic t7 , t9 ,53 .


Nu vi cc s t nhin a, b, c ta c ta2 tb2 tc2 , th c th kim tra

2a 1 1 2b 1 1 2c 1 1 .
2

CHNG 2. GII TCH DIOPHANTE BC HAI V CAO HN | 35

Phng trnh

1 y 2 1 z 2 1 c nghim t nhin l l x 263, y 287, z 329 .


2

Phng trnh ny cng c nghim khc m tt c cc s x, y, z u l, v d x 10, y 13, z 14 .


Ta khng bit phng trnh ny c v hn nghim hay khng
D dng chng minh rng khng c tam gic Pythagoras nguyn thy m nu cng thm 1 vo
di cnh huyn th ta c mt bnh phng ng. Tht vy theo nh l 1 th cnh huyn ca mt
tam gic nguyn thy c dng m2 n2 vi mt trong hai s m, n l chn v s cn li l l. Xt s
d ca m2 n2 1 khi chia cho 4 l 2 do n khng th l bnh phng ng.

c v hn nghim t nhin.
2n 1 2n 1 1 2n 2n 1 .
2

D dng chng minh rng phng trnh x 2 1 y 2 1 z 2 1


Kt qu ny c th rt ra t ng thc

2n

Thay n 1, 2,..., vo ln lt ta thu c

1 32 1 42 1
2

12

24

1 52 1 122 1 ,
2

1 72 1 242 1
2

Ch rng cc s 2n2 2n v 2n 1 u c th l cnh gc vung ca tam gic Pythagoras v ta c

2n

2n 2n 1 2n2 2n 1 vi n 1, 2,... .
2

z
2

Phng trnh x 2 1 y 2

8n
7

c bit

1 cng c v hn nghim. Kt qu ny suy ra t ng thc


2

1 1 2n
2

8n 1 1

6 2

1 82 92 1 .
2

Tuy nhin khng tn ti tam gic Pythagoras no m khi em cc cnh gc vung tr i 1 ta u


nhn c cc bnh phng ng. iu ny c suy ra t tnh cht tam gic Pythagoras lun c
t nht mt cnh gc vung chia ht cho 4.
Bn cnh c th chng minh rng vi mi tam gic Pythagoras a, b, c v s t nhin n cho
trc th u tn ti tam gic ng dng vi tam gic v tha mn mi cnh ca n l ly tha
bc m vi m n . xy dng tam gic ny cn nhn cc cnh ca tam gic a, b, c vi

b4n n1 2n1c 4n 2n1

2 4 n2 1

Vi n 2,

. S dng ng thc a 2 b2 c 2 ta c

a 2 nb n1 2 n1c n 2 n1

a b c

2
4 5 6 4

2n 2

a5b7c8

a 2 n1b2 n 1c 2 n

3 2

a3b 4c5

2 n1 2

a 2 n1b2 n1nc 2 n2 2 n1

2 n1 2

5 2

Ta cha bit c tn ti cc nghim t nhin ca phng trnh x 4 y 4 z 4 t 4 hay khng. Phng


trnh ny v nghim vi t nh hn 220000 (Lander, Parkin v Selfridge [1]).
Ta c ng thc 304 1204 2744 3154 3534 (Norrie, 1991) v 1334 1344 594 1584 (Euler,
1778). Ta khng bit phng trnh x 4 y 4 z 4 t 4 u 4 c v hn nghim t nhin tha mn
x, y, z, t 1 hay khng. Ngoi cc nghim trn th c chnh xc 81 nghim na vi u 20469

36 | V cc bnh phng c tng v hiu u l bnh phng

v x, y, z, t 1 (Rose v Brudno [1]), chng hn 2404 3404 4304 599 4 6514 (J.O.Patterson,
1942). Mt khc tn ti v hn b s x, y, z, t tha mn x, y, z, t 1 v x 4 y 4 z 4 t 4 (Lander
v Parkin [1], Lander, Parkin v Selfridge [1], Zaita [1]).
Ta cng c

24 24 34 44 44 54 ,
44 64 84 94 144 154 ,
14 84 124 324 644 654.
Tr li vi H qu 1 ta ch phng trnh x 4 y 4 z 3 c nghim t nhin. Tht vy, vi mi s t

nhin k ta u c k k 4 1

2 4

k 4 1

2 4

k 4 1

3 3

. c bit k 2, 450 225 15

E.Swift [1] chng minh phng trnh x 4 y 4 z 3 khng c nghim t nhin m x, y 1 .


H qu 3. Khng tn ti ba bnh phng lp thnh mt cp s cng vi cng sai bnh phng.
Chng minh. Nu tn ti cc s t nhin x, y, z, t m y 2 x 2 t 2 v z 2 y 2 t 2 th
y t x , y t z , mu thun vi nh l 3.
2

H qu 4 (nh l Fermat). Khng tn ti tam gic Pythagoras m din tch l bnh phng (1).
Chng minh. Phn chng. Gi s tn ti tam gic a, b, c nh vy. Khi a 2 b2 c 2 v ab 2d 2 ,
trong d v c l cc s t nhin. Khng mt tnh tng qut gi s a b . Khng xy ra trng
hp a b v khng th c 2a 2 c 2 . V vy c 2 2d a b , c 2 2d a b , mu thun
2

vi nh l 3.
C th chng minh khng tn ti cc s hu t khc 0 m tng v hiu bnh phng ca chng l
cc bnh phng hu t.
Cng vy, c th chng minh khng tn ti cc s hu t a, b, c, khc 0 tha mn a 4 b4 c 4 .
Phn chng. Gi s tn ti cc s a, b, c nh vy. C th gi s chng u dng. t a l / m ,

b r / s , c u / v , l , m, r, s, u, v l s t nhin. Do a 4 b4 c4 suy ra lvs rvm uvm2 s 2 ,


4

mu thun vi H qu 1.
dng chng minh rng khng tn ti cc bnh phng hu t khc 0 m to thnh cp s cng
vi cng sai bnh phng hu t. T y suy ra khng tn ti s hu t x m cc s x , x 1 , x 2
u l bnh phng hu t.
7. Phng trnh x 4 y 4 z 2
Mt cu hi kh t nhin c t ra l c tn ti hay khng cc tam gic Pythagoras m cc cnh
gc vung u l cc bnh phng. Cu tr li ph nh c trong nh l Fermat di y.
nh l 4. Phng trnh
(30)

x4 y 4 z 2

khng c nghim t nhin x, y, z .


Chng minh. Phn chng. Gi s phng trnh (30) c nghim t nhin v k hiu z l s t
nhin nh nht c bnh phng l tng ca hai ly tha bc 4. Ta c x, y 1 v nu ngc li t
(1)

C.M.Walsh vit mt bi bo di v nh l ny [1]. Bi bo ny bao gm cc ch dn lch s chi tit cng vi cc


ghi ch ca tc gi.

CHNG 2. GII TCH DIOPHANTE BC HAI V CAO HN | 37

x, y d 1,

ta c x dx1 , y dy1 , x1 , y1 l cc s t nhin, nh th z 2 d 4 x14 y14 , suy ra

d 4 | z 2 , do d 2 | z v th z d 2 z1 , z1 l s t nhin. Do theo (30) ta c x12 y12 z12 z 2 , mu

thun vi gi thit v tnh nh nht ca z . Vy x, y 1 . Suy ra x 2 , y 2 1 . Cc s x 2 , y 2 , z lp


thnh mt nghim nguyn thy ca phng trnh
(31)

x y
2 2

2 2

z2 .

Theo nh l 1 th c th gi thit y 2 chn v ta c


(32)

x 2 m2 n2 , y 2 2mn, z m2 n2 ,

vi m, n 1, m n, trong hai s m, n c mt s chn, mt s l. Nu m chn v n l th theo h


qu ca (32) phng trnh x 2 n2 m2 c c x v n u l. iu ny dn ti mu thun. Bi v
theo nhng g ta chng minh trong mc 3 th bnh phng ca mt s l chia 8 d 1, do v
tri ca phng trnh x 2 n2 m2 chia 8 d 2 v v th khng th l mt bnh phng ng. V
vy m l v n 2k trong k l s t nhin. Do m, n 1 suy ra m, k 1 . Theo ng thc th
hai ca (32) ta kt lun rng y 2 22 mk t suy ra y chn v c dng y 2l , suy ra l 2 mk. Do
m, k 1 , theo nh l 8 Chng 1 th suy ra cc s m v k l cc bnh phng ng, ngha l

m a 2 , k b2 , trong a, b l cc s t nhin. Ta c n 2k 2b2. . V vy theo (32), x n2 m2


trong m, n 1 suy ra x, n 1 .
Vy cc s x, n, m to thnh mt nghim nguyn thy v theo nh l 1, n chn suy ra
(33)

n 2m1n1 ,

m m12 n12

trong m1 , n1 l cc s t nhin nguyn t cng nhau. Do n 2b2 , ta c b2 m1n1 , m

m1 , n1 1 suy ra cc s m1 , n1

l bnh phng ng, vy m1 a12 , n1 b12 v do m a 2 , s dng

(33) ta kt lun a 2 m12 n12 a14 b14 . Nhng a a2 m m2 n2 z , suy ra a z , mu thun


vi tnh nh nht ca z . Vy phng trnh (30) khng c nghim t nhin.
T nh l 4 suy ra khng tn ti cc tam gic Pythagoras m cc cnh gc vung u l bnh
phng. Cng c th chng minh khng tn ti cc tam gic Pythagoras m cc cnh gc vung l
lp phng. Chng minh ny th kh hn.
Mt khc ta c 124 154 204 4812 . Tng qut hn nu ta c x 2 y 2 z 2 th
(34)
Nu

xy xz yz
4

x, y x, z y, z 1

th suy

z 4 x2 y 2 .
2

xy, xz, yz 1.

V th theo (34) v v tn ti v hn cc

nghim nguyn thy ca phng trnh Pythagoras nn ta kt lun phng trnh t 2 u 2 v2 w2


c v hn nghim t nhin t , u, v, w, vi t , u, v 1 . Ta lu rng 24 44 64 74 632 . Hn na
nh ta chng minh trong mc 6 th tng ca bn ly tha bc bn c th l ly tha bc bn.
Mt khc ta cha c phn v d cho gi thuyt Euler ni rng khng tn ti ly tha bc bn c th
biu din nh l tng ca ba ly tha bc bn khc.
Ta lu h phng trnh

x4 y 4 z 4 2t 4 ,

x2 y 2 z 2 2t 2

c v hn nghim t nhin x, y, z, t. iu ny c suy ra t cc ng thc

1 2n 1 n2 2n 2 n2 n 1
4

4.

38 | Phng trnh

x4 y 4 z 2

1 2n 1 n2 2n 2 n2 n 1
2

4n

3n2 1 3n2 2n 1 2 3n2 1 ,

4n

3n2 2n 1 3n2 2n 1 2 3n2 1 .

c bit ta c

34 54 84 2 7 4 ,

32 52 82 2 7 2 ,

7 4 84 154 2 134 ,

7 2 82 152 2 132.

Theo nh l 4 ta lu rng phng trnh x 4 y 4 2 z 2 c nghim tm thng x y, z x 2 , x l


s t nhin ty . Legendre chng minh rng y l tt c cc nghim ca phng trnh ny.
Tht vy nu ta c x 4 y 4 2 z 2 vi cc s t nhin x, y, z m x y, gi s x y, khi cc s

x, y c cng tnh chn l. V th a 12 x 2 y 2 v b 12 x 2 y 2 l cc s t nhin. V vy

x 2 a b, y 2 a b, 2z 2 x 4 y 4 2 a 2 b 2 v a 2 b2 z 2 , a 2 b2 xy , mu thun vi
2

nh l 3. T suy ra khng tn ti ba s t nhin phn bit m ly tha bc bn ca chng to


thnh mt cp s cng. Php chng minh cho s khng tn ti ba lp phng lp thnh mt cp s
cng kh hn v c th tm thy trong mc 14.
D thy phng trnh x 4 y 4 3z 2 khng c nghim t nhin. iu ny c suy ra t vic
phng trnh x 2 y 2 3z 2 khng c nghim t nhin. Cng vy, phng trnh x 4 y 4 4 z 2 cng
khng c nghim t nhin. chng minh ta vit phng trnh ny thnh x 4 y 4 2 z v s
2

dng nh l 4. Tng t phng trnh x 4 y 4 9 z 2 cng khng c nghim t nhin. By gi ta


chng minh rng phng trnh x 4 y 4 5z 2 cng khng c nghim t nhin. C th gi s khng
c s no trong hai s x, y chia ht cho 5, do chng s c dng 5k 1 hoc 5k 2. Ta c

5k 1

5 5k 2 2k 1, 5k 2 5 5k 2 4k 1 1 . V th khi chia ly tha bc bn ca x, y


2

cho 5 ta nhn c s d l 1. Do x 4 y 4 chia 5 d 2 v v vy x 4 y 4 5z 2 khng th xy ra.


Cng c th chng minh c nu k l s t nhin 8 tha mn 3 k 16 th phng trnh
x 4 y 4 kz 2 khng c nghim t nhin. Mt khc phng trnh x4 y 4 17 z 4 c nghim t
nhin x 2, y z 1. Phng trnh x 4 y 4 8z 2 ch c cc nghim tm thng x y 2k , trong
k l s t nhin, z x 2 2.

T ng thc a3 3ab 2

3a b b a
2

3 2

b2 suy ra phng trnh x 2 y 2 z 3 c v hn


3

nghim t nhin. D dng chng minh cc s x 8n n2 4 ,

y n4 24n2 16, z n2 4, vi

n l s l 1 , l nguyn t cng nhau v tha mn phng trnh x 2 y 2 z 4. .


8. V ba bnh phng c tng i mt l bnh phng ng
Gi x, y, z l nghim ca phng trnh Pythagoras. t
(35)

a x 4 y 2 z 2 . b y 4 x 2 z 2 . c 4 xyz .

Do x 2 y 2 z 2 ta c a 2 b2 z 6 , a 2 c 2 x 2 4 y 2 z 2

, b2 c 2 y 2 4 x 2 z 2 .
2

V vy vi nghim cho trc ca phng trnh Pythagoras ta nhn c cc s a, b, c m tng i


mt ca cc bnh phng ca chng l bnh phng ng. Cc s a, b, c khi l di cc cnh

CHNG 2. GII TCH DIOPHANTE BC HAI V CAO HN | 39

ca mt hnh hp ch nht m ng cho cc mt ca n u l s t nhin. c bit khi cho


x 3, y 4, z 5 ta c a 117, b 44, c 240, a 2 b2 1252 , a 2 c 2 2672 , b2 c2 2442 .
Cc s ny tm c bi P.Halcke vo nm 1719. C th chng minh s tn ti cc s a, b, c khng
c dng (35) m tng i mt cc bnh phng ca chng u l bnh phng ng. Chng hn
cc s a 252, b 240, c 275, a 2 b 2 3482 , a 2 c 2 3732 , b2 c 2 3652 ; vi c khng th bng
4 xyz, mt khc v x z, y z , c phi l s ln nht trong ba s a, b, c c dng (35).
Ta bit i vi nghim u, v, w ca phng trnh u 2 v 2 w2 th c t nht mt trong hai s u, v
chia ht cho 3 v t nht mt s chia ht cho 4. V vy nu tng i mt cc bnh phng ca a, b, c
l bnh phng th t nht hai trong cc s a, b, c phi chia ht cho 3 v t nht hai trong s phi
chia ht cho 4. Gi s ngc li cc s a v b u khng chia ht cho 3 th tng cc bnh phng
ca chng khng phi bnh phng. H qu l khng phi tt c cc cp nhn c t (35) l
nguyn t cng nhau. Tuy nhin nu x, y, z l nghim nguyn thy ca phng trnh Pythagoras
th cc s a, b, c nhn c t (35) tha mn a, b 1 . iu ny chng t tn ti v hn cc b s

a, b, c tha mn a, b, c 1 v tng i mt bnh phng ca chng l bnh phng ng.


D dng chng minh nu a, b, c l cc s t nhin tha mn tng i mt cc bnh phng ca
chng l bnh phng th cc s ab, ac, bc cng c tnh cht . M.Kraitchik pht trin cch tm cc
b ba a, b, c nh vy trong cc chng t 4 ti 6 trong [3]. Xem thm Leech [2], Korec [1].
Ta cha bit c tn ti hay khng ba s t nhin a, b, c tha mn cc s a 2 b2 , a 2 c 2 , b2 c 2 v

a 2 b2 c 2 u l bnh phng. Hay ni cch khc chng ta cha bit c tn ti hnh hp ch nht
m ng cho cc mt v ng cho chnh ca n u l s t nhin hay khng.
Mt khc tn ti ba s t nhin a, b, c, chng hn a 124, b 957, c 13852800, m cc s

a 2 b2 , a 2 c, b2 c v a 2 b2 c u l bnh phng ng (Bromhead [1]). Cng tn ti bn s


t nhin x, y, z, t m bt k ba bnh phng no ca chng cng u l bnh phng ng. S.Tebay
(xem thm Dickson [7] tp 2 trang 55) tm ra cng thc cho cc s nh th

x s 2 1 s 2 9 s 2 3 ,

y 4s s 1 s 3 s 2 3 ,

z 4s s 1 s 3 s 2 3 , t 2s s 2 1 s 2 9 ,
trong s l s t nhin ln hn 3. Bng tnh ton trc tip ta c

s 3 s 6s 9 ,
s 1 s 3 s 2s 10s
s 1 s 3 s 2s 10s
2s 3s 2s 27 .

x2 y 2 z 2
x2 y 2 t 2
x2 z 2 t 2
y2 z2 t 2

6s 9 ,

6s 9 ,
2

Khi s 4 ta c x 1995, y 6384, z 1520, t 840 .


Euler tm ra nghim x 168, y 280, z 105, t 60, nghim ny khng c dng nh trn (Jean
Lagrange [1]). Euler cng quan tm ti vic tm ba s t nhin x, y, z m cc s x y, x z, y z
u l bnh phng. y l mt b s nh vy ( x 434657, y 420968, z 50568) . C v hn cc
b ba nh vy (Dickson [7] tp 2 trang 449).
kt thc mc ny ta s chng minh tn ti v hn dy s t nhin a1, a2 ,.... tha mn mi s

a12 a22 .... an2 , vi n 1, 2,..., u l bnh phng.

40 | V ba bnh phng c tng i mt l bnh phng ng

Chng minh bng quy np. Gi s vi n cho trc ta c cc s a1 , a2 ,...an tha mn tnh cht s

a12 a22 ... an2 l bnh phong ca mt s l 1 . Ngha l a12 a22 ... an2 2k 1 , vi s t
2

2k 1

nhin k no . Vi n 1 ly a1 3 . S dng

2k 2 2k 2k 2 2k 1 , t
2

an1 2k 2 2k th a12 a22 ... an21 2k 2 2k 1 , y li l mt bnh phng l. iu phi


2

chng minh. C th vi a1 3 ta c a2 4, a3 12, a4 84, a5 3612 v tip tc nh th. V vy ta


c chui 32 42 52 , 32 42 122 132 , 32 42 122 842 852 ,
9. Cc s iu ha
S t nhin h c gi l s iu ha nu tn ti s hu t v m cc s v 2 h, v 2 h l bnh
phng ca cc s hu t.
Gi s h l s iu ha ngha l tn ti cc s t nhin a, b, c tha mn z 2 hc2 a 2 , z 2 hc2 b2 .
Ta c a b v 2z 2 a 2 b2 . Suy ra a v b c cng tnh chn l. Vy a b v a b u chn, t
a b 2x , a b 2 y, trong x, y l cc s t nhin.
Ta c a x y, b x y nn 2 z 2 a 2 b2 x y x y 2 x 2 2 y 2 , vy z 2 x 2 y 2 .
2

Hn na t z 2 hc2 a 2 , z 2 hc2 b2 , suy ra 2hc 2 a 2 b2 x y x y 4 xy , v vy


2

hc 2 2 xy . Do nu h l s iu ha th tn ti nghim ca phng trnh x 2 y 2 z 2 tha mn


hc 2 2 xy . Ngc li nu cc s t nhin x, y, z tha mn phng trnh x 2 y 2 z 2 th ta c
z 2 2 xy x y .
2

Kt hp cc kt qu nu trn ta c mnh : mi nghim t nhin ca phng trnh x 2 y 2 z 2


ng vi mt s iu ha h 2 xy . Ngc li, mi s iu ha cng c th nhn c bng cch ny,
sai khc mt t l bnh phng.
Nghim nh nht ca phng trnh Pythagoras l 3, 4,5 cho ta s iu ha 2 3 4 24 22 6 (ta
c 52 24 72 ,52 24 12 ). Nghim

5,12,13

cho s iu ha 2 5 12 120 22 30 (v

132 120 172 ,132 120 72 ). Nghim khng nguyn thy 6,8,10 cho s iu ha 96 42 6

8,15,17 cho s diu ha


9, 40, 41 cho s 720 122 5

(vi 102 96 142 ,102 96 22 ). Nghim

172 240 232 ,172 240 72 ). Nghim

41
49
41 720 49 , 41 720 31 . Suy ra 5 ,
12
12
2

240 42 15 ( y
vi cc ng thc
2

41
31
5 .
12
12

Bi ton sau y xut hin t nhng nm 1220: tm s hu t r m r 2 5 v r 2 5 u l bnh


41
phng hu t. Ngay khi bi ton c t ngi ta tm c nghim r
. Mt nghim khc
12
2

3344161
4728001
113279
2
c tm ra bi J.D.Hill [1] l r
. Khi r 2 5
v r 5
.
1494696
1494696
1494696
J.V.Uspensky v M.A.Heaslet ([1] trang 419-427) chng minh hai nghim nu trn l hai nghim
ti gin vi mu s nh nht c th. H cng tm ra nghim khc vi t s v mu s c ti 15
ch s thp phn v cng trnh by phng php ch ra v hn nghim s nh vy.
Ta s chng minh tn ti v hn cc s hu t r m cc s r 2 5 , r 2 5 l bnh phng hu t.

CHNG 2. GII TCH DIOPHANTE BC HAI V CAO HN | 41

Gi s rng r x / y , vi cc s t nhin x, y v y chn, x, y 1 tha mn tnh cht cc s

r 2 5 v r 2 5 u l bnh phng hu t. Do cc s x 2 5 y 2 / y 2 v x 2 5 y 2 / y 2 u l bnh


phng hu t suy ra iu cng ng vi x 2 5 y 2 v x 2 5 y 2 . Nhng v chng l cc s t
nhin nn chng l bnh phng ca cc s t nhin, t x 2 5 y 2 z 2 , x 2 5 y 2 t 2 v

x 4 25 y 4
r1
.
2 xyzt

(36)

x 4 10 x 2 y 2 25 y 4
4
4
Tnh ton trc tip ta c r 5
. x1 x 25 y v y1 xyzt l cc s t
2 xyzt

2
1

nhin, y1 chn v ln hn y . C th chng minh x1 , y1 1 .

V vy vi mi s hu t r biu din di dng phn s ti gin x / y trong x l s t nhin, y


chn v c hai s r 2 5 v r 2 5 u l bnh phng hu t. Theo (36) ta tm c s hu t r1
cng c tnh cht nu trn v mu s ti gin ln y . T suy ra tn ti v hn s hu t r m c
hai s r 2 5 v r 2 5 u l bnh phng hu t.
Bt u vi s

r 1241

c tm ra bi Leonardo Pisano (Fibonacci), theo (36) ta nhn c s

3344161
r1 1494696
, s ny tm c bi Hill. Tip tc s dng (36) cho r1 ta nhn c r2

l mt phn
s c t s c ti 27 ch s. Nh trn cp, Uspensky v Heaslet tm ra s hu t r tha
mn c hai s r 2 5 v r 2 5 u l bnh phng hu t v t s trong dng biu din phn s ti
gin ca r ch c 15 ch s. Nh th da vo cng thc (36) ta khng th thu c tt c cc s r
m r 2 5 v r 2 5 u l bnh phng hu t.
L do chnh khin mi ngi quan tm ti vic tm cc s r m r 2 5 u l bnh phng hu t
c l l v vi cc s h 5 th khng tn ti cc s hu t r m r 2 h l bnh phng hu t.
Chng minh kt qu ny khi h 1 hoc h 4 c suy ra trc tip t nh l 3. Chng minh cho
trng hp h 2 kh hn mt cht. Gi s tn ti s hu t r m cc s r 2 2 v r 2 2 l bnh
phng hu t. t r x / y , vi x, y l cc s t nhin th r 2 2 y 2 v x 2 2 y 2 l bnh phng
hu t. M chng l cc s t nhin nn chng phi l bnh phng ca cc s t nhin. Vy tn ti
2
2
2
2
2
2
2
2
2
z v t m x 2 y z , x 2 y t . V vy 2x 2 z 2 t 2 , 4y z t , suy ra

4x 2 z t z t . T 2 x z t z 2 t 2 z t 2 y z t . Nhng
nn ta c mu thun vi nh l 4. Chng minh khi h 3 cng kh hn mt cht.
2

z t

5
7 5
1 337
113
463 337
Mt khc ta c 6 , 6 ,
7
,
7
.
2
2 2
2 120
120
10 120
Bng y cc s iu ha nh hn 1000 c trnh by bi Tunnel trong [1].
D dng chng minh rng khng tn ti cc s t nhin x, y m x 2 y v x y 2 u l bnh
phng ng. Tht vy nu ta c x 2 y t 2 th

tx

do t x 1 , suy ra t 2 x2 2 x 1 . V vy

y t 2 x2 2 x 1 x . Lp lun tng t ta c x y , mu thun.


Mt khc tn ti v hn s hu t dng x, y m x 2 y v x y 2 u l bnh phng hu t.

Tht vy, vi x n2 8n /16 n 1 , y 2 x 1, n l s t nhin 8 , ta c

x2 y 2 z 2 t 2

42 | Cc s iu ha. Phng trnh

n 2 8n 16
n 2 2n 8
2
x y
, x y
.
16 n 1
8 n 1
2

Da vo mi lin h c cp trn gia cc s iu ha vi nghim ca phng trnh


Pythagoras v cng thc nghim ca phng trnh Pythagoras trong mc 3, th iu kin cn v
h l s iu ha l hc 2 4mn m2 n2 l 2 trong c, m, n, l l cc s t nhin, m, n 1 ,

m n , 2 | mn . Ta c

n2 l hc 2 m2 n2 2mn l . Nu h l s iu ha, z 2 hc2 a 2 ,


2

z 2 hc2 b2 , th cc s b2 , z 2 , a 2 to thnh mt cp s cng cng sai hc 2 . T nu cc s

b2 , z 2 , a 2 to thnh mt cp s cng vi cng sai hc 2 th h l s iu ha. V vy s iu ha c th


nh ngha sai khc mt t l bnh phng nh l cng sai ca mt cp s cng bao gm ba bnh

phng ng. Tt c cc cp s cng nh vy u c dng l 2 m2 n2 2mn , l 2 m2 n2

l 2 m2 n2 2mn vi m, n l cc s t nhin, m n .
2

C th chng minh vi s t nhin k cho trc th iu kin cn v tn ti s t nhin x tha


mn k x 2 v k x 2 u l bnh phng ng l k 4m4 n4 l 2 , trong m, n, l l cc s t

nhin (khng gim tng qut c th gi s m, n nguyn t cng nhau). Vi m n 1 ta c

5 22 32 , 5 22 12 , vi m 1, n 2 ta c 20 42 62 , 20 42 22 , vi m 2, n 1 ta c
65 42 92 , 65 42 72 , vi m 1, n 3 ta c 85 62 112 , 85 62 72 .
10. Phng trnh x2 y 2 z 2 t 2
Ta s tm tt c cc nghim t nhin ca phng trnh
(37)

x2 y 2 z 2 t 2

u tin ta ch rng t nht hai trong cc s x, y, z l chn. Gi s ngc li l tt c cc s x, y, z


u l, khi t 2 l tng ca ba bnh phng l s c dng 8k 3 bi v mi bnh phng l chia 8
u d 1. iu ny l khng th v bn thn t 2 cng l mt bnh phng l. Nu ch c duy nht
mt trong ba s x, y, z l chn th tng x2 y 2 z 2 t 2 s c dng 4k 2 , iu ny cng khng
th xy ra v bnh phng ca mt s chn s c dng 4k . Vy ta c th gi s y v z chn. Khi
(38)

y 2l ,

z 2m ,

trong l v m l cc s t nhin. T (37) ta thy


(39)

t x . t

tx u

Ta nhn c s t nhin u v t (37), (38), (39) ta c

x u

x 2 4l 2 4m2 v suy ra

2 xu u 2 4l 2 4m2 , hn na
(40)

u 2 4l 2 4m2 2 xu .

V phi ca ng thc (40) l tng ca cc s chn, v th u chn. t


(41)

u 2n ,

vi s t nhin n no . Th (41) vo (40) v chia c hai v cho 4 ta c n2 l 2 m2 nx . Hay l


(42)

l 2 m2 n 2
,
n

CHNG 2. GII TCH DIOPHANTE BC HAI V CAO HN | 43

l 2 m2 n 2
. Hn na v x l s t nhin, t (42) ta suy ra
n
n2 l 2 m2 . Vy tt c cc nghim t nhin x, y, z, t , vi y, z chn, ca (37) u c dng

M t (39) suy ra t x u x 2n

(43)

l 2 m2 n2
l 2 m2 n 2
, y 2l , z 2m , t
,
n
n

vi m, n, l l cc s t nhin v n l c s ca l 2 m2 v nh hn

l 2 m2 .

By gi ta chng minh iu ngc li, ngha l nu l , m, n tha mn iu kin nu trn th cc s


x, y, z, t thu c t (43) s tr thnh nghim t nhin ca phng trnh (37). T cc iu kin
trn ta c x, y, z l cc s t nhin. chng minh chng tha mn (37) ta s dng ng thc
2

l 2 m2 n 2
l 2 m2 n2
2
2

2l 2m
.
n
n

D dng chng minh rng mi nghim t nhin x, y, z, t vi y, z chn ca (37) u thu c duy
y
z
tx
nht mt ln t cng thc (43). S dng (43) ta c l , m , n
v v vy cc s l , m, n
2
2
2
xc nh duy nht theo x, y, z, t . Lp lun ny dn ti kt qu sau
nh l 5. Tt c cc nghim t nhin x, y, z, t vi y, z chn ca phng trnh x2 y 2 z 2 t 2 u
c dng x

l 2 m2 n2
l 2 m2 n 2
, y 2l , z = 2m, t
, l , m l cc s t nhin ty , n l c s
n
n

ca l 2 m2 v nh hn

l 2 m2 . Hn na mi nghim u c lit k duy nht theo cch ny.

nh l 5 khng ch ra s tn ti nghim ca phng trnh (37) nhng li cho chng ta phng


php tm chng. B qua cc cp l , m m m l v ch ly cc gi tr ca n m x l. Ta cng loi
ra cc nghim m x, y, z, t u chn. thu li cc nghim ta ch cn nhn cc nghim m x ln
lt vi cc ly tha ca 2. Di y l 10 nghim u tin ca (37) nhn c theo cch ny

R.D.Carmichael (xem [4] trang 39-43) ch ra rng tt c cc nghim t nhin ca (37) u c


th thu c t ng thc

d 2 m2 n2 p 2 q 2 d 2 2mn 2 pq d 2 2mp 2nq


2

d 2 m2 n 2 p 2 q 2
11. Phng trnh xy zt

CHNG 2. GII TCH DIOPHANTE BC HAI V CAO HN | 44

Gi s rng cc s t nhin x, y, z, t tha mn xy zt v t x, z a . Th th x ac , z ad ,


trong c v d l cc s t nhin v c, d 1 . V vy acy adt , ngha l cy dt v t c, d 1 ,
ta suy ra d | y v nh th y bd trong b l s t nhin, hn na t bc . iu ny chng t
rng nu cc s t nhin x, y, z, t tha mn phng trnh xy zt th tn ti cc s t nhin
a, b, c, d m c, d 1 v x ac , y bd , z ad , t bc . Ngc li th vi cc s t nhin cho
trc a, b, c, d ta nh ngha cc s x, y, z, t theo cng thc trn th xy zt . Ta c nh l
nh l 6. Tt c cc nghim t nhin ca phng trnh xy zt c cho bi cng thc x ac ,
y bd , z ad , t bc vi a, b, c, d l cc s t nhin ty . Hn na kt qu vn ng khi b sung
thm iu kin c, d 1 .
D dng chng minh c, d 1 , vy cng thc trn cho ta tt c cc nghim v mi nghim nhn
c ng mt ln.
tm c cc nghim ca phng trnh xy zt , ta bt u vi cc s t nhin ty x, z .
Do

x
x
x
z
z
y
t ta thy
,
nguyn t cng nhau v da vo ng thc
y v
x, z x, z
x, z x , z
x, z

do y

uz
ux
uz
suy ra t
. Mt khc, ly cc s t nhin ty x, z, u v t y
,
x, z
x, z
x, z

ux
ta nhn c nghim t nhin ca xy zt . V vy tt c cc nghim t nhin ca xy tz
x, z
uz
ux
c cho bi cng thc y
,t
vi x, z, u ty .
x, z
x, z
t

Khng kh nhn thy tt c cc nghim t nhin ca phng trnh xy z 2 u c cho bi


cng x u 2t , y v 2t , z uvt vi u, v, t ty . Ta gi thit thm u, v 1 th khi mi nghim
u nhn c ng mt ln t cc cng thc ny. C th chng minh rng tt c cc nghim t
nhin ca phng trnh xy z 3 u c cho bi cng thc x uv 2t 3 , y u 2vw3 , z uvtw vi
u, v, t , w ty .
Tng qut hn tn ti cng thc nghim tng qut ca phng trnh x1 x2 ...xn z k vi n 2 v s
t nhin k cho trc (Ward [1], Schinzel [4]).
D dng chng minh vi cc s t nhin cho trc n v m th tt c cc nghim t nhin ca
x1 x2 ...xn y1 y2 ... ym u c cho bi

xn

y1 y2 ... ym1t
x1 x2 ...xn 1t
, ym
x1 x2 ...xn1 , y1 y2 ... ym1
x1 x2 ...xn1 , y1 y2 ... ym1

vi x1 , x2 ,..., xn1 , y1 , y2 ,..., ym1 ,

l cc s t nhin ty . Phng trnh ny cn c cng thc

nghim khc nh sau: vi mn tham s t nhin ty tij i 1, 2,..., m, j 1, 2,..., n th cc nghim


u c dng yi ti ,1ti ,2 ...ti ,n

i 1, 2,..., m ,

x j t1. j t2. j ...tm. j

j 1, 2,..., n .

chng minh rng

cng thc ny cho ta tt c cc nghim ca phng trnh ban u l kh phc tp (Bell [1]).
Bi tp. 1. Tm tt c cc nghim nguyn ca phng trnh x y z x3 y 3 z 3 .
3

x y z x3 y 3 z 3 3 x y y z z x .
phng trnh nghim nguyn x y y z z x 0 .
Li gii. Ta c ng thc

Vy ch cn gii

CHNG 2. GII TCH DIOPHANTE BC HAI V CAO HN | 45

2. Tm tt c cc nghim nguyn ca h phng trnh

x y z t , x 2 y 2 z 2 t 2 , x3 y 3 z 3 t 3

(44)

Li gii. T h (44) suy ra xy yz zx 0 v x y y z z x 0 (so snh vi bi tp 1).


Nu x y 0 th do xy yz zx xy x y z 0 , suy ra xy 0 cho nn x y 0 . V vy nu
cc s nguyn x, y, z, t tha mn h (44) th hai trong s x, y, z phi bng 0 v s th ba bng
vi t ty . V vy h (44) ch c cc nghim tm thng.

3. Tm tt c cc cp s t nhin x, y m xy chia ht cho x y .


Li gii. Tt c cc cp s nh vy c cho bi cng thc
x k m n m,

(45)

y k m n n,

vi k l s t nhin ty v m, n nguyn t cng nhau. T (45) suy ra xy / x y kmn do


x y | xy . Mt khc nu vi cc s t nhin x, y, x y | xy c tha mn th ta c th t
d x y , x dm, y dn, v thu c m, n 1 , hn na d m n | d 2 mn , suy ra m n | dmn .
T m, n 1 suy ra m n, mn 1. H qu l m n | d v do d k m n vi s t nhin k
no . V vy t x dm v y dn ta nhn li cng thc (45). D dng chng minh rng vi cc
s t nhin k , m, n m m, n 1 th tt c cc cp x, y tha mn iu kin x y | xy c nhn
li ng mt ln t cng thc (45). Tht vy, theo cng thc (45) ta c m, n 1 ,
ti gin v bng vi

m
l phn s
n

x
. H qu l cc s x, y xc nh m, n mt cch duy nht. V vy theo (45) th
y

k cng xc nh mt cch duy nht bi x, y .


4. Tm tt c cc nghim t nhin ca phng trnh
(46)

1 1 1

x y z

Li gii. Tt c cc nghim t nhin ca (46) c cho bi cng thc sau y


(47)

x k m n m,

vi k l s t nhin v

x y z xy

m, n 1 .

y k m n n, z kmn,

Tht vy, nu cc s t nhin x, y, z tha mn (46) th

suy ra x y | xy v theo bi tp 3 th ta thy cng thc (45) c tha mn vi x, y .

V vy z xy / x y kmn suy ra cng thc (47). Mt khc d dng kim tra li cc s x, y, z


tha mn cng thc (47) cng tha mn phung trnh (46).
5. Tm tt c cc nghim nguyn ca phng trnh
(48)

x y z

x2 y 2 z 2

Li gii. Phng trnh (48) tng ng vi


(49)

xy yz zx 0 .

Nu cc s nguyn x, y, z tha mn phng trnh (49) v t nht mt trong s chng, gi s l x ,


bng 0 th theo (49) ta c yz 0 suy ra mt trong hai s z, y cng bng 0. V vy nu mt trong
cc s x, y, z bng 0 th t nht hai trong s chng bng 0. Mt khc nu hai trong cc x, y, z bng 0
v s th ba ty th phng trnh (48) c tha mn. By gi ta gi s x, y, z u khc 0. Khi
theo (49) suy ra hai trong cc s ny phi cng dng hoc cng m v s cn li c du ngc
li. i du nu cn thit ta c th gi s x 0, y 0, z 0 . T (49) ta suy ra xy x y z . Nh

46 | Phng trnh

xy zt

th x y | xy . By gi ta c th p dng cng thc (45) trong bi tp 3 vi z

xy
kmn . V
x y

vy nu cc s nguyn x, y, z tha mn phng trnh (48), x 0, y 0 , th vi s cc s t nhin


k , m, n no m m, n 1 ta nhn c
(50)

x k m n m,

y k m n n,

z km.

Mt khc, tnh ton trc tip cho thy cc s t nhin k , m, n trong cng thc (50) cho ta nghim
ca phng trnh (48). V vy tt c cc nghim x, y, z vi x 0, y 0 ca (48) u c cho bi
cng thc (50) vi k , m, n l cc s t nhin. Hn na ta c th gi thit m, n 1 . T y tt c
cc nghim ca phng trnh (48) c th tm c mt cch d dng.
6. Chng minh hai mnh sau l tng ng
(i) Tn ti cc s nguyn dng a, b, c, d , ef , g tha mn
(51)

a 2 b2 e2 ,

b2 c 2 f 2 , a 2 c 2 g 2 , a 2 b2 c 2 d 2

(ii) Tn ti cc s hu t x, y, z ln hn 1 tha mn
2

x v z
.

2
2
2
1 x 1 y 1 z
2

(52)

Li gii (M.Skalba). (i) (ii). Khng mt tnh tng qut gi s a, b, c, d 1 . Suy ra d l v c


ng mt trong cc s a, b, c l, gi s l a .
T nh l 1 v (51) suy ra tn ti cc s nguyn dng di , mi , ni i 1, 2,3 tha mn

a d1 m12 n12 ;
(53)

b d2 2m2 n2 ,

c d3 2m3n3,

d d1 mi2 ni2 vi i 1, 2,3.

ng thc a 2 b2 c 2 d 2 tr thnh d2 m2 n2 d3m3n3 d1m1n1 .


2

Chia c hai v cho d 2 d12 mi2 ni2

m2
1,
n2

ta thu c (52) vi

m3
1,
n3

m1
1
n1

(ii) (i). Vi cc s hu t x, y, z ln hn 1 tha mn (52). Ta vit

m2
,
n2

m3
,
n3

vi mi , ni l cc s nguyn dng. t d

m
3

i 1

2
i

m1
,
n1

ni2 .

Nu ta nh ngha di i 1, 2,3 v a, b, c theo cng thc (53) v ly

e d3 m32 n32 ,

f d1 2m1n1 ,

g d2 m22 n22 ,

th ta nhn c cc s nguyn dng a, b, c, d , e, f , g tha mn (51).


12. Phng trnh x 4 x 2 y 2 y 4 z 2
Phng trnh

CHNG 2. GII TCH DIOPHANTE BC HAI V CAO HN | 47

x4 x2 y 2 y 4 z 2

(54)

c nghim tm thng x y, z y 2 trong y l s t nhin ty .


Gi s rng x, y, z l nghim t nhin ca (54) vi x y . R rng ta c th gi s x, y 1 v nu
ngc li ngha l x, y d 1 th ta c x dx1 , y dy1 suy ra theo (54) th d 4 | z 2 v do

z dz12 . Chia c hai v ca (54) cho d 4 ta nhn c x1 , y1 1 v x14 x12 y12 y14 z12 . Gi x, y, z
l nghim t nhin ca (54) tha mn x, y 1 v x y . Hn na gi s rng i vi nghim
x, y, z ny th tch xy t gi tr b nht c th.
Ta gi s rng mt trong cc s x, y l chn, gi s l y . Do x, y 1 suy ra x l. Phng trnh

(54) c th vit li di dng x 2 y 2

xy
2

z 2 trong x 2 y 2 0 (v x y ). T x, y 1

suy ra x 2 y 2 , xy 1. Hn na do xy chn nn p dng cng thc nghim nguyn thy ca


phng trnh Pythagoras ta suy ra tn ti cc s t nhin m, n tha mn

m, n 1, 2 | mn ,

x2 y 2 m2 n2 , xy 2mn . Do x l v y chn nn s x 2 y 2 v m2 n2 c dng 4k 1 suy ra n


chn v m l. t y 2 y0 vi y0 l s t nhin. T xy 2mn ta tm c xy0 mn vi
x, y0 m, n 1 . T nh l 6 suy ra tn ti cc s t nhin a, b, c tha mn x ac , y0 bd ,
m ad , n bc vi c, d 1 . T x, y0 m, n 1 suy ra cc s a, b, c, d l nguyn t cng nhau
i mt. Do x, m l nn cc s s a, c, d l. V do n chn nn b chn.
Thay x ac, y 2 y0 2bd , m ad , n bc vo phng trnh x 2 y 2 m2 n2 ta thu c

b2 c 2 a 2 4b2 d 2 . t a 2 b2 , a2 4b2 . Ta c | a 2 4b2 a 2 b2 3b2 v

| 4 a 2 b2 a 2 4b2 3a 2 . V do a, b 1 suy ra | 3 . Nhng 3 khng phi c s ca

a 2 b2 v nu 3 | a 2 b2 th a, b u chia ht cho 3, mu thun vi a, b 1 . Vy 1 ngha l

b2 , a 2 4b2 1 , t a 2 b2 c 2 a 2 4b2 d 2 suy ra a 2 b2 | d 2 v c 2 | a 2 4b2 .

Mt khc t c, d 1 suy ra d 2 | a 2 b2 v c 2 | a 2 4b2 . V vy a 2 b2 d 2 v a 2 4b2 c 2 .


Nhng a, b 1 v do a l nn a, 2b 1 . V vy theo cng thc nghim nguyn thy ca phng
trnh Pythagoras th t ng thc a 2 2b c 2 suy ra s tn ti ca cc s t nhin x1 , y1 tha
2

mn x1 y1 1, 2 | x1 y1 , a x12 y12 , b x1 y1 . Ta c a 2 b2 d 2 . V vy x14 x12 y12 y14 d 2 v mt


trong cc s x1 , y1 l chn. Nhng x1 y1 b 2bd y xy suy ra x1 y1 xy . iu ny mu thun
vi gi thit ban u ca nghim x, y, z . Chng t cc s x, y u l.

Do x y nn ta c th gi s x y . T x 2 y 2

xy
2

z 2 v s x 2 y 2 0 l chn nn tn

ti cc s t nhin m, n tha mn m, n 1 , 2 | mn , x 2 y 2 2mn v xy m2 n2 . T

m4 m2 n2 n4 m2 n2 m2 n 2
2

xy

x2 y 2 x2 y 2

2 2

v m, n 1 , mt trong cc s m, n l chn. Nhng iu ny l khng th. Ta c nh l sau y


nh l 7. Phng trnh x 4 x 2 y 2 y 4 z 2 ch c cc nghim t nhin tm thng x y , z x 2 .
Chng minh trn thuc v H.C.Pocklington [1].

48 | Phng trnh

x4 x2 y 2 y 4 z 2 . Phng trnh x4 9 x2 y 2 27 y 4 z 2

T nh l ny Pocklington i ti nh l Fermat sau y


nh l 8. Khng tn ti bn bnh phng khc nhau to thnh mt cp s cng.
Chng minh. Gi s phn chng x 2 , y 2 , z 2 , w2 l cc s t nhin tha mn tnh cht cp s cng

y 2 x2 z 2 y 2 w2 z 2 . Do ta c 2y 2 x 2 z 2 , 2z 2 y 2 w2 suy ra 2y 2 w2 x2 w2 z 2 w2 ,
2x2 z 2 x2 y 2 z 2 w2 t 2 x2 z 2 2 y 2 w2 x2 y 2 z 2 w2 . S x 2 y 2 z 2 w2 chn suy ra xy v zw c
cng tnh chn l. t u xz , v yw , r xy zw / 2 , s xy zw / 2 . R rng u, v, r , s u l s

t nhin. D dng kim tra u 2 v 2 2rs , uv r 2 s 2 . Suy ra u 4 u 2v 2 v 4 r 2 s 2

v theo

nh l 7 suy ra u v . Do x 2 , y 2 , z 2 , w2 lp thnh mt cp s cng phn bit nn ta c th gi s


x y z w suy ra xz yw ngha l u v , mu thun.
13. Phng trnh x 4 9 x 2 y 2 27 y 4 z 2
Li gii sau y thuc v J.Cel [1]. Ta s chng minh phng trnh ny khng c nghim t nhin.
Gi s phng trnh

x4 9 x2 y 2 27 y 4 z 2

(55)

c nghim nguyn dng v gi x, y, z l nghim m z t gi tr nh nht. Nu x, y d 1 th

x dx1 , y dy1 v t (55) suy ra d 4 | z 2 , d 2 | z , z d 2 z1 , x1 , y1 , z1 l cc s nguyn dong. Chia c


hai v ca (55) cho d 4 ta nhn c x14 9 x12 y12 27 y14 z12 . Mu thun vi gi thit v tnh nh
nht z . V vy x, y 1 . Nu 2 | x th t (55) suy ra 4 | 27 y 2 z 2 v vy 2 | y mu thun vi

x, y 1 . Do x

l. Nu y cng l th t (55) suy ra 8 | z 2 5 . iu ny khng th c. V vy

(56)

x l, y chn.

Nu 3 | x th r rng ta c 27 | z 2 v vy 9 | z , 81| 27y 4 , 3 | y mu thun vi x, y 1 . Vy x,3 1 .


Ta cng c

x, z 1 .

Tht vy, t

x, z

bi d v t (55) th d | 27 y 4 . T

x,3 y 1

ta c

d , 27 y 1 v vy d 1 . t y 2 y . Phng trnh (55) c th biu din di dng


4

z x2
z x 2

27 y14
9 y12
9 y12 .
2
2

Cc nhn t v phi l dng v tng v tch ca chng l dng. t d1 l c chung ln nht


ca chng. Ta c d12 | 27 y14 v vy theo H qu 2 nh l 6a Chng 1 ta c d1 | 9 y12 v v vy

d1 | x12 , z . T x, z 1 ta c th p dng nh l 6a suy ra x 2 , z 1 v vy d1 1 v theo nh l

8 Chng 1 tjh ta s c mt trong hai kh nng sau xy ra


(571)

z x2
z x2
9 y12 27a 4 ,
9 y12 b4 , y1 ab ,
2
2

(572)

z x2
z x2
9 y12 a 4 ,
9 y12 27b4 , y1 ab ,
2
2

vi a, b l cc s nguyn dng nguyn t cng nhau.


H (571) khng th xy ra v n dn ti x2 18a 2b2 27a 4 b4 , 3 | b4 1 . H (572) dn ti
(58)

x2 18a 2b2 a 4 27b4 ,

CHNG 2. GII TCH DIOPHANTE BC HAI V CAO HN | 49

suy ra a hoc b chn. Lu x l. Nu a chn th a4 x2 18a2b2 27b4 8k 4 , mu thun.


a 2 x 9 2 a 2 x 9 2
a2 x 9 2 a2 x 9 2
Vy b chn suy ra 27b 4
b
b . t d 2
b ,
b .
2 2
2
2
2
2
2
2

Ta c d 22 | 27b4 v vy d 2 | 9b2 v d 2 | x . Suy ra d 2 | 9 y 2 , x v do 3 y, x 1 ta suy ra d 2 1 . Nu

a x 9 2
a2 x 9 2 a2 x 9 2
b u m ta s c a 2 9b2 mu thun vi (58). Vy
b ,
b
2
2
2
2
2
2
2
a x 9 2
b m4 ,
l cc s nguyn dng nguyn t cng nhau. p dng nh l 8 chng 1 ta c
2
2
2
a x 9 2
b 27n 4 , b mn , vi m, n l cc s nguyn dng. Hn na a2 m4 9m2n2 27n4
2
2
v a y1 y z , mu thun vi gi thit ban u ca nghim x, y, z . iu phi chng minh.
2

cc s

y ta ch ti hai bi bo ln (Lind [1], Reichardt [1]) nghin cu cc phng trnh dng

ax4 bx2 y 2 cy 4 dz 2 .
14. Phng trnh x y 2 z
3

Gi s rng phng trnh ny c nghim nguyn x, y, z tha mn x y v z 0 . Ta c th gi s

x, y 1 v nu x, y d 1 th ta c th t x dx1 , y dy1 v suy ra d 3 | 2 z 3 do d | z v v


3
3
3
3
3
3
vy z dz1 . Ta s c x1 y1 2 z1 vi x1 , y1 1 . Do x y 2 z nn cc s x y v x y l
chn suy ra cc s u x y / 2 v v x y / 2 nguyn. Hn na ta c x u v , y u v v do
3
3
x, y 1 nn u, v 1 . Ta cng c u v u v 2 z 3 . V vy u u 2 3v2 z 3 m v x y v
z 0 suy ra uvz

1 2
x y2 z 0 .
4

Nu u,3 1, th v u, v 1, ta c u, u 2 3v 2 1 . Hn na tn ti cc s nguyn z1 v z2 tha

2
2
3
2
2
3
3
6
2
mn u z1 v u 3v x2 . V vy z2 z1 3v v do z2 z1 z2 z1

3z2 z12 3v 2 . t

t z2 z12 khi v z1 , z2 1 ta c t , z1 1 v t t 2 3tz12 3z14 3v 2 . Suy ra 3 | t . Ta t t 3t1

2
2
4
v t1 9t12 9t1 z12 3z14 v 2 suy ra 3 | v . V vy v 3v1 v v z1 ,3 1 suy ra s 9t1 9t1 z1 3z1

khng chia ht cho 9, m 9 | v ta suy ra 3 | t1 . V vy t1 3t2 . Ta c t2 (27t22 9t2 z12 z14 ) v12 m
2

t , z1 1 suy ra t2 , z1 1 v (t2 , 27t22 9t2 z12 z14 ) 1 . Hn na t2 b2

4
2 2
4
2
v 27b 9b z1 z1 c .

2
Cc s b v z1 l cc s t nhin v nu b 0 th t2 0 v t t 0 , do z2 z1 m

z1 , z2 1 nn z1 1, z2 1 suy ra

v 0 do x y mu thun vi gi thit ca x, y, z . Mt

2
khc nu z1 0 th u 0 , do 3v z2 v h qu l v 0 , mu thun. Vy ta kt lun rng
2

phng trnh x 9 x y 27 y z khng c nghim t nhin.


4

2 2

Nu 3 | u th v u, v 1 ta c v,3 1 do u 3u1 v bi v u u 2 3v 2 z 3 nn z 3z1 v

u1 3u12 v 2 3z13 m v,3 1 suy ra 3 | u1 . H qu l u1 3u2 v u2 27u22 v 2 z13 . Nhng do

u2 , v 1 suy ra u2 , 27u22 v2 1, ta c u2 a3 , 27u22 v2 b3 vi a, b 1 v bi v v,3 1


nn b,3 1 . Ta c 27a6 v2 b3 . t t b 3a 2 ta nhn c t ,3 1 v
t t 2 9a 2t 27a 4 v 2 . Nhng v a, b 1 ta suy ra a, t 1 . T t ,3 1 ta suy ra

50 | S tam gic

t, t

2
2
2
4
2
4
2 2
4
2
9a 2t 27a 4 1 . H qu l t a1 v t 9a t 27a b1 . Do a1 9a a1 27a b1 vi

a1 0 , a 0 v nu a1 0 th t 0 mu thun vi t ,3 1 v nu a 0 th u 0 v h qu l
z 0 mu thun vi z 0 . Vy ta kt lun rng phng trnh x4 9 x2 y 2 27 y 4 z 2 khng c
nghim t nhin.
T hai trng hp trn suy ra iu phi chng minh. Li gii ny thuc v Antoni Wakulicz [1].
nh l 9. Phng trnh x y 2 z khng c nghim nguyn vi x y v z 0 .
3

nh l ny suy ra khng c ba lp phng phn bit lp thnh mt cp s cng.


Cho y 1 hoc y 1 ta nhn thy phng trnh x3 2 z 3 1 khng c nghim nguyn z, x no
ngoi tr x z 1 v x 1 , z 0 v phng trnh x3 2 z 3 1 khng c nghim nguyn no
ngoi tr x z 1 v x 1 , z 0 .
H qu 1. Khng c s tam gic 1 no l lp phng ca mt s t nhin.
Chng minh. Gi s tn ti s tam gic 1 l lp phng ng. Th th tn ti cc s t nhin
m 1 v n tha mn m m 1 2n3 . Nu m chn th m 2k , k l s t nhin v k 2k 1 n3 ,
do v k , 2k 1 1 ta suy ra tn ti s t nhin x, z tha mn k z 3 , 2k 1 x3 , do

x3 2 z 3 1 m nh chng ta chng minh trn th iu ny l khng th. Nu m l th


m 2k 1 trong k l s t nhin 1 (v m 1 ) v 2k 1 k n3 , do v 2k 1, k 1 ta suy
ra tn ti cc s t nhin x, z tha mn 2k 1 x3 , k z 3 . V vy x3 2 z 3 1. iu ny cng
khng th xy ra. H qu c chng minh xong.
H qu 2. Phng trnh x y 1 khng c nghim t nhin no ngoi tr x 3 , y 2 .
2

Chng minh. Gi s tn ti s t nhin x 3 v y tha mn x y 1 . Nu x chn th ta c


2

x 1, x 1 1
x 1 a3 ,

x 1 x 1 y3 suy ra tn ti cc s t nhin a v b tha mn


2
2
x 1 b3 . Do b a b2 ab a 2 b3 a3 2 v t b ab a | 2 , v l. V vy
v bi v

x l v do x 2k 1 trong k l s t nhin 1 (nu k 1 th x 3 , mu thun vi gi


2
3
thit). T x 1 y suy ra y chn v do y 2n , ta c k k 1 2n3 vi k l s t nhin 1 ,
mu thun vi H qu 1. H qu 2 c chng minh.
Lin quan ti H qu 2 ta nhc li gi thuyt Catalan ni ting ni rng phng trnh x z y t 1
ch c duy nht mt nghim t nhin x, y, z, t m mi s u ln hn 1 l x 3 , y 2 , z 2 ,
t 3 . Gi thuyt ny cha c chng minh.
R.Tijdeman [1] hn ch c vic gii phng trnh thnh cc tnh ton hu hn bng cch t
ra mt chn hu hiu cho cc nghim. Gii hn ny c xc nh c th bi M.Langevin [1].
Langevin cng chng minh gi thuyt trong trng hp x z exp exp exp exp 730.
Nu z v

t l cc s nguyn t th theo J.W.S.Cassels [3] ta c z | y , t | x .

A.Makowski [6] v S.Hyyro [1] da vo nhn xt ny chng minh khng tn ti ba s t


nhin lin tip m mi s u l mt ly tha khng tm thng. Mt khc rt d chng minh
khng tn ti bn s t nhin lin tip m mi s trong chng u l cc ly tha khng tm
thng. Tht vy trong bn s t nhin lin tip lun c mt s chia 4 d 2, ngha l khng th l
mt ly tha khng tm thng.
Gi thuyt ca S.S.Pillai ni rng nu u1 , u2 ,... l dy v hn cc s t nhin phn bit m mi s
trong chng u l ly tha khng tm thng th lim un1 un (Pillai [8]). Gi thuyt ny
n

CHNG 2. GII TCH DIOPHANTE BC HAI V CAO HN | 51

tng ng vi mnh : vi mi s t nhin m th ch c hu hn cc b s t nhin x, y, z, t m


mi s u ln hn 2 v tha mn phng trnh x y z t m .
Mt vn kh th v c t ra l vi gi tr no ca m th tn ti cc s t nhin x, y, z, t ln
hn 1 tha mn phng trnh nu trn. D dng chng minh rng tnh cht ny l kh d vi mi
s t nhin khng c dng 4k 2 vi k 0,1, 2,...
Trong mi lin h ny mt cu hi khc c t ra l c phi vi mi s t nhin n u tn ti s
t nhin m m phng trnh x y z t m c t nht n nghim t nhin phn bit x, y, z, t m mi
s u ln hn 1 hay khn? Cu tr li l khng nh. Vi k 1, 2,..., n v m 22 n th

m 22 n 22 nk 1 2k 1 22 nk 1 2k 1 .
2

Ta c

32 n 22 n 22 n k 22 n k
2k

2k

vi k 1, 2,..., n.

Trong dy un c cp trn cc phn t nh hn 400 ln lt l 1, 4, 8, 9, 16, 25, 27, 32, 36,


49, 64, 81, 100, 121, 125, 128, 144, 169, 196, 216, 225, 243, 256, 289, 324, 343, 361, 400 v tng
ng dy un 1 un l 3, 4, 1, 7, 9, 2, 5, 4, 13, 15, 17, 19, 21, 4, 3, 16, 25, 27, 20, 9, 18, 13, 33, 35, 19,
18, 39.
H qu 2a . Phng trnh x y 1 khng c nghim hu t ngoi tr cc nghim x 0 , y 1 ,
x 1 , y 0 , x 3 , y 2 .
2

Chng minh. Gi s cc s hu t x, y tha mn x y 1 . t x h / g , y r / s vi g , s l cc


2

s t nhin v h, r l cc s nguyn tha mn

h2 s3 g 2 r 3 g 2 s3 . V vy h2 s3 g 2 r 3 s3 . V

h, g r, s 1 . Do x2 y3 1
g , h 1 ta c g 2 | s3 . Mt

ta c
khc,

g 2 r 3 h2 g 2 s3 do v r , s 1 ta c s3 | g 2 . T y ta suy ra g 2 s3 . V vy vi s t nhin

m no ta c g m3 , s m2 v suy ra h2 r 3 m6 . Vy r 3 h m3 h m3 vi m, h 1 .

Nu mt trong cc s h v m l chn v s cn li l l th h m3 , h m3 1 v t suy ra tn


ti a v b tha mn h m3 a3 , h m3 b3 , do a3 b 2m3 . Nhng v m 0 suy ra
3

h m3 h m3
,
1 v 2 | r
2
2

a b do h 0 v suy ra x 0 , y 1 . Nu c m v h u l th

do r 2r1

h m3 h m3
v 2r13

. Suy ra tn ti cc s nguyn a v b tha mn


2 2

3
3
h m3 4a3 , h m 2b . V vy b3 m 2a3 . Nu a 0 th h m3 g do x 1 ,
3

y 0 . Nu a 0 th nh ta bit b bng m a . V vy h 4a3 m3 3m3 3g do


x 3 , y 2 . H qu 2a c chng minh.

H qu 3. Nu n l s t nhin ln hn 1 th 13 23 ... n3 khng phi lp phng ng.

n n 1
2
Chng minh. Ta bit 1 2 ... n
tn .
2

2
Nu cc s t n l lp phng ng th t n cng l lp phng ng . Mu thun vi H qu 1. y
cn nhc li nh l trong chng trc (h qu ca nh l 16) ni rng nu cc s t nhin
a, b, l , m tha mn l , m 1 , al bm th tn ti s t nhin n tha mn a nm v b nl .

52 | S tam gic

Kh hn mt cht chng minh rng vi n 1 th s 12 22 ... n2 l bnh phng ca mt s


t nhin ch trong trng hp n 24 (1). C mt bi ton lin quan l khi no th phng trnh

1n 2n ... m 1 mn c nghim t nhin m, n 1 . Bi ton ny vn cha c gii. Gi thuyt


n

ca P.Erdos a ra cu tr li ph nh. L.Moser [2] chng minh gi thuyt trong trng hp

m 1010 (xem thm Best v Riele [1]). Cui cng ta ch rng c th chng minh phng trnh
x3 y 3 z 3 khng c nghim nguyn x, y, z 0 . T y suy ra 1 khng phi l tng ca hai lp
6

phng hu t (Chng 11 mc 10).


15. Phng trnh x3 y3 az 3 vi a 2
nh l 10. Nu a l s t nhin ln hn 2 v khng chia ht cho lp phng no ln hn 1 v
(59)

x3 y3 az 3

c nghim x, y, z vi x, y 1 , z 0 , th phng trnh ny c v hn nghim nh th (xem thm


Nagell [5], trang 246).
Chng minh. Gi s x, y, z, x, y 1 , z 0 tha mn (59). Ta c x, z 1 . t d x, z ta c

d 3 | az 3 x3 y3 do d | y m do x, y 1 suy ra d 1 . Tng t y, z 1 . t
(60)

x x3 2 y 3 , y 2 x3 y 3 , z x3 y 3 .

Ta c
(61)

x x3 2 y 3 x1 ,

(62)

y 2 x3 y 3 y1 ,

(63)

z x3 y 3 z1 ,

trong x1 , y1 , z1 l cc s nguyn v x1 , y1 , z1 1 .
Da vo ng thc

xx

2 y 3 y 2 x3 y 3 x3 y 3 x3 y 3
3

3
3
3
T (59) suy ra cc s x1 , y1 , z1 tha mn phng trnh x1 y1 az1 . Nu x y th v x, y 1 ta

c x y 1 v t (59) suy ra az 3 2 , v l v a 2 . V vy x y v theo (63) suy ra z1 0 .

3
3
3
3
3
3
Nu d x1 , y1 th d | x1 y1 az1 . Nu d 1 v d , z1 1 th ta c d 3 , z13 1 v v d | az1

suy ra d 3 | a , mu thun vi gi thit a khng c c lp phng ln hn 1. V vy hoc d 1


hoc d 1 v d , z1 1 do x1 , y1 , z1 1 , v l.
V vy ta kt lun d 1 v do x1 , y1 1 .
3
3
3
Hn na do x1 y1 az1 ta thy

y1 , x 1 (nu d1 | y1 v d1 | x

x1 , z1 1. T x, y 1

suy ra

x, y 1 v v (62) suy ra
4

4
th theo (62) suy ra d1 | y v nh th , x 1 .

Tng t t (61) ta suy ra x1 , y 1 do , y 1 .

(1)

Bi ton ny c t ra bi E.Lucas [1]. Li gii u tin da trn l thuyt v cc hm elliptic c a ra bi


G.N.Watson [1]. Li gii da trn l thuyt v trng s hc c a ra bi Ljunggren [5]. Xem thm Trost [1]

CHNG 2. GII TCH DIOPHANTE BC HAI V CAO HN | 53

x, z y, z 1

nn ta c

xy , z 1 .
3

Nu d | v d | z th theo (59) v (61) ta c

d | x3 y3 | x 4 xy3 v d | x 4 2 yx3 do d | xy 3 . T v d | z v xy 3 , z 1 nn d 1 , ngha l

, z 1 . V vy , x , y , z 1 v t (61), (62), (63) suy ra

l c s ca x3 2 y 3 ,

2x3 y 3 , x3 y 3 v do n cng l c s ca x3 2 y3 2 x3 y3 3x3 .

V vy t , x 1 ta thy | 3 . Vy 1 hoc 3 v trong mi trng hp ta u c 3 .


Nu x 0 th v x, y 1 ta c y 1 , mu thun vi (59) v a 2 . Tng t ta c y 0 . Mi
mt trong cc s x v y l khc 0 v v x y nn ta c x3 y 3 1 .
Nu cc s x, y cng dng hoc cng m th x 2 xy y 2 x y 3xy 1 3xy 4 v
2

x3 y 3 x y x y 3xy 4 . Nu mt trong cc s x, y l dng v s cn li l m th


2

xy 0 v x 2 xy y 2 x y 3xy 4 ; v x y 0 , gi s ngc li x y th theo (59) v


2

a 2 suy ra z 0 , mu thun vi gi thit. V vy trong mi trng hp ta u c x3 y 3 4 .

Do 3 nn t cng thc (63) suy ra z1 z . Chng t nu a tha mn iu kin bi ton th t


mi nghim nguyn x, y, z vi x, y 1 v z 0 ca (59) ta c th nhn c mt nghim khc

x1 , y1 , z1 vi x1 , y1 1 v z1 z . T y suy ra c v hn nghim nh vy. nh l 10 c


chng minh.
Cc phng trnh x3 y3 3z 3 , x3 y3 4 z 3 , x3 y3 5z 3 u khng c nghim nguyn x, y, z
vi z 0 (Selmer [1], [2]). Mt khc t nh l 10 suy ra mi phng trnh x3 y3 6 z 3 ,

x3 y3 7 z 3 , x3 y3 9 z 3 u c v hn nghim nguyn x, y, z vi x, y 1 v z 0 . Tht vy,


ta s dng nh l 10 v ch rng cc s 17 , 37 , 21 tha mn phng trnh th nht, cc s 2 ,
1 , 1 tha mn phng trnh th hai v cc s 2 , 1 , 1 tha mn phng trnh th ba (Nagell [5],
trang 247-248). T y ta suy ra mt s h qu trong chng 11 mc 9.
16. S tam gic
Nh ta bit trong mc 4, s t n n n 1 / 2 c gi l s tam gic th n. Danh sch 20000 s
tam gic c lit k vo nm 1762 bi E. de Joncourt [1]. K.Zarankiewicz [1] lu rng cc s
67
66 67
666 667
21, 2211, 222111, u l s tam gic. Ta c 21
, 2211
, 222111
. Bn
2
2
2
c c th t chng minh nhn xt ny. Cc v d tng t c a ra bi T.Jozefiak [1]

D dng chng minh tn ti v hn cc cp s tam gic m tng i mt ca chng u l s tam


gic. Tht vy, d dng kim tra rng vi s t nhin k ta c tk 1 k tk (vi t0 0 ). V vy vi

k tn n 1, 2,... ta c ttn 1 tn ttn . c bit t2 t2 t3 , t5 t3 t6 , t9 t4 t10 , t14 t5 t15 .


M.N.Khatri [1] tm ra t3k t4 k 1 t5k 1 , t5k 4 t12k 9 t13k 10 , t8k 4 t15k 9 t17 k 10 vi k 0,1, 2,...

54 | S tam gic

c bit t6 t9 t11 , t9 t13 t16 , t9 t21 t23 , t12 t24 t27 . Ta cng c t4 k 2 5k 2 t4 k 2 5k t4 k 2 vi

k 1, 2,... . Ta s chng minh tn ti v hn cp cc s t nhin x, y tha mn h phng trnh


(64)

t x t2 y t3 y v t x t2 y t y 1 .

D dng chng minh mi phng trnh trong (64) u tng ng vi


(65)

x2 x 5 y 2 y .

Cho nn ta ch cn chng minh (65) c v hn nghim t nhin x, y . S dng ng thc

161x 360 y 116

161x 360 y 116 5 72 x 161y 52

72 x 161y 52 x 2 x 5 y 2 y
Suy ra nu x, y to thnh nghim t nhin ca (65) th cc s u 161x 360 y 116 v
v 72 x 161y 52 l nghim t nhin u, v ca (65) m ln lt ln hn x, y . Do x 2 v y 1
tha mn (65) suy ra (65) c v hn nghim t nhin x, y (Sierpinski [32]).
J.Browkin [1] s dng cc kt qu ca P.F.Teilhet [1] a ra mt phng php tm tt c cc
cp s tam gic m tng v hiu ca cc s trong mi cp u l s tam gic.
Vi x 100 th tn ti t x , t y vi x, y 6,5 , 18,16 , 37, 27 , 44,39 , 86,65 , 91,54 .
Nh ta bit (so snh vi Chng 1 mc 4) tn ti v hn cc s tam gic l bnh phng ng.
Nhc li rng t thi Euler ta bit vi mi s t nhin n th s

3 2 2

32 2

l s t
4 2
nhin v bnh phng ca n l s tam gic (Sierpinski [30]). Mt khc, W.Ljunggren [4] chng
minh rng ch tn ti hai s tam gic m bnh phng l s tam gic, k hiu l t1 v t6 .
nh l 11. Khng tn ti s tam gic 1 l ly tha bc bn.
Chng minh. Gi s phn chng tn ti s t nhin m v n 1 m

1
2

n n 1 m4 c tha

mn. Khi n n 1 2m4 v do n 2k suy ra k 2k 1 m4 . V k , k 1 1 nn suy ra tn


4
4
4
4
ti cc s t nhin x, y tha mn k y , 2k 1 x v do 2 y 1 x . Nu n l th

n 2k 1 v do 2k 1 k m4 . V 2k 1, k 1 suy ra tn ti cc s t nhin x, y tha mn

2k 1 x4 , k y 4 . T y ta c 2 y 4 1 x4 v t 2k 1 n 1 suy ra y 1 . Vy y 4 k 1 .
hon tt chng minh ta cn chng minh rng
1)

4
4
Khng tn ti x, y tha mn 2 y 1 x ,

2)

4
4
Khng tn ti x v y 1 tha mn 2 y 1 x .

4
4
chng minh 1) ta ch rng nu 2 y 1 x th ta c y 2

x 4 y 4 1 , mu thun vi H
2

4
4
qu 2 mc 6. chng minh 2) ta gi s 2 y 1 x v do y 2

x 4 y 4 1 . Nhng do
2

y 4 1, y 4 1 l s t nhin, mu thun vi H qu 1 mc 6. nh l uc chng minh.


Tuy nhin vn c th xy ra tnh hung vi cc s hu t t v u th
2

1
t (t 1) u chng hn vi
2

1
32
4
2
6
ta c t (t 1) . Ch rng phng trnh 2 y 1 z khng c nghim t nhin y, z
49
2
7

CHNG 2. GII TCH DIOPHANTE BC HAI V CAO HN | 55


4
2
ngoi tr 2 134 14 2392 . C th chng minh rng phng trnh 2 y 1 z ch c hai nghim
t nhin y, z l y z 1 v y 13, z 239 (Ljunggren [1]).

C th s dng kt qu quen thuc v phng trnh x y 2 z (Denes [1]) suy ra rng cc


s tam gic u khng th l ly tha bc n ca mt s t nhin vi 2 n 30 . Mt khc theo nh
l tng qut hn (Schinzel v Tijdeman [1]) th phng trnh P x y m , trong P l a thc
n

vi h s hu t vi t nht hai h s khc 0, c th c v hn nghim nguyn x, y vi y 1, m 2 .


V vy s tam gic khng th l ly tha bc n ca mt s t nhin vi n ln hn mt gic tr n0
8
no . Theo E.Z.Chein [2] th c th chn n0 7.877 10 .

D dng nhn thy vi s t nhin n th n n 1 khng th l bnh phng ng. Tht vy, nu ta
2
2
c n n 1 a 2 th v n, n 1 1 suy ra cc s n , n 1 u l bnh phng. Vy n k , n 1 l

v do l k l k l 2 k 2 1 . V l. Tuy nhin vi n

2
3

th ta c

1
3

13 1 32

Tng t ta c th chng minh tch ca hai s t nhin lin tip khng th l ly tha bc ln hn
1. Chng minh cho nh l Goldbach ni rng tch ca ba s t nhin lin tip khng th l bnh
phng cng tng i n gin. Tht vy, ta c th chng minh rng tch ca ba s t nhin lin
tip khng th l ly tha bc ln hn 1. Gi s tn ti cc s t nhin n, k v s 1 m

n n 1 n 2 k s . Do n 1, n n 2 1 p dng nh l 8 Chng 1 suy ra tn ti a, b tha

mn n 1 a s v n n 2 b s . T 1 n 1 n n 2 a 2
2

b s v l.

P.Erdos v J.L. Selfridge [1] chng minh tch ca k s nguyn dng lin tip vi k 1 khng
th l ly tha bc ln hn 1 v tch ca k s l lin tip vi k 1 cng khng th l ly tha bc

n
k

ln hn 1 (Erdos [5]). Hn na vi s t nhin k 3 v n 2k th s khng th l ly tha


bc ln hn 1 (P.Erdos [11]).
Cc s c dng Tn 16 n n 1 n 2 (vi n l s t nhin) c gi l s t din. Tn gi ny c
t theo s cc hnh cu c cng bn knh c th cha trong cc t din. Mi s t din u tin
l 1, 4,10, 20,35,56,84,120,165, 220 . Vi n 1, 2, 48 ta nhn c cc s t din 12 ,22 ,1402 l bnh
phng ng. C th chng minh y l tt c cc s t din c tnh cht ny. Tnh cht ny c
2
2
2
chng minh bi A.Meyl [1] da trn s kin l cc s sn 1 2 ... n l bnh phng khi v ch
khi n 1 hoc n 24 (xem thm mc 14). Ngc li, gi s vi s t nhin n ta c sn m2 vi m l
s t nhin. Khi ta c th kim tra 4sn T2n . Suy ra T2 n 2m v 2n bng 2 hoc 48 ngha l
2

n 1 hoc n 24 .
Tn ti cc s t nhin va l s t din va l s tam gic. E.T.Avanesov [1] chng minh tt c
cc s nh vy l n 1,10,120,1540, 7140 . Vi cc s ny th ta c n 12 x x 1 16 y y 1 y 2
ln lt vi x 1, 4,15,55,119; y 1,3,8, 20,34 .
Ta c Tn Tn1 tn v Tn Tn1 12 22 ... n 1 .
2

C th chng minh tn ti v hn cp cc s t din m tng (hoc hiu) ca cc s trong mi cp


cng l s t din (Rohr [1], Sierpinski [33], Wunderlich [1], Bremner [1]). Ta cha bit c cp s
t din no m c tng v hiu ca chng u l s t din hay khng. H.E.Salzer [1] a ra gi
thuyt ni rng mi bnh phng u l tng ca nhiu nht l bn s t din v ng ta chng
minh tnh cht ny vi mi bnh phng 106 . c bit

56 | Phng trnh x 2 Dy 2 1

12 T1 , 22 T2 ,32 T1 T2 T2 , 42 T1 T1 T2 T3 ,52 T1 T2 T4 T1 T2 T3 T3 ,
62 T1 T5 , 7 2 T2 T3 T5 ,82 T2 T2 T6 ,92 T1 T2 T4 T6 ,102 T2 T4 T4 T6 .
D dng chng minh mi s t nhin u l tng ca bn s t din. Tht vy ta c
1 T1 T4 T3 T3 ,2 T4 T3 T3 T2 v vi s t nhin n > 2 ta c n Tn Tn2 Tn1 Tn1 . Tuy nhin chng minh
mi s t nhin u l tng ca nhiu nht 8 s t din l kh hn (Watson [2]). Cc s t nhin
107 u l tng ca nhiu nht 5 s t din (Salzer v Levine [1]).
17. Phng trnh x2 Dy 2 1
Trong mc ny ta nghin cu cc phng trnh nghim nguyn vi s t nhin D cho trc
(66)

x2 Dy 2 1

Phng trnh ny c gi l phng trnh Fermat hoc l phng trnh Pell. Phng trnh ny c
cc nghim tm thng x 1, y 0 v x 1 , y 0 . Ta ch cn tm cc nghim t nhin ca
(66). Nu D l bnh phng th (66) c th vit thnh x ny x ny 1, do x ny , v l
v x, y l cc s t nhin. V vy nu D l bnh phng th phng trnh (66) khng c nghim t
nhin. chng minh phng trnh cng v nghim trong trng hp cn li ta xt b
B . Nu D khng phi bnh phng ng th tn ti v hn cp s phn bit x, y tha mn

y 0 v | x2 Dy 2 | 2 D 1

(67)

Chng minh. Vi mi k 0,1, 2,..., n k hiu lk l s t nhin ln nht k D 1 .


Ta c lk k D 1 v lk 1 k D 1. V vy

0 lk k D 1

(68)

Ta c n 1 s lk k D k 0,1, 2,..., n l phn bit v nu lk k D lk ' k ' D th ta c

lk lk ' k k ' D trong k k ' l khng th xy ra v nu ngc li th

D l bnh phng

hu t v suy ra D l bnh phng ng, mu thun vi gi thit.


Theo (68) th cc s u lk k D k 0,1, 2,..., n phi tha mn mt trong cc bt ng thc
n 1
n
u . Suy ra t nht c hai gi tr phn bit u ' v u '' tha
n
n
j 1
j
j 1
j
mn cng mt c lng, ngha l
u' ,
u '' , trong j = 1, 2,..., n .
n
n
n
n
0u

1
,
n

1
2
u , ...,
n
n

T gi thit u ' u '' ta c th gi s rng u ' u '' . Bt ng thc u ' k / n v u '' k 1 / n suy ra
1
0 u ' u '' . Do u ' lk k D , u '' li i D trong k , i thuc v 0,1, 2,..., n th t
n

x lk li , y i k ta suy ra
(68a)

0 x y D

1
n

Hin nhin v x, y l cc s nguyn v y i k . V vy y l hiu ca hai ch s khc nhau t dy


0,1, 2,..., n , nn n khc 0 v khng ln hn n ngha l
(69)

0 y n

CHNG 2. GII TCH DIOPHANTE BC HAI V CAO HN | 57

1
Theo (68a) ta c y D x y D .
n

1
n

1
n

Theo (69) suy ra n D n D x n D , v h qu l x n D


T theo (69) th x y D x y

1
.
n

1
D 2n D .
n

Nhn hai v vi (68a) suy ra x 2 Dy 2 2 D 1. V vy tn ti x, y tha mn (67) v (68a). T


y suy ra tn ti v hn cp s t nhin x, y tha mn (67) v
(70)

0 x y D.

Gi s ngc li, ngha l ch tn ti hu hn cp s nh vy. K hiu


(71)

x1, y1 , x2 , y2 ,

...,

xs , ys

l tt c cc cp s . Khi cc s
(72)

x1 y1 D , x2 y2 D , ..., xs ys D

u l dng. Gi l s nh nht trong . S t nhin n tha mn


(73)

1
.
n

trn ta chng minh tn ti t nht mt cp s x, y tha mn c (67) v (68a). T (68a) v


(73) ta c 0 x y D . Nhng do l nh nht trong cc s dy (72) nn x y D
khng thuc v dy , ngha l cp x, y khng trng vi cp no trong (71) v u tha mn
(67),(68a) v do tha mn c (70). iu ny mu thun vi nh ngha trong (71). Vy c v
hn cc cp s x, y tha mn (67) v (70). B c chng minh.
nh l 12. Nu s t nhin D khng phi bnh phng ng th phng trnh x2 Dy 2 1 c v
hn nghim t nhin x, y .
Chng minh. Do s cc s nguyn vi tr tuyt i nh hn 2 D 1 l hu hn v theo b th
tn ti v hn cp x, y tha mn bt ng thc (67) nn tn ti v hn cp s nguyn x, y m

x2 Dy 2 bng vi mt s k xc nh v khc 0 (trng hp D x2 / y 2 c loi b). K hiu tp


hp tt c cc cp s x, y nh vy l Z . Vi s nguyn t k hiu r t l phn d nhn c khi
chia

cho k . Vi x, y thuc Z ta xt cp r x , r y . R rng c nhiu nht k 2 cp s phn bit

nh vy. By gi ta chia Z thnh cc lp vi gi thit x, y v x ', y ' thuc cng mt lp nu v ch


nu r x r x ' v r y r y ' . Do s cp r x , r y phn bit l hu hn m Z v hn nn
t nht mt trong cc lp l v hn. Trong lp tn ti hai cp a, b v c, d m t nht mt trong
cc ng thc a c , b d sai bi v vi cp a, b cho trc tn ti nhiu nht 4 cp c, d nh
vy. Cc hiu a 2 Db2 v c 2 Dd 2 u bng k (v cc cp a, b v c, d u thuc Z ). Nhng do
a, b v c, d thuc cng mt lp nn suy ra r a r c v r b r d . V vy tn ti cc s
nguyn t v v tha mn a c kt v b d kv . T
(74)

a c kt , b d kv ,

Trong t v v u nguyn. Nhn cc ng thc

58 | Phng trnh x 2 Dy 2 1

a 2 Db2 k , c2 Dd 2 k

(75)

v s dng ng thc a 2 Db2

ac Dbd

(76)

Dd 2 ac Dbd D ad cb ta suy ra
2

D ad cb k 2
2

2
2
T (74) v (75) suy ra ac Dbd c kt c D d kv d c Dd k ct Ddv v cng c

ad cb c kt d c d kv k dt cv . V vy nu ta chia c hai v ca (76) cho k 2 th ta c

1 ct Ddv

2
2
D dt cv 1 . T y t x 1 ct Ddv , y dt cv ta suy ra x Dy 1 . Ta
2

s chng minh y 0 . Nu y 0 suy ra | x | 1 v vy 1 ct Ddv 1, dt cv 0.

Nhn ng thc th nht vi c v th hai vi Dd sau cng li ta suy ra c c 2 Dd 2 t c,


t theo (74) v (75) suy ra a c , ngha l a c .
Tng t nhn ng thc th nht vi d v th hai vi c sau cng li ta c

d c 2 Dd 2 v d , do theo (74) v (75) ta suy ra b d , ngha l b d . Nhng t ng

thc ny v a c suy ra mu thun vi nh ngha ca a, b v c, d .


V vy ta chng minh tn ti t nht mt cp x, y tha mn x 2 Dy 2 1 vi y 0 (nh th
2
2
x 0 ). i du nu cn ta thu c nghim t nhin ca (66). Nu ng thc x Dy 1 ng

vi cc s t nhin x, y no th r rng 2 x 2 1 D 2 xy 2 1 vi 2xy y . V vy t mt


2

nghim t nhin x, y bt k ca (66) ta thu c nghim t nhin x ', y ' m x ' x v y ' y .
Suy ra (66) c v hn nghim t nhin. nh l 12 c chng minh.
tm cc nghim ca (66) mt cch hu hiu ta p dng th tc sau: trong 1 Dy 2 ta ln lt
thay y bi cc s t nhin 1, 2,3,... v k hiu u l s y u tin m 1 Dy 2 l bnh phng ng.
Khi ta t 1 Du 2 t 2 . Cp t , u l nghim ca (66) vi t , u l cc s t nhin nh nht nh
th v vi mi nghim x, y khc ca (66) ta c y u suy ra x 1 Dy 1 Du 2 t nn

xt .

Trong cc trng hp ring th rt d tm nghim ca (66). c bit nu D c dng a 2 1


trong a l s t nhin 1 th nghim nh nht ca (66) l t a, u 1 . Tng t nu

D a a 1 vi a l s t nhin th nghim nh nht l t 2a 1, u 2 v ta c

2a 1

D 22 1 v nu vi s t nhin x, x2 D 12 1 th ta s c x 2 a 2 a 1 do

x 2 a 2 suy ra x a nn x a 1 v v vy x2 a2 2a 1 a2 a 1 mu thun. Trong


2
trng hp D a 2 vi a l s t nhin th ta chng minh c nghim nh nht ca (66) l
t a2 1, u a v cng vy nu D a 2 1 th cc nghim nh nht l t 2a2 1, u 2a .
V d. Vi D 2 phng trnh (66) tr thnh x2 2 y 2 1. Ln lt thay 1 v 2 cho y trong 1 2y 2
ta nhn c 2 v 9 . Nghim nh nht l x 3, y 2 . Vi D 3 phng trnh (66) tr thnh

x2 3 y 2 1 . Thay 1 cho y trong 1 3y 2 ta nhn c bnh phng ca 2. V vy nghim nh nht l


t 2, u 1 . Vi D 5 ta c phng trnh x2 5 y 2 1 , thay 1, 2,3, 4 cho y trong 1 5y 2 ta nhn
c 6, 21, 46,81 . Nghim nh nht l t 9, u 4 . Vi D 11 ta c phng trnh x2 11y 2 1 ,
thay 1, 2, 3 cho y trong 1 11y 2 ta nhn c 12, 45,100 . Nghim nh nht l t 10, u 3 .

CHNG 2. GII TCH DIOPHANTE BC HAI V CAO HN | 59

Phng php tm nghim trn mc d n gin nhng trn thc t n khng thc s hu dng v
trong nhiu trng hp th phng php ny yu cu mt lng php th rt ln. V d xt
phng trnh x2 13 y 2 1 ta tm c t 649, u 180 vi 180 php th. c bit vi phng trnh
(77)

x2 991y 2 1

Nghim nh nht l

t 379516400906811930638014896080,
u 12055735790331359447442538767.
Nh th nu ta s dng 1028 php th sau kt lun rng phng trnh (77) v nghim th ta s
thu c mt kt lun khng ng. Trong Chng 8 mc 5 ta s trnh by mt phng php hu
hiu hn tm nghim nh nht ca (66). Phng php ny s khng qu tn thi gian tnh
ton. Mt khc ta rng nu D khng phi bnh phng ng th c th tm tt c cc nghim
hu t ca (66). Tht vy, vi s hu t r ty ta t x r 2 D / r 2 D , y 2r / r 2 D th
2r
1 Dy 1 D 2

r D
2


r D 4Dr r

r
r D
2

D
2
x ,
2
D

Do x 2 Dy 2 1 . D dng chng minh tt c cc nghim ca (66) u thu c theo cch ny.


Vic tm tt c cc nghim hu t ca (66) tng ng vi vic tm nghim nguyn ca phng
trnh x2 Dy 2 z 2 . By gi ta tr li vi vic tm tt c cc nghim t nhin ca phng trnh (66).
nh l 13. Tt c cc nghim t nhin ca phng trnh
(78)

x2 Dy 2 1 l

(t0 , x0 ),(t1 , x1 ),(t2 , x2 ),...

Trong t0 , u0 l nghim nh nht v tk , uk c xc nh bi

tk 1 t0tk Du0uk , uk 1 u0tk t0uk , k 0,1, 2,...

(79)

Chng minh. chng minh cc b s trong (78) u tha mn phng trnh (66) th ta gi s
vi k 0 cp tk , uk tha mn (66). R rng cc s trong (79) l t nhin v t ng thc

tk 1 , uk 1 cng tha
x, y ca phng trnh

tk21 Duk21 (t0tk Du0uk )2 D(u0tk t0uk )2 (t02 Du02 )(tk2 Duk2 ) suy ra
mn phng trnh (66). V vy ta ch cn ch ra tt c cc nghim

x2 Dy 2 1 u cha trong dy (78). Ta chng minh b sau y


2
2
B . Nu x, y l nghim t nhin ca x Dy 1 m

u0 y th vi

t0 x Du0 y, u0 x t0

(80)

vi , u l cc s t nhin, y v 2 D 2 1 .
Chng minh. T (80) ta c 2 D 2 t0 x Du0 y D u0 x t0 y t02 Du02 x 2 Dy 2 .
2

H qu l t t02 Du02 1 v x2 Dy 2 1 suy ra 2 D 2 1 . Do ch cn chng minh rng nu v

l cc s t nhin v y th cc bt ng thc 0 t0 x Du0 y v 0 u0 x t0 y y l ng. u

tin

ta
2

ch

D2u02 y 2 t02 1 x 2 1 t02 x 2


2

do

Du0 y t0 x

u0 y ta c

x
1
1 t0
D 2 D 2 . H qu l x / y t0 / u0 suy ra u0 x t0 y do 0 u0 x t0 y .
y
u0 u0
y

60 | Phng trnh x 2 Dy 2 1

chng minh u0 x t0 y y ta ch v t02 Du02 1 suy ra t0 1 do x 2 2 2t0 0 t0 1 .


2

Cng x 2 t02 1 vo mi v ca bt ng thc cui ta suy ra x 2 t02 2t0 1 x 2 t02 1 t0 1 .


2

Do x 2 1 t0 1 x 2 t02 1 v h qu l Dy 2 t0 1 x 2 Du02 suy ra y 2 t0 1 x 2u02 v vy


2

y t0 1 xu0 ngha l xu0 t0 y y . B c chng minh.


2
2
By gi gi s tn ti nghim t nhin ca phng trnh x Dy 1 m khng thuc dy (78) th

trong s ta chn nghim x, y m y ly gi tr nh nht. Khi y vn ln hn

t0 , u0 l nghim nh nht v nu

u0 v nghim

y u0 suy ra x t0 , mu thun vi gi thit x, y khng thuc

dy (78). T b ly cc s , c dng (80) nh ngha t nghim x, y ta nhn thy chng tha

mn phng trnh x2 Dy 2 1 v y . T nh ngha ca nghim x, y suy ra , thuc dy


(78). V vy tn ti k 0 m tk , uk . Khi theo cc cng thc (79), (80) v t02 Du02 1 suy ra

tk 1 t0 Du0 t0 t0 x Du0 y Du0 u0 x t0 y t02 Du02 x x,


uk 1 u0 t0 u0 t0 x Du0 y t0 u0 x t0 y t02 Du02 y y,
T suy ra x, y li thuc v dy (78). Mu thun. nh l 13 c chng minh.
c bit vi phng trnh x2 2 y 2 1 m t0 3, u0 2 th theo cng thc (79) ta tm c dy tt c
cc nghim ca n l t1 32 2 22 17 , u1 2 3 3 2 12 , t2 99 , u2 70 , t3 577 , u3 408 , .
Antoni Wakulicz ch ra t (79) suy ra tk 1 2t0tk tk 1 , uk 1 2t0uk uk 1 vi k 0,1, 2,...
By gi ta chng minh rng

tn1 un1 D (t0 u0 D )n vi n 0,1, 2,...

(81)

Cng thc (81) ng vi n 1 . Gi s n ng vi n. S dng (79) vi k n 1 suy ra


tn un D t0tn 1 Du0un 1 u0tn 1 toun 1 D

t0 u0 D tn 1 un 1 D ,

Do theo (81) suy ra tn un D t0 u0 D

n 1

, chng t (81) ng vi n 1 v v vy theo

quy np suy ra iu phi chng minh. T nh l 13 v cng thc (81) dn ti nh l sau


nh l 14. Nu t0 , u0 l nghim t nhin nh nht phng trnh x2 Dy 2 1 th cp s t nhin t , u
l nghim ca phng trnh ny khi v ch khi

t u D (t0 u0 D )n vi n no .

(82)

Vi cc s t nhin a, b, c, d ty ng thc a b D c d D suy ra a c, b d (v D


v t). V vy khai trin v phi ca (81) theo cng thc nh thc v thu gn n v dng c d D
vi c, d l cc s t nhin ta suy ra tn1 c, un1d . Ta ch rng theo cng thc (82) ca tt c cc
nghim t nhin ca (66) th ta c th thu c tt c cc nghim nguyn ca n. Tht vy nu
t , u l nghim t nhin ca (66) th theo nh l 14 ng thc (82) ng vi s n no . M

t u D 1/ t u D

(ta gi s t

Du 2 1 ) suy ra t u D 1/ t0 u0 D

. Cc s t , u thu

c t t , u bng cch i du, hai nghim cn li thuc lp ny l t , u , t , u . Ta c

CHNG 2. GII TCH DIOPHANTE BC HAI V CAO HN | 61

nh l 15. Mi nghim nguyn ca phng trnh (66) u c dng t u D (t0 u0 D )k vi k


l s nguyn thch hp v u0 , t0 l nghim t nhin nh nht. Ngc li, tt c cc b s nguyn t , u
nhn c t cng thc trn u l nghim ca phng trnh (66). Nghim t 1, u 0 c c khi
k 0.
Cc nghim ca phng trnh (66) cho ta mt phng php xp x cn bc hai ca mt s t nhin
bi cc s hu t. Tht vy, t (66) ta c x y D 1/ x y D , cho nn

0 x / y D 1/ y x y D 1/ y 2 D 1/ y 2 .

V vy nu x, y l nghim t nhin ca (66) th phn s x / y xp x s D vi sai s nh hn


nghch o bc 2 ca mu s. R rng x / y l ti gin. c bit bn nghim c lit k ca
phng trnh x 2 y 1 cho ta phn s 577 / 408 l xp x ca
sau du phy thp phn (v 4082 105 ).
2

2 vi chnh xc 5 ch s

Tip theo c c lng chnh xc vi t bc th hn ta s dng cng thc sau y. Cng thc
ny cho php ta nhy trc tip t nghim

T (81) ta c t2 n1 u2 n1 D t0 u0 D

tn1 , un1 ti nghim t2n1 , u2 n1 .

t
2n

n 1

un1 D , do t tn21 Dun21 1 suy ra

t2 n1 tn21 Dun21 tn21 tn21 1 2tn21 1, u2n1 2tn1un1.

V vy ta c t2 n1 / u2 n1 2tn21 1 / 2tn1un1 . c bit do t2 / u2 99 / 70 l mt xp x


s t5 / u5 19601/13860 l xp x

2 , ta c phn

2 vi chnh xc 8 ch s sau du phy thp phn.

V s 2 ta lu rng nm 1950 R.Coustal [1] tm ra c lng vi 1033 ch s (xem thm


E.Borel [2]) v nm 1951 H.S.Uhler [1] tm ra khai trin vi 1543 ch s ca n. Trong cng
trnh tc gi cng ch ra biu din vi 1301 ch s ca
din

2 vi 106 ch s v 24576 ch s i vi

3 . Cho ti by gi th ta bit biu

3 (Beyer, Metropolia v Neuregerard [1]).

Tr li vi phng trnh x2 2 y 2 1 ta s chng minh rng phng trnh ny khng c cc nghim


t nhin x, y no m x l bnh phng ng. Tht vy nu tn ti nghim t nhin x, y m x u 2

th u phi l s l ln hn 1 . T u 2 8k 1 . Hn na do u 2 1 u 2 1 u 4 1 2 y 2 suy ra

8k 4k 1 y m 2k , 4k 1 1 suy ra 2k a 2 . V vy u 1 8k 2a 2 , v l v hai s t
2

nhin lin tip khng th u l bnh phng ng. Suy ra phng trnh x 2 y 1 khng c
nghim t nhin. D dng chng minh phng trnh v4 2u 2 1 khng c nghim t nhin
4

ngoi tr u v 1 . Tht vy, nu u 1 v v tha mn v4 2u 2 1 th u 4 v 4 u 2 1 vi


2

u, v, d 2 1 l cc s t nhin. Nhng iu ny mu thun vi H qu 1 nh l 3 mc 6. Tuy nhin c


th chng minh cc phng trnh x 2 y z , u 2v w u c v hn nghim t nhin.
4

c bit 3, 2,7 v 113,84,7967 l cc nghim ca phng trnh th nht, 1,13, 239 v

1343,1525, 2165017

l cc nghim ca phng trnh th hai. Hu ht cc phng trnh

Diophante bc hai vi hai bin u c th bin i v phng trnh Pell (Skolem [2] trang 46).
Xt phng trnh
(83)

( x 1)3 x3 y 2

62 | Phng trnh

x2 k y3

vi

k nguyn

R rng phng trnh ny tng ng vi 2 y 3 2 x 1 1 . Do ta ch cn tm nghim


2

nguyn u, v ca phng trnh u 2 3v2 1 m u chn, v l. Phng trnh ny c nghim tm


thng u 1, v 0 . Nghim t nhin nh nht ca n l u0 2, v0 1 do theo nh l 13 th tt

c cc nghim t nhin ca phng trnh ny c xc nh bi dy v hn uk , vk , k 0,1, 2,... ,


trong uk 1 2uk 3vk v vk 1 uk 2vk . Suy ra nu
Ngc li nu

uk chn v vk l th uk 1 l v vk 1 chn.

uk l v vk chn th uk 1 chn v vk 1 l. T y ta suy ra tt c cc nghim t

nhin ca phng trnh u 2 3v2 1 vi u chn v l l u2 k , v2 k trong k 0,1, 2,... Tng t tt


c cc nghim t nhin ca (83) c cha trong dy v hn xk , yk , k 0,1, 2,... , vi x0 0, y0 1
v xk 7 xk 1 4 yk 1 3, yk 12 xk 1 7 yk 1 6, k 0,1,2,... . Ngoi ra nu cc s t nhin x, y tha mn
phng trnh (83) th cc s y u l tng ca hai bnh phng t nhin lin tip. c bit ta c
83 73 22 32 ,1053 1043 92 102 .
2

A.Rotkiewicz [3] nhn xt rng vic gii phng trnh nghim t nhin
(

(84)

vi u v c th c quy v vic gii phng trnh (83) trong tp s t nhin.


Tht vy, ta ch rng nu cc s t nhin x, y tha mn (83) th t u y x 1 , v yx , ta c

u v y v u 3 v3 y3 x 13 x3 y 5 u v 5 . y chnh l cng thc (84).

x, t 1 v bi v u v, t x suy ra theo (84) ta c


y t x t x / t x v t t x / t x t x 3tx
t , x 1 ta suy ra t x 1 , h qu l t x 1, u y x 1 v

t y u, v , x v / y, t u / y , ta c
y 5 t x y 3 t 3 x3 do
5

suy ra t x |3tx . V vy t
2

y 2 x 1 x3 suy ra (83). Do tt c cc nghim t nhin ca (84) vi u v u thu c t


3

cc nghim ca (83) bng php t u y x 1 , v yx .


18. Phng trnh

x2 k y3 vi k nguyn

Cc phng trnh ny c pht hin v nghin cu bi nhiu tc gi. Ta bt u vi nh l tng


qut sau. nh l ny c th p dng cho nhiu gi tr k khc nhau (xem thm Mordell [1]).
nh l 16. Nu a l v b l s chn khng chia ht cho 3 v khng c c s chung dng 4t 3 vi
a v nu k b2 a3 v k khng c dng 8t 1 th phng trnh x 2 k y 3 v nghim.
Chng minh. Gi s phn chng rng tn ti cc s nguyn x, y m x 2 k y 3 . Do b chn v a l

nn s k b2 a3 l. V vy nu y chn th x l v suy ra 8 x 2 1,8 y 3 do t k 1 y 3 x 2 1


ta suy ra 8|k 1 , mu thun vi gi thit k khng c dng 8t 1 . Vy y l v h qu l
x 2u v t b 2c ta c

chn. Do

x 2 b2 4 u 2 c2 y3 a3 y a y 2 ay a 2 .
Do y a chn v a l nn y ay a y a y a l. H qu l 4|y a v y a 4v . V vy
2

y a 4v 2a, y 4v a v

y a y 4w 2a2 . Suy ra

y 2 ay a2 4w 3a 2 . Do a l nn v phi

c dng 4t 3 . H qu l (Chng 5) n c c s nguyn t p c cng dng m ly tha cao

CHNG 2. GII TCH DIOPHANTE BC HAI V CAO HN | 63

nht

tha mn

p s l c ca 4w 3a 2 l ly tha bc l. t s 2 1 th do p2 1|y 2 ay a2

v y 2 ay a2 |x2 b2 ta c p 2 1|x2 b2 . t d x, b , x dx1 , b db1 th x1 , b1 1 v p 2 1|d 2 x12 b12 .


Ta bit (Chng 11) tng bnh phng ca hai s m t nht mt s khng chia ht cho s
nguyn t p c dng 4t 3 khng th chia ht cho p . Ta c p 2 1|d 2 do p 2 |d 2 v p |d . T

p |x v p |b do p 2 | y a y 2 ay a 2 . Do s ln nht m s s | y 2 ay a 2 l l nn ta c
2
2
2
p|y a . Cng vy t p|y ay a y a y 2a 3a ta suy ra p3a . Li do

pb v b khng chia

ht cho 3 suy ra pa , mu thun vi gi thit ca a v b . nh l 16 c chng minh.


2
3
H qu. Phng trnh x k y khng c nghim t nhin vi k 3,5,17, 11, 13 ( rng

3 22 13 ,5 22 1 , 11 42 33 ,17 42 1 , 13 702 173 .


3

nh l 17. Nu a l s nguyn c dng 4t 2 v b l khng chia ht cho 3 v khng c c s


chung dng 4t 3 vi a th vi k b2 a3 phng trnh x 2 k y 3 v nghim.
Chng minh. Phn chng. Gi s tn ti cc s nguyn x, y tha mn x 2 k y 3 . Do k b2 a3 v
t gi thit ca a v b suy ra s k c dng 8t 1 . T nu y chn th x2 y3 k c dng 8t 1 ,
v l. Vy y l v do x chn. Nu y c dng 4t 1 th y a c dng 4t 3 v do n c c
s nguyn t p c dng ny m ly tha ca p trong phn tch thnh c s nguyn t ca
2
2
3
3
2 1 2
2
y a l s l, ngha l 2 1 . Hn na t x b y a ta c p |x b . Do nh trong

chng minh nh l 16 ta suy ra p |b v p |x . Vy p|3a . Nhng p|b m b khng chia ht cho 3

suy ra p 3 . Vy ta c pa , mu thun vi gi thit. Do ta ch cn xt trng hp y c dng

4t 3 . Khi y a c dng 4t 1 v y y a c dng 4t 3 . Do y 2 ay a 2 c dng 4t 3 nn

tng t chng minh nh l 16 th ta chng minh c x 2 b2 y3 a3 y a y 2 ay a 2

c c s nguyn t p c dng 4t 3 vi ly tha l trong phn tch thnh c s nguyn t. Mu


thun. nh l 17 c chng minh.
2
3
H qu. Phng trnh x k y khng c nghim nguyn vi k 9 v k 7 ( 9 12 2

v 7 1 2 ) (V.A.Lebesgue [2] tm ra mt chng minh cho trng hp k 7 vo nm


1869)
2

nh l 18. Phng trnh x2 12 y3 khng c nghim nguyn x, y .


2
3
Chng minh. Gi s phn chng tn ti x, y tha mn x 12 y . Nu x chn th x 2 x1 v y
2
3
cng chn th y 2 y1 . Vy x1 3 2 y1 v

x1 l, t x12 c dng 8t 1 suy ra 2 y13 x12 3 c dng

8t 4 nn y1 c dng 4t 2 . V l v lp phng ca mt s chn chia ht cho 4. Suy ra x v y

u l. Ta c x 2 4 y 2 8 y 2 y 2 2 y 4 . Do y l nn suy ra s y 2 2 y 4 c dng 4t 3 .
Do s x 2 22 , vi x, 2 1 , c c s c dng 4k 3 , v l. nh l 18 c chng minh.
Ta lu rng Mordell chng minh nh l tng qut hn: nu k 2a 2b vi a l s l
2

khng chia ht cho 3 v b c dng 4t 3 , hn na a, b khng c c s c dng 4t 3 th phng


3
2
3
trnh x k y v nghim. c bit v 12 22 2 , 20 142 63 nn phng trnh x k y

khng c nghim nguyn vi k 12, k 20 .


nh l 19. Phng trnh x 16 y v nghim.
2

64 | Phng trnh

x2 k y3

vi

k nguyn

Chng minh. Nu x chn th y cng chn v do x 2 x1 , y 2 y1 , x1 v y1 l cc s nguyn. V

x1 chn v do x1 2 x2 suy ra 2 x22 2 y13 . T y1 2 y2 suy ra

2
3
vy x1 4 2 y1 v h qu l

x22 1 4 y23 , v l. Vy x l suy ra

y 3 c dng 8t 1 . Nhng t y suy ra

y c dng 8t 1 . H qu

l y 2 c dng 8t 1 . Do y 2y3 8 x 2 8 nn cc s x 2 8 c c s c dng 8t 1 . Suy ra


x 2 8 c c s nguyn t

p c dng 8k 5 hoc 8k 7 . Nhng nh th th px 2 8 m y li l

iu l khng th xy ra (Chng 9). nh l 19 c chng minh.


nh l 20. Phng trnh x 16 y khng c nghim no ngoi tr x 4, y 0 .
2

2
3
Chng minh. Gi s x, y tha mn x 16 y . Nu x l th ta c ( x 4, x 4) 1 do t

( x 4)( x 4) y3 suy ra tn ti cc s l a, b tha mn x 4 a3 , x 4 b3 do a3 b3 8 v l v 8


khng phi hiu ca hai lp phng l. Suy ra x chn tc l x 2 x1 . T y cng chn, y 2 y1 . V
vy x12 4 2 y13 , Suy ra
T y suy ra

x1 chn, x1 2 x2 v do y1 cng chn, ngha l y1 2 y2 . Ta c x22 1 4 y23 .

x2 l, ngha l x2 2 x3 1 . V vy 4 x23 4 x3 4 y23 v t

x3 ( x3 1) y23 , m ( x3 , x3 1) 1

3
3
suy ra tn ti cc s nguyn a v b tha mn x3 a , x3 1 b nhng t y suy ra y2 0 v t y
= 0 v x 4 . nh l 20 c chng minh..

A.Thue [2] (xem Mordell [2]) chng minh rng vi mi s nguyn k 0 th phng trnh

x2 k y3 ch c hu hn nghim nguyn.
2
3
H qu 2 nh l 9 cho ta li gii hon chnh cho phng trnh x 1 y . Phng trnh x 1 y

khng c nghim nguyn v cng khng c nghim hu t. Phng trnh

x2 2 y3 c nghim t

nhin duy nht x 5, y 3 . Kt qu ny mc d c bit ti t thi Fermat nhng li gii ca


n vn l rt kh (Fermat [1] trang 345 v 434; chng minh y u tin c cho bi T.Pepin
[1] v sau bi Euler). Li gii ny c th tm thy trong Uspensky v Heaslet [1]. Chng minh
c trnh by y khng s dng ti trng Q( 2) .
Chng minh phng trnh x 2 y ch c nghim x 1, y 1 cng l rt kh. Li gii u tin
c trnh by bi A.Brauer [1] nm 1926 da trn l thuyt v cc idean, li gii trong Uspensky
v Heaslet [1] trnh s dng l thuyt .
2

S nghim ca phng trnh x 2 k y 3 c th ln ty .


T.Nagell [3] vo nm 1930 chng minh rng vi k 17 th phng trnh c ng 16 nghim
l ( x, y) (3, 2), (4, 1), (5, 2), (9, 4), (23,8), (282,3), (375,52), (378661,5234) .
Cng v cc phng trnh ny, O.Hemer bo v lun n ca mnh (Hemer [1]). Cc chnh sa v
thng tin b sung c tm thy trong bn ghi ch sau (Hemer [2]) v cun sch ca London v
Finkelstein [1]. Hermer tm tt c cc nghim ca phng trnh x k y vi mi k m
100 k 0 . Vi s dng k 100 cng vic tng t c hon thnh bi F.B.Conghlan v
N.M.Stephens [1].
2

3
2
10
10
V mt l thuyt bi ton cng c gii bi A.Baker [1]: x y 10 (log x )

nu

x3 y 2

vi mi s nguyn x, y (Stark [2]). M.Hall, Jr [3] t ra gi thuyt rng vi s c 0 no th


bt ng thc 0 x3 y 2 c x khng c nghim nguyn. Gn y Danilov [1] chng minh
rng vi v hn cc s nguyn x, y th 0 x3 y 2 0.97 x . tng ca Danilov dn ti nh l
mnh hn sau y

CHNG 2. GII TCH DIOPHANTE BC HAI V CAO HN | 65

nh l 21. Vi v hn s t nhin x, y ta c bt ng thc 0 x3 y 2

54 x
25 5

Chng minh. Ta c (t 2 6t 11)3 (t 2 5)2 [(t 9)2 4] 1728t 3456. Theo nh l 14 vi 0 930249,0 83204 ,

n l th phng trnh 2 125 2 1 c v hn nghim t nhin , m 125 | 1.

t t 1364 1525 9,
khi ta thy
t x

u 61 682 ,

t l, 125 | t 2 , (t 9)2 4 500u 2 , t

t 2 6t 11
,
20

6t 11 (t 2 5)
27
4
(t 2) .

20

4
125

(t 2 5)
u.
4

Ta c t 20 x 20 3 20x , 0 t 2 20x v t suy ra iu phi chng minh.


Phng trnh x2 k y3 vi 2 k 20 c nghim x, y 0 vi k = 4,7,11,13,15,18,19,20, -3, -5, -8, 9, -10, -12, -15, -17, -18, -19 do 22 + 4 = 23, 12 + 7 = 23, 42 + 11 = 33 (cng vy 582 + 11 = 153), 702 +
13 = 173, 72 + 15 = 43, 32 + 18 = 33, 182 + 19 = 73, 142 + 20 = 63, 22 - 3 = 13, 22 - 5 = (-1)3, 42 - 8 = 23,
12 - 9 = (-2)3, 32 - 10 = (-1)3, 22 - 12 = (-2)3, 42 - 15 = 13 (cng vy 11382 - 15 = 1093), 42 - 17 = (-1)3
(cng vy 32 - 17 = (-2)3), 192 - 18 = 73, 122 - 19 = 53, vi tt c cc gi tr khc ca k m 2 k 20
th phng trnh khng c nghim hu t x, y 0 ngoi tr vi k 11 th khng c nghim
2

19
7
nguyn nhng c nghim hu t, c th 11 . T ng thc
8
4
2

27 y 6 36 x 2 y 3 8 x 4
9 y 4 8x2 y
3
2

2
8 x3

4x

suy ra mi nghim hu t ca phng trnh x 2 k y3 m x, y 0 u cho thm mt nghim khc.


Theo R.Fueter [1] th nu tn ti mt nghim nh vy th vi k 1, 432 ta s c v hn nghim.
Hin nhin cc nghim hu t ca phng trnh x2 k y3 thu c t cc nghim nguyn ca
phng trnh u 2 kw6 v3 vi w 0 bng cch t x u w3 , y v w2 .
Tht vy, d dng kim tra c x2 k y3 . t x m / n, y r / s vi m, r l cc s nguyn v n, s
l cc s t nhin. Khi t u mn2 s3 , v rn 2s, w ns , ta thy cc s u, v, w nguyn, w 0 v tha
3
2
mn u 2 kw6 v3 vi u w m n , v w r s .

Cc nghim hu t ca phng trnh x 2 k y3 c tm ra bi J.W.Cassels [1], [2] v E.S.Selmer


[3]. J.W.Cassels [1] ch ra cc nghim c bn ca phng trnh u 2 kw6 v3 vi cc gi tr k c
gi tr tuyt i 50 m tn ti nghim khng tm thng (trang 268). Selmer tip tc pht
trin bng ca Cassels ti 100 .
Ta ch rng vic phng trnh u 3 v3 w3 khng c nghim nguyn vi uvw 0 l tng
ng vi vic phng trnh x2 432 y3 khng c nghim hu t no ngoi tr x 36, y 12 .
Tht vy, gi s cc s hu t x, y tha mn x 432 y , x 36 . Hin nhin y 0 . Cc s x/36
v y/12 l hu t, 7/12 > 0. Gin c ta c x 36 k n , y 12 m n vi k l s nguyn v m, n
l cc s t nhin. Khng gim tng qut gi s k v n chia ht cho 2 do ta c th thay n, k , m bi
2

2n, 2k , 2m ln lt nu cn. t u

nk
nk
,v
, w m . R rng u, v, m nguyn v hn na
2
2

66 | Phng trnh

x2 k y3

vi

k nguyn

v vy
w 0 . Ta c u 3 v3 w3 n k n k m3 n 3nk m3 . Nhng k
,m
46
12
4
4
2 2
n3 3n3 x 2 n3 y 3
n3
3
3
3
u v w
3
432 x 2 y 3 0 . Suy ra nu phng trnh x2 432 y3 c nghim

2
4 4.36 12
1728
hu t x, y v x 36 th phng trnh u 3 v3 w3 c nghim nguyn u, v, w vi uvw 0 . Mt
khc gi s cc s nguyn u, v, w vi uvw 0 tha mn u 3 v3 w3 . Ta c
3

nx

ny

u 3 v3 u v u 2 uv v 2 v w 0 suy ra u v 0 . V vy t x 36(u v) / (u v) ,
y 12w / (u v ) ta c cc s hu t u, v tha mn

123 u 3 v3 362 u v 2
y x

3
2
u v
u v
3

123 u 2 uv v 2 362 u 2 2uv v 2

u v

432

v suy ra x2 432 y3 .
Tng t chng minh ny ta c nhn xt rng phng trnh u 3 v3 Aw 3 vi A l s t nhin l
c nghim nguyn vi uvw 0 khi v ch khi phng trnh x 432 A y c nghim hu t. T
2

y suy ra chng minh phng trnh x y z khng c nghim nguyn 0 ta ch cn


3

chng minh phng trnh x3 16 y3 khng c nghim hu t x, y khc 0. rng nu u, v, w

khc 0 v tha mn u 3 v3 w3 th cc s hu t x v3 w3 / v3 v y 4vw 2 u khc 0 v


tha mn x2 16 y3 .
n
n
n
T.R.Bendz [1] ch ra nh l Fermat ln ni rng phng trnh x y z khng c nghim t
4
n 1
n
nhin vi n 2 tng ng vi vic phng trnh x 4 y khng c nghim hu t khc 0.

kt thc mc ny ta lu nh l ni rng cc phng trnh x 2 k y n vi 0 k 10, k 7 u


v nghim vi n 3 c chng minh bi O.Korhonen [1], V.A.Lebesgue [1], W.Ljunggren
[2],[3] v T.Nagell [2],[8],[9],[10].
Bi tp. 1. Chng minh nh l V.Bouniakowsky [1] (nm 1848) ni rng phng trnh
(i)

x mt n y mu n z mu n

vi cc s m, n nguyn t cng nhau s c v hn nghim t nhin x, y, z, t , u, v .


Chng minh. Theo nh l 16 Chng 1 tn ti cc s t nhin r , s m mr ns 1 . Chn a, b l
cc s t nhin ty . t c a b . D thy x ar , y br , z cr , t bsc s , u a sc s , v a sbs tha mn (i).
2. Chng minh rng phng trnh x2 y3 z 5 c v hn nghim t nhin.
Chng minh. Cc s x n10 n 1 , y n7 n 1 , z n4 n 1 , n 1, 2,... , tha mn phng
8

trnh.
3. Chng minh rng vi n 1 phng trnh xn y n z n1 c v hn nghim t nhin.
Chng minh. Ta c

1 k k 1 k n 2 1 k
n
n n2

4. Chng minh rng phng trnh

n 1
n n 1

xn y n z n1 c v hn nghim.

vi n 2 .

CHNG 2. GII TCH DIOPHANTE BC HAI V CAO HN | 67

Chng minh. Ta c 1 k n [k 1 k n ]n 1 k n
n

n 1

m
n
p
Ghi ch. Phng trnh Ax By z v tng qut hn l

Aix i 0

c cp ti bi mt s

9
t

t 1

tc gi (Tchacaloff et Karanicoloff [1], Vijayaraghavan [1], Georgiev [1], Schinzel [12]).


5. Chng minh mnh dng Fermat ln sau y: nu n l s t nhin ln hn 2 th phng trnh

x n x 1 x 2 khng c nghim t nhin.


n

Chng minh. Gi s n l s l 2 ; nu vi x t nhin no ta c x n x 1 x 2 th vi


n

n
n
y x 1 ta c y y 1 y 1 , do y 2
n

y
n
1

n 1

2 3n y n3 ... 2 nn2 y 2 2 . Suy ra y 2

l c s ca 2 m y x 1 1 suy ra v l. Nu n chn v > 2 th t y x 1 suy ra

y n 2 1n y n1 2 nn1 y 0 . Do y n1 2 1n y n2 2 3n y n4 ... 2n 0 . ng thc th nht

suy ra

y n 2ny n1 do

y 2n ; ng thc th hai suy ra

y l c ca 2n ; mu thun.

Ghi ch. B.Leszczynski [1] chng minh b s nguyn dng duy nht n, x, y, z vi y 1 m

n x n 1 n 2 l n 1 , x ty , y 3, z 2 v n 3, x x z 2 . Trng hp y 1
c t ra bi Demyanenko [4] v theo cch n gin hn bi Chain [1].
y

19. Mt s phng trnh m

x y y x . Ta tm tt c cc nghim hu t dng ca phng trnh ny m y x .


y
x
Ta c r x / y x l s hu t dng v y 1 1/ r x. Do x x v t x y suy ra
1. Phng trnh

11/ r x

11/r x

y , t x

11/ r

y 1 1/ r x. V vy x

1/ r

1
1 1/ r v h qu l x 1 ,
r

m n t m . C hai v u l phn s
t
1
mn
,
do

y 1 . t r n / m, ta c

nn
sm
s
r
n
ti gin v m, n 1 , ta c m n, n 1 , do (m n)n , nn 1, v v t , s 1 , ta c t m , s m 1 .
r 1

n/m

Vy m n t m v nn s m . T y theo H qu 1 nh l 16 Chng 1, do m, n 1 suy ra tn


n

m
n
ti cc s t nhin k v l tha mn m n k , t k v n l , s l . Do m l m k m . T y

suy ra k l 1 . Nu m 1 th ta s c k m l 1 l m ml m1 1 l m m k m , v l. H qu l
m

m 1 do r n / m n . Suy ra
n

1
1
x 1 , y 1
n
n

(85)

n 1

trong n l s t nhin. Ngc li, d thy cc x, y c nh ngha bi (85) tha mn phng


trnh ban u. V vy tt c cc nghim hu t ca phng trnh x y vi y x 0 u c
cho bi (85) vi n l s t nhin. T y suy ra n 1 l trng hp duy nht m phng trnh c
nghim t nhin. Trong trng hp ny nghim s l x 2, y 4 . V vy ta kt lun rng phng
y

trnh x y y x c duy nht mt nghim t nhin x, y vi y x . Tnh cht ny cng c suy ra t


3

3 2 2 4 4 5 5 6 6 ... 1 1 . Phng trnh x y li c v hn nghim hu t x, y vi y x .


y

27

9 8
27 4
Khi n 2 ta c .
4
8

2. Phng trnh x y y x 1 . Theo nh l Moret Blanc [1] th phng trnh

68 | Mt s phng trnh m

(86)

xy yx 1

c ng hai nghim t nhin l x 2, y 1 v x 3, y 2 .


Ta s chng minh nh l ny. Gi s cc s t nhin x, y tha mn (86). Khi x y 1 v do

x 1 . Nu x 2 th theo (86) ta c 2 y 1 suy ra y l v h qu l 4 y 2 1. T y suy ra


y

4 2 y 2 v

2 | 2 y1 1 . Vy

(87)

y 1 . Ta c

3 2 2 4 4 5 5 6 6 ... 1 1.

V theo (86) th x y , x
y

1/ x

y1/ y . Cc s

x 3, y 1 khng tha mn (86) nhng x 3, y 2

tha mn. V vy nu x, y l nghim ca (86) khc vi 2,1 v 3, 2 th x 3, y 4 hoc, v

x1/ x y1/ y v (87), x 4, y x 1. Vy trong c hai trng hp ta u c y x 1. t


y x a . Hin nhin a l s t nhin v cc ng thc sau l ng
xy
x xa
xa

y x x a x a x
1
x

(88)

M et 1 t vi t 0 suy ra vi t a / x ta c 1 a / x ea . Vy theo (88) v x 3 e suy ra


x

x y xa x
x 3
a 1.1.
x
y
e
e e
e

y x 43
1 , mu thun vi gi thit v b x, y l nghim ca (86). Suy ra phng
10 10
trnh (86) khng c nghim no ngoi tr x 2, y 1 v x 3, y 2 .
Do x y y x

3. Phng trnh x x y y z z . Phng trnh ny c v hn nghim t nhin khc 1. Nh ch ra bi


Chao Ko [2] th vi s t nhin n ta c cc s
x2

n 1 2n n 1 2 n

2 2n1

, y2

2 2 1 1
2 n 1 2 n 1 n 1
2 n 1 2 2 2 ,
z2

2n 1 .

2 n 1 2n n 1

n1

12
6
tha mn phng trnh x x y y z z . Vi n 2 ta c x 2 3 2985984 , y 28 38 1679616 ,

z 211 37 4478976 . Chao Ko cng chng minh rng phng trnh x y z khng c
nghim t nhin x, y, z m mi s u ln hn 1 v x, y 1 .
x

V.A.Demyanenko [3] chng minh rng nu x, y, z

l cc s t nhin ln hn 1 tha mn

phng trnh x y z th x, y phi c cng cc c s nguyn t (Chng 3 mc 1).


x

Ta cha bit phng trnh x x y y z z c nghim l ln hn 1 hay khng.


4. Phng trnh x ! y ! z ! . Khng kh chng minh rng phng trnh ny c v hn nghim
t nhin x, y, z m mi s u ln hn 1. Tht vy vi mi s t nhin n ln hn 2 th cc s
x n! 1, y n, z n! tha mn phng trnh. Khi n 3 ta c 5!3! 6!. Phng trnh ny cn
c cc nghim khc khng c dng ny chng hn 6!7! 10! . Ta cha bit c tn ti cc nghim
khc na hay khng (Guy [1] trang 44 ). Mt khc, rt d tm tt c cc nghim t nhin ca
phng trnh x ! y ! z ! . Tht vy nu x, y, z l nghim th ta c th gi s x y v do z y ,
ngha l z y 1 , suy ra z ! ( y 1)!. Nhng z ! x ! y ! y !2 , do y !2 y 1! y ! y 1

CHNG 2. GII TCH DIOPHANTE BC HAI V CAO HN | 69

v h qu l y i 2 , ngha l y 1 , suy ra x 1 v z 2 . Phng trnh x ! y ! z ! ch c


ng mt nghim t nhin l x 1, y 1, z 2 . Cc phng trnh dng giai tha c nghin
cu bi P.Erdos, R.Oblath [1] v R.M.Pollack v H.N.Shapiro [1].

CHNG 3
S NGUYN T
1. S nguyn t v phn tch s t nhin thnh tch cc s nguyn t
Cc s t nhin ln hn 1 khng c c s no ngoi 1 v chnh n c gi l s nguyn t hoc
gn hn l nguyn t. iu kin cn v mt s t nhin m 1 l nguyn t l m khng th
phn tch thnh tch ca hai s t nhin nh hn m. Tht vy nu m l s nguyn t th m khng
th biu din thnh tch a b ca hai s t nhin nh hn m v nu ngc li th cc s a v b l
cc c s t nhin ln hn 1 v nh hn m ca m. iu kin cn c chng minh. Mt khc nu
m khng phi s nguyn t th n c c s a vi 1 a m v do m a.b vi b l s t nhin
nh hn m v a 1 . iu kin c chng minh.
T nh ngha ca cc s nguyn t ta c ngay mt phng php quyt nh mt s t nhin
cho trc n 1 l nguyn t hay khng. Theo ta ch cn ln lt chia n cho cc s 2,3,..., n 1
v nu c mt php chia ht th n khng phi s nguyn t, ngc li th n l s nguyn t.
Mt s t nhin khng bng 1 v cng khng nguyn t c gi l hp s. l cc s c th biu
din nh l tch ca hai s nguyn dng ln hn 1.
Xt hp s n a.b ta c th gi s a b khi a2 ab n, suy ra a n. V vy ta c nh l
nh l 1. Nu s t nhin n l hp s th n c c s a tha mn 1 a n .
Do xt xem mt s t nhin n 1 c l s nguyn t hay khng ta ch cn em s chia
cho cc s ln hn 1 v khng vt qu

n . By gi ta chng minh nh l

nh l 2. Mi s t nhin 1 c t nht mt c s nguyn t.


Chng minh. Xt s t nhin n 1 . Hin nhin n c cc c s ln hn 1. K hiu p l c s
nh nht ln hn 1 ca n . Nu p khng phi s nguyn t th ta c p a.b vi cc s t nhin
1 a, b p v vy a l c s ln hn 1 ca n v nh hn p . Mu thun suy ra p l c s
nguyn t ca n . nh l c chng minh.
T hai nh l c bn ny ta c
H qu 1. Mi hp s n c t nht mt c s nguyn t

n.

H qu 2. Mi s t nhin 1 u l tch ca hu hn s nguyn t.


Chng minh. Phn chng. Gi s n 1 l s t nhin nh nht m khng l tch ca cc s
nguyn t. T nh l 2 suy ra n c c s nguyn t p tc l n p.n1 vi n1 l s t nhin. Ta
khng th c n1 1 v nh th n p mu thun vi gi thit. V vy n1 1 v n n1 v t gi thit
v tnh nh nht ca n suy ra n1 l tch ca hu hn cc s nguyn t. Khi n p.n1 li l tch
ca hu hn cc s nguyn t. Vy H qu 2 c chng minh.
Mt vn c t ra l lm sao tm c cch biu din mt s t nhin cho trc thnh tch
ca cc s nguyn t. Ta s ch ra mt phng php nh vy mc d cc php tnh ton c th c
th s rt di. Ta s chng minh rng vic tm phn tch thnh tha s nguyn t ca s t nhin
cho trc c th chuyn v bi ton tm phn tch thnh tha s nguyn t ca cc s t nhin nh
hn s cho trc . Gi s n l s t nhin 1 . Chia n ln lt cho 2,3,..., n ta s tm c c
s nguyn t nh nht p ca n. Ta c n p.n1 vi n1 l s t nhin. Nu n1 1 th n p v ta c
biu din cn tm. Nu ngc li ta lp li qu trnh ny vi n1 n . Sau hu hn bc ta s thu
( k 1)

c biu din thnh tha s nguyn t n pp ' p ... p . Nu trong tch ny cc tha s lp li th
ta thay chng bng cc ly tha thch hp v ta nhn c
''

72 | S nguyn t v phn tch s t nhin thnh tch cc s nguyn t

(1)

n q1a1 q2a2 ...qsas

trong q1 , q2 ,..., qs u l cc s nguyn t phn bit, ngha l q1 q2 ... qs v ai (i 1,2,..., s) l


cc s t nhin. Biu din dng ny c gi l phn tch thnh tha s nguyn t ca s t nhin
n . R rng q1 , q2 ,..., qs l tt c cc c s nguyn t ca n . Tht vy nu n c c s nguyn t q
khng trng vi s no trong cc s

q1 , q2 ,..., qs th vi

i 1, 2,..., s ta c

q, qi 1

v do s

nguyn t q ch c hai c s l q v 1 vi q qi nn chng l cc s nguyn t khc nhau nguyn

t cng nhau. Ta cng c q, qiai 1 vi i 1, 2,..., s v vy t (1) v nh l 6a Chng 1 ta c

q, n 1 mu thun vi gi thit. Ta cng c cc s qi i 1, 2,..., s c xc nh duy nht theo n


(nh l cc c s nguyn t ca n ). Hn na cc ly tha a1 , a2 ,..., as cng xc nh duy nht theo
a 1
n . c bit s a1 c th nh ngha nh l ly tha ln nht m q1a1 n v nu q1 1 | n ta c

q1 q2a2 ...qsas | n l iu v l. V ta gi s

q1 , q2 ,..., qs l dy tng nn phn tch (1) l duy nht.

Ta c nh l sau y
nh l 3. Mi s t nhin c th biu din duy nht thnh tch ca cc s nguyn t nu khng tnh
ti th t ca cc tha s nguyn t trong phn tch.
Mc d ta c th xc nh phn tch thnh tha s nguyn t ca mi s t nhin nhng cc php
tnh ton c th c th s rt di. c bit trong trng hp ca s 2293 1 . Ta bit s ny c 89
ch s v l hp s v c s nh nht ca n c 11 ch s. Tuy nhin ta vn cha bit tt c cc
c s ca s . Ta cng cha bit F20 22 1 c nhng c s nguyn t no, hn na ta cng
20

cha bit n c phi s nguyn t hay khng. Ta bit mt c s ca F9448 l 19 29450 1 nhng
ta cha xc nh c tt c cc c s nguyn t ca n. Mt hp s khc cha xc nh c cc
2

c s nguyn t l F20 .
nh l 4. Nu s t nhin n 2 th gia n v n ! c t nht mt s nguyn t.
Chng minh. V n 2 nn s N n! 1 ln hn 1 v vy theo nh l 2 th n c c s nguyn t
p . c s ny khng th nh hn hoc bng n v nu ngc li n chia ht 1, v l. Do p n .
Mc khc ta li c p N v p l c s ca N suy ra n p n! 1 n! .
T y suy ra vi mi s t nhin u c s nguyn t ln hn n. Vy c v hn s nguyn t. c
bit ta bit tn ti cc s nguyn t c hng trm nghn ch s, nhng ta cha bit mt s no nh
vy. S nguyn t ln nht tm c l 2216091 1 c 65050 ch s. S ny c tm ra nm 1985
Bi tp. 1. Chng minh rng s ch s ca mt s nguyn t m biu din thp phn ca n gm
ton ch s 1 phi l s nguyn t (lu rng iu ngc li khng ng).
Chng minh. Gi s n l s nguyn t nh vy c s ch s 1 trong biu din thp phn. Gi s s
s
ab
l hp s, ngha l s ab vi a, b l cc s t nhin ln hn 1 . Khi ta c n 10 1 10 1 .

Nhng 10a 1|10ab 1 v vy

10 1
10 1
n .
9
9
a

l s t nhin 1 v a 1 . V b 1 ta c

10a 1
10a 1 10ab 1
nh hn n v ln hn 1 . Mu thun.

n . T y suy ra n c c s
9
9
9
Chiu ngc li khng ng chng hn 111 3 37 v 11111 41 271. Ta cha bit dy s
11,111,1111,... c cha v hn s nguyn t hay khng. M.Kraitchik [2] (Chng 3) chng

CHNG 3. S NGUYN T | 73

minh s 1023 1 9 l nguyn t. Williams v Dubner [1] chng minh vi p 10000 th

10

1 9 nguyn t ch trong cc trng hp p bng 2,19, 23,317 hoc 1031.

2. Chng minh rng tn ti v hn cc s t nhin khng c dng a 2 p vi a l s nguyn v p


nguyn t.
Chng minh. Cc s 3n 2 vi n 1, 2,... khng c dng . Phn chng. Gi s vi s t
2

nhin n no ta c

3n 2

a 2 p vi a l s nguyn dng, p nguyn t. Khi

3n 2 a, suy ra 3n 2 a 0 . Nhng p 3n 2 a 3n 2 a suy ra 3n 2 a 1 v


3n 2 a p , suy ra p 6n 3 3(2n 1) . iu ny l khng th.
Ghi ch. C th chng minh rng vi mi s t nhin k tn ti v hn cc ly tha bc k cc s t
k
nhin khng c dng a p vi a nguyn v p nguyn t (Clement [2]). Euler chng minh

rng mi s t nhin l n vi 1 n 2500 u khng c dng n 2a 2 p vi n nguyn v p


nguyn t. iu ny khng ng vi 5777 v 5993 (Dickson [7] tp 1 trang 424). Ta cha bit c
tn ti v hn cc s t nhin l khng c dng 2a 2 p vi a nguyn, p nguyn t hay khng.
3. Chng minh rng tt c cc s c dng 8n 1 u l hp s.
Chng minh. Ta c 2n 1| 23n 1 8n 1 v r rng 1 2n 1 8n 1 . Suy ra 8n 1 l hp s.
Ghi ch. Ta cha bit c tn ti v hn cc s nguyn t c dng 10n 1 hay khng. Ta cng cha
bit c phi mi s c dng 12n 1 u l hp s hay khng (n 1) .
2. Sng Eratosthenes v bng cc s nguyn t
T H qu 1 trong mc 1 suy ra nu s t nhin n 1 khng chia ht cho mi s nguyn t n
th n l s nguyn t. Vy tm c tt c cc s nguyn t trong dy 2,3, 4..., m, vi s t
nhin cho trc m th ta ch cn b ra khi dy tt c cc bi s kp ca cc s nguyn t p m
vi k 1 . V vy trong trng hp ring tm c tt c cc s nguyn t trong dy 2,3...,100
ta ch cn b ra khi dy tt c cc bi s ca 2,3,5 v 7 .
Mt phng php n gin tm ra tt c cc s nguyn t lin tip c a ra bi nh ton
hc Hy Lp Eratosthenes. Xt dy 2,3, 4,... th v 2 l s nguyn t u tin k hiu l

p1 ta b ra

khi dy tt c cc s chn ln hn 2 . S u tin cn li l 3 p2 . Ta li b i tt c cc s ln hn

p2 v chia ht cho p2 . S u tin cn li l 5 p3 . Gi s sau bc th n ta tm c s nguyn t


th n l pn th ta loi b ra khi dy tt c cc s ln hn pn v chia ht cho pn . S u tin thu
c trong cc s cn li pn 1 chnh l s nguyn t th n 1 . Nu ta ch xt dy 2,3,..., N , th
qu trnh trn s dng li bc th k vi

pk l s nguyn t ln nht N . Ta nhn c

p1 2, p2 3, p3 5, p4 7, p5 11, p6 13, p7 17, p8 19, p9 23, p10 29, p25 97,


p100 541, p200 1223, p1000 7917, p1229 9973, p1230 10007.
Gn y ta tnh c p6000000 104395301 (xem ghi ch [1]).
D.Blanusa [1] ch ra phng php hnh hc tng ng cho sng Eratosthenes.

74 | Sng Eratosthenes v bng cc s nguyn t. Hiu ca cc s nguyn t lin tip

1
Trong h trc ta Castesian ta xt tp hp A gm cc im 0, , m 1, 2,..., v tp hp B
m

gm cc im n 1,0 , n 1, 2,..., mi im ca tp hp A c ni vi mi im ca tp hp

B bng mt ng thng. Khi tp hp cc honh ca cc giao im ca cc ng thng ny

vi ng thng y 1 l tp hp cc hp s. Tht vy phng trnh ng thng i qua

1
0, v n 1,0 l x / n 1 my 1. ng thng ny ct ng thng y 1 ti im c
m
honh x m 1 n 1 . V m v n l cc s t nhin nn x l hp s. Ngc li, nu x l hp
s th x m 1 n 1 vi m, n l cc s t nhin v do n l honh giao im ca ng

1
v n 1,0 vi ng thng y 1 .
m

thng ni 0,

c mt bng y cc s nguyn t nh hn 7 triu (D.N.Lehmer [1]). Trong bng ny cc c


s ln hn 2,3,5, 7 ca tng s t nhin khng ln hn 10170000 c trnh by y . Trong
Kulik, Poletti [1] cc s nguyn t nh hn 11 triu c trnh by y . Nh ton hc Ba Lan
Jacob Philip Kulik sinh nm 1793 ti Lww v mt nm 1863 ti Prague dnh 20 nm son
mt bn tho c tn l Magus Canon Divisorum pro omnibus numbers par 2,3,5 non
divisilibus et numerorum primorum interjacentium ad Millies centum millia, accuratius ad
100330201 usque. Authore Jacobo Philippo Kulik Galiciano Leopolensis Universitate Pragensi
Matheseos sublimioris Prof. publ.ac ord. Hin nay bn tho ny c bo qun bi Vin khoa
hc Vienna. Bng lit k cc s nguyn t nh hn 7 triu c s dng bn tho ny vi nhng
chnh sa mt s li. Bi bo ni v J.P.Kulik v cng vic ca ng cng cc phc tho chn dung
va c pht hnh gn y bi I.Ya.Depman [1]. V lch s ca bng cc s nguyn t xem ti liu
dn trang 594 - 601 . Vo nm 1959 C.L.Baker v F.J.Gruenberger xy dng mt bng cha
tt c cc s nguyn t nh hn 104395301 (Baker v Gruenberger [1]).
3. Hiu ca cc s nguyn t lin tip

pn l s nguyn t th n v t dn pn1 pn vi
Cc s u tin ca dy v hn d1 , d 2 ,... l
K hiu

n 1, 2,...

S 2 l s nguyn t chn duy nht. V vy s pn vi n 1 l l v do dn pn1 pn chn.


Quan st bng trn mt cu hi t ra l vi s t nhin k no th tn ti n m d n 2k ? Ta cha
bit cu tr li. Di dy l bng cc s t nhin nh nht n m d n 2k vi 2k 30 v cc s
nguyn t pn, pn1 tha mn pn1 pn 2k (Lander v Parkin [3]).

CHNG 3. S NGUYN T | 75

Cc s nguyn t lin tip nh nht c hiu bng 100 l 396733 v 396833 . Bng cc s d n 1 vi

n 600 c trnh by bi P.Erdos, A.Renyi [1]

(1).

Bng d n vi n 1233 c trnh by bi

M.Colombo [1].
Bng cc s nh nht pn m pn1 pn 2k vi 2k 314 c trnh by bi Lander v Parkin [3]
v Brent [1] (xem thm Brent [4], Weintraub [1]).
Hn mt trm nm trc gi thuyt sau c t ra: vi mi s chn 2k tn ti v hn cc s t
nhin n tha mn dn 2k (de Plignac [1]).
Vi k 2 gi thuyt ny tng ng vi vic tn ti v hn cp s nguyn t sinh i, ngha l
cp cc s l lin tip m mi s u l s nguyn t. Mi cp s u tin nh vy l (3,5), (5,7),
(11,13), (17,19), (29,31), (41,43), (59,61), (71, 73), (101,103), (107,109). H.Tietze trnh by
bng cc cp s nguyn t sinh i nh hn 300000 . C 2994 cp nh vy (Tietze [1] v Frucht
[1], xem Selmer v Nesheim [1] trong cc s n tha mn 6n 1 v 6n 1 u l s nguyn t
nh hn 200000 . So snh vi Sexton [1] v [2].) Brent [3] tm ra c 152892 cp cc s nguyn
t sinh i nh hn 1011 . Cp s ln nht c bit l 260497545 26625 1 (Atkin v Rickert, xem
Yates [1]). Bi ton chng minh tn ti v hn cc cp s nguyn t sinh i tng ng vi vic
chng minh tn ti v hn cc s t nhin n m n2 1 c ng 4 c s t nhin.
T dy cc s t nhin lin tip 1, 2,..., n tm c s nguyn t p m p 2 cng l s nguyn
t th vi mi hp s k c b ra khi sng Eratosthenes ta cng b i s k 2 (Golomb [1]),
W.A.Golubew [2] t ra cu hi vi s t nhin n no th tn ti t nht mt cp s nguyn t
nm gia n 3 v n 1 . Ta bit chui tng cc cp nghch o ca cc cp s nguyn t sinh i
3

l hi t (Brun [1]) (1). Chui

1 1 1 1 1 1 1 1 1 1
...
3 5 5 7 11 13 17 19 29 31
c tnh chnh xc ti 6 ch s thp phn bi Brent [2]. Trong mc 17 ta s thy tng nghch
o ca tt c cc s nguyn t l phn k.
Mt cu hi khc cha c cu tr li l c tn ti v hn cc s nguyn t p m p, p 2, p 6
v p 8 u l s nguyn t hay khng? Mt b bn s nguyn t nh vy gi l mt b s
nguyn t sinh bn. Su b s nguyn t sinh bn u tin nhn c vi
p 5,11,101,191,821,1481. K.Fruchtl [1], [2], [3], [4] lit k tt c cc b nh vy nh hn
(1)

Di y l mt s li trong bng c trch dn (theo

J . Galgowski v L. Kacperek):

Thay v

d265 12 ta c d256 2, thay v d314 6 ta c d314 4, thay v d344 12 ta c d344 22,

Thay v

d429 18

Ta c
(1)

ta c

d429 28, thay v d465 4 ta c d465 6, thay v d462 18 ta c d462 28,

d579 2.

Chng minh s cp ca nh l Brun c trong cun sch ca E. Landau [2], tp 1

76 | Hiu ca cc s nguyn t lin tip. Gi thuyt Goldbach

15000000. C tt c 1209 b s nh vy. Gn y J.Bohuman [2] ch ra c ng 49262 b bn


nh hn 2.109 . Mt b bn s nguyn t nh vy m s nh nht ln hn 5 s c cc ch s tn
cng ln lt l 1, 3, 7 v 9.
R rng mi b bn nh vy cho ta hai cp s nguyn t sinh i. Tuy nhin c cc cp s nguyn
t sinh i m gia chng khng c s nguyn t no v khng to thnh b bn, V d nh cp
(179,181) v (191,193). Cp th hai cng vi (197,199) to thnh mt b bn nguyn t. Gia hai
cp (419,421) v (431,433) khng c s nguyn t no.
Cc cp 809,811,821,823 v 1019 1021; 1031, 1033 cng nh vy. Mt cu hi c t ra l c
tn ti s lng ty cc cp s nguyn t sinh i lin tip m gia chng khng c s nguyn t
no hay khng. Ta bit c cc b ba nh vy l
179,181,191,193,197,199
809,811,821,823,827,829 3359,3361,3371,3373,3389,3391 ; 4217, 4219, 4229, 4231, 4241, 4243 ;
6761, 6763, 6779, 6781, 6791, 6793.

Mt b bn nh vy l 9419,9421,9431,9433, 9437,9439,9461,9463.
C th chng minh rng nu p 5 v cc s p, p 2, p 6 v p 8 l s nguyn t th khi
chia p cho 210 ta nhn c s d l 11,101 hoc 191.
C th chng minh cc s d n c th ln ty . Tht vy k hiu m l s t nhin ln ty . K hiu

pn l s nguyn t ln nht m! 1. S
k 2,3,..., m ). V vy

m! k l hp s vi k 2,3,..., m (v k | m! k vi

pn1 m! m 1 v h qu l dn pn1 pn m .

Vy ta c th chng minh d n n 1, 2,... tin ti v hn.


C cc s t nhin n m dn dn1 . Chng hn n 2,15,36,39, 46 .
Tn ti cc s t nhin n m dn dn1 dn2 chng hn vi n 54, 464, 682, 709,821,829 .
Tuy nhin ta khng bit vi s t nhin k no th tn ti s t nhin n tha mn
dn dn1 dn2 ... dnk (Lander v Parkin [4] v Bohman [2]). P.Erdos v P.Turan [2] chng
minh rng tn ti v hn cc s t nhin n m dn dn1 . Vi mi s t nhin m v k th tn ti s
t nhin n m cc s dn , dn1 ,..., dnk u ln hn m . Ni cch khc tn ti v hn cc cp s nguyn
t sinh i m hiu ca chng ln ty (Erdos [7]).
Cc hiu ca cc s nguyn t sinh i c nghin cu rt rng bi G.Ricci (Ricci [1],[2])
4. Gi thuyt Goldbach
Gi thuyt Goldbach ni rng mi s chn ln hn 2 u l tng ca hai s nguyn t. Gi thuyt
ny c kim tra vi cc s chn nh hn 108 (Light, Forrest, Hammond, Roe [1]). Nm 1973
Chen [2] chng minh mi s chn ln u l tng ca mt s nguyn t v mt s t nhin
c nhiu nht hai c s nguyn t. Kt qu u tin thuc dng ny c tm ra bi Brun [2] vo
nm 1920.
Gi thuyt Golbach ko theo mi s l c th biu din v hn cch di dng p q r vi p, q, r
l cc s nguyn t. Kt qu khng n gin ny c tm ra bi J.G.Van der Corput [2]. ng ta
cng chng minh rng hu ht cc s chn l tng ca hai s nguyn t l. Ngha l vi mi s
dng th vi s t nhin ln N ta c s cc s chn N m khng phi tng ca hai s
nguyn t s nh hn N (Van der Corput [1]).
A.Desboves [1] ch ra mi s t nhin 10000 c dng 4k 2 u l tng ca hai s nguyn
t, mi s u c dng 4k 1 . iu ny ch ng nu ta coi 1 cng l s nguyn t. Khi
2 1 1, 6 1 5,14 1 13,38 1 37, 62 1 61 .

CHNG 3. S NGUYN T | 77

Mt bi ton kh gn vi gi thuyt Golbach l c phi vi s t nhin cho trc n th s G n tt


c cc php phn tch n thnh tng ca hai s nguyn t s tin ti v cng khi n tng. N.Pipping
[1], [2] tnh hm G n vi mi s chn n nh hn 5000 v mt s s khc. Cc tnh ton cho
hm G n vi n 2000000 c thc hin bi M.L.Stein v P.R.Stein (Stein v Stein [1]).
Ta c G(4) G(6) 1 , G(8) 2 , G(10) 3 , G(12) 2 , G(14) 3 , G(16) G(18) G(20) 4 ,
G(22) 5 , G(24) 6 . Hn na G(158) 9 v G(2n) 10 vi 2n 158 . Tng t G(188) 10 v
G(2n) 10 vi 2n 188 . S chn nh nht 2n m G(2n) 100 l 840 . Tht ra ta c

G 840 102. S 2n ln nht m G(2n) 100 c th l s 2n 4574 .


T gi thuyt Golbach suy ra mi s l ln hn 7 u l tng ca ba s l. Tht vy, nu n l s l
>7 th n 3 l s chn > 4 v theo gi thuyt Goldbach th n l tng ca hai s nguyn t l. V vy
mi s l ln hn 7 u l tng ca ba s nguyn t l. Nhn xt ny cng cha c chng minh
mt cch cht ch. Tuy nhin cc kh khn ch l vn k thut v vo nm 1937 I.Vinogradov
chng minh cc s l ln hn mt hng s xc nh c a th u c tnh cht nh vy. Sau
K.G. Borozdkin [1] chng minh rng a exp exp16, 038 33 . Vy ta ch cn kim tra li vi
15

cc s 7 n a, tuy nhin cc php tnh ton ln nh vy l cha thc hin c.


Tnh hung li rt khc i vi cu hi rng c phi mi s chn u l hiu ca hai s nguyn t?
Hin ti ta cha c phng php no tip cn bi ton ny. A.Schinzel [11] chng minh rng
gi thuyt Goldbach suy ra mi s l >17 u l tng ca ba s nguyn t l phn bit. T kt qu
trn ca Vinogradov suy ra tnh cht ny l ng vi cc s ln. Gi thuyt ni rng mi s chn
>6 u l tng ca hai s nguyn t phn bit tng ng vi gi thuyt ni rng mi s t nhin
>17 u l tng ca ba s nguyn t phn bit (Sierpinski [23]).
Nm 1930 L.Schnireiman [1] a ra mt chng minh s cp rng tn ti cc s s m mi s t
nhin >1 u l tng ca nhiu nht s s nguyn t. Riesel v Vaughan [1] s dng li phng
php ca Schnireman chng minh rng mi s chn >1 u l tng ca nhiu nht 19 s nguyn
t. T kt qu nu trn ca Vinogradov ta thy mi s t nhin ln u c th biu din nh l
tng ca nhiu nht 4 s nguyn t, cc trng hp khc c th kim tra bng my tnh nhng n
yu cu nhiu thi gian. C th d dng chng minh tn ti v hn cc s t nhin khng th biu
din nh l tng ca t hn 3 s nguyn t (so snh vi bi tp 2 di). Mt gi thuyt khc c
t ra l c phi mi s l >5 u l tng ca mt s nguyn t v mt s c dng 2 p, vi p
nguyn t (Dickson [7] trang 424)? Mayah [1] kim tra gi thuyt ny vi n 42.105.
Bi tp. 1. Chng minh rng mi s t nhin >11 u l tng ca hai hp s.
Chng minh. Xt s t nhin n >11. Nu n chn tc l n 2k , th k 6 v n 6 2 k 3 , suy ra

n 6 l hp s. Nu n l ngha l n 2k 1, th k 6 v n 9 2 k 4 l hp s.
2. Chng minh rng tn ti v hn s t nhin l khng biu din c thnh tng ca t hn 3 s
nguyn t.
Chng minh. Cc s 14k 3 , vi k 1, 2,... c tnh cht nh vy. Tht vy cc s ny u khng
2

nguyn t. Hn na chng khng th biu din thnh tng ca hai s nguyn t v nu ngc li
2
th do chng l l nn mt trong hai hng t nguyn t phi bng 2 v ta c 14k 3 2 p, suy

ra p 7 28k 2 12k 1 , nhng y khng phi s nguyn t. Mu thun.


Ghi ch. C th chng minh mt cch s cp rng tn ti v hn cc s l l tng ca ba s nguyn
t phn bit nhng khng l tng ca t hn ba s nguyn t (Sierpinski [31]).
3. Chng minh rng gi thuyt Golbach tng ng vi gi thuyt ni rng mi s chn >4 l
tng ca ba s nguyn t.

78 | Gi thuyt Goldbach. Cc s nguyn t lp thnh cp s cng

Chng minh. T gi thuyt Goldbach suy ra vi s t nhin n 1 ta c 2n p q, vi p v q l


cc s nguyn t. V vy 2 n 1 2 p q, ngha l mi s chn c chn ln >4 u c
biu din thnh tng ca ba s nguyn t. Mt khc nu mi s chn >4 u l tng ca ba s
nguyn t ngha l nu vi n 2 ta c 2n p q r , vi p, q, r l cc s nguyn t th t nht mt
trong cc s p, q, r l chn v do bng 2. Gi s r 2. Khi 2 n 1 p q vi n 1 1, suy
ra gi thuyt Goldbach.
4. Chng minh rng cc phng trnh x2 y 2 z 2 , x2 y 2 z 2 t 2 , x 2 y 2 z 2 t 2 u 2 u
khng c nghim l cc s nguyn t.
Chng minh. chng minh phng trnh th nht khng c nghim nguyn t ta nh li trong
chng 2 mc 3 ta chng minh mi nghim t nhin ca phng trnh ny c t nht mt s
chia ht cho 4. By gi xt phng trnh th hai. Gi s tn ti cc s nguyn t x, y, z, t tha mn

x2 y 2 z 2 t 2 . Nh chng minh trong Chng 2 mc 10 th t nht hai trong cc s x, y, z l


chn, m v chng l nguyn t nn chng u bng 2. V vy t 2 z 2 8. Nhng do z , t l nguyn
t v t nht c 1 s l nn t ng thc t z t z 8 suy ra t z 2 v h qu l t z 4 , v
l v trong hai s t , z c mt s nguyn t l. Cui cng xt phng trnh th ba. Gi s tn ti cc
s nguyn t x, y, z, t , u tha mn x 2 y 2 z 2 t 2 u 2 . R rng u > 2 v do l l. V vy t nht
mt trong cc s x, y, z, t l. Nu ch c ng mt s trong chng l l, gi s l t , th ta c

x y z 2, suy ra 12 t 2 u 2 v t t u t a 12, suy ra t u 2, t u 6. Nhng iu


ny khng th xy ra v u, t l cc s nguyn t l phn bit. Ngc li nu ba trong cc s nguyn
t x, y, z, t l l v s cn li chn th u 2 x2 y 2 z 2 t 2 u c dng 4k 3 , v l.
5. Tm cc nghim nguyn t ca phng trnh x 2 y 2 z 2 t 2 v 2 vi x y z t u v.
Li gii. C duy nht mt nghim l 22 22 22 22 32 52 , v d dng chng minh ch c ng
mt trong cc s x, y, z, t , u, l l v do 4 22 u 2 v 2 , suy ra v u v u 16, v u 8, v
vy u 3, v 5.
5. Cc s nguyn t lp thnh cp s cng
Mt cp s cng gm 18 phn t u l s nguyn t l 4808316343 71777060k , k 0,1, 2,...,17.
P.A. Pritchard [1] tm ra cc s 4180566390k 8297644387 k 0,1, 2,..,18 lp thnh cp s
cng gm 19 s nguyn t phn bit. Ta cha bit c tn ti cp s cng c 100 s nguyn t hay
khng. Ta s chng minh nu cp s cng nh th tn ti th cng sai ca cp s ny s phi l mt
s c nhiu hn 30 ch s thp phn.
nh l 5. Nu n v r l cc s t nhin, n 1 v nu n phn t ca cp s cng
m, m r,..., m n 1 r u l s nguyn t l th cng sai r chia ht cho mi s nguyn t nh hn

n (Dickson [7] tp 1 trang 425).


Chng minh. Gi s m, n 1 v r l cc s t nhin cho trc v cc s m, m r ,..., m n 1 r
u l s nguyn t l. Ta phi c m n, v nu ngc li th hp s m mr m 1 r s l phn
t ca dy s trn. K hiu p l s nguyn t nh hn n v t r0 , r1 ,..., rp 1 l phn d nhn c
khi ln lt chia cc s m, m r ,..., m p 1 r cho p . Cc s d ny l nh hn p v hn na
chng u khc 0 v nu ngc li th mt trong cc s nguyn t s khng nh hn m n p v
chia ht cho p, mu thun. V vy cc s d trn ch c th nhn cc gi tr 1, 2,..., p 1, ngha l c

p 1 kh nng. T y suy ra tn ti hai s nguyn k v l tha mn 0 k l p 1 m rk rl .

CHNG 3. S NGUYN T | 79

H qu l p | m lr m kr v v vy p | l k r. Nhng 0 l k p 1 p, do p | r. V p
l s nguyn t ty nh hn n, ta c iu phi chng minh.
T nh l 5 ta c h qu sau y
H qu. Nu tn ti cp s cng tng cha n 2 s nguyn t th cng sai ca dy ny s chia ht
cho tch Pn tt c cc s nguyn t nh hn n, v v vy bn thn cng sai Pn .
c bit, cng sai ca cp s cng gm ba s nguyn t phn bit phi P3 2. Tn ti duy nht
mt cp s cng cng sai 2 l 3,5,7. Tn ti v hn cp s cng gm ba s nguyn t. Chng minh
kt qu ny khng n gin (xem Van der Corput [2] v Chowla [2]).
Bi ton chng minh tn ti v hn cp s cng nh vy l tng ng vi cu hi khi no th
phng trnh p r 2q c v hn nghim nguyn t p, q, r, vi p r. T gi thuyt H (mc 8)
suy ra vi mi s t nhin n v s nguyn t p n th tn ti v hn cp s cng tng c n phn
t l s nguyn t m phn t th nht l p.
Ta lit k y mt s cp s cng gm ba s nguyn t m s u tin l 3: 3, 7, 11; 3,11,19; 3, 13,
23; 3, 31, 59; 3, 37, 71; 3, 23, 43; 3, 31, 59; 3, 37, 71; 3, 41, 79; 3, 43, 83. Cng sai ca cp s cng
gm 4 s nguyn t phi P4 6. C rt nhiu cp s cng 4 s nguyn t cng sai bng 6 chng
hn: 5, 11, 17, 23; 11, 17, 23, 29; 41, 47, 53, 59; 61, 67, 73, 79. T gi thuyt H suy ra tn ti v hn
cp s cng nh vy, hn na chng cn l cc s nguyn t lin tip. c bit ta c dy 251, 257,
263, 269; 1741, 1747, 1753, 1759. Cng sai ca cp s cng gm 5 s nguyn t phi ln hn hoc
bng 6. Tn ti duy nht mt cp s cng gm 5 s nguyn t m cng sai bng 6 l 5, 11, 17, 23,
29. Ta lu rng trong mt cp s cng nh vy th phi c mt s chia ht cho 5. Tng t ta d
dng chng minh rng tn ti ng mt cp s cng gm 5 s nguyn t m cng sai bng 12 l 5,
17, 29, 41, 49. Khng c cp s cng no nh vy c cng sai l 18 hoc 24. Tuy nhin t gi thuyt
H suy ra tn ti v hn cp s cng gm 6 s nguyn t m cng sai bng 30. Chng hn 7, 37, 67,
97, 127, 157; 541, 571, 601, 631, 661, 691. T h qu trn suy ra mi cp s cng gm 7 s
nguyn t th cng sai ca dy chia ht cho 30. D dng chng minh rng khng c cp s cng no
nh vy m cng sai nh hn 150. C ng mt cp s cng gm 7 s nguyn t cng sai 150 l 7,
157, 307, 457, 607, 757, 907. L do l v trong by s nguyn t ny s c mt s chia ht cho 7. T
h qu cng suy ra mt cp s cng 10 phn t nguyn t th c cng sai P10 210. Cp s cng
nh vy c cng sai 210 l 199 210k , vi k 0,1, 2,...,9. T gi thuyt H suy ra c v hn cp s
cng nh vy. T h qu suy ra cng sai ca cp s cng gm 100 s nguyn t phi chia ht cho
tch ca mi s nguyn t nh hn 100, v do cng sai ny c hn 30 ch s trong biu din
thp phn. Ta cha tm c mt cp s cng no nh vy. Ta cng cha bit c tn ti cp s
cng nh vy hay khng (Grosswald v Hagis [1]).
6. Cc s nguyn t trong mt cp s cng cho trc
Khc vi mc 5, trong mc ny ta xem xt vn vi cc s t nhin a v b no th cp s cng
ak b 1, 2,..., cha v hn s nguyn t? R rng nu a, b d 1, th khng c s nguyn t
no trong cp s cng ak b 1, 2,.., bi v vi mi k , ak b d ka d b d u l hp s (
a d , b d l cc s t nhin).

V vy iu kin cn tn ti v hn s nguyn t trong cp s cng ak b l a, b 1.


Nm 1837 Lejeune Dirichlet chng minh iu kin trn cng l iu kin . Li gii n gin
nht ca nh l ny (vn rt phc tp) c trnh by trong Chng 8 cun sch ca E.Trost [3].
Ta s chng minh nh l ny trong mt vi trng hp ring. Trong Chng 5 l vi a 4, b 1,3
(nh l 7 v 7a), trong Chng 6 vi b 1, a ty (nh l 11a), trong Chng 9 vi
a 8, b 3,5,7 (nh l 1,2,3) v vi a 5, b 4 (nh l 4).
Hai nh l sau l tng ng

80 | Cc s nguyn t trong mt cp s cng cho trc. Tam thc Euler x x 41 . Gi thuyt H


2

T. Nu a , b v k l cc s t nhin, a, b 1, th tn ti v hn s nguyn t c dng ak b .


T1. Nu a v b l cc s t nhin tha mn a, b 1, th tn ti t nht mt s nguyn t p c dng

ak b vi k t nhin (2).
Chng minh. R rng T suy ra T1. Ta ch cn chng minh chiu ngc li, ngha l T1 suy ra T. Ta
gi s a 1 v vi a 1 th T hin nhin ng. Gi s a, b l cc s t nhin cho trc tha mn

a, b 1.

Khi r rng a m , b 1. V vy theo T1 suy ra tn ti s nguyn t p tha mn

p a m k b, vi s t nhin k . Nhng do a 1, a m 2m m. Suy ra p m. V vy ta chng


minh vi mi s t nhin m th u tn ti s nguyn t c dng ak b v ln hn m. T y suy
ra tn ti v hn s nguyn c dng nh vy.
Trong Chng 5 nh l 9 ta s chng minh vi mi s nguyn t c dng 4t 1 u l tng ca
hai bnh phng hon ho. S dng kt qu ny ta chng minh h qu sau y ca nh l T
H qu. Vi mi s t nhin n u tn ti s nguyn t p tha mn p a 2 b2 , vi a, b l cc s
t nhin > n.
Chng minh. Gi s n l s t nhin. Theo T th tn ti s nguyn t q n c dng 4t 1. Suy ra

4 1 q 2 q ... n q , q 1. V vy theo T suy ra tn ti s t nhin k tha mn s


2

p 4 12 q 22 q ... n2 q k q l s nguyn t v c dng 4t 1. V vy tn ti a, b tha


2

mn p a 2 b2 , vi a b, gi s a n. th

b 2 p a 2 4 12 q 22 q ... n 2 q k a 2 q
2

2
2
2
2
2
a 2 q 4 12 q ... a 1 q a 1 q ... n 2 q k 1 ,

Vi cc nhn t v phi u l nguyn t cng nhau. H qu l chng u l cc bnh phng


ng. Nhng iu ny l khng th v nhn t th hai c dng 4t 1. V vy b a n, suy ra h
qu c chng minh.

Ta lu rng theo nh l ca E.Hecke [1] th vi mi s thc c d 0 u tn ti s nguyn t p


a
tha mn p a 2 b2 vi a, b l cc s t nhin v c d (Maknis [1]).
b
7. Tam thc Euler x 2 x 41
m
m1
D dng chng minh rng khng tn ti a thc f x a0 x a1 x ... am1 x am vi h s

nguyn v a0 m 0 m cc s f x l nguyn t vi mi gi tr nguyn ca x. Tht vy vi gi tr


ln ca x, gi s x x0 , hm f x tng. Vi gi tr x1 x0 , f x1 p l s nguyn t th

p | f x1 p , m f x1 p f x1 p, suy ra f x1 p l hp s. Ta cng chng minh c


khng c hm hu t no nhn mi gi tr nguyn t vi i s nguyn ngoi tr hm hng (Buck
[1]). Tuy nhin tn ti a thc bc hai vi h s nguyn nhn gi tr nguyn t vi dy rt di cc
2
s t nhin lin tip. V d tam thc Euler f x x x 41, nhn mi gi tr nguyn t vi

x 0,1,...,39. rng f x 1 f x 2 x 1 . T y suy ra vi x 0,1, 2,... th f ( x) nhn


cc gi tr ring ca dy 41 2.1 2.2 2.3 ... v vy ta c 41, 43, 47, 53, 61, 71, 83,, 1601. C th
kim tra trong bng cc s nguyn t thy tt c cc s ny u nguyn t.
(2)

Chng minh s tng ng ca T v T1 c ti (Sierpinski) a ra vo nm 1950 (Sierpinski [12] trang 526). Su


nm sau bi ton v s tng ng ny c trnh by trong The Amer. Math. Monthly trong s E1218 (1956) trang
342 v c chng minh bi D.Zeitlin (1957, trang 46). Xem thm V.S.Hanly [1].

CHNG 3. S NGUYN T | 81

Do f x f x 1 , suy ra f x nguyn t vi x 1, 2,..., 40.


Vy vi x 40, 39,..., 1,0,1,...,39 th f x nhn gi tr nguyn t (khng nht thit phn bit).
Hm f x cn c mt tnh cht th v khc: vi mi s nguyn x th khng c s d no m

1 d 41 l c ca f x . Tht vy, gi s vi s nguyn x ta c d | f x , vi 1 d 41. K


hiu r l s d nhn c khi chia x cho d . Th th x kd r , vi k l s nguyn v 0 r d .
Nhng f kd r kd kd 2r 1 f r , do d | f x suy ra d | f r ; tuy nhin t y ta c
mu thun. Tht vy, bi v 0 r d 41, ta phi c 0 r 39; suy ra nh ta bit f r l s
nguyn t 41, v n khng c c s d tha mn 1 d 41. V vy vi mi s nguyn x th

f x khng c c s d tha mn 1 d 41.


Tnh cht ny c lin quan ti vic tm xem vi s t nhin x 40 no th f x l s nguyn t.
Vi x 40 ta c f 40 40 41 41 412 , do f x l hp s. S f 41 41 42 41 41 43
cng l hp s. Nu x 41 v nu s

f x l hp s th v

x 1

x 2 2 x 1v

x 2 x 41 f x , suy ra f x x 1 . Do s f x c c s p x 1 v do ta chng
2

minh trn 41 p x (khi chia

f x cho x ta nhn c s d l 41). V vy

f 42 42 43 41 nguyn t.
E.Trost trong [3] trang 41 ch ra vi x khng vt qu 11000 th hm f x nhn 4506 gi tr
nguyn t khc nhau. Ta cha bit dy f x x 1, 2,... c cha v hn s nguyn t hay khng.
Kt qu ca cu hi ny c suy ra t gi thuyt H trong mc 8.
T cc tnh cht ca f x suy ra tam thc g x f x 40 x 2 79x 1601 nhn cc gi tr
nguyn t (khng nht thit phn bit) vi x 0,1, 2,..., 79. Ta c g t g 79 t vi mi t.
T cc kt qu ca G.Frobenius [1] v H.M.Stark [1] suy ra khng tn ti s A ln hn 41 m tam
thc x 2 x A nhn cc gi tr nguyn t vi x 0,1, 2,..., A 2. Vi x 0,1,..., 28 th cc gi tr

6 x2 6 x 31 l cc s nguyn t phn bit c dng 6k 1; chng nm gia 31 v 4909 vi cc gii


hn km theo (C.Coxe, Van der Pol v Speziali [1]).
Cc gi tr ca 2 x 2 29 u l nguyn t vi 28 x 28.
D dng chng minh rng tn ti a thc bc n nhn cc gi tr nguyn t vi x 0,1,..., n; tuy
nhin ta cha bit a thc bc 2 hoc cao hn vi bin s x no nhn v hn gi tr nguyn t vi
cc gi tr ca x. c bit ta cng khng bit a thc x 2 1 c tnh cht hay khng.
W.A.Golubew [5] trnh by mt danh sch cc s t nhin x 120000 m x 2 1 nguyn t.
M.Wunderlich [2] tm ra c ng 624535 s x 14 106 c tnh cht nh vy. H.Iwaniec [1]
chng minh rng tn ti v hn s x 2 1 l tch ca nhiu nht hai s nguyn t v B.M.Bredihin
[1] chng minh rng tn ti v hn s nguyn t c dng x 2 y 2 1.
Nu a thc f x vi h s nguyn nhn cc gi tr nguyn t vi v hn cc gi tr ca x th h
s a0 ca ly tha bc cao nht ca x phi l dng v vi gi tr ln ca x th a thc c cng

du vi a0 . Hn na a thc f x khng th l tch ca hai a thc vi h s nguyn v nu


ngc li th vi gi tr ln ca x th s f x s l hp s. V vy a thc f x l bt kh quy.

82 | Cc s nguyn t trong mt cp s cng cho trc. Tam thc Euler x x 41 . Gi thuyt H


2

Tuy nhin cc iu kin ny khng suy ra f x nhn cc gi tr nguyn t thm ch l vi t


nht mt gi tr ca x. Tht vy, a thc x 2 x 4 bt kh quy v khng c nghim thc, nhng
vi mi gi tr nguyn ca x th cc s x 2 x 4 u l hp s bi v chng u l cc s chn ln
hn 3 v x 2 x x 1 x , tch ny lun chn v khng m.
Nm 1857 W.Bouniakowsky [2] t ra gi thuyt sau: nu f x l a thc bt kh quy h s
nguyn v N k hiu c chung ln nht ca cc s f x , x nhn mi gi tr nguyn th a thc

f x N nhn gi tr nguyn t vi v hn gi tr ca x. (Dickson [7] tp 1 trang 333).


Xt a thc f x x 2 x 4. V f 0 4, f 1 6 v f x l s chn suy ra c chung ln nht
ca f x khi x nhn mi gi tr nguyn l 2. Do gi thuyt trn ni rng c v hn s nguyn
t c dng x x 1 2 2 .
8. Gi thuyt H
K hiu s l s t nhin v f1 ( x), f 2 ( x),..., f s ( x) l cc a thc h s nguyn. Gi s tn ti v hn
s t nhin x m cc s f1 ( x), f 2 ( x),..., f s ( x) u l s nguyn t. Nh ta bit trong mc 7 th
cc a thc fi ( x), i 1, 2,..., s l bt kh quy v cc h s cao nht u dng. Do vi gi tr
ln ca x th tt c cc s f1 x , i 1, 2,..., s, l ln ty . C th kim tra rng tnh cht ny suy ra

khng tn ti s t nhin d 1 l c ca P x f1 x f 2 x ... f s x vi mi s t nhin x . Tht


vy nu tn ti c s nh th th n s l c s ca tch s s nguyn t ln ty , iu ny l
khng th. V vy ta chng minh rng nu s l s t nhin v f1 x , f 2 x ,..., f s x l cc a
thc h s nguyn v c v hn cc gi tr t nhin ca x m cc s f1 x , f 2 x ,..., f s x u
nguyn t th cc a thc ny phi tha mn
iu kin C. Cc a thc fi x ,(i 1, 2,..., s ,) u bt kh quy vi h s cao nht dng v khng
tn ti s t nhin d 1 l c ca P x f1 x f 2 x ... f s x , x nguyn ty .
Nm 1958 A.Schinzel t ra gi thuyt sau
Gi thuyt H. Nu s l s t nhin v f1 x , f 2 x ,..., f s x l cc a thc h s nguyn tha mn
iu kin C th tn ti v hn s t nhin x m cc s f1 x , f 2 x ,..., f s x u nguyn t
(Schinzel et Sierpinski [3], trang 188).
Trng hp ring khi cc a thc f1 l tuyn tnh ta thu li gi thuyt t ra sm hn bi
L.E.Dickson [1]. Ta trnh by y mt s h qu ca gi thuyt H.
Vi s t nhin cho trc n v f1 x x 2 1, f 2 x x 2 3, f3 x x 2 7, f 4 x x 2 9.
n

Vi P x f1 x f 2 x f3 x f4 x ta c P 0 1 3 7 9 v P 1 2 4 8 10. H qu l

P 0 , P 1 1. Suy ra iu kin C c tha mn v gi thuyt H suy ra: vi mi s t nhin n

tn ti v hn s t nhin x m cc s x 2 1, x 2 3, x 2 7, x 2 9 u l nguyn t (Sierpinski


[34]). T y suy ra tn ti v hn b cc s nguyn t sinh bn (xem mc 3) v tn ti v hn cc
s nguyn t c dng x 2 1 hoc dng x 4 1.
n

W.A.Golubew [6] tnh ton v ch ra ch c 5 s t nhin x nh hn 10.000 m cc s


x2 1, x2 3, x3 7, x 2 9 u l nguyn t. l x 2,10,1420, 2080, 2600.

CHNG 3. S NGUYN T | 83

K hiu k l s nguyn ty v f1 x x, f 2 x x 2k. Vi P x f1 x f 2 x ta c

P 1 2k 1, P 2 4 k 1 . R rng 2k 1, 4 k 1 1, suy ra cc a thc ny tha mn iu


kin C. T gi thuyt H suy ra tn ti v hn cc s t nhin x m cc s p x v q x 2k u
l s nguyn t. V vy 2k p q, suy ra 2k c th biu din v hn cch nh l hiu ca hai s
nguyn t. Ngha l t gi thuyt H c th suy ra mi s chn u c th biu din v hn cch nh
l hiu ca hai s nguyn t. T gi thuyt H cng c th suy ra mi s chn c th biu din di
dng hiu ca hai s nguyn t lin tip (Schinzel v Sierpinski [3] trang 190).
T gi thuyt H suy ra nu a v b l cc s t nhin tha mn a, b a, b b 2 1, th tn ti
v hn s nguyn t p c dng ak b, vi k l s t nhin tha mn p 2 cng l s nguyn t.
Tht vy, xt cc a thc

f1 x ax b, f 2 x ax b 2. Vi P x f1 x f 2 x ta c

P 0 b b 2 , P 1 a b a b 2 v P 1 P 1 2a 2 2b b 2 . Nu tn ti s nguyn
t q tha mn q | P x vi mi s nguyn x, th nu b l ta c P(0) l ko theo q l v nu b
chn th v a, b 1, a l; v vy c a b v a b 2 u l v do P 1 l suy ra q l. Do
trong mi trng hp q u l. Vy ta c th gi s q | P 0 , ngha l q | b b 2 v

q | P 1 P 1 , ta c q | 2a 2 do v q l nn q | a. Nhng iu ny v l v a, b b 2 1. V
vy iu kin C c tha mn. Do t gi thuyt H suy ra tn ti v hn s t nhin x m cc s
f1 x ax b v f 2 x ax b 2 u nguyn t. H qu c chng minh.
D dng nhn thy iu kin a, b b 2 1 l cn thit i vi s tn ti v hn cc s p c
dng ak b m p 2 cng l s nguyn t.
Xt s nguyn ty k v t f1 x x, f 2 x 2k 1 2 x. Vi P x f1 x f 2 x ta c

P 1 2k 3, P 1 2k 1 . Do 2k 1, 2k 3 1 vi mi s nguyn k , ta nhn thy cc a


thc ny tha mn iu kin C. Khi theo gi thuyt H th tn ti v hn cc s t nhin x m cc
s q x v p 2k 1 2 x u nguyn t. V vy 2k 1 p 2q. Do t gi thuyt H suy ra mi
s l ( 0 hoc 0 ) u c th biu din v hn cch nh l hiu ca mt s nguyn t v bi 2
ca mt s nguyn t khc.

G.de Rocquigny [1] t ra cu hi c phi mi s nguyn chia ht cho 6 u l hiu ca hai s


nguyn t c dng 6k 1. Cu tr li khng nh l h qu ca gi thuyt H. Tht vy vi
f1 x 6 x 1 v f 2 x 6 x 6k 1, P x f1 x f 2 x ta c P 0 6k 1, P k 6k 1
v 6k 1,6k 1 1 vi mi s nguyn k .
T gi thuyt H suy ra tn ti cp s cng di v hn m cc phn t l cc s nguyn t lin tip
(Schinzel v Sierpinski [13]). C rt nhiu h qu khc c suy ra t gi thuyt H, v d gi thuyt
Bouniakowsky (xem Schinzel v Sierpinski [3] v Schinzel [13]).
Bi tp. Chng minh rng t gi thuyt H suy ra: vi hai s nguyn nguyn t cng nhau a v b
tha mn mt trong hai s l chn v a 0. Khi tn ti v hn s nguyn t p tha mn ap b
u l s nguyn t.
Chng minh. t

f1 x ax b, f 2 x x. Vi P x f1 x f 2 x ta c P 1 a b,

P 1 a b, v do mt trong hai s a, b l chn, s cn li l l v a, b 1. V t a, b 1 suy

ra a b, a b 1. Do P 1 , P 1 1 v iu kin C c tha mn. Khi t gi thuyt H


suy ra tn ti v hn x m f 2 x x v f1 x ax b u l cc s nguyn t. Ta c iu phi
chng minh.

84 | Hm s

9. Hm s x
Vi mi s thc x k hiu x l s cc s nguyn t khng vt qu x.
Ta c

1 0, 2 1, 3 4 2, 5 6 3, 7 8 9 10 4,

100 25, 1000 168, 10000 1229, 105 9592, 106 78498, 107 664579,

108 5761455, 109 50847534.


ca Lebmer [8] vo nm 1958), 10 4118054813, 10 37607912018. Gn y
J.C.Lagarias, V.S.Miller v A.M.Odlyzko [1] tnh c 10 346065536839 (y l kt qu
chnh sa t kt qu ca Bohman [1]), 10 3204941750802, 10 298445704422669 v
10 279238341033925. Hin nhin ta c p n vi n 1, 2,...
Nm 1972 J.Bohman [1] tnh c 109 455052511 (y l kt qu chnh sa t kt qu
11

12

13

14

15

16

P.Erdos (Trost [3] trang 52-53) tm ra li gii n gin ca bt ng thc


(2)

log n
vi n 1, 2,...
2log 2

Nh ta chng minh trong Chng 1 mc 14, mi s t nhin u c th biu din duy nht
di dng k 2l , vi k v l l cc s t nhin v hn na l l khng c c chnh phng. Vi mi
s trong cc s 1, 2,..., n, ta c biu din k 2l n; do k 2 n. V vy k n . Suy ra k c th nhn
nhiu nht n gi tr khc nhau. S l khng c c s chnh phng v nh hn n nn c th
biu din nh l tch ca mt s s nguyn t thuc dy p1 , p2 ,..., p n . S lng cc s nh vy
n
(tnh c 1) l 2 . H qu l s l c th nhn nhiu nht 2 n gi tr khc nhau. Do s lng
n
cc tch lk 2 phn bit v khng ln hn n, ti a l n 2 . V mi s t nhin n u c biu
n

din di dng suy ra n n 2


1
log n n log 2, t y suy ra (2).
2

. Vy

n 2 n v ly logarithm c hai v ta c

Sau y trong mc 14 ta s chng minh cc bt ng thc cht hn cho n . Tuy nhin iu th


v ca bt ng thc (2) nm tnh n gin trong cch chng minh ca n.
K hiu k l s t nhin ty v n pk . Theo cng thc (2) v pk k , ta c

k log pk 2log 2. Do pk 22 k vi k 1, 2,..., hn na 22 k l hp s vi mi k 1, 2,..., suy ra


(3)

pk 22 k vi k 1, 2,...

Bi tp. 1. Chng minh rng vi mi s t nhin n 1 bt ng thc


(4)

n 1
n 1

n
n

ng nu v ch nu n l s nguyn t. Vi mi hp s n ta c
(5)

n 1
n 1

n
n

CHNG 3. S NGUYN T | 85

Chng minh. Nu n l hp s th n n 1 v (5) ng. Nu n l s nguyn t th

n n 1 1,
(6)

n n 1
n

1 n 1
1
.
n 1
n
n 1

Nhng k k vi k 1, 2,... nn t (6) suy ra (4).


2. Cho trc s t nhin m, tm tt c cc nghim t nhin ca phng trnh n m .
Li gii. Cc nghim t nhin n tha mn pm n pm1. Vy c ng pm1 pm nghim cn tm.
10. Chng minh nh Bertrand (nh l Tchebycheff)
Vi s thc cho trc x k hiu x l s nguyn ln nht x.

3
3
Ta c 0, 1, 2 1, 3.

4
4
T nh ngha suy ra vi mi s thc x ta c x 1 x x. ng thc x x xy ra nu v ch
nu x l s nguyn. Nu k l s nguyn th vi cc s thc x ta c x k x k. Vi mi s

1 2 1 2
1 1 1 1
thc x, y ta c x y x y . V d 0 1 v 0.
2 3 2 3
3 2 3 2
nh l 6. Ly tha ca s nguyn t p trong phn tch thnh tha s nguyn t ca n ! l
(7)

n n n
a 2 3 ...
p p p

Chng minh. Gi s n, k l hai s t nhin cho trc v s nguyn t p n. Cc s trong dy

1, 2,..., n chia ht cho p k u c dng lp k , vi l l s t nhin tha mn lp k n, ngha l

l n p k . S cc s l nh vy l n p k . Mt khc r rng s m a ca s nguyn t p trong


phn tch thnh tha s nguyn t ca n ! chnh l tng ca s cc s trong dy 1, 2,..., n m chia
ht cho p vi s cc s chia ht cho p 2 v s cc s chia ht cho p 3 v c nh th. T y suy ra
(7).
ng dng n gin nht ca nh l 6 l tnh s ch s 0 tn cng ca 100!. Theo cng thc (7)
(vi n 100 v p 2 ) th s m ca 2 trong phn tch thnh tha s nguyn t ca 100! L
100 100 100
2 22 23 ... 50 25 12 6 3 1 97.

100 100
S m ca 5 l
2 20 4 24. V vy s 100! c 24 ch s 0 tn cng.
5 5
B 1. Vi mi s t nhin n 1 ta c
(8)

2n
4n

.

n 2 n

4
2

Chng minh. Bt ng thc (8) ng vi n 2 bi v 6


ng vi s t nhin n 1. Ta c

42
. Gi s bt ng thc (8)
2 2

86 | Chng minh nh Bertrand (nh l Tchebycheff)


n
2n 2
2n 1 2n 2 2n 1 4

2

n 1 n n 1 2 n
n 1

2 2n 1 4n

4n n 1 n 1

4n 1
.
2 n 1

V 2n 1 4n n 1 , suy ra 2n 1 4n n 1. Theo quy np suy ra (8) c chng minh.


2

B 2. Tch Pn ca cc s nguyn t n, vi s t nhin n cho trc, l khng vt qu 4n.


Chng minh. B hin nhin ng vi n 1 v n 2. K hiu n l s t nhin 2. Gi s b
ng vi cc s t nhin n. Nu n l s chn 2, th Pn Pn1. V vy b ng vi n. Nu

n 2k 1, vi k l s t nhin th mi s nguyn t p tha mn k 2 p 2k 1 u l c s


ca
2k 1 2k 1 2k 2k 1 ... k 2
.

1 2...k
k

(9)

2k 1 2k 1
2k 1
2k 1 k

2
, ta c
4 . Vy tch ca tt c cc s nguyn
k k 1
k
k
t phn bit tha mn k 2 p 2k 1 l c ca (9) khng vt qu 4k . Nhng theo gi thit
th b ng vi mi s nh hn n, nn tch ca cc s nguyn t k 1 l nh hn 4k 1 , ta c
Pn P2k 1 4k 4k 1 42k 1 4n. V vy Pn 4n. Theo quy np b c chng minh.
Do 1 1

2 k 1

B 3. Nu p l c s nguyn t ca s 2n vi p 2n , th p 2n ch khi ly tha ca p


n

2n
trong phn tch thnh tha s nguyn t ca bng 1.
n

Chng minh. Theo nh l 6 th ly tha ca p trong phn tch thnh tha s nguyn t ca

n n n
3 ... Do
2
p p p
x
x
2n
n x 2n
n
2n
2n 2n !
a

2
suy
ra
ly
tha
ca
trong
l
p

p k .
k
k
k
2
k 1 p
k 1 p
k 1 p

n
n n !

2n !

2n 2n 2n
2 3 ... v trong phn tch ca n !
p p p

Nu p 2n , th

ch khi n 2. Do vi mi n 2 th ta c p 2n , t y suy ra

2n
n
a 2 2. H qu l a 2, ngha l a 1 (do a l s nguyn).
p
p

4
2

B 3 c chng minh vi n 2. Vi n 2, ta c 2 3.

2n
2n
B 4. Mi c s pr (p nguyn t, r l s t nhin) ca u 2n. Ta c (2n) (2 n ) .
n
n

2n
, ly tha ca p trong phn tch thnh tha
n

Chng minh. Vi s nguyn t p tha mn p r |


2n
n
2n
l a k 2 k r.
k 1 p
p
n

s nguyn t ca

CHNG 3. S NGUYN T | 87
r 1
2n
n
2n
n

0
vi
suy
ra
k

r
;
a

k 2 k .

k
k
k 1 p
p
p
p

Nu p r 2n, th ta c

Nhng vi mi s thc x, 2 x 2 x 1, ng thc cui cng suy ra a r 1, mu thun vi tnh


cht a r. V vy p r 2n.
chng minh phn th hai ca b ta ch rng trong phn tch thnh tha s nguyn t ca
2n
2n
(2 n )
iu phi chng minh.
th ch c cc s nguyn t 2n xut hin, ta c (2n)
n
n

2n
2
khng c c s nguyn t p no m n p n .
3
n

B 5. Nu n l s t nhin 2, th

2n
n
n
1. Do 2, 1, suy ra
p
p
p
2n
n
2n 9
4 2
(3). Vi k 1 ta c p k
1 vi n 4. V vy
n v do k
p 2 p 0.
p
2n
9


2n
n
p 2 p 0. vi mi k 1 v n 4. Vy ta suy ra vi n 4 th ly tha ca s nguyn t p


2n
2n
trong bng 0, ngha l khng chia ht cho p. Suy ra b ng vi n 4. Vi n 3
n
n
6
8
2
hoc n 4 ta c t n p n suy ra p 3 v 3 khng phi c s ca 20 v 70.
3
3
4
Chng minh. Nu

2n
2
3 v
n p n, th
p
3

B c chng minh.

2n
l bng 1.
n

B 6. Ly tha ca s nguyn t p m n p 2n trong

2n
n
2n
n
2, 1. Suy ra 1, 0 . Vi k 2 ta c
p
p
p
p
2n
n
2n 2n 2
2n
2 . Suy ra vi n 1, k 1 v h qu k 0, suy ra k 0. Vy ly tha a ca
k
p
p
n
p
p
p
2n
s nguyn t p trong phn tch ca thnh tha s nguyn t l bng 1. Vi n 1 th
n
n p 2n khng tha mn. B c chng minh.
Chng minh. Vi n p 2n ta c 1

B 7. Vi n 14 ta c n

1
n 1.
2

14
1. Suy ra B 7 ng vi n 14. Gi s n l s
2
n
t nhin khng nh hn 15. Trong dy 1, 2,..., n cc s chn 4, 6,8,..., 2 u l hp s. S cc s
2
Chng minh. D dng kim tra 14 6

(3)

Tht vy vi s thc

nguyn nn ta c

ta c

2 x 2 x, 2 x 2 x 1, suy ra 2 x 2 x 1, v h qu l v v tri l s

2 x 2 x 0.

88 | Chng minh nh Bertrand (nh l Tchebycheff)

n
nh vy l 1. Hn na trong dy 1, 2,..., n vi n 15, c cc s l nhng khng nguyn t l
2

n
n
n
n n
1,9,15. V vy n n 1 3 n 2 1 (v 1 ). Do n 1 vi
2
2
2
2 2
2

n 15, suy ra b c chng minh.


B 8. K hiu Rn l tch tt c cc s nguyn t p tha mn n p 2n. Nu khng c s nguyn
t no nh vy th t Rn 1. Khi vi n 98.

Rn

(10)

4n 3
2 n 2n

n 2

2n
2n
. H qu l Qn Rn , vi Qn l s t
n
n
nhin. V vy theo B 6 ta suy ra khng c s nguyn t p no m n p 2n li xut hin
trong phn tch thnh tha s nguyn t ca Qn . Suy ra cc s nguyn t p khng xut hin trong
2
phn tch ny u n, v vy theo B 5 th s phi n. Suy ra tch ca tt c cc s nguyn
3
t phn bit p tha mn p | Qn l khng ln hn tch ca tt c cc s nguyn t khng vt qu
Chng minh. T nh ngha ca Rn suy ra Rn |

2n
2
n . Tch ny theo B 2 th khng vt qu 42 n 3. Theo b 3 v Qn | , ly tha ca s
3
n
nguyn t p trong Qn c th ln hn 1 ch khi p 2n . S cc s nguyn t nh vy theo B 7
(vi 2n thay th cho n v vi n 98, ta c 2n 14) l nh hn 2n 2. Theo B 4 th tch

ca tt c cc ly tha ca cc s nguyn t xut hin trong phn tch thnh tha s nguyn t ca

2n
l 2n
n

2n 2

. Ta cng c bt ng thc nh vy cho tch ca cc ly tha ca cc s nguyn

t xut hin trong phn tch thnh tha s nguyn t ca Qn . V vy Qn 42 n 3 2n

2n 2

. Nhng v

2n
n
Qn Rn , v theo B 1 suy ra Qn Rn 4 2 n t cng thc (10) c chng minh.
n
k
B 9. Vi s t nhin k 8 ta c 2 18 k 1 .

Chng minh. Chng minh bng quy np. Ta c 28 256 18 9. Nu 2k 18 k 1 , th

2k 1 2k 2k 18k 18 18k 18 18k 36 18 k 2 .


B 10. Vi mi s thc x 8 ta c 2x 18x.
Chng minh. Vi mi s thc x 8 ta c x 8. Theo B 9, 2x 2 x 18 x 1 18 x, suy ra

2 x 18 x, .
k
B 11. Vi s t nhin k 6 ta c 2 6 k 1 .

Chng minh. Theo B 9 th ch cn chng minh B 11 vi k 6 v k 7. Ta c


26 64 6 7 v 27 128 6 8.
B 12. Vi s thc x 6 ta c 2 x 6 x.
Chng minh tng t B 10.

CHNG 3. S NGUYN T | 89

B 13. Nu n l s t nhin 648, th Rn 2n.


Chng minh. Theo B 8 ch cn chng minh nu n 648, th 4n 3 4n n 2n

n2

. Lu rng

2n , suy ra 2n 3 2n

n2

. Nhng do

nu n 648, th

2n 6 6 v theo B 12, 2

2n 6

n3
n 648, ta c 2n 9 8, s dng B 10 ta nhn c 22 n 9 4n, suy ra 2 4n 4n 4n n.

Vy vi n 648, ta c 4n 3 4n n 2n

n2

. B c chng minh.

B 14. Nu n 648, th gia n v 2n c t nht hai s nguyn t khc nhau.


Chng minh. T nh ngha ca Rn th nu c nhiu nht mt s nguyn t gia n v 2n, th ta c

Rn 2n, vi n 648, mu thun vi B 13.


nh l 7. Nu n l s t nhin 5, th gia n v 2n c t nht hai s nguyn t khc nhau.
Chng minh. Vi n 6 nh l ng v gia 6 v 12 ch c hai s nguyn t l 7 v 11. V vy theo
B 14, nh l c chng minh cho cc s t nhin n m 7 n 648. kim tra iu ny ta
khng cn phi th vi cc trng hp 7,8,..., a 647 trc tip. Ta ch cn xc nh dy cc s
nguyn t q0 , q1 ,..., qm m q0 7, qk 2qk 2 vi k 2,3,..., m v qm1 a. K hiu n l s t nhin
ty tha mn 7 n a. Phn t u tin ca dy q0 , q1 ,..., qm l n v phn t cui l a n.
V vy tn ti ch s ln nht k vi k m 1 tha mn qk n. Ta c k 2 m, n qk 1 v v vy do

qk 2 2qk 2n, suy ra gia n v 2n c t nht hai s nguyn t l qk 1 v qk 2 .


Dy trn chnh l 7,11,13,19,23,37,43,73,83,139,163,277,317,547,631,653,1259.
H qu trc tip ca nh l 7 l
nh l 8 (Tchebycheff). Nu n l s t nhin 3, th gia n v 2n 2 c t nht mt s nguyn
t.
Chng minh. Vi n 4 v m 5 th nh l ng. Nu n 5, th theo nh l 7, gia n v 2n tn
ti t nht hai s nguyn t. Nu s ln hn l q 2n 1, th s kia l 2n 2, v 2n 2, vi n 5,
l hp s nn ta c n p 2n 2. Nu q 2n 1, th v q 2n 1, ta c n p 2n 2 .
nh l 8 c t ra nh l mt gi thuyt bi J.Bertrand vo nm 1845 v c chng minh ln
du bi P.Tchebycheff vo nm 1850. Chng minh trn l mt bin th ca chng minh ca
P.Erdos [1] c trnh by bi L.Kalmar.
H qu 1. Nu n l s t nhin 1, th gia n v 2n c t nht mt s nguyn t.
Chng minh. Theo nh l 8 th h qu ng vi cc s t nhin 3. Kim tra trc tip vi n 2
v n 3 .
Nm 1892 J.J.Sylvester [1] chng minh m rng ca H qu 1: nu n k , th trong dy
n, n 1, n 2,..., n k 1 c t nht mt s c c s k . H qu 1 nhn c vi n k 1. M
rng ny c chng minh bi I.Schur [2] nm 1924. Chng minh s cp v ngn hn c trnh
by bi P.Erdos [2] nm 1934 (Erdos [12]).
H qu 2. Vi mi s t nhin k 1 ta c pk 2k .
Chng minh. Ta c p2 3 22. Vi mi s t nhin k , pk 2k , s dng H qu 1 ta thy gia 2k
v 2k 1 c t nht mt s nguyn t, s ny khng ln hn pk . V vy pk 1 2k 1 v theo quy np
h qu c chng minh.
H qu 2 mnh hn bt ng thc (3) mc 9; tuy nhin chng minh ca n th phc tp hn.

90 | Chng minh nh Bertrand (nh l Tchebycheff)

H qu 3. Trong phn tch thnh tha s nguyn t ca n ! vi n 1 tn ti t nht mt s nguyn t


vi ly tha bng 1.
Chng minh. Vi n 2 th h qu hin nhin ng. Nu n 2k 1, vi k l s t nhin 1 th
theo H qu 1 th tn ti s nguyn t p m k p 2k , suy ra p n 2 p v h qu l p l c
s ca ch mt trong cc nhn t ca tch 1 2 ... n. Mt khc nu p tha mn k p 2k n, suy
ra 2k 2 p v do 2k 1 2 p, ngha l p n 2 p, suy ra H qu 3 c chng minh.
H qu trc tip ca H qu 3 l
H qu 4. Vi mi s t nhin n 1 th s n ! khng phi ly tha bc k vi k 1 l s t nhin.
T nh l 7 suy ra
nh l 9. Vi mi s t nhin k 3 ta c pk 2 2 pk .
Chng minh. K hiu k l s t nhin 3. Ta c pk p3 5. Theo nh l 7 th gia pk v 2 pk
c t nht hai s nguyn t khc nhau. Nhng hai s nguyn t nh nht ln hn pk li l pk 1 v

pk 2 , vy ta c pk 2 2 pk .
Ta lu rng t nh l 9 cng suy ra nh l 7. Tht vy gi s nh l 9 l ng th nu k hiu n
l s t nhin ty 6 th p4 7 n. t pk l s nguyn t ln nht tha mn pk n. Ta c

k 3 v pk 1 n. V vy theo nh l 9 th pk 2 2 pk 2n. Do gia n v 2n c t nht hai s


nguyn t l pk 1 v pk 2 . Cui cng ch cn kim tra nh l 7 vi n 6. Ta chng minh cc
nh l 7 v 9 l tng ng.
H qu 1. Ta c pk 1 2 pk vi mi k 1, 2,...
Chng minh. Vi k 4,5,... th H qu 1 c suy ra t nh l 9. Ta kim tra trc tip H qu 1
vi k 1, 2,3; p2 3 4 2 p1 , p3 5 6 2 p2 , p4 7 10 2 p3 .
H qu 2. Vi mi s t nhin k 1 ta c pk 2 pk pk 1.
Chng minh. Vi k 3 th kt qu ny c suy ra t nh l 9 vi pk 2 2 pk pk pk 1 (do

pk pk 1 ). iu ny cng ng vi k 2 v k 3. Tht vy ta c p4 7 3 5 p2 p3 v
p5 11 5 7 p3 p4 .
Bi tp. 1. Tm s t nhin n l tng ca tt c cc s nguyn t nh hn n.
Li gii. S nh nht c tnh cht ny l 5 2 3. Gi s n 5 tha mn. Nu pk l s nguyn t
ln nht nh hn n , th pk 5. Do k 2 v p1 p2 ... pk n pk 1. Do k 2, t H qu 2
nh l 9 suy ra pk 1 pk 1 pk nn p1 p2 ... pn pk 1 pn , v l. Vy ch c 5 tha mn.
2. Chng minh rng nu n 1 v k l s t nhin th

1
1
1
khng phi s nguyn.

...
n n 1
nk

Chng minh. Nu biu thc trn l nguyn th ta c

1
1
1

...
1, m ta li c c
n n 1
nk

1
1
1
k 1

...

, do k 1 n, suy ra k n. K hiu p l s nguyn t ln


n n 1
nk
n
nht n k. Ta c 2 p n k ; v theo H qu 1 nh l 8 th gia p v 2 p c s nguyn t q, v
do 2 p n k , ta c p q n k , mu thun vi nh ngha ca p. V k n, suy ra n k 2n, v
theo H qu 1 th tn ti s nguyn t r nm gia n v 2n. V vy r 2n n k v t nh ngha
ca p suy ra r p. Nhng v n r , ta c n p n k 2 p. Suy ra trong s cc tng con ca tng
lng

CHNG 3. S NGUYN T | 91

1
1
1
c ng mt tng m mu s chia ht cho s nguyn t p . T y suy ra tng

...
n n 1
nk
ny khng phi s nguyn. Tht vy, quy ng cc phn s vi mu s chung l n n 1 ... n k ,
ta thy tt c cc t s tr ra mt s l u chia ht cho p, vy tt c cc tng ring ca chui iu
1 1 1
ha ... u khng phi s nguyn.
1 2 3
3. Chng minh H qu 1 nh l 8 l tng ng vi mnh T: mi dy hu hn cc s t nhin
lin tip cha t nht mt s nguyn t th cng cha t nht mt s nguyn t cng nhau vi tt c
cc s cn li trong dy (xem thm Zahlen [1]).
Chng minh. Gi s k , k 1,..., l (i) l dy cc s t nhin lin tip v p l s nguyn t ln nht
cha trong dy. Nu 2 p l , th theo H qu 1 nh l 8 tn ti s nguyn t q tha mn
p q 2 p 1, mu thun vi nh ngha p l s nguyn t ln nht trong dy (i). Vy ta c
l 2 p. Do c th thy p nguyn t cng nhau vi 1, 2,...,l v suy ra n nguyn t cng nhau
vi mi phn t ca dy (i). Vy H qu 1 nh l 8 suy ra T. By gi gi s T ng. K hiu n 1
l s t nhin. Xt dy (ii) cc s t nhin lin tip 2,3,..., 2n, dy ny cha s nguyn t 2 nn
theo T th tn ti t nht mt s p nguyn t cng nhau vi cc s cn li trong dy. Ta lu p
phi l s nguyn t. Tht vy nu p ab, vi a v b l cc s t nhin 1, th s a p thuc
dy (ii) v khng nguyn t cng nhau vi p. Hn na nu p n, th 2 p 2n v s 2 p p
thuc (ii) v 2 p khng nguyn t cng nhau vi p. V vy ta c p n. Nhng v p thuc dy (ii)
v p 2n. Hn na p 2n v n 1 v p nguyn t. T y suy ra n p 2n. Ta chng minh
T suy ra H qu 1 nh l 8. Vy hai mnh ny tng ng v ta c iu phi chng minh.
4. S dng H qu 1 nh l 8 chng minh rng vi mi s t nhin k v n 2k th k s nh nht
1 khng chia ht cho s no trong cc s 2,3,..., n u l s nguyn t.
Chng minh. Nu n 2k , th n2 2k n v theo H qu 1 nh l 8 th gia hai phn t lin tip
ca dy n, 2n, 23 n,..., 2k n u c t nht mt s nguyn t, gia n v n 2 c t nht k s nguyn t
khc nhau. Vy gia n v n 2 cng tn ti t nht k s khng chia ht cho bt k s no trong cc
s 2,3,..., n. Mi s nh vy u l s nguyn t v nu l l mt s nh th v l ab, vi a, b l
cc s t nhin 1 , a b, th ta c a n (do l khng chia ht cho bt k s no trong cc s

2,3,..., n. ). Vy ta phi c b a n, suy ra l ab n2 , v l.


11. nh l H.F.Scherk
nh l 10 (H.F.Scherk). Vi mi s t nhin n v cc du v thch hp ta c
(11)

p2n 1 p1 p2 ... p2n2 p2 n1

(12)

p2n1 1 p1 p2 ... p2n1 2 p2n .

Cc cng thc ny c tm ra bi H.F.Sherk [1] vo nm 1830, chng minh ca H.F.Scherk c


cng b bi S.S.Pillai [1] nm 1928. Chng minh di y c tc gi cng b nm 1952
(Sierpinski [14]). Chng minh tng t c trnh by bi R.Teuffel [1] nm 1955.
Chng minh. Ta ni dy v hn q1 , q2 ... l c tnh cht P nu n l dy tng cc s t nhin, tt c
u l s l tr ra s u tin tha mn
(13)

q1 2, q2 3, q3 5, q4 7, q5 11, q6 13, q7 17

(14)

qn1 2qn vi n 1, 2,...

92 | nh l H.F.Scherk. nh l H.E.Richert

Theo H qu 1 nh l 9 th dy qn pn (vi n 1, 2,... ) c tnh cht P. Do chng minh nh


l Scherk ta ch cn chng minh vi la chn cc du thch hp th cc cng thc (11),(12) ng
vi mi dy c tnh cht P.
B . Nu q1 , q2 ,... l dy v hn c tnh cht P, th vi n 3 mi s t nhin l q2 n1 , c dng

q1 q2 ... q2n1 q2n vi cc du v thch hp.


Chng minh b . T (13) suy ra b ng vi n 3, v

Ta lu vi n 2 th b khng ng v khng th c 5 2 3 5 7 .
Gi s b ng vi s t nhin n 3 v t 2k 1 l s l q2 n3 .
T (14) ta c q2 n3 2q2 n 2 v h qu l q2n2 2k 1 q2n2 q2 n2 . Do chn cc du v

thch hp ta c 0 2k 1 q2 n2 q2 n2 . Theo (14) ta c q2 n 2 2q2 n1 v do

q2n1 2k 1 q2n2 q2n1 q2n1. Hn na chn cc du v thch hp ta c


(15)

0 2k 1 q2n2 q2n1 q2n1.

Cc s q2 n 1 v q2 n 2 u l v do s gia bt ng thc (15) cng l l v q2 n 1. H qu l


theo gi thit quy np ta suy ra vi la chn cc du v thch hp ta c
2k 1 q1 q2 ... q2n q2n1 q2n . V vy la chn cc du v thch hp ta c

2k 1 q1 q2 ... q2n q2n1 q2n2 , suy ra b ng vi n 1 v theo quy np ta c iu phi


chng minh vi n 3.
H qu. La chn cc du v thch hp ta c
(16)

q2n1 q1 q2 ... q2n1 q2 n .

Chng minh. V q2 n 1 l s l nn vi n 3 cng thc (16) c suy ra trc tip t b . Vi

n 1 v n 2 th tnh ton trc tip cho thy q3 q1 q2 v q5 q1 q2 q3 q4 .


By gi ta chng minh cc cng thc (11),(12).
Chng minh (12). Theo (14) th vi n 3 s q2 n1 q2 n 1 l s l q2 n 1. Do p dng b , ta
thy vi la chn cc du + v thch hp ta c q2n1 q2 n 1 q1 q2 ... q2 n1 q2 n v t y
suy ra (vi qi pi , i 1, 2,... ) cng thc (12) ng. Vi n 1 v n 2 tnh ton trc tip ta c

q3 1 q1 2q2 , q5 1 q1 q2 q3 2q4 .
Chng minh (11). Theo (14) ta c q2 n 2 2q2 n1 v ta thy q2 n2 q2 n1 1 l s l 0 v q2 n 1.
p dng b ta thy vi n 3 v la chn cc du + v - thch hp th

q2n2 q2n1 1 q1 q2 ... q2n1 q2n ,


(17)

q2n2 1 q1 q2 ... q2n1 q2n q2n1.

Hn na theo (13) ta thy

CHNG 3. S NGUYN T | 93

q2 1 q1 , q4 1 q1 q2 q3 ,
q6 1 q1 q2 q3 q4 q5 ,
Suy ra (17) ng vi n 0,1 v 2. H qu l (17) ng vi n 0,1, 2,... do ( qi pi , i 1, 2,... )
cng thc (11) ng vi n 1, 2,3,... nh l Scherk c chng minh.
12. nh l H.E.Richert
B 1. Nu m1 , m2 ,... l dy v hn tng cc s t nhin tha mn vi s t nhin k no bt
ng thc
(18)

mi 1 2mi for i k

ng, v nu s t nhin a 0 v s t nhin r v sr 1 mk r tha mn cc s


(19)

a 1, a 2,..., a sr 1

u l tng ca cc s khc nhau trong dy m1 , m2 ,..., mk r 1 th vi sr sr 1 mk r cc s


(20)

a 1,

a 2, ..., a sr

u l tng ca cc s khc nhau trong dy m1 , m2 ,..., mk r v hn na sr mk r 1.


Chng minh. Gi s cc iu kin trong gi thit ca b c tha mn. K hiu n l s t
nhin thuc dy (20). Nu n a sr 1 , th theo gi thit n l tng ca cc phn t khc nhau ca
dy m1 , m2 ,..., mk r 1. By gi gi s n a sr 1 th t sr 1 mk r ta c n a 1 mk r do

n mk r a 1. Hn na v n l phn t ca dy (20) nn ta c n a sr a sr 1 mk r . Do
n mk r a sr 1. V vy s n mk r , l phn t ca dy (19) v do n l tng ca cc phn t
khc nhau ca dy m1 , m2 ,..., mk r 1. Vy n l tng ca cc phn t khc nhau ca dy
m1 , m2 ,..., mk r . Hn na theo (18) ta c mk r 1 2mk r , do sr sr 1 mk r 2mk r mk r 1.
B 2. Nu m1 , m2 ,... l dy v hn cc s t nhin tha mn (18) ng vi s t nhin k v nu
tn ti s nguyn a 0 v s t nhin s0 mk 1 tha mn cc s
(21)

a 1, a 2,..., a s0

u l tng cc phn t khc nhau ca dy m1 , m2 ,...mk th mi s t nhin a cng l tng cc


phn t khc nhau ca dy m1 , m2 ,...
Chng minh. Gi s iu kin ca b c tha mn. Ln lt p dng B 1 vi r 1, 2,.., l
trong l l s t nhin, ta suy ra cc s
(22)

a 1, a 2,..., a sl

u l tng cc phn t khc nhau ca dy m1 , m2 ,...mk 1 . Nhng sr sr 1 , r 1, 2,..., l , nn ta thy


vi mi s t nhin n th u tn ti s t nhin l tha mn n a sl . H qu l mi s t nhin

n a u l mt trong cc s ca dy (22) vi l thch hp. Cc s ny u l tng cc phn t


khc nhau ca dy m1 , m2 ,... B c chng minh.
By gi t mi pi vi i 1, 2,... Theo H qu 1 nh l 9 th cc iu kin trong B 2 c
tha mn vi a 6, s0 13, k 5; bi v 13 p6 v cc s 7,8,19 u l tng ca cc s nguyn t
phn bit p5 . Tht vy

7 2 5,8 3 5,9 2 7,10 3 7,11 11,

94 | Gi thuyt v cc s nguyn t

12 5 7,13 2 11,14 3 11, 15 2 5 7,


16 5 11,17 2 3 5 7,18 7 11,19 3 5 11.
R rng ta c th loi ra cc tng ch c mt hng t: s 11 khng phi tng ca hai hay nhiu hn
cc s nguyn t. T B 2 ta c h qu
nh l 11. Mi s t nhin 6 u l tng ca cc s nguyn t khc nhau (Richert [1],[2]).
Gi s mi pi 1. Cc iu kin ca B 2 c tha mn vi a 9, s 0 19, k 6, t 19 p8 m,
nn s0 m61 v hn na mi s 10,11,...28 u l tng ca cc s nguyn t l m6 19. Tht vy

10 3 7,11 11.12 5 7,13 13,14 3 11, 15 3 5 7,


16 5 11,17 17,18 5 13,19 3 5 11, 20 7 13, 21 3 5 13, 22 5 17 ,
23 3 7 13, 24 11 13, 25 5 7 13, 26 3 5 7 11, 28 3 5 7 13.
nh l 12. Mi s t nhin 10 u l tng ca cc s nguyn t l khc nhau.
Nu ta b sung s 2 nh l mt hng t th ta c
nh l 13. Mi s t nhin 12 u l tng ca hai hoc nhiu hn cc s nguyn t khc nhau.
D dng thy 11 khng phi tng ca hai hay nhiu hn cc s nguyn t khc nhau. S 17 khng
phi tng ca hai hoc ba s nguyn t khc nhau (nhng 17 2 3 5 7 ). Ta c th chng minh
mt cch s cp rng tn ti v hn s l khng l tng ca t hn ba s nguyn t.
Sau y l bn nh l c trnh by bi R. Dressier, A.Makowski v T.Parker [1]: Mi s t nhin
>1969 u l tng ca cc s nguyn t khc nhau c dng 12k 1. Mi s t nhin >1349 u l
tng ca cc s nguyn t khc nhau c dng 12k 5. Mi s t nhin >1387 u l tng ca cc s
nguyn t khc nhau c dng 12k 7. Mi s t nhin >1475 u l tng ca cc s nguyn t khc
nhau c dng 12k 11. Cc chn di l khng th thay bng cc s nh hn.
Mt s kt qu c lin quan c th xem trong J.L.Brown Jr, [2].
13. Gi thuyt v cc s nguyn t
Vi nm trc ti (Sierpinski) c t ra gi thuyt P sau y
Gi thuyt P. Nu cc s 1, 2,3,..., n2 vi n 1 c xp thnh n dng, mi dng cha n s

th mi mt dng cha t nht mt s nguyn t (Schinzel et Sierpinski [3]).


Dng th nht ca bng (23) cha s 2 n 1 . Mnh ni rng dng th hai cha s nguyn t
vi n 1 l h qu 1 ca nh l 8. T cc bt ng thc ca J.B.Rosser v L.Schoenfeld (mc 15)
th vi n ek th k dng u tin cha s nguyn t. S dng cc bng ca Lander v Parkin [3]
v [1],[4] c th kim tra gi thuyt P ng vi 1 n 21105. Do hai dng cui ca bng u cha
cc s n 1 , n 1 1,..., n2 , nn gi thuyt P suy ra rng gia hai bnh phng lin tip cc s
2

t nhin c t nht hai s nguyn t. Hn na trong mi khong m im kt thc l lp phng


ca hai s t nhin lin tip u c hai bnh phng ca hai s t nhin lin tip. Mnh cui
cng cha c chng minh nhng theo kt qu ca A.E.Ingham vo nm 1937 th s cc s
3
nguyn t nm gia n3 v n 1 tin ti v cng theo n.
H qu trc tip ca gi thuyt P l gia hai s tam gic c t nht mt s nguyn t. Ngha l nu
ta xp cc s t nhin thnh tam gic vung m dng th n cha n s t nhin lin tip. tc l

CHNG 3. S NGUYN T | 95

th tr ra dng u tin cc dng cn li u cha s nguyn t. Gi thuyt ny cha c cu tr li.


Nm 1932 R.Haussner [1] t ra gi thuyt rng vi mi s t nhin k , th gia hai bi s lin tip
ca s nguyn t pk u nh hn pk21 tn ti t nht mt s nguyn t. Gi thuyt ny c kim
chng bi Haussner vi cc s nguyn t pk 100. Gi thuyt P cho s nguyn t n l h qu trc
tip ca gi thuyt ca Haussner.
L.Skula lu rng t gi thuyt P suy ra vi mi s t nhin n 1 th cc hng th n 1 v
hng n 2 u cha t nht mt s nguyn t. T gi thuyt P vi n 1 th gia cc s
n2 1, n2 ,..., n n 1 c t nht mt s nguyn t v vi n 2 th hai phn t u tin ca dy u
l hp s v c t nht mt s nguyn t nm trong cc s n2 1, n2 ,..., n n 1 . iu ny cng
ng vi n 2. T gi thuyt P vi n 1 th gia cc s n2 n 1, n2 n 2,..., n 1 c t nht
2

mt s nguyn t, v vy c t nht mt s nguyn t nm gia n2 n 1, n2 n,..., n2 2n v

n 1

l hp s. A.Schinzel t ra gi thuyt rng nu n l s t nhin 1 v k l s t nhin

nh hn n v nguyn t cng nhau vi n, th trong ct th k ca bng (23) c t nht mt s


nguyn t (Schinzel v Sierpinski [3]). Ni cch khc nu k v n l cc s t nhin nguyn t cng
nhau v k n, th trong cc s k , k n, k 2n,..., k n 1 n lun c t nht mt s nguyn t.
Theo bng ca Wagstaff [2] th iu ny ng ti n 500000.
Nm 1947 Yu.V.Linnik chng minh s tn ti ca hng s C tha mn nu k , n 1 v 1 k n
th s nguyn t nh nht trong cp s cng k , k n, k 2n,... nh hn nC . J.R.Chen [13] chng
minh rng bng vic thay th nC bi AnC vi gi tr A thch hp ta c th chn C 17 . (S.Graham
[1]). Gn y Chen a ra kt qu mi vi C 14 .
A.Schinzel [13] t ra mt gi thuyt mnh hn gi thuyt P. Theo nu x l s thc 117
th gia x v x x c t nht mt s nguyn t. Gi thuyt ny (k hiu l P1) c kim tra bi
cc bng ca Lander v Parkin v bng ca Brent vi 117 x 4, 44 1012. Legendre l ngi t ra
gi thuyt ni rng vi x ln th c t nht mt s nguyn t nm gia x v x x .
By gi ta chng minh gi thuyt P vi n 117 c suy ra t gi thuyt P1. K hiu n l s
nguyn 117 v k l s t nhin nh hn n. Ta c kn 117 v do theo gi thuyt P1 tn ti s
nguyn t p m kn p kn kn. Nhng v k n, ta c kn n; do tn ti t nht mt s
nguyn t trong dy kn 1, kn 2,..., k 1 n. Do iu ny ng vi mi s t nhin k n, ta thy
vi n 117 th mi hng ca bng (23) t dng th hai c t nht mt s nguyn t. Vy gi thuyt
P vi n 117 c suy ra t gi thuyt P1. Vi n 117 gi thuyt P c kim tra trc tip.
A.Schinzel [13] nhn thy c th lm mnh hn gi thuyt P1 thnh: vi mi s thc x 8 th gia

x v x log x c t nht mt s nguyn t. S dng cc bng ca Lander, Parkin v bng ca


2

Brent c th kim tra gi thuyt ny vi mi x 4, 44 1012. Nu ta t x pn vi n 4 th ta c


bt ng thc

pn1 pn log pn

lim pn1 pn log pn 1.


2

vi mi n 4 . H.Cramer [1] t ra gi thuyt

96 | Gi thuyt v cc s nguyn t. Bt ng thc ca hm

Di y l mt gi thuyt khc v hiu ca hai s nguyn t lin tip c a ra bi


N.L.Gilbreath vo nm 1958. Ta lp bng cc s t nhin theo cch sau: dng th nht vit hiu cc
s nguyn t lin tip, ngha l cc s pn1 pn , n 1, 2,... , dng th hai ta vit gi tr tuyt i ca
hiu ca cc s lin nhau trong dng th nht. Trong cc dng tip theo ta li vit tip nh vy. Gi
thuyt ca Gilbreath ni rng cc s u tin ca mi dng u l 1. Di y l 10 dng u tin

Gi thuyt ca Gilbreath c kim tra cho 63418 dng u tin vi my tnh SWAC. Gi thuyt
ny vn cha c chng minh (Killgrove v Ralston [1]).
14. Bt ng thc ca hm x
By gi ta xt cc h qu ca B 9 mc 10. Do Rn k hiu tch ca cc s nguyn t p m

n p 2n v s cc s nguyn t nh vy l 2n n (v theo H qu 1 nh l 8 mc 10

th vi mi s t nhin n tn ti t nht mt s nguyn t p nh th). Hn na cc s nguyn t


2 n n

u nh hn 2n, suy ra Rn 2n

T cng thc (10) mc 10 suy ra vi cc s t nhin n 98 ta c (2n) (2 n ) ( n )

4n /3
2 n (2n)

n /2

Ly logarithm hai v ta suy ra vi n 98 th


(24)

2n n

n
3log 4n 3log 2n
log 4

3log 2n
2n
2n

log x
0; do lim 2n n . T y suy ra vi mi s t nhin
n x
x x
x
k th u tn ti s t nhin mk m vi mi n mk th u tn ti t nht k s nguyn t nm gia

Nhng ta bit lim

n v 2n. Hn na v log x x (vi x e. ) l hm gim theo x, ta c vi n 2500


log 4n log 2n
3log 4n 3log 2n

2n
4
n
2n
2n

log 4 2500 log 2 2500


6

0,37;
2 2500
4 2500
V vy
(25)

log 4

3log 4n 3log 2n

1,38 0,37 1.
2n
2n

Theo (24) th cng thc (25) cho thy bt ng thc Finsler


(26)

2n n

n
.
3log 2n

CHNG 3. S NGUYN T | 97

2n
n
4
n

ng vi mi s t nhin n 1. By gi ta ch rng vi mi s t nhin n th ta c

2n
2n
). Do Rn | nn ta thy Rn 4n v
n
n
2 n n
2 n n
t nh ngha ca Rn c th suy ra Rn n . T n 4n v do
log 4
7n
7
2n n

v log 4 . S dng (26) ta c (Finsler [1] v Trost [3], Satz 32)


log n 5log n
5
(p dng trc tip cng thc nh thc suy ra 1 1

2n

n
7n
(2n) (n)
vi mi n 1
3log 2n
5log n

(27)

n
vi n 1 v vi n 4 ta c n n / 2 [n/2] n / 2 1 n / 4 v do
3log 2n
n 2 n vi mi n 4,
n
log 2 n 2 log n, ta c n 2
2 3log 2 n 2 12log n

T (27) suy ra (2n)

( n)

(28)

n
vi mi n 1
12log n

D dng kim tra bt ng thc ng vi n = 2 v n = 3. Ta s chng minh

(2k )

(29)

2k 1
k log 2

C th thy cng thc (29) ng vi cc s t nhin k 6 v log 2 1. Gi s n ng vi s t


nhin

k 6.

2k 1 2k
2k 1

Theo (27) (vi 2k thay th v tr ca n )


7 2k
2k 1
7

1 . Nhng vi k 6 ta c
5k log 2 k log 2 10

(29)

k 1 1

ta

7
2k ,
10

k 2

2
v theo quy np bt ng thc (29) ng.
k 1 log 2

k
k 1
K hiu n l s t nhin 1 . Tn ti s t nhin k tha mn 2 n 2 , suy ra

k 1 log 2 log n. V vy theo (29) ta c n 2k 1


n

(30)

2k 1
4n

. T y ta suy ra
k 1 log 2 log n

4n
vi mi s t nhin n 1
log n

Thay n bi pn trong (28) v (30) v do pn n ta c

pn
4 pn
n
; suy ra v pn n
12 log pn
log pn

(vi n 1, 2,... ).
Ta c pn

n
n log n
v pn 12n log pn . suy ra log pn log12 log n log log pn .
log pn
4
4

n
Nhng theo H qu 2 nh l 8 mc 10 ta thy pn 2 , suy ra log pn n log 2 v

log log pn log n log log 2.

Do

log 2 1,

vi

n 12

ta

n 12log 2

do

log n log12 log log 2. V vy vi n 12, ta c log pn 2log n log12 log log 2 3log n. H
qu l pn 36n log n vi mi n 12 v cng vy vi 2 n 12. Ta c kt lun

98 | Bt ng thc ca hm

n log n
pn 36n log n vi mi n 1 .
4

(31)

T cng thc (28) ta c h qu sau: vi mi s t nhin s th u tn ti s t nhin c th biu din


nh l tng ca hai s nguyn t theo nhiu hn s cch.
Chng minh. Gi s rng vi s t nhin s m khng tn ti s t nhin no c th biu din di
dng tng ca hai s nguyn t nhiu hn s cch. K hiu n l s t nhin 1 . Xt tt c cc cp

p, q vi

p, q l cc s nguyn t m khng c s no ln hn n. S cc cp nh vy l n .
Ta chia tp cc cp s p, q thnh cc lp theo cch p, q thuc lp k nu p q k. Do p n
2

v q n nn ta c k 2n. Theo gi thit vi s cho trc k 2n th trong lp th k tn ti nhiu


nht s cp khc nhau. Do s cc lp l nh hn 2n, s cc cp p, q l nh hn 2ns. H qu l

n 2ns v theo cng thc (28) ta c n n2 122 log n , suy ra 2 122 s log n n.
2

x
3
Nhng e x 3! vi mi x 0, thay x log n, ta c 6n log n . V vy 123 s(log n) 2 (log n) 3
3

vi n 1 suy ra log n 12 s vi mi n 1, m vi n ln th iu ny khng ng. H qu l t


gi thit ban u suy ra mu thun. Vy h qu c chng minh. C mt gi thuyt c t ra l
s cc cch phn tch mt s chn n thnh tng ca hai s nguyn t s tin ti v cng theo n .
3

Ghi ch. Cc s c th biu din thnh tng ca hai s nguyn t nhiu hn mt cch phi l chn
vi lu l ta khng tnh cc biu din sai khc mt hon v. Tht vy nu s l n l tng ca hai
s nguyn t th mt trong hai s phi chn ngha l bng 2 suy ra ta ch c nhiu nht mt biu
din. Thay i mt cht chng minh B 1 ta c th chng minh vi mi s t nhin s th u
tn ti s t nhin c th biu din thnh tng ca ba bnh phng cc s nguyn t vi nhiu hn
s cch. P.Erdos [4] chng minh vi mi s t nhin s th u tn ti s t nhin c th biu
din thnh tng (tng ng, hiu) ca cc bnh phng ca hai s nguyn t theo nhiu hn s
( n)
cch. T (30) suy ra lim
0 . M log n log log n log 4 log pn log n log log n log36 nn
n
n
(32)

lim
n

log pn
1
n

By gi ta suy ra mt s h qu t bt ng thc (31). Theo (31) ta c


n

k 2,3,..., t y suy ra vi s t nhin n 2 th

k 2

1
1

vi
pk 36k log k

1
1 n
1

.
pk 36 k 2 k log k

1 1
Nhng log 1 x x vi 0 x 1, suy ra vi k 2,3,..., log k 1 log k log 1 , t
k k

log k 1
1

log(k 1)
1
1
1
log 1

v log log(k 1) log log k log


. V

log k
k log k
log k
k log k k log k
1
log log k 1 log log k vi k 2,3,..., n. Suy ra vi s t nhin n 2 ta u c
vy
k log k
n
n
1
1
1
log log n 1 .
(
).
Vy
log
log
2

log
log
n

log
log
2

log
log
n

36
k 2 pk
k 2 k log k
1 1 1 1 1 1
Suy ra chui tng nghch o cc s nguyn t lin tip, tc l chui ...,
2 3 5 7 11 13
phn k.

CHNG 3. S NGUYN T | 99

15. nh l s nguyn t v cc h qu
T cc cng thc (28) v (30) mc 14 suy ra tn ti cc s dng ( a

a n :

1
, b 4 ) tha mn
12

n
b vi mi s t nhin n 1. Nm 1896 J.Hadamard v Poussin chng minh
log n

x
lim x :
1.
x
log x

(33)

Ngy nay vi phng php mi c t ra bi A.Selberg [1] v P.Erdos [9], cng thc ny (c
bit di tn nh l s nguyn t) c chng minh mt cch s cp hn (mc d vn rt phc
tp). Ta khng trnh by chng minh y (4).

n
h n , th h(103 ) 1.159 , h(104 ) 1.132 , h(105 ) 1.104 , h(106 ) 1.084 ,
log n
7
h(10 ) 1.071 , h(108 ) 1.061, h(109 ) 1.053 , h(1010 ) 1.048 . Xp x tt hn ca hm x c

Nu n :

dt
dt
thu bi hm
. J.E.Littlewood chng minh hiu x
nhn v hn gi tr dng v
log t
log t
0
0
v hn gi tr m khi x nhn tt c cc gi tr t nhin. Chng minh nh l ny v cc nh l c
cp trong chng ny vi phng php gii tch c th xem trong cun sch ca K.Prachar [1].
n log pn
Trong cng thc (33) t
th ( )
v ta c lim
1 khi theo (32)
n
pn
(34)

pn
1
n n log n

lim

T y suy ra ta c th xp x pn bi n log n, vi n ln. T (34) suy ra ngay lim

pn 1
1.
pn

J.B.Rosser [1] chng minh rng vi mi s t nhin n th ta c bt ng thc pn n log n . Cc


thng tin nhiu hn v hm n c suy ra t (33) c cho bi nh l J.B.Rosser v
L.Schoenfield [1] ni rng
(35)

n
log n

1
2

n
log n

3
2

vi mi s t nhin n 67. R rng cng thc (33) suy ra t (35). Nhng ngay c t bt ng
thc (35) ta cng khng quyt nh c nhiu tnh cht n gin ca hm n . Mt v d l
nh l ca E.Landau (xem Landau [3] tp 1 trang 215-216) ni rng 2n 2 n ng vi gi
tr ln ca n, ngha l c nhiu s nguyn t trong khong 0 x n hn l trong khong
n x 2n, vi n ln. Rosser v Schoenfeld [2] ch ra rng ch cn gi thit n 11. Mt cu
hi c t ra l vi cc s t nhin x 1 v y 1 no th bt ng thc sau ng
(36)

x y x y

T bt ng thc ny suy ra ngay 2n 2 n ng vi mi s t nhin n.

(4)

Xem Trost [3], chng 7: Elementarer Beweis des Primzahlsatces, trang 66-73: xem LeVeque [1], tp 11 trang 229263, chng 7: The prime number theorem.

100 | nh l s nguyn t v cc h qu

Bt ng thc (36) c chng minh bi A.Schinzel [13] vi min x, y 146, v bi J.L.Selfridge


vi lp rng hn (cha cng b) v c kim tra bi S.L.Segal [1] vi x y 100000. Tuy nhin
D.Hensley v I.Richards [1] chng minh rng (36) l khng tng thch vi gi thuyt H. S
dng phng php ny gn y T.Vehka [1] chng minh s khng tng tch xut hin vi

min x, y 11763.

Tip tc vi hm x ta lu rng hm s xc nh s cc s nguyn dng x c ng k c


s nguyn t (tng ng k c s t nhin) vi k v x l cc s t nhin c nghin cu v
cng thc m t dng iu ca n c tm ra (Sathe [1], tng ng LeVeque [1]).

log ax
1, theo (33) ta c
x log bx

By gi vi hai s thc a v b tha mn 0 a b. Ta c lim

bx b
. T v 0 a b, bx ax , vi n ln. T y suy ra mnh sau: nu
x ax
a
a v b l hai s thc dng v a b, th vi s thc ln x s c t nht mt s nguyn t nm
gia ax v bx. c bit nu a 1 v b 1 vi l s thc dng ty th suy ra c t nht
mt s nguyn t nm gia n v n 1 vi n ln. Gi s c1 , c2 ,..., cm l dy hu hn cc ch
lim

s ty . K hiu a l mt s vi cc ch s l ca n c1 , c2 ,..., cm . p dng h qu suy ra t cng


thc (33) ta c an a 1 n ng vi gi tr ln ca n. H qu l tn ti s t nhin s

s
tha mn a 10

a 1 10 . V vy tn ti s nguyn t
s

p m a 10s p a 1 10s. V

vy m ch s u tin ca p trng vi cc ch s tng ng ca a. Ngha l m ch s u tin


ca p l c1 , c2 ,..., cm . T y ta c mt h qu khc ca cng thc (33) l: vi dy hu hn

c1 , c2 ,..., cm cc ch s ty th tn ti s nguyn t m m ch s u tin ca n l c1 , c2 ,..., cm

(5)

K hiu x l s thc 0 . Vi s t nhin ln n ta c nx 2 do (nx) 1 . T (34) suy ra


(37)

lim
n

p ( nx )

(nx) log (nx)

Nhng t (33) ta c
(38)

lim
n

(nx) log (nx)


nx

Suy ra lim log nx log log nx log nx 0, vy


n x

(39)

lim
n

log (nx)
1
log(nx)

T cc cng thc (37), (38) v (39) ta suy ra lim

P ( nx )
nx

1 . Vy ta chng minh cng thc (33)

suy ra vi mi s thc x 0 th u tn ti dy v hn cc s nguyn t q1 , q2 ,... tha mn


q
lim n x . Tnh cht ny c pht hin bi H.Steinhaus. Cui cng nu a v b l cc s thc ty
n n
tha mn
th t h qu ca cng thc (33) suy ra nu q l s nguyn t ln th tn ti
s nguyn t p tha mn aq p bq, t y suy ra a p q b. iu ny chng t tp hp cc
t s p q , p v q l cc s nguyn t, l tr mt trong tp cc s thc dng.
(5) Xem Sierpinski [10] v Trost [3] trang 42 nh l 20, xem Sierpinski [25]. nh l mnh hn c chng minh.

CHNG 4
S CC C S V TNG CA CHNG
1. S cc c s
S cc c s ca mt s t nhin cho trc n c k hiu l d (n) . lp bng cc gi tr ca
hm s d (n) ta c th s dng phng php di y. Phng php ny c bin i t
phng php sng Eratosthenes. u tin tnh cc gi tr d (n) vi n a ta vit cc s 1, 2,..., a
v nh du chng. Sau ta nh du tt c cc s chia ht cho 2, sau l cc s chia ht cho 3
v c nh vy. Cui cng ta nh du s a. S cc c s ca s n chnh l s du m n c
gch di (Harris [1]). Chng hn vi a 20 ta c
1, 2, 3, 4, 5, 6, 7, 8, 9, 10, 11, 12, 13, 14, 15, 16, 17, 18, 19, 20.

V vy ta c

Vi s t nhin n ln hn 1 xt phn tch thnh tha s nguyn t ca n l


(1)

n q11 q22 ...qkk

Gi s d l c s ca n. Khi mi c s ca d cng l c s ca n, do phn tch thnh


tha s nguyn t ca d cng c dng (1). Hn na s m ca cc ly tha ca biu din l
nh hn cc ly tha tng ng trong (1). Ngha l mi c s d ca n c th vit di dng
(2)

d q11 q22 ...qkk ,

Vi i i 1, 2,..., k l cc s nguyn tha mn cc bt ng thc


(3)

0 i xi vi i 1, 2,..., k

Mt khc mi s t nhin c th biu din dng (2) vi cc s i tha mn (3) u l c s t




nhin ca n. v theo (3) th n d q1 1 1 q2 2 2 ...qk k k l s nguyn.

Cui cng cc b s nguyn phn bit


(4)

1 , 2 ,..., k

xc nh cc s khc nhau trong (2). Ta c nh l


nh l 1. Nu n l s t nhin m phn tch thnh tha s nguyn t ca n c vit di dng
(1) th vi tt c cc b s phn bit gm k s nguyn trong (4) tha mn (3) ta xc nh c tt
c cc c s ca n cho bi (2). Hn na mi b s ng vi ng mt c s ca n .
H qu l s cc c s ca s t nhin n c biu din thnh tha s nguyn t c dng (1) bng
vi s cc b s nguyn (4) tha mn cc bt ng thc (3). S b s nh vy c th tnh mt cch
n gin. Tht vy s nguyn i tha mn (3) khi v ch khi i thuc dy 0,1, 2,..., i . V vy vi

i 1, 2,..., k cho trc th i nhn i 1 gi tr phn bit. Suy ra


nh l 2. S d n cc c s ca s t nhin n c biu din thnh tha s nguyn t (1) l
(5)

d n 1 1 2 1 ... k 1 .

102 | S cc c s

Chng hn tnh d 60 . Ta c 60 22 3 5. Do theo (5) ta c d 60 2 11 11 1 12.


2
2
Tng t v 100 2 5 , suy ra d 100 2 1 2 1 9.

T (5) suy ra vi mi s t nhin s 1 c v hn cc s t nhin m c ng s c s. Tht vy


s 1
s 1
s. R rng ng thc d n 1 suy ra
nu n p , vi p l s nguyn t th d n d p

n 1. Cng thc (5) chng t d n 2 ch khi k 1 v 1 1, ngha l n l s nguyn t. Cc


nghim ca phng trnh d n 2 u l s nguyn t. Vy vi hp s n ta c d n 3. T (5)
suy ra d n l s l khi v ch khi tt c cc s xi vi i 1, 2,..., k u chn, ngha l n l bnh
phng ng.
Bi tp. 1. Chng minh rng vi mi s t nhin n th d n 2 n .
iu ny l hin nhin v trong hai c s i nhau ca n th c mt s khng vt qu

n.

2. Tm tt c cc s t nhin c ng 10 c s.
Li gii. Nu d n 10, th theo (5) ta c

1 12 1 ...k 1 10.

C th gi s

1 2 ... k . V c hai cch biu din 10 thnh tch ca hai s t nhin 1 theo th t
khng gim nn 10 2 5 v 10 10, nn hoc k 2, 1 1, 2 4, hoc k 1, 1 9. Suy ra cc s
9
t nhin c 10 c s c dng pq 4 vi p, q p l cc s nguyn t ty hoc c dng p , vi p
l s nguyn t ty .

3. Tm s t nhin n nh nht m d n 10.


9
4
Li gii. Theo bi tp 2 th cc s 2 , 2 3 , v 3 24 (s cui cng l nh nht) l cc s t nhin

nh nht n m d n 10 . Vy n 3 24 48.
Ghi ch. Vi hai s nguyn t p, q cho trc, q p , th s t nhin nh nht c ng pq c s
l 2q 1 3 p 1.
4. Chng minh rng nu n l s t nhin 1 th trong dy v hn n, d n , d d n , ddd n ,... tt
c cc phn t ca dy s bng 2 k t mt v tr no . Chng minh rng v tr ny c th ty .

Lu rng nu n l s t nhin ln hn 2 th d n n, v d 2 2. Mt khc d 2n 1 n.


5. Chng minh rng vi mi s t nhin m th tp hp cc s t nhin n m s c s ca n chia
ht cho m cha mt cp s cng v hn.
m
m1
Chng minh. Ta lu rng cc s 2 t 2 t 1, 2,... to thnh cp s cng v hn v thuc v

tp xc nh nh bi bi m. Tht vy, ly tha ca 2 trong phn tch thnh tha s nguyn t


ca n 2m t 2m1 l m 1. V vy theo (1) ta c m | d n .
Ghi ch. H qu trc tip ca nh l trn l vi mi s t nhin m th tp hp cc s t nhin n
m m | d n c chn di tr mt. Ngha l tn ti s dng a m s Sm x cc s t nhin n x
m m | d n l ln hn ax vi mi x ln. E.Coben [1] chng minh rng vi mi s t nhin

Sm x
tn ti v l mt s dng.
x x
x

m th gii hn lim

CHNG 4. S CC C S V TNG CA CHNG | 103

Nm 1940 bng d n vi n 10000 c trnh by (Glaisher [2]). Da vo bng ny ta tm


thy d n d n 1 d n 2 d n 3 8 ng vi n 3655, 4503,5943,6853,8393,9367.
J.Mycielski ch ra vi n 40311 ta c d n d n 1 d n 2 d n 3 d n 4 .
Chng minh nhn xt ny suy ra ngay t 40311 33 1493, 40312 23 5039, 40313 7 13 443,
40314 2 3 6719, 40315 5 11 733. Tnh hung tng t cng xy ra vi n 99655.
Cu hi t ra l c th c bao nhiu gi tr lin tip d n bng nhau (Erdos v Mirsky [1]).
Ta c d 2 d 3 , d 14 d 15 , d 33 d 34 d 35 4, d 242 d 243 d 244 d 245 6.
D.R.Heath-Brown [1] chng minh s tn ti v hn n m d n d n 1 . Ta cha bit c tn
ti dy v hn tng cc s t nhin nk k 1, 2,... m lim d (nk 1 1) / d (nk ) 2 hay khng. Ta cng
k

cha bit cc s d n 1 d n c to thnh tp tr mt trong tp cc s thc dng hay khng.


Tuy nhin P.Erdos chng minh rng tp hp tr mt trong mt khong khng tm thng
(Erdos [14] ghi ch (1)). Vi n 10000 ta c d n 64 v gi tr ln nht d n 64 nhn c
ch vi n 7560 v 9240. A.Schinzel [2] chng minh rng vi mi s t nhin h v m tn ti
s t nhin n h tha mn d (n) / d (n 1) m vi i 1, 2,..., h .
2. Cc tng d (1) d (2) ... d (n)
Vi s thc x 1 k hiu T x l tng
(6)

T x d k d 1 d 2 ... d x .
k 1

tnh tng ny u tin ta chng minh rng vi s t nhin k cho trc th d (k ) l s cc


nghim t nhin ca phng trnh
(7)

mn k

Tht vy nu s t nhin n l c s ca k , th m k n l s t nhin v cp m, n l nghim t


nhin ca (7). Ngc li nu cp cc s t nhin m, n tha mn (7) th n l c s ca k . Vy mi
c s t nhin ca k tng ng vi ng mt nghim ca phng tnh (7). Suy ra s d k bng
vi s cc nghim t nhin ca (7). H qu l t (6) suy ra T x chnh l s nghim t nhin ca
bt ng thc mn x . Bt ng thc ny tng ng vi
(8)

mn x.

Tt c cc nghim t nhin ca (8) c chia thnh cc lp trong nghim m, n c gi l


thuc lp n . K hiu k n l s nghim thuc lp n th r rng
(9)

T x k1 k2 k3 ...

By gi ta tnh s cc nghim thuc lp n .


Vi n cho trc th m ch c th nhn cc gi tr t nhin tha mn (8), tc l m

x
c th l 1, 2,..., , c
n

x
x
n s nh vy suy ra kn n , v t (9) ta c

x
. Do m ch
n

104 | Cc tng

d (1) d (2) ... d (n)


x x x
T x ...
1 2 3

(10)

V phi khng phi l tng v hn v ch c x phn t u tin ca n l khc 0 .


V vy (10) c th vit thnh
x

x
T x .
k 1 k

(11)

Cc tnh ton T x da theo (11) ch thun tin khi ta mun tm cc gi tr lin tip ca d k ,
nhng ni chung l khng tt vi x ln. Chng hn tnh T 100 ta cn cng ln lt hn mt
trm s. Ta s tm mt cng thc thun tin hn cho T x . u tin ta chia lp cc nghim t
nhin ca (8) thnh hai lp m lp u tin cha cc nghim vi n x v lp kia cha cc
nghim cn li, ngha l n x . Ta tnh s cc nghim trong mi lp. Nu n nhn gi tr t nhin

x v nu m, n l nghim t nhin ca (8) ngha l nu m l s t nhin tha mn m x n , th


x
m, n thuc lp th nht. Do vi mi s t nhin n x s cc nghim ca lp th nht l .
n
Do n nhn cc gi tr 1, 2,..., x , nn s nghim thuc lp th nht l

n .

Ta tnh s

n 1

nghim thuc lp th hai. Ngha l s cc cp s t nhin m, n tha mn mn x v n x , hay


(12)

x n

x
.
m

Nu m x , th x m x v cc bt ng thc (12) khng tha mn vi mi n . Theo k hiu

m l s t nhin c nh x . tm tt c cc gi tr dng ca n m (12) tha mn th ta ch


x
cn loi ra khi cc s t nhin n
(c x s nh vy) cc s n khng tha mn x n,
m
m
x
ngha l s cc ch s n m n x (c x s nh vy). Vy x l s cc cp m, n vi

m

m x tha mn (12). Nhng m ch nhn cc gi tr 1, 2,..., x , nn s cc nghim thuc lp th


x

2
x
x
hai l x x . Hng t th hai trong v phi bng vi x v

m 1 m
m1 m m1
n l tng ca x hng t v mi hng t u bng x . Do s nghim trong lp th hai l

x 2

x . V vy
m 1 m

T x . Ta c

x x 2

m x l s nghim t nhin ca (8), tc l gi tr ca


n 1 n
m 1

x
x x v c hai tng u bng x x

v vy

n
1 2 ... ;
m 1 m
n 1
x

2
x
T x 2 x .
n 1 n
Cng thc ny c tm ra bi Lejeune Dirichlet. Ta tnh T 100 nh sau

(13)

CHNG 4. S CC C S V TNG CA CHNG | 105


10
100
T 100 2
102 2(100 50 33 25 20

n 1 n
16 14 12 11 10) 100 2 291 100 482.

Tng t ta tnh c T 200 1098, T 500 3190, T 1000 7069. Vi cc tnh ton di hn ta
tm c T 5000 43376, T 10000 93668. T (11) ta c th nhn c mt xp x trung bnh

cho hm d n . Trong v phi ca (11) ta thay bi , khi cc sai s trong mi hng t l


k
k
nh hn 1 v v vy sai s ca c tng l nh hn s hng t, ngha l nh hn
x

x x. Do

T x c xp x bI

n , vi sai s nh hn x. Vi cc gi tr l s t nhin x k ta c

(14)

d 1 d 2 ... d k 1 1
1
... ,
k
1 2
k

n 1

vi sai s nh hn 1. Do v phi ca (14) tng ti v hn theo k , nn t s gia v tri v v phi


1 1
1
tin ti 1. Ta bit tng ... c th xp x bi log k , vi sai s nh hn 1 vi k 1. Do
1 2
k

1 1
1
... log k tin ti gii hn l hng s Euler
1 2
k
C 0.57721566... (ta cha bit s ny c phi l s v t hay khng). T y kt hp vi cng thc
(13) ta tm c xp x x log x 2C 1 x ca T x vi sai s nh hn bi s hu hn ca x .
log k l xp x ca v tri (14). Hiu

G.Voronoi chng minh rng sai s ny l khng ln hn bi s hu hn ca


gi khc tm ra nh gi chnh xc ca sai s ny (Kolesnik [1]).

x log x. Mt s tc

3. Cc chui vi cc h s d n
Trong gii tch, cc hm d n xut hin nh l h s ca cc chui v hn.
Chng hn xt cc chui Lambert (hi t vi x 1 )

xk
x
x2
x3

...

k
1 x 1 x 2 1 x3
k 1 1 x

Khai trin mi hng t ca chui thnh chui ly tha


chui lp

kl

xk
x k x 2 k x3k ... ta nhn c
k
1 x

m vi mi s t nhin n th ly tha x n xut hin vi s ln ng bng s

k 1 l 1

nghim t nhin ca phng trnh kl n , ngha l d n ln.


V vy vi x 1 ta c

xk

d ( n) x n .

k
1

x
k 1
n 1

Ta thy d n l h s ca x n trong biu din ca chui Lambert thnh chui ly tha.


Hm d n cng l h s trong biu din ca cc hm .

1 1 1 1
s s s ... (chui ny hi t vi s 1 ).
s
1 2 3
k 1 k

Vi s 1 ta xt chui v hn ( s)

106 | Cc chui vi cc h s

d n . Tng cc c s

By gi ta p dng tch Dirichlet cho s s . Tch Dirichlet c cho nh sau: vi hai chui
a1 a2 ... v b1 b2 ..., ta nhn a1 a2 ... vi b1 b2 ..., v nhm cc tch ak bl vi cc b ch s

c tch bng nhau, ngha l a1 a2 ... b1 b2 ... a1b1 a1b2 a2b1 a1b3 a3b1 a1b4 a2b2 a4b1
a1b5 a5b1 a1b6 a2b3 a3b2 a6b1 a1b7 a7b1 ... T y ta c
(15)

d ( n)
s
n 1 n

( ( s)) 2

4. Tng cc c s
Tng cc c s t nhin ca s t nhin n k hiu l n . T nh l 1 suy ra nu (1) l phn
tch thnh tha s nguyn t ca n, th
(16)

n q1 q2 ...qk ,
1

Trong cc tng ly trn mi b k s nguyn (4) tha mn (3). Nhng mi hng t ca (16) u
xut hin trong khai trin tch 1 q1 q12 ... q11 1 q2 q22 ...q22 ... 1 qk qk2 ... qkk v

hn na xut hin ng mt ln. V vy


nh l 3. Tng n cc c s t nhin ca s t nhin n q1 q2 ...qk l
1

(17)

q11 1 1 q22 1 1 qkk 1 1

...
.
q1 1
q2 1
qk 1

23 1 53 1

7 31 217. T nh l 3 suy ra vi cc s t nhin nguyn t


2 1 5 1
cng nhau a, b ta c ab a b . Mt khc nu a, b 1, th ab a b .
c bit 100

S dng nh l 3 ta tnh c 1 1, 2 3, 3 4, 4 7, 5 6, 6 12, 7 8,


8 15, 9 13, 10 18. Vi n 1 th n n. Suy ra n 5 vi n 4. Vi n 4 th n

nhn cc gi tr 1,3, 4 v 7. Khng tn ti n m n 5.


nh l 4. Tn ti v hn s t nhin khng phi gi tr x vi mi s t nhin x.
Chng minh. Vi n l s t nhin 9 xt s t nhin k m
(18)

n
n
1 k
3
2

S cc s k nh vy l ln hn n 2 n 3 n 6. Theo (18) ta c
(19)

2k n v 3k 3 n,

V v n 9, ta c 3k 6, suy ra k 3. V vy 2k c t nht 4 c s phn bit l 1, 2, k , 2k. Do


2k 1 2 k 2k , v theo (19) suy ra 2k n. Do s cc s t nhin k m tha mn (18),(19)
v do 2k n l nhiu hn n 6, nn trong cc s 1 , 2 ,..., n c nhiu hn n / 6 s
ln hn n. V vy trong dy 1, 2,..., n c nhiu hn n 6 s t nhin khng l gi tr ca hm x
vi x n. Cc s ny cng khng th l gi tr x vi x n. v cc s ny n m

x 1 x n vi x n . Do vi mi cc s t nhin n 9 c nhiu hn n / 6 s t nhin


trong dy 1, 2,..., n khng l gi tr ca x vi mi s t nhin x.
Vy tn ti v hn s t nhin m m phng trnh x m l khng c nghim t nhin x.

CHNG 4. S CC C S V TNG CA CHNG | 107

C th chng mnh ng tt c cc s m 3k k 1 u c tnh cht ny (Sierpinski [27]). C ng 59


s m 100 nh vy. l 2, 5, 9, 10, 11, 17, 19, 21, 22, 23, 25, 26, 27, 29, 33, 34, 35, 37, 41, 43, 45,
46, 47, 49, 50, 51, 52, 53, 55, 58, 59, 61, 64, 65, 66, 67, 69, 70, 71, 73, 75, 76, 77, 79, 81, 82, 83, 85,
86, 87, 88, 89, 92, 94, 95, 97, 99, 100.
Trong cc s m 100 c ng 25 s m phng trnh x m c duy nht nghim. l m
1,3,4,6,7,8,13,14,15,20,28,30,36,40,44,57,62,63,68,74,78,91,93. Mt cu hi t ra l c phi tn
ti v hn cc s t nhin m m phng trnh x m c duy nht nghim hay khng.
Chng minh di y cho ta kt qu tng qut hn. P.Erdos [14] (trang 12) chng minh rng
vi mi s k cho trc tn ti m m phng trnh x m c ng k nghim th tn ti v hn
cc s m nh vy. Tuy nhin tn ti v hn s t nhin m m phng trnh

x m c nhiu

hn 1 nghim. Chng hn vi cc s m 3 5k 1 , k 1, 2,... , th ta c 6 11 12 v

6,5 11,5 1, nn 6 5k 1 11 5k 1 3 5k 1.
k 1

k 1

D dng chng minh tn ti v hn cc s t nhin m m phng trnh x m c nhiu hn 2


nghim. Chng hn vi cc s 2 13k 1 , vi k 1, 2,... th ta c 14 13k 1 15 13k 1

23 13k 1 2 13k 1 .

Ta vn cha bit c phi vi mi s t nhin k tn ti s t nhin mk m phng trnh x mk


c ng k nghim t nhin x hay khng. Kt lun ny cng c suy ra t gi thuyt H (Schinzel
[13]). C th chng minh rng nu mk l s nh nht m x mk c ng k nghim th
m1 1, m2 12, m3 24, m4 96, m5 72, m6 168, m7 240, m8 432,

m9 360, m10 504, m11 576, m12 1512, m13 1080, m14 1008, m15 720, m16 2304, m17 3600,
m18 5376, m19 2160, m20 1440.

Phng trnh ( x) m c ng 3 nghim t nhin vi 6 trng hp ca m 100, l cc s


24,42,48,60,84,90. Vi m 100 phng trnh ( x) m c ng 4 nghim ch khi m 96. Cng
vy vi m 100 th phng trnh c ng 5 nghim ch khi m 72 . Khng tn ti m 100 m
phng trnh c nhiu hn 5 nghim t nhin. Tuy nhin H.J. Kanold [2] chng minh vi mi s
t nhin k th tn ti s t nhin m m phng trnh ( x) m c k nghim t nhin x.
Phng trnh (n) (n 1) c ng 9 nghim n 10000. l n 14, 206, 957, 1334, 1364,
1634, 2685, 2974, 4364 (Makowski [4]). C ng 113 nghim n 107 (Hunsucker, Nebb v Stream
[1], Guy v Shanks [1]). Ta cha bit c tn ti v hn nghim nh vy hay khng.
A.Makowski t ra cu hi c phi vi mi s nguyn k th tn ti s t nhin n m
n 1 n k v tng qut hn vi mi s t nhin m v k nguyn tn ti s t nhin n
m n m n k.
Cc kt qu vi m 5 c trnh by bi Mientka v Vogt [1]. Nu n v n 2 l cc s nguyn
t sinh i th (n 2) (n) 2 . Phng trnh ny cng ng vi n 434 m 434 v 436 khng
phi s nguyn t. Tnh hung tng t xy ra vi n 8575 v n 8825.
Da vo gi thuyt Catalan ( chnh sa bi Dickson [3]) th nu f (n) (n) n th vi s t
nhin n 1 dy v hn n, f (n), ff (n), fff (n) hoc l dy tun hon hoc l dy dng ti phn t 1.
Kt qu ny ng vi mi n 275 (Devitt [1]). Theo L.Alaoglu v P.Erdos [2] th gi thuyt ny
khng nhng kh chng minh m cn kh kim tra trong cc trng hp c bit. Chng hn

108 | Cc chui vi cc h s

d n . Tng cc c s

vi n 276. Vi n 12496 24 11 71 th tt c cc s n, f (n), ff (n), fff (n), ffff (n) l phn bit
nhng fffff (n) n . Vi n 12 ta c f 12 16, f 16 15, f 15 9, f 9 4, f 4 3, f 3 1, chng
t dy dng. i vi s nguyn t n th f n 1. Vi n 100 ta c f 100 117, f 117 65,
f 65 19, f 19 1. Vi n 6, th f (n) n nn dy tun hon chu k mt phn t. Vi n 95 ta
c f 95 25, f 25 6, f 6 6 v dy tun hon t phn t th 4 vi chu k mt phn t. Vi
n 220 ta c f 220 284, f 284 220 n, v do dy tun hon ngay t phn t u tin vi

chu k hai phn t. Trong mt bn tho cha cng b P.Poulet [3] thng bo rng vi n 936
th dy 936,1794,2238,2250,,74,40,50,43,1 cha 189 phn t, s ln nht l 33289162091526.
Mt cu hi c t ra l c phi tn ti dy di ty m dng ti 1 v c phi tn ti v hn s
t nhin n m dy trn l tun hon. Cu tr li cho cu hi ny l khng nh nu gi thuyt ni
rng mi s chn ln 6 u l tng ca hai s nguyn t phn bit l ng. Tht vy, gi s gi
thuyt ng v k hiu 2k 1 l s l ty 7 . Khi 2k 2 6 v theo gi thuyt tn ti hai s
nguyn t l phn bit p v q , m 2k 2 p q . V vy f pq pq pq 1 p q 2k 1. V

p, q l cc s nguyn t l phn bit, gi s p q, v do p q 2 vi q 3. V vy


pq 3 p 2 p p 2 p 1 2 p q 1 2k 1 do pq 2k 1. Vy vi mi s l n 7 tn ti s
l m n m f m n. t m g n . Tn ti dy v hn tng g n , gg n ,... Nu vi s t nhin

k t n g k 11 ta nhn c dy n g k 11 , f n g k 1 11 ,... f k n 11, f 11 1. Do ta nhn


c dy gim n, f (n), ff (n),... cha k 2 phn t v phn t cui cng bng 1. Vi s t nhin k
t n g k 25 , ta c dy tun hon n g k 25 , f n g k 1 25 ,... f k n 25, f 25 6, f 6 6,6,...
vi k 1 phn t gim trong chu k tun hon.
Mt cu hi khc c t ra l c phi tn ti v hn cc s t nhin phn bit m dy
n, f (n), ff (n),... l tun hon m khng c phn t no xut hin trc chu k tun hon. Ta mi
ch tm ra cc chu k c di l 1,2,4,5 v 28 nh vy (H.Cohen [1]).
Ta va chng minh t gi thuyt ni rng mi s t nhin chn 6 l tng ca hai s nguyn t
phn bit suy ra mi s t nhin l 7 l phn t ca dy f n n 1, 2,... . Hn na
f 3 1, f 4 3, f 8 7. Mt khc d dng chng minh 5 khng xut hin trong dy

f n n 1, 2,... . Tht vy nu vi s t nhin n ta c f n n n 5 th n l hp s (bi v

1 1 0 v vi s nguyn t n, n n 1 ). Do n ab, vi 1 a b n. Khi v 1,b v n


l cc c s phn bit ca n ta c n 1 b n, suy ra 5 n n 1 b b, v b 5. Suy
ra n ab vi 1 a b 4. Nhng iu ny l v l v khng c cc s t nhin a, b c tnh cht
trn m n n 5 .
Nu khng s dng gi thuyt ni rng mi s t nhin 6 l tng ca hai s nguyn t phn bit
th ta cha chng minh c mi s l khc 5 l c dng n n n 1, 2,... . P.Erdos [17]
chng minh tn ti v hn s t nhin khng thuc dy ny.
C th chng minh tnh cht m | mn 1 ng vi mi s t nhin n khi v ch khi

m 3, 4,6,8,12 hoc 24 (Gupta [1]).


Ta cha bit c tn ti v hn s t nhin n m n l bnh phng hay khng. T gi thuyt H
(Chng 3 mc 8) suy ra cu tr li khng nh. Tht vy t f ( x) 2 x 2 1 th a thc f ( x) l
bt kh quy v v f (0) 1 nn n tha mn iu kin C trong Chng 3. Do theo gi thuyt H

CHNG 4. S CC C S V TNG CA CHNG | 109

suy ra tn ti v hn s t nhin x m p 2 x2 1 l s nguyn t 7 . Vi cc s ny ta c

(7 p) 8( p 1) (4 x)2 . Suy ra (7 p) l bnh phng ng.


Ta bit mt s nghim t nhin ca phng trnh ( x 2 ) y 2 chng hn x 7, y 20. Ta cng
bit mt s nghim t nhin ca phng trnh ( x 2 ) y 3 chng hn x 2 3 11 653, y 7 13 19.
Bi tp. 1. Chng minh rng n n 1 khi v ch khi n l s nguyn t.
Chng minh. Nu p l s nguyn t th n c 2 c s l p v 1. Do p p 1. Mt khc
nu n l hp s ngha l n ab, vi cc s t nhin a v b 1 th n c t nht ba c s t nhin
phn bit l 1, a v n . Do n 1 a n n 1. Cui cng nu n 1, th n 1 n 1.
2. Chng minh vi mi s t nhin m tn ti cc s t nhin x, y m x y m v ( x 2 ) ( y 2 ) .
Chng minh. Xt s t nhin n ty m m n,10 1. Vi x 5n, y 4n ta c x y n m v

x 2 y 2 31 n2 .

3. Tm tt c cc nghim t nhin m tng cc c s ca n l l.

Li gii. Gi s n l s t nhin m n l. t n 2 k , vi k l v l s nguyn khng m.

Ta c n 2 1 1 k v do k l. V k l nn cc c s ca n l l. M tng cc c s
ca n k l nn d k l s cc c s ca n cng l. V vy theo mc 1 suy ra k l bnh

phng ng, ngha l k m2 . V vy n 2 m2 . Nu chn ngha l 2 , th n 2 m .

2
Nu l th 2 1 v do n 2 2 m nn hoc n l 2 hoc n 2l , vi l l s t nhin.
2

2
Mt khc nu n l 2 hoc n 2l , vi l l s t nhin, n 2 q11 q22 ...qkk l phn tch thnh tha

s nguyn t ca n vi q1 , q2 ,..., qk l cc s nguyn t l. Ta c n 22 1 1 q12 ... qk2


1

hoc n 22 2 1 q12 ... qk2 . Nhng v qi2 1 qi qi2 ...qi2 , l tng ca l hng t
1

l nn n l. Do n l l khi v ch khi n l bnh phng ng hoc hai ln mt bnh


phng ng.
4. Chng minh rng nu n l hp s th n n n .

Chng minh. Hp s n c c s d tha mn 1 d n. V vy 1 n d n . Nu d n , th

d n n . Nhng v n d cng l c s ca n (khng nht thit khc d ) v 1 n d n. nn

n n n 1, suy ra n n n . iu phi chng minh.


Ghi ch. Ta c h qu lim pn 1 pn v lim pn pn 1 .
n x

n x

5. Chng minh vi s t nhin k 1 phng trnh n n k c s nghim l hu hn dng.


Chng minh. Nu n n k vi k l s t nhin 1 th n l hp s v theo bi tp 4 th

n n n . suy ra n k 2 . c bit phng trnh n n 2 v nghim v phng trnh

n n 3 c duy nht nghim n 4.


6. Chng minh rng lim
n x

n !
n!

110 | Cc chui vi cc h s

d n . Tng cc c s

Chng minh. D dng chng minh m m l tng ca cc nghch o cc c s ca m. v n !


nhn tt c cc s t nhin

lm c s nn

n !
1
1 1
lim ... , suy ra lim
.
n x
n x 1
n!
2
n

n ! 1 1

1
... .
n!
1 2
n

Nhng

7. L.Alaoglu v P.Erdos [1] gi cc s t nhin n l rt phong ph (superabundant) nu


n n k k vi mi k n. Chng minh rng tn ti v hn s nh vy.
Chng minh. t un n n vi n 1, 2,... t bi tp 6 suy ra dy u1 , u2 ,... khng c chn trn. V
vy chng minh bi ton ta ch cn chng minh nh l tng qut hn: mi dy v hn cc s
thc khng c chn trn cha v hn cc phn t ln hn tt c cc phn t ng trc n.
Tht vy gi s dy u1 , u2 ,... khng c chn trn khi ta c lim max u1 , u2 ..., un v vi mi
n

s t nhin m tn ti s t nhin l m m al max u1 , u2 ,..., ul max u1 , u2 ,..., um . Trong


dy u1 , u2 ,..., ul tn ti cc phn t khng bng al . Gi un l phn t u tin nh vy. Khi ta c

n m, n l v un uk vi k n. Ta chng minh vi mi s t nhin m th tn ti s t nhin

n m m un uk vi mi k n. iu phi chng minh.


8. A.K.Srinivasan [1] gi cc s t nhin n l s c s (practical number) nu mi s t nhin n
n 1
2n 1 l s c s.
l tng cc c s khc nhau ca n. Chng minh vi s t nhin n 1 th 2

n
Chng minh. Nu k l s t nhin 2 1, th ta bit k l tng cc s khc nhau trong dy

1, 2, 22 ,..., 2n1. Mt khc nu 2n 1 k 2n1 2n 1 , th k 2n 1 t r , vi t l s t nhin

2n1 v 0 r 2n 1, do t v r l cc tng cc phn t khc nhau trong dy 1, 2, 22 ,..., 2n1.


iu phi chng minh.
iu kin cn v mt s t nhin n l s c s c trong Sierpinski [16]. Stewart [2],
Margenstern [1]. S 10 khng phi s c s 100 v 1000 l s c s.
9. Tm s t nhin m m phng trnh x m c nhiu hn 1000 nghim.
Li gii. Ta s dng phng php c a ra bi S.Mazur. Gi s ta tm c s b ba cc s
nguyn t pi , qi , ri i 1, 2,..., s , m tt c 3s s ny u phn bit v hn na
(20)

pi 1 qi 1 ri 1,

i 1, 2,..., s.

t
(21)

ai pi qi , ai ri , i 1, 2,..., s.
0

Vi mi dy 1 , 2 ,..., s cha s s bng 0 hoc 1 ta t


(22)

n1 ,2 ,..,s a11 a22 ...as s .

V cc s pi , qi , ri i 1, 2,..., s , l cc s nguyn t phn bit, cc iu kin (21) v (22) suy ra


(23)

n ,
1

2 ,.., s

a a ... a .
1

CHNG 4. S CC C S V TNG CA CHNG | 111

Theo

(21)

ta

ai 0 pi 1 qi 1 , ai1 ri 1, i 1, 2,..., s,

theo

(20)

th

ai 0 ai1 ri , vi i 1, 2,..., s, do ai ri , i 1, 2,..., s, v vy ta thy cng thc

(23) suy ra n1 ,2 ,... s r1 r2 ... rs r1r2 ...rs vi tt c 2 s dy 1 , 2 ,... s .


Cc s n1 ,2 ,..., s , c 2 s s nh vy, u phn bit v theo (21) v (22) th phn tch thnh tha s
nguyn t ca chng l phn bit. Do ta nhn c 2 s s t nhin phn bit c chung tng cc
c s. Do tm chng hn 1024 s c tng cc c s bng nhau ta ch cn tm 10 b ba cc
s nguyn t pi , qi , ri i 1, 2,...,10 m 30 s l khc nhau v tha mn (20). D dng kim tra
cc b ba sau y tha mn cc iu kin cn thit
2,3,11;5,7,47;13,17,251;19,23,479;29,41,1259;31,83,2687;
43,71,3167;59,61,3719;53,101,5507;83,97,8231.
Suy ra vi m 12 48 252 480 1260 2688 3168 3720 5508 8232 th phng trnh x m c t nht
1024 nghim t nhin x.
5. Cc s hon ho
Tn ti v hn s t nhin n m tng cc c s ca n (khng tnh n ) l nh hn n . Chng hn
cc s nh vy l cc s nguyn t v ly tha ca chng. Cng tn ti v hn cc s t nhin n m
tng tng ng ln hn n . Chng hn cc s c dng n 2k 3, vi k 2,3,... Tuy nhin ta cha bit
c tn ti v hn cc s t nhin n m tng cc c s ca n (khng tnh n ) bng n hay khng.
Cc s c tnh cht ny c gi l cc s hon ho. C 30 s hon ho c tm ra. Tt c
chng u l s chn v ta cha bit c tn ti s hon ho l hay khng. Ta chng minh c
nu mt s hon ho l l tn ti th n phi ln hn 1050 (Buxton v Elmore [1] ch ra n cn
phi ln hn 10200 ) v c t nht 8 c s nguyn t phn bit (Hagis [1],[2]). S hon ho ln
nht c bit l s 2216090 2216091 1 . S ny c 130100 ch s. S hon ho nh nht l

6 1 2 3 v s tip theo l 28 1 2 4 7 14. Tng cc c s ca n (khng tnh n ) r rng


l

n n. Do mt s t nhin l hon ho khi v ch khi


n 2n.

(24)

s 1
2s 1 , vi s l s t nhin v
nh l 5. Mt s chn l s hon ho khi v ch khi n c dng 2

2s 1 l s nguyn t.
s 1
Chng minh. Gi n l s hon ho chn. Khi n 2 l , vi s 1 v l l s l. V vy

n 2s 1 l v theo (24) th 2s 1 l 2s l. V 2s 1, 2s 1, suy ra l 2s q, vi q

s
l s t nhin. V vy (2s 1) q l vi l 2 q, suy ra l l q. Nhng v

1 q l , ta

c q | l v q l (v s 1 ) do s l c t nht hai c s t nhin phn bit l q v l . Cng thc

l l q chng t n khng c nghim khc na. H qu l q 1 v l l s nguyn t. Nhng


l 2s 1 q 2s 1. Do n 2s 1 l 2s 1 2s 1 , v v vy 2s 1 l s nguyn t. iu kin cn

c chng minh. chng minh iu kin gi s 2s 1 l s nguyn t l. Hn na t


n 2s 1 2s 1 . Ta c n 2s 1 2s 1 2s 1 2s v 2s 1 l s nguyn t. Do

n 2n, suy ra n l s hon ho. iu kin c chng minh.

112 | Cc s hon ho. Cc s bn b. Tng 1 2 ... n

D dng chng minh nu 2s 1 l s nguyn t th s cng l s nguyn t. Tht vy nu s ab,

b 1 a
vi a v b l cc s t nhin 1 th 2s 1 2a 1 1 2a 22 a ... 2 , m a 2, nn v

2a 1 22 1 3, suy ra 2s 1 l hp s. nh l 5 suy ra h qu sau y


p
H qu. Mi s hon ho chn cho bi cng thc 2 p 1 2 p 1 , vi p v 2 1 l cc s nguyn t.

Cc s hon ho c nghin cu bi Euclid, ngi ra phng php sau tm nhng s


nh vy: ta tnh cc tng cc ly tha lin tip 1 2 4 8 16 32 ... . Nu tng tr thnh mt s
nguyn t th ta nhn n vi hng t cui cng v nhn c mt s hon ho.
S dng nh l 5 ta thy phng php ca Euclid thc s cho bit mi s hon ho chn.
By gi ta tnh mt s s hon ho chn. u tin ta xt p ln lt l cc s nguyn t tip theo
tnh t 2 v kim tra xem 2 p 1 c phi s nguyn t hay khng. Ta thy vi p 2,3,5,7 th
2 p 1 3,7,31,127

cc

nguyn

t.

Do

hon

ho

tin

2 2 1 6, 2 2 1 28, 2 2 1 496, 2 2 1 8128. vi p 11 th 2 1 23 89 l hp


2

11

s do ta khng c s hon ho tng ng.


T nh l 5 suy ra vic tm tt c cc s hon ho chn tng ng vi vic tm tt c cc s
Mersenne (l cc s nguyn t c dng 2s 1 ). Ta s nghin cu bi ton ny trong chng 10.
K hiu V x , x l s thc, l s cc s hon ho x.
B.Hornfeck v E.Wirsing [1] chng minh lim
x

ti s t nhin A m V x

log V x
0 v E.Wirsing [1] chng minh tn
log x

Ae A log x loglog x .

Ta khng bit c tn ti v hn s t nhin n m n | n , hoc c tn ti s t nhin l c tnh


cht ny hay khng. Ta chng minh c khng tn ti s t nhin l n nh th vi n 1050
(Beck v Najar [1]).
S t nhin n tha mn n mn, vi m l s t nhin 1 c gi l s hon ho Pm . Cc s
ny c nghin cu bi Mersenns, Fermat, Descartes, Legendre v mt s ngi khc. Cc s
hon ho P2 l s hon ho thng thng. P.Poulet [1] (trang 9-27) tm ra 334 s hon ho Pm
vi m 8. Nm 1953 B.Franqui v M.Garcia [1] tm ra 63 s na (Franqui v M.Garcia [2],
A.L.Brown [1],[2]). Cc s P3 c nghin cu bi R.Steurwald [1].
P.Cattaneo [1] gi mt s l gi hon ho nu n bng tng cc c s t nhin khng tm thng
ca n, ngha l cc c s khc 1 v chnh n. Theo s gi hon ho l cc s t nhin n m
n 2n 1. Ta cha bit c tn ti cc s nh vy hay khng. P.Hagis Jr. v G.Cohen [1]
chng minh nu tn ti cc s nh vy th chng u ln hn 1035 v c t nht 7 c s nguyn t
phn bit.
Tuy nhin d dng chng minh tn ti v hn s t nhin n m n 2n 1. Chng hn cc s

2k , k 0,1, 2,... c tnh cht ny. A.Makowski [5] nghin cu nghim t nhin ca phng trnh
n 2n 2 . ng ta lu rng nu 2k 3 l s nguyn t th n 2k 1 2k 3 l nghim cn tm.
Cc s 2k 3 l nguyn t vi cc gi tr k 24 : k 2,3, 4,5,6,9,10,12, 14, 20, 22. Phng trnh
ny cn c cc nghim khc chng hn n 650.
Phng trnh tng qut n kn a c nghin cu bi C.Pomerance [1] v A.Makowski [9].

CHNG 4. S CC C S V TNG CA CHNG | 113

Bi tp. 1. Chng minh rng tn ti v hn s t nhin l n m n 2n.


Chng minh. Cc s n 945m, vi m l s t nhin khng chia ht cho 2,3,5,7, tha mn iu
kin v 945 33 5 7, m,945 1 do n 945 m 945 m 1920m 2n. V m khng
chia ht cho 2 nn n l s l. C th chng minh 945 l s t nhin l nh nht m n 2n .
2. Tm tt c cc s t nhin n m n l tch ca tt c cc c s t nhin ca n tr ra n .
Li gii. K hiu Qn l tch tt c cc nghim t nhin ca n . Ta tm cc s t nhin n m

Qn n n, , ngha l vi n ta c Qn n2 . Nu d1 , d2 ,...d s l tt c cc c s t nhin ca n (c

s d n s nh vy) th cc s n d1 , n d2 ,... n d s cng l cc c s t nhin ca n . Suy ra

s 2
Qn d1 , d2 ,..., d s ns Qn , v do Qn n n

d n

. V Qn n2 , ta c n2 n

d n

, suy ra d n 4,

2
iu ngc li cng ng, ngha l nu d n 4, th Qn n , do mt s t nhin n bng tch

cc c s ca n tr ra n khi v ch khi n c ng 4 c s t nhin.


T cng thc tnh s c s ca mt s t nhin cho bi cng thc (5) vi (1) l phn tch thnh
tha s nguyn t ca n ta c 1 1 2 1 ... k 1 4 . Do cc s m 1 , 2 ,..., k l cc s t
nhin suy ra k 2, ngha l k 1 hoc k 2 . Nu k 1 th 1 1 4, suy ra 1 3 v n l lp
phng mt s nguyn t. Nu k 2 th 1 2 1 v n l tch ca hai s nguyn t. Vy mi s
t nhin cn tm l cc lp phng nguyn t v tch ca hai s nguyn t phn bit.
Cc s nh vy nh hn 30 l 6,8,10,14,15,21,22,26,27.
3. Chng minh cc nh l Descartes sau (th Descartes gi Mersenne ngy 15 thng 11/ 1638).
1. Nu n l s hon ho P3 v khng chia ht cho 3 th 3n l s hon ho P4 .
2. Nu n chia ht cho 3 nhng khng chia ht cho 5 v 9 v hn na n l s hon ho P3
th 45n l s hon ho P4 .
3. Nu n khng chia ht cho 3 v 3n l s hon ho P4k th n l s hon ho P3k .
Chng minh. 1. Nu n l s hon ho P3 th n 3n v nu n khng chia ht cho 3 th

3n 3 n 4 3n v h qu l 3n l s hon ho P4 .
2. Nu n l s hon ho P3 v n 3k , vi k khng chia ht cho 3 v 5 th

45n 33 5k 33 5 k 40 6 k . Nhng v n 3k v k khng chia ht cho 3


nn ta c n 3 k 4 k . H qu l 45n 60 4 k 60 n . V vy do n l s
hon ho P3 nn n 3n v ta c 45n 180n 4 45n, chng t 45n l s hon ho P4 .
3. Nu n khng chia ht cho 3 v 3n l s hon ho P4k th 3n 4k 3n , suy ra

3n 3 n 4 n v do n 3kn, chng t n l s hon ho P3k .


4. Chng minh rng 120 v 672 l cc s hon ho P3 . S 25 33 5 7 l s hon ho P4 v

27 34 5 7 112 17 19 l s hon ho P5 .
Li gii. Ta c cc phn tch thnh tha s nguyn t 120 23 3 5 v 672 25 33 7. C th
chng minh 120 l s hon ho P3 nh nht.
5. Chng minh rng nu n 5n, th n c nhiu hn 5 c s nguyn t phn bit.

114 | Cc s hon ho. Cc s bn b. Tng 1 2 ... n

Chng minh. Gi s (1) l phn tch thnh tha s nguyn t ca n . Khi theo (17) ta c
n

q11 1q22 1...qkk 1


q
q
q
1 2 ... k n.
q1 1 q2 1 ... qk 1 q1 1 q2 1 qk 1

2 3 5 7 11
77
n
n 5n mu thun vi n 5n.
1 2 4 6 10
16
6. nh l Mersenne: nu n khng chia ht cho 5 v l s hon ho P5 th 5n l s hon ho P6 .
Nu k 5, th ta c n

6. Cc s bn b
Hai s t nhin gi l s bn b nu mi s bng tng tt c cc c s ca s kia tr ra chnh s
. D dng thy hai s t nhin n, m l cc s bn b khi v ch khi m n m n. (1)
Cp s bn b u tin l 220 v 284 c tm ra bi Pythagoras. Cp 24 23 47 v 24.1151 c
tm ra bi Fermat (2). Cp 27 191 383 v 27 73727 tm ra bi Decartes. Euler tm ra 59 cp s
nh vy, trong c cc cp 23 17 79 , 23 23 59 v 23 19 41 , 25 199. E.J.Lee v J.S.Madachy [1]
trnh by mt danh sch 1107 cp s bn b tm c qua 25 thin nin k. Danh sch c hon
thin ti 108. Hn 5000 cp s b bn c xy dng bi W.Borho, H.Hoffman v H.J.J te Riele (te
Riele [1],[2]). Ta bit tn ti cp s bn b cng l chng hn 33 5 7 11,3 5 7 139. Nhng ta cha
bit c tn ti cp s bn b khc tnh chn l hay khng. Ta cng cha bit c tn ti v hn cp
s bn b hay khng. nh ngha ca cp s bn b c m rng cho b k s bn b. nh ngha
ny c trnh by bi L.E.Dickson, ngi gi b k s t nhin n1 , n2 ,...nk l b k s bn b nu

n1 n2 ... nk n1 n2 ... nk (Dickson [2], Mason [1]).

A.Makowski

[4]

tm

ra

ba

bn

22 32 5 11, 25 32 7, 22 32 71 v

23 3 5 13, 22 3 5 29, 22 3 5 29 (trong b s th hai c 2 s bng nhau). Tn ti b ba m c ba


s l bng nhau chng hn n1 n2 n3 120.
nh ngha khc ca b k s bn b c cho bi B.F.Yanney [1]: b k s t nhin n1 , n2 ,...nk gi l
b k s bn b nu n1 n2 ... nk (ni ) (n1 ) (n2 ) ... ( nk ) vi i 1, 2,..., k . iu kin
ny tng ng vi iu kin n1 n2 ... nk (k 1) (ni ) vi i 1, 2,..., k .
Vi k 2 cc nh ngha u quy v trng hp nh ngha cp cc s bn b. Vi k 2, cc nh
ngha ny khng trng nhau. Chng hn mt b ba s bn b theo nh ngha ca Yanney l
308,455,581. Ta c 308 22 7 11, 455 5 7 13,581 7 83, do n1 n2 n3 672 v

n1 n2 n3 1344 2 672.
Ta cha bit c cc s nguyn t cng nhau no l s bn b hay khng. H.J.Kanold [1] chng
minh nu cp s bn b m1 , m2 tha mn m1 , m2 nguyn t cng nhau th mi s u ln hn 1023
v s m1m2 c nhiu hn 20 c s nguyn t phn bit. P.Erdos [13] chng minh rng nu

A x l s cc cp s bn b x, th lim A x x 0. C.Pomrance [2] chng minh rng nu


x

A x l s cc cp s bn b x, th vi x ln A x x exp log x 1 3 .
7. Tng 1 2 ... n
Trong mc ny ta xt cng thc tng

(1)
(2)

Ghi ch ca ban bin tp. hu ht cc tc gi u gi thit n m.


Theo W.Borcho [1] th cc s ny c tm ra bi Ibn Al Banna (1256-1321).

CHNG 4. S CC C S V TNG CA CHNG | 115

S x 1 2 ... x ,

(25)

vi x l s thc 1. Gi s n l s t nhin. S n l phn t ca tng k khi v ch khi n l


c s ca k . Do tnh s cc hng t k xut hin trong tng S x m n xut hin nh l
mt hng t th ch cn tnh s cc s k x m chia ht cho n. Nhng cc s k nh vy c dng
k nl x, vi l l s t nhin m l x n . R rng c [x/n] s nh vy. Do s t nhin n l
hng t ca tng k ng vi [x/n] s t nhin phn bit k x. T y suy ra
x

x
S x n .
n 1 n

(26)

C mt phng php khc tnh (25). S k c th xt nh l tng cc s t nhin n tha


mn phng trnh mn k , vi m l s t nhin. Do (25) l tng cc s n m tn ti cc s t
x
nhin m tha mn mn x . Nh th vi cc s c nh m v n thuc 1, 2,3,..., , th tng ca
m

x 1x 1x
chng bng 1 2 ... . H qu l nu ta cho m nhn mi gi tr dng m
m 2 m 2 m
mn x th tng tt c cc s n , ngha l S x , bng vi
S x

(27)

1 x 1 x

.

2 m1 m 2 m1 m
2

So snh (26) v (27) ta nhn c


x



x 1 x 1 x
n

2 m 2 m . R rng cng thc ny c th


n 1 n
m 1
m 1
x


x
x
vit li di dng 2n 1 . Tuy nhin cc cng thc (26) v (27) u khng ng
n
n 1 n
n 1
dng c tnh cc gi tr ca tng S x vi s cho trc x. Cng thc thch hp hn c
tm ra theo cch tng t cng thc (13)
2

x
x
2
3
2
1 x
x
S x 2n 1 x x .

2 n 1 n
n
n 1

Chng hn s dng cng thc ny ta tnh c S 100 8249. By gi nu trong (28) ta b i k

(28)

hiu [] v thay tng

1 n2 bi tng ca chui v hn
n 1

1 n

2 6, sau mi ln tnh th sai s

n 1

gim, th ta nhn c x 12 l xp x ca tng S x , vi sai s khng ln hn Ax x , vi A


2 2

l hng s dng c lp vi x.
8. Cc chui vi h s n
Hm n (tng t d n ; mc 3) xut hin nh l h s ca cc chui v hn. Ta c chui lp

(29)

kx
k 1 l 1

kl

116 | Cc chui vi h s

hi t tuyt i vi x 1. rt gn chui ny thnh chui ly tha th vi s c nh n ta nhm


cc hng t m x n xut hin. Khi cc h s ca cc hng t trong nhm th n l nhn t ca
tch n kl. Do (29) tr thnh tng

( n) x

n 1

Mt khc v

kxkl kxk 1 xk , ta thy (29) bng vi tng


l 1

x
kx k
V vy ta nhn c cng thc

n xn ,

k
k 1 1 x
n 1

kx

kl

kx k / (1 x k ) .

l 1

x 1.

V (29) hi t tuyt i vi x 1, ta c th hon v cc phn t ca chui, p dng ng thc

kxkl xl 1 xl vi x 1, ta nhn c cng thc


2

k 1

l 1

xl

1 x

l 2

n xn ,

x 1.

n 1

Trong mc 3 ta gii thiu tch Dirichlet ca hai chui v hn a1 a2 ... v b1 b2 ... .


s 1

Ta s dng tch vi ak 1 k , b1 1 l , k v l l cc s t nhin v s l s thc 2 . Ta c


1 1
k
ak bl s 1 s
. By gi nhm cc tch ak b1 m kl bng s t nhin cho trc n, ta thy
s
k
l
kl
s

s
cc t s bng vi cc c s t nhin k ca n; tng ca chng l n n . V vy

( n)

n 1

ns

( s 1) ( s)

vi s 2 .

9. Tng ca cc hng t xc nh bi cc c s t nhin ca mt s t nhin n


Xt f (n) l hm ty xc nh vi mi s t nhin n. Nu d1 , d2 ,...d s l tt c cc c s t nhin
ca n th tng f d1 f d2 ... f d s k hiu bi

f d

l tng cc hng t f d vi d

d n

nhn mi gi tr l cc c s t nhin ca n . Chng hn

1 d n , d n
d |n

d n .

Vi hm

f n

cho trc ta t

n 1

n 1 d |n

F n f d . Ta s tnh tng
d |n

d |n

nhng

d |n

F x f d vi s thc

x 1. Tng v phi bao gm cc hng t f k , vi k l s

t nhin x. Vi s t nhin cho trc k x th hng t f k xut hin trong tng

f d khi
d |n

v ch khi k l c s ca n. R rng n xut hin nhiu nht mt ln. S cc s t nhin n x


x
x
nh vy l . T s hng t f k trong tng lp l suy ra
k
k
(30)


x
F
n

f k

k
n 1
k 1
x

s
c bit nu f n n vi s l s nguyn c nh th F n l tng cc ly tha bc s ca cc c

s t nhin ca s t nhin n. Tng ny thng c k hiu l s n . Cng thc (30) suy ra

CHNG 4. S CC C S V TNG CA CHNG | 117


x

n 1

k 1


s n k s k . Ta c ngay 0 n d n ,1 n n vi n 1, 2,... v ta thy cc cng
x

thc (11) v (26) l cc trng hp ring ca cng thc ny.


10. Hm Mobius
Hm Mobius l cc hm s hc n xc nh bi cc iu kin

10. (1) 1
20. (n) 1 nu s t nhin n chia ht cho bnh phng mt s t nhin 1
30. (n) (1)k nu s t nhin n l tch ca k s nguyn t phn bit
Theo 1 1, 2 3 1, 4 0, 5 1, 6 1, 7 1, 8 9 0, 10 1
Ta trnh by mt tnh cht ca hm n : vi n l s t nhin 1 m phn tch thnh tha s
nguyn t ca n cho bi (1) th vi s l s nguyn cho trc xt tch

1 q 1 q ...1 q ,

(31)

s
1

s
2

s
k

Khai trin ca tch (31) cha hng t 1 v cc s d s vi d l c s ca n l tch cc c


nguyn t khc nhau. Cc du + v ca mi s tng ng vi vic trong tch c chn hay l
s
tha s nguyn t. Theo tnh cht 30 ta thy cc h s ca d s l bng d . Ch 1 1 1
v gi thit d bng 1 hoc l tch cc s nguyn t phn bit (tong ng vi vic n khng c
c s l bnh phng mt s t nhin 1 ) th theo tnh cht 20 ta thy tch (31) bng tng
d d s . Ngha l 1 q1s 1 q2s ...1 qks d .d s , suy ra vi s 0 ta c
d |n

d |n

d 0

(32)

d |n

vi mi s t nhin n 1 . R rng vi n 1, ta c

d 1 1.

Ta thy nu

d |1

F n d th F 1 1 v F n 0 vi cc s t nhin n 1 . H qu l (30) suy ra


d |n

(33)

k k 1 vi x 1.
k 1

Cc bt ng thc 0 t t 1 ng vi mi s thc t v v | (k ) | 1 vi cc s t nhin k ta

x
x

cc k hiu [] trong cc hng t ca (33) th sai s nhn c l nh hn 1 v hng t th nht
thy (k ) ( k ) 1 vi mi s thc x 1 v k l s t nhin. T y suy ra nu ta b i
k
k
ng bng vi x x . M c tt c

x 1

hng t tr ra hng t u tin nn ta c

x
x
(k )
(k )
x
(k ) x
x [x] [x] 1 x 1 suy ra theo (33) th 1 x
x 1 suy ra

k
k
k
k 1
k 1
k 1
[x]

x k / k x do
k 1

(34)

k / k 1. Chng t mi tng ring ca chui v hn


k 1

1 2 3
1

...

118 | Tng ca cc hng t xc nh bi cc c s t nhin ca mt s t nhin

n . Hm Mobius

c gi tr 1 . H. von Mangoldt vo nm 1897 chng minh tng (34) bng 0. Euler t ra gi

(k )
1
thuyt ny vo nm 1748. By gi ta p dng tch Dirichlet cho cc chui s v s vi s
k 1 k
l 1 l

(k )
1
l s t nhin 1 . Do 1 1 v t cng thc (32) ta nhn c s . s 1 ngha l cng
k 1 k
l 1 l

(k )
1
2
thc s
vi s l s thc 1 . c bit 2 / 6 , ng thc cui cng suy ra
( s)
k 1 k

k 1

(k )
k

. Trong mi lin h ny ta thy d dng chng minh ng thc

k 1

vi s l s thc 1 . Rt gn chui lp

2 k
k

s
2s

k x

kl

thnh chui ly tha bng phng php

k 1 l 1

tng t nh i vi chui (29), vi x 1 ta nhn c cng thc

( n) x n

1 x
n 1

x.

nh l 6. Vi mi hm s hc F n tn ti duy nht mt hm s hc f n m vi mi s t nhin

n ta c
(35)

F n f d
d |n

Chng minh. Nu vi n 1, 2,..., cng thc (35) ng th dy v hn cc ng thc sau l ng


F 1 f 1 ,
F 2 f 1 f

(36)

F 3 f 1 f
F 4 f 1 f
F 5 f 1 f
F 6 f 1 f

2 ,
3 ,
2
5 ,
2

4 ,

f (6)

.....................................

ng thc u tin cho f 1 F 1 . Do f

c th tnh da vo ng thc th hai. Khi

v f 1 v f 2 c tnh nn f 3 tnh c da vo ng thc th ba v c nh vy.


ng thc th n cho gi tr ca f (n) da vo cc gi tr f (k ) vi k n c tnh. Do tn
ti hm tha mn (35) suy ra tn ti duy nht mt hm nh vy. Mt khc d thy t cc tnh ton
gi tr f 1 , f 2 , t (36) ta nhn c hm f n tha mn (36) v do tha mn (35).
nh l c chng minh.
Cc phng trnh (36) cho ta cch tnh f (n) da vo F 1 , F 2 ,, F n .
Cng thc tng qut hn l
(37)

n
f n d F
d
d |n

Hoc c th vit di dng


(38)

n
f n F d
d
d |n

Hoc
(39)

f n k F l ,
kl n

CHNG 4. S CC C S V TNG CA CHNG | 119

Trong tng c ly vi mi cp k , l cc s t nhin m kl n . chng minh cc cng thc


ny ch cn chng minh hm xc nh bi (39) tha mn (35) vi mi s t nhin n. Tht vy t
(39) suy ra f d k F l k F l F l k F n v theo tnh cht
d |n kl d

d |n

l |n

lk |n

k |n / l

k l khc 0 (do bng 1 ) ch khi n / l 1 ngha l l n . c bit vi F 1 1

ca ta c

k |n / l

v F n 0 , n 2,3, , nh l 6 suy ra tn ti duy nht hm f , gi l hm Mobius, n f n ,


m f (1) 1 ,

f (d ) 0 vi n 2,3,
d |n

11. Hm Liouville n
Hm Liouville l hm s hc xc nh bi cc iu kin

10 . (1) 1
1 2 ... k

20 . (n) (1)

nu n c phn tch thnh tha s nguyn t dng (1)

Ta c 1 1, 2 3 1, 4 1, 5 1, 6 1, 7 8 1, 9 10 1 .
Gi s vi s t nhin n 1 c phn tch thnh tha s nguyn t dng (1).
Xt tch

1 q
k

s
i

t 1

qi2 s qi3s ... 1 i qii s

vi s l s nguyn ty . Khai trin tch ta nhn c

tng i s ca cc hng t q11 q22 ...qkk vi h s tng ng 1 1

... 2

q11 q22 ...qkk trong

tng ly trn tp mi cc c s d q1 q2 ...qk ca n . H qu l tch bng

d d

d |n

Mt khc ta c cng thc tng chui ly tha 1 q q q ... 1 qi


s
i

2s
i

3s
i

i s

1 1 i qi
s dng cng thc ny cho cc nhn t ca tch ta c
1 qis
i 1
k

i 1 s

1 1 i qi

1 qis

i 1 s

d d s . c bit
d |n

vi s 0

1 1 1 1 1 2 1 1
.
.
2
2
2

(40)
1 1
2

d .
d |n

bng 0 hoc 1 ty thuc l hay chn. Do v tri ca (40) khc 0 (do bng 1 ) khi

v ch khi tt c cc ly tha 1 , 2 ,... k , l chn, ngha l n l bnh phng mt s t nhin. Ta c


nh l
nh l 7. Tng

(d ) bng 0 trong trng hp n l bnh phng mt s t nhin hoc bng 1.


d |n

Trong chng minh nh l 7 ta gi s n 1 nhng nh l vn ng vi n 1 v 1 1 . t

F n d h qu l F n 1 ng vi mi n l bnh phng mt s t nhin v F n 0


d |n

x
trong cc trng hp khc. Theo (30) (vi f n k ) ta nhn c k F n
k n 1
k 1

120 | Hm Liouville

vi mi x 1. Tng trong v phi cha s cc hng t bng 1 ng bng s cc t nhin x m l


bnh phng. Do tng ny bng x . V vy

k k
k 1

x vi x 1.

CHNG 5
NG D
1. ng d v cc tnh cht
Gi s a v b l cc s nguyn. Ta ni rng a ng d vi b theo modulo m nu hiu ca a v b
chia ht cho m . S dng k hiu c xut bi Gauss ta vit
(1)

a b mod m

Cng thc (1) tng ng vi m | a b


R rng nu hai s nguyn l ng d modulo m th chng c cng s d khi chia cho m .
K hiu ng d c s dng kh ging k hiu
Ta lit k di y mt s mi lin h gia cc ng d thc v cc ng thc

I . Tnh phn x: mi s nguyn l ng d vi chnh n theo mi modulo, ngha l a a mod m


vi mi s nguyn a v mi s t nhin m v r rng a a 0 chia ht cho mi s t nhin m .
II . Tnh i xng: ng d thc (1) tng ng vi ng d thc b a (mod m ) v r rng cc
s a b v b a cng chia ht hoc cng khng chia ht cho m

III . Tnh kt hp: nu a b mod m v b c mod m th a c mod m v ta c ng thc


a c a b b c v lu tng ca hai s chia ht cho m l mt s chia ht cho m .
Ta c mt s tnh cht khc ca ng d sau y.
Ta chng minh hai ng d thc vi cng modulo c th cng hoc tr tng ng cc v. Tht vy
gi s
(2)

a b mod m v c d modm .

chng minh a c b d (mod m ) v a c b d (mod m ) ta lu cc ng thc


a c b d a b c d v a c b d a b c d . Tng t s dng ng
thc ac bd a b c c d b, ta chng minh c t (2) suy ra ng d thc

ac bd mod m . H qu l ta c th nhn theo v hai ng d thc vi cng modulo.


Cc nh l v cc tnh cht cng, tr, nhn cc ng d thc trn c th m rng cho hu hn
cc ng d thc.
nh l v php cng cc ng d thc chng t ta c th chuyn v i du mi hng t trong
mt ng d thc bi v php ton ny tng ng vi vic tr cc hng t trong c hai v.
T tnh cht php nhn cc ng d thc chng t c th nhn mt ng d thc vi mi s
nguyn ty v do ta c th ly tha c hai v ca ng d thc vi s m bt k.
Tuy nhin ta khng th chia mt ng d thc cho mt ng d thc khc (ngay c khi thng s
l cc s nguyn). Chng hn 48 18 mod10 v 12 2 mod10 nhng khng c 4 9 mod10 .
Do mt c s ca c s ca mt s nguyn cng l c s ca s nguyn nn nu d | m th
ng d thc a b mod m chng t a b mod d .
Tnh kt hp ca cc ng d thc cng vi tnh cht cng v nhn cc ng d thc chng t
trong ng d thc cho trc ta c th thay mi hng t hoc nhn t bi cc s ng d vi n.

122 | ng d v cc tnh cht

Quy tc ny khng ng vi cc ly tha. Chng hn ng thc thc 26 4 mod 5 khng th


thay th bi ng d thc 21 4 mod 5 mc d 6 1 mod 5 .
Gi s f x A0 x n A1 x n1 ... An1 An l a thc bc n vi h s nguyn. K hiu m l s t
nhin v a, b l cc s nguyn tha mn a b mod m . T nh l v cc ly tha t nhin v tnh
cht nhn cc ng d thc suy ra

A0 a n A0bn mod m ,
n 1
A1a n1 Ab
mod m ,
1

An1a An1b mod m ,

An An mod m .
n 1
... An1b An mod m . Ngha l
Cng li ta c A0 a n A1a n1 ... An1a An A0bn Ab
1

f a f b mod m . Ta chng minh c


nh l 1. Nu f x l a thc mt bin x vi h s nguyn th ng d thc a b mod m suy ra
ng d thc f a f b mod m .
nh l 1 cho ta quy tc cho bit mt s c chia ht cho 9,7,11,13,27,37 hay khng.
K hiu N l s t nhin. Biu din ca N trong h thp phn c cho bi biu din c dng
N c110n1 c210n2 ... cn110 cn . t
(3)

f x c1 x n1 c2 x n2 ... cn1x cn

Khi f x l a thc h s nguyn v


(4)

f 10 N

Theo nh l 1 th v 10 1 mod 9 ta c
(5)

f 10 f 1 mod 9 .

Nhng f 1 c1 c2 ... cn v h qu l theo (4) v (5) th N c1 c2 ... cn mod 9 chng t


mi s t nhin N sai khc tng cc ch s ca n trong h c s 10 mt bi s ca 9. Do N
chia ht cho 9 khi v ch khi tng cc ch s ca n chia ht cho 9. Tng qut hn nu k hiu S N l
tng cc ch s ca N (trong h thp phn) th vi cc s t nhin N v N ' ta c N sN mod 9 ,

N ' sN ' mod 9 suy ra NN ' sN sN ' mod 9 . V NN ' sN N ' mod 9 nn sNN ' sN sN ' mod 9 .
T (3) v ng d thc 10 1 mod 11 , nh l 1 suy ra f 10 f 1 mod 11 , do theo
(4) v (3) ta c 1

n 1

N c1 c2 c3 c4 ... mod11 . T y ta nhn c quy tc v tnh chia

ht cho 11. By gi ta tm cc quy tc v tnh chia ht cho 7 hoc 13.


K hiu c1 , c2 ,..., cn 10 l s trong h thp phn c cc ch s l c1 , c2 ,..., cn (k hiu ny l cn
thit phn bit vi tch cc ch s c1 , c2 ,..., cn ). Mi s t nhin u c dng

N cn2cn1cn 10 cn5cn4cn3 10 .1000 cn8cn7cn6 10 .10002 ... .

1000 1 mod 7

CHNG 5. NG D | 123

1000 1 mod 13 nn ta c N cn2cn1cn 10 cn5cn6cn7 10 cn8cn7cn6 10 ... mod 7 v


ng d thc tng t cng nhn c khi thay modulo 7 bi modulo 13.
Cc ng d thc ny cng cho ta bit quy tc v tnh chia ht cho 7 hoc 13. Chng hn ta c
N 8589879056 56 879 589 8 theo c modulo 7 v modulo 13. V s trong v phi cc ng
d thc ny (bng -242) l khng chia ht cho c 7 ln 13 nn N khng chia ht cho c 7 ln 13.
Cc quy tc vi 27 v 37 c da trn tnh cht 100 1 theo mod 27 v mod37 . T y ta nhn
c cc quy tc tng t vi cc trng hp trn. Chng hn ta c
N 24540509 509 540 24 theo mod 27 v mod37 . S trong v phi l 1073 c th vit
thnh 1073 73 1 theo mod 27 v mod37 . S 74 chia ht cho 37 nhng khng chia ht cho 27
v do s N cng chia ht cho 37 nhng khng chia ht cho 27.
Bi tp. 1. Tm hai ch s tn cng ca s 21000 .
Li gii. Ta c 210 1024 24 mod100 v vy 220 242 76(mod100) . Nhng 762 76 mod100
suy ra theo quy np 76k 76 mod100 , k 1, 2,... . Do 21000 220.50 7650 76 mod100 . Vy
hai ch s tn cng ca 21000 l 7 v 6.
2. Chng minh rng t nht mt trong su ng d thc sau l ng (Erdos [10]): 1) x 0 mod 2 ,
2) x 0 mod 3 , 3) x 1 mod 4 , 4) x 3 mod8 , 5) x 7 mod12 , 6) x 23 mod 24 vi s
nguyn x bt k.
Chng minh. Nu s nguyn x khng tha mn c 1) v 2) th n khng chia ht cho 2 v 3 do
c dng 24t r vi t l s nguyn v r l mt trong cc s 1,5,7,11,13,17,19,23. Khi d dng
kim tra s x 24t r tha mn cc ng d thc 3),3),5),4),3),3),4),6) tng ng.
Ghi ch. P.Erdos [10] t ra bi ton sau y: cho trc s t nhin n , c tn ti hay khng tp
hp hu hn cc ng d thc vi modulo phn bit ln hn n m mi s nguyn tha mn t
nht mt trong s chng? H.Davenport [2] t ra gi thuyt rng cu tr li l khng nh nhng
s khng c mt li gii n gin. P.Erdos t chng minh gi thuyt ny trong trng hp
n 2 bng cch s dng mt s ng d thc vi modulo l c s ca 120. D.Swift cho li
gii vi n 3 bng cch s dng cc ng d thc vi modulo l c s ca 2880. Gi thuyt
c chng minh vi mi n 20 (Choi [1]).
9

9
3. Tm hai ch s tn cng ca s 9 .
2
4
2
8
2
9
10
Li gii. Theo modulo 100 th 9 81,9 81 61,9 61 21,9 21.9 89,9 89.9 1. Ta
9
10
c 9 9 mod 10 suy ra 99 10k 9 vi k l s t nhin. V vy t 9 1 mod100 suy ra

9
99 910 k 9 99 89 mod 100 chng t ch s tn cng ca 9 l 9 v ch s lin trc l 8.
9

9
4. Tm hai ch s tn cng ca s 9 .

9
Li gii. T bi tp 3 suy ra 99 9 mod 10 do 9 10t 9 vi t l s t nhin. Suy ra
9

9
9
99 910t 9 99 89 mod 100 . V vy hai ch s tn cng ca 9 l hai ch s tn cng ca 9 .
9

Ghi ch. Theo W.Lietzmann ([1] trang 118) th s cc ch s ca s ny ln hn mt phn t ca


mt triu. Gauss gi s ny l s ln v hn.
2. Nghim ca cc ng d thc v h thng d y
K hiu f x l a thc bc n vi h s nguyn v m l modulo cho trc. Tt c cc s a m

f a 0 mod m c gi l nghim ca ng d thc

124 | Nghim ca cc ng d thc v h thng d y

(6)

f x 0 mod m

T nh l 1 suy ra nu a l nghim ca ng d thc (6) th mi s ng d vi a theo modulo


m cng l nghim ca (6). Do c th coi lp tt c cc s c cng ng d l mt nghim n
ca ng d thc. Nghim n ny c th c chn bi mi s trong .
Mi s nguyn l ng d theo modulo m vi mt trong cc s thuc dy
(7)

0,1, 2, , m 1 .

a
. S r ng d vi a theo modulo m .
m
a
a a
V t 1 t t vi mi s thc t nn ta c 1 , suy ra 0 r m . V vy r thuc dy
m
m m
(7) v do mi s t nhin a l ng d theo modulo m vi t nht mt trong cc phn t thuc
dy (7). Mt khc, cc phn t thuc (7) l cc ng d phn bit modulo m nn mi s nguyn a
ng d vi ng mt phn t thuc (7). S ny gi l s d ca a theo modulo m .
Tht vy, k hiu a l s nguyn cho trc v r a m

Tt c cc s nguyn c cng s d r modulo m u c dng mk r vi k l s nguyn.


gii ng d thc (6) ( f x l a thc h s nguyn) ta ch cn tm cc s trong dy (7) l
nghim ca ng d thc . Vy (6) c th c gii sau hu hn php th. Do v l thuyt ta
c th tm mi nghim ca (6) (f (x) l a thc h s nguyn) hoc chng minh f (x) v nghim.
V d. 1. Gii ng d thc
(8)

x5 3x 2 2 0 mod 7 .

Ta s tm xem trong cc s 0,1,2,3,4,5,6 th s no tha mn (8). Ln lt th 0 v 1 vo (8) ta thy


1 l nghim v 0 khng phi nghim ca (8). Tng t 2 khng phi nghim. Ta c 32 2 mod 7
suy ra 34 4 mod 7 v 35 12 5 mod 7 . Do 35 3.32 2 5 3.2 2 1 mod 7 v v vy 3
khng phi nghim. Vi 4 ta c 4k 3 mod 7 suy ra 45 35 5 mod 7 v do

45 3.42 2 5 3.2 2 3 mod 7 nn 4 khng phi nghim ca (8). Ta c 5 2 mod 7 suy


ra 55 25 3 mod 7 v 55 3.52 2 3 3.4 2 0 mod 7 do 5 l nghim. Ta c

6 1 mod 7 suy ra 65 3.62 2 1 3 2 5 mod 7 nn 6 khng phi nghim. Vy ng d


thc (8) c hai nghim l 1 v 5. Do cc s nguyn x tha mn (8) u c dng 7k 1 hoc
7k 5 vi k l s nguyn ty .
2. Gii ng d thc
(9)

x 2 x 0 mod 2

Ta ch cn kim tra xem (9) c ng vi 0 hoc 1 hay khng. C hai trng hp u tha mn suy
ra mi s nguyn x u l nghim ca (9). Kt qu ny cng c suy ra t tnh cht x 2 v x c
cng tnh chn l v do tng ca chng lun chn. Ta ni ng d thc ng vi mi s nguyn.
y l mt v d cho thy mt ng d thc c th lun ng cho d cc h s khng phi bi s
3
3
ca modulo. Mt v d khc l ng d thc x x 0 mod 3 . Tht vy x x x 1 x x 1 l
tch ca ba s nguyn lin tip nn trong c mt s chia ht cho 3 v v vy tch ca chng chia
3
ht cho 3. Ta c x x 0 mod 3 vi mi s nguyn x .

CHNG 5. NG D | 125

3. Do (9) lun ng nn ng d thc x 2 x 1 0 mod 2 khng c nghim. Tng t ng d


thc x 2 3 mod 8 cng khng c nghim nguyn x v bnh phng mt s l chia 8 d 1 v bnh
phng mt s chn chia 8 d 0 hoc 4.
K hiu m l modulo cho trc, k l s t nhin cho trc m v a1 , a2 ,, ak l cc s nguyn
khng m m . Cu hi t ra l khi no th ng d thc f x 0 mod m ( f x l a thc h
s nguyn) c tt c cc nghim l a1 , a2 ,, ak (v cc ng d ca chng modulo m ).
Nu m l s nguyn t th r rng hm cn tm l f x x a1 x a2 ... x ak .
Nu m 4 v a1 , a2 ,, ak , k 4 , l cc s nguyn khng m cho trc 4 th cc nghim ca

ng d x a1 x a2 ... x ak 0 mod 4 l cc s a1 , a2 ,, ak (v cc ng d ca chng


theo mobulo 4). Tuy nhin M.Chojnacka Pniewska [1] chng minh rng khng tn ti a thc
f x a0 x n a1 x n1 ... an1 x an m f x 0 mod 6 tha mn vi 2 v 3 nhng khng tha
mn vi mi s nguyn 6 khc. Tht vy, gi s

f x c tnh cht . Khi

f 2 f 3 0 mod 6 suy ra 3 f 2 3 f 3 0 mod 6 . Ta c 3.2k 2.3k 0 mod 6 vi mi


k 1, 2, nn 3 f 2 3an mod 6 v 2 f 3 2an mod 6 . Do 3 f 2 2 f 3 an mod 6
suy ra an 0 mod 6 nn f 0 0 mod 6 . Ta chng minh ng d thc f x 0 mod 6 c
nghim x 0 , mu thun vi gi thit 2 v 3 l tt c cc nghim.
C th chng minh rng (Sierpinski [15]) nu m l hp s 4 th tn ti hai s nguyn a v b
khng c cng s d khi chia cho m v nu f x l a thc h s nguyn th ng d thc

f a f b 0 mod m ko theo ng d thc f 0 0 mod m . T y suy ra nu m l hp


s 4 th tn ti a thc bc hai f x x 2 a1 x a2 vi h s nguyn m ng d thc

f x 0 mod m c nhiu hn hai nghim.


y l mt tnh hung kh ging nhau gia l thuyt cc ng d thc v l thuyt cc phng
trnh Diophante tuyn tnh mt bin. Tht vy, s nguyn x tha mn ng d thc (6) khi v ch
khi tn ti s nguyn y tha mn f x my . V vy ng d thc f x 0 mod m tng
ng vi phng trnh Diophante f x my 0 .
Vi nhng lp lun tng t ta chng minh c mt ng d thc m v tri l mt a thc
nhiu bin h s nguyn v v phi l mt s cho trc l gii c v l thuyt. Chng hn i vi
ng d thc f x, y 0 mod m vi f x l a thc vi cc bin x, y th ch cn tm xem trong

m 2 b s x, y vi x v y nhn mi gi tr 0,1,, m 1 th b s no tha mn ng d thc .


Cc php th ny c th lm n gin hn da vo nhn xt nu a c mod m v b d mod m
4
4
th f a, b f c, d mod m . Mt v d n gin l xt ng d thc x y 1 mod 5 . Tt c

x, y 0,1 , 0, 2 0,3 , 0, 4 , 1,0 , 2,0 , 3,0 ,(4,0). V


3
3
3
vy trong mi nghim ca n c ng mt s chia ht cho 5. ng d thc x y z 4 mod9
cc nghim ca phng trnh ny l

v nghim v lp phng ca mt s nguyn th ng d vi 0,1, 1 theo modulo 9 v do tng


ca ba lp phng khng th ng d vi 4.
3. Nghim ca a thc v nghim ca ng d thc

126 | Nghim ca a thc v nghim ca ng d thc

Nu phng trnh f x, y 0 vi f x, y l a thc h s nguyn c cc nghim nguyn x, y th


r rng vi mi s t nhin m u tn ti cc s nguyn x, y m s f x, y chia ht cho m ,
ngha l ng d thc f x, y 0 mod m c nghim t nhin m . V vy nu tn ti modulo m
m ng d thc f x, y 0 mod m khng c nghim nguyn th phng trnh f x, y 0
cng khng c nghim nguyn.
2
n
V d vi mi s t nhin n th phng trnh x +1 3y 0 khng c nghim nguyn v

x 2 1 3y n 0 mod 3 khng c nghim. Tht vy, v mt bnh phng ca s nguyn th chia 3


d 0 hoc 1, do v tri ca ng d thc chia 3 d 1 hoc 2.
Tuy nhin nu phng trnh f x, y 0 khng c nghim vi f x, y l a thc h s nguyn
th khng suy ra tn ti modulo m m ng d thc f x, y 0 mod m v nghim. Chng hn

2x 1 3y 1 0 khng c nghim nguyn nhng ng d thc


2x 1 3y 1 0 mod m li c nghim vi mi s t nhin m . Tht vy, mi s t nhin m
u c th biu din c di dng m 2k 1 2x 1 vi k , x l cc s t nhin. S 22k 1 1 chia
phng

trnh

ht cho 2 1 3 v do tn ti s t nhin

y tha mn 22k 1 1 3y . Do

2x 13y 1 2k 2m v t y suy ra ng d thc trn l gii c. D dng chng minh kt


lun tng qut v cht hn: nu a1 , a2 l cc s t nhin m a1 , a2 1 , b1 , b2 l cc s nguyn ty
th ng d thc a1 x b1 a2 y b2 0 mod m l gii c vi mi s t nhin m ( Skolem [1]).
2
2
D dng chng minh phng trnh 2 x 219 y 1 khng c nghim nguyn x, y bi v ng d

thc 2 x 2 219 y 2 1 mod 3 l khng gii c (tht vy, nu x l s nguyn, x 2 chia 3 d 0 hoc
2
2
1 v do v 219 3 73 , s 2 x 219 y chia 3 d 0 hoc 2).

Kh hn mt cht chng minh phng trnh 2 x 219 y 1 khng c nghim nguyn.


T.Nagell [7] suy ra kt lun ny bng cch s dng mt nh l tng qut hn vi chng minh
2
2
rt kh. Tuy nhin ng d thc 2 x 219 y 1 mod m (ti liu dn trang 62) l gii c
2

vi mi s t nhin m .
Ta trnh by chng minh trc tip phng trnh 2 x 219 y 1 l khng c nghim nguyn
x, y . Gi s phn chng phng trnh ny c nghim nguyn. Khi cc s x, y u khc 0 . Ta
gi s cc s ny u dng. Hn na gi s x, y c chn vi y nh nht c th. t
2

x1 293 x 3066 y , y1 28x 293 y . Khi 2 x12 219 y12 2 x2 219 y 2 . H qu l x1 , y1 tha
mn phng trnh. Ta khng th c x1 0 v do x1 l s t nhin. Ta cng khng th c y1 0
2

y
y
293
85849 2
2
2
v nu ngc li th x
nn 1
, v
y suy ra x 2
y v do 2 x 219 y
392
392
28
784
l. Vy x1 , y1 l cc s t nhin. Theo gi thit th y y1 nn 28x 293 y y suy ra
292
73
539 2
73 2
73
x
y
y v vy x 2
y v 2 x 2 219 y 2
y
1 , mu thun vi gi thit
28
7
49
49
49
x, y l nghim ca phng trnh. Vy phng trnh khng c nghim nguyn x, y .
2
2
By gi ta chng minh rng ng d thc 2 x 219 y 1 mod m l gii c vi mi s t

nhin m . Vi m l s t nhin t m m1.m2 vi m1 11 ( nguyn 0 ) v m2 ,11 1 . t

CHNG 5. NG D | 127
m1 1

x1 5.13

m 1
, y1 13 1 . V

13, m1 1

nn theo nh l Euler (Chng 6) th

13 m1 1 mod m1 . Do 132 2 x12 219 y12 2 25 132 m 2 25 219 132 mod m1 suy ra v
1

13, m1 1 ta c 2 x12 219 y12 1 mod m1 . t x2 7.11 m 1, y2 11 m 1 . V 11, m2 1 nn


11 m 1 mod m2 vy 112 2 x22 219 y22 2.49.112 m 219.112 m 2.49 219 112 mod m2
v do v 11, m2 1 ta nhn c 2 x22 219 y22 mod m2 . By gi v m1 , m2 1 nn theo
2

nh l s d Trung Hoa (Chng 1 mc 12) suy ra tn ti cc s nguyn x, y tha mn

x x1 mod m1 , x x2 mod m2
y y1 mod m1 , y y2 mod m2
V vy 2 x 2 219 y 2 2 x12 219 y12 1 mod m1 v 2 x 2 219 y 2 2 x22 219 y22 1 mod m2 do
v m1 , m2 1 v m m1m2 , 2 x 2 219 y 2 1 mod m chng t ng d thc trn l gii
c vi mi s t nhin m .
By gi ta gii mt ng d thc khc m v tri khng c dng a thc. Xt

2x x 2 mod3 .

(*)

V 22 1 mod 3 ta c 2 x 2 k 2 x mod 3 vi mi s nguyn khng m x v k 0,1, 2, V

x 3l

x 2 mod 3 vi mi s nguyn x, l nn ta thy nu x l nghim ca ng d thc (*) th

x 6t , t 0,1, 2,..., cng l cc nghim ca (*). Trong cc s 0,1,2,3,4,5 th ch c 2 v 4 l nghim


ca ng d thc (*). Vy mi nghim ca ng d thc l 2 6t hoc 4 6t vi t 0,1, 2,... .
Ghi ch. Cc s ng d vi nghim ca (*) theo modulo ca ng d thc c th khng phi
nghim ca (*) chng hn s 5.
4. ng d thc bc mt
t
(10)

ax b mod m ,

vi m l modulo cho trc v a, b l cc s nguyn cho trc. Ta bit trong mc 2 th ng d


thc (10) tng ng vi phng trnh Diophante
(11)

ax my b

T nh l 15 Chng 1 th phng trnh (11) c nghim nguyn x, y khi v ch khi a, m | b . Do


y cng l iu kin cn v i vi tnh gii c ca ng d thc (10).
Gi s cc iu kin c tha mn ta s tm phong php tnh tt c cc nghim ca (10) v tnh
s nghim .
t d a, m th s b / d nguyn. Vi x0 l nghim ca ng d thc (10) v x l nghim ty
ca n th ta c ax0 b mod m v theo (10) th ta c a x x0 0 mod m . Do m | a x x0

m a
m a
m
l c s ca x x0 suy ra
| x x0 . Nhng v d a, m nn , 1 do
d d
d
d d
m
x x0 t vi t l s nguyn.
d

suy ra

128 | ng d thc bc mt

Ngc li, xt s nguyn ty t v nghim ty x0 ca ng d thc (10) v t x x0


nhn c nghim ca ng d thc (10) v ax ax0

m
t ta
d

a
tm ax0 b mod m .
d

By gi cho t ln lt nhn cc gi tr 0,1, 2,, d 1 . Ta chng minh cc s


(12)

xt x0

m
t
d

l phn bit modulo m .


Tht vy, nu xt xu mod m th theo (12) ta c x0

m
m
t x0 u mod m v do
d
d

m
t u mz vi z l s nguyn, suy ra t u dz , v l v t , u l cc s phn bit trong dy
d
0,1, 2,, d 1 .
Cui cng ta chng minh rng mi nghim ca ng d thc (10) ng d theo modulo m vi
mt trong cc nghim x0, x1, ..., xd 1 (xc nh trong (12)). Tht vy, nu x l nghim ca ng d
thc (10) th vi s nguyn t ta c x x0

m
t . t r l s d nhn c khi chia t cho d .
d

Khi r l mt trong cc s 0,1, 2,, d 1 . Ta c t f du vi u l s nguyn.


V vy x x0
chng minh.

m
m
m
t x0 r du x0 r mu x r mu , suy ra x xr mod m , iu phi
d
d
d

T cc kt qu ny ta c
nh l 2. ng d thc bc mt ax b mod m l gii c khi v ch khi b chia ht cho c s
chung ln nht d ca h s a ca x v modulo m . Nu iu kin ny c tha mn th ng d
thc c ng d nghim phn bit theo modulo m .
Nu a v m nguyn t cng nhau th d 1 . Ta c h qu
H qu. Nu h s ca x nguyn t cng nhau vi modulo m th ng d thc bc mt
ax b mod m c ng mt nghim.
Nu ng d thc ax b mod m l gii c v

a, m d 1

th ng d thc

a
b
m
a m
x mod nhn c t ng d thc ban u vi , 1 cng gii c. Do
d
d
d
d d
gii cc ng d thc bc mt (trong trng hp ng d thc l gii c) ta c th gi s h s
ca bin s v modulo l nguyn t cng nhau. C.Sardi [1] trnh by mt phng php gii
cc ng d thc nh vy. Xt ax b mod m vi a 1 v a, m 1 . Hn na t

m
m
a1 m a th 0 a1 a v m khng chia ht cho a . Nhn c hai v vi ta nhn c
a
a
m
a1 x b mod m , ngha l ng d thc vi a1 a . Qu trnh ny lp li v ta nhn c
a
an 1 , ngha l ng d thc x c mod m vi nghim duy nht x c .
5. nh l Wilson v nh l Fermat nh
K hiu p l s nguyn t l v D l s nguyn khng chia ht cho p .

CHNG 5. NG D | 129

Cc cp s m, n thuc dy
(13)

1, 2,3,, p 1

c gi l i tch khi v ch khi ng d thc sau tha mn


(14)

mn D mod p

T nh ngha ny suy ra nu m l s i tch vi n th n i tch vi m .


Ta s chng minh vi mi s thuc dy (13) th tn ti ng mt s i tch vi n. Xt m l s
thuc dy (13). iu kin cn v x thuc dy (13) l i tch ca m l mx D mod m . T

ng d thc mx D mod p (vi m l s thuc dy (13)) v theo h qu ca nh l 2 th ng


d thc cui cng c ng mt nghim. Do trong dy 0,1, 2,3,, p 1 c ng mt s tha
mn ng d thc ny. S ny khc 0 v D khng chia ht cho p . Suy ra trong dy (13) c ng
mt s tha mn ng d thc trn.
C th xy ra trng hp cc s i tch l bng nhau. Khi (14) tr thnh m d mod p . iu
2

ny ch c th xy ra nu tn ti bnh phng ng d vi D theo modulo p . Khi D c gi


l thng d bc hai modulo p . Trong trng hp ngc li, ngha l khng c bnh phng no
ng d vi D theo modulo p th ta ni D khng phi thng d bc hai modulo p . Ni cch
khc s D khng chia ht cho p c gi l thng d bc hai hay khng phi thng d bc hai
ty thuc vo vic ng d thc x D mod p l gii c hay khng. u tin ta xt trng
2

hp D khng phi thng d bc hai modulo p nguyn t. Kh mi cp cc i tch m, n cha


hai s phn bit thuc dy (13). Do tt c cc s thuc dy (13) c th chia thnh cc cp i
tch, s cc cp nh vy l ng bng p 1 / 2 . T (14) ta c p 1 / 2 ng d thc

m1n1 D mod p ,
m2 n2 D mod p ,

m p 1 n p 1 D mod p .
2

Khi nhn cc ng d thc ny theo v v lu tch m1n1m2 n2 ...m p 1 n p 1 chnh l tch cc phn
2

t thuc dy (13) sai khc nhiu nht mt hon v. Ta c


1

(15)

p 1! D 2

p 1

mod p .

By gi xt trng hp D l thng d bc hai modulo p . Khi ng d thc


(16)

x2 D mod p

l gii c. Ta tnh s cc s trong (13) m tha mn (16). Ta c th gi s (16) l gii c, khi


trong dy 0,1, 2,, p 1 c t nht mt s k l nghim ca (16). Khng th c k 0 v theo gi
thit th D khng chia ht cho p . Do s k l mt trong cc s thuc dy (13) v do p k
cng thuc dy ny. S ny l khc k v p l s l. Vi l p k ta c l k
2

mod p nn t ng

2
2
d thc k D mod p suy ra l D mod p .

Vy nu D l thng d bc hai modulo p thi trong dy (13) c t nht hai s khc nhau m cng
tha mn ng d thc (16). Ta chng minh c ng hai s nh vy.

130 | nh l Wilson v nh l Fermat nh

Gi s x thuc dy (13) tha mn ng d thc (16). V k 2 D mod p nn ta c x 2 k 2 mod p


suy ra p | x 2 k 2 x k x k . Nhng v p l s nguyn t nn p | x k hoc p | x k . Nu

p | x k th v x v k cng thuc dy (13) suy ra x k . Nu p | x k th v 0 x p v 0 k p


do 0 x k 2 p nn ta c x k p suy ra x p k l .
Vy ta chng minh c k v l l tt c cc s trong dy (13) m tha mn ng d thc (16).
Do : nu D khng chia ht cho s nguyn t l p v l thng d bc hai modulo p th ng d
thc (16) c ng hai nghim.
By gi ta b cc s k v l ra khi dy (13). Khi khng c s no trong p 3 s cn li tha
mn ng d thc (16) do c th chia chng thnh p 3 / 2 cp i tch. Do ta nhn c

p 3 / 2 ng d thc
m1n1 D mod p ,
m2 n2 D mod p ,

m p 3 n p 3 D mod p .
2

V kl k p k k 2 D mod p nn ta c th cng ng d thc kl D mod p vo cc ng


d thc trn sau nhn li. Khi v tri ca ng d thc nhn c bng p 1! . Do
1

(17)

p 1 D 2

p 1

mod p

Ta thy (15) hoc (17) ng hay khng ph thuc vo vic D l thng d bc hai modulo p hay
khng. Kt hp li ta vit
(18)

p 1! D

1
p 1
2

mod p ,

Cc du v + trong v phi c ly ty thuc D l thng d bc hai modulo p hay khng.


c bit vi D 1 ta thy 1 l thng d bc hai theo mi modulo p nn
(19)

p 1! 1 mod p .

Trong chng minh tnh cht (19) c s dng ti gi thit p l s nguyn t l, nhng kt qu vn
ng vi p 2 v ta thy 2 1 ! 1 1 mod 2 . Vy ta c nh l
nh l 3 (Wilson). Nu p l s nguyn t th p 1! 1 chia ht cho p .
Mnh ngc li cng ng. Tht vy, nu p l s t nhin 1 v nu p 1! 1 chia ht cho

p th p l s nguyn t. Tht vy, gi s phn chng p khng phi s nguyn t. Khi tn ti


c s q ca p m 1 q p . S p 1! 1 chia ht cho p nn n chia ht cho q nhng v
q p, q p 1 nn q | p 1! suy ra q | 1 , mu thun. Vy ta c
nh l 3a. iu kin cn v s t nhin n 1 l s nguyn t l n 1! 1 chia ht cho n .
Vy v l thuyt th quyt nh xem mt s t nhin n 1 c phi s nguyn t hay khng ta
ch cn s dng ng mt php chia.

CHNG 5. NG D | 131

T nh l 3 suy ra vi s nguyn t p th s wp p 1! 1 / p l s t nhin. C.E.Froberg [2]


tnh cc s d nhn c khi chia w p cho p vi cc s nguyn t p 50000 . Cc s nguyn t
tha mn p 2 | p 1! 1 c gi l cc s nguyn t Wilson. T bng cho bi Froberg th trong
cc s nguyn t p 50000 ch c ba s nguyn t Wilson l 5,13 v 563.
T nh l 3a v lu vi n 2 th n 1! n 2 ! n 1 n 2 ! mod n ta c
nh l 3b (Leibniz). S t nhin n 1 l s nguyn t khi v ch khi n 2 ! 1 mod n .( 0! 1)
C th chng minh s t nhin p 1 l s nguyn t khi v ch khi tn ti s t nhin n p m

n 1! p n ! 1 mod p (Dickson [7] tp 1 trang 64).


n

R rng nu n l s t nhin tha mn n | n 1! th n l hp s. D dng chng minh nu n l


hp s 4 th n | n 1! . Tht vy, nu n l hp s th tn ti cc s t nhin a v b m

n ab, 1 a b, 1 b n . Nu a b th a v b l cc nhn t phn bit ca tch n 1! v do

n ab l c s ca n 1!. Nu a b th n a 2 v v n l hp s 4 nn a 2 . V vy
n a2 2a v do a v 2a l cc nhn t phn bit ca tch n 1!. Vy n 1! chia ht cho
2a 2 v do cng chia ht cho a 2 n . Vi n 4 th ta c n 1! 3! 6 2 mod 4 .
T nh l 3 suy ra tn ti hu hn cc s t nhin n m n ! 1 l hp s. Chng hn cc s
n p 1 vi p l s nguyn t 3 . Khi p 1! 2 p 1 p p 2 p . A.Schinzel [14]
chng minh vi mi s hu t c 0 th lun tn ti v hn hp s nguyn c dng cn! 1 . Tuy
nhin ta cha bit c tn ti v hn cc s nguyn t c dng n ! 1 hay khng? Vi n 546 th cc
s nguyn t c dng ny nhn c vi n 1, 2,3,11, 27,37, 41,73,77,116,154,320,340,399, 427
(Buhler, Crandall v Penk [1]).
Ta cng cha bit c tn ti v hn cc s t nhin k m cc s Pk p1 p2 ... pk 1 nguyn t hay
khng? Cu hi tng t vi Pk l hp s. Ta bit mt s s nguyn t c dng Pk l P1 3,

P2 7, P4 211, P5 2311, nhng cc s sau y P6 59 509, P7 19 97 277, P8 347 27953,


P9 317 703763, P10 331 571 34231 khng phi s nguyn t. Vi k nm gia 10 v 442 th Pk
l s nguyn t ch vi k 11,75,171,172,284 (Buhler, Crandall v Penk [1]). T nh l 3b suy ra
tn ti v hn s t nhin n m n !1 l hp s. Chng hn vi cc s n p 2 vi p l s nguyn
t 5 . Ta cha bit c tn ti v hn cc s nguyn t dng ny hay khng. Nu n 546 , th n !1
l s nguyn t ch vi n 3, 4, 6, 7, 12, 14, 30, 32, 33, 94, 166, 324, 379, 427 (Buhler,
Crandall v Penk [1]).
T cc cng thc (15), (17) v nh l 3 suy ra
nh l 4. Nu s nguyn D khng chia ht cho s nguyn t l p th
1

(20)

D2

p1

1 mod p ,

Trong cc du + hoc c ly ty thuc D c phi l thng d bc hai modulo p hay khng.


Bnh phng hai v ca (20) ta nhn c
nh l 5. Nu s nguyn D khng chia ht cho s nguyn t p th
(21)

D p 1 1 mod p

132 | nh l Wilson v nh l Fermat nh

y l nh l nh Fermat. nh l ny c Fermat pht biu (khng km theo chng minh) vo


nm 1640. Li gii u tin c trnh by bi L.Euler nm 1736.
Chng minh trn ca cng thc (20) khng ng khi p 2 nhng ta c th thy (21) vn ng
v vi D khng chia ht cho p 2 nn n l s l v do D 1 mod 2 .
c bit t nh l 5 suy ra nu p l s nguyn t l th 2 p1 1 chia ht cho p . Cc nghin cu
xem khi no th 2 p1 1 chia ht cho p c tin hnh. Vi p 6 10 th ch c hai s c
tm ra l p 1093, p 3511 (Brillhart, Tonascia v Weinberger [1] v Lehmer [9]).
2

ng dng n gin ca nh l 5 cho php tnh tt c cc nghim ca ng d thc ax b mod p


vi p l s nguyn t v a khng chia ht cho p . Tht vy, x a p 2b l nghim v theo nh l 5
ta c a

p 1

1 mod p suy ra ax a p1b b mod p .

H qu trc tip ca nh l 5 l
p
nh l 5a . Nu p l s nguyn t th vi mi s nguyn a ta c p | a a .

Ngc li, nh l 5 c suy ra t nh l 5a . Tht vy, nu a l s nguyn khng chia ht cho s


nguyn t p th t p | a p a a(a p 1 ) 1 suy ra a p 1 1 (mod p) .
Cc nh l Wilson v Fermat c kt hp thnh nh l duy nht c dng sau y (Moser [4])
nh l 6. Nu p l s nguyn t v a l s nguyn th

p | a p p 1!a

(22)

Tht vy, t nh l 3 suy ra p 1! 1 mod p v do a p 1!a a a mod p . Theo


p

nh l 5a suy ra a a 0 mod p nn (22) c ko theo.


p

Mt khc, nu nh l 6 ng th vi a 1 t cng thc (22) c th suy ra nh l 3. V vy vi mi


s nguyn a m ng d thc a p p 1!a a p a mod p c tha mn th t (22) c th
suy ra a a 0 mod p . Do nh l 5a ng v n tng ng vi nh l Fermat nh.
p

D dng chng minh dng chung ca nh l Fermat nh v nh l Wilson tng ng vi


nh l 6a . Nu p l s nguyn t v a l s nguyn th p | p 1!a a
p

Trong mi lin h ny cc tc gi Dickson [7] tp 1 trang 84-86 v T.Szele [1] chng minh dng
tng qut sau y ca nh l 5a, dng tng qut ny c t ra bi J.A.Serret nm 1855: vi mi
s t nhin m v s nguyn a th s d a d |m chia ht cho m .
d |m

pq
p
q
V vy, vi mi s nguyn a v hai s nguyn t phn bit p v q ta c pq | a a a a .

Ta c mt h qu khc ca nh l 5
nh l 7. Tn ti v hn s nguyn t c dng 4k 1 vi k l s t nhin.
Chng minh. Xt n l s t nhin ty 1 v t
(23)

N n ! 1
2

S N l v 1 . K hiu p l c s nguyn t nh nht ca N . Theo (23) th p n . Do p l nn


n c dng 4k 1 hoc 4k 3 . Theo (23) ta c n ! 1 mod p v ly tha c hai v ln
2

CHNG 5. NG D | 133

( p 1) / 2 ln ta c n!
ta c n !

p 1

p 1

p 1 /2

mod p . Nhng n ! khng chia ht cho p

v theo nh l 5,

1 mod p suy ra
1

1 2

(24)

p 1

1 mod p
1

Khng th c p 4k 3 v nu nh vy th t (24) suy ra 1 2

p 1

2 k 1

1 1 mod p do

p | 2 , v l. Vy p phi c dng 4k 1 .
Do ta chng minh c vi mi s t nhin n 1 th lun tn ti s nguyn t p n c
dng 4k 1 . Cc c s nguyn t ca s cho bi (23) tha mn tnh cht ny. nh l 7 c
chng minh.
Ta cng chng minh c c v hn cc s nguyn t c dng 4k 3 . Tht vy, k hiu n l s t
nhin ty 3 v t

N1 n! 1

(25)

th N1 l s l 1 v v th cc c s nguyn t ca n u l. Nu tt c cc c s ca n u c
dng 4k 1 th N1 cng c dng 4k 1 . Nhng iu ny khng ng vi n 3 . Vy vi mi s t
nhin n 3 th tn ti s nguyn t p n c dng 4k + 3. V vy

nh l 7 a . Tn ti v hn s nguyn t c dng 4k 3 vi k l s t nhin.


Vi s thc cho trc x 1 k hiu 1 x l s cc s nguyn t x c dng 4k 1 v 3 x l
s cc s nguyn t x c dng 4k 3 . t x 3 x 1 x . Nm 1914 J.E.Littlewood
chng minh rng tn ti v hn s t nhin n m n 0 v tn ti v hn s n m n 0 .
Tuy nhin ti tn nm 1957 th ta vn cha tm c s n no m n 0 . S dng my tnh
in t EDSAC, J. Leech [1] tnh cc s n vi n 3000000 v chng minh c s t nhin
nh nht n m n 0 l n 26861 . Vi s n ta c 1 n 1473, 3 n 1472 v n 1 .
Ta tnh c 623681 8 , 627859 627860 ... 627900 0 , 2951071 256 .
p 1
T nh l 5 suy ra nu p l s nguyn t th a 1 mod p vi a 1, 2,... p 1. Cng p 1 ng

d thc ta c 1p 1 2 p 1 ... p 1

p 1

p 1 mod p . V vy p |1p 1 2 p 1 ... p 1

p 1

1 vi

mi s nguyn t p . G.Giuga [1] t ra gi thuyt rng tnh cht ny khng ng vi cc hp s.


1700
Gi thuyt ny c E.Bedocchi [1] (1) chng minh vi p 10 .

nh l sau y l h qu ca nh l 3.
nh l 8. Nu p l s nguyn t c dng 4k 1 vi k l s t nhin th
2

p 1
p
! 1
2

(26)
Chng minh. T

1
2

p 1 2k

suy ra

p 1
p 1
1 2 3... 12 p 1 1 2 ...
mod p ;
p 1 p 2 ...
2
2

(1)

Xem ph lc mc b sung cho cc chng minh

134 | nh l Wilson v nh l Fermat nh


2

p 1
p 1 p 1
Do
.
... p 1 p 1! 1 mod p v suy ra (26).
! 1 2...
2
2
2
Da theo nh l 8 ta chng minh
nh l 9 (Fermat). Mi s nguyn t p c dng 4k 1 u l tng ca hai bnh phng.

p 1
! . Theo nh l 8 ta c
2

Chng minh. Gi s p l s nguyn t c dng 4k 1 v a

p | a 2 1 do a nguyn t cng nhau vi p . Theo nh l Thue (Chng 1 mc 13) th tn ti


hai s t nhin x, y p m vi cch chn cc du + v thch hp th ax y chia ht cho p . V
vy suy ra s a 2 x 2 y 2 ax y ax y chia ht cho p . Do a 2 x 2 y 2 ax y ax y chia
ht cho

p v a 2 x 2 x 2 a 2 1 x 2 chia ht cho

p (v

p | a 2 1 ). H qu l s

x 2 y 2 a 2 x 2 x 2 a 2 x 2 y 2 chia ht cho p . Nhng v x, y l cc s t nhin


u

p nn chng

p v p l s nguyn t v khng phi bnh phng mt s t nhin. Do x 2 y 2 l s t

2
2
nhin 1 v 2 p v hn na n chia ht cho p do n bng p , ngha l p x y . Chng t
p l tng ca hai bnh phng cc s t nhin.

Cc s c dng 4k 3 (khng nht thit nguyn t) khng th biu din c thnh tng ca hai
bnh phng v mt bnh phng chia 4 d 0 hoc 1, do tng hai bnh phng chia 4 d 0,1
hoc 2. Chng t trong cc s nguyn t th ch c 2 12 12 v cc c dng 4k 1 l tng ca hai
bnh phng cc s t nhin.
H.Davenport [2] (trang 120-122) trnh by bn cch phn tch s nguyn t c dng 4k 1
thnh tng hai bnh phng. Cc cch c cho bi Legendre (1808), Gauss (1825), Serret
(1848) v Jacobsthal (1906). Cch xy dng n gin nht (khng km theo chng minh) thuc v
2
Gauss. Nu p 4k 1 l s nguyn t ta ly cc s nguyn x, y m x 2k !/ 2 k ! mod p v

y 2k ! x mod p vi x 12 p, y 12 p . Khi p x 2 y 2 . Cc chng minh c trnh by bi


Cauchy v Jacobsthal u khng n gin. Cc tnh ton cc s x, y l khng d. Chng hn vi
p 29 th x 14!/ 2 7! 1716 5 mod 29 , y 14! x 14! 5 2 mod 29 suy ra x 5, y 2 .
2

Ta khng bit c tn ti v hn s nguyn t p m p x 2 x 1 vi x l s t nhin hay


2

khng. Cu tr li khng nh c suy ra t gi thuyt H (Chng 3 mc 8). Chng hn ta c


2
2
5 12 22 , 13 22 32 , 41 42 52 , 61 52 62 , 113 72 82 , 181 92 102 , 313 12 13 ,
2
2
421 142 152 , 613 172 182 , 761 19 20 .

Gi thuyt ni rng tn ti v hn s nguyn t m mi s u l tng ca hai bnh phng lin


2
tip l tng ng vi gi thuyt ni rng tn ti v hn s nguyn t p m 2 p a 1 vi a l
s t nhin. Tht vy, gi s p x 2 x 1 vi x l s t nhin th 2 p 2 x 1 1 . Ngc li
2

2
nu 2 p a 1 vi a l s t nhin th vi p 2 s a l 1 v do a 2 x 1 vi x l s t

nhin. Suy ra 2 p 2 x 1 1 ngha l p x 2 x 1 .


2

2
2
T gi thuyt H suy ra tn ti v hn s nguyn t p m p a b vi a v b l cc s nguyn

t. Chng hn 13 22 32 , 29 22 52 , 53 22 72 , 173 22 132 , 293 22 172 , 1373 22 372 .


Cng t gi thuyt H suy ra tn ti v hn cc s nguyn t l tng ca ba bnh phng lin tip
cc s t nhin. Chng hn 29 22 32 42 , 149 62 72 82 , 509 122 132 142 ,
677 142 152 162 , 1877 242 252 262 . Trong mi lin h ny ta lu rng gi thuyt H suy

CHNG 5. NG D | 135

ra tn ti v hn s nguyn t m mi s u l tng ba bnh phng cc s nguyn t l phn bit.


Chng hn 83 32 52 72 , 179 32 72 112 , 419 32 112 172 , 563 32 52 232 . D dng
chng minh trong cc bnh phng ny lun phi c s 32.
Mt h qu khc c th suy ra t gi thuyt H l vi mi s t nhin n th lun tn ti v hn s t
nhin x m x 2 n2 l cc s nguyn t.
C th chng minh rng vi mi s t nhin n th lun tn ti s nguyn t p tha mn

p a 2 b2 vi a n v b n (Chng 3 mc 7).
Nu mt s nguyn t l tng ca hai hoc bn bnh phng cc s nguyn t phn bit th mt
trong cc s nguyn t phi bng 2. Nu mt s nguyn t l tng ca ba bnh phng cc s
nguyn t phn bit th mt trong cc s bng 3. Tuy nhin t gi thuyt H suy ra vi mi s t
nhin n th tn ti s nguyn t q pn3 m s p pn 2 pn21 pn22 pn23 q 2 nguyn t. Chng
hn 373 32 52 72 112 132 , 653 52 72 112 132 172 , 1997 72 112 132 172 372 .
By gi ta chng minh nu php phn tch mt s nguyn t thnh tng ca hai bnh phng t
nhin l tn ti th biu din l duy nht sai khc cc hon v. Ta chng minh mnh tng qut
sau y
nh l 10. Gi s a v b l cc s t nhin th nu tn ti biu din ca s nguyn t p di dng

p ax 2 by 2 vi x, y l cc s t nhin th biu din ny l duy nht (khng tnh hon v x v y


nu a b 1 ).
Chng minh. Gi s vi s nguyn t p
(27)

p ax 2 by 2 ax12 by12

Vi x, y, x1, y1 l cc s t nhin. R rng x, y x1, y1 1 . T (27) ta c

p 2 axx1 byy1 ab xy1 yx1 axx1 byy1 ab xy1 yx1


2

Nhng axx1 byy1 xy1 yx1 ax 2 by 2 x1 y1 ax12 by12 xy p x1 y1 xy . Do t nht mt


trong cc nhn t v tri phi chia ht cho p . Nu p | axx1 byy1 th t cng thc th nht ca

p 2 suy ra xy1 yx1 0 . Do x / y x1 / y1 m x, y x1, y1 1 suy ra x x1 , y y1 . Nu


p | xy1 yx1 th t cng thc th hai ca p 2 suy ra p 2 abp 2 . iu ny ch c th khi a b 1 .

Nhng xx1 yy1 0 nn x / y y1 / x1 m x, y x1, y1 1 suy ra x y1 , y x1 . Vy cc phn


2
2
2
2
tch p x y v p x1 y1 ch sai khc th t cc hng t. nh l 10 c chng minh.

H qu trc tip ca nh l 10 l nu mt s t nhin c hai (hoc nhiu hn) biu din thnh
2
2
dng ax by vi x, y l cc s t nhin th n phi l hp s. Mnh ngc li khng ng.
2
2
Chng hn s 14 c duy nht mt biu din c dng 14 2 x 3 y vi x, y l cc s t nhin

x 1, y 2

2
2
v s 15 l hp s nhng khng c biu din no di dng 15 2 x 3 y vi x, y

2
2
l cc s nguyn. S 18 c biu din duy nht di dng 18 x y vi x, y l cc s t nhin vi
2
2
x y 3 . Cc s 25 v 45 c biu din duy nht khng tnh cc hon v c dng x y vi x, y l
2
2
2
2
cc s t nhin, l 25 3 4 , 45 3 6 .

Tuy nhin ta c nh l sau y


nh l 11. S t nhin c dng 4k 1 1 l s nguyn t khi v ch khi n c ng mt biu din
(khng tnh cc hon v) di dng tng ca hai bnh phng cc s nguyn 0 v trong biu din
cc bnh phng l nguyn t cng nhau.

136 | nh l Wilson v nh l Fermat nh

Chng minh. Gi s s p 4k 1 nguyn t. Th th theo nh l 9 v 10, s p c ng mt biu


2
2
din (khng tnh cc hon v) di dng p x y vi x, y l cc s t nhin. R rng trong biu

2
din ny th x, y l cc s nguyn t cng nhau. V nu ngc li x, y d 1 th d | p v l. Vy

ta chng minh c iu kin cn. chng minh iu kin ta chng minh b sau y
B . Nu hai s t nhin c dng 4k 1 vi k 0 u l tng ca hai bnh phng cc s nguyn
th tch ca chng khng tha mn cc iu kin ca nh l 11.
2
2
2
2
Chng minh b . Gi s m a b , n c d vi a, b, c, d l cc s nguyn. Ta c

(28)

mn ac bd ad bc ac bd ad bc
2

Gi s hai biu din mi nhn c ny ca s mn ch sai khc th t cc hng t. Th th


ac bd ad bc hoc ac bd ac bd . Trong trng hp th nht ta c a c d b c d
nhng c d v nu c d th n 2c 2 mu thun vi vic n l s l, nn a b . Nhng iu ny l
khng th v m l s l. Trong trng hp cn li, ngha l ac bd ac bd ta c

ac bd ac bd hoc ac bd bd ac . Trong trng hp th nht th bd 0 v do b 0


2
2
hoc d 0 . Nu b 0 th m a 2 vi a 1 v mn ac ad vi ac v ad c c s chung

1 v do mn khng tha mn cc iu kin t ra trong nh l 11. Trong trng hp th hai


ta c ac 0 v do a 0 hoc c 0 , chng minh tng t trng hp trc suy ra mn khng
tha mn cc iu kin t ra trong nh l 11. V vy ch cn xt trng hp phn tch (28) khng
ch sai khc th t cc hng t. Trong trng hp ny r rng mn khng tha mn cc iu kin
trong nh l 11. B c chng minh.
Ta chng minh iu kin . Gi s phn chng rng s 4k 1 1 tha mn cc iu kin trong
nh l 11 v khng phi s nguyn t. Gi p l c s nguyn t ty ca s . R rng p l s l.
2
2
2
2
Nu p bng 4t 3 th v gi thit s a b vi a, b 1 ta c a b mod p t y ly tha

1
p 1 2t 1 th theo nh l 5 ta c 1 1 mod p , ngha l 2 | p , iu
2
ny l khng th. V vy p phi c dng 4t 1 v do theo nh l 9, p l tng ca hai bnh
phng cc s t nhin. Do mi c s nguyn t ca s u l tng hai bnh phng cc s
nguyn nn theo (28) cc c s ca s cng c tnh cht ny. Nu s s l hp s th n l tch ca
cc s t nhin n, m 1 m mi s l tng bnh phng hai s nguyn c dng 4t 1 (v cc c s
nguyn t ca n u c dng ny) nn theo b th s s mn khng tha mn cc iu kin
trong nh l 11, mu thun vi gi thit. nh l 11 c chng minh.
c hai v ln s m

p dng nh l 11 ta thy kim tra xem mt s t nhin cho trc n c dng 4k 1 c phi l
s nguyn t hay khng th ta xt dy n 02 ,

n 12 ,

...,

n n v kim tra xem c s no

l bnh phng hay khng.


S dng nh l 11 theo cch ny th vo nm 1960 T.Kulikowski, vi s tr gip ca my tnh
EMC ti Warsaw Polytechnic, ch ra s 239 7 l nguyn t v n ch c duy nht mt biu din
39
2
2
thnh tng hai bnh phng cc s nguyn l 2 7 64045 738684 v cc s nguyn ny l
n
nguyn t cng nhau. Cc s 2 7, n 4,5,...38 , u l hp s. Bi ton tm s nguyn t c dng

2n 7 c t ra bi P.Erdos vo nm 1956.
Bi tp. 1. Chng minh rng cc s t nhin n 1 v n 2 to thnh mt cp s nguyn t sinh
i khi v ch khi ng d thc
(29)

4 n 1! 1 n 0 mod n n 2

CHNG 5. NG D | 137

ng (Clement [1], vi n 3 xem Coblyn [1]).


Chng minh. Gi s n v n 2 u l s nguyn t. Theo nh l 3 th n 1! 1 0 mod n v

n 1! 1 0 mod n 2 . Nhng v n 2 mod n 2 v n 1 1 mod n 2


n 1! n 1!2 mod n 2 . Suy ra v tri ca (29) chia ht cho n do

nn suy ra ta c

4 n 1! 1 n n 1!2 2 n 2 2 n 1! 1 n 2 0 mod n 2
V vy v tri ca (29) cng chia ht cho n 2 . Nhng cc s n , n 2 l cc s nguyn t phn
bit nn v tri ca (29) chia ht cho n n 2 v do (29) c tha mn.
By gi gi s vi s t nhin n 1 th ng d thc (29) ng. Nu n chn ngha l n 2k vi k
l s t nhin th khi n 1 k suy ra k | n 1! v 2k | n 1!4 . H qu l n 1!4 0 mod n
m theo (29) suy ra 4 0 mod n v 2k | 4 , suy ra k | 2 nn k 1 hoc k 2 do n 2 hoc

n 4 . Nhng ng d thc (29) khng ng vi n 2 v n 4 . V vy (29) ko theo


n 1! 1 0 mod n v theo nh l 3a th n l s nguyn t. Cui cng vi s t nhin n th

ng d thc 4 n 1! 1 n 2 n 1! 1 mod n 2 ng, suy ra t (29), s dng tnh cht

n 2 l, ta c n 1! 1 0 mod n 2 . V vy s dng nh l 3a ta c n 2 l s nguyn t. Vy


n , n 2 l cp s nguyn t sinh i.

2
2
2
2
2. Chng minh rng nu n a b c d vi a, b, c, d l cc s t nhin tha mn

a b, c d , a c, a, b c, d 1, th s

(30)

ac bd
ac bd , ab cd

l c s ca n tha mn 1 n .
2
2
2
2
Chng minh. Nu n a b c d th
2
2
2
2
2

n ac bd ad bc ad bc ac bd

ac bd ad bc n ab cd

(31)

V vy n | ac bd ad bc . Nu n | ac bd th theo (31) ta c ad bc 0 suy ra a / b c / d , m

a, b c, d 1,

suy ra a c , v

a, b c, d 1

suy ra a d , mu thun vi gi thit

a c d . Cc s n1 ac bd v n2 ad bc u khng chia ht cho n , v t tnh cht n | n1 n2


bi tp 2 Chng 1 mc 6 v cng thc (31) suy ra l c s ca s n v 1 n .
3. Chng minh nh l sau y ca Liouville [1]: nu p l s nguyn t 5 th s p 1! 1 khng
phi ly tha bc k ca p vi mi s t nhin k .
Chng minh. Nh chng minh trn, nu s t nhin n l hp s 4 th n | (n 1)!. Do nu
p l s nguyn t 5 th p 1| p 2 ! suy ra p 1 | p 1! . Mt khc p dng cng thc nh
2

k
thc cho 1 p 1 p k vi k l s t nhin th p 1 |1 k p 1 p k . Nu p 1! 1 p
k

th p 1 | k p 1 p 1! , theo cng thc p 1 | p 1! suy ra p 1 | k p 1 v do


2

k
p 1
p 1| k suy ra k p 1 nn p 1! 1 p p , v l v p 1 p 1

p 2

138 | nh l Wilson v nh l Fermat nh

4. Chng minh rng nu p l s nguyn t 5 th s p 1! 1 c t nht hai c s nguyn t


phn bit.
Chng minh. Theo nh l 3 th s p 1! 1 c t nht mt c s nguyn t p . Theo bi tp 3
th s ny khng phi ly tha bc k ca p vi mi s t nhin k do n phi c c s nguyn
t khc.
5. Chng minh nh l Lerch [1] ni rng nu p l s nguyn t l th

1p 1 2 p 1 ... p 1

p 1

p p 1! mod p 2

Chng minh. Vi p l s nguyn t l th theo nh l 3 s


nhn c khi chia s ny cho p th ta c
Theo nh l 5 vi a 1, 2,p 1 th s
cho p th

p 1! 1 r
p

p 1

p 1! 1
p

nguyn. Gi r l s d

mod p . V vy p 1! pr 1 mod p 2 .

nguyn, gi ra l s d nhn c khi chia s

a p 1 1
r (mod p) . Do
p
a p 1 pra 1 mod p 2

(32)
Suy ra

p 1!

p 1

1p 1.2 p 1... p 1

pr1 1 pr2 1 ... prp 1 1 1 p r1 r2 ... rp 1 mod p 2

p 1

Nhng v p 1! pr 1 mod p 2 suy ra

p 1!
So snh vi cng thc ca

p 1

pr 1

p 1!

p 1

p 1

1 p 1 pr 1 pr mod p 2

ta c p r1 r2 ... rp 1 pr mod p 2 suy ra theo (32)

1p 1 2 p 1 ... p 1

p 1

p r1 r2 ... rp 1 p 1

pr p 1 p 1! p mod p 2
6. Chng minh rng mi s nguyn t p 5 u l c s ca s n p 111...1 c biu din trong
h thp phn vi p 1 ch s 1.
Chng minh. Xt s nguyn t p 5 . Khi 10, p 1 v 9n p 10 p 1 1 . Theo nh l 5 th

10 p 1 1 mod p suy ra p | 9n p . Nhng v p,9 1 (vi p l s nguyn t 5 ) ta c p | n p .


7. Chng minh rng nu p l s nguyn t v c l s nguyn th tn ti v hn s t nhin x tha
mn dy v hn cc ng d thc sau
(*)

x c mod p ,

x x c mod p ,

x x c mod p ,...
x

Chng minh. Vi p l s nguyn t v c l s nguyn cho trc th v p, p 1 1 suy ra tn ti


k
v hn s t nhin x 1 m x c mod p v x 1 mod p 1 . V vy x 1 mod p 1 vi

mod p .

k
k 1, 2,... Do x 1 p 1 lk m x 1 nn lk l s t nhin. V vy x x x xlk
x

p 1

CHNG 5. NG D | 139

Nu p | c th x 0 mod p v r rng x tha mn cc ng d thc (*). Nu c khng chia ht

cho p th c, p 1 v v x, p 1 nn xlk , p 1 . Vy theo nh l 5 suy ra xlk

p 1

1 mod p

do x x x c mod p vi mi k 1, 2,... Th 1, x, x x , x x ,... cho k ta nhn c (*).


x

Cc ng d thc dng (*) cng c nghin cu vi cc modulo ty (Schinzel v Sierpinski [4]).


8. Tm tt c cc s t nhin m mi s c ng mt biu din di dng tng hai bnh phng cc
s t nhin nguyn t cng nhau (khng xt cc biu din sai khc hon v)
Li gii. Ta s chng minh rng cc s cn tm l cc ly tha t nhin ca cc s nguyn t c
dng 4k 1 .
k
B 1. Nu p l s nguyn t c dng 4t 1 th vi k 1, 2,, s p c ng mt biu din thnh
tng hai bnh phng cc s t nhin nguyn t cng nhau.

Chng minh b 1. Theo nh l 11 th b ng vi k 1 . Xt k l s t nhin ty v gi s


k
2
2
b ng vi k . Khi tn ti cc s t nhin c v d tha mn c, d 1 v p c d . T
2
2
nh l 11 suy ra tn ti cc s t nhin a, b m a, b 1 v p a b . V vy

(33)

p k 1 a 2 b2 c 2 d 2 ac bd ad bc ad bc ac bd
2

Nu mi s ad bc v ac bd u chia ht cho p th ad bc mod p v ac bd mod p v suy


k
2
2
ra a 2cd b2cd (mod p) do p | cd (a 2 b2 ) . Nhng v p c d v c, d 1 nn c hai s c
2
2
2
2
v d u khng chia ht cho p . H qu l t p | a b v p | a b suy ra p | a v v

p a 2 b2 nn p | b , mu thun vi gi thit a, b 1 . V vy t nht mt trong cc s ad bc v


ac bd khng chia ht cho p . Nu l s ad bc th theo (33) s ac bd cng khng chia ht
cho p . Khi cc s ac bd v ad bc nguyn t cng nhau v theo (33) th cc c s chung ca
k 1
chng phi l c s ca p nhng p khng phi c s ca bt k s no trong chng. Tng
t nu ac bd khng chia ht cho p th cc s ad bc v ac bd nguyn t cng nhau. V vy
k 1
trong mi trng hp th cng thc (33) cho ta biu din ca p thnh tng hai bnh phng
k
cc s t nhin nguyn t cng nhau. Vy theo quy np chng t vi mi k 1, 2, th s p l
tng hai bnh phng cc s t nhin nguyn t cng nhau.
k
By gi gi s vi s t nhin k th s p c hai biu din phn bit thnh tng hai bnh phng
k
2
2
2
2
cc s t nhin nguyn t cng nhau. t p a b c d vi

a, b c, d 1

a b, c d , a c . Ta c
(34)

p 2k ac bd ad bc ad bc ac bd
2

k
v ac bd ad bc ab cd p .

Do t nht mt trong cc s ac bd v ad bc chia ht cho p . Nu c hai s u chia ht cho p

k
2
2
th theo (34) ta c ad bc mod p v ac bd mod p suy ra p | cd a 2 b 2 v t p c d
2
2
2
2
2
v c, d 1 suy ra p | a b m p | a b suy ra p | 2a m p l s l nn suy ra p | a . Nhng
2
2
khi v p | a b ta c p | b , mu thun vi a, b 1 . V vy c ng mt trong cc s ac bd
k
v ad bc chia ht cho p . Nhng tch ca chng l bi s ca p nn s chia ht cho p phi chia
k
k
ht cho p . Nu p | ac bd th theo (34) suy ra ad bc 0 do a / b c / d m

140 | nh l Wilson v nh l Fermat nh

a, b c, d 1

k
suy ra a c , mu thun vi gi thit. Nu p | ad bc th theo (34) ta c

ac bd 0 v suy ra a / b d / c m a, b c, d 1 suy ra a d , mu thun vi a c d . B


1 c chng.
chng minh nh l ta ch cn chng minh rng nu s t nhin l no c biu din duy nht
(khng tnh cc hon v) thnh tng bnh phng hai s t nhin nguyn t cng nhau th s l
ly tha t nhin ca mt s nguyn t c dng 4k 1 .
B 2. Nu m v n l cc s t nhin l nguyn t cng nhau v mi s u c th biu din thnh
tng bnh phng hai s t nhin nguyn t cng nhau th tch ca chng c t nht hai cch biu
din thnh tng bnh phng hai s t nhin nguyn t cng nhau (khng tnh cc hon v).
Chng minh b 2. Gi s m v n l cc s t nhin l nguyn t cng nhau v a, b, c, d l cc
s t nhin tha mn a, b c, d 1, m a 2 b2 , n c 2 d 2 . Gi s a b, c d ta c
(35)

mn ac bd ad bc ad bc ac bd
2

v
(36)

ac bd ad bc cdm abn

Cc biu din mn thnh tng cc bnh phng cho bi (35) l phn bit bi v nu
ac bd ad bc th ta c a b c d 0 v do a b hoc c d , iu ny l khng th v
cc s m v n l v nu ac bd ac bd (s ac bd 0 v a b, c d ) th ta c ac 0 , v l.
Vy chng minh B 2 ta ch cn chng minh ac bd , ad bc 1 v ad bc, ac bd 1.
Nu ac bd , ad bc 1 th cc s ac bd v ad bc c c s nguyn t chung p . V vy theo
(35) th p | mn v do p | m hoc p | n . Nu p | m th theo (36) ta c p | abn m p | m v

m, n 1 suy ra p | ab , do p | a hoc p | b m p | m a2 b2 suy ra p | a v p | b , mu thun


vi gi thit a, b 1 . Nu p | n th theo (36) ta c p | cdm m m, n 1 suy ra p | cd m
p | c 2 d 2 v c, d 1 suy ra mu thun. B 2 c chng minh.
By gi gi s s l n c biu din duy nht thnh tng hai bnh phng cc s t nhin nguyn t
cng nhau. Gi s n a 2 b2 l biu din duy nht v p l c s nguyn t ca n . Khi p l
s l. Nu p 4k 3 th ly tha c hai v ca ng d thc a 2 b2 mod p ln
1
2

p 1 2k 1

ln th ta c a p 1 b p 1 mod p nhng a, b 1 v a, p b, p 1, s

dng nh l 5 ta c a p 1 b p 1 1 mod p . V vy 1 1 mod p suy ra p | 2 , v l. Vy mi c


s nguyn t ca n u c dng 4k 1 . Do phn tch thnh tha s nguyn t ca n l
n q11 q22 ...qkk vi 1 , 2 ,..., k v k l cc s t nhin v cc s nguyn t qi i 1, 2,..., k u c
dng 4t 1 . Nu k 1 th khng cn g chng minh. Gi s k 1 . Khi cc s q11 , q22 ,...qkk l
i mt nguyn t cng nhau. T B 1 suy ra mi s u l tng hai bnh phng cc s t
nhin nguyn t cng nhau. Nn theo B 2 suy ra s q11 q22 ...qkk11 cng l tng hai bnh phng

cc s t nhin nguyn t cng nhau. V q11 q22 ...qkk11 , qkk 1 nn s q11 q22 ...qkk n c t nht
hai biu din phn bit thnh tng hai bnh phng cc s t nhin nguyn t cng nhau, mu
thun vi gi thit. Vy ta c k 1 v nh l c chng minh (Sierpinski [29]).
6. Cc s idonei

CHNG 5. NG D | 141

Ta s dng tn gi ny cho cc s d c tnh cht sau y: nu s l n 1 c biu din duy nht


2
2
(khng tnh cc hon v) di dng x y d vi x, y l cc s nguyn khng m v cc hng t l
nguyn t cng nhau th n l s nguyn t (1).
T nh l 11 suy ra 1 thuc lp cc s ny. Euler tm ra 65 s nh vy: 1, 2, 3, 4, 5, 6, 7, 8, 9, 10,
12, 13, 15, 16, 18, 21, 22, 24, 25, 28, 30, 33, 37, 40, 42, 45, 48, 57, 58, 60, 70, 72, 78, 85, 88, 93, 102,
105, 112, 120, 130, 133, 165, 168, 177, 190, 210, 232, 240, 253, 273, 280, 312, 330, 345, 357, 385,
408, 462, 520, 760, 840, 1320, 1365, 1848.
Cc s d c nghin cu ti 5 1010 (Weinberger [1]) nhng ta vn cha tm c s idonei
no ln hn 1848. Vo nm 1934 S.Chowla [1] chng minh rng s cc s idonei l hu hn,
sau Schowla cng vi W.E.Briggs chng minh rng c nhiu nht mt s nh vy ln hn
1065 m khng c c s chnh phng (Chowla v Briggs [1]). Cui cng P.Weinberger [1]
thay 1065 bi 1365. Cc s idonei m c c s chnh phng th hoc l nh hn 100 hoc c dng
4d vi d l s idonei chn khng c c s chnh phng (Grube [1] hoc Grosswald [1]). Cc
thng tin khc v cc s idonei c th tm trong cc bi bo ca I.G.Melnikov [1] v J.Steinig [1].
7. Cc s gi nguyn t v gi nguyn t tuyt i
T nh l 5 ta c nu n l s nguyn t th n | 2 2 . Cc nh ton hc Trung Hoa 25 th k
trc cho rng mnh ngc li cng ng. Mnh ny ng vi cc s t nhin n 300 (2). S
341
341 l hp s (bng 11 31 ) v 341| 2 2 . Tht vy v 11 v 31 l cc s nguyn t l nn theo
n

nh l 5 ta c 210 1 mod11 v r rng 210 1 mod 31 . V vy 2341 2 2340 2 mod11 v

2341 2 mod 31 . Do 2341 2 chia ht cho 11 v 31, nn n chia ht cho tch 11 31 341 .
n
Cc hp s n m n | 2 2 c gi l cc s gi nguyn t.

Tt c cc s gi nguyn t 2000 l: 341 11 31 , 561 3 1117 , 645 3 5 43 , 1105 5 13 17 ,


1387 19 73 , 1729 7 13 19 , 1905 3 5 127. P.Poulet [2] lp bng tt c cc s gi nguyn
t l nh hn 108 v C.Pomerance, J.L.Selfridge v S.S.Wagstaff Jr. [1] nng ln thnh 25 109 .
nh l 12. Tn ti v hn cc s gi nguyn t

(1)

B . Nu n l s gi nguyn t l th s m 2n 1 cng l s gi nguyn t t l. R rng m n .


Chng minh b . Gi s n l s gi nguyn t th n l hp s v do n c c s q m

1 q n . Ta c 1 2q 1 2n 1 m . T y suy ra m l hp s l v n l. Do v n l s gi

n
nguyn t nn 2n 2 / n l s nguyn. Ta thy s 2n 2 / n chn. T y suy ra 2n | 2 2 do

n | 2

n 1

1 . H qu l vi s nguyn k ta c 2

n 1

1 kn . V vy 2m1 22

22 kn v do

n
m1
m
2m1 1 2n 1 suy ra 2 1| 2 1 nn m | 2 2 , ngha l m l s gi nguyn t. R rng
2k

m n v n 2 ( n l hp s) nn ta c 2n n 1 v do m n . B c chng minh.
nh l 12 l h qu trc tip ca b vi lu tn ti cc s gi nguyn t l, v d n 341 .
K hiu P x l s cc s gi nguyn t nh hn x ta c c lng sau y bi C.Pomerance

log x log log log x


[3],[4]: exp(log x)5/14 P x x exp
vi x ln.
2log log x

(1)

Mt s nh ngha khc c trnh by bi rt nhiu tc gi nhng ni chung l khng chnh xc. nh ngha chnh
xc u tin c trnh by bi F.Grube [1] tng i phc tp. Theo cc s ny c gi l s Euler.
(2)

Lu rng vo cc nm 1680-81 Leibniz chng minh rng


t. Tuy nhin chng minh l sai (Dickson [7] tp 1 trang 64).
(1)
Cipolla [1], D.H.Lehmer [3], Sierpinski [6].

2n 2 khng chia ht cho n tr khi n l s nguyn

142 | Cc s idonei. Cc s gi nguyn t v gi nguyn t tuyt i

Ti tn nm 1950 cc s gi nguyn t l mi c bit ti. D.H.Lehmer l ngi u tin tm cc


s gi nguyn t chn. l s n 161038 . Khng d tm c s ny, nhng chng minh
n l s gi nguyn t th kh n gin v s cp. Tnh ton trc tip ta c

n 2 73 1103, n 1 32 29 617, 29 1 7 73, 229 1 233 1103 2089


V 9 | n 1 v 29 | n 1 suy ra 29 1| 2n1 1 v 229 1| 2n1 1 . Lu rng 73 | 29 1 v

1103 | 229 1 suy ra 2n 2 chia ht cho 73 v 1103. Nhng y l s chn nn n chia ht cho 2, v
n
vy n | 2 2 . Suy ra n l s gi nguyn t.

N.G.W.H Beeger [1] chng minh rng tn ti v hn s gi nguyn t chn, sau A.Rotkiewicz
[2] chng minh vi cc s t nhin bt k a v b th lun tn ti v hn s gi nguyn t chn
n tha mn n | a nb abn . Suy ra vi mi s t nhin l a th tn ti v hn s t nhin chn n m

n | a n a . (A.Rotkiewicz [7] chng minh vi cc s t nhin a, b ty v s nguyn t p th


lun tn ti v hn s n chia ht cho p m p | a nb abn ). A.Rotkiewicz [5],[6] chng minh tn
ti v hn s gi nguyn t c dng ax b x 0,1, 2,... vi a,b l cc s nguyn t cng nhau,
a 0 (Rotkiewicz [8]).
Cc s gi nguyn t cn c gi l cc s Poulet v Poulet lp bng cc s . Cc s c mi
a
c s d tha mn d | 2 2 c gi l cc s siu-Poulet (Duparc [2]). V d n 2047 . Tht vy
11
8
ta c 2047 2 1 23 89 suy ra theo nh l 5 th 11| 211 2 suy ra 211 2 | 22

11

2 v chng

a
t 2047 l s gi nguyn t. Cc c s t nhin ca 2047 l 1, 23,89 v 2047 . Theo nh l 5

23 | 223 2 v 89 | 289 2 suy ra 2047 l s siu-Poulet. Tn ti cc s Poulet nhng khng phi


siu-Poulet. Chng hn 561 3 1117 . Tht vy, 560 chia ht cho 2,10 v 16 nn theo nh l 5
suy ra 3| 22 1| 2560 1, 11| 210 1| 2560 1| 17 | 216 1| 2560 1 . V vy 561 3.11.17 | 2560 1| 2561 2 suy
ra 561 l s Poulet. Tuy nhin c s 33 ca 561 th khng phi c s ca 233 2 v 233 2 khng
chia ht cho 11. V vy 561 khng phi s siu-Poulet.
a

T nh l 5 suy ra cc s Poulet l tch ca hai s nguyn t phn bit th u l s siu-Poulet.


pq
Cu hi c t ra l c tn ti v hn cp s nguyn t phn bit p, q m pq | 2 2 hay khng.
Cu tr li cho cu hi ny l khng nh v c suy ra t nh l tng qut hn ca A.Rotkiewicz
[1]: cho trc ba s t nhin ty a, b, s . Khi tn ti v hn s t nhin n l tch ca s s nguyn
n 1
n 1
t phn bit tha mn n | a b .

nh l ny suy ra vi cc s t nhin ty a v s th tn ti v hn s t nhin n l tch ca cc s


n
nguyn t v n | a a (vi s = 2 xem Schinzel [9], D.H.Lehmer [3], Erdor [8]). T y suy ra tn
ti v hn s siu-Poulet.
Mt khc c th chng minh tn ti v hn s Poulet m khng phi siu-Poulet (bi tp 1)
n
Hp s n c gi l s gi nguyn t tuyt i nu vi mi s nguyn a th a a chia ht cho n.

S gi nguyn t tuyt i l s gi nguyn t, nhng iu ngc li khng ng. Chng hn ta c


341 l s gi nguyn t nhng n khng phi s gi nguyn t tuyt i v 11341 11 khng chia ht
cho 341 v ta c 112 3(mod31) suy ra 1110 (3)5 1130 1(mod31) , 11341 1111 1(mod 31) .
Nhng v 1130 1(mod31) , 11341 1111 7(mod 31) suy ra 11341 11 18(mod31) ..
D dng chng minh rng nu n l tch ca k s nguyn t phn bit q1 , q2 ,...qk vi k l s t nhin

1 v nu qi 1| n 1 , i 1, 2,..., k th n l s gi nguyn t tuyt i. Tht vy t nh l 5 suy ra

CHNG 5. NG D | 143

nu i 1, 2...k v s nguyn a khng chia ht cho qi th qi | a qi 1 1 m qi 1| n 1, qi | a n1 1 v ta


c qi | a n a . Tnh cht cui cng ng vi qi | a .
V vy 561 3 1117 l s gi nguyn t tuyt i v 560 chia ht cho 2,10 v 16. C th chng
minh 561 l s gi nguyn t tuyt i nh nht.
D thy vi mi s t nhin m th vi n 6m 112m 118m 1 s n 1 chia ht cho 36m nn
suy ra n chia ht cho 6m,12m v 18m. V vy nu cc s 6m+1,12m+1 v 18m+1 u l s nguyn
t th n 6m 112m 118m 1 l s gi nguyn t tuyt i (Chernick [1]).
Ta khng bit c tn ti v hn s gi nguyn t tuyt i hay khng. Tuy nhin t gi thuyt H
(Chng 3 mc 8) suy ra tn ti v hn s t nhin m m cc s 6m + 1,12m +1 v 18m +1 u l
s nguyn t. V vy t gi thuyt H suy ra tn ti v hn s gi nguyn t tuyt i.
Cc s 6m 1 , 12m 1 v 18m 1 u l cc s nguyn t vi m 1, 6, 35, 45, 51. Suy ra cc s
1729 7 13 19, 294409 37 73 109, 211 421 621, 271 541 811, 307 613 919 u l cc s gi
nguyn t tuyt i. Di y l mt s s gi nguyn t tuyt i khc
5 29 73, 5 17 29 113, 5 17 29 113 337, 5 17 29 113 337 673, 5 17 29 113 337 673 2689,
7 23 41, 7 31 73, 7 73 101, 7 13 31, 7 13 31 61181, 7 13 31 61181 541, 7 13 31 61181 541 2161,
13 37 61181 541 2161, 13 37 61, 13 37 91, 13 37 241, 13 61 397, 13 97 421, 43 3361 3907
n
n
Nu n l s gi nguyn t tuyt i th n | 2 2 v n | 3 3 . Ta cha chng minh c c tn ti

v hn cc hp s n m n | 2n 2 v n | 3n 3 hay khng?
Nu n l s gi nguyn t tuyt i v a l s nguyn nguyn t cng nhau vi n th v
n 1
n 1
a n n a a n1 1 chia ht cho n nn a 1 chia ht cho n . Hp s n tha mn n | a 1 vi

a, n 1 c gi l cc s Carmichael. Carmichael l ngi u tin lu s tn ti cc s nh


vy vo nm 1909. Ta thy mi s gi nguyn t tuyt i u l s Carmichael. iu ngc li
cng ng v c th chng minh s t nhin n l s Carmichael khi v ch khi n q1q2 ...qk vi k 3
v q1 , q2 ,..., qk l cc s nguyn t l phn bit tha mn qi 1| n 1, i 1, 2..., k (Carmichael [2],[3],
Sispanov [1], Dupare [1], Knodel [1], Sierpinski [12] trang 186-188). c lng tt nht v s cc
s Carmichael nh hn mt s cho trc c trnh by bi Pomerance [3].
n2
C tn ti cc s t nhin n 2 m vi mi s nguyn a, n | a a . Chng hn n =195. V

195 3 5 13 nn ta ch cn chng minh vi mi s nguyn a th s a193 a chia ht cho 3,5 v 13.


193
Gi p l mt trong cc s 3,5, 13. Khi p 1|192 v 192 4.48 . Nu p | a th p | a a . Nu p
p 1
khng chia ht cho a th theo nh l 5, p | a 1 v v p 1|192 , p | a192 1 suy ra p | a193 a . Do

193
193
p | a a vi mi s nguyn a v p=3,5,13. V vy 195 | a a vi mi s nguyn a.

Tng t v 399 3 7 19,18 | 396,1023 3 11 31,30 |1020 nn ta chng minh c vi mi s


397
1021
nguyn a th 399 | a a,1023| a a .

n2
Nu n l s t nhin 3 tha mn n | a a vi mi s nguyn a th vi

a, n 1

ta c

1 vi a, n 1 c D.C Morrow [1] gi l cc D-s. Ta chng


minh rng mi s c dng n = 3p vi p l s nguyn t 3 u l cc D-s. Nu p 3 ngha l
n|a

p 3

1 . S n 3 m n | a

n 3

n 9 th ta thy 9 | a6 1 vi mi a m a,9 1 . Gi s p l s nguyn t 3 v a l s nguyn


p 1
3 p 3
1 . Nhng v
tha mn a,3 p 1 . Khi a, p 1 v theo nh l 5, p | a 1 suy ra p | a

a,3 p 1

nn s a khng chia ht cho 3 v s p 1 chn (v p l s nguyn t l) do

144 | Cc s idonei. Cc s gi nguyn t v gi nguyn t tuyt i

3 | a3 p 1 1 . Chng t s a3 p 3 1 chia ht cho p v 3 m v p,3 1 nn s chia ht cho 3p.


3 p 3
1 vi mi a tha mn a,3 p 1 v do 3p l D-s.
Suy ra 3 p | a

A.Makowski [7] chng minh nh l tng qut hn ni rng vi mi s t nhin k 2 th lun


nk
tn ti v hn hp s n m vi mi s nguyn a tha mn a, n 1 th n | a 1 (chng minh
nh l ny c trong Chng 6 mc 5).
Bi tp. 1. Chng minh rng khng tn ti s siu-Poulet chn.
2n
2n
Chng minh. Gi s phn chng rng 2n l mt s siu-Poulet. Khi 2n | 2 2 suy ra n | 2 1
n
n 1
v chng t n l s l. V 2n l s siu-Poulet nn n | 2 2 suy ra v n l nn ta c n | 2 1 . Do

n 1
n 1
v n | 2 1 nn n | 22 n1 2n1 2n1 2n 1 . Li v n l nn ta c n | 2 1 v so snh vi

n | 2 2 chng t n 1 , iu ny l v l v 2n l hp s.
n

Ta cp ti nh l Beeger ni rng tn ti v hn s Poulet chn.


Bi tp 1 suy ra cc s ny khng phi s siu-Poulet.
2. Chng minh rng n 2 73 1103 2089 l s gi nguyn t. (S.Maciag)
Chng minh. Ta c n = 2089m vi m

l s gi nguyn t v 9 | m 1, 29 | m 1. V vy

n 1 m 1 2089 2088. Do 2088 23.32.29 m 9 | m 1 v 29 | m 1 nn suy ra 9 | n 1 v


29 | n 1 . V vy t 29 1 7.73 v 229 1 233.1103.2089 suy ra 73| 2n1 1, 1103| 2n1 1 v v

2089 | 229 1, 2089 | 2n1 1. By gi da vo phn tch thnh tha s nguyn t ca n suy ra
n | 2n 2 .
3. Chng minh rng tn ti v hn s Mersenne l s Poulet.
Tnh cht ny suy ra trc tip t b trong chng minh nh l 12 v tnh cht tn ti cc s
Poulet l (chng hn 341).
Tuy nhin ta khng bit c tn ti v hn s Mersenne l s siu-Poulet hay khng.
n
4. Chng minh rng n | 2 1 khng ng vi mi s t nhin n 1 .

n
Chng minh. Gi s phn chng rng n l s t nhin ln hn 1 v tha mn n | 2 1 . Gi p l

c s nguyn t nh nht ca n v l s nh nht m p | 2 1 . V p >1 suy ra 1 . Hn na


n
t p | 2 1 suy ra | n . Nu n chia cho d r vi 0 r th n=k +r suy ra 2n 1 2k 2r 1 .

r
n

n
Nhng v p | 2 1 nn ta c 2 1 mod p v suy ra 2 1 1 mod p m v p | 2 1 suy ra

p | 2r 1 v ta c mu thun vi nh ngha ca . T nh l Fermat nh suy ra p | 2 p 1 1 v ta


r
c p | 2 1 (v n l v do p l). V vy t nh ngha ca suy ra p 1 nn 1 p , mu
thun vi nh ngha ca p.
n
Ghi ch. D dng chng minh rng tn ti v hn s t nhin n tha mn n | 2 1 chng hn cc

k
n
s n 3 vi k=0,1, 2, . Khng kh chng minh tn ti v hn cc s t nhin n m n | 2 2 .
n
Tht vy, ta thy tnh cht ny ng vi n = 2 v nu n l s t nhin chn tha mn n | 2 2 v

n 1| 2n 1 th cc s m 2n 2 tha mn m | 2m 2 v m 1| 2m 1 . V vy ta nhn c cc s
n=2,6,66, .
n 1
C th chng minh khng tn ti s t nhin n 1 m n | 2 1 .
n 1
5. Chng minh rng tn ti v hn hp s n tha mn n | a a vi mi s nguyn a.

CHNG 5. NG D | 145

Gi . Ch cn t n=2p vi p l s nguyn t l.
8. nh l Lagrange
nh l 13 (Lagrange). Nu n l s t nhin v f(x) l a thc bc n bin x vi cc h s nguyn, v
nu h s ca x n khng chia ht cho p th ng d thc f x 0 mod p c nhiu nht n nghim.
Chng minh. T h qu ca nh l 2 suy ra nh l 13 ng vi n=1. K hiu n l s t nhin ty
1 v gi s nh l 13 ng vi a thc bc n 1 . Gi s f x ao x n a1 x n1 ... an1 x an l
a thc h s nguyn tha mn ao khng chia ht cho s nguyn t p v gi s ng d thc
(37)

f x 0 mod p

c nhiu hn n nghim. Khi tn ti n 1 s x1 , x2 ,..., xn1 l cc nghim phn bit ca ng d


thc (37). c bit f x1 0 mod p . Ta c

f x f x1 a0 x n x1n a1 x n1 x1n1 ... an1 x x1 .

Nhng v x k x1k x x1 x k 1 x k 2 x1 ... x1k 1 , suy ra


(38)

f x f x1 x x1 g x ,

vi g x l a thc bc n 1 vi bin s x v cc h s nguyn. Hn na h s ca x n 1 l a0 v


theo gi thit th h s ny khng chia ht cho p . V vy theo (38) v lu f x1 0 mod p ,
ng d thc (37) tng ng vi ng d thc
(39)

x x1 g x 0 mod p .

Do cc s x1 , x2 ,..., xn1 l nghim ca ng d thc (39). Vi i 2,3,..., n 1 ta c

p xi x1 g xi m x1 , x2 ,..., xn1 l cc nghim phn bit ca (37) suy ra p g xi vi

i 2,3,..., n 1. Chng t ng d thc g x 0 mod p c t nht n nghim phn bit, mu


thun vi gi thit nh l 13 ng vi cc a thc bc n 1 .
Do ta kt lun ng d thc (37) c khng qu n nghim, v theo quy np suy ra nh l 13.
Trong nh l 13 th gi thit modulo p nguyn t l cn thit. Chng hn ng d thc

x 2 1 0 mod8 c bn nghim 1,3,5,7. Tng t ng d thc x 2 3x 2 0 mod 6 c bn


nghim 1,2,4,5 trong khi h s ca ly tha cao nht l nguyn t cng nhau vi modulo.
C th chng minh nu m l hp s th ch khi m 4 th nh l sau y ng: nu f x l a
thc bc n vi h s nguyn m h s ca ly tha cao nht nguyn t cng nhau vi m th ng d
thc f x 0 mod m c nhiu nht n nghim phn bit (Sierpinski [12] trang 180-181)
H qu. Nu ng d thc bc n vi h s nguyn v modulo nguyn t p c nhiu hn n nghim
th tt c cc h s ca n u chia ht cho p .
Chng minh. Gi s (37) l ng d thc tha mn cc iu kin trn v t

f x a0 x n a1 x n1 ... an1 x an
Gi s trong cc s a0 , a1 ,...an c s khng chia ht cho p khi gi am l s u tin trong dy

a0 , a1 ,...an m khng chia ht cho p . Khi vi mi s nguyn x ta c


f x am xnm am1 x nm1 ... an1 x an mod p

146 | nh l Lagrange

Nu n m th f x an mod p v v ng d thc (37) c nhiu hn n nghim nn tn ti s


nguyn x tha mn f x 0 mod p suy ra an 0 mod p . Do m n . H qu l a thc

am xnm ... an1 x an tha mn cc iu kin ca nh l 13 do n c nhiu nht n m n


nghim phn bit, mu thun vi gi thit. H qu c chng minh.
Nu tt c cc h s ca ng d thc l chia ht cho modulo th hin nhin ng d thc ng.
Tuy nhin iu ngc li khng ng. Chng hn ng d thc x 2 x 0 mod 2 lun ng.
Tng t theo nh l 5a th ng d thc x17 x 0 mod17 lun ng.
ng dng n gin ca nh l 5a suy ra mi ng d thc vi modulo nguyn t p l tng
ng vi mt ng d thc bc khng ln hn p . Tht vy, theo nh l 5a th vi s nguyn x
ta c x p x(mod p) , x p 1 x 2 (mod p) v c nh vy. Chng t mi ly tha bc p ca bin x
c th thay bi mt ly tha bc p 1 ca x .
nh l 14. Nu m ab vi a, b l cc s t nhin nguyn t cng nhau th s cc nghim ca ng
d thc
(40)

f x 0 mod m

vi f x l a thc bin x h s nguyn l bng vi tch cc nghim ca cc ng d thc


(41)

f x 0 mod a

v s nghim ca ng d thc
(42)

f x 0 mod b

Chng minh. Nu x l nghim ca ng d thc (40) th n l nghim ca cc ng d thc (41)


v (42) bi v nu m | f ( x) th a | f ( x) v b | f ( x) . V vy mi nghim ca ng d thc (40)
tng ng vi mt cp u, v , u l nghim ca ng d thc (41) v v l nghim ca ng d thc
(42). C th hn u l s d nhn c khi chia x cho a, v l s d nhn c khi chia x cho b .
D thy cc cp u, v phn bit tng ng vi cc nghim phn bit ca ng d thc (40). Tht

u, v th x y mod a v
x y mod m , mu thun vi gi thit

vy nu hai nghim phn bit x, y cng tng ng vi cp

x y mod b m a, b 1 suy ra m ab | x y v do
cc nghim x, y phn bit.

By gi gi s u l nghim ca ng d thc (41) v v l nghim ca ng d thc (42) th v


a, b 1 , theo nh l s d Trung Hoa (Chng 1 mc 12) suy ra tn ti s nguyn x tha mn

x u(mod a) v x v(mod b) . Suy ra (theo nh l 1) f ( x) f (u)(mod a) v f ( x) f (v)(mod b) .


Nhng v f (u) 0(mod a) v f (v) 0(mod b) nn f ( x) 0(mod a) v f ( x) 0(mod b) .
H qu l v (a, b) 1 v ab m ta c f ( x) 0(mod m) .
Vy ta chng minh c mi cp u, v vi u l nghim ca ng d thc (41) v v l nghim
ca ng d thc (42) tng ng vi ng mt nghim ca ng d thc (40).
T quan h mt-mt ny suy ra s nghim ca ng d thc (40) ng bng s cc cp u, v vi

u l nghim ca ng d thc (41) v v l nghim ca ng d thc (42).


nh l 14 c chng minh.

CHNG 5. NG D | 147

H qu. Nu m q11 q22 ...qkk l phn tch thnh tha s nguyn t ca s nguyn m th s nghim
ca ng d thc (40) ng bng vi tch cc s nghim ca k ng d thc sau y

f x 0 mod q11 , f x 0 mod q22 , ..., f x 0 mod qkk

T y ta c mt phng php quy vic tnh cc nghim ca ng d thc modulo m ty v


vic tm cc nghim ca cc ng d thc vi modulo l ly tha ca cc s nguyn t.
Bi tp. Chng minh rng vi mi s t nhin n th lun tn ti modulo m m ng d thc
x 2 1 mod m c nhiu hn n nghim.
Chng minh. Nu p l s nguyn t l th ng d thc x 2 1 mod p c ng hai nghim l 1 v

p 1 (xem mc 5). T h qu ca nh l 14 suy ra ng d thc x 2 1 mod p2 p3.... ps 1 c ng

2 s nghim. V vy ta ch cn chn s t nhin s m 2s n . Chng hn ng d thc


x 2 1 mod105 c tm nghim v 105 p2 p3 p4 . Cc nghim ny l 1, 29, 34, 41, 64, 71, 76, 104.
9. ng d thc bc hai
Xt ng d thc bc hai

ax2 bx c 0 mod m

(43)

vi m l s t nhin cho trc v a, b, c l cc s nguyn. Gi s a 0 mod m v nu ngc li,


2
tc l a 0 mod m , th (43) tr thnh ng d thc bc nh hn hai. V m | ax bx c tng

ng vi 4am | 4a ax2 bx c nn ng d thc (43) tng ng vi ng d thc


(44)

4a ax2 bx c 0 mod 4am

2
t D b 4ac th t ng thc 4a ax 2 bc c 2ax b b 2 4ac suy ra ng d thc

(44) c th vit li di dng


(45)

2ax b

D mod 4am

Gi x l nghim ca ng d thc (43) v t z 2ax b . Khi theo (45) th z l nghim ca


ng d thc trng phng sau
(46)

z 2 D mod 4am

V vy mi nghim x ca cc ng d thc (43) tng ng vi nghim ca ng d thc (46).


chng minh iu ngc li cng ng, ngha l vi nghim cho trc z ca ng d thc (46)
ta tm tt c cc nghim x ca (43) tng ng vi z . Ta s gii ng d thc
2ax b z mod 4am . ng d thc ny l gii c khi 2a, 4am | z b , ngha l 2a | z b . V
vy ta kt lun rng nghim ca ng d thc bc hai c th quy v cc nghim ca mt ng d
thc bc mt v mt ng d thc trng phng dng (46). Theo ghi ch ca nh l 14 th
nghim ca ng d thc (46) c quy v nghim ca cc ng d thc
(47)

z 2 D mod p

vi p l s nguyn t v l s t nhin.
Ta s gii ng d thc (47). u tin gi s p | D . Khi D p D1 vi l s t nhin v D1

khng chia ht cho p . Nu th D 0 mod p

v do (47) tr thnh z 0 mod p .

148 | ng d thc bc hai

ng d thc ny c th gii mt cch n gin. Nu th ng d thc (47) tng ng


vi phng trnh
(48)

z 2 p D1 tp

vi t l s nguyn ty v D1 tp khng chia ht cho p (v D1 khng chia ht cho p ). V vy

l ly tha cao nht ca p m p l c s ca z 2 . Do chn. Ta c 2 vi l s t


nhin. V vy z p z1 v do theo (48) th z12 D1 tp . T y suy ra z12 D1 (mod p ) . V
vy nghim ca ng d thc (47) c th quy v nghim ca ng d thc vi cng dng trong
v phi khng chia ht cho p . Ta gi s trong ng d thc (47) th D 0 mod p . Nu z tha
mn ng d thc ny th n tha mn ng d thc z 2 D mod p , chng t D l thng d bc
hai modulo p . T y suy ra iu kin cn ng d thc (47) gii c ( D khng chia ht cho
p ) l D l thng d bc hai modulo p . Ta chng minh iu kin ny cng l iu kin . Tht
vy, ta ch cn chng minh rng nu ng d thc
(49)

z 2 D mod p 1

vi l s t nhin 1 l gii c th ng d thc (47) cng gii c. Xt ring hai trng


hp p l s nguyn t l v p 2 . Trong trng hp th nht p l th gi y l s nguyn tha
mn (49). Khi
(50)

y 2 D mod p 1

V vy suy ra
(51)

y2 D
p 1

l s nguyn. K hiu x l nghim ca ng d thc


(52)

2 xy M 0 mod p

V D khng chia ht cho p, y khng chia ht cho p , suy ra v p l nn 2 y khng chia ht cho p
do ng d thc (25) l gii c. t z y p 1 khi z 2 y 2 2 p 1 xy p2 2 x2 .
Theo (51) th y 2 D Mp 1 suy ra ng d thc sau ng
(53)

z 2 D 2 xy M p 1 x2 p2 2

Theo (52) th s 2xy M chia ht cho p . V 2 2 2 (do 1), p | p 2 2 . Do


theo (53) th z tha mn ng d thc (47). iu kin c chng minh.
Ta c kt qu sau y
nh l 15. ng d thc (47), vi p l s nguyn t l, l s t nhin v D l s nguyn khng
chia ht cho p , l gii c khi v ch khi D l thng d bc hai modulo p .
Ta chng minh vi cc iu kin trong nh l 15 th ng d thc (47) c ng hai nghim.
Nu z l nghim ca ng d thc (47) th r rng z1 z l nghim ca ng d thc . Hn
na z v z1 khng ng d vi nhau modulo p v nu ngc li th ta c p | 2 z , m p l nn

p | z v do p | D , mu thun vi gi thit. Vy ta thy tn ti t nht hai nghim phn bit ca


ng d thc (47) l z v z1 . Ta s chng minh chng l tt c cc nghim ca (47). Gi s t l

nghim ca ng d thc (47) th t D mod p

m z

D mod p suy ra t 2 z 2 mod p .

CHNG 5. NG D | 149

V vy p | t z t z . Nu cc s t z v t z u chia ht cho p th p 2 z m p l nn p z
v do p D , mu thun vi gi thit. Vy mt trong cc s t z v t z l khng chia ht cho

p . Nu t z khng chia ht cho p th t p | t z suy ra t z mod p , nu t z khng chia

ht cho p th p | t z suy ra t z mod p . Vy mi nghim ca ng d thc (47) ng d

modulo vi z hoc z . Chng t (47) c ng hai nghim.


By gi t p 2 . Khi vi 1 th t cng thc (47) suy ra z 2 D mod 2 vi D khng
chia ht cho 2, tc l D l. H qu trc tip l ng d thc ny c ng mt nghim l z 1.
Vi 2 th ng d thc c dng z 2 D mod 4 . Nhng bnh phng ca mt s nguyn l
ng d vi 0 hoc 1 modulo 4 m D l nn ng d thc l gii c ch trong trng hp D
c dng 4k 1 . Khi ng d thc c ng hai nghim l z 1 v z 3 .
Vi 3 th ng d thc c dng z 2 D mod8 . V D l nn z l suy ra v bnh phng ca
mt s l l 1 mod8 nn ng d thc ban u l gii c ch khi D c dng 8k 1 . Trong
trng hp th ng d thc c bn nghim l 1,3,5,7.
Vi 3 ta xt ng d thc

z 2 D(mod 2 ) vi 3

(54)

Ta thy t ng d thc (54) suy ra ng d thc z 2 D mod8 . ng d thc ny gii c ch


khi D 8k 1 . Ta chng minh rng y cng l iu kin cho tnh gii c ca (54). Gi s
D 8k 1 v ng d thc

z 2 D mod 2 1

(55)

2
1
l gii c (iu ny ng vi 4 ). Khi tn ti s nguyn y tha mn y D mod 2
v

v D l nn y l. t
(56)

y2 D
2 1

Khi M nguyn. Hn na gi x l nghim ca ng d thc bc mt bin x


(57)

xy M 0 mod 2

ng d thc ny gii c v h s y ca bin x v modulo 2 l nguyn t cng nhau.


t z y x2 2 . Theo (56) ta c
(58)

z 2 y 2 xy 2 1 x2 22 4 D xy M 2 1 x 2 22 4

v v 2 4 4
0 mod 2 . V vy t (58) suy ra

Nhng theo (57) th xy M chn suy ra xy M 2 0 mod 2


2 2 4
2 2 4
(ng v 4 ) nn x 2
chia ht cho 2 . H qu l x 2

(54) v nh th chng t vi mi 3 th tnh gii c ca (55) suy ra tnh gii c ca (54).


2
3
Nhng ta gi s D 8k 1 nn ng d thc z D mod 2 gii c v do theo quy np

(vi D 8k 1) th ng d thc (54) l gii c vi mi s t nhin 3 . Ta c nh l

nh l 16. ng d thc z D mod 2 , vi D l v l s t nhin, l gii c khi v ch khi

D c dng 2k 1 , 4k 1 hoc 8k 1 tng ng vi 1 , 2 hoc 2 .

150 | ng d thc bc hai

Ta s chng minh vi 3 th ng d thc (54) (vi D 8k 1) c ng bn nghim.


Ta chng minh (vi cc gi thit trn) ng d thc ny c t nht mt nghim. K hiu
nghim ny l z0 . Gi s z l nghim bt k ca ng d thc (54). Ta c z02 D mod 2 v suy

ra theo (54) th 2 | z z0 z z0 . V D l nn z v z0 cng l v suy ra z z0 v z z0 chn.


Cc s ny khng cng chia ht cho 4 v nu ngc li th 2 z , v l. Vy mt trong cc s khng

1
1
z z0 l. Nhng v 2 1 | z z0 z z0
2
2

1
1
nn 2 | z z0 v do z z0 2 t vi t l s nguyn. Nu t chn th z z0 mod 2 . Nu
chia ht cho 4. Nu z z0 khng chia ht cho 4 th

t l th z z0 2 1 mod 2 . By gi ta xt trng hp cn li, ngha l z z0 khng chia ht

1
1
z z0 l v do v 2 1 | ( z zo ) z zo ta suy ra 2 1 | z zo v do
2
2
1
z z0 2 u vi u l s nguyn. Nu u chn th z z0 mod 2 . Nu u l th

cho 4. Khi s

z z0 2 1 mod 2 . Vy ta chng minh c mi nghim z ca ng d thc (54) u phi

tha mn mt trong cc ng d thc sau


(59)

z z0 mod 2 , z z0 2 1 mod 2
z z0 mod 2 ,

z z0 2 1 mod 2

Chng t s cc nghim ca ng d thc ny l khng ln hn bn. Mt khc d dng kim tra


rng cc s c cho bi cc ng d thc (59) u tha mn ng d thc (54) (nu n ng vi
z0 ) v v 3 nn cc s i mt khc nhau modulo 2 . Vy chng l cc nghim phn bit
ca ng d thc (54).
Ta c nh l sau y
nh l 17. ng d thc z 2 D mod m , vi D l s nguyn v D, m 1 , l gii c khi v
ch khi (i) D l thng d bc hai vi mi modulo l c s nguyn t l ca m v (ii) D c dng
4k 1 trong trng hp m chia ht cho 4 nhng khng chia ht cho 8 v c dng 8k 1 nu m
chia ht cho 8. S cc nghim ca ng d thc ny l 2 vi l s cc c s nguyn t l ca
m v 0 trong trng hp m khng chia ht cho 4 v 1 nu m chia ht cho 4 nhng khng
chia ht cho 8 v cui cng 2 vi m chia ht cho 8.

CHNG 6
HM CH EULER V NH L EULER
1. Hm ch Euler
Vi mi s t nhin n th s cc s t nhin n v nguyn t cng nhau vi n c k hiu l n .
Hm s n c gi l hm ch Euler (Euler l nh ton hc u tin nghin cu v cc tnh cht
ca hm s ny vo nm 1760). K hiu n c xut bi Gauss vo nm 1801 do i khi
hm s ny cng c gi l hm Gauss.
T nh ngha ta c ngay

1 1, 2 1, 3 2, 4 2, 5 4, 6 2, 7 6, 8 4, 9 6, 10 4
Nu n l s nguyn t th mi s t nhin nh hn n u nguyn t cng nhau vi n do

n n 1

(1)

Nu s t nhin n l hp s, ngha l n c c s d tha mn 1 d n , khi trong cc s


1, 2, n c t nht hai s l n v d khng nguyn t cng nhau vi n do n n 2 . Vi n 1
ta c n n n 1 . Vy (1) ng khi v ch khi n l s nguyn t.
T y suy ra tnh cht: s t nhin n 1 l s nguyn t khi v ch khi vi mi s t nhin a n th
a n1 1 mod n .
Tht vy, t ng d thc ny suy ra a, n 1 v do nu n ng vi mi a n th n n 1 v
do n l s nguyn t. iu kin c chng minh. iu kin cn c suy ra t nh l Fermat
nh (nh l 5 Chng 5).
D dng tnh gi tr n vi mi ly tha cc s nguyn t n p k , k l s t nhin.
Tt c nhng s trong dy 1, 2, , p k m khng nguyn t cng nhau vi p k l cc s chia ht cho p
tc l cc s c dng pt vi t l s t nhin bt k tha mn pt p k , ngha l t p k 1 . R rng c
ng p k 1 s nh th. V vy trong dy 1, 2,..., p k c ng p k 1 s khng nguyn t cng nhau vi p k

k
k
k 1
v suy ra p p p .

k
k 1
nh l 1. Nu p l s nguyn t v k l s t nhin th p p p 1 .

tnh n vi n l s t nhin ty ta chng minh b sau


B . Vi m l s t nhin v l l s t nhin nguyn t cng nhau vi m, r l s nguyn ty th khi
chia cc s
(2)

r, l+r, 2l+r, ., (m 1) l+r

cho m ta s nhn c dy cc s d
(3)

0, 1, 2, ,m 1

Chng minh. Gi s vi cc s nguyn k v h vi 0 k h m ta c cc s kl + r v hl + r c cng s


d khi chia cho m. Khi hiu h k l chia ht cho m suy ra v l , m 1, m | h k , v l v

152 | Hm ch Euler

0 h k m . V vy khi chia cc s trong (2) cho m ta nhn c cc s d khc nhau. M c tt c


m s trong (2) nn dy cc s d nhn c chnh l (3). B c chng minh.

nh l 2. Nu l v m l cc s t nhin nguyn t cng nhau th


(4)

lm l m

Chng minh. V 1 1 nn nh l 2 ng nu t nht mt trong cc s l , m bng 1 . Gi s l 1 v

m 1. Ta bit lm l s cc s trong bng

m nguyn t cng nhau vi lm , ngha l s cc s trong bng m nguyn t cng nhau vi c l v m.


Vi r l s t nhin cho trc l ta xt ct th r trong bng. Nu r , l 1 th tt c cc s trong ct
ny u nguyn t cng nhau vi l . Nu r , l 1 th khng c s no trong ct l nguyn t cng
nhau vi l . S cc s t nhin r l m r , l 1 l l . y cng l s cc ct m mi phn t trong
ct u nguyn t cng nhau vi l . Xt mt trong cc ct nh vy, gi s l ct th r . Theo b
th cc s d nhn c khi chia cc s trong ct cho m chnh l cc s 0,1,..., m 1 v suy ra s cc s
trong ct m nguyn t cng nhau vi m chnh l m . Suy ra trong l ct nh vy th mi ct c

m s nguyn t cng nhau vi m . Vy tng s cc s trong bng m nguyn t vi c m v l l

l m v nh vy nh l c chng minh.
T nh l 2, theo quy np, ta c h qu
H qu. Nu m1 , m2 ,..., mk l cc s t nhin i mt nguyn t cng nhau th

m1m2...mk m1 m2 ... mk
Vi s t nhin n 1 c phn tch thnh tha s nguyn t l n q11 q22 ...qkk th p dng cng thc

va chng minh vi cc s m1 qii , i 1, 2,..., k ta c n q1 1 q2 2 ... qk k .

Nhng theo nh l 1 th qi i qi i1 qi 1 vi i 1, 2, k , ta c nh l sau y

nh l 3. Vi s t nhin n 1 c phn tch thnh tha s nguyn t l n q11 q22 ...qkk th


(5)

n q1 1 q1 1 q2
1

2 1

q2 1 ...qk 1 qk 1
k

Cng thc ny c th vit li thnh


(6)

n n 1

1
1
1
1 ... 1
q1 q2 qk

T nh l 3 d thy nu a, b 1 th ab a b v nu m | n th (m) | (n) .

CHNG 6. HM CH EULER V NH L EULER | 153

nh l 4. Ta c lim n .
n

1
n vi mi s t nhin n . R rng bt
2
l phn tch thnh tha s nguyn t ca n

Chng minh (J. Browkin). Ch cn chng minh rng n


ng thc ng vi n 1. Gi s n 1 v 20 q11 q22 ...qkk

vi 0 l s nguyn khng m v 1 , 2 ,... k l cc s t nhin. Vi s t nhin ty a 2 ta c


1
a 1 a v vi mi s t nhin b th b 2b . V vy theo nh l 3 ta c
2

n 2 1 q1 1q2 1...qk
0

20 1 q1

1
2

q2

1
2

...qk

q1 1 q2 1 ... qk 1
1
2

20 1 q12 q22 ...qk2

1
n
2

Trong mi lin h vi nh l 4 ta lu rng tn ti v hn s t nhin n m n n 1 .


chng minh tnh cht ny ta chng minh
nh l 5. Nu n l hp s t nhin th
(7)

n n n

Chng minh. K hiu n l hp s v p1 l c s nguyn t nh nht ca n. Ta bit p1 n do

1
n
theo cng thc (6) th n n 1 n
suy ra (7) ng. By gi gi s n l s nguyn t
p1
n

7 . Khi n 1 l hp s v n 1 9 . Vy

n 1 3 v theo (7) th n 1 n 1 n 1 n 2 .

Nhng v n n 1 nn n n 1 . Bt ng thc ng vi mi s nguyn t n 7 . N cng


ng vi n 5 v n 7 . Do n ng vi v hn s t nhin n .
ng thc n n 1 vi n l s t nhin c nghin cu bi nhiu tc gi (Klee [2], Moser
[1], Lal, Gillard [1], Yorigana [1], Baillie [1],[2]). Tt c cc nghim n 10000 ca phng trnh ny l
n 1, 3, 15, 104, 164, 194, 255, 495, 584, 975, 2204, 2625, 2834, 3255, 3705, 5186, 5187 . S t
nhin n nh nht m n n 1 n 2 l 5186. D thy 5186 tha mn tnh cht ny v ta
c cc phn tch thnh tha s nguyn t 5186 2.2593 , 5187 3.7.13.19 , 5188 2 2.1297 v
2592 2.6.12.18 2.1296 .
8
Ta cha bit c tn ti cc s t nhin n m n n 1 hay khng. Vi n 2 10 th c 391 s

nh vy v trong ch c n 1586 tha mn n n 1 n 2 . Vi phng trnh

n 2 n th ta bit c 7998 nghim n 4 106 . Vi n 100 th n 4,7,8,10, 26,32,70,74 .


6
Phng trnh n 3 n ch c hai nghim n 3 v n 5 vi n 10 .

D dng chng minh vi mi s t nhin cho trc k th phng trnh n k n c t nht mt


nghim t nhin n (bi tp 11). T gi thuyt H (Chng 3 mc 8) th tn ti v hn nghim t nhin
ca phng trnh trn vi k chn (Schinzel v Sierpinski [3] trang 195). A.Schinzel v Adrzej

154 | Hm ch Euler

Wakulicz [1] chng minh rng vi mi s t nhin k 2 1058 th phng trnh n k n c


t nht hai nghim t nhin n (Shinzel [8]).
Nu mt trong hai s n v n 2 l nguyn t th n 2 n 2 . Phng trnh ny cng ng
vi mt s hp s chng hn n 12,14, 20, 44 . Moser [1] chng minh rng khng tn ti hp s l
n 10000 tha mn phng trnh ny. T y t ra gi thuyt ni rng khng tn ti s l n no
ngoi tr cc cp s nguyn t sinh i n, n 2 m n 2 n 2 . Trong mi lin h ny
A.Makowski [4] t cu hi cn tn ti hp s t nhin n m (n 2) (n) 2 v
(n 2) (n) 2 hay khng.
Nu n l s nguyn t th (n) n 1 do (n) | n 1 . Ta cha bit c tn ti v hn cc hp s n
m (n) | n 1 hay khng. D.H.Lehmer [1] t ra gi thuyt khng tn ti nhng s nh vy.
G.L.Cohen v P.Hagis Jr. [1] chng minh nu tn ti cc s nh vy th cc s c t nht 14 c s
nguyn t phn bit. Mt khc d dng tm c cc s t nhin n m n | n . Tt c cc s c tnh
cht ny l n 2 , 0,1, 2,... v n 2 3 vi , l cc s t nhin (Sierpinski [26], 196-197).

T (5) suy ra nu n 2 vi l s t nhin 1 th n 2 1 . Do 2 | (2 ) vi 2,3,... . Tuy

nhin nu n c c s nguyn t l p th p 1 chn v do theo (5) th p 1| (n) v do


2 | (n) . V mi s t nhin 2 hoc l ly tha bc k ca 2 vi k 1 hoc c c s nguyn t l nn
suy ra vi mi s t nhin n 2 th 2 | n .
Do 1 2 1 suy ra phng trnh x m, m l, l gii c ch khi m 1. V vy tn ti v
hn s t nhin (l ) m m phng trnh x m khng c nghim t nhin x .
Mt khc c th chng minh tn ti v hn s t nhin chn m m phng trnh x m khng c
nghim t nhin x . Ta chng minh tnh cht ny bng cch chng minh phng trnh khng c
nghim khi m 2 52 k vi k 1, 2, . T (5) suy ra nu n 2 52 k vi k l s t nhin th n c
ng mt c s nguyn t v nu q1 v q2 l cc c s nguyn t phn bit ca n th theo (5) suy ra

(q1 1)(q2 1) | (n) 2.52 k v do 4 | (n) , v l. V vy n 2 p vi l s nguyn 0 v l

s t nhin. Hn na 1 v nu ngc li th vi 2, 2 1 p 1 | n v do 4 | n , v l.

Nu 0 ta c n p v nu 1, n 2 p th n p 1 p 1 2 52 k . Nu 1, p 5 v do
2k
p 1 4 , v l. Vy 1 suy ra p 2.5 1 , v l v s 52 k 5k ng d 1 modulo 3, suy ra
2

3 | p v do p 3 , iu ny khng ng. Vy phng trnh n 2 52 k , vi k 1, 2, khng c


nghim t nhin.
S dng phng php tng t, nh l mnh hn sau y c chng minh bi A.Schinzel [6].
nh l ch ra vi mi s t nhin s th tn ti s t nhin m chia ht cho s v phng trnh
n m khng c nghim t nhin n. . nh l ny l h qu trc tip ca kt qu c trnh by
bi S.S.Pillai [2] theo mt cch khc: nu g x k hiu s cc s t nhin m x m phng trnh

n m l gii c th lim
x

g x
0.
x

CHNG 6. HM CH EULER V NH L EULER | 155

Tht vy, H.Maier v C.Pomerance [1] chng minh tn ti s thc C 0,81781465 m vi mi

0 v x x0 ( ) th

x
2
exp C log log log x g x
log x
x
2

exp C log log log x


log x

T nh l 4 suy ra vi mi s t nhin m th s cc nghim t nhin ca phng trnh n m l


hu hn. Ngc li, nh l 4 l h qu trc tip ca tnh cht ny. T nh l ca Pillai suy ra
nh l 6. Vi mi s t nhin s th lun tn ti s t nhin m m phng trnh n m c nhiu hn

s nghim t nhin n phn bit.


Chng minh. Ta trnh by mt chng minh s cp thuc v A.Schinzel [5].
K hiu s l s t nhin v m p1 1 p2 1 ... ps 1 vi pi l s nguyn t th i . Ta s chng
minh cc s x1 , x2 ,..., xs 1 vi xi p1... pi 1 ( pi 1) pi 1... ps , i 1, 2,..., s v xi 1 p1 p2 ... ps l nghim ca
phng trnh n m .

Tht vy, vi i l mt trong cc s 1, 2,..., s th s pi 1 khng chia ht cho mi s nguyn t pi v


do

1 1

xi p1

pi 1 p11 p2 2 ... pii11

vi

1 , 2 ,..., i 1

cc

nguyn

khng

m.

vy

p2 ... pi 1 pi 1 pi 2 ... ps nn xi p1 p2 ... pi 1 p1 1 p2 1 ... pi 1 1 pi 1 1 ... ps 1


2 1

i1 1

i1

suy ra theo nh ngha ca m th ( xi ) m vi i 1, 2,..., s . Ta thy cc s x1 , x2 ,..., xs 1 l cc s


nguyn dng phn bit. nh l c chng minh.
Tn ti dy v hn tng cc s t nhin mk k 1, 2,... m s nghim ca phng trnh (n) mk vi
mi k 1, 2,... l ln hn mkc vi c l hng s dng (P.Erdos [3]). P.Erdos t ra gi thuyt vi
7
mi 0 th hng s ny c th chn l 1 . A.Balog [1] chng minh iu ny ng vi
.
20
Cu hi t ra l c phi vi mi s t nhin s th u tn ti s t nhin m m phng trnh
n m c ng s nghim t nhin hay khng. Ta cha bit cu tr li ngay c khi s 1 . Tht vy,
ta cha bit s t nhin m no m phng trnh n m c ng mt nghim. Gi thuyt
Carmichael [5] ni rng khng tn ti s t nhin m nh vy. P.Masai v A.Valette [1] chng minh
10000
khng tn ti s m 10
no nh vy. Tuy nhin c th chng minh tn ti v hn s t nhin m
m phng trnh n m c ng hai (hoc ba) nghim t nhin n (bi tp 12).
Vi mi s t nhin s 1 k hiu ms l s t nhin nh nht m m phng trnh n m c ng

s nghim t nhin (nu s ms tn ti). Ta tnh c


m2 1, m3 2, m4 , m5 8, m6 12, m7 32, m8 36, m9 40, m10 24, m11 48, m12 160
m13 396, m14 2268, m15 704
Ta t ra gi thuyt rng vi mi s t nhin s 1 th tn ti v hn s t nhin m m phng
trnh n m c ng s nghim t nhin n. Tnh cht ny c suy ra t gi thuyt H (Schinzel

156 | Hm ch Euler

[13]). Kh khn duy nht trong chng minh nm vic chng t s tn ti ca cc s ms v P.Erdos
[14] chng minh nu vi s t nhin s cho trc m tn ti s t nhin m m phng trnh
n m c ng s nghim t nhin n th c v hn s t nhin m c tnh cht ny.
Ta cha bit c tn ti v hn cc s t nhin khng c dng n n vi n l s t nhin hay khng.
C th chng minh cc s 10, 26, 34 v 50 u khng c dng ny. Ta khng bit c tn ti s l c
dng ny hay khng. Cu tr li l khng nh nu ta chng minh c mi s chn ln hn 6 u l
tng ca hai s nguyn t phn bit.
n 1
1
Bi tp. 1. Chng minh cng thc N.C.Scholomiti [1] n
vi mi s t nhin n 1 .
k 1 n, k

1
Chng minh. Lu nu n 1, k n v (n, k ) 1 th
1 . Mt khc nu n, k 1 th
(n, k )
1

0 . Do v phi ca cng thc bng vi s cc s t nhin n nguyn t cng nhau vi


n, k
n 1 v do bng n .
2. Tm cc s t nhin n m n khng chia ht cho 4.
Li gii. Cc s l 1, 2, 4 v p v 2 p vi p l s nguyn t c dng 4t 3. Chng minh l hin
nhin (Carmichael [1], Klee [1]).
3. Chng minh tn ti v hn cp s t nhin x, y, y x, m d ( x) d ( y) , ( x) ( y) , ( x) ( y) .
Chng minh. Cc cp s x 3k.568 , y 3k .638 vi k 0,1, 2,... u tha mn (Jankowska [1]).
4. Chng minh rng tn ti v hn cc b s x, y, z m x y z v

d x d y d z , x y z , x y z
Chng minh. t

x 5k 23 33 71113, y 5k 23 3 29 37 71
z 5k 2 33 11 29 113
P.Erdos [15] chng minh rng vi mi s t nhin s th u tn ti s s t nhin phn bit
a1 , a2 ,..., as tha mn d (ai ) d (a j ) , (ai ) (a j ) , (ai ) (a j ) vi mi 1 i j s . T gi thuyt
Erdos [16] th suy ra cc s a1 , a2 ,...as c th chn i mt nguyn t cng nhau.
5. Chng minh rng vi mi s t nhin m th tn ti s t nhin n m (n) (n 1) m v
(n) (n 1) m .

p l s nguyn t c dng 4k 3 v ln hn 2m 3. th ta c
p 4k 2, p 1 4k 2 2k 1 2k 1. Do p p 1 2k 1 m . Ta cng
1
c p 1 4(k 1) 2 l vi 2 v l l s l. V vy p 1 2 1 l 2 1 l p 1 v do
2
1
1
p p 1 p 1 p 1 p 3 m .
2
2
Chng minh. Vi

CHNG 6. HM CH EULER V NH L EULER | 157

Lu rng: tn ti s t nhin n 1 m (n 1) / (n) m v (n 1) / (n) m v tng t tn ti


s t nhin n 1 m n / n 1 m v n / n 1 m (Schinzel v Sierpinski [1]).
C th chng minh (Erdos v Schinzel [1]) vi hai s t nhin m v k 1 th tn ti s t nhin n
(n i )
(n i 1)
tha mn
m vi i 1, 2,..., k v s t nhin n tha mn
m vi i 1, 2,..., k .
(n i 1)
(n i )
6. Chng minh rng vi cc s t nhin a, b ty th tn ti v hn cc cp s t nhin x, y m

x : y a : b .
Chng minh. Vi a v b l hai s t nhin cho trc. Khng gim tng qut gi s cc s ny nguyn
t cng nhau. K hiu c l s t nhin nguyn t cng nhau vi ab (c v hn s nh vy chng hn
cc s kab 1 vi k 1, 2,... ). t x a 2bc, y ab2c . V cc s a, b, c i mt nguyn t cng nhau
nn ( x) (a 2 ) (b) (c) v ( y) (a) (b2 ) (c) . T nh l 3 suy ra vi mi s t nhin n ta c

n2 n n v suy ra (a 2 ) a (a) , (b2 ) b (b) nn x : y a : b .

T gi thuyt H suy ra s tn ti v hn cc s nguyn t x, y tha mn x : y a : b vi cc cp


s cho trc a, b (Schinzel v Sierpinski [3] trang 192).
7. Chng minh rng nu n l s t nhin 1 th tn ti v hn s t nhin m m m / m n / n .
Chng minh. S n l s t nhin 1 c c s nguyn t p do n p n1 vi l s t nhin v

p 1 p 1 n1 p 1 n1
. t m p n1 vi l s t nhin. Lp

n
p n1
p
n1
(m) (n)
(m) p 1 (n1 )

lun tng t ta c
do
v suy ra iu phi chng minh.

m
n
m
p
n1

n1 , p 1 . V vy

C th chng minh cc s n / n, n 1, 2,... to thnh mt tp hp tr mt trong khong 0,1 . Mt


khc tn ti tp hp tr mt trong khong 0,1 cha cc s hu t m khng c dng n / n
(Schoenberg [1], Sierpinski [26] trang 210). K.Zarankiewicz t ra cu hi c phi tp hp cc s
n 1 / n , n 1, 2,... l tr mt trong tp hp cc s thc hay khng. A.Schinzel [3] chng
minh cu tr li l khng nh (Erdos v Schinzel [1]).
8. Tim tt c cc nghim t nhin ca phng trnh n 2n .
Li gii. l cc s l.
9. Tm tt c cc nghim t nhin ca phng trnh 2n 3n .
Li gii. l cc s t nhin chn khng chia ht cho 3.
10. Tm tt c cc nghim t nhin ca phng trnh 3n 4n .
Li gii. l cc s khng chia ht cho 2 hoc khng chia ht cho 3.
11. Chng minh rng vi mi s t nhin k th tn ti t nht mt s t nhin n m n k n .
Chng minh. Nu k l s l th mnh ng v 2k k v ta t n k. Gi s k chn v k
hiu p l s nguyn t nh nht khng phi c s ca k . Khi mi s nguyn t p u l c s

158 | Hm ch Euler

ca k . V vy p 1 k p 1 k (iu ny suy ra t nh l 3 v nu m l s t nhin m mi


c s nguyn t ca n u l c s nguyn t ca k th mk m k m

pk p k p 1 k p 1 k . Do t

p, k 1 suy ra
n p 1 k ta c n k n , iu

phi chng minh (Sierpinski [18] trang 184).


Ta chng minh c vi mi s t nhin m th lun tn ti s t nhin k m phng trnh
n k n c nhiu hn m nghim t nhin n (ti liu dn trang 184-185).
12. Chng minh rng tn ti v hn s t nhin m m phng trnh n m c ng hai nghim t
nhin n .
Chng minh. Cc s m= 2 36 k 1 vi k 1, 2,... tha mn. Tht vy, gi s n l s t nhin tha mn

n 2 36k 1 . R rng n khng phi ly tha ca 2 (v 2 2 1 ) nn n c c s nguyn t l


p v hn na n khng c nhiu hn mt c s nh vy v n khng chia ht cho 4.

Nu p 3 th n 3 hoc n 2 3 vi , l cc s t nhin. Khi v n 2 36 k 1 suy ra

2.3 1 2.36k 1 hoc 2 .3 1 2.36 k 1 v do 1 v 1 6k 1 . V th n 36k 2 hoc n 2.36k 2


v trong mi trng hp ta c n 2 36 k 1 m .
Nu p 3 th p 3 v do n khng chia ht cho p 2 v nu ngc li th p | n 2 36 k 1 mu
thun v p 3 . V vy n p hoc n 2 p vi l s t nhin. V vy t n 2 36 k 1 suy ra ta c

p 1 2 36 k 1 hoc 2 1 p 1 2 36 k 1 . M p 1 chn suy ra 1 v trong mi trng hp th

p 2 36k 1 1 , v l v vi k 1 ta c p 7 v theo nh l Fermat nh th 36 1(mod 7) suy ra

p 2.36k 1 1 2.3 1 0(mod 7) , do p | 7 . V vy phng trnh n 2 36 k 1 vi k l s t nhin


c ng hai nghim l n 36 k 2 v n 2 36 k 2 .
Phng trnh n 2 3 c bn nghim l n 7,9,14,18 v phng trnh n 2 32 cng c bn
nghim l n 19, 27,38,54 .
Ghi ch. A.Schinzel [6] tm ra v hn s t nhin m m phng trnh n m c ng ba
nghim t nhin. l cc s m 712k 1 12 vi k 0,1, 2,... . Khi n m vi n 712k 2.3 , 712 k 2.4
v 712 k 2.6 . chng minh y l tt c cc nghim bng cch s cp th tng i di.
13. Tm tt c cc nghim t nhin ca phng trnh n 210 .
Li gii. Gi s n l s chn v n 2 q11 q22 ...qkk11 vi q1 , q2 ,...qk 1 l cc s nguyn t l l phn tch
thnh tha s nguyn t ca n .

t q1 q2 ... qk 1 (c th b qua trng hp k 1 , ngha l n 2 ). V n 210 nn

2 1 q11 1q22 1...qkk11 1 q1 1 q2 1 ... qk 1 1 210 suy ra 1 2 ... k 1 1 v qi 2 1 ,

i 1, 2,..., k 1 v i vi i 1, 2,.., k 1 l cc s t nhin v 1 1 2 ... k 1 10 do


i 10 vi i 1, 2,..., k 1 .
Cc s nguyn t l c dng 2 1, 10 l cc s 2 1 vi 1, 2, 4,8 . V th k 5 .

CHNG 6. HM CH EULER V NH L EULER | 159

Nu k 1 th nu n 2 ta c 1 10 suy ra 11 v h qu l n 211 2048 .


Nu k 2 th 1 1 10 v vi 1 1, 2, 4,8 ta c 10,9,7,3 tng ng. Do cc gi tr ca

n l 210.3 3072 , 29.5 2560 , 27.17 2176 hoc 23.257 2056 .


Nu k 3 th 1 1 2 10 . V vy 1 khng th 2 v nu nh vy th 1 ln hn hoc bng
4. Nhng 1 2 (vi q1 q2 ) ta c 2 4 do 2 8 v 1 2 12 , v l. V vy 1 bng 1 hoc
2. Nu 1 1 th 2 10 v 2 1 1 do 2 2, 4,8 suy ra 8,6, 2 v do

n 28.3.5, 26.3.17, 22.3.257 . Nu 1 2 th 2 9 , 2 4 hoc 8 suy ra 5 hoc 1 v do

n 25.5.17 hoc 2.5.257 .


Nu k 4 th 1 1 2 3 10 . V 1 2 3 (suy ra t q1 q2 q3 ) v 1 , 2 , 3 ch c th
l 1, 2, 4,8 nn 1 1, 2 2, 3 4 chng t 4 v do n 24.3.5.17 .
Cui cng ta thy khng th c k 5 v nu k 5 suy ra 1 1, 2 2, 3 4, 4 8 mu thun vi
ng thc 1 1 2 3 10.
By gi gi s n l th n q11 q22 ...qkk11 vi q1 , q2 ,..., qk 1 l cc s nguyn t l q1 q2 ... qk 1 . Theo
gi thit th q11 1q22 1...qkk11 1 q1 1 q2 1 ... qk 1 1 210 nn 1 2 ... l 1 1 v qi 2i 1
vi i 1, 2,..., k 1 . Hn na 1 2 ...k 1 10 .
Nu k 2 th 1 10 , v l. Nu k 3 th 1 2 10 suy ra 1 2, 2 8 v do n 5 257 .
Nu k 4 th 1 2 3 10, v l v 1 , 2 , 3 l cc s phn bit thuc dy 1,2,4,8.
Tng t khng th c k 5 . Vy phng trnh n 210 c ng 12 nghim t nhin l

n 211 , 210 3, 29 5, 27 17, 23 257, 28 3 5, 26 3 17, 22 3 257, 25 5 17,5 257, 2 5 257, 23 3 5 17.
Ghi ch. C th chng minh vi 0 m 31 ( m l s nguyn) th phng trnh n 2m c ng
20
m 2 nghim t nhin n . Vi 31 m 2 th phng trnh lun c ng 32 nghim. Chng minh

2
da trn tnh cht cc s 2 1 5 n 20 u l hp s (1)
n

14. Chng minh rng tn ti v hn s t nhin m m phng trnh n m c t nht mt nghim


t nhin v mi nghim ca n u l s chn.
Chng minh. t m 232 2 vi s 6,7,... . Nu tn ti s t nhin l n tha mn n m th n l
s

tch ca cc c s nguyn t khc nhau v c dng Fh 22 1 v nu p l s nguyn t v p | n th


k

p 1| n m suy ra p 1 l ly tha c 2 v do p Fk . Gi s l cc s Fh1 , Fh2 ,..., Fhk . Khi

2h1 2h2 ... 2hk 25 2s vi h1 , h2 ,..., hk l cc s t nhin phn bit. S 25 2s vi s 5 ch c mt


biu din thnh tng cc ly tha phn bit ca 2 nn mt trong cc s Fh1 , Fh2 ,..., Fhk bng F5 , v l v

33 2
m.
F5 l hp s. Vy phng trnh n m khng c nghim l. Nu n chn th ta c 2

(1)

Carmichael [1] trnh by chng minh vi m 2 . Vi 2


10

10

m 220 th chng minh tng t.

160 | Cc tnh cht ca hm ch Euler

15. Chng minh nu p 2 v 2 p 1 l cc s nguyn t th vi n 4 p ta c n 2 n 2 .


Chng minh. Nu cc s p 2 v 2 p 1 l cc s nguyn t th 4 p 4 p 2 p 1 v

4 p 2 2 2 p 1 2 p 1 2 p suy ra 4 p 2 4 p 2 .

Ghi ch. T gi thuyt H (Chng 3 mc 8) suy ra tn ti v hn cp s nguyn t sinh i. Tng t,


t gi thuyt H cng suy ra tn ti v hn s nguyn t p m s 2 p 1 cng l s nguyn t. Do
gi thuyt H suy ra tn ti v hn s t nhin l v v hn s t nhin chn n m n 2 n 2 .
2. Cc tnh cht ca hm ch Euler
By gi vi s t nhin cho trc n ta s tnh s cc s t nhin n m c s chung ln nht ca
chng vi n l bng d vi d | n .
c s chung ln nht ca m n v n l d khi v ch khi m kd vi k l s t nhin n / d . H qu
l s cc s t nhin m n tha mn iu kin m, n d l bng vi s cc s t nhin n / d m
nguyn t cng nhau vi n / d , tc l bng vi n / d .
V vy ta thy trong dy 1, 2,..., n th vi mi c s t nhin d ca s t nhin n lun c ng

n / d s t nhin m m m, n d .
Gi s d1 , d2 ,..., d s l tt c cc c s t nhin ca s t nhin n . Cc s 1, 2,..., n c th chia thnh s

n
lp theo quy tc s m thuc lp i khi v ch khi n, m di . S cc phn t ca lp i l
di

n
n
n
Hn na v s cc s trong dy 1, 2,..., n l bng n nn .. n . Nhng nu
d1
d2
ds
n
cng nhn mi gi tr l c s ca n . V vy
d1 nhn mi gi tr l c s ca n th
di

d1 d2 ... d s n , ngha l

d n

(8)

d |n

Vy ta chng minh c
nh l 7. Tng cc gi tr ca hm ch Euler ng vi mi c s ca n l bng n .
S dng tch Dirichlet (Chng 4 mc 3) cho cc chui a1 a2 ... v b1 b2 ..., vi s thc s 2 ,

an n / ns , bn1/ ns n 1, 2,... ta nhn c t (8)

cn ad bn
d |n

V vy

cn s 1 v do
n1

d |n

d
x

n 1

(d ) d s

1
n
1
d s s 1

n d |n
n
n

s 1
vi s 2 .
s

S dng (8) c th chng minh ng thc Liouville

n 1

n xn
1 x

1 x

vi x 1 .

CHNG 6. HM CH EULER V NH L EULER | 161

T nh l 6 mc 10 Chng 4 th hm ch Euler l hm s duy nht tha mn nh l 7.


Cng thc (8) v cng thc (37) cho ta cng thc

n n

(9)

d |n

ng vi mi s t nhin n . Cng thc ny c th vit li di dng

n l k

(10)

kl n

vi tng ly trn mi cp s t nhin k v l m kl n . Vi x 1 th t cng thc (10) suy ra


[ x]

n l k

(11)

n 1

vi

kl x

l tng tnh trn mi cp s t nhin k , l m kl x . Nhng

kl x
[ x/k ]

1 x x

[ x]
[ x/k ]

k
l v t

kl x
k 1
l 1

l 2 k k 1 theo cng thc (33) Chng 4, cng thc (11) suy ra


i 1

[ x]

1
2

(12)

n 1

2
1 [ x]
x

k
k
2 k 1

Cng thc ny c th s dng tnh tng cc gi tr lin tip ca hm v tm xp x cho tng . S

dng cng thc

k 1

[ x]

n v 3x

k
k

c chng minh trong Chng 4 mc 10 ta c th thy t s ca

/ 2 tin ti 1 khi x tng v hn.

n 1

Dng tng qut ca hm s n l hm s k n c xc nh vi cc cp s t nhin k , n l s cc


dy a1 , a2 ,...ak cha k s t nhin n m a1 , a2 ,...ak , n 1 .
D dng chng minh c nh l C.Jordan [1] (trang 95-97) ni rng nu n q11 q22 ...qss l phn tch

1
1
1
thnh tha s nguyn t ca n th k n n k 1 k 1 k ... 1 k v
q1 q2 qs

d n
k

d |n

Mt dng tng qut khc ca hm l hm k n c cho bi V.L.Klee, Jr. [3]. Hm s ny c xc


nh (vi cc s t nhin k v n ) l s cc s h xut hin trong dy 1, 2,..., n m h, n khng chia ht
cho ly tha bc k bt k ln hn 1.
D dng chng minh nu n q11 q12 ... qss l phn tch thnh tha s nguyn t ca n th

k n qii qii k qik 1 . Ta cng c k n n 1 q k v


i k

ai k

3. nh l Euler
Cho trc s t nhin m 1 v t

q k |n
qprime

d n
k

d |n

162 | nh l Euler

(13)

r1 , r2 ,..., r m

l dy cc s t nhin nguyn t cng nhau vi m v nh hn m . K hiu a l s nguyn ty


nguyn t cng nhau vi m . K hiu Qk l s d nhn c khi chia s ark cho m k 1, 2,..., m .
Ta c

Qk ark mod m vi k 1, 2,..., m .

(14)
v

Qk ark mtk ,

(15)

vi tk k 1, 2,..., m l cc s nguyn.
Ta s chng minh cc s
(16)

Q1 , Q2 ,..., Q m

v cc s (13) l trng nhau ch sai khc mt hon v. Ta ch cn chng minh


(i)
(ii)

cc phn t ca dy (16) l cc s t nhin nguyn t cng nhau vi m v nh hn m ,


cc phn t ca (16) l phn bit.

t dk Qk , m . Theo (15) th ark Qk mtk suy ra d k | m . Nhng v a, m rk , m 1, ark , m 1


m d k | m v d k | ark suy ra d k 1 , ngha l Qk , m 1. Mt khc, s Qk l s d nhn c khi chia
mt s cho m nn 0 Qk m . Hn na v Qk , m 1 v m 1 nn Qk khc 0 . V vy mi phn t
ca dy (16) u tha mn tnh cht (i).
By gi gi s vi hai ch s phn bit i v j trong dy 1, 2,..., m th Qi Q j . Khi theo (14) ta

c ari arj mod m v do m | a ri rj

nn v a, m 1 ta c m | ri rj , v l v ri v rj l hai

phn t phn bit ca (13) (vi i j ) l cc s t nhin phn bit m . V vy ta chng minh c
cc phn t ca dy (16) u c tnh cht (ii).
T y suy ra cc phn t ca dy (16) v cc phn t ca dy (13) l ng nht sai khc mt th t.
Do Q1 Q2 ...Q m r1 r2 ... r m . K hiu P l gi tr chung ca cc tch s ny. S P nguyn t cng
nhau vi m v cc nhn t ca n nguyn t cng nhau vi m . Nhn cc ng d thc nhn c t
(14) bng cch thay 1, 2,..., m cho k ta nhn c Q1 Q2 ...Q m a m r1 r2 ... r m mod m , ngha l

P a m P mod m . iu ny tng ng vi m | P a

1 suy ra v P, m 1 ta c m | a m 1 .

Vy ta chng minh c
m
nh l 8 (Euler). Vi mi s nguyn a nguyn t cng nhau vi s t nhin m th a 1 mod m .

Nu p l s nguyn t th p p 1 do nh l Euler l tng qut ca nh l Fermat nh


(Chng 5 mc 5).
nh l 8a (Redei). Vi mi s t nhiin m 1 v a nguyn ta c (Szele [1] ghi ch 2)
(17)

m | a m a m m .

CHNG 6. HM CH EULER V NH L EULER | 163

Chng minh. t m q11 q22 ... qkk l phn tch thnh tha s nguyn t ca m . K hiu i l mt
qi
trong cc s 1, 2,..., k . Nu a, qi 1 th theo nh l 8 ta c qii | a i 1 v t nh l 3 suy ra

m
qi | m v ta c qii | a 1 . Nu v q 2 l cc s t nhin th theo quy np ta c
i

q 1 . Mt khc vi i 1, 2,..., k , ta c qi 1 | m v qii 1 | m suy ra qii 1 | m m . Hn na


i

m m dng vi m ln hn 1, nn t tnh cht cui cng suy ra m m qii 1 i . V vy

nu a, qi 1 th vi qi | a ta c qi i | qi

m m

|a

m m

m m
m

Vy vi mi s nguyn a th qii | a a 1 vi mi i 1, 2,3,..., k . Ngha l qi i | a m a

m m

suy ra theo phn tch thnh tha s nguyn t ca a th cng thc (17) ng. nh l 8a c chng
minh.
nh l Euler l h qu ca nh l 8a . Tht vy, theo nh l 8a th vi mi s t nhin m 1 v mi

m m
m
m m
s nguyn a ta c m | a a 1 . Do v a, m 1 nn a , m 1 suy ra m | a m 1 .

Bi tp. 1. Chng minh rng t mt cp s cng v hn cc s nguyn th c th chn ra mt chui ly


tha.
Chng minh. Gi s ta c cp s cng v hn cc s nguyn

a, a r , a 2r ,...

(18)

Nu r 0 th khng c g chng minh v dy (18) lp thnh mt cp s nhn.


Nu r 0 th bi ton c quy v trng hp r 0 bng cch i du tt c cc phn t ca dy.
Vy ch cn xt trng hp r l s t nhin. Hn na c th gi s a, r 1 v nu d a, r 1 th ta
c a da, r dr vi a, r 1 v do ch cn chng minh vi cp s cng a, a r , a 2r ,... .
By gi xt r 0 , khi t phn t no dy (18) s gm ton s ln hn 1. V vy ta c th gi s
a 1 . V a, r 1 nn theo nh l 8 th a r 1(mod r ) . V vy vi cc s t nhin n, a n r mod r

v do s kn aa

n r

a / r nguyn vi mi n 1, 2,... . Nhng a kn r a a

vi n 1, 2,...,

v do v a 0 , 0 k1 k2 ... v cc s a kn r n 1, 2,... to thnh mt cp s nhn.


Bi ton ta va chng minh suy ra trong mi cp s cng v hn th c v hn cc phn t c chung
c s nguyn t (Polya v Szego [1] trang 344). Mt h qu khc ca bi ton ny l mi cp s cng
v hn cc phn t hu t th tn ti cc phn t lp thnh mt cp s nhn s vn.
2. Chng minh rng nu m, a, r l cc s t nhin vi a, r 1 v Z l tp hp v hn cc phn t
ca cp s cng a kr (k 1, 2,...) th cp s ny cha cc phn t l tch ca nhiu hn m phn t
phn bit trong Z .
Chng minh. Ly ra s m r 1 s phn bit trong tp Z v k hiu cc s l t1 , t2 ,..., ts . Cc s
ny l phn t ca cp s cng a kr k 1, 2,... nn chng ng d vi s a mod r . Do

t1 t2 ... ts a s a a m r mod r m

a, r 1

nn theo nh l 8 suy ra a

1 mod r . V vy

164 | nh l Euler

t1 t2 ... ts a mod r v h qu l s t1 t2 ... ts l phn t ca cp s cng a kr k 1, 2,... . Hn na


s m r 1 m v do ta c iu phi chng minh.
3. Chng minh rng mi s t nhin khng chia ht cho 2 hoc 5 u l c s ca s t nhin no
vi cc ch s (trong h thp phn) u bng 1.
Chng minh. Nu n,10 1 th 9n,10 1 v v vy theo nh l 8 th 10 9 n 1 mod 9n . Do
9 n

10

9n
1 9nk vi k l s t nhin. V vy nk 10 1 / 9 v do cc ch s trong h thp

phn ca s ny u bng 1.
4. Chng minh rng mi s t nhin u c bi s vi cc ch s trong h thp phn u bng 1 hoc
0 v cc ch s 1 ng lin trc cc ch s 0.
Chng minh. Mi s t nhin u biu din c di dng n n1 2 5 vi n1 ,10 1 . Theo bi tp
3 th n1 l c s ca s m m cc ch s ca n trong h thp phn u bng 1. Mt khc 2 5 |10
vi max , nn n | m 10 .

5. Tm tt c cc nghim t nhin ca ng d thc x x 3 mod10 .


Li gii. Nu s t nhin x tha mn ng d thc trn th v 3,10 1 ta c x,10 1 . H qu l

x 20k ,10 1 vi k 0,1, 2,... . V vy theo nh l 8 th vi 10 4 ta c x 20k 4 1 mod10


20 k
x
do x 20k 1 mod10 . Mt khc ng d thc x 20k x x mod10 ng vi mi s t
x 20 k
nhin x . Do nhn hai ng d thc cui cng theo v suy ra x 20k
x x mod10 vi mi
k 0,1, 2,... . Nu s t nhin x tha mn ng d thc x x 3 mod10 th mi phn t ca cp s
cng x 20k k 0,1, 2,... l tha mn tnh cht ny. D thy trong cc s nguyn x m 0 x 20
ch c 7 v 13 tha mn ng d thc nn tt c cc nghim t nhin ca ng d thc
x x 3 mod10 l 7 20k v 13 20k vi k 0,1, 2,...
4. Cc s vi s m cho trc theo mt modulo cho trc
T nh l 8 suy ra nu a l s nguyn nguyn t cng nhau vi s t nhin m th ng d thc
(19)

a x 1 mod m

c v hn nghim t nhin x chng hn dy v hn cc s x k m vi k 1, 2,... u l nghim.


Mt khc ng d thc (19) c nghim t nhin ch khi a, m 1 .
Nu x l nghim t nhin nh nht ca ng d thc (19) th ta ni s ny c s m theo
modulo m .
R rng nu hai s ng d theo modulo m th chng c cng s m modulo m v vi a b mod m
x
x
x
v vi x ta c (19) th b 1 mod m v t a b mod m suy ra a b mod m vi mi x 1, 2,... .

nh l 9. Nu a, m 1 th mi nghim ca ng d thc (19) u chia ht cho s m ca a theo


modulo m .

CHNG 6. HM CH EULER V NH L EULER | 165

Chng minh. Gi s phn chng rng tn ti nghim x ng d thc (19) m khng chia ht cho .
Ngha l x chia d r . Theo x k r vi k l s nguyn khng m. Theo (19) ta c

a k r 1 mod m hay a a r 1 mod m .


k

(20)

Theo nh ngha ca th ng d thc a 1 mod m ng. V vy theo (20) th a mod m . V


vy t gi thit suy ra tn ti nghim r ca ng d thc (19) nh hn , nhng iu ny mu thun
vi nh ngha ca . nh l c chng minh.
Theo nh l 8 th m l nghim ca ng d thc (19). nh l 9 suy ra
H qu. S m ca mt s nguyn t vi m theo modulo m l c s ca m .
c bit nu tn ti cc s nguyn t cng nhau vi m v c s m l m theo modulo m (cng
tc l cc s c s m ln nht theo modulo m ) th cc s c gi l cn nguyn thy ca m .
V d 3 l cn nguyn thy ca 10 v 31 3,32 9,33 7,34 1 mod10 v 10 4 . Tuy nhin 10
khng phi l cn nguyn thy ca 3 v 10 1 mod 3 chng t 10 c s m l 1 theo modulo 3 v

3 2 . S 7 l cn nguyn thy ca 10 v 71 7,72 9,73 3,74 1 mod10 v 10 cng l cn


nguyn thy ca 7 v 10 3,102 2,103 6,104 4,105 5,106 1 mod 7 v 7 6 .
T nh l 8 suy ra vi mi s t nhin m th u tn ti s t nhin nh nht m m m | a m 1
vi a, m 1 (cc s ny l cn thit khi xc nh cc hm Liouville trong Chng 4 mc 11). S

m c gi l ly tha ph qut (universal exponent) nh nht modulo m . Theo nh l 8 th

m m vi mi s t nhin m . Ta c 2 1, 22 2, 2 2 2 , 3, 4,... . Nu
m 2 q11 q22 ... qss , 2 q1 q2 ... qs l phn tch thnh tha s nguyn t ca m

th

m 2 , q1 ,..., qs v vi mi s t nhin m th lun tn ti s t nhin c s m


0

m theo modulo m (Ore [1] trang 292-293).

Trong Mathematical Tables v Aids to Computation 4 (1950) trang 29-30, S.Whitten [1] tnh c
n vi n 1200 . Di y l bng m vi m 100 .

166 | Cc s vi s m cho trc theo mt modulo cho trc

C th chng minh m m ch khi m 1, 2, 4, p v 2 p vi p l s nguyn t l v l s t


nhin. Mt bo co khc gn y ch ra s tn ti dy tng v hn cc s t nhin nk k 1, 2,... m

lim nk / nk 0 . Chng hn dy nk p1 p2 ... pk k 1, 2,... . C th chng minh m l hp s


k

Carmichael (v do l s gi nguyn t tuyt i) khi v ch khi m | m 1 (Carmichael [2] trang


237 cng thc (18)).
Carmichael thng bo (ti liu dn trang 236) phng trnh n 2 c ng su nghim

n 2, 4,6,8,12, 24, v phng trnh n 4 c ng 12 nghim m nghim nh nht l n 5 v


nghim ln nht l n 240 . Phng trnh n 12 c 84 nghim m nghim nh nht l n 13 v
nghim ln nht l n 65520 . Ta c 100 20 . Vi n 100 th ng thc n 1 n ng ch
vi n 3 ,15 v 90.
Vi mi s t nhin s th u tn ti s t nhin ms m phng tnh n ms c nhiu hn s
nghim t nhin n . Theo nh l 11 (s c chng minh trong mc tip theo) th vi mi s t
nhin s u tn ti s t nhin k m p 2s k 1 l s nguyn t. Vi j 0,1, 2,..., s, s 1 ta c

2 j 2s k 1 2s k do t ms 2s k ta nhn c tnh cht trn.

D dng chng minh vi mi s t nhin n 2 th cc s n u chn. Tn ti v hn s chn


khng phi gi tr ca hm n . C th chng minh cc s 2 7 k vi k 1, 2,..., u c tnh cht ny
(Sierpinski [26] trang 191-192).
nh l 10. Nu p l s nguyn t 2 th mi c s t nhin ca s 2 p 1 u c dng 2kp 1 vi k
l s nguyn.
Chng minh. Do tch ca hai (hoc nhiu hn) cc s c dng 2kp 1 th cng c dng ny v v 1 c
dng (vi k 0 ) nn ch cn chng minh mi c s nguyn t q ca s 2 p 1 u c dng

2kp 1 . Nu q | 2 p 1 th 2 p 1 mod q v do theo nh l 9 th | p vi l s m ca 2 theo


modulo q . Ta khng th c 1 v nu vy th 2 1 mod q v q |1 , v l. Do v | p v p l s
nguyn t suy ra p . Mt khc h qu ca nh l 9 suy ra | q , ngha l | q 1 . V vy

p | q 1 v v q l c s ca mt s l v p, 2 1 (v p l s nguyn t 2 ) nn suy ra 2 p | q 1 ,
ngha l q 1 2kp do q 2kp 1 vi k l s nguyn. nh l c chng minh.
Ta lu trong nh l 10 th gi thit p l s nguyn t 2 l cn thit. Cc c s 3,5 v 15 ca

24 1 khng c dng 8k 1 v c s 7 23 1 ca 215 1 khng c dng 30k 1.


Bi tp. 1. Chng minh nh l Fermat sau y: nu p l s nguyn t 3 th mi c s t nhin 1

ca s 2 p 1 / 3 u c dng 2kp 1 vi k l s t nhin.

Chng minh. S 2 p 1 / 3 l s t nhin v vi s l p, 2 1| 2 p 1 . K hiu d l c s 1 ca

q
2p
1 / 3 v q l c s nguyn t ca d . Nu q 3 th 2 1 0 mod 9 suy ra 2 1 mod 9 v

theo nh l 9 th s 2 p chia ht cho s m ca 2 theo modulo 9. Nhng ta li d dng tnh c


6 v do 6 | 2 p suy ra 3 | p , mu thun vi gi thit p 3 . V vy cn phi c q 3 . V

CHNG 6. HM CH EULER V NH L EULER | 167

2 p 1 0 mod q ta c 22 p 1 mod q . K hiu l s m ca 2 theo modulo q . Ta khng th c

1 hoc 2 v q 3 . Vy 2 . Nhng theo nh l 9 th | 2 p v v 2q 1 1 mod q , | q 1 .


Do cc s 2 p v q 1 c c s chung 2 v suy ra p v q 1 c c s chung 1 . Nhng v
p l s nguyn t nn suy ra p | q 1 v do q pt 1 vi t l s nguyn v v p , q u l nn t
chn. Do q 2kp 1 vi k l s t nhin v v th mi c s ca d u c dng 2kp 1 . H qu l
d cng c dng 2kp 1 . nh l c chng minh.
2. Chng minh rng nu a , b v n l cc s t nhin tha mn a b, n 1 th mi c s nguyn t
ca s a n bn s c dng nk 1 vi k l s nguyn hoc l c s ca a n1 bn1 vi n1 | n v n1 n .
Chng minh. t a, b d . V a b nn a a1d , b b1d vi a1 , b1 1 v a1 b1 . Gi s p l c

s nguyn t ca s a n bn . Khi p | a n bn d n a1n b1n . Nu p | d n th p | d v v vy p | a b .


nh l c chng minh. Gi s p | a1n b1n khi v a1 , b1 1 nn ta c a1 , p b1 , p 1 . Gi p
l c s nguyn thy ca a1 b1 (ngha l p | a1 b1 v ta khng c p | a1m b1m vi mi 0 m ).
Lu rng | n . Tht vy, gi s n khng chia ht cho . Khi n k r vi k l s nguyn 0

v 0 r . Nhng p | a1 b1 v do p | a1k b1k . Do a1k r b1k r a1k b1k a1r b1k a1r b1r
k
1

suy ra p | b

r
1

r
1

m b , p 1 suy ra
1

p | a b vi 0 r , mu thun vi gi thit p l c
r
1

r
1

s nguyn thy ca a1 b1 .
Nu n th | n v p | a1n1 b1n1 | a n1 bn1 vi n1 , n1 | n v n1 n . t n khi theo nh l
Fermat nh th p | a1p 1 1, p | b1p 1 1 suy ra p | a1p 1 b1p 1 . H qu l n | p 1 v do p c dng
nk 1 .
3. Chng minh rng nu a, b, n l cc s t nhin m a b, n 1 th mi c s nguyn t ca a n bn
n
n
hoc l c dng 2nk 1 vi k l s nguyn no ; hoc l c s ca s a 1 b 1 vi n1 l thng s
nhn c khi chia n cho mt s l ln hn 1 no .

Chng minh. Tng t nh bi tp trc.


5. S tn ti v hn cc s nguyn t trong cp s cng nk 1
nh l 11. Nu p l s nguyn t v s l s t nhin th tn ti v hn s nguyn t c dng 2 p s k 1
vi k l s t nhin.
s 1

Chng minh. Gi s p nguyn t v s l s t nhin. t a 2 p . Gi s q l c s nguyn t ty


p 1
p 2
ca s a p 1 a p 2 .... a 1 . Nu a ng d vi 1 mod q th q | a a ... a 1 p mod q do
p
q | p m p v q l cc s nguyn t suy ra q p v do a 1 mod p nn 2 p 1 mod p .
s

p
Nhng theo nh l 5a Chng 5 th suy ra 2 2 mod p nn theo quy np th 2 p 2 mod p v
s

suy ra 1 ng d vi 2 mod p do p |1 , v l.
Vy a 1 mod q , ngha l 2 p

s1

1 mod q . K hiu l s m ca 2 theo modulo q . V q | a p 1,

ngha l 2 p 1 mod q nn | p s v v s p
s

s 1

1 mod q nn khng th c | p s 1 . Vy bng p s .

Theo h qu ca nh l 9 ta c | q , ngha l p s | q 1. V 2 p 1 mod q nn q l v do p 1


s

168 | S tn ti v hn cc s nguyn t trong cp s cng nk 1

chn. Nu p l s nguyn t 2 th

p, 2 1

v do v p s | q 1 suy ra 2 p s | q 1 chng t

q 2 p s k 1 vi s t nhin k no . Nu p 2 th 2s | q 1 suy ra q 2s k 1 vi k l s t nhin.


Vy nu p l th tn ti t nht mt s nguyn t c dng 2 p s k 1 . Nu p 2 th tn ti t nht mt
s nguyn t c dng 2s k 1 . V s ty nn nh l 11 c chng minh.
Chng minh nh l tng qut hn sau y l kh hn
nh l 11a . Vi mi s t nhin n th tn ti v hn s nguyn t c dng nk 1 vi k l s t nhin.
Chng minh (theo A.Rotkiewicz [4], xem thm Estermann [2]). u tin ta lu rng chng
minh nh l th ch cn chng minh rng vi mi s t nhin n th tn ti t nht mt s nguyn t c
dng nk 1 vi k l s t nhin v khi vi hai s t nhin n, m th tn ti t nht mt s nguyn t
c dng nmt 1 vi t l s t nhin v s nguyn t ny m v c dng nk 1 vi k l s t nhin.
Khng gim tng qut c th gi s n 2 v trong dy tt c cc s l th tn ti v hn s nguyn t.
t n q11 q22 ... qss l phn tch thnh tha s nguyn t ca n vi q1 q2 .... qs . Gi s vi mi
c s nguyn t p ca a n 1 th n c s m n theo modulo p . t

Pn nd 1

(21)

n/ d

d |n

vi l hm Mobius (Chng 4 mc 10). Ta biu din cc nhn t nd 1 thnh tch cc c s


nguyn t ca n. Khi tch (21) tr thnh tch ca cc c s nguyn t v s m ca chng du
nguyn (dng, m hoc bng 0 ). Xt p l mt trong cc c s nguyn t . Khi tn ti s t
nhin d | n tha mn p | nd 1. V d | n nn p | nn 1 v n, p 1 . K hiu l s m ca n theo
modulo p . T gi thit suy ra n . H qu trc tip ca nh l 9 l trong cc s nd 1 vi d | n
th tt c cc s chia ht cho p l cc s m | d , ngha l d k vi k l s t nhin tha mn
n
k | n do k | . V p | nn 1 nn ta c | n v suy ra n / l s t nhin 1 (v n ).

Gi l ly tha ln nht m p l c s ca n 1 . Ta c p | n 1 v khng c p 1 | n 1 . Nu

k 1
k 2
n k 1
n 1 n 1 ... n 1 k ,

n 1
p | k , v l, v k | n v n, p 1 . Do vi mi s t nhin k | n / , l ly tha ln nht m

vi s t nhin k | n / ta c p 1 | nk 1 th v

p | n k 1 . T y suy ra trong phn tch (21) th s m ca s nguyn t p l

k . Nhng
k|

v n / l s t nhin 1 , theo cng thc (32) Chng 4 mc 10 ta c

k k 0 . V
k|

k|

tnh cht ny ng vi mi c s nguyn t p ca s (21) nn Pn 1 . Nhng theo (21) ta c


(22)

Pn nn / d 1
d |n

n/d

d |q1 q2 ... qs

v d 0 vi mi s d chia ht cho bnh phng mt s t nhin 1 .

CHNG 6. HM CH EULER V NH L EULER | 169


q11 q2 2 1 ...qs s 1

t b n 1

q q ...q
ta c b n 2 v b 1 2 s nn v vy theo (22) th Pn

d |q1 q2 ...qs

q1 q2 ...qs / d

Ta thy Pn l thng ca hai a thc bin b vi h s nguyn. By gi ta tm ly tha nh nht ca b


trong c t s v mu s ca thng s. Ta xt hai trng hp: s l s chn v s l. Trong trng
hp th nht th ly tha t nhin nh nht ca b trong t s nhn c vi d q1 q2 ... qs . Do s
m trong ly tha ny bng 1 v t s chia cho b 2 c s d l b 1 hoc b2 b 1. Trong mu s th v
q1 q2 ... qs nn ly tha nh nht nhn c vi d q2 q3 ...qs . Do s m trong ly tha ny l

q1 . T s chia b 2 d 1 hoc b2 1 . Nhng v Pn 1 suy ra mu thun v nu b 2 th cc s b 1 v

b2 b 1 l khc 1 v b2 1 . Nu s l th ly tha nh nht ca b t c trong t s l vi


d q2 q3 ... qs v trong mu s l vi d q1 q2 ... qs v ta li thu c mu thun.
Vy t gi thit vi mi c s nguyn t p ca nn 1 th s n c s m nh hn n theo modulo p
dn ti mu thun. Vy nn 1 c t nht mt c s nguyn t p m n c s m n theo modulo p .
Nhng n, p 1 v theo nh l Fermat nh th p | n p 1 1 suy ra theo nh l 9 ta c n | p 1 , ngha
l p nk 1 vi k l s t nhin. Vy ta chng minh vi mi s t nhin n 1 th tn ti t nht
mt s nguyn t c dng nk 1 vi k l s t nhin, suy ra nh l 11a c chng minh,
S dng nh l 11a ta chng minh c nh l A.Makowski (Chng 5 mc 7): vi mi s t nhin
k 2 th tn ti v hn hp s t nhin n m n | a nk 1 vi mi s nguyn a m a, n 1 .

Chng minh. t k q1 1 q2 ... qs s vi q1 q2 ... qs l phn tch thnh tha s nguyn t ca s t

nhin

k 2 . Theo nh l 11a

q1 1 q2 1 ... qs 1 t 1

th tn ti v hn s nguyn t

pk

c dng

vi t l s t nhin. Ta s chng minh nu p l mt s nh vy th s

n kp l hp s tha mn cc iu kin trong bi. Tht vy, ta c


n k k p 1 q11 q22 ... qs s q1 1 q2 1 ... qs 1 t
q1 q2 ... qs k t ,
nk
S dng nh l Euler v nh l Fermat nh suy ra vi a, n 1 th a 1 chia ht cho k v p v

do chia ht cho kp n .
Ta c mt ng dng khc ca nh l 11a .
Ta gi dy p, p 2, p 6 m cc phn t u l s nguyn t l b ba nguyn t loi mt v dy
p, p 4, p 6 m cc phn t u l s nguyn t l b ba nguyn t loi hai. Ta chng minh nu t
tp hp tt c cc s nguyn t ta b i cc s nguyn t thuc vo cc b ba nguyn t loi mt hoc
loi hai th vn cn li v hn s nguyn t trong dy
Tht vy, t nh l 11a suy ra tn ti v hn s nguyn t q c dng q 15k 1 vi k l s t nhin.
Hin nhin vi mi s q th 3| q 2, 5 | q 4, 3| q 4, 5 | q 6 . V vy do q 15 nn ta thy cc s
q 2, q 4, q 4 v q 6 u l hp s. Vy q khng th thuc b ba nguyn t loi mt hoc loi
hai. Tht vy, nu q l mt trong cc s , ngha l q p hoc q p 2 hoc q p 6 v cc s
p, p 2, p 6 l nguyn t th trong trng hp u tin s p 2 q 2 l hp s v trong trng
hp th hai s p 6 q 4 l hp s v trong trng hp th ba th s p q 6 l hp s. Vy c ba

170 | S tn ti v hn cc s nguyn t trong cp s cng nk 1

trng hp u khng xy ra. Tng t nu p, p 4, p 6 l nguyn t th nu p q, p 4 q 4 l


hp s, nu q p 4 th p 6 q 2 l hp s v nu q p 6 th p q 6 l hp s.
6. S tn ti cn nguyn thy ca s nguyn t
K hiu p l s nguyn t cho trc. Theo h qu ca nh l 9 th cc phn t ca dy

1, 2,3,..., p 1

(23)

u c s m (theo mod p ) l c s ca p p 1 . Vi mi c s t nhin ca p 1 k


hiu l s cc phn t trong dy (23) c s m theo modulo p . V mi phn t ca dy (23)
l nguyn t cng nhau vi p nn chng c s m l c s ca p 1 . H qu l

p 1 .

| p 1

T nh l 7 ta c

p 1 suy ra

| p 1

0 .

(24)

| p 1

Ta s chng minh vi | p 1 . R rng iu ny ng vi 0 . Gi s 0 ,

ngha l dy (23) cha t nht mt s a c s m theo modulo p . Khi ta c a 1 mod p . H

qu l a l mt trong cc nghim ca ng d thc

x 1 0 mod p .

(25)
Gi s
(26)

r1 , r2 ,..., r

k
l cc s d nhn c khi chia cc s a k 1, 2,..., cho p . Cc s (26) l phn bit v nu ngc

k l
k
k
l
li th nu rk rk 1 vi k , l l cc s t nhin v k l th p | a a a a 1 suy ra v

a, p 1 ta c

l
p | al 1 , ngha l a 1 mod p , v l v a c s m theo modulo p v l l s t

nhin nh hn (tht vy, k l v k 1 suy ra l ). Theo nh ngha (26) vi k 1, 2,..., th

ta c rk a mod p . V vy t a 1 mod p suy ra rk a

1 mod p , chng t (26) l cc

nghim ca ng d thc (25). ng d thc (25) c bc v tha mn cc iu kin ca nh l


Lagrange (nh l 13 mc 8 Chng 5) nn n khng c nghim no ngoi cc nghim cho bi (26).
Mt khc mi s x c s m theo modulo p u tha mn ng d thc (25) do n l mt
trong cc s (26). Ta cn tm rk c s m theo modulo p . Ta chng minh l s rk vi k , 1 .
Gi s k , 1 . Khi rk l nghim ca ng d thc (25) v c s m theo modulo p . V

k
k
vy rk 1 mod p . Nhng rk a mod p suy ra a 1 mod p . Ta thy k l mt trong cc

x
nghim ca ng d thc a 1 mod p . Vy theo nh l 9 th | k m theo gi thit k , 1

suy ra | v v chng t . Vy nu k , 1 th rk c s m theo modulo p .

CHNG 6. HM CH EULER V NH L EULER | 171

k , d 1 . t k k1d , 1d
k
a 1 mod p . iu ny chng

vi 1 th k1 k1d1 k1 . H qu l

By gi gi s

rk1 a k1 a k1

t rk 1 1 mod p vi 1 v do s rk

khng c s m theo modulo p .


Vy ta chng minh k , 1 l iu kin cn v rk c s m theo modulo p . Ni cch
khc tt c cc s cc s rk trong dy (26) m c s m theo modulo p l cc s m c ch s k

nguyn t cng nhau vi . S cc s nh vy l . V vy (vi c s t nhin cho trc ca


s p 1 ) nu 0 th . T y suy ra mi hng t ca (24) l khng m, m tng
ca chng l bng 0 nn tt c cc hng t u bng 0 .
Do vi | p 1 .
Ta chng minh c
nh l 12. Gi s p l s nguyn t v l c s t nhin ca p 1 . Khi tn ti ng s
phn bit trong dy 1, 2,..., p 1 m c s m theo modulo p .
Trong trng hp ring quan trng p 1 ta c
H qu. Mi s nguyn t p c ng p 1 cn nguyn thy trong dy 1, 2,..., p 1 .
T chng minh ca nh l suy ra nu g l cn nguyn thy ca s nguyn t p th tt c cc cn
nguyn thy ca p thuc dy (23) c th tm c trong cc s d nhn c khi chia cc s trong
dy g , g 2 , g 3 ,..., g p 1 vi s m nguyn t cng nhau vi p 1 cho p .
K hiu p l cn nguyn thy nh nht ca s nguyn t p . Bng di y cho cc gi tr ca hm

p vi cc s nguyn t l p 100

Ta chng minh c lim p (Pillai [7]) v hn na vi v hn s p, p c log p (Turan


[1]). Mt khc, ta cha bit c tn ti v hn cc s nguyn t m 2 l cn nguyn thy ca n hay
khng. E.Artin t ra gi thuyt ni rng mi s nguyn g 1 khng l bnh phng u l cn
nguyn thy ca v hn s nguyn t (Hasse [1] trang 68). Kt qu ny c th suy ra t gi thuyt H
(Schinzel v Sierpinski [3] trang 199-201).
Bng trn cho thy p 7 vi mi s nguyn t p 100 . Vi p 191 ta cng c p 7 .
Nhng 191 19 . Nu p 409 thi p 19 nhng 409 21 . Vi cc s nguyn t p 3361
ta c p 21 nhng 3361 22 . Vi p 5711 ta c p 22 nhng (5711) 29 . Nu p l
s nguyn t 5881 th p 29 nhng 5881 31 (Wertheim [1] trang 406-409).

172 | S tn ti cn nguyn thy ca s nguyn t

Nu g l cn nguyn thy ca s nguyn t p th cc s g 0 , g1 , g 2 ,..., g p 1 chia cho p cho cc s d


phn bit v mi s u khc 0 . H qu l s cc s d nh vy bng vi s cc s g 0 , g1 , g 2 ,..., g p 1 ,
ngha l bng p 1 . Do vi mi s x g 0 , g1 ,..., g p 1 th tn ti y trong dy 0,1, 2,..., p 2 m

g y x mod p .
Ta tm tt c cc s t nhin m 1 c cn nguyn thy. Ta c nh l: s t nhin m 1 c cn nguyn
thy khi v ch khi n l mt trong cc s 2, 4, p , 2 p vi p l s nguyn t l v l s t nhin.
S cc cn nguyn thy ca m c dng ny l m (Sierpinski [12] trang 193).
Litver v Yudina [1] kim tra vi mi s nguyn t p 106 ngoi tr p 40487 th p l cn
nguyn thy ca p vi mi .
T nh l v s tn ti ca cc cn nguyn thy ca cc s nguyn t l ta tm tt c cc s t nhin
m m t a b mod m v c d mod m suy ra ac bd mod m vi mi a, b, c, d nguyn dng.
n gin ta gi cc tnh cht trn l tnh cht P . Gi s s t nhin m c tnh cht P . K hiu a
m
l s t nhin cho trc. Do m | a a v m | m 1 1 ta c m | a m1 a , ngha l m | a a 1 . Mt

m
khc gi s m l s m vi mi s nguyn a ta c m | a a 1 . Ly a, b, c, d l cc s nguyn tha

mn m | a b v m | c d . Nu c d th v m | a b ta c m | a c bd . Gi s c d . i vai tr (nu
cn thit) ca c v d , ta gi s c d . Khi v m | c d , c d mk vi k l s t nhin th ta c

a | a d v a m 1| a mk 1 . Hn na, v m | a ma 1 suy ra m | a d a mk 1 a c a d . Nhng v m | a b

nn ta c m | a d bd m theo cng thc m | a c a d suy ra m | a c bd . Ta thy s m c tnh cht P .

m
Vy iu kin cn v iu kin mt s m c tnh cht P l vi mi s nguyn a, m | a a 1 .

By gi ta tm tt c cc s m c tnh cht P . Hin nhin 1 v 2 u c tnh cht P . Gi s m l s t


2
m
nhin 2 . Nu m chia ht cho bnh phng s nguyn t p th vi a p ta c p | p p 1 , v l

m
v p, p 1 1 . H qu l m phi l tch ca cc nhn t l cc s nguyn t phn bit m tt c u

ln hn 2. Tch ny cha s nguyn t l p . K hiu g l cn nguyn thy ca s nguyn t p . V

p | m | g g m 1 v p, g 1 suy ra p | g m 1 . Nhng do g c s m p 1 theo modulo p nn ta c

p 1| m . Do m chn v l tch ca t nht hai s nguyn t phn bit l 2 v p . Nu m l tch ca


ng hai s nguyn t th m 2 p . V p 1| m v p 1, p 1 ta c p 1| 2 nn v p 3 ( p l s
nguyn t l) ta suy ra p 3 v do m 2 3 6 . S 6 c tnh cht P bi v vi mi s nguyn a ta

2
6
c 6 | a 1 a a 1 a a 1 v a 2 1| a6 1 suy ra 6 | a a 1 .

By gi gi s m l tch ca ba s nguyn t phn bit, ngha l m 2 p1 p2 vi 2 p1 p2 . Ta bit

p1 1| m suy ra p1 1| 2 p1 p2 . Nhng p1 1 1 (v p1 2 ) v p1 1 p1 p2 . S nguyn t p2 khng


chia ht cho p1 1 do p1 1| 2 p1 suy ra tng t trng hp trn ta c p1 3 v do m 6 p2 .
Do p2 1| m 6 p2 v p2 1, p2 1 suy ra p2 1| 6 m p2 p1 , ngha l p2 3 v do p2 1 2

suy ra p2 1 3 hoc p2 1 6 . Nhng p2 1 3 l khng th xy ra v p2 l s nguyn t do ta


phi c p2 1 6 . Suy ra p2 7 v do m 2 3 7 42 . C th kim tra trc tip s 42 c tnh cht

CHNG 6. HM CH EULER V NH L EULER | 173

6
42
P. Tht vy, 6 | a a 1 vi mi s nguyn a suy ra 6 | a a 1 . Nu a khng chia ht cho 7 th

42
theo nh l Fermat nh ta c 7 | a 6 1 v li suy ra 7 | a a 1 . V vy vi mi s nguyn a th t

6 | a a 42 1 v 7 | a a 42 1 v lu 6, 7 1 suy ra 42 | a a 42 1 suy ra 42 c tnh cht P.


Hn na gi s m l tch ca bn s nguyn t. Tc l m 2 p1 p2 p3 vi 2 p1 p2 p3 . Khi ta c

p1 1| 2 suy ra p1 3 , tng t p2 1| 2 p1 6 suy ra p2 7 v p3 1| 2 p1 p2 42 . Do v


p3 p2 7 ta c p3 1 7, 14, 21 hoc 42 m v p3 nguyn t suy ra p3 1 42 , ngha l p3 43

42
suy ra m 1806 . D thy s 1806 c tnh cht P v ta chng minh 42 | a a 1 vi mi s nguyn

a nn suy ra 42 | a a1806 1 . Nu a chia ht cho 43 th ta c 43 | a a1806 1 . Nu a khng chia ht


cho 43 th tnh cht trn l h qu ca nh l Fermat nh v 43 | a 42 1 suy ra 43 | a1806 1 . Do

42 | a a1806 1 v 43 | a a1806 1 ng vi mi s nguyn a v lu 42, 43 1 v 1806 42 43

1806
suy ra 1806 | a a 1 . Vy 1806 c tnh cht P.

Cui cng gi s m l tch ca nhiu hn bn s nguyn t. Kh m 2 p1 p2 ... pk vi

k 4, 2 p1 p2 ... pk . trn ta thy p1 3, p2 7, p3 43 . Hn na p4 1| m suy ra


p4 1| 2 p1 p2 p3 , ngha l p4 1|1806 . Mt khc p4 1 p3 1 42 v hn na p4 1 chn. Cc c
s ca s 1806 2 3 7 43 khng ln hn 42 l cc s 86, 258, 602, 1806. Do p4 phi l mt trong
cc s 87,259,603 hoc 1807 nhng cc s ny u khng phi s nguyn t v ta c
87 3 29, 259 7 37, 603 32 67, 1807 13 139 . V vy gi thit m l tch ca nhiu hn bn s
nguyn t dn ti mu thun.
Vy ta chng minh c nh l J.Dyer Bennet [1] ni rng cc s 1,2,6,42,1806 l tt c cc s c
tnh cht P. Do chng l tt c cc modulo m m cc ng d thc a b mod m v c d mod m
suy ra ac bd (mod m) vi mi s nguyn dng a, b, c, d .
D thy cc s m c tnh cht P l nhng s khng c c s chnh phng m m | m vi m l
ly tha ph qut nh nht theo modulo m (mc 4).
2
Bi tp. Chng minh rng 2 khng phi cn nguyn thy ca mi s nguyn t c dng 2 1 vi n
l s t nhin 1 .
n

2
2
Chng minh. Nu p l s nguyn t v p 2 1 th 2
n

n1

1 mod p . Nhng p 1 22 2n1 vi


n

n 1 v d dng chng minh bng quy np 2n n 1 vi n 2,3,... Do 2 c s m p 1 theo


modulo p v khng phi cn nguyn thy ca p .
7. Thng d bc n ca mt s nguyn t theo modulo p
Nu p l s nguyn t, n l s t nhin 1 th s nguyn a c gi l thng d bc n theo modulo

p nu tn ti s nguyn x tha mn x n a mod p .


R rng s 0 l thng d bc n vi mi modulo p nguyn t v n l s nguyn ty .
Do ta gi thit cc thng d bc n y u khc 0 .

174 | Thng d bc n ca mt s nguyn t theo modulo

T quan im l thuyt th tn ti phng php quyt nh xem mt s t nhin a 0 c l thng d


bc n theo modulo p hay khng. Tht vy, ch cn kim tra xem c tn ti s x trong dy

1, 2,..., p 1 m tha mn x n a mod p hay khng.


Trong mi lin h ny ta c
nh l 13 (Euler). S nguyn a khng chia ht cho s nguyn t p l thng d bc n theo modulo p
nguyn t khi v ch khi
(27)

a p 1/ d 1 mod p vi d p 1, n

Chng minh. Gi s s nguyn a khng chia ht cho s nguyn t p v l thng d bc n theo


n
modulo p . Khi tn ti s nguyn x khng chia ht cho p tha mn a x mod p . V vy

(28)

a p 1/ d x n

p 1 / d

x p 1

n/ d

p 1
V d | n v theo nh l Fermat nh x 1 mod p , t (28) suy ra (27). iu kin cn c chng

minh. By gi gi s cng thc (27) ng. K hiu g l cn nguyn thy ca p . Trong mc 6 ta


h
h p 1 / d
bit tn ti s nguyn h m 0 h p 2 v a g mod p , theo (27) suy ra g 1 mod p .

V g l cn nguyn thy ca s nguyn t p , t tnh cht cui cng trn suy ra p 1|

d | h v do h kd vi k l s nguyn khng m.

h p 1
nn
d

Theo nh ngha th d p 1, n v theo nh l 16 Chng 1 suy ra tn ti hai s t nhin u, v tha


k p 1 v
mn d nu p 1 v suy ra kd knu k p 1 v . Theo nh l Fermat nh th g 1 mod p .
h
V vy s dng a g g

kd mod p

ta tm c a ag

k p 1v

g kd k p 1v g knu g ku mod p chng


n

t a l thng d bc n theo modulo p . iu kin c chng minh.


Nu a l thng d bc n theo modulo p th r rng mi s ng d vi a mod p cng l thng d
bc n theo modulo p . Do thng d bc n theo modulo p c th c hiu l lp cc s ng d
vi mt thng d bc n theo modulo p .
nh l 14. Nu p l s nguyn t, n l s t nhin v d n, p 1 th s cc thng d bc n phn
bit theo modulo p (tnh c 0 ) l p 1 / d 1 .
Chng minh. K hiu g l cn nguyn thy ca p . K hiu d p 1, n , n dm, p 1 ds vi m, s
l cc s t nhin v

m, s 1 .

K hiu k , l l hai s trong dy 1, 2,..., s m k l . Nu

g kn g ln mod p th p | g kn g ln g ln g k l n 1 do p, g 1 , g k l n 1 mod p . V vy do g l
cn nguyn thy ca s nguyn t p nn p 1| k l n m n dm, p 1 ds suy ra s | k l m do
v m, s 1, s | k l , v l v k v l l hai s phn bit trong dy 1, 2,..., s . Vy ta suy ra cc s

g n , g 2n ,..., g sn c cc s d khc nhau khi chia cho p . Hn na cc s u l thng d bc n theo


n
kn
modulo p (v ng d thc x g mod p c nghim x g k ). Do tn ti t nht s thng d
bc n phn bit theo modulo p . Cc thng d ny u khc 0 .

CHNG 6. HM CH EULER V NH L EULER | 175

By gi k hiu a l thng d bc n ty theo modulo p v khc 0 . Khi tn ti s nguyn x


n
(khng chia ht cho p) tha mn x a mod p . Ta bit trong dy 0,1,..., p 2 tn ti s y m

x g y mod p suy ra a g ny mod p . K hiu r l s d nhn c khi chia y cho x . Ta c

y ks r vi k

l s nguyn khng m v 0 r s . V vy ny nks nr . Nhng v

n dm, p 1 ds nn ns p 1 m . H qu l ny k p 1 m nr suy ra a g ny g nr mod p v

n
2n
chng t khng c thng d bc n theo modulo p khc 0 no ngoi cc thng d 1, g , g ,... g

s 1 n

T sn p 1 m , thng d 1 c th thay bi g . Vy ta chng minh vi s nguyn t p cho


sn

trc th tn ti ng

p 1
1 thng d bc n phn bit theo modulo p .
n, p 1

H qu trc tip ca nh l 14 l vi s t nhin cho trc n th mi s nguyn u l thng d bc


n theo modulo nguyn t p cho trc khi v ch khi n nguyn t cng nhau vi p 1 .
Theo khi n 3 th mi s nguyn u l thng d bc ba theo modulo p nguyn t khi v ch khi
p khng c dng 3k 1 vi k l s t nhin, ngha l p l 2,3 hoc c dng 3k 2 vi k t nhin.
D dng chng minh rng tn ti v hn s nguyn t c dng 3k 2 .
Tht vy, k hiu n l s t nhin ty v t N 6n!1 . R rng N l s t nhin 1 . D thy mi
c s ca N u c dng 6k 1 hoc 6k 1. Khng phi tt c cc c s ca N u c dng 6k 1
v nu nh vy th tch ca chng cng c dng nh vy, iu ny khng ng v N khng c dng .
H qu l N c t nht mt c s nguyn t p 6k 1 vi k l s t nhin. Do p | N 6n! 1 suy ra
p n . V vy do n ty nn tn ti s nguyn t ln ty c dng 6k 1 3(2(k 1) 1) 2 . iu
phi chng minh.
C th chng minh rng nu n l s nguyn t v m l s t nhin 1 th mi s nguyn u l
thng d bc n theo modulo m khi v ch khi m l tch ca cc s nguyn t phn bit m khng c
s no c dng nk 1 (vi k l s t nhin) (Sierpinski [9]).
Bi tp. Chng minh rng nu p l s nguyn t, n l s t nhin v d p 1, n th thng d bc

n theo modulo p nguyn t trng vi thng d bc d theo modulo p .


Chng minh. V d p 1, n nn ta c d | p 1 v do d p 1, d v theo nh l 13 th iu
kin cn v mt s nguyn a khng chia ht cho p l thng d bc n theo modulo p l trng
vi iu kin a l thng d bc d theo modulo p . Do tp hp cc thng d bc n v thng d
bc d l trng nhau.
c bit, nu p l s nguyn t c dng 4k 3 vi k 0,1, 2,..., th (v 2 p 1, 4 ) cc thng d bc
hai theo modulo p trng vi cc thng d bc bn theo modulo p .
8. Cc tnh cht v ng dng ca hm ch s
Trong mc 4 ta nh ngha cn nguyn thy ca s t nhin m l s nguyn g c s m m
theo modulo m . T suy ra cc s g 0 , g1 ,..., g

m 1

l khng ng d mod m . V s cc s trong

dy ny l m tc l ng bng vi s cc s nguyn t cng nhau vi m trong dy 1, 2,..., m, nn


vi mi s nguyn x nguyn t cng nhau vi m th tn ti ng mt s y trong dy

176 | Cc tnh cht v ng dng ca hm ch s

0,1, 2,..., m 1 m g y x mod m . Ta ni y l ch s ca x theo cn nguyn thy g v k hiu l


ind x , hoc nu khng c g gy nhm ln th k hiu n gin l ind x . Ta gi g l c s ca ch s.
C nh s t nhin m 1 c cn nguyn thy g v xt hm ch s ind x vi s nguyn x nguyn t
cng nhau vi m . Ta chng minh cc tnh cht sau y ca hm ch s
I. Ch s ca cc s nguyn ng d mod m v nguyn t cng nhau vi m l bng nhau.
Tht vy, nu a b mod m v g inda a mod m th g inda b mod m . Nhng v b, m 1 nn ng
d thc g x b mod m c ng mt nghim trong cc s 0,1,..., m 1 v l ind b suy ra

ind a ind b . V vy trong bng cc gi tr ca ind x ch xut hin cc s x nh hn modulo v


nguyn t cng nhau vi modulo.
II. Ch s ca tch l ng d mod m vi tng cc ch s ca cc nhn t, ngha l
(29)

ind ab ind a ind b mod m .

Tht vy, theo nh ngha ca ch s ta c g ind a a mod m , g ind b b mod m vi cc s a v b


nguyn t cng nhau vi m . V vy nhn hai ng d thc cui cng ta nhn c
g ind a ind b ab mod m . Nhng v g ind ab ab mod m ta suy ra
(30)

g ind {ab ) g ind a ind b mod m .

Gi s vi mi s nguyn khng m ,v th ng d thc g g v mod m ng. Nu v th

m | g v g v 1 m g , m 1 suy ra g v 1 mod m . S g l cn nguyn thy ca m c s m

m theo modulo m . V vy theo nh l 9 suy ra m | v .


Tnh cht cui cng cng ng vi v . V vy t (30) ta c ng d thc (29).
Tnh cht ny c m rng cho hu hn nhn t. V vy
III. Ch s ca ly tha bc n (n l s t nhin) l ng d mod m vi tch ca n vi ch s ca
c s. Ta c ind a n n ind a mod m .

By gi ta trnh by quan h gia cc ch s ly vi cc cn nguyn thy khc nhau ca mt s c nh


ind a
m . Theo nh ngha ca ch s ta c a g g mod m . V vy s dng cc tnh cht I v II ta c

ind a ind g a ind g mod m


vi l cn nguyn thy ca m . Do i c s ca ch s th ta ch cn nhn mi s vi mt s
c nh (gi l ch s ca ch s c i vi c s mi) v tnh ng d theo modulo m ca tch.
nh l 15. mt s a khng chia ht cho p l thng d bc hai ca mt s nguyn t l p th iu
kin cn v l ind a chn.
Chng minh. Gi s ind g a 2k vi k l s nguyn khng m. Ta c g 2 k a mod p chng t ng
d thc x 2 a mod p c nghim x g k . Do a l thng d bc hai modulo p .

CHNG 6. HM CH EULER V NH L EULER | 177

1
p 1 s c ch s chn v cc ch s ca cc s trong dy trng vi cc s
2
1
0,1, 2,..., p 2 (sai khc mt th t) m trong c ng p 1 s chn. Cc s u l thng d
2
1
bc hai ca p . Nhng theo nh l 14 vi d 2, p 1 2 th ch c p 1 thng d bc hai trong
2
dy 1, 2,..., p 1 . Vy khng c s no c ch s l m l thng d bc hai theo modulo p nguyn t.
nh l c chng minh.
Trong dy 1, 2,..., p 1 c

H qu trc tip ca nh l 15 l khng tn ti cn nguyn thy ca s nguyn t l p li l thng d


bc hai ca p .
Ta lu rng mnh tng t vi thng d bc n vi n ln hn 2 l khng ng. Chng hn, trong
cc ch s theo modulo 5 th c hai s l 0 v 3 l chia ht cho 3 v cc s 1,2,3,4 u l thng d bc
ba modulo 5 v 1 13 mod 5 , 2 33 mod 5 , 3 23 mod 5 , 4 43 mod 5 . Vi modulo 7 th cc s

1 14 mod 7 , 2 24 mod 7 , 4 34 mod 7 l thng d bc bn vi ch s theo modulo 7 li l 0


hoc 4 u chia ht cho 4.
Cc ch s c s dng gii cc ng d thc.
Xt s nguyn t p v cc s a , b khng chia ht cho p . Xt ng d thc ax b mod p .
T cc tnh cht I v II ta c ind a ind x ind b mod p 1 suy ra ind x ind b ind a mod p 1 .
S ind x l s d nhn c khi chia hiu ind b ind a cho p 1 . V vy khi bit gi tr ind x ta tnh

x theo x g ind x mod p . Trong thc hnh phng php ny yu cu s dng bng cc ch s.
t a l s nguyn khng chia ht cho p v n l ly tha t nhin. Xt ng d thc x n a mod p .
Theo I v III th ng d thc ny tng ng vi n ind x ind a mod p 1 v vy bi ton gii
ng d thc c quy v bi ton gii ng d thc tuyn tnh.
Xt ng d thc a x b mod p vi a, b l cc s nguyn khng chia ht cho s nguyn t p . Khi
ng d thc ny tng ng vi ng d thc tuyn tnh x ind a ind b mod p 1 .
V d. Ta trnh by bng cc ch s mod13 . u tin ta tnh cc cn nguyn thy ca 13. Ta bt u
vi s 2. Ta tnh thng d mod13 ca cc ly tha lin tip ca 2. R rng ch cn tnh 2n vi s m
t nhin n . Nu rk l s d nhn c khi chia 2k cho 13 th s d nhn c khi chia 2k 1 cho 13 l

2rk . Theo cch ny ta tm c 2 2 , 22 4 , 23 8 , 24 3 , 25 6 , 26 12 , 27 11 , 28 9 , 29 5 ,

210 10 , 211 7 , 212 1 mod13 . Vy 2 l cn nguyn thy ca 13. Ta c bng cc s x tng ng


vi ch s ca n l ind2 x k (vi k 0,1,...,11 ) nh sau

S dng bng ny ta c bng cc ch s ng vi x 1, 2,...,12 nh sau

178 | Cc tnh cht v ng dng ca hm ch s

Vi 6 x 5 mod13 ta c ind 6 ind x ind 5 mod12 suy ra ind x ind 5 ind 6 mod12 . Kim
tra trong bng th hai ta c ind 5 9 v ind 6 5 v vy x 9 5 4 mod12 v do ind x 4 v
s dng bng th nht suy ra x 3 .
8
Xt ng d thc x 3 mod13 . Ta c 8 ind x ind 3 mod12 . Mt khc, s dng cc bng trn

ta c ind 3 4 suy ra nu t ind x y th ta nhn c ng d thc 8 y 4 mod12 . ng d


thc ny tng ng vi 12 | 8 y 4 do 3 | 2 y 1 , ngha l 2 y 1 mod 3 . V vy 4 y 2 mod 3 .
Nhng v 4 1 mod 3 , y 2 mod 3 v do y 2 3k vi k l s nguyn. Cc s c dng ny m
thuc dy 0,1, 2,...,11 l cc s 2,5,8 v 11. H qu l chng l cc gi tr ca y ind x . S dng bng
th nht ta tnh c x bng 4,6,9 v 7. V vy ng d thc ban u c ng bn nghim l 4,6,7,9.
x
Cui cng xt ng d thc 6 7 mod13 . Ta c x ind 6 ind 7 mod12 . Kim tra trong bng th

hai ta c ind 6 5 , ind 7 11 . V vy ng d thc tr thnh 5 x 11 mod12 , iu ny ch ng vi

x 7 vi cc s x thuc dy 0,1,,11. Vy ng d thc ban u c nghim 7 12k vi k 0,1, 2,... .


Bi tp. 1. Chng minh rng vi mi s nguyn t l p nguyn t vi mi cn nguyn thy ca p th

ind 1 ind p 1

1
p 1
2

Chng minh. Theo nh l Fermat nh th vi mi cn nguyn thy g ca s nguyn t l p th


1
12 p 1 12 p 1
p 1
p | g 1 g
1 g
1 . Nhng v p | g 2
1 l khng th v g l cn nguyn thy ca

1
1
p 1
p 1
1
1 , ngha l g 2
p nn p | g 2
1 mod p chng t ind 1 p 1 .
2
p 1

2. Chng minh rng iu kin cn v s nguyn g nguyn t cng nhau vi s nguyn t l p


p 1 / q
l cn nguyn thy ca p l g 1 mod p vi mi c s nguyn t q ca p 1 .

p 1 / q
1 mod p th q c s m
Chng minh. Nu vi s nguyn t q m q | p 1 v g

p 1 / q p 1 v do g khng phi cn nguyn thy ca p . iu kin cn c chng minh.


Mt khc, gi s s nguyn g nguyn t cng nhau vi p khng phi cn nguyn thy ca p . Khi
ly tha ca g theo modulo p l p 1 . Do l c s ca p 1 nn p 1 / l s t nhin

1 v do n c c s nguyn t q . Vy ta c q | p 1 / suy ra | p 1 / q v v p | g 1 ( g

c s m theo modulo p ) nn p | g

iu kin c chng minh.

p 1 / q

1 , ngha l g p 1/ q 1 mod p .

CHNG 7
BIU DIN H C S TY
1. Biu din ca s t nhin trong c s ty
Xt g l s t nhin 1 . Biu din c s g ca s t nhin N c cho bi cng thc
(1)

N cm g m cm1 g m1 ... c1 g c0

Trong m l s nguyn 0 v cn n 0,1, 2,..., m l cc s nguyn tha mn tnh cht


(2)

0 cn g 1

vi

n 0,1,..., m

cm 0 .

Mi s thuc dy
(3)

0,1, 2,..., g 1

c gi l cc ch s v cng thc (1) c th vit thnh N m m1... 1 0 g , vi n l ch s k


hiu cho cn . Nu g 10 cc s 0, 1, 2, 3, 4, 5, 6, 7, 8, 9 c dng lm k hiu cho cc ch s trong
(3). V d

N 10010 2 ngha l
N 56037 ngha l

N 1 24 0 23 0 22 1 2 0 18,

N 5 73 6 72 0 7 3 2012 .

nh l 1. Mi s t nhin u c biu din duy nht trong c s g ( g l s t nhin 1 ) ngha


l c th biu din di dng (1) trong cc s nguyn cn n 0,1,..., m tha mn cc bt ng
thc (2).
Chng minh. Gi s s t nhin N c th biu din nh trong (1) vi cn n 0,1,...m nguyn
tha mn (2). K hiu n l mt trong cc s 0,1, 2,..., m 1 . T (1) ta c
(4)

c
c
c
N
cm g mn cm1 g mn1 ... cn n1 n22 ... 0n .
n
g
g
g
g

cn1 cn2
c
g 1 g 1
g 1
1
2 ... 0n
2 ... n 1 n . V vy t (4) ta suy
g
g
g
g
g
g
g
N
N
ra n cm g mn cm1 g mn 1 cn 1 g cn v tng t n 1 cm g mn 1 cm1 g mn 2 ... cn 1 .
g
g
Cc cng thc ny suy ra
Nhng theo (2) th 0

(5)

N
N
cn n g n 1 vi mi n 0,1,..., m .
g
g

T (1) v (2) ta cng c

g m N g 1 g m g m1 ... g 1 g m1 1 g m1
suy ra m log g log N m 1 log g v v vy m
(6)

log N
m
.
log g

log N
m 1 . Do
log g

180 | Biu din ca s t nhin trong c s ty

Cc cng thc (6) v (5) chng t rng nu N c biu din di dng (1) v iu kin (2) c
tha mn th cc s m v cn n 0,1,..., m xc nh duy nht theo N . iu ny chng t vi s t
nhin cho trc N (vi s t nhin xc nh g 1 ) tn ti nhiu nht mt biu din dng (1) tha
mn (2). V vy chng minh nh l ta ch cn ch ra rng vi mi s t nhin N v g 1 tn
ti t nht mt biu din dng (1) tha mn (2). Gi N1 v c0 l thng s v s d nhn c
khia chia N cho g . Ta c N c0 gN1 . Thay N bng N1 ta tm c thng s N 2 v phn d

c1 t php chia N1 cho g . Tip tc nh vy vi N 2 v c nh th. R rng tt c cc thng s


ln lt thu c s gim bi v Nn1 Nn / g . Do cc s ny u l cc s nguyn khng m nn
vi k 1 no ta phi c N k 0 . K hiu m l ch s ln nht m N m 0 . Ta c dy ng thc

N c0 gN1 , N1 c1 gN2 , , Nm1 cm1 gNm , N m cm .


2
m
V vy ta thu c biu din ca N c dng N c0 c1 g c2 g ... cm g trong cm 0 bi v

N m 0 v cn n 0,1,..., m l cc phn d thu c khi chia cho g nn n tha mn (2).

Vy ta chng minh c nh l 1 v ta cng c ngay thut ton tm biu din ca N trong c


s g nh sau: ta chia N cho g v k hiu phn d l c0 v thng s l N1 , sau ta chia N1
cho g v k hiu phn d l c1 v thng s l N 2 . Tip tc qu trnh ny cho ti khi nhn c
thng s N m1 0 . Khi ta thu c biu din ca N theo dng (1). Trng hp ring khi

g 2 th ch c hai ch s l 0 v 1. T nh l 1 ta c h qu sau y
H qu. Cc s t nhin c th biu din duy nht di dng tng cc bnh phng ly tha phn
bit ca 2.
n
n 1
n2
0
V d: 100 26 25 22 , 29 24 23 22 20 , M n 2 1 2 2 ... 2 2 .

Bi tp. 1. Tm tt c cc ch s trong biu din c s 2 ca 12 s nguyn t u tin.


Li gii. 10, 11, 101, 111, 1011, 1101, 10001, 10011, 10111, 11101, 11111, 100101.
2. Chng minh rng vi mi s t nhin m tn ti s nguyn t m biu din ca n trong c s 2
tha mn ch s cui cng ca n l 1 v m ch s lin trc l 0.
Li gii. Theo nh l 11 Chng 6, vi s t nhin m tn ti s nguyn t p c dng mm1k 1 ,
k t nhin. Biu din ca s nguyn t ny trong c s 2 c ch s cui cng l 1 v m ch s lin
trc bng 0.
Ghi ch. Tn ti nhng s nguyn t m biu din c s 2 ca chng gm ton ch s 1. C 30 s
nh th c tm ra, s ln nht c 216091 ch s 1 trong h c s 2. Ta cha bit c tn ti v
hn cc s nh vy hay khng. R rng nhng s nguyn t nh vy u c dng 2n 1 . Mt khc
c nhng s nguyn t m trong biu din h c s 2 ca chng th ch c ch s u v ch s
cui bng 1 cn li tt c u l 0. V d cc s 11, 101, 10001, 100000001 v
10000000000000001. Ta cha bit ngoi cc s ny ra th c s nguyn t no nh vy hay khng.
2
Tt c cc s nh vy u c dng 2 1 .
n

3. Chng minh vi s t nhin bt k s 1 tn ti t nht 2 s nguyn t m biu din c s 2 c


ng s ch s.
Chng minh. Vi s 2 v s 3 kt qu c suy ra t bi tp 1. Nu s 4 th 2s1 5 v theo
nh l 7 Chng 3 suy ra gia 2n1 v 2 s tn ti t nht hai s nguyn t. Mt khc nu n l s t
nhin m 2s 1 n 2s th n c ng s ch s trong h c s 2.
4. Chng minh rng ch s cui cng trong biu din c s 12 ca s chnh phng l mt bnh
phng ng.

CHNG 7. BIU DIN H C S TY | 181

Chng minh. Nu ch s cui cng ca s t nhin l 0,1,...,11 th ch s cui cng trong h c s


12 ca bnh phng ca n l 0,1, 4,9, 4,1,0,1, 4,9, 4,1, tng ng.
Ghi ch. Tt c cc c s c tnh cht trn l 2, 3, 4, 5, 8, 12, 16 (xem Muller [1]).
5. Chng minh rng tn ti v hn s t nhin n khng chia ht cho 10 v tha mn s n nhn
c t n bng cch o ngc th t cc ch s thp phn ca n l c s ca n v n : n 1 .
Chng minh. Cc s 9899...9901 9 1099...9989 v 8799...9912 4 2199...9978 trong s ch
s 9 gia l ty v bng nhau c hai v c tnh cht . C th chng minh s nh nht 9 c
tnh cht trn l 8712 4 2178 v cc s trn l tt c cc s c tnh cht nh vy (xem Subba
Rao [1]). Vn khi no th tn ti cc s nh vy c t ra bi D.R.Kaprekar.
6. Chng minh rng mi s t nhin c th biu din duy nht di dng
(*)

n a1 1! a2 2! ... am m!,

vi m l s t nhin, am 0 v a j j 1, 2,..., m l cc s nguyn tha mn 0 a j j vi

j 1, 2,..., m .
Chng minh. Gi s s t nhin n c hai biu din dng (*). Ta c

a1 1! a2 2! ... am m! a1 1! a2 2! ... am m!.


K hiu k l s t nhin ln nht tha mn ak ak , ngha l ak ak suy ra ak ak 1 , v vy

k ! ak k ! ak k ! a1 1! ... ak 1 k 1! a1 1! ... ak 1 k 1!
11! 2 2! ... k 1 (k 1)! k ! 1 k ! ,
V l. Xt s l s t nhin. Xc nh tt c cc biu din dng (*) vi m s v 0 a j j vi

j 1, 2,..., m . S cc biu din nh vy l 1 1 2 1 ... s 1 s 1! . V vy s cc biu din (b


qua trng hp n 0 ) l s 1! 1. trn ta chng minh cc biu din (*) khc nhau s cho
cc s n khc nhau. Mt khc mi biu din dng (*) vi m s cho ta s t nhin
11! 2 2! ... m m! m 1! 1 s 1! 1 . V vy hin nhin mi s t nhin s 1! 1 c
th biu din di dng (*) nh l n trn vi m s .
7. Vi cc s t nhin g v s c nh t f n l tng cc ly tha bc s ca cc ch s trong h
c s g ca s t nhin n . Chng minh rng dy v hn
(i)

n, f n , ff n , fff n ,...

tun hon.
Chng minh. R rng ta ch cn chng minh trong dy trn c hai phn t bng nhau. Xt s t
k 1
nhin n a0 a1 g ... ak 1 g
l biu din ca n trong h c s g . Ta c

f n a0s a1s ... aks 1 k g 1 kg s . Nhng g k / k tin ti v cng theo k do vi k


s

ln ta c g k / k g s 1 . V vy kg s g k 1 n . T y ta suy ra vi n ln, k hiu l n m , ta c


f (n) n . Vy vi mi s x m th tn ti k ( x) m fff .... f ( x) m ( f c tnh k ( x) ln). Do
trong dy ban u c v hn phn t khng vt qu m . Suy ra tn ti hai phn t bng nhau.
Ghi ch. Vi g 10 v s 2 , Porges [1] chng minh dy (i) tun hon s cha mt phn t
bng 1 hoc 8 phn t 4,16, 37, 58, 89, 145, 42, 20. V d vi n 3 ta c dy 3, 9, 81, 65, 61, 37, 58,
, 16, 37, . Nu n 5 ta c dy 5, 25, 29, 85, 89,145, , 58, 89, . Nu n 7 ta c dy 7, 49, 97,
130, 10, 1, 1, 1, Tng qut ha kt qu ca Porges c trnh by bi B.M.Stewart [1]. Trng

182 | Biu din ca s t nhin trong c s ty

hp g 10 v s 3 c gii quyt bi K.Iseki [1], ngi chng minh rng c 9 vng lp nh


vy. l: mt phn t trong cc trng hp 1, 153, 370, 371, 407; hai phn t vi cp 136 v
244 hoc 919 v 1459; ba phn t vi cc b ba 55, 250, 133, hoc 160, 217, 252 (xem Iseki [2]).
K.Chikawa, K.Iseki v T.Kusakabe [1] chng minh rng trong trng hp g 10 , s 4 th c 6
vng lp nh vy. l: mt phn t thuc 1, 1634, 8208, 9474; hai phn t 2178, 6514; by phn
t 13139, 6725, 4338, 4514, 1138, 4179, 9219 (Chikawa, Iseki, Kusakabe v Shibamura [1] tm ra
tt c cc vng lp vi g 10 , s 5 , Avanesov, Gusev [1] gii quyt cc trng hp g 10 , s 6
hoc 7, Takada [1] gii quyt trng hp g 10 , s 8 , Iseki v Takada [1] vi g 10 , s 9 v
cui cng Avanesov, Gusev [2] gii quyt trng hp g 10 , s 10 hoc 11).
8. Chng minh vng lp ca dy (i) c th di ty .
Chng minh. Vi s t nhin n ty tn ti s t nhin m n tha mn f m n . Tht vy vi
mi s t nhin s th tng cc ly tha bc s ca cc ch s ca n trong h c s g ca

g n 1
l n v hn na nu n 1 ta c m n . Nu n 1 ta t m g .
g 1

9. Tnh bng tng v tch cc ch s trong h c s 7.


Li gii.

2. Biu din trong h c s m


nh l 2. Nu g l s nguyn 1 th mi s nguyn N 0 c th biu din duy nht di dng
(1) trong cn n 0,1,..., m nguyn tha mn
(7)

0 cn g

vi n 0,1,..., m v cm 0 .

nh l ny c Z.Pawlak v Andrzej Wakulicz [1] tm ra bng cch s dng my tnh in t.


Chng minh. Xt s nguyn g 1 v x N l s nguyn ty . K hiu c0 l phn d khi chia

x cho g . Ta c 0 c0 g v x c0 gx1 trong x1 nguyn. V vy gx1 x c0 v do


gx1 x c0 x g 1 trong x1 x g 1 / g .

Nu

x g 1 / g x

th x g 1 g x , ngha l g 1 g 1 x suy ra v g 1 ta c

x 1 do x 0,1 hoc 1 . Nu x 0 hoc x 1 th

x c0 . Nu

x 1

th

x g 1 g c0 g suy ra c0 g 1 . V vy ch cn xt trng hp x g 1 / g x . Ta c
x1 x v c th p dng th tc va s dng vi x cho x1 .
Tip tc qu trnh ny v sau hu hn bc ta nhn c biu din ca N trong dng (1) vi
cn n 0,1,..., m nguyn tha mn (7).
Tip theo chng minh biu din ca N trong dng (1) tha mn (7) l duy nht, ta ch cn
nhn xt N chia cho g c phn d c0 , N c0 / g chia cho g c phn d c1 v c th suy ra

c0 , c1 , c2 ,... xc nh duy nht theo N v vy biu din ny l duy nht.

CHNG 7. BIU DIN H C S TY | 183

nh l 2 c chng minh.
V d. 1 112 , 10 111102 , 10 1010 2 , 16 100002 , 16 110000 2 ,

25 11010012 , 25 1110112 , 100 1101001002 102013 .

3. Phn s v hn trong h c s cho trc


Xt s t nhin g 1 v s thc x . t x1 x [ x] . Ta c 0 x1 1 . Hn na vi x2 gx1 gx1

th 0 x2 1 . Tip tc ta xc nh x3 bi gx2 gx2 v c nh vy ta thu c dy v hn

xn n 1, 2,... nh ngha bi cc iu kin


x1 x x ,

(8)

xn1 gxn gxn

vi

n 1, 2,...

Cc cng thc ny suy ra


(9)

0 xn 1

vi

n 1, 2,...

cn gxn

vi

n 1, 2,...

t
(10)

Theo (9) ta c 0 gxn g v vy theo (10) 0 cm g v do

0 cn g 1

(11)

T (8) v (11) suy ra x x x1 , x1

x x

(12)

vi

n 1, 2,...

c x3
c xn 1
c1 x2
, x2 2
, , xn n
. V vy vi n 1, 2,...,
g
g
g

c
x
c1 c2
2 ... nn n n1 .
g g
g
g

xn 1 1
x
n v t g 2, g n tng v hn theo n , ta suy ra lim n n1 0 . V theo (12)
n
n

g
g
g
ta nhn c khai trin ca x thnh chui v hn
Theo (9) th 0

x x

(13)

c1 c2 c3

...
g g2 g3

trong theo (11) cc s cn l cc ch s trong h c s g .


V vy ta chng minh mi s thc x c t nht mt biu din c dng (13) vi mi h c s
g 1 trong cn l cc ch s trong h c s g .
Gi s rng s thc x c biu din trong dng (13) vi cn l cc s nguyn tha mn (11) th
vi mi n 1, 2,... t
(14)
Ta c x rn

rn x

c
c1 c2
2 ... nn .
g g
g

cn1 cn 2
g 1 g 1
1
n 2 ... suy ra ra theo (11), 0 x rn n1 n 2 ... n .
n 1
g
g
g
g
g

n
Biu thc x rn 1/ g dng ch trong trng hp cn1 cn ... g 1 ngha l tt c cc ch s
n
trong biu din u bng g 1 vi n xc nh. Vy x rn 1/ g v theo (14) suy ra x l thng

s ca mt s nguyn v mt ly tha ca g . Nu m l s t nhin nh nht m cn g 1 vi

n m th nu m 1 th theo (13) ta c x x 1 , v l. Nu m 1 th cm1 g 1 suy ra theo

184 | Phn s v hn trong h c s cho trc

(11) th cm1 g 1 ngha l cm1 g 2 suy ra cm 1 cm1 1 cng l ch s trong h c s g . H


qu l ta c biu din khc vi (13)

x x

c
c
c1 c2
0
0
2 ... mm22 mm11 m m1 ... ,
g g
g
g
g
g

D dng chng minh iu ngc li. Nu x l thng s ca mt s nguyn v mt ly tha ca g


th x c hai biu din dng (13) vi cn l cc s nguyn tha mn (11). Trong c mt biu din
m ch c hu hn s cn khc 0. Trong biu din cn li th tt c cc h s ny bng g 1 .
Nu s thc x khng l thng s ca mt s nguyn v mt ly tha ca g th 0 x rn

1
gn

n
n
n
n
n
vi n 1, 2,..., v vy 0 g x g rn 0 . Suy ra t (14) s g rn nguyn. Ta c g rn g x . iu

ny cng ng vi n 0 t nh ngha r0 = x . Ta c
(15)

g n rn g n x

Nhng theo (14) th rn rn 1


(16)

g n1rn1 g n1 x vi

n 1, 2,...

cn
n
n 1
vi mi n 1, 2,..., vy cn g rn gg rn1 v theo (15) suy ra
n
g

cn g n x g g n1 x ,

n 1, 2,...

Chng t mi s thc x khng l thng s ca mt s nguyn v mt ly tha ca g c ng


mt biu din dng (13) trong cn l cc s nguyn tha mn (11). Biu din ny k hiu bi
(17)

x x 0, c1 c2 c3 ... g .

Cng thc (16) cho ta cch tnh ch s th n kh n gin. Tuy nhin khng d tnh ton cc
gi tr v phi ca n. V d vi g 10 cng thc (16) cho ch s th 1000 ca

2 l

c1000 101000 2 10 10999 2 .


Ta chng minh rng thu c biu din dng (17) ca mt s thc ty ta c th p dng
thut ton: x1 x x , c1 gx1 , x2 gx1 c1 , c2 gx2 , x3 gx2 c2 , , xn gxn1 cn1 ,

cn gxn , .

Ta cng chng minh rng biu din (13) l hu hn (ngha l tt c cc ch s trong biu din
u bng 0 tr ra mt s hu hn) nu v ch nu x l thng s ca mt s nguyn v mt ly
tha ca g .
D dng chng minh iu kin ny tng ng vi vic ni rng x l s hu t c dng biu din
phn s ti gin vi mu s l tch ca cc s nguyn t m mi s u l c s ca g .
iu kin cn l hin nhin.
Mt khc nu x l / m vi l l s nguyn v m l s t nhin tha mn mi c s nguyn t ca
m l c ca g th nu g q11 q22 ...qss l phn tch thnh cc tha s nguyn t ca g ta c

m q11 q22 ... qss vi 1 , 2 ,...s l cc s nguyn khng m. Vi s t nhin k tha mn ki i


vi mi i 1, 2,..., s . Th m | g k do g k hm vi h l s t nhin. V vy x l / m hl / g k suy ra
iu kin .
V vy nu s thc x khng phi s hu t m biu din dng phn s ti gin ca n c mu s l
tch ca cc c s nguyn t u l c ca g th x c ng mt biu din dng (13) vi

CHNG 7. BIU DIN H C S TY | 185

cn n 1, 2,... l cc ch s trong h c s g . Hn na biu din ny l v hn v c v hn ch s


khc g 1 . Biu din ny thu c bng thut ton trn.
Thut ton tm biu din ca s thc x vn c hiu lc trong trng hp g l s thc 1 . Khi
cc cng thc (8), (9), (10) v (12) vn ng. Mnh v cc s cn cng vn ng nhng cn
thm iu kin 0 cn g v cn nguyn. V d vi g 2 , x 2 th biu din cho bi thut
ton l

2 1

dng (13) l

2 2 2 2
3

2 2 1 1

4
2

12

21

... Tuy nhin

2 cn c biu din khc

... Vi g 2 v x 2 2 1 / 4 ta c hai biu din c

...

... , trong ng thc sau c

cho bi thut ton trn. Ta cng c

1
2

2 4

...

...

Vi biu din th hai cho bi thut ton trn. Xem Gelfond [1].
4. Biu din ca cc s hu t
Xt s hu t x c dng phn s ti gin l / m v gi s biu din ca x c dng (13) vi
cn n 1, 2,... l cc ch s trong h c s nguyn g 1 . t xn n 1, 2,... l cc s nh ngha
bi (8). Khi cc cng thc (9), (10) ng. Theo (8) ta c mx1 l [ x] . T mx1 l s t nhin
v t (8) ta c mxn1 gmxn m gxn vi mi n 1, 2,..., t theo quy np ta suy ra mi s mxn

u nguyn v hn na theo (9) th chng tha mn cc bt ng thc 0 mxn m vi n 1, 2,...


Nu vi n no ta c xn 0 th theo (8) ta c x j 0 vi mi j n . V vy theo (10) th c j 0
vi j n v biu din (13) ca x l hu hn. Hn na gi s xn 0 vi mi n 1, 2,... ta c

0 mxn m vi n 1, 2,... v do cc s mx1 , mx2 ,..., mxm ch c th nhn m 1 gi tr phn bit


1, 2,..., m 1 . T y suy ra tn ti cc s t nhin h v s tha mn h s m v mxh mxh s , m
theo (8) suy ra xn xn s vi n h do theo (10) th cn cn s vi n h . Suy ra dy v hn cc
ch s ca (17) l tun hon. Ta c nh l sau y
nh l 3. Biu din ca s hu t trong dng (13) vi g l s t nhin ln hn 1 s tun hon. S
cc ch s ng trc vng lp nh hn mu s ca s ban u.
Xt dy v hn ty c1 , c2 ,..., vi cn n 1, 2,... l cc ch s trong h c s g . Th th cn tha
mn (11), v vy suy ra chui v hn (13) hi t v tng x ca chng l s thc. T nh l 3 suy
ra nu dy c1 , c2 ,... khng tun hon th x l s v t. chng minh iu ngc li ta ch cn
chng minh nu dy cc ch s c1 , c2 ,... tun hon th s (17) hu t. Gi s dy c1 , c2 ,... l tun
hon. Ngha l vi s t nhin s v h no ta c cn s cn v do n h . Suy ra

186 | Biu din ca cc s hu t

c1 c2

...
g g2
c
c
c
c
c
c
c
c c
1 22 hh11 hh hh11 ... hhss11 hh s hhs11 ... hh2s s 11 ...
g g
g
g
g
g
g
g
g
c

c
c
c
c1 c2
1
1
2 ... hh11 hh hh11 ... hhss11 1 s 2 s ...
g g
g
g
g
g
g
g

s
c
g
c
c
c
c c
1 22 ... hh11 hh hh11 ... hhss11 s
g g
g
g
g
g
g 1

c1 g h s 2 c2 g h s 3 ... ch 1 g s ch g s 1 ... ch s 1 c1 g h 1 c2 g h 2 ... ch 1

g h 1 g s 1
g h 1 g s 1

c1 c2 ... ch s 1 g c1 c2 ... ch1 g


g h 1 g s 1

g h 1 g s 1

V vy ta thy tng ca chui trn l s hu t

c1 c2 ... ch s 1 g c1 c2 ... ch1 g


g h 1 g s 1

. Tuy nhin phn s

ny cha chc ti gin.


Cng thc cho ta quy tc rt gn cc biu din tun hon trong h c s g 1 . Ta c
nh l 3a . Vi h c s t nhin g 1 cho trc, cc s c biu din dng (13) tha mn dy cc
ch s ca n l tun hon u l s hu t (biu din hu hn c coi nh tun hon vi chu k
bng 0 hoc g 1 ).
H qu ca nh l 3a l nu x c biu din khng tun hon trong h c s g th x l s v t.
T nhn xt ny ta c th chng minh s a 0.1234567891011121314... (cc s 1,2,3, c vit
cnh nhau) l s v t. chng minh iu ny ch cn ch rng trong biu din ca a s xut
n
hin s 10 n 1, 2,... vi n ty , do n cha dy s 0 di v hn, do khng th tun hon
chu k hu hn c.
Bi tp. 1. Biu din s

1
di dng thp phn
99 2

1
0.00010203...0809101112...969799 trong cc du chm pha trn cc ch s ch
992
ra vng lp. Vng lp bt u t ngay sau du phy thp phn v thu c bng vic vit lin tip
cc s t 0 ti 99 tr ra s 98. Tng qut hn J.W.L.Glaisher [1] a ra cng thc

(0.0123...
g

3
g

1) g vi g l s t nhin 2 .
2
g 1
Li gii.

2. S dng e 2.718281828... hy biu din e trong c s 2 chnh xc ti 24 ch s.


Li gii. e (10.101101111110000101010001...) 2 . Biu din ny c a ra bi G.Peano trong
[1] trang 154. Trong ta c e !.,!.!!.!!!!!!....!.!.!...!2 .

3. S dng 3.14159265... hy biu din trong c s 2 chnh xc ti 24 ch s.


Li gii. 11.001001000011111101101010...2 (xem G.Peano [1] trang 177).
4. Chng minh rng trong mi biu din thp phn v hn tn ti dy di ty cc ch s xut
hin v hn ln.

CHNG 7. BIU DIN H C S TY | 187

Chng minh. K hiu 0. c1 c2 c3 ... l biu din thp phn v hn v m l s t nhin. Xt cc khi
c m ch s xut hin trong dy c1 c2 ... , ngha l cc dy
(18)

ckm1 , ckm2 ,..., ckm m

vi

k 0,1,...

Ta chia tt c cc dy nh vy thnh hai lp vi tnh cht hai dy thuc cng mt lp nu v ch


nu cc phn t ca dy ny u tng ng bng cc phn t ca dy kia. R rng s cc lp cc
dy cha m phn t khng vt qu 10m do hu hn. Nhng mt khc tn ti v hn dy c
dng (18) suy ra t nht mt dy xut hin v hn ln.
Ghi ch. Trong trng hp ring ca nh l trn ta ch rng trong mi biu din thp phn v
hn t nht c mt ch s xut hin v hn ln. Hn na nu l mt s v t th c t nht hai
ch s xut hin v hn ln. Tuy nhin ta khng bit vi 2 v th hai ch s l bao nhiu.
L.E.J.Brouwer lu rng ta khng bit dy 0123456789 c xut hin trong biu din thp phn
ca hay khng. Biu din thp phn ti ch s th 100000 ca e v c trnh by ln lt
bi Shanks v Wrench trong [2] v [1]. S c tnh chnh xc ti ch s th 1000000 bi
Gilloud v Bourger [1] v ti 4196239 trong Tamura v Kanada [1].
5. Chng minh rng s c c ... c 10 vi cc ch s trong h c s 10 l c trong c 2 , c 5 hoc

c 6 khng c dng m n , vi m v n l cc s t nhin 1 .


Chng minh. Cc s 2, 5, 6 khng chia ht cho bt k bnh phng no 1 . V vy chng khng c
dng m n vi cc s t nhin m , n 1 . Cc s c ch s tn cng l 22, 55, 66 ln lt khng chia
ht cho cc s 4, 25 v 4 suy ra chng khng c dng m n vi cc s t nhin m , n 1 . Mt s
4 vi cc ch s trong biu din thp phn u bng 4 th chia ht cho 4 nhng khng chia ht
cho 8. Do khng th l ly tha bc n ca mt s t nhin m vi n 3 . Nu 44...4 m2 th s
1111 l bnh phng ng, nhng iu ny l khng th v hai ch s tn cng ca bnh phng
khng th l 11.
Ghi ch. R.Oblath [1] ch ra rng nu cc s 333, 777, 888, 999 ln hn 10 th chng
khng c dng m n vi m, n l cc s t nhin 1 . Ta cha bit 111 c th c dng khng
(xem Shorey v Tijdeman [1]).
6. Biu din
Li gii.

1
0.00011 0.0022 .
2
3
10

7. Biu din
Li gii.

1
trong h c s 2 v 3.
10

1
trong h c s 10.
61

1
0.016393442622950819672131147540983606557377049180327868852459
61

10

Ghi ch. C th chng minh rng vng lp ca 1/ 97 cha 96 ch s v ca 1/1913 cha 1912
ch s. Ta vi iu kin no ca s t nhin n 2 th biu din thp phn ca n tun hon v c
vng lp cha n 1 ch s. Trong lp ny cn c cc s n 313,1021,1873, 2137,3221,3313. C
th chng minh cc s nguyn t nhn 10 lm cn nguyn thy th c tnh cht ny.
5. S chun tc v s chun tc tuyt i
Xt s t nhin g 1 . Ta vit s thc x : x x 0.c1 c2 c3 ... g l biu din trong h c s g . Vi
mi ch s c (trong h c s g ) v vi mi s t nhin n ta k hiu l c, n l s cc ch s ca

188 | S chun tc v s chun tc tuyt i. Phn s thp phn trong c s bin thin

l c, n 1

vi g gi tr ca c th s x c gi l chun tc
n
n
g
1234567890
1
trong c s g . V d s
l chun tc c s 10, s
chun tc c s 2 nhng khng
9999999999
10
chun tc c s 3. Nu x l chun tc c s 10 th x / 2 cha chc l chun tc. V d
x 0.1357982046 l chun tc nhng x / 2 0.0678991023 th khng.
dy c1 , c2 ,..., cn bng c . Nu lim

S chun tc trong mi h c s c gi l s chun tc tuyt i. S tn ti ca cc s chun tc


tuyt i c chng minh bi E.Borel [1]. Chng minh ny s dng l thuyt o v ch thun
ty ch ra s tn ti m khng xut mt phng php no xy dng nhng s .
V d hu ch u tin v cc s chun tc tuyt i c a ra nm 1916 (Sierpinski [5],
H.Lebesgue [1]). Nh c chng minh bi Borel th hu ht (theo ngha ca l thuyt o)
cc s thc l s chun tc tuyt i.
Tuy nhin vi hu ht cc s thc quen thuc th ta li khng bit chng c phi l s chun tc
hay khng. V d, ta khng bit cc s 2 , , e l chun tc trong h c s 10 hay khng. V vy
mc d Borel ch ra hu ht cc s thc u l s chun tc tuyt i nhng khng d dng
xy dng nhng s nh th.
D.G.Champernowne [1] vo nm 1933 chng minh rng s a ( mc 4 ta chng minh s
ny l v t) l chun tc c s 10. Champarnowne cng t ra gi thuyt rng s 0.2357111317,
(tt c cc s nguyn t c t lin tip sau du phy thp phn) l chun tc c s 10. Gi
thuyt ny v nh l tng qut hn c chng minh bi A.H.Copeland v P.Erdos [1]. Cc tnh
cht khc ca s chun tc c nghin cu bi W.M. Schmidt [1].
6. Phn s thp phn trong c s bin thin
Xt g1 , g2 ,... l dy v hn cc s t nhin 1 , x l s thc. Xt dy v hn c1 , c2 ,... v x1 , x2 ,...
nh sau
(19)

c0 x , x1 x c0 , c1 g1 x1 , x2 g1 x1 c1 ,

c2 g2 x2 ,..., cn gn xn , xn1 gn xn cn ,

n 1, 2,...

R rng 0 xn v 0 cn gn 1 vi mi n 1, 2,... . .
So snh (19) v thut ton trong mc 3 ta thy ch s c1 l ch s trong h c s g1 , c2 l ch s
trong h c s g 2 v c nh th. T (19) ta c
(20)

x c0

c3
cn
xn1
c1
c
2
...

.
g1 g1 g 2 g1 g 2 g3
g1 g 2 ... g n g1 g 2 ... g n

vi n 1, 2,... ta c g n 2 v 0 xn1 1 , hng t cui cng trong (20) khng m v nh hn

1/ 2n suy ra n tin ti 0 khi n tin ti v cng. Suy ra ta c biu din ca x thnh chui v hn
(21)

x c0

c3
c1
c
2
...
g1 g1 g 2 g1 g 2 g3

Nu g1 g2 ... g ta nhn li biu din nguyn thy ca x trong h c s g . t gn n 1 ,

n 1, 2,... th (21) tr thnh


(22)

x c0

c1 c2 c3
... ,
2! 3! 4!

vi c0 , cn n 1, 2,... l cc s nguyn v

CHNG 7. BIU DIN H C S TY | 189

0 cn n

(23)

n 1, 2,...

R rng nu x l s hu t th thut ton (19) dn ti biu din hu hn ca (22) trong


cn n 1, 2,... tha mn bt ng thc (23). Tuy nhin mi s hu t cng c biu din v hn
thuc dng (22) v

c0

c
cn
c1 c2
... n 1
2! 3!
n ! n 1!

c0

c
c 1
c1
n 1
n2
n3
... n 1 n

...
2!
n ! n 1! n 2 ! n 3! n 4 !

Biu din dng (21) c nghin cu trong E.Strauss [1] v G.Cantor [1], biu din (22) c
nghin cu trong C.Stephanos [1] v G.Faber [1].
Ta hy xem xt thm mt s khai trin khc cc s thc thnh chui v hn.
Xt s thc dng x . K hiu k1 l s t nhin nh nht tha mn k1 x 1 . t k1 x 1 x1 v ta c

x1 0 . Ta tip tc qu trnh ny vi x1 , ngha l ta tm s t nhin nh nht k 2 tha mn k2 x1 1


v t k2 x1 1 x2 v c nh vy. Khai trin ny ca x l chui v hn c dng
1
1
1
x

... , vi kn n 1, 2,... l cc s t nhin v kn1 kn vi n 1, 2,... .


k1 k1 k2 k1 k2 k3
C th chng minh rng mi s thc dng c duy nht mt biu din dng ny v iu kin cn
v s x l v t l lim kn (Sierpinski [3]).
n

1 1
1
1

... .
Khai trin nhn c ca s e l e
1 1 1 1 1 2 1 1 2 3
2
2
a a2 4 1 a 2 a 2 4

Xt a l s t nhin 2 . S dng ng thc
suy ra vi
2
2
2a
2
a1 a , an1 an 2 n 1, 2,... ta c
2

(24)

a a2 4 1
1
1

...
2
a1 a1 a2 a1 a2 a3
n1

Chui ny hi t rt nhanh v bng quy np ta c an 22 , n 1, 2,...


t bit vi a 3 ta c a1 3 , a2 7 , a3 47 , a4 2207 , a5 4870847 v c th.
V vy

3 5 1 1
1
1

...
2
3 3 7 3 7 47 3 7 47 2207

Khai trin ny c gi l chui Pell (E.Lucas [2] trang 331).


2
Nu a chn, a 2b , b 1 th t (24) ta c khai trin b b 1

1
1
1

...
2b1 2b1 2b2 2b1 2b2 2b3

2
vi b1 b v bn1 2bn 1 vi n 1, 2,...

Ta c khai trin thnh tch v hn sau y

b 1 1
1
1
1 1 1 ...
b 1 b1 b2 b3

Mt s trng hp ring ca khai trin ny (v d b 2 , b 3 v mt vi trng hp khc) c


trnh by bi G.Cantor [2] nm 1869.

190 | S chun tc v s chun tc tuyt i. Phn s thp phn trong c s bin thin

By gi t x0 l s v t tha mn 0 x0 1 . Xt a1 l s t nhin ln nht tha mn x0

1
. t
a1

1
x0 . Ta c 0 x1 1 . Ta tip tc qu trnh ny vi x1 v tm c s t nhin ln nht a2
a1
1
1
x1 v tip tc nh vy. Ta thu c dy v hn cc s t nhin
tha mn x1
. t x2
a2
a2
1
a1 , a2 ... v dy v hn cc s v t x1 , x2 ,... tha mn 0 xn 1 vi n 0,1, 2,... v xn xn 1
an
1
1
1
xn 1
xn 1
vi n 1, 2,... Hn na
vi n 1, 2,... V vy
do
an 1
an
an 1
1
1
1
1
1
xn xn1

. Suy ra an1 1 an (an 1) v v th


an1 1
an
an an 1 an an 1
x1

an1 an an 1 vi n 1, 2,... T y bng quy np ta suy ra an 2 22 vi n 1, 2,... S an tng


n

nhanh chng ti v hn theo n . T nh ngha ca an v xn n 1, 2,... suy ra


(25)
Do 0 xn

1
1 1 1
x0 ...
a1 a2 a3
an

n 1

1 xn .
n

1
m lim an1 suy ra lim xn 0 . V vy cng thc (25) cho ta khai trin ca s
n
n
an 1

v t x0 thnh chui hi t rt nhanh


(26)

x0

1 1 1 1
...
a1 a2 a3 a4

Trong an n 1, 2,... l cc s t nhin tha mn bt ng thc


(27)

an1 an an 1 vi n 1, 2,...

Ta chng minh rng mi s v t x0 ,0 x0 1 u biu din c dng (26). C th chng


minh mi s v t nm gia 0 v 1 c ng mt biu din dng ny v s thc x0 c th biu din
dng (26) vi an n 1, 2,... l cc s t nhin tha mn (27) s l s v t (Sierpinski [4]).

CHNG 8
LIN PHN S
1. Lin phn s v s hi t ca chng
Cc lin phn s n (hoc gn hn l lin phn s) c xc nh khi nghin cu thut ton
Euclid mc 9 Chng 1. y ta trnh by mt phng php biu din cc s hu t nh l cc
lin phn s n. Xt lin phn s n dng tng qut
(1)

a0

1| 1|
1|

...
,
| a1 | a2
| an

vi n l s t nhin cho trc, a0 l s thc v a1 , a2 ,..., an l cc s dng. S


(2)

Rk a0

1| 1|
1|

...
,
| a1 | a2
| ak

vi k 1, 2,..., n , c gi l hi t th k ca lin phn s (1). Suy ra hi th th 0 l R0 a0 . T


cng thc (2) suy ra hi t th k Rk ca lin phn s l mt hm k 1 bin a0 , a1 ,..., ak v vi
1
th hi t Rk tr thnh hi t Rk 1 . t
k n nu thay s ak bi s ak
ak 1
(3)

P0 a0 ,

Q0 1,

P1 a0 a1 1,

Q1 a1 ,

Pk Pk 1 ak Pk 2 ,

Qk Qk 1 ak Qk 2

vi k 2,3,..., n . Bng quy np ta chng minh c Pk l hm s ca cc bin a0 , a1 ,..., ak , Qk l


hm s ca a1 , a2 ,..., ak . Hn na Pk v Qk l cc a thc h s nguyn vi cc bin ban u. Kim
tra trc tip ta c

P0 a0
R0 ,
Q0 1

P1 a0 a1 1
1

a0 R1 .
Q1
a1
a1

Ta chng minh vi mi s dng a1 , a2 ,..., an th


(4)

Pk / Qk Rk , k 0,1, 2,..., n,

trn ta thy ng thc ny ng vi k 0 v k 1 . Vi k 2 ng thc cng ng t (3); ta c


a a 1 a2 a0 a 1 R .
Pa P
P2
1 2 0 0 1
0
2
1
Q2 Q1 a2 Q0
a1 a2 1
a1
a2
Gi s (4) ng vi k m , 2 m n . Suy ra vi mi s dng a1 , a2 ,..., am ta c Rm Pm / Qm .
Theo (3) th ng thc
(5)

Rm

Pm1 am Pm2
Qm1 am Qm1

ng vi mi s dng a1 , a2 ,..., am . ng thc (5) vn ng nu am thay bi am

1
am 1

trong c

hai v (v am1 0 ). Khi Rm tr thnh Rm 1 v do trong v phi ca ng thc th cc hm s

Pm1 , Pm2 , Qm1 , Qm2 khng ph thuc am , ta c

192 | Lin phn s v s hi t ca chng

1
Pm1 am
Pm 2
am1
P a Pm2 am1 Pm1 .

Rm1
m 1 m

Qm1 am Qm2 am1 Qm1


1
Qm1 am
Qm 2
am1

Do theo (3) ta c Rm1

Pm am1 Pm1
P
m1 , suy ra (4) ng vi k m 1.
Qm am1 Qm1 Qm1

Theo quy np th n ng vi mi k 0,1, 2,..., n .


Vit k Pk 1 Qk Qk 1 Pk , k 1, 2,..., m . Ta c 1 P0 Q1 Q0 P1 a0 a1 a0 a1 1 1 . Nhng
theo (3) ta c k Pk 1 Qk 1 ak Qk 2 Qk 1 Pk 1 ak Pk 2 Pk 1 Qk 2 Qk 1 Pk 2 k 1 vi

k 2,3,..., n , suy ra k 1 vi k 1, 2,..., n . Ta chng minh c


k

(6)

k Pk 1 Qk Qk 1 Pk 1 vi k 1, 2,..., n .
k

2. Biu din mt s v t thnh lin phn s


Gi s x l mt s v t. t a0 x . V x l s v t nn 0 x a0 1 suy ra x1 1/ x a0 l

s v t 1 . t a1 x1 . R rng x1 l s t nhin v lp lun tng t nh trn suy ra s

x2 1/ ( x1 a1 ) l s v t 1 . Tip tc qu trnh ny ta nhn c dy v hn x1 , x2 ,... cc s v

t ln hn 1 v dy cc s t nhin an xn tha mn xn 1/ xn1 an1 , n 1, 2,...; x0 = x .


Ta c xn 1 an 1
(7)

1
1
1
1
, n 1, 2,... Dy ng thc x a0 , x1 a1 , , xn 1 an 1
suy ra
x1
x2
xn
xn
x a0

1| 1|
1|
1|

...

.
| a1 | a2
| an1 | xn

t
(8)

Rn a0

1| 1|
1|

...
.
| a1 | a2
| an

So snh (7) v (8) ta thy nu an trong (8) c thay bi xn , Rn tr thnh x . Pk v Qk c


P
P a Pn 2
nh ngha theo (3), vi a0 ty v cc s dng a1 , a2 ,..., an th ta c Rn n n 1 n
.
Qn Qn 1 an Qn 2
Hn na v Pn 1 , Pn 2 , Qn 1 v Qn 2 khng ph thuc an , thay an bi xn trong c hai v ng
thc trn ta nhn c
(9)

Pn 1 xn Pn 2
.
Qn 1 xn Qn 2

Cng thc ny ng vi mi s t nhin n 1 v vy thay n bi n 1 , ta c x


ra theo (6) th

1
P x P
P
x Rn n n 1 n 1 n
.
Qn xn 1 Qn 1 Qn Qn xn 1 Qn 1 Qn
n

(10)

T ng thc ny v bt ng thc xn1 an1 suy ra

Pn xn 1 Pn 1
, suy
Qn xn 1 Qn 1

CHNG 8. LIN PHN S | 193

x Rn

(11)

1
1

Qn an1 Qn1 Qn Qn1 Qn

Ta s chng minh Qk k vi mi k 1, 2,... Hin nhin iu ny ng vi k 1 v Q1 a1 l s


t nhin. Nu vi s t nhin k bt ng thc Qk k l ng th theo (3), Qk k 0,1, 2,... l s
t nhin v ta c Qk 1 Qk ak 1 Qk 1 Qk 1 k 1 . V vy theo quy np bt ng thc Qk k
ng vi mi k 1, 2,... Theo (11) ta c x Rn

1
vi n 1, 2,... . V vy x lim Rn .
n
n n 1

Ta ni x c biu din bi lin phn s v hn

x a0

(12)

1| 1| 1|

...
| a1 | a2 | a3

Vy ta chng minh mi s v t x c th biu din nh l mt lin phn s n v hn, biu


din ny nhn c da vo thut ton nu trn. Do an1 xn1 xn1 1 v v vy xn1 an1 1 .
T cng thc (10) suy ra

x Rn
(13)

1
1

Qn xn1 Qn1 Qn Qn an1 1 Qn1 Qn


1

Qn Qn 1 Qn

Nhng do an 2 1 , thay n bi n 1 trong (11) ta nhn c

x Rn1

(14)

1
.
Qn1 Qn Qn1

p dng tnh cht Qn1 Qn an Qn1 Qn cho (13) v (14) suy ra

x Rn1 x Rn , vi mi n 1, 2,3,...

(15)

Ngha l vi mi hi t lin tip ca x , th hi t th hai s cho xp x tt hn hi t trc. Cng


thc (10) chng t x Rn l s dng vi n chn v l s m vi n l. Ngha l cc hi t chn th
nh hn x , trong khi hi t l th ln hn x . Kt hp vi bt ng thc (15) suy ra cc hi t
chn lp thnh dy tng nghim ngt v tin ti x , trong khi dy hi t l gim nghim ngt.
By gi k hiu a0 l s nguyn ty v a1 , a2 ,... l dy v hn cc s t nhin ty . S dng lp
lun trn vi mt s thay i ta kt lun nu cc s Rk xc nh bi (2), th vi mi s t nhin

n, m n , ta c Rm Rn

1
. Suy ra dy v hn Rn n 1, 2,... hi t, ngha l tn ti gii
n n 1

hn x lim Rn . Ta xt cng thc (12). V vy mi lin phn s v hn (12) (vi a1 , a2 ,... l cc s


n

t nhin) u biu din mt s thc. By gi gi s (12) ta vit


(16)

xn an

1|
1|

... vi
| an 1 | an 2

n 0,1, 2,...,

vi x0 x . t
(17)

Rk n an

1|
1|
...
| an 1
| an k

vi k 1, 2,...

194 | Biu din mt s v t thnh lin phn s

Th th

lim Rk n1 xn v lim Rk n1 xn1 .

(18)


Nhng r rng Rk 1 an

n 1

Rk

xn an

(19)

Ta cng c Rk 2 an
n

1
n 1

Rk 1

suy ra theo (18)

1
xn 1

an

vi n 0,1, 2,...

1
an 1

n 2

an 2 nn Rk n2 an

Rk n 2

n
suy ra v lim Rk 2 xn , ta kt lun xn an
k

, nhng v Rk

1
an 1

1
an 1

an 2

. H qu l xn an vi mi n 0,1, 2,... V

an 2

vy xn1 an1 v do xn 1 1 vi n 1, 2,...


Mt khc theo (19) ta c xn an 1 . V vy an xn an 1 vi n 0,1, 2,..., suy ra an xn vi

n 0,1, 2,... T y theo (19) chng t nu (12) l biu din ty ca x thnh lin phn s th
1
1
x1
, xn 1
vi n 1, 2,..., v
x a0
xn an
(20)

an xn vi n 0,1, 2,...

Suy ra mi s v t ch c th biu din mt cch duy nht thnh lin phn s.


By gi ta chng minh mi lin phn s v hn u biu din mt s v t. Tht vy ta gi s s
hu t x l / m (vi (l , m) 1 ) c biu din dng (12).

1
m
l l
l

Ta thy t (12) suy ra (20). Do a0 , x1


. Nhng 1 , suy
l l
l
m m
m
l m
m m
m
1
l

ra l m l m 1 m . H qu l nu x1 l1 / m1 , l1 / m1 l phn s ti gin th m1 m . V
m
m
vy suy ra mu s ca cc s hu t x0 , x1 , x2 ,... lp thnh dy gim nghim ngt, v l. T y ta
kt lun rng s hu t khng th biu din thnh lin phn s v hn.
Ta c nh l sau y
nh l 1. Mi s v t c th biu din duy nht thnh mt lin phn s v hn c dng (12) (vi a0
l s nguyn v a1 , a2 ,... l cc s t nhin cho bi cng thc (20)). Ngc li mi lin phn s v
hn u biu din mt s v t.
Vi s v t vi ly tha bc 2 th cc biu din thnh lin phn s cng c tnh (ta s tho lun
k hn trong mc 4). Rt t s v t c ch r lin phn s ca n. S e l mt v d. Ta c

e 2
v

1| 1| 1| 1| 1| 1|
1| 1| 1|
...
...
|1 | 2 |1 |1 | 4 |1
|1 | 2k |1

e2 1 1| 1| 1| 1|
...
e2 1 |1 | 3 | 5 | 7

CHNG 8. LIN PHN S | 195

Quy lut ca dy a0 , a1 , a2 ,... trong lin phn s biu din e 2 cng c bit. l dy

7, 2, 1, 1, 3, 18, 5, 1, 1, 6, 30,..., 2 3k , 1, 1, 3 3k, 18 12k ,...


Khng c quy lut tng t cho s . Trong trng hp ny G.Lochs [1] tnh cc s ak vi

k 0,1,...968 . S ln nht l a431 20776; tt c cc s 34 u xut hin trong dy v s 1 xut


hin 393 ln. Sau y l danh sch ca 30 s u tin: 3, 7, 15, 1, 292, 1, 1, 1, 2, 1, 3, 1, 14, 2, 1, 1, 2,
2, 2, 2, 1, 84, 2, 1, 1, 15, 3, 13, 1, 4. R.W.Gasper Jr. [1] tnh ak vi k 204103 . S ln nht l

a156381 179136 .
D dng tm iu kin cn v ca s khng nguyn x m lin phn s biu din x bt u vi

1
a1 bng vi s t nhin cho trc m . Tht vy ta c a1 x1
; suy ra a1 m khi v ch
x a0
1
1
x a0 . c bit iu kin s x vi
khi m 1/ x a0 m 1 , ngha l a0
m 1
m
1
1
x . H qu
0 x 1 c tham s u tin trong biu din dng lin phn s bng m l
m 1
m
l x phi thuc khong c di 1/ m 1/ m 1 1/ m m 1 .
T y ta suy ra xc sut phn t u tin bng m l 1/ m m 1 . T vi m 1 th xc

1
1
1
, vi m 2 xc sut l , vi m 3 xc sut ch cn
, v c nh vy. Ta thy xc
2
6
12
sut ny gim dn khi m tin ti v cng. D dng chng minh xc sut m phn t u tin 10
1
l
. y l l do v sao phn t u tin a1 thng l cc ch s nh.
11
sut ny l

Php tnh ton cc xc sut phn t th hai bng mt s t nhin cho trc m l kh hn. Lu
rng xc sut ch s th k trong biu din lin phn s ca mt s thc trong h thp phn
1
bng vi ch s c l
vi mi k v mi ch s c .
10
S dng l thuyt o c th chng minh xc sut gia cc phn t ca biu din mt s v t
thnh lin phn s n c hu hn (hoc khng c) phn t no bng 1 l 0 (xem Hausdorff [1]
trang 426). Tng t xc sut gia cc phn t ch c hu hn s phn bit cng bng 0.
3. Lut xp x tt nht
By gi ta chng minh nh l cho bit tm quan trng ca lin phn s trong vic tm xp x tt
nht ca cc s v t. Cho trc s v t x v biu din ca n di dng lin phn s (12). t
r / s l s hu t xp x x tt hn hi t th n Rn ca x . Ni cch khc gi s
(21)

r
x Rn .
s

Theo (15) ta c x Rn x Rn1 , suy ra theo (21) ta c


(22)

r
x Rn 1
s

Nhng ta bit x nm gia Rn 1 v Rn . V vy cc bt ng thc (21) v (22) chng minh rng

r / s cng nm gia Rn 1 v Rn . V vy ta c

196 | Lut xp x tt nht. Lin phn s biu din cc cn bc hai

(23)

r
Rn 1 Rn 1 Rn .
s

Nhng t (4) v (6) th Rn 1 Rn


(24)

P Q Qn1 Pn
Pn 1 Pn
1

n 1 n

, theo (23) suy ra


Qn 1 Qn
Qn1 Qn
Qn1 Qn

r Qn 1 s Pn 1
1

.
s Qn 1
Qn 1 Qn

S r Qn1 s Pn1 l s nguyn khc 0 v nu ngc li th r / s Rn1 , mu thun vi (22). V vy ta


c r Qn1 s Pn1 1 ; kt hp vi (24) suy ra s Qn . Ta chng minh c
nh l 2. Gi s s hu t r / s , r l s nguyn v s l s t nhin, l mt xp x tt hn hi t th
n Rn n 1 ca s v t x . Th th mu s s l ln hn mu s ca hi t Rn .
nh l ny c gi l lut xp x tt nht.

22
22
v vy t s
xp x
7
7
tt hn mi phn s c mu s 7 . Tng t xp x th 3 ca n l 355/113 suy ra y l xp
x ca tt hn mi xp x hu t khc c mu s 113 .
V d, biu din thnh lin phn s n ta thy hi t th 2 ca n l

4. Lin phn s biu din cc cn bc hai


K hiu D l s t nhin khng phi bnh phng ng ca mt s t nhin. Ta s dng thut
ton trnh by trong mc 2 v nhn c biu din thnh lin phn s ca s v t x D . Ta c
(25)
Suy ra x1

a0 D ,

D a0

D a0
1

D a02
D a0

1
;
x1

D b1
2
, y b1 a0 , c1 D a0
c1

v c1 0 (bi v

a0 D D , v D khng phi bnh phng ng). V vy


(26)

D b12 c1 .

Hn na ta c a1 x1 v x1 a1

x2

1
, suy ra theo (26)
x2

x1 a1

c1

D b1
a1
c1

D a1 c1 b1

D b a c 2a1 b1 c1
2
1

2
1

2
1

D a1 c1 b1

1 a12 c1 2a1 b1

2
Vi b2 a1 c1 b1 v c2 1 a1 c1 2a1 b1 . Vi s t nhin n 1 ta vit

(27)

bn1 an cn bn , cn1 cn1 an2 cn 2an bn .

Ta s chng minh vi n 1 th ng thc sau l ng


(28)

D bn2 cn1 cn

c1 D a1 c1 b1
c1

2
D b1 a1 c1
D a1 c1 b1

D b2
c2

CHNG 8. LIN PHN S | 197

Tht vy

D b22 D a1 c1 b1 D b12 a12 c12 2a1 b1 c1


2

c1 a12 c12 2a1 b1 c1 c1 1 a12 c1 2a1 b1 a1 c2 .

2
Nu vi s t nhin n 1 ta c D bn cn1 cn th theo (27) ta c

D bn21 D an cn bn D bn2 an2 cn2 2an bn cn


2

cn1 cn an2 cn2 2an bn cn cn cn1 an2 cn 2an bn cn cn1 ,


v theo quy np suy ra (28).
Gi thit ca D suy ra theo (28) th cn 0 vi mi n 1, 2,... Ta chng minh

D bn
cn

xn

(29)

vi n 1, 2, ,,,

Ta chng minh (29) ng vi n 1 v n 2 . Gi s n ng vi n 1 .


Khi theo (27) v (28) ta c

xn 1

xn an

cn D an cn bn
cn

2
D bn an cn
D an cn bn

D bn
an
cn

D bn 1
cn 1

v cng thc (29) ng theo quy np. Ta bit c1 l s t nhin, do v b1 a0 D D


v v vy 0 D b1 1, ta c 0
Vy ta thy 0

D b1
1
c1

D b1 / c1 1 v v x1 1 , ta c

D b1 / c1 1 .

D b1
.
c1

By gi ta chng minh cng thc trn ng vi mi s t nhin n , ngha l


(30)

D bn
1
cn

D bn
cn

Cng thc ny ng vi n 1 .
Gi s n ng vi s t nhin ty n . Theo (29) ta c

D bn 1
D bn21

cn 1
cn 1 D bn 1

nn 0

D bn 1
1 , v v (30) ta c
cn 1

cn
D bn 1

D bn 1
xn 1 1 . T (27) v (28) ta c
an 1

cn
D an cn bn

1
,
D bn
an
cn

D bn
an an 1 .
cn

Vy bt ng thc (30) ng theo quy np.


Nu cn 0 vi s t nhin n , th theo (30) ta c
l. V vy cn 0 vi mi n 1, 2,... . H qu l

D bn 0 v

D bn 0 , suy ra 2 D 0 , v

D bn cn D bn , v vy

D bn D bn v

do bn 0 vi n 1, 2,... . H qu l t (30) suy ra bn D v cn D bn 2 D . T y ta

198 | Lut xp x tt nht. Lin phn s biu din cc cn bc hai

suy ra s b cc s t nhin bn v cn l nh hn 2D . V vy gia cc phn t ca dy v hn (29)


vi n 1, 2,... ch c hu hn s phn bit v mi s l nh hn 2D . Suy ra gia cc s
x1 , x2 ,..., x2 D c t nht hai s bng nhau. H qu l tn ti k v s 2D tha mn

xk xk s ;

(31)
T xn 1

1
vi n 1, 2,..., (31) suy ra xk 1 xk s 1 v tng qut hn xn xn s vi n k .
xn xn

V vy dy v hn x1 , x2 ,... v dy a1 , a2,... ( an = xn , n 1, 2,... ) u l dy tun hon.


(32)

xn

D bn
cn

vi

T (29) suy ra nu ta i du ca

n 1, 2,...
D , s xn tr thnh xn

v h qu l ng thc

xn an 1/ xn1 tr thnh xn an 1/ xn 1 , ng thc ny c vit li thnh 1/ xn 1 an xn .


T (32), (30) v 0 xn 1 , ta c
(33)

1
D bn
xn
an

cn
xn 1

vi

n 1, 2,...
1 1

ak s 1 .
xk xk s

Hn na t (31) suy ra xk xk s , ta c theo (33) vi k 1 th ak 1

Do xn an 1/ xn1 v (31) suy ra xk 1 xk s 1 . Lp li lp lun ny vi k 2 , ta nhn c

xk 2 xk s 2 v c nh th. T y suy ra dy x1 , x2 ,... v do dy a1 , a2 ,... u l cc dy tun


hon ngay t phn t u tin (ngha l ti a1 ch khng phi a0 ). Vy
(34)

xn s xn v an s an vi n 1, 2,...

Dy cc cng thc

x1 a1

1
1
1
1
as
, x2 a2 ,, xs as
x2
xs 1
x1
x3

v x1 a1
hoc tng ng

1
1
1
1
as
, x2 a2 ,, xs as
xs1
x1
x2
x3

1
1
1
1
1
1
a1
, a2
, , as
x1
1
x2
1 x3
1



x1
x2
xs

l h qu trc tip ca cng thc

x1 a1
(35)

1|
1| 1|
...

,
| a2
| as | x1

1
1|
1|
1|
as
...

.
x1
| as 1
| a1 1
|
x1

CHNG 8. LIN PHN S | 199

Nhng theo (25),


thc (35) suy ra

D a0 1/ x1 v D a0 1/ x1 , suy ra D a0 1/ x1 . V vy t cng
1| 1|
1| 1|
1|
1|
1|
1|
D a0

...

...

, D as a0
.
| a1 | a2
| as | x1
| as 1 | as 2
| a1 1
|
x1

V x1 1 , 1/ x1 1 , suy ra

as 2 D , a1 as 1 , a2 as 2 ,, as 1 a1 .

(36)

Vy ta thy dy a1 , a2 ,..., as 1 l i xng. Ta c nh l sau y


nh l 3. Nu D l s t nhin khng phi bnh phng ng th trong biu din thnh dng lin
1| 1|

... , dy a1 , a2,... l tun hon. Hn na nu chu k tun hon bt u


phn s D a0
| a1 | a2
ngay t a1 v cha s phn t a1 , a2 ,..., as , th s 2 D, as 2 D v dy a1 , a2 ,..., as 1 i xng.
Biu din lin phn s ca

D a0 ; a1 , a2 ,..., as , du gch ngang trn

D thng c vit l

u ch ra l chu k tun hon. Tuy nhin khng phi ch cc cn bc hai ca cc s t nhin


khng phi bnh phng ng mi c tnh cht nh trn. C th chng minh rng lp cc s v t
dng cng c cc tnh cht trn th trng vi lp cc cn bc hai ca cc s hu t ln hn 1.
V d c th kim tra

26
5
2;3,1,1,3, 4 .
1;3, 2 ,
5
3

13
2;1,1, 4 ,
2

Cc cn bc hai v t khng c tnh cht trn chng hn l

1 13
0;1, 6,1,1,1 ,
4

1
2 19
1 365
1 17
0;1, 2 ,
0;1,3,1, 2,8, 2 ,
1; 2,3, 2 ,
2;1,1, 2 .
2
5
14
2

By gi ta trnh by phng php tnh cc lin phn s ca

D . u tin ta c

B . Nu k l s t nhin v x l s thc th
(37)

x x
k k .

Chng minh. V x x , ta c

x x , do x x . chng minh chiu ngc li ta s


k k
x . Ta c x x 1 , suy ra x k x k v h qu
dng bt ng thc t [t] 1 vi t


k k
k
k
x
x
l cc s c hai v u l s nguyn, x k k 1 . Do x x 1 , suy ra x k k v
k
k
x x
x x
v 1 , suy ra , ta c iu phi chng minh.
k k
k k
k

D bn [ D ] bn a0 bn
Theo b v s dng (29) ta c an xn

, ngha l
c
c
c
n
n
n

200 | Lut xp x tt nht. Lin phn s biu din cc cn bc hai

a b
an 0 n vi n 1, 2,...
cn

(38)

Vy theo (27) v (28), ta nhn c thut ton sau y tnh lin phn s

D.

2
t a0 D , b1 a0 , c1 D a0 v ta ln lt tm cc s an 1 , bn , v cn bng cch s dng

a b
D bn2
an 1 0 n 1 , bn an1 cn1 bn1 , cn
. Ta xt dy a2 , c2 , b3 , c3 , b4 , c4 , v tm ch
c
c
n 1
n 1
s nh nht s m bs 1 b1 v cs 1 c1 ; khi biu din ca

D thnh dng lin phn s chnh l

D a0 ; a1 , a2 ,..., as . Ta nhn c kt qu ny sau mt s hu hn php tnh hu t.


Ghi ch. V chu k khi b ra phn t cui cng m ta bit l bng 2 D l mt dy i xng,

nn tnh chu k ta ch cn tnh cc phn t thuc na u ca dy. Lu ny rt quan trng


1
trong thc hnh. C th chng minh rng nu s l s phn t ca chu k l chn th s s bng
2
1
vi ch s k u tin m bk 1 bk ; nu s l th s 1 l ch s k u tin m ck 1 ck (T.Mulr,
2
xem Perron [1] trang 91).
V d. Tnh biu din ca

a 2 2 vi a l s t nhin 3 . V a 1 a 2 2a 1 a 2 2 a 2
2

2
2
do a0 a 2 2 a 1 nn b1 a0 a 1 , c1 D a0 a 2 a 1 2a 3 suy ra
2

a b 2a 2
1
a1 0 1
1

1 (v a 3 , ta c 2a 3 3 ).
c1 2a 3 2a 3

2
D b22 a 2 a 2
4a 6

2,
Vy b2 a1 c1 b1 2a 3 a 1 a 2 , c2
c1
2a 3
2a 3
2

a b a 1 a 2
3
a2 0 2
a a 2 , suy ra b3 a2 c2 b2 a 2 2 a 2 a 2 ,

2
2

a2
2
a b a 1 a 2
D b32 a 2 a 2
4a 6
c3

2a 3 , a3 0 3
1 , suy ra
c2
2
2
c3 2a 3
2

2
D b42 a 2 a 1
b4 a3 c3 b3 2a 3 a 2 a 1, c4

1,
c3
2a 3
2

a b a 1 a 1
a4 0 4
2a 2 . v vy b5 a4 c4 b4 2a 2 a 1 a 1 b1 ,
1
c4
2
D b52 a 2 a 1
c5

2a 3 c1 . Vy b5 b1 v c5 c1 , suy ra s 4 .
c4
1
2

Vy biu din nhn c l


(39)

a 2 2 a 1;1, a 2,1, 2a 2 vi mi s t nhin a 3 .

Lu rng a1 v a3 (tng qut hn l an , n l) khng ph thuc vo a .

CHNG 8. LIN PHN S | 201

Cng thc (39) khng ng vi a 2 . Tht vy

2 1, 2 . Thay 3,4,5

1
v do
1 2

7 2;1,1,1, 4 , 14 3;1, 2,1, 6 , 23 4;1,3,1,8 . Cc

vo (39) ta nhn c cc cng thc

a 2 1 a 1;1, 2a 2 ,

a 2 1 a; a, 2a vi mi s t nhin a ;

biu din sau y tm c bng cch tng t:

2 1

a 2 a a 1; 2, 2a 2 vi a 2,3,...;

1
a 2 4 a; a 1 ,1,1, a 1 , 2a vi mi s l a 1 ;
2
2

1
1

a 2 4 a 1;1, a 3 , 2, a 3 ,1, 2a 2 vi mi s l a 3 ;
2
2

4a 2 4 2a; a, 4a vi mi s t nhin a ;

na

na

a na; 2n, 2an ,

2 2

na

2a na; n, 2na vi mi s t nhin a, n ;

a na 1;1, 2n 2,1, 2 na 1 vi mi s t nhin a v n 1 .

By gi ta tm tt c cc s t nhin D m biu din lin phn s ca


phn t trong chu k tun hon. T tnh cht (36) ta c

D ch cha ng mt
1
D a; 2a , suy ra D a
,
a D

v do D a 2 1 . Vy cn bc hai D ca mt s t nhin ch c biu din lin phn s vi chu


k tun hon gm 1 phn t khi v ch khi D a 2 1 , vi a l s t nhin.
D dng tm tt c cc s t nhin D m biu din ca

D thnh lin phn s c chu k tun


hon cha ng hai phn t. Tht vy theo (36) ta c D a; b, 2a vi b 2a . V vy

2a
1|
1|
2

v h qu l D a
. Suy ra 2a kb , vi k l s t nhin 1 v
b
|b |a D
b 2a . V vy ta kt lun rng tt c nhng s c cn bc hai biu din thnh lin phn s n
tun hon chu k hai phn t u c dng D a 2 k , vi k l c s ln hn 1 ca 2a.

D a

By gi ta tm tt c cc s t nhin m biu din lin phn s cn bc hai ca n c 3 phn t


trong chu k tun hon. Gi s D l s nh vy. Khi

D a0 ; a1 , a2 , 2a0 . Theo nh l 3 th

dy a1 , a2 i xng. Ta c a1 a2 v hn na a1 2a0 v nu ngc li th chu k ca lin phn


s ca
(40)
Lu
(41)

D ch cha 1 phn t. Suy ra cng thc sau l ng


D a0

1| 1|
1|

| a1 | a1 | a0 D

D l s v t nn ng thc trn tng ng vi


D a02

2a0 a1 1
.
a12 1

y l tt c cc nghim cn tm.
Ta s chng minh rng s t nhin D c dng (41) nu v ch nu a1 l s chn v

202 | Lut xp x tt nht. Lin phn s biu din cc cn bc hai

1
a0 a12 1 k a1 , vi k 1, 2,...
2

(42)

iu kin l ng. Nu a1 l s chn th cng thc (42) ng, do a0 l s t nhin tha mn

2a0 a1 v 2a0 a1 1 2 a12 1 a1 k a12 1 a12 1 2a1 k 1 , suy ra s D c dng (41) l s t

nhin. Mt khc nu vi s t nhin a0 no m a1 2a0 v s D c dng (41) l s t nhin


2
th v 2a0 a1 1 l nn s a1 1 (v cc c s ca n) cng l, do a1 chn v v D c dng (41)

a a / 2 2a1 nguyn, s a 2 1 l c ca a a / 2 2a .
2a0 a1 1
1 0 21
0
1
1
1
2
a1 1
a1 1

l nguyn suy ra

2a , a 1 1 (v a chn); do a a / 2 chia ht cho a 1 v ta c


a / 2 a 1 k vi s nguyn k no . Suy ra cng thc (42) ng. Nhng v 2a a , ta

Nhng

a0

2
1

2
1

2
1

phi c k 0 v do k l s t nhin. iu kin cn c chng minh.


nh l 4. Tt c cc s t nhin D c biu din lin phn s ca
phn t c cho bi cng thc D

k 1, 2,... Khi

a 1 k a / 2
2
1

D c chu k tun hon gm ba

2a1k 1 , vi a1 l s t nhin chn,

D a0 ; a1 , a1 , 2a0 .(Tng qut ca nh l ny i vi trng hp chu k tun

hon gm s lng phn t ty c th xem trong Perron [1] trang 88, Satz 3,17, ti liu dn
trang 89-90, xem thm Drittes Beispiel i vi trng hp k 3 .)
Khng kh chng minh D

a 1 k a / 2
2
1

t nhin D m biu din lin phn s ca

2a1k 1 . c bit t nh l 4 suy ra mi s


2

D c chu k cha 3 phn t v 2 phn t u tin u

bng 2 l cc s D 5k 1 4k 1 , vi k 1, 2,... S dng nh l 4 d dng kim tra trong cc


2

s D 1000 tn ti ng 7 s tha mn D c th biu din thnh lin phn s chu k tun hon
gm 3 phn t. l 41,130, 269,370, 458,697,986.
nh l 5. Nu s l s t nhin 1, a1 , a2 ,..., as 1 l phn i xng trong chu k ca biu din dng
lin phn s ca

D0 , D0 l s t nhin, th tn ti v hn s t nhin D m a1 , a2 ,. . ., as 1 l phn

i xng trong chu k ca lin phn s


Chng minh. Nu

th k ca

D0 a0

D (xem Kraitchik [1] trang 57-58).

1 | 1|
1 |
1
|

. . .

, th nu k hiu Pk / Qk l hi t
| a1 | a2
| as 1 | ao D

1 | 1|
1 |

. . .
ta c
| a1 | a2
| as 1

D0 a0

D Q

Ps 1 a0 D0 Ps 2
Qs 1

suy ra v

s 2

nn Qs 2 Ps 1 v Qs 1D0 a0 Qs 1a0 Qs 2 Ps 1a0 Ps 2 suy ra D0 a02

a0 Qs 2 Ps 1 Ps 2
.
Qs 1

t a a0 Qs 1k vi k 1, 2,3,... th

a Qs 2 Ps 1 Ps 2 a0 Qs 2 Ps 1 Ps 2

Qs 2 Ps 1 k
Qs 1
Qs 1
D0 a02 Qs 2 Ps 1 k
l s t nhin v 2a 1 . Lu rng

D0 l s v t

Qs 2 Ps 1 2Qs 1
P

2 v s 2 1.
Qs 1
Qs 1
Qs 1

CHNG 8. LIN PHN S | 203

Suy ra D a 2

a Qs 2 Ps 1 Ps 2
l s t nhin v D a .

Qs 1

Hn na Qs 2 Ps 1

a D Q

Ps 1 a D Ps 2

D a

Qs 1

D a

nn

s 2

1| 1|
1|
1 |

...

.
| a1 | a2
| as 1 | a D

Suy ra s

D a0 Qs 1k D0 a02 Qs 2 Ps 1 k
2

Qs21k 2 2a0Qs 1 Qs 2 Ps 1 k D0
vi k 1, 2,...., tha mn iu kin nh l. iu phi chng minh.
nh l 6. Vi mi s t nhin s , tn ti v hn s t nhin D m biu din
phn s c chu k cha ng s phn t.

D di dng lin

B . Nu n l s t nhin 1 v a1 , a2 ,..., an l dy i xng cc s t nhin v hn na nu

Pk / Qk k hiu hi t th k ca lin phn s

1| 1|
1|

...
th Pn Qn1 .
| a1 | a2
| an

Chng minh b . Theo (3) th Qn Qn1an Qn2 , Qn1 Qn2 an1 Qn3 ,..., Q2 a2 a1 1, Q1 a1.
Qn
1|
1|
1| 1|
an

...

V vy
. Nhng v dy a1 , a2 ,..., an i xng suy ra
Qn 1
| an 1 | an 2
| a2 | a1

Qn
1|
1|
a1
...
Qn1
| a2
| an
chng minh.

v do

Qn1
1| 1|
1 | Pn

...

suy ra Pn Qn1 . iu phi


Qn
| a1 | a2
| an Qn

Ghi ch. Nu k hiu 1 l Q0 th b cng ng vi n 1 .


Chng minh nh l 6. Cho trc hai s t nhin k , n v t a1 , a2 ,..., an l dy cc phn t u

Pn Qn1 .
T b ta c
1|
1|
1|
1
|
yt Qn t k

...

.
| 2k | 2 k
| 2k | Qn t k y

bng

2k .

V Qn1 Pn , ta c yt Qn t k

Do Qn yt2 Qn t k

nguyn

t 0

hiu

Pn Qn t k yt Pn 1
.
Qn Qn t k yt Pn

2P Q t k P
n

Vi

n 1

c bit vI t 0 ta nhn c Qn y02 k 2 2Pn k Pn1 .

Mt khc, theo nh ngha ca yt , y0 k ; 2k k 2 1. H qu l Qn 2Pn k Pn1 v do

yt2 Qn t k 2Pn t 1, suy ra yt


2

Qn t k

2 Pn t 1 . V vy vi s t nhin k v s

nguyn t 0 th biu din lin phn s ca cn bc hai ca D Qn t k 2Pn t 1 c chu k


2

cha ng n 1 phn t v mi phn t u bng 2k . Lu chu k k ; 2k k 2 1 ch c duy


nht mt phn t. Suy ra nh l 6 c chng minh.
V d vi k 1 v n 1, 2,3, 4,5,6 ta c vi t 0,1, 2,..., tng ng (xem Kraitchik [1] trang 57)

204 | Lut xp x tt nht. Lin phn s biu din cc cn bc hai

V vy vi t 1 , ta c

D dng chng minh vi mi s n c dng 3k hoc 3k 1 tn ti v hn s t nhin D tha mn


biu din ca D thnh lin phn s c chu k cha n 1 phn t m n phn t u tin u
bng 1 (Sierpinski [26] trang 300).

89t 44

V d ta c vi t 1, 2,... th
V vy vi t 1 th

110t 54 89t 44;1,1,1,1,1,1,1,1,1,1,178t 88 .

2081 45;1,1,1,1,1,1,1,1,1,1,90 .

W.Patz [1] lp bng biu din thnh lin phn s ca tt c cc s v t


bng cho thy trong 100 s t nhin u tin th chu k di nht l

D , vi D 10000 . T

94 9;1, 2,3,1,1,5,1,8,1,5,1,1,3, 2,1,18 ,


Chu k ny cha 16 phn t.
S

919 c chu k 60 phn t

(Kraitchik [1] trang 57 tnh ton sai s thnh 62).


S

991 c chu k 60 phn t

D thy 1000 31;1,1,1,1,1,6, 2, 2,15, 2, 2,15, 2, 2,6,1,1,1,1,1,62 .


Mi nghim v t ca a thc bc hai vi h s nguyn gi l s v t bc hai. Nu x l s thc
2
tha mn phng trnh Ax Bx C 0 , vi A, B, C l cc s nguyn th ta bit

D B2 4 AC 0 v D khng l bnh phng ng. Ta c x B D / 2 A.


nh l Lagrange sau y c chng minh vi mt s thay i t chng minh nh l 3: biu din
lin phn s ca mt s v t bc hai thc l tun hon. Ngc li mi lin phn s tun hon biu
din mt s v t bc hai thc (Lagrange, Kratchik [1] trang 9-13).

CHNG 8. LIN PHN S | 205

V d. Ta c

1
2

5 1 1; 1 . Kt qu ny suy ra t

1
2

1
5 1 1 1/
2

5 1 .

Bi tp. 1. Chng minh rng mi s thc l tng ca hai s m mi s u c biu din lin phn
s vi thng s u tin = 1.
Chng minh. Trong mc 3 ta thy phn t u tin ca lin phn s = 1 khi v ch khi t t

1
(v
2

1| 1|
1
1
). Vi s thc x ta t u x x , v x 1 u .
|1 |1
2
2
1
1
Khi x u v v v 0 x x 1 ta c u 1 suy ra v x 1 , v do d v v u . T
2
2
cc bt ng thc ny ta suy ra iu phi chng minh.
nu t [t] 1/ 2 th suy ra t [t]

Ghi ch. M.Hall, Jr. [1] chng minh rng mi s thc l tng ca hai s m mi s u c biu
din lin phn s vi phn t u tin khng ln hn 4. Tuy nhin ngay c khi x tnh c
100
chnh xc ti 1/10 th mt cch tng qut ta vn cha tm c phn t u tin trong biu din
100
100
lin phn s ca n. Tht vy v ta mi ch bit 0 x 1/10 nn ta ch c th kt lun 1/ x 10
100
ngha l phn t u tin l 10 .
2. Chng minh rng khng tn ti s t nhin D m D c th biu din thnh lin phn s vi
chu k tun hon cha 6 phn t m 5 phn t u tin bng 1.
Chng minh. Gi s
lin phn s

D a0

1| 1|
... . Ta c
|1 |1

1| 1| 1| 1| 1|
1 |

. K hiu Pn / Qn l hi t th n ca
|1 |1 |1 |1 |1 | a0 D
D a0

Q a

DQ

P5 a0 D P4
5

8a

D5

5 a0 D 3
0

nn D a02

10a8 3
,
8

v l v s l 10a0 3 khng chia ht cho 8.


3. K hiu f s l s t nhin nh nht D m chu k ca lin phn s

D cha s phn t. Tnh

gi tr ca f s vi s 10
Li gii.

5. S dng lin phn s

D gii cc phng trnh x 2 Dy 2 1 v x 2 Dy 2 1

Gi s D l s t nhin khng phi bnh phng ng. t


hi t th k . Ta c

D a0

D a0 ; a1 , a2 ,...., as v Pk / Qk l

1| 1|
1|
1
|

...

v vy
| a1 | a
| as 1 | as a0 D

D Ps 1 as a0 D Ps 2Qs 1 as a0 D Qs 2

Pks 1 ( D a0 ) Pks 2
vi k 1, 2,3,...., m D l s v t
Qks 1 ( D a0 ) Qks 2
v DQks 1 a0 Pks 1 Pks 2 . Nhn ng thc th nht vi Pks 1 v ng

v tng qut hn v a0 as a0 ,
nn a0Qks 1 Pks 1 Qks 2

thc th hai vi Qks 1 sau cng li v s dng (6) ta suy ra

206 | S dng lin phn s

D gii cc phng trnh x 2 Dy 2 1 v x 2 Dy 2 1

Pks21 DQks2 1 Qks 2 Pks 1 Pks 2Qks 1 1 .


ks

Nu s l th suy ra
(43)

Pks21 DQks2 1 bng 1 vi k l v bng 1 vi k chn

Nu s chn th
(44)

Pks21 DQks2 1 1 vi mi k 1, 2,3,...

V vy ta thy cc hi t ca lin phn s ca

D l nghim t nhin ca x 2 Dy 2 1 .

Ta chng minh iu ngc li cng ng, ngha l mi nghim t nhin ca phng trnh trn cho
ta t s v mu s ca hi t ca lin phn s D .
Tht vy gi s t v u l nghim t nhin ca x 2 Dy 2 1 . Ta c t u . t
(45)

t
1| 1|
1 |

...
u | b1 | b2
| bk 1

l biu din ca s t / u thnh lin phn s ta c k chn. Biu din nh vy l tn ti v nu k 1


1|
chn th vi bk 1 1 s bk 1 1 c th thay th v tr ca bk 1 , v vi bk 1 1 th s bk 2 1 c
|1
1 |
th thay th v tr ca bk 2
.
| bk 1
K hiu t '/ u ' l hi t cui cng ca lin phn s (45). Th th
(45a)

t'
1| 1|
1 |
b0

...
u'
| b1 | b2
| bk 2

Ta c u ' u . Vi k 2 ta c t '/ u ' b0 ) v k chn s dng (6) ta c tu ' ut ' 1 .


2
2
By gi tr c hai v ca dng thc cui cng cho t Du 1, ta nhn c

(46)

t u ' t u t ' Du

Theo (45) ta c 0 t / u b0 1 , suy ra


(47)

0 t b0u u

2
2
Do t v u nguyn t cng nhau (v t Du 1) suy ra vi s nguyn l th cc ng thc sau l
ng

(48)

u ' t lu,

t ' Du lt

V vy
(49)

u ' t b0u l b0 u

T cc bt ng thc 0 u ' u v (47) ta suy ra u ' t b0u u , v theo (49) ta c l b0 0 , do


l b0 , v vy theo (48) th u ' t b0u,
(50)

t ' Du b0t , v h qu l

t D Du
u b D u ' t u D
t b0 D t '
0

D;

CHNG 8. LIN PHN S | 207

Nhng theo (45) v (45 ) th v tri ca (50) tr thnh b0


theo (50) th ta c lin phn s

1| 1|
1 |
1 |

...

; do
b1 | b2
| bk 1 | b0 D

D b0 ; b1 , b2 ,..., bk 1 , 2b0 , hi t th k 1 ca n l (45).

trn ta ch ra s k bng vi s cc phn t trong chu k ca lin phn s D . Chu k ny


khng nht thit l nh nht. K hiu s l chu k nh nht ca lin phn s. R rng s | k v do

k sn , vi n l s t nhin. Vi mi nghim t nhin ca phng trnh x 2 Dy 2 1 l t v u , th


phn s t / u l hi t ca lin phn s D ; ng hn n l hi t th ns 1 vi s l s cc phn
t ca chu k ngn nht ca lin phn s v n l s t nhin. trn ta chng minh (cng thc
(44)) nu s l s chn th mi hi t th ns 1 n 1, 2,... u l nghim t nhin ca
x 2 Dy 2 1 . Ta c nh l sau y
nh l 7. Nu chu k ca lin phn s D cha s chn phn t th t s v mu s ca hi t th
ns 1 vi n 1, 2,..., to thnh nghim t nhin ca phng trnh x 2 Dy 2 1 . Hn na tt c cc
nghim u c th nhn c theo cch ny.
T y ta thy nghim nh nht ca phng trnh cho bi hi t th s 1 . Nu s l th cng thc
(43) chng t t s v mu s ca hi t th ns 1 to thnh nghim ca x 2 Dy 2 1 ch khi n
chn. V vy ta c
nh l 8. Nu chu k ca lin phn s D cha l phn t th t s v mu s ca hi t 2ns 1 ,
n 1, 2,. . . to thnh nghim t nhin ca x 2 Dy 2 1 . Hn na mi nghim l nhn c theo
cch ny.
V vy nghim nh nht c cho bi hi t th 2ns 1 . Biu din ca 991 thnh lin phn s
c trnh by trn. Ta thy chu k ca n gm 60 phn t. S dng biu din ny v nh l 7
c th tnh c nghim t nhin nh nht ca phng trnh x 2 991y 2 1 (Chng 2 mc 15).
Nghim ny l x c 30 ch s v y c 29 ch s. By gi ta chuyn sang phng trnh
(51)

x 2 Dy 2 1

2
Gi s D a 1 , vi a l s t nhin 1 . Nh ta bit ta c

a 2 1 a; 2a . V vy nu

Pk / Qk l hi t th k ca a; 2a , th theo (43), vi s 1 , ta c
Pk21 DQk21 1,

k 1,3,5,...

V vy nghim t nhin nh nht ca phng trnh l s t P0 a, u Q0 1 . Vi cc nghim t


nhin khc ca (51) l t , u ta c u 1 . Nu D a 2 1, a l s t nhin th nu t v u l nghim t
2
2
nhin ca phng trnh (51) th ta c u 1 v nu u =1 ta c t D 1 suy ra D t 1 , mu
thun vi gi thuyt ca D . V vy ta c th gi s t v u l nghim t nhin ca (51) vi u 1 .
K hiu (45) l lin phn s ca t / u , th k l. Ta xt t '/ u ' theo (45 ). V k l ta c tu ' ut ' 1 ,
2
2
suy ra v t Du 1, ta nhn li (46). Lp lun tng t c s dng chng t (45) l hi t
th k 1 ca lin phn s D v k sn , trong s l s cc phn t (nh nht) trong chu k ca
lin phn s D v n l s t nhin. Nhng nu s chn th theo (44) ta thy khng c hi t th
sn 1 no cho ta nghim ca (51). Nu ngc li l s l th theo (43) hi t th sn 1 cho nghim
ca (51) vi n l. Vy ta c nh l

nh l 9. Nu chu k ca lin phn s D c s phn t v s chn th phng trnh (51) khng c


nghim t nhin. Nu s l th t s v mu s ca cc hi t th ((2n 1) s 1) , n 1, 2,... to thnh
nghim t nhin ca (51). Hn na tt c cc nghim l nhn c theo cch ny.

208 | S dng lin phn s

D gii cc phng trnh x 2 Dy 2 1 v x 2 Dy 2 1

V d. 1. t D 2 . V D 1; 2 , ta c s 1 v do theo nh l 7 suy ra t s v mu s ca hi
t th 2n 1, n 1, 2,... to thnh nghim t nhin ca x 2 2 y 2 1 v hn na mi nghim l
1 3
nhn c theo cch ny. Hi t u tin l 1 cho nghim t nhin nh nht x 3, y 2 .
2 2
Theo nh l 9 th t s v mu s ca mi hi t th 2n 2 . n 1, 2,..., to thnh nghim t
nhin ca phng trnh x 2 2 y 2 1 v mi nghim nhn c theo cch ny. Hi t th 0 l
1/1 cho nghim t nhin nh nht ca phng trnh.
2. t D 3 . Khi

3 1;1, 2 . Ta c s 2, v theo nh l 7 th t s v mu s ca hi t th

2n 1, n 1, 2,... to thnh nghim ca phng trnh x 2 3 y 2 1 v tt c cc nghim nhn dc


1 3
theo cch ny. Nghim t nhin nh nht c cho bi hi t u tin, ngha l 1 , suy ra
1 2
2
2
x 3, y 2 . Tuy nhin theo nh l 9 th phng trnh x 3 y 1 khng c nghim t nhin.

3. t D 13 . Khi 13 3;1,1,1,1, 6 . Ta c s 5 v theo nh l 8 th t s v mu s ca mi


hi t th 10n 1 , n 1, 2,..., cho nghim ca phng trnh x 2 13 y 2 1 , v tt c cc nghim
nhn c theo cch ny. Nghim t nhin nh nht cho bi hi t th 9, ngha l s
1| 1| 1| 1| 1| 1| 1| 1| 1| 649
3
suy ra x 649, y 180 . T nh l 9 suy ra t s v
|1 |1 |1 |1 | 6 |1 |1 |1 |1 180
mu s ca mi hi t th 10n 6 , n 1, 2,..., u l nghim ca phng trnh x 2 13 y 2 1 v
tt c cc nghim nhn c theo cch ny. Nghim t nhin nh nht cho bi hi t th 4, ngha
1| 1| 1| 1| 18
l s 3
suy ra x 18, y 5 .
|1 |1 |1 |1 5
Khng kh tm tt c cc nghim t nhin nh nht ca phng trnh x 2 Dy 2 1 bng cch
s dng biu din lin phn s ca D vi D 100 . Bng cc nghim vi D 1003 c cho bi
Legendre [1]. y l bng cc nghim t nhin nh nht ca phng trnh x 2 Dy 2 1 vi D 40

T nh l 8 suy ra phng trnh x 2 Dy 2 1 l c nghim t nhin vi D 100 ch khi D


nhn cc gi tr 2,5,10,13,17, 26, 29,37, 41,50,53,58,61,65, 73,74,82,85,89,97.
6. Lin phn s dng phc
Ta nghin cu cc phn s c dng
(52)

a0

b |
b1 | b2 |

... n
| a1 | a2
| an

CHNG 8. LIN PHN S | 209

vi a0 , a1 ,..., an , b1 , b2 ,..., bn cc cc s thc hoc phc ty . Ta cn c iu kin sau cc biu


thc l c ngha

an 0, an 1

bn
b | b
b
b
0, an 2 n2 n 0, . . ., a1 2 ... n 0 .
an
| an 2 an
a2
an

Ta thy mt s (hoc tt c) cc s a1 , a2 ,..., an1 c th bng 0.


V d lin phn s

1| 1|
1| 1|
...
= 2.
|0 |0
|0 |2

C th chng minh lin phn s


(53)

Rn a0

b |
b1 | b2 |

... n
| a1 | a2
| an

c nh ngha tt nu cc s Pk v Qk k 0,1,..., n c cho bi cng thc truy hi

P0 a0 , Q0 1, P1 a0 a1 b1 , Q1 a1 ,
Pk Pk 1ak Pk 2bk , Qk Qk 1ak Qk 2bk , k 2,3,..., n,
Th th

Rn

Pn
v Pk 1Qk Qk 1Pk (1)k bb
1 2 ...bk vi k 1, 2,..., n .
Qn

Lu rng nu lin phn s (52) c nh ngha tt th cc hi t ring ca n c th khng c


1| 1| 1|
1| 1|
tnh cht . V d lin phn s
khng xc nh.
= 2 nhng cc tng ring
|1 | 1 |1
|1 | 1
Nu cc dy a0 , a1 , a2 ,... v b1 , b2 ,... u l dy v hn v nu dy cc s (53) hi t ti gii hn l
x , th x c gi l gii hn ca lin phn s v hn
(54)

x a0

b1 | b2 |

...
| a1 | a2

Cc lin phn s nh vy c trnh by bi cc cng thc ca Brouncker cho / 4 vo nm 1655.


Ta c

1| 12 | 32 | 52 |

...
|1 | 2 | 2 | 2

log 2

1| 12 | 22 | 32 |

...
|1 | 1 | 1 | 1

Cng thc th nht c suy ra t cng thc

1 1 1
(1)n1 1| 12 | 32 | 52 |
(2n 3) 2 |
...

...
1 3 5
2n 1 |1 | 2 | 2 | 2
|2
v cng thc quen thuc ca Leibniz cho / 4 .
Cng thc sau cho bi cng thc

1
1 1 1
...
1 2 3
n

n 1

n 1 |
1| 12 | 22 | 32 |

...
|1 | 1 | 1 | 1
| 1
2

vi mi s t nhin n (Sierpinski [7] phn 2 trang 140).


By gi ta chuyn sang nghin cu mt s trng hp c bit ca lin phn s c dng (54).

210 | Lin phn s dng phc

Vi s thc x0 th k hiu G x0 l s nguyn nh nht x0 . Ta c x0 G x0 x0 1 , do

0 G x0 x0 1 v h qu l x1
thay cho x0

G x0 x0

1 . V vy G x1 2 . Lp li th tc ny vi x1

v c nh vy. V vy nu xn

G xn 2, n 1, 2,... Hn na x0 G x0

G xn 1 xn 1

vi n 1, 2,..., ta c xn 1 v

1 |
1 |
1 | 1|

...

.
| G x1 | G x2
| G xn1 | xn

T y suy ra lin phn s v hn ca x0

x0 G x0

(55)

1 |
1 |
1 |

...
| G x1 | G x2 | G x2

V vy ta thy mi s thc x u c th biu din thnh lin phn s v hn

x a0

1| 1| 1|

... ,
| a1 | a2 | a3

vi a0 l s nguyn v an l cc s t nhin 2 . C th chng minh rng mi s thc c ng mt


1| 1| 1|
biu din nh vy. c bit ta c 1 2 ...
|2 |2 |2
Vi mi s hu t th trong biu din dng (55) ca chng ta c G xn 2 vi n ln.

1|
1 |

cho ta biu din ca


|2 |2 2
1| 1| 1| 1|
2 2 ... .
|2 |4 |2 |4

Cng thc
phn t

2 2

2 thnh lin phn s vi chu k gm 2

Mt dng biu din khc ca s thc x thnh lin phn s nhn c vi a0 l s nguyn gn x
nht v x1 l s cho bi cng thc x a0 1/ x1 vi cc du v c chn ty thuc x a0
hay x a0 . Vi x1 xc nh ta nh ngha a1 v x2 tng t nh vy v c nh th (Hurwitz [1]).
Biu din dng ny ca 2 trng vi biu din lin phn s n 2 . Vi 3 ta c
1| 1|
3 2 ... ngha l biu din vn c dng (55). Vi 5 ta nhn li biu din lin phn
|4 |4
1| 1| 1|
s n ca 5 . Vi 7 ta c 7 3 ... , ngha l c dng (55). Nhng vi 13 ta c
|3 |6 |3

13 4

1| 1| 1| 1| 1| 1|
. . ., biu din ny khng c dng (55) v cng khng
|3 |2 |7 |3 |2 |7

phi lin phn s n.


kt thc chng ny ta xt lin phn s

a1
a0

a2 ...
a3
b2
|a |a
a
a
a0 1 2 ... a0 1 2
... .
b1
b1 | b2 |
b1 b1 b2 b1 b2 b3

K hiu b1 , b2 ,... l dy v hn cc s t nhin m trong c v hn phn t khc 1. K hiu x0 l


s thc v t a0 x0 , a1 b1 x0 a0 . R rng a1 l s nguyn b1 . t x1 b1 x0 a0 a1 .

CHNG 8. LIN PHN S | 211

Ta c 0 x1 1 . Trong trng hp tng qut gi s vi s t nhin n 1 ta c s xn 1 ; khi t

an bn xn1 v xn bn xn1 an . V vy dy a1 , a2 ,... c nh ngha bng quy np v cc phn t

ca n l cc s nguyn khng m tha mn an bn , cng vy dy x1 , x2 ,... l dy cc s thc vI

0 xn 1 , vi mi n 1, 2,... .
Ta nhn c
(56)

x0 a0

an
xn
a1
a
2 ...

b1 b1 b2
b1 b2 ...bn b1 b2 ...bn

Theo gi thit cc s b1 , b2 ,... u l s t nhin v c v hn s trong cc s l 2 . V vy tch

b1 b2 ...bn tng ti v hn theo n . Hn na v 0 xn 1 , cng thc (56) cho biu din ca x0 thnh
chui v hn
(57)

x0 a0

a3
a1 a2

...,
b1 b1 b2 b1 b2 b3

Ngha l lin phn s v hn


(58)

x0 a0

| a1 | a2

...
b1 | b2 |

T y ta c nh l: vi mi dy v hn cc s t nhin b1 , b2 ,... m v hn cc phn t trong l


khc 1 th mi s thc x0 u c th biu din thnh lin phn s c dng (58) vi

a0 x0 , an n 1, 2,... l cc s nguyn 0 an bn vi n 1, 2,... . D thy biu din dng (57)


.

trng vi biu din thp phn vi c s bin thin trong Chng 7 mc 6.

212 | Lin phn s dng phc

CHNG 9
K HIU LEGENDRE V K HIU JACOBI

D
v cc tnh cht
p

1. K hiu Legendre

Gi s p l mt s nguyn t l v D l s nguyn khng chia ht cho p , k hiu Legendre

nhn gi tr bng 1 nu D l thng d bc hai modulo p v nhn gi tr bng -1 trong trng hp


ngc li. Theo nh l 4 Chng 5 ta c
1
p 1
D
2

D
mod p

p

(1)

D
p 1 /2
chia p d 1. Theo nh l 15 Chng 6 ta c
bng 1 nu v ch nu D
p

Suy ra

D
ind D
,
1
p

(2)

trong ch s
c ly tng ng theo cn nguyn thy ca p . Nu D v D ' l cc s t
nhin khng chia ht cho p th t (1) suy ra cc tnh cht sau

D D'
I. Nu D D '(mod p) th

p p
T (2) suy ra nu D v D ' l cc s nguyn khng chia ht cho p th

DD '
D D '
idcDD '
v (1)idc Didc D '

(1)
p
p p

(3)

Nhng theo tnh cht II ca cc ch s (Chng 6 mc 8) ta c ind DD ' ind D ind D ' mod p 1 .
Do p l s nguyn t l ta c ind DD ' ind D ind D ' mod 2 , suy ra 1

ind DD '

ind D ind D '

DD ' D D '
. Vy ta chng minh
p p p

T theo (3) ta c

DD ' D D '
.
p p p

II. Nu D v D ' l cc s nguyn khng chia ht cho p th

D
By gi ta chng minh (Sierpinski [2]) nu l s thc nh ngha bi s nguyn t l p v
p
mi s nguyn D khng chia ht cho p th n khc 0 vi t nht mt gi tr ca D v khc 1 vi t
nht mt gi tr ca D . Hn na n cng tha mn cc iu kin
D D '
1. Nu D D '(mod p) th
p p
DD ' D D '
2.
vi mi D v D ' khng chia ht cho p
p p p
Khi vi mi s nguyn D khng chia ht cho p ta c

214 | K hiu Legendre v cc tnh cht

D D
.
p p

(4)

Gi s g l cn nguyn thy ca p .
ind D
Vi mi s nguyn D khng chia ht cho p ta c D g
mod p .

D
Theo cc tnh cht 1,2 ca ta c
p
ind D

D g ind D g


p p p

(5)

g
g
p 1
p 1
t a . Do g 1 mod p , theo 1,2 ta c a
p
p

p 1

g p 1 1

, nhng theo 2
p p

1
1
1
1 1
th suy ra 0 hoc 1 . Ta khng th c 0 v nu nh vy th theo
p
p
p
p p
D D 1
D
2 (vi D ' 1 ) ta s c 0 mu thun vi gi thit khng ng nht 0
p p p
p

1
p

g
p

p 1
(nu D khng chia ht cho p ). V vy 1 v do a 1 . Nhng a l s thc v

phng trnh x p 1 1, p l, c t nht hai nghim l 1 v 1 . Suy ra a 1 hoc a 1 . Nu a 1

D
1 , mu thun vi gi thit
p

th theo (5) vi mi s nguyn D khng chia ht cho p ta c

D
khng ng nht 1 ( D khng chia ht cho p ). T ta phi c a 1 suy ra theo (5) th
p
D
D D
ind D
. Bi vy t (2) ta c . nh l c chng minh.
1
p
p p
D
khng
p

T ta thy mi tnh cht ca k hiu Legendre c th nhn c t I, II v nhn xt


ng nht 1 hoc 0 vi mi s nguyn t l p .
T cng thc (1) suy ra

1
p 1 /2
1
p

III.

c thm mt s tnh cht ca k hiu Legendre ta chng minh kt qu sau

1 vi l s cc d modulo p xut hin trong dy


p

B Gauss.
(6)

D, 2 D,3D,...,

1
p 1 D
2

m khng vt qu p / 2 .
Chng minh. Vi k 1, 2,..., p 1 / 2 , k hiu rk l s d ca kD khi chia cho p . t

rk p / 2 v

rk nu

p rk nu rk p / 2 . Khng xy ra trng hp rr p / 2 v p l s nguyn t l.

CHNG 9. K HIU LEGENDRE V K HIU JACOBI | 215

Do D khng chia ht cho p v trong dy (6) th cc h s ca D l s t nhin p 1 / 2 nn


cc tng v hiu ca cc phn t trong dy (6) u khng chia ht cho p . V vy d thy tng v
hiu ca cc phn t ca dy sau cng th
(7)

,...,

Nhng theo nh ngha ca


nn 2

p, ngha l 2

p1
2

th chng u ln hn 0 v nh hn p 1 / 2 (v

p 1 hoc

p rk v rk p / 2 li suy ra

ra cc phn t ca (7) chnh l cc s 1, 2,..., p 1 / 2 . V vy


(8)

,...,

p 1
2

rk p / 2

p / 2 ). T ta suy

p 1
p 1
p 1

!
! D mod p
2
2

p 1
Cc ng d thc c suy ra t nh l Fermat nh D 1 mod p .

K hiu k bng 0 hoc 1 nu nh rk p / 2 hoc rk p / 2 tng ng.


Theo nh ngha ca

ta c

1 k rk mod p

(9)

Nhng theo nh ngha ca rk , rk kD mod p . V vy theo (9) ta c


(10)

p 1
2

1 1

2 ... p1/2

mod p

p 1 p 1 /2
Cc cng thc (8) v (9) kt hp vi vic
khng chia ht cho p suy ra
!D
2
(11)

p 1
2

1 1

2 ... p1/2

mod p

p
nhn
2
c khi chia cc s trong dy (6) ln lt cho p . Mt khc, v tri ca (11) ng d vi
Nhng theo nh ngha ca k th s 1 2 ... p 1/2 chnh l s cc phn d

D
D

mod D . T (11) tr thnh ng d thc 1 mod p .


p
p

D
D

1 ta ch cn bng 1 hoc -1 v p l s nguyn t l


p
p

By gi chng minh

3 . B c chng minh.

2 kD / p

Cc s k xc nh nh trn tha mn 1 k 1

. Tht vy, nu rk p / 2 th k 0 v

mt khc theo nh ngha ca rk suy ra vi s nguyn t k ta c kD ptk rk . T

2kD / p 2tk 2rk / p v do 0 2rk p, 2kD / p 2tk ta c 1 k 1

2 kD / p

. Nu rk p / 2 th

1 2rk / p 2 (v rk p ) suy ra 2rk / p 1 v 2kD / p 2tk 1 . Nu rk p / 2 ta c k 1 suy


2 kD / p

ra cng thc 1 k 1

1 2 ... p1/2

. V cng thc trn ng vi mi k 1, 2,..., p 1 / 2 ta c


p1/2

1
k 1

2 kD / p

. Do b Gauss dn ti

216 | K hiu Legendre v cc tnh cht


p1/2

D
H qu. 1
k 1
p

2 kD / p

c bit nu D 2 . T h qu ta c
p 1 /2
2

1 vi 4k / p
k 1
p

(12)

Nu 1 k p / 4 th 0 4k / p 1 v do 4k / p 0 . ng thc k p / 4 khng th c v p l.
Vi p / 4 k p 1 / 2 ta c 1 4k / p 2 p 1 / p 2 ; h qu l 4k / p 1. T y ta suy ra
trong cc hng t ca trong cng thc (12) c p 1 2 p / 4 hng t bng 1, cc hng t cn
li bng 0. H qu l p 1 / 2 p / 4 . Nhng vi p l ta c

p 1 p p2 1

mod 2 .
2
8
4

Tht vy mi s p l u c dng 8k 1,8k 3,8k 5,8k 7 , vi k l s t nhin.


Vit f ( p)

p 1 p
p2 1
v g ( p)
th ta c
2
8
4
f 8k 1 4k 2k 2k ,
f 8k 3 4 k 1 2 k 2 k 1
f 8k 5 4k 2 2k 1 2k 1,
f 8k 7 4k 3 2k 1 2k 2,
g 8k 1 k 8k 2 ,
g 8k 3 4k 1 2k 1 ,
g 8k 5 2k 1 4k 3 ,
g 8k 7 4k 3 2k 2 ,

Suy ra trong mi trng hp f p g p mod 2 .

p2 1
mod 2 v v vy theo (12) suy ra
8
2
2
IV. (1)( p 1)/8
p

Vy 2 l thng d bc hai ca mi s nguyn t p c dng 8k 1 v khng l thng d bc hai ca


mi s nguyn t p c dng 8k 3 (vi k l s nguyn).
p dng tnh cht IV chng minh nh l sau
nh l 1. Tn ti v hn s nguyn t c dng 8k 1 , vi k 1, 2,...
Chng minh. Xt n l s t nhin 1 . S N 2 n ! 1 l ln hn 1 v c t nht mt c s
2

nguyn t l p khng c dng 8k 1 . V nu ngc li th tt c cc c s nguyn t ca N u


c dng 8k 1 do s N t n cng c dng , nhng N c dng 8k 1 , mu thun.
Ta c p | N , ngha l 2 n! 12 mod p suy ra 2 n ! l thng d bc hai theo modulo p .
2

CHNG 9. K HIU LEGENDRE V K HIU JACOBI | 217

2 n !2
2 n !2
V vy
1 , suy ra theo tnh cht II ta c

p
p

theo IV th p c dng 8k 1 . Nhng t nh ngha ca

2 n!
2
nn
p p
p
p suy ra n c dng

2
1 v
p
8k 1 . M

p | N 2 n ! 1 suy ra p n . T ta thy vi mi s t nhin n 1 tn ti s nguyn t p ln


2

hn n c dng 8k 1 . nh l c chng minh.


nh l 2. Tn ti v hn s nguyn t c dng 8k 3 , vi k 0,1, 2,...
2
Chng minh. Gi s n l s t nhin 1 . t a p2 p3 ... pn . Do a l nn bnh phng ca n l a
2
c dng 8t 1. S N a 2 c dng 8t 3 . Nu mi c s nguyn t ca N u c dng 8t 1
th N cng c dng , v l. Vy s l N c c s nguyn t l p khng c dng 8k 1 suy ra

p c dng 8k 3 hoc 8k 5 . Gi s p 8k 5 . T p | N a 2 2 suy ra a 2 2 mod p v do


2
2 1 2
p 2 1 /8
p 1 /2
1 . Do p 8k 5 suy
1 . Nhng theo II, III, IV th 1
p p p
p
2
1
1
ra s p 1 chn v p 2 1 l do vy
1 , mu thun. Vy p c dng 8k 3 . M
2
8
p
p | a 2 2, a p2 p3 ... pn nn p pn . Do n c chn ln ty nn ta c iu phi chng minh.

nh l 3. Tn ti v hn s nguyn t c dng 8k 5 , vi k 0,1, 2,...


2
Chng minh. Xt s t nhin n 1 v t a p2 p3 ... pn . Do a l s l, s N a 4 c dng
8k 5 . Nu tt c cc c ca n c dng 8t 1 th N cng c dng , v l. Vy N c c
nguyn t l p c dng 8k 3 hoc 8k 5 . Nu p 8k 3 th v p | N a 2 4 suy ra

4 1 2
4
p 1 /2
a 4 mod p v do 1 . Theo II, III th 1
. T v
p p p
p
4
p 8k 3 ta c 1 , mu thun. Vy p c dng 8k 5 . Nhng v p | a 2 4 v
p
a p2 p3 ... pn suy ra p pn . V n c th chn ty nn nh l 3 c chng minh.
2

2. Lut tng h bc hai


Cho hai s nguyn t l phn bit p v q .
Xt cc cp s kq, lp vi k 1, 2,..., p 1 / 2, l 1, 2,..., q 1 / 2 . S cc cp ny l
Vi mi cp ta c kq lp v nu kq lp ta c p | kq , suy ra, v

p 1 q 1
.

2
2

p, q 1, p | k ,

v l v

k p 1 / 2 . Ta chia tt c cc cp ny thnh hai lp, lp th nht cha cc cp m kq lp , lp


cn li cha cc cp m kq lp . Ta tnh s cc cp trong mi lp.
Cho trc s l trong dy 1, 2,..., q 1 / 2 . Nu cp kq, lp thuc lp th nht th k lp / q . Do

lp q 1 p p
lp p
lp

, suy ra , ta c 2 p, ngha l
q
2q
2
q
q 2
1
lp
lp p 1
2 p 1 suy ra
. T vi s cho trc l , l q 1 , k c th nhn cc gi tr
2
2
q
q

lp / q khng nguyn v v

218 | Lut tng h bc hai

lp
1, 2,..., , suy ra s cp thuc lp th nht l
q
p 1 /2

k 1

q 1 /2

l 1

lp
q . Tng t s cp thuc lp th hai l

kq
p 1 q 1

suy ra
p . Do s cp trong c hai lp l
2
2

q 1 /2
p 1 /2
p 1 q 1 lp kq


2
2
l 1 q
k 1 p

(13)

T b Gauss v cc tnh cht I, II ta c

2 q p
2q 2 p q 2 2 q p / 2


p
p
p
p

p1/2

1
k 1

k pq

p1/2

1
k 1
p 1 /2

ng thc cui cng suy ra t ng thc

k 1

kq

p

p1/2

p1/2

k 1

1
k 1

kq p 2 1

8
p

1 2
p 1 .
8

p 1
2q 2 q q
2q
V q l nn t II, IV ta c 1 8 , kt hp vi cng thc trn ca

p p p p
p
2

p1/2

suy ra 1 k 1
p

kq

p

. T (13) hai cng thc ny suy ra

p 1 q 1
p q

V. 1 2 2
q p

ng vi mi cp s nguyn t l phn bit p v q . Cng thc ny c gi l lut tng h bc

p 1 q 1

l l nu v ch nu mi s p v q u c dng 4k 3 v vy ng thc V suy


2
2
q
p
ra nu hai s nguyn t l phn bit p v q c dng 4k 3 th v nu trong hai s
p
q
hai. S

q p
. Gauss t mnh a ra 7 li gii cho nh l ny. Bng 45
p q
li gii c trnh by t nm 1796 ti nm 1897 c tng hp bi P.Bachmann trong [2] trang
203. S li gii cho lut tng h ny ngy mt tng. Ta p dng tnh cht V cho nh l sau.
c s c dng 4k 1 th

nh l 4. Tn ti v hn s nguyn t c dng 5k 1 vi k l s t nhin.


Chng minh. Xt n l s t nhin ty 1 . t N 5 n ! 1 . R rng N l s l 1 v khng
2

c dng 5t 1 , do n c t nht mt c s nguyn t l p (khc 5) v khng c dng 5t 1 . Ta

5
p
1 . Theo V ta c 5 1 . S nguyn t

p
p khc 5 phi c dng 5k 1 hoc 5k 2 . Nu p 5k 2 th theo
I,II ta c
2
p 2 1 2
1
p
. Nhng t III th 1 v theo IV th 1 ta suy ra 1
5
5 5 5 5
5
5

c p n . Do p | N , suy ra 5 n ! 1 mod p v th
2

CHNG 9. K HIU LEGENDRE V K HIU JACOBI | 219

mu thun. V vy p c dng 5k 1 do theo gi thit suy ra n c dng 5k 1 . V vy vi mi s


t nhin n tn ti s nguyn t p n c dng 5k 1 . iu phi chng minh.
Nu p 5k 1 ( k l s t nhin) l s nguyn t th k chn (nu ngc li th p chn v 2 ). V
vy k 2t , vi t l s t nhin v p 10t 1. T nh l 4 suy ra tn ti v hn s nguyn t c
dng 10t 1 vi t l s t nhin. Hay ni cch khc tn ti v hn s nguyn t c ch s tn cng
l 9. D dng kim tra rng tn ti v hn s nguyn t c dng 5k 2 vi k l s t nhin. Tht
vy, xt s t nhin ty n 2 . t N p2 p3 ... pn 2 . Th N l s l 1 m khng chia ht cho
5. Nu tt c cc c s nguyn t ca n u c dng 5k 1 th s N cng c dng , v l. Vy
tn ti t nht mt c s nguyn t p ca N khc 5 v khng c dng 5k 1 . Do p c dng
5k 2 . Nhng do p pn ta suy ra iu phi chng minh. Chng minh trong cp s cng tn ti v
hn s nguyn t c dng 5k 2 v 5k 2 khng kh. Li gii trn thc ra t c kt qu
mnh hn, v k phi l s l nn thc ra ta chng minh rng tn ti v hn s nguyn t c ch
s tn cng l 7 v 3.
nh l 5. Mi s nguyn t p c dng 6k 1 u c dng p 3x 2 y 2 vi cc s t nhin x, y .
Chng minh. Gi s p l s nguyn t c dng 6k 1 . Theo tnh cht V ca k hiu Legendre suy

3
p 1 /2 p
1
. T I ta c
3
p

ra

p 1
1 . Kt hp hai ng thc ny ta c
3 3

1
3 1 3
p 1 3
p
1 2
1,
3
p p p
p

Suy ra 3 l thng d bc hai modulo p . Suy ra tn s nguyn a tha mn a 2 3 0 mod p . T


nh l Thue (Chng 1 mc 13) suy ra tn ti cc s t nhin x, y

p m vi cch chn du

ph hp th ax y chia ht cho p . Vy ta c p | 3x y . Nhng do p | a 2 3 nn p | a 2 x 2 3x 2 ,


2

ta c p | 3x 2 y 2 . Li c x

p v y

p . H qu l bi v p l s nguyn t nn ta c x 2 p v

y 2 p , dn ti 3x 2 y 2 4 p . Do p | 3x 2 y 2 ta c 3x 2 y 2 pt vi t l s t nhin 4 . Nu
t 3 th 3 | y v do y 3z trong z l s t nhin.Vy p x 2 3z 2 . Nu t 2 th cc s
x, y cng tnh chn l. Trong c hai trng hp th 2 p 3x 2 y 2 u chia ht cho 4 suy ra 2 | p ,
v l. Nu t 1 ta c p 3x 2 y 2 . iu phi chng minh.
D dng chng minh rng nu s nguyn t p c dng p 3x 2 y 2 vi x, y l cc s t nhin th
p c dng p 6k 1 vi k l s t nhin. T nh l 10 Chng 5 suy ra mi s nguyn t c
dng 6k 1 c ng mt biu din dng 3x 2 y 2 vi x v y l cc s t nhin. B.Van der Pol v
P.Speziali [1] tnh mi biu din dng 3x 2 y 2 ca cc s nguyn t c dng 6k 1 khng vt
qu 10000 . c bit ta c

7 3 12 22 ,

13 3 22 12 ,

19 3 12 42 ,

31 3 32 2 2

37 3 22 52 ,

43 3 32 42 ,

61 3 2 2 7 2 ,

67 3 12 82 ,

73 3 42 52 ,

79 3 52 22 ,

97 3 42 7 2.

A.Makowski lu rng t nh l 5 suy ra h qu: vi mi s nguyn t p c dng 6k 1 th s

2 p 4 l tng ca ba trng phng. Tnh cht ny suy trc tip t nh l 5 v ng nht thc

2 3x 2 y 2 3x 2 2 xy y 2 3x 2 2 xy y 2 4 xy
4

v lu rng vi p 3x 2 y 2 ta c 3x2 2 xy y 2 p 2 y 2 2 xy . V phi khc 0 v p 6k 1 l.

220 | Tnh ton k hiu Legendre

Ta ch ng thc

2 3x 2 y 2 3x 2 2 xy y 2 3x 2 2 xy y 2 4 xy .
2

Do vi x 1, y 2 ta c

2 74 34 54 84 ,

2 72 32 52 82

V vi x 2, y 1 ta c

2 134 154 74 84 ,

2 132 152 72 82

Ta cng c hai ng thc

2 3x 2 y 2 x y x y 2 x ,
2

2 3x 2 y 2 x y x y 2 x ,
2

t y suy ra vi mi s nguyn t p c dng 6k 1 th s 2 p 2 l tng ca ba trng phng. V d


vi x 1, y 2 ta c

2 72 34 14 24 ,

2 7 32 12 22

V vi x 2, y 1 ta c

2 132 34 14 44 ,

2 13 32 12 42

3. Tnh ton k hiu Legendre


Ta tnh ton cc k hiu Legendre da vo 5 tnh cht c suy trc tip t nh ngha ca n v
lu rng gi tr ca cc k hiu ny bng 1 hoc -1. Vi s nguyn t l cho trc p v D l s
nguyn khng chia ht cho p . t r l phn d ca D khi chia cho p . Suy ra 0 r p v theo

D r
I, . K hiu a 2 l bnh phng ln nht l c s ca r . Ta c r ka 2 vi k 1 hoc
p p
k l tch ca cc s nguyn t khc nhau, ngha l k q1 q2 ... qs vi q1 q2 . . . qs . Hn na v
2

D ka 2 k a 2 k a k
r p , ta c qs p . T II ta c
. S ny bng
p p p p p p p
q1 q2 qs
1
q1
1 hoc bng ... . Nu q1 2 th c tnh da vo tnh cht IV. Nu
p p p
p
p
q
q1 2 th gi tr ca vi q v p l cc s nguyn t l v q p c tnh da vo tnh cht
p
p 1 q 1
q p
D

1
2 2 . Bi vy vic tnh ton cc k hiu Legendre quy v vic
p q
p
D'
tnh cc k hiu
vi q l s nguyn t l nh hn p . Do sau mt s hu hn php rt
p

V v ta c

D
. Qu trnh ny c bt tin v phi s dng ti php
p

gn ta c th tnh c gi tr ca k hiu

phn tch thnh tha s nguyn t. trnh vn ny Jacobi a ra mt k hiu tng qut
hn. Ta s tm hiu k hn trong mc tip theo.
4. K hiu Jacobi v cc tnh cht

CHNG 9. K HIU LEGENDRE V K HIU JACOBI | 221

D
Jacobi nh ngha k hiu cho cc s l P 1 v s nguyn D nguyn t cng nhau vi P
P
1 2
nh sau: nu P q1 q2 ... qss l phn tch thnh tha s nguyn t ca P (cc tha s u l) th
1

D
D D D
... ,
P q1 q2
qs

(14)

trong v phi l cc k hiu Legendre. T y suy ra ngay nu P l s nguyn t th k hiu


Jacobi trng vi k hiu Legendre. Tuy nhin cc quy tc v thng d bc hai ca k hiu Jacobi
D
khng suy trc tip t k hiu Legendre. L do l t ng thc 1 suy ra D khng phi
P

D
trong v phi ca (14) phi bng
qi

thng d bc hai ca P bi v t nht mt trong cc nhn t

1 , suy ra ng d thc x2 D mod qi l khng c nghim do ng d thc x2 D mod P

D
cng khng c nghim, ng thc 1 khng suy ra rng D l thng d bc hai ca P , v d
P
2 2 2
2
1 1 1 v ng d thc x 2 mod15 l khng gii c v phng
15 3 5
trnh x 2 2 mod 3 v nghim. K hiu Jacobi cng c 5 tnh cht tng t k hiu Legendre.
Trc khi chng minh chng ta lu (14) c th vit thnh dng

D D D D
... ,
P q1 q2 qs

(15)

trong P q1 q2 ...qs v cc s nguyn t q1 , q2 ,..., qs khng nht thit phn bit.

D D'
Tnh cht I. Nu D D ' mod P th
.
P P
Chng minh. Theo (15) ta c

D D D D
... ,
P q1 q2 qs

(16)

D' D' D'


... ,
P q1 qs

Nu D D ' mod P th suy ra D D ' mod qi vi mi i 1, 2,..., s . Do theo tnh cht I ca cc

D D'
D D'
vi i 1, 2,..., s , suy ra theo (16) th .
P P
qi qi

k hiu Legendre th

DD ' D D '
Tnh cht II.
vi mi s nguyn D v D ' khng chia ht cho P .
P P P

DD ' DD ' DD ' DD '


.

...

P q1 q2 qs

S dng tnh cht II ca k hiu Legendre, cng thc (16) v

1
H qu trc tip ca tnh cht II l 1 .
P
P 1 /2
1
Tnh cht III. 1
.
P

Chng minh. Theo (15) v tnh cht III ca cc k hiu Legendre ta c

222 | K hiu Jacobi v cc tnh cht


q 1
q1 1 q2 1

... s
1 1 1 1
2
2
2

...


P
q
q
q
1 2 s

(17)

Ta c P q1q2 ...qs q1 1 1 q2 1 1 ... qs 1 1 . Tt c cc s q1 1, q2 1,..., qs 1 u


chn, do tch ca hai trong s chng u chia ht cho 4.
V vy P 4k 1 q1 1 q2 1 ... qs 1 , v do

P 1
2

q 1
q1 1 q2 1

... s
2
2
2

q 1
q 1 q 1
P 1
2k 1 2 ... s . Suy ra
2
2
2
2

. Theo (17), ta c tnh cht III.

2
P2 1 /8 .
Tnh cht IV. 1
P
Chng minh. Theo (15) v tnh cht IV ca cc k hiu Legendre ta c

2 2 2 2
...
P q1 q2 qs

(18)

q 1
q1 1 q2 1

... s
8
8
8

Do bnh phng l lun c dng 8k 1 , ng thc P 2

q 1 1 q 1 1... q 1 1
2
1

2
2

2
s

chng t mi hiu q12 1, q22 1,..., qs2 1 u chia ht cho 8. Suy ra tch ca hai trong s chng u
chia ht cho 64.
q2 1
q 2 1 q22 1
P2 1
8k 1

... s
V vy P 2 64k 1 q12 1 q22 1 ... qs2 1 v v
8
8
8
8

suy ra 1

P 2 1
8

q 2 1
q12 1 q22 1

... s
8
8
8

. Theo (18) suy ra tnh cht IV.

P 1 Q 1
P Q
.
2
2 vi mi cp s l nguyn t cng nhau P, Q 1 .


Q P

Tnh cht V.

Chng minh. t Q r1 r2 ... rt trong r1 , r2 ,..., rt khng cn l cc s nguyn t l phn bit. Theo
(15), tnh cht II v tnh cht V ca k hiu Legendre ta c
(19)

s t
qi 1 r j 1
P Q s t qi rj

2
2

1
j

1

Q P i 1 j 1 rj qi

Nhng

qi 1 rj 1 s qi 1

2
2
2
i 1 j 1
i 1
s

(20)

rj 1

j 1

qi 1 t rj 1 P 1 Q 1

2h . Kt hp

2
2
2
2
i 1
j 1
s

rng trong chng minh tnh cht III th ta c


vi (19), (20) suy ra tnh cht V.
5. Lut Eisenstein

Cc tnh cht trn ca k hiu Jacobi c s dng dn ti lut Eisenstein m theo gi tr


ca cc k hiu Jacobi c th c tnh (v do cc k hiu Legendre cng vy) m khng cn s
dng ti cc phn tch thnh tha s nguyn t.

CHNG 9. K HIU LEGENDRE V K HIU JACOBI | 223

D
vi P l s l 1 v D nguyn t cng nhau vi
P
Q
P c th rt gn thnh vic tnh cc gi tr vi Q l s t nhin l.
P

u tin ta ch rng vic tnh gi tr ca

Tht vy nu 2 (vi l s nguyn 0 ) l ly tha bc cao nht ca 2 l c s ca D th

D 1 2 Q trong 0 hoc 1 , Q l s t nhin l. R rng tnh Q ta khng cn bit

phn tch thnh tha s nguyn t ca D m ch cn chia D lin tc cho 2. Theo cc tnh cht ca
P 1 P 2 1

Q
D
Q
8
k hiu Jacobi v theo cng thc ca D ta c 1 2
. V vy ch cn tnh P

P
P
vi Q, P l cc s l nguyn t cng nhau.
K hiu R l phn d ca Q khi chia cho P . Khi R l mt trong dy 1, 2,..., P 1 . S P R
cng thuc v dy ny. V vy vi s nguyn t ta c Q Pt R v Q P t 1 P R . Do tng
ca R v P R l nn mt trong hai s l l v s cn li chn. K hiu s l l P1 . Nu P1 R

th Q Pt P1 . Nu P1 P R th Q P t 1 P1 . Trong c hai trng hp ta c Q Pk 1P1


vi k l s nguyn v 1 bng 1 hoc 1 .
Ta ch rng k l s chn v nu ngc li th s Q P1 l, v l v cc s Q v P1 u l. Suy ra

k 2k1 vi k1 nguyn. Ta c Q 2k1P 1P1 . Nu P1 1 th ta c th lp li lp lun trn vi P v


P1 suy ra P 2k2 P1 2 P2 vi k 2 nguyn v 2 1, P2 l s t nhin. Nu P2 1 th theo trng
hp trn P1 2k3 P2 3 P3 v li tip tc nh th.
Dy cc s P, P1, , P2 ,... gim nghim ngt v P1 P 1, P2 P1 1,... Do dy cc bt ng thc
ca cc s P, P1 , P2 ,... khng th ko di v hn v s cc s l P l hu hn. V vy ta suy ra bt
ng thc cui cng Pn2 2kn Pn1 n Pn vi Pn phi bng 1 v nu ngc li th bt ng thc tip
theo s nhn c. Vy ta c dy ng thc
Q 2k1P 1P1 , P 2k2 P1 , P1 2k3 P2 3 P3 , ...

(21)

Pn3 2kn1Pn2 n1Pn1 , Pn2 2kn Pn1 n Pn ,

vi Pn 1 . ng thc u tin trong (21) theo tnh cht I, II ca k hiu Jacobi suy ra
(22)

Q 1 P1

P P P

P 1
P 1 1
P 1 11
P 1 1
Nu 1 1 th 1 1 1 2 2 1 2 2 ; Nu 1 1 th 1 1 2 1 2 2 .
P
P
1

P 1 11


Trong mi trng hp ta u c 1 1 2 2 .
P

T tnh cht V ca k hiu Jacobi v nhn xt bnh phng ca k hiu Jacobi lun bng 1 ta c
P 1 P1 1

P1 P
2
2 suy ra theo (22) th


P
P
1

Nhng v 12 1 ta c

P 1 11 P 1 P1 1

Q P
2
2
2
2 .


P P1

224 | Lut Eisenstein

P 1 1 1 P 1 P1 1 P 1 P1 1 P 1 1 P1 12

2
2
2
2
2
2
2
21

Hin nhin 1

a / 1

vi 1 1 do

P 1 1 P1 1

2
21

P 1 11 P 1 1P1 1

2
2
2
2

P 1 1P1 1

2
2

; vy ta c

P 1 1P1 1
P

Q
1 2 2 . Tng t t ng thc th hai trong (20) suy ra cng thc
P
P1

cng thc

P1 1 2 P2 1
P
P1

1
2
v c tip tc nh th. ng thc p
P1
P2
Pn2 1 n1 Pn1 1
Pn 3
Pn 2

2
2

. ng thc cui cng vi Pn 1 suy ra


Pn 2
Pn1

cht cho ta cng thc


Pn1 1 n 1
n

2
2 . Vy

Pn1

Pn1 1 n Pn 1
Pn3

2
2

P
n2

ta thu c

Pn2
Q P
ta c cng thc
, ,....,
P P1
Pn 1

By gi kt hp cc cng thc thu c cho

P 1 P 1
P 1 1P1 1 P1 1 2 P2 1

... n1 n n
Q
2
2
2
2
2
2

1

P

Gi tr ca v phi ca ng thc ph thuc vo s cc hng t l ca s m. Tch


nu v ch nu mi s P v 1 P1 u c dng 4t 3 . Vy ta c th vit
(23)

P 1 1 P1 1

l
2
2

m
Q
1 ,
P

trong m l s cc cp Pi 1 , i Pi ( i 1, 2,..., n , v P0 P ) m c Pi 1 v i Pi u c dng 4t 3 .

Q
Lut Eisenstein. tnh ta s dng ng thc (21) v tm m l s tt c cc cp Pi 1 v i Pi
P
m Pi 1 v i Pi u c dng 4t 3 , sau ta thay m vo (23).
S dng lut ny ta c th tnh cc k hiu Jacobi m khng cn s dng ti cc phn tch thnh
tha s nguyn t.

641
V d. 1. S dng lut Eisenstein tnh
. ng thc (21) l
257

641 2 257 127, 257 2 127 3,

127 42 3 1

Trong cc cp 257,127;127,3;3,1, ch c 2 cp m c hai u c dng 4t 3 .

641
V vy m 1 do
1 suy ra 641 khng phi thng d bc hai modulo 257 .
257
65537
2. Tnh
. Ta c
274177

CHNG 9. K HIU LEGENDRE V K HIU JACOBI | 225

65537 0 274177 65537, 274177 4 65537 12029, 65537 6 12029 6637,


12029 2 6637 1245, 6637 6 1245 883, 1245 2 833 421, 833 2 421 9,
421 46 9 7, 9 2 7 5, 7 2 5 3, 5 2 3 1
Trong cc cp Pi 1 , i Pi ch c 7, 5 v 3, 1 u c dng 4t 3 .

65537
Suy ra m 2 v vy
1.
274177
104 1 2 13
104
3
3. Tnh
ta tm c 104 1 2 13 . Do

. S 997 c
997 997 997 997
997
1
104
13
2
dng 4t 1 nn
1 . S 997 c dng 8t 5 nn
1 . V vy

.
997
997
997
997

13
tnh
ta xt dy ng thc 13 0 997 13, 997 76 13 9, 13 2 9 5, 9 2 5 1.
997
Trong dy ny khng c cp Pi 1 , i Pi no m hai phn t u c dng 4t 3 .
13
104
Vy m 0 suy ra
1 v ta c
1 .
997
997

226 | Lut Eisenstein

CHNG 10
CC S MERSENNE V CC S FERMAT
1. Mt s tnh cht ca cc s Mersenne
Cc s Mersenne M n 2n 1 c tm hiu trong Chng 4 mc 5. nh l 5 Chng 5 ni
rng mt s chn l s hon ho khi v ch khi n c dng 2n 1 M n vi n l s t nhin v M n l s
nguyn t Mersenne. y l l do khin cc s nguyn t Mersenne c quan tm. Hn na s
nguyn t ln nht c bit hin nay cng l s Mersenne.
Trong Chng 4 mc 5 ta bit nu s Mersenne M n l s nguyn t th n cng l s nguyn t,
tuy nhin iu ngc li khng ng chng hn M11 23 89 .
D dng chng minh mt s t nhin m l s Mersenne khi v ch khi m 1 khng c c s
nguyn t l. Golomb [1] lu rng t nhn xt ny ta c th xy dng mt phng php tm
tt c cc s Mersenne tng t phng php sng Eratosthenes. By gi ta chng minh mt nh
l m trong mt s trng hp gip ta quyt nh c mt s Mersenne l hp s hay khng.
nh l 1. Nu q l s nguyn t c dng 8k 7 th q | M q 1 /2 .

2
q 1 /2
mod q .
q
Nu q l s nguyn t c dng 8k 7 th theo tnh cht 4 ca k hiu Legendre (Chng 9 mc 1)
Chng minh. T cng thc trong Chng 9 ta c nu q l s nguyn t th 2

2
q

ta c 1 . H qu l 2

q 1 /2

1 mod q suy ra q | 2 q 1/2 1 .

7
Bng php quy np n gin ta c 24 k 3 8 k 1 . Tht vy 2 8 2 v nu 24 k 3 8 k 1 th

24 k 13 24 8 k 1 8 k 2 . Do nu q 8k 7 7 th 2 q 1/2 1 8k 7 q suy ra nu q
l s nguyn t c dng 8k 7 7 th M q 1 /2 l hp s chia ht cho q . Ta c h qu
H qu. Nu s nguyn t n 3 c dng 4k 3 v q 2n 1 l s nguyn t th M n l hp s chia
ht cho q .
Bng cch ny ta xc nh c cc s Mersenne sau y u l hp s, cc c s nguyn t ca
chng cng c tm ra c th

T gi thuyt H (Chng 3 mc 8) ta bit tn ti v hn s nguyn t p c dng 4k 3 m


q 2 p 1 l s nguyn t. V vy s dng h qu trn ta thy gi thuyt H suy ra s tn ti v
hn cc s nguyn t p m cc s M p l hp s (Schinzel v Sierpiniski [3] trang 198 C9 ).
T nh l 1 ch rng vi cng phng php lp lun ta c th suy ra nu q l s nguyn t c
dng 8k 1 th q | M q 1 /2 . Tuy nhin

q 1 / 2 4k

khng phi s nguyn t. Chng hn

17 | M 8 , 41| M 20 , 89 | M 44 , 97 | M 48 .
Ta cha bit s Mersenne no l hp s vi ch s nguyn t v khng phi tch ca cc s nguyn
t phn bit. Ta cng cha chng minh c c tn ti v hn cc s Mersenne khng c c s
chnh phng hay khng.

228 | Mt s tnh cht ca cc s Mersenne

nh l 2. Nu n l s t nhin 1 th M n khng th l ly tha bc m ca mt s t nhin vi m


l s t nhin 1 (Gerono [1]).
Chng minh. Gi s rng 2n 1 k m vi k v m 1 l cc s t nhin. V n 1 nn k l. Nu m
chn th k m c dng 8t 1 suy ra k m 1 2 4t 1 . Nhng v n 1, k m 1 2n chia ht cho 4 suy

ra mu thun. Vy m l v 2n k m 1 k 1 k m1 k m2 ... k 1 , nhn t th hai l tng ca


l hng t l nn n l s l, m n li l c ca 2n nn phi bng 1 . Do 2n k 1 v m 1
mu thun vi gi thit. nh l 2 c chng minh.
nh l 2 suy ra khng tn ti s Mersenne l bnh phng ngoi tr M1 12 .
Mt khc tn ti cc s Mersenne l s tam gic. Tuy nhin ch tn ti bn s nh vy l
M1 t1 , M 2 t2 , M 4 t5 , M12 t90 (Ramanujan [1], Nagell [4],[12] v Hasse [2]).
D dng chng minh vi x

1
1
M1 M 2 x M 3 x 2 ... .
th
1

x
1

2
x
2

Bi tp. 1. Chng minh rng mi s l l c s ca v hn cc s Mersenne.


Chng minh. Nu m l s l th theo nh l Euler, vi mi s t nhin k ta c m | M k m .
2. Tm s Mersenne nh nht chia ht cho bnh phng mt s t nhin 1 .
Li gii. S l M 6 26 1 63 32 7 v M1 1, M 2 3, M 3 7, M 4 15 3 5 v M 5 31.
3. Tm s Mersenne nh nht c ch s l m n chia ht cho bnh phng mt s t nhin 1 .
Li gii. l s M 21 7 127 337 v

M 7 127, M 9 7 73, M11 23 89, M13 8191, M15 7 31151,


M17 131071, M19 524287
Ghi ch. Cc s Mersenne tip theo sau M 21 c ch s l v chia ht cho bnh phng mt s t
2
nhin 1 l M 63 v M 105 . Chng u chia ht cho 7 v M 21 | M 63 v M 21 | M105 .
n
4. Chng minh rng nu a v n l cc s t nhin 1 th nu a 1 l s nguyn t th n l s
Mersenne.
n
Chng minh. Nu a 2 ta c a 1| a n 1 do v n 1, 1 a 1 a n 1 suy ra a 1 khng phi
n
s nguyn t. Do v a 1 nguyn t suy ra a 2 v vy a 2 (v 1 1 khng phi s nguyn
t). Vy a n 1 M n .

5. Chng minh rng nu m l s t nhin ty , s l s cc ch s ca m trong h thp phn th


tn ti s Mersenne M n m s ch s u tin ca n trng vi s ch s ca m tng ng.
Chng minh suy ra ngay t tnh cht ca cc s 2n ( Sierpinski [11] nh l 2).
6. Chng minh rng vi mi s t nhin s th s ch s tn cng ca cc s M n 1, 2,... to thnh
s1
dy v hn tun hon chu k gm 4 5 phn t.

Chng minh suy ra t nh l 1 trong bi bo trch dn trn.


Rt nhiu cc nh l v cc c s ca M n c tng hp bi E.Storchi trong [1].
2. nh l ca E.Lucas v D.H.Lehmer

CHNG 10. CC S MERSENNE V CC S FERMAT | 229

nh l 3. S M p , p l s nguyn t l, l nguyn t khi v ch khi n l c s ca phn t th p 1


ca dy s1 , s2 ,..., vi s1 4, sk sk21 2, k 1, 2,... (Lehmer [2], Kraitchik [1] trang 141 v Trost [3])
Chng minh. t a 1 3, b 1 3 . Ta c a b 2, ab 2, a b 2 3 . Xc nh dy cc s
t nhin un , vn n 1, 2,... c cho bi cng thc bi

un

a n bn
,
a b

vn a n b n

Cc cng thc ny suy ra vi mi n 1, 2,... ta c

n n
n
un 3 32 ...,
1 3
5

n
vn 2 1 3 32 ...
5
3

V vy vi mi s t nhin k , l ta c

Vi s nguyn t l q k hiu q l s t nhin nh nht n tha mn q | un (s ny l tn ti).


B 1. S nguyn t l q l c ca un , n l s t nhin, khi v ch khi q | n .
Chng minh b 1. Cho trc s nguyn t l q . K hiu S l tp tt c cc s t nhin n tha
mn q | un . T (1) v (2) nu hai s k v l thuc tp S th k l cng thuc tp S , hn na nu
k l th k l cng thuc S . Do tp S c tnh cht: tng v hiu (dng) ca hai phn t
thuc S cng thuc S .
K hiu d l s t nhin nh nht thuc S . T tnh cht trn suy ra cc s kd , k 1, 2,..., cng
thuc S . Mt khc gi s n thuc S v n chia d d r 0 . Khi n td r vi t l s nguyn
0 v r d . Nu t 0 th r rng l v l v r nh hn d khng th bng n v v vy khng
thuc S theo nh ngha ca d . H qu t l s t nhin v v vy td thuc S suy ra theo tnh
cht ca S th s r n td l hiu ca hai phn t thuc S vi n td nn cng thuc S , iu
ny mu thun vi nh ngha ca d . T y suy ra r 0 , ngha l tp S l tp hp cc bi s
dng ca phn t ca n.
Vy nu n thuc S th q | n v c nh vy. B c chng minh.
B 2. Nu q l s nguyn t 3 th
(7)

q | uq 3 q 1 /2

V
(8)

q | vq 2

Chng minh b 2. chng minh (7) ta s dng ng thc

1
uq
1 3
2 3

1 3

q 1 /2

k 0

q k

3
2k 1

230 | nh l ca E.Lucas v D.H.Lehmer

Trong tng ny tt c cc h s nh thc ch tr ra h s u tin th u chia ht cho q do suy


ra (8).
B 3. Nu vi s nguyn t q 3 s q tn ti th q q 1 .
Chng minh b 3. V u1 1, v1 2 , theo (1) v (2) vi k q, l 1 ta c 2uq 1 2uq vq v
2
2
2
q 1
2
4uq 1 2uq vq v vy 8uq 1uq 1 4uq vq . Nhng t B 2 ta c q | uq 3 v q | vq 4 . V q
2
l s nguyn t 3 , s dng nh l Fermat nh ta nhn c q | 3q 1 1 . V vy q | uq 1 v do

q | 4uq2 vq2 . H qu l q | 8uq 1uq 1 suy ra v q 3 ta c hoc q | uq 1 hoc q | uq 1 . Trong trng hp


th nht theo B 1 ta c q q 1 v trong trng hp sau ta c q q 1 . V vy ta lun
c q q 1 .
By gi ta chng minh iu kin trong nh l 3 cng l iu kin .
Gi s p l s nguyn t l v M p | s p 1 . Khi
(9)

M p | 22

p 2

s p 1
n1

Ta c 2s1 v2 . Vi s t nhin n gi s 22 sn v2n , iu ny ng vi n 1 . Do sn1 sn2 2 ta c

n1

22 sn1 22 sn
n

22

v22n 22

n
. Nhng t (4) vi k 2 ta c v2n1 v22n 22

. V vy

n1

22 sn1 v2n1 . Cng thc 22 sn v2n c suy ra theo quy np. V vy vi n p 1 ta c


n

(10)

p1

22 s p 1 v2 p1

T (10) v (9) ta c
(11)

M p | v2 p1

p 1
Suy ra theo (3) vi k 2

(12)

M p | u2 p

p
K hiu q l c s nguyn t ty ca M p . V p l nn s M p 2 1 khng chia ht cho 3 , ta
p
c q 3 . Do q | M p v cng thc (12) suy ra q | u2 p do theo b 1 ta c q | 2 .

Mt khc q khng phi c ca 2 p1 v nu ngc li th theo B 1 suy ra q | u2 p1 do


theo (5) vi k 2 p 1 , p l c s ca mt ly tha ca 2, iu ny khng th v q l s nguyn t
p
3 . V vy q 2 .

Theo B 3 ta c 2 p q 1 suy ra M p q v v q | M p suy ra M p q ngha l M p l s nguyn


t. Vy iu kin trong nh l 3 cng l iu kin . chng minh iu kin cn ta chng minh
B 4. Nu p l s nguyn t c dng 12k 7 th p | 3

p 1 /2

1 .

Chng minh b 4. Xt s nguyn t p c dng 12k 7 vi s nguyn k 1 . Khi p 3 v

p 1
theo tnh cht 1 ca k hiu Legendre (Chng 9 mc 1) ta c 1 . Theo tnh cht 5 k
3 3

CHNG 10. CC S MERSENNE V CC S FERMAT | 231

3
p 3
p 1 /2
1 mod p suy ra
1 suy ra 1 . H qu l 3
3 p
p

hiu Legendre ta c

p | 3 p 1 /2 1 .

By gi ta chng minh iu kin trong nh l 3 l iu kin cn. Gi s p l s nguyn t 2 v

q M p l s nguyn t. V p 2 ta c 8 | 2 p q 1 . V vy q 8t 7 vi s nguyn t 0 . Ta c

q 1 2 p 2 2 2 p 1 1 . V p 1 chn ngha l p 1 2s vi s l s t nhin nn ta c

2 p 1 1 3 1 1 3u vi u l s nguyn. V vy 3 | 2 p 1 1| q 1 8t 6 suy ra 3 | t ngha l


s

t 3k vi k l s nguyn. Do q 8t 7 24k 7 . Theo (4) vi k 2 p 1 ta c


(13)

v2 p v22p1 4 22

p1

Nhng t q 24k 7 8 3k 7 v theo nh l 1 ta c q | M q 1 /2 , ngha l q | M 2 p1 1 2 2 p11 1


suy ra theo (13) th
(14)

q | v2 p v22p1 4

Nhng theo (6) vi k q, l 1 v v q 1 2 p ta c 2v2 p vq v1 12uqu1 2vq 12uq . H qu l


(15)

v2 p vq 6uq vq 2 6 uq 1 4

Do q 24k 7 ta p dng B 4 cho q do q | 3 q 1/2 1 v v vy theo (7) ta c q | uq 1 v


2
theo (8) th q | v p 2 . V vy theo cng thc (15) ta c q | v2 p 4 suy ra theo (14) th q | v2 p1 . V

vy t (10) ta c q M p l suy ra M p | s p 1 . nh l 3 c chng minh.


D dng chng minh nh l 3 tng ng vi nh l Lucas sau y
nh l 3 . Gi s p l s nguyn t l th M p l s nguyn t nu v ch nu M p l c s ca
phn t th p 1 ca dy t1 , t2 ,..., , vi t1 2, tk 1 2tk2 1 vi k 1, 2,...
S tng ng ny c suy ra ngay t vic dy sk k 1, 2,... tr thnh dy tk k 1, 2,... nu

sk c thay bi 2tk . Do v M p l nn M p | s p 1 tng ng vi M p | t p 1 . Chng minh ca


nh l 3 da trn cc hm lng gic cc bin phc c trnh by bi T.Bang [1].
3. S nguyn t ln nht tm c
nh l 3 khng thun tin khi s dng nghin cu cc s Mersenne c ch s ln hn 10. L do
l v cc phn t ca dy sk k 1, 2,... tng rt nhanh theo k . Theo quy np t nh ngha ca

dy s1 4, sk sk21 2, k 2,3,..

ta c s

102

k 2

4 vi mi k 2,3,... suy ra s10 102 10256


8

27
ngha l phn t th 10 l s10 c hn 250 ch s. S s100 c hn 10 ch s. V vy s dng

nh l 3 nghin cu xem khi no th s M p ( p l s nguyn t 2 ) l s nguyn t th ta lm


nh sau.
Vi mi s nguyn t k hiu t l phn d nhn c khi chia t cho M p . Ta c ngay M p | t t .
By gi ta xt dy rk k 1, 2,... nh ngha bi
___

(16)

r1 4; rk 1 rk2 2

V theo quy np ta chng minh

vi

k 1, 2,...

232 | S nguyn t ln nht tm c

(17)

M p | sk rk

k 1, 2,...

vi

Ta thy (17) ng vi k 1 . Gi s tnh cht ny ng vi s t nhin k no . Khi t


M p | sk2 rk2 suy ra M p | sk2 2 rk2 2 . V sk2 2 sk 1 v v M p | t t vi t rk2 2 v theo (16)

2
th M p | rk 2 rk 1 nn ta nhn c M p | sk 1 rk 1 . Cng thc (17) c chng minh.

Theo (17) th cng thc M p | s p 1 tng ng vi M p | rp 1 . Theo (16) th tnh rp 1 ta c th


tnh p 2 bnh phng cc s d nhn c khi chia cho M p , cc s ny khng c nhiu ch s
hn M p , v tnh s d bi cc bnh phng ca chng tr i 2 khi chia cho M p .
Cc my tnh in t hin nay c th tnh ton theo th tc trn vi cc s nguyn t p ti
200.000. Theo cch ny s M 293 c ch ra l hp s v n khng phi c s ca r293 . Ta cha
bit s nguyn t no l c s ca s ny. Tnh hung tng t xy ra vi M 347 (Brillhart,
Lehmer, Selfridge, Tuckerman and Wagstaff [1]).
Ti tn nm 1950 s nguyn t ln nht c tnh l M 127 , s ny c 39 ch s. S ny c
nghin cu bi E.Lucas nm 1876 v nm 1914 E.Fauquembergue chng minh n l s nguyn
t. Thng 1 nm 1952 vi s tr gip ca my SWAC cc s M 521 v M 607 c chng minh l
s nguyn t. S th nht c 157 ch s v s kia c 183 ch s. Cng nm vo thng 7 s
M1279 c chng minh l s nguyn t. N c 376 ch s. Thng 10 nm 1952 cc kt qu tng
t c R.M.Robinson chng minh cho cc s M 2203 v M 2281 . S u tin c 664 ch s v s kia
c 687 ch s (H.S.Uhler [2],[3]). S nguyn t Mersenne tip theo l M 3217 c tm ra bi
H.Riesel vo nm 1957 bng my BESK v nm 1962 Alexander Hurwitz tm ra M 4253 v M 4423
bng my IBM 7090. Cc tnh ton khc c thc hin bi D.B.Gillies trn my ILLIAC II dn ti
cc s nguyn t M 9683 , M 9941 v M11213 vo nm 1964. Nm 1971 B.Tuckerman tm c s
Mersenne tip theo l M19937 bng my IBM 360/91. Cc s nguyn t M 21701 , M 23209 c tm ra
bi E.Nickel v C.Noll bng my CDC Cyber 174 nm 1978 v 1979. Nm 1979 D.Slowinski s
dng my CRAY 1 tm ra M 44497 , nm 1983 l M 86243 , M132049 v nm 1985 l s M 216091 . S cui
cng l s nguyn t ln nht c tnh hin nay v n c 65050 ch s.
Vy 30 s nguyn t Mersenne M n c tnh l

n 2,3,5, 7,13,17,19,31, 61,89,107,127,521, 607,1279, 2203, 2281,3217, 4219, 4423,


9689,9941,11213,19937, 21701, 23209, 44497,86243,132049, 216091
(Brillhart, Lehmer, Selfridge, Tuckerman v Wagstaff [1]).
Vi mi n 263 phn tch thnh tha s nguyn t ca 2n 1 cng c tnh. Chng hn M 101 l
tch ca hai s nguyn t, s b hn l 7432339208719 (Brillhart et al. [1]).
C mt gi thuyt ni rng nu M n l s nguyn t th M M n cng l s nguyn t. Gi thuyt ny
ng vi 4 s nguyn t Mersenne u tin nhng vi s th 5 l M13 8191 th gi thuyt sai.
8191
Cu tr li ph nh c trnh by bi D.J.Wheeler nm 1953. S M M13 2 1 (c 2466 ch

s) l hp s (Robinson [1] trang 844). Kt qu ny c c bng cch s dng nh l Lucas v


Lehmer, cc tnh ton thc hin bi my tnh in t trong 100 gi ng h. Cha c c s
nguyn t no ca s ny c tnh. Tuy nhin nm 1957 ta chng minh c M 17 l s

nguyn t nhng M M17 l hp s v n chia ht cho 1768 217 1 1. Tng t vi M 19 l s

nguyn t nhng M M19 l hp s chia ht cho 120 2 1 1 .


19

CHNG 10. CC S MERSENNE V CC S FERMAT | 233

Trong mi lin h ny c mt gi thuyt khc c t ra (cha c cu tr li): dy q0 , q1 , q2 ,...,


vi q0 2, qn1 2qn 1, n 0,1, 2,..., ch cha cc s nguyn t?
Gi thuyt ny c kim tra vi qn m n 4 , s q5 c hn 1037 ch s. Hn na v cc c s
ca q5 u c dng 2kq4 1 2q4 , s q5 khng c c s nguyn t c t hn 39 ch s. V vy ta
vn cha bit q5 c phi s nguyn t hay khng.
4. c s nguyn t ca cc s Fermat
n
Cc s Fermat Fn 22 1 n 0,1, 2,... l trng hp c bit ca cc s c dng a 1 vi a l
n

m
s t nhin 1 . Gi s s a 1 vi s t nhin m 1 l s nguyn t. Nu m c c s l k 1

th n kl suy ra al 1| al

1 a m 1 v v k 1 s a m 1 l hp s. H qu l nu a m 1 (vi

m l s t nhin 1 ) l s nguyn t th m l ly tha ca 2, ngha l m 2n vi n l s t


nhin. c bit nu 2m 1 (vi m l s t nhin) l s nguyn t th n phi l s Fermat. V vy
suy ra s t nhin s l s nguyn t Fermat khi v ch khi s l s nguyn t 2 v s 1 khng c
c s nguyn t l. T y ta c phng php tm tt c cc s Fermat l s nguyn t.
Phng php ny l ng dng th hai ca phng php sng Eratosthenes (so snh vi phng
php tm tt c cc s Mersenne trong mc 1).
2
nh l 4. Nu a l s nguyn chn, n l s t nhin v p l s nguyn t tha mn p | a 1 th
n

p 2n1 k 1 , vi k l s t nhin.
n1

2
2
2
Chng minh. V p | a 1 ta c p | a 1; p | a 1 l khng th v nu p | 2 , th p 2 , mu
n

2
thun v t p | a 1 suy ra
n

p, a 1

v a chn. K hiu l ly tha nh nht ca a m

n1

2
p | a 1 . V p | a 1 nn theo nh l 9 chng 6 ta c | 2n 1 . Khng th c | 2n v ta khng
2
n 1
c p | a 1 . T y suy ra 2 v theo nh l Fermat nh p | a p 1 1 suy ra | p 1 ngha l
n

2n1 | p 1 v vy p 2n1 k 1 vi k l s t nhin.


n2
nh l 5. Mi c s 1 ca s Fn vi s nguyn n 1 u c dng 2 k 1 vi k l s t nhin.

Chng minh. T chng minh ca nh l 4 (vi a 2 ) ta c nu p l s nguyn t v p | Fn th 2


n 1
c cp 2n1 mod p . Mt khc t nh l 4 suy ra p c dng 2 t 1 vi t l s t nhin. H qu l

nu n 1 th n c dng 8k 1 suy ra theo mc 1 th p | M p 1 /2 ngha l p | 2 p 1 /2 1 . Nhng 2 c


n 1
cp 2n1 mod p nn 2 | p 1 / 2 v do 2n 2 | p 1 suy ra p 2n 2 k 1 vi k l s t nhin.
n2
V vy ta thy mi c s nguyn t ca Fn n 1 u c dng 2 k 1 . Hn na v mi c s

1 ca Fn u l tch cc c s nguyn t ca Fn nn bn thn Fn cng c dng trn (v tch ca


cc s c dng mk 1 th cng c dng ). nh l 5 c chng minh.
nh l 5 c s dng nghin cu xem khi no th mt s Fermat cho trc c phi l s
nguyn t hay khng. Chng hn cc c s nguyn t ca F4 (theo nh l 5) u c dng

26 k 1 64k 1 . kim tra F4 c l s nguyn t khng ta c th chia n ln lt cho cc s


nguyn t c dng trn m khng vt qu

F4 (ngha l nh hn 28 ). S duy nht tha mn cc

iu kin ny l 193 do v F4 65637 khng chia ht cho 193 nn n l s nguyn t.


7
By gi xt s F5 . Theo nh l 5 th mi c s nguyn t ca s ny c dng 2 k 1 128k 1.
Thay k 1, 2, 3, 4, 5 ta nhn c cc s nguyn t vi k 2 v k 5 . Cc s ny l 257 v 641 .

234 | c s nguyn t ca cc s Fermat

Chia F5 232 1 cho cc s ny ta thy n chia ht cho s th hai. H qu l F5 l hp s. C th

641| F5 . Chng minh s cp tnh cht ny nh sau, ta c 641 54 24 | 54 228 232 v


641 5 27 1| 52 214 1| 54 228 1 , suy ra 641 l c s ca hiu cc s 54 228 232 v

54 228 1 ngha l s 232 1 F5 . Ta c F5 641 6700417 . V 6700417 2600 v cc c s


nguyn t ca 6700417 (l cc c s ca F5 ) u c dng 128k 1 vi k 5,6,..., nn kim
tra 6700417 c phi s nguyn t hay khng ch cn chia s ny cho 128k 1 vi 5 k 20 . Tuy
nhin tt c cc php chia ny u c s d dng. V vy 6700417 l s nguyn t. Vy F5 l tch
ca hai s nguyn t. Kt qu ny c tm ra bi Euler vo nm 1732.
Cc c s nguyn t ca F6 phi c dng 256k 1 . c s u tin dng ny nhn c khi

k 1071 v l 274177 . Do F6 l hp s. Kt qu ny c Landry tm ra nm 1880. C th


chng minh F6 cng l tch ca hai s nguyn t ging nh F5 . Cc c s nguyn t ca F7 phi
c dng 512k 1 . c s u tin ng vi k 1165031037646443 c tm ra bi M.J.Morrison v
J.Brillhart nm 1975 vi s h tr ca my tnh IBM 360/91. Nhn t kia cng l s nguyn t.
Trc , nm 1905, J.C.Morehead chng minh F7 l hp s bng cch s dng nh l 6 mc
5. Cc c s nguyn t ca F8 c dng 1024k 1 . c s u tin c dng ny ng vi
k 1208689024954 uc tm ra bi R.P.Brent nm 1980. Trc , nm 1908, J.C.Morehead v
A.E.Western chng minh F8 l hp s bng cch s dng nh l 6. Sau nm 1981 Brent v
H.C.Williams chng minh n l tch ca hai s nguyn t. S F9 l hp s. Western tm ra vo
11
5
nm 1903 s 2 k 1 vi k 2 37 l c s nguyn t ca F9 . S F10 l hp s c pht hin
bi J.L.Selfridge nm 1953. Vi s h tr ca my tnh in t SWAC ng ta tnh c
212 11131 1 l mt c s nguyn t ca n. Mt c s nguyn t khc ca s ny l
214 395937 1 c tm ra bi J.Brillhart nm 1962 vi s h tr ca my IBM 704.

Bi ton tm dy cc c s c v d dng hn.


13
13
Nm 1899 Cunningham tm ra hai c s nguyn t ca F11 l 2 39 1 v 2 119 1 . Bn c
14
s nguyn t ca F12 c tm ra l: 2 7 1 tm ra bi Pervouchine v Lucas nm 1877; cc
16
16
14
c s 2 397 1 v 2 973 1 tm ra bi Western nm 1903; c s 2 11613415 1 tm ra
bi Hallyburton v Brillhart nm 1975.

S F13 c chng minh l hp s bi G.A.Paxson v F14 bi J.L.Selfridge v Alexander Hurwitz,


nhng cha c c s nguyn t no ca cc s ny c tm ra. Gn y J.R.Hallyburton v
16
J.Brillhart tm ra c s 2 41365885 1 ca F13 . S F15 c chng minh hp s nm 1925
21
bi Kraitchik. ng ta tnh c 2 579 1 l mt c s ca n. F16 c chng minh l hp s
19
bi Selfridge nm 1953. S dng my SWAC ng ta tnh c c s 2 1575 1 ca n.

Kt qu sau cng ny quan trng v n cho cu tr li ph nh cho gi thuyt ni rng tt c cc


2

phn t ca dy 2 1, 2 1, 2 1, 2 1, 2
(c 19729 ch s) l phn t th 5 ca dy.
2

22

22

22

22

1, ... u l s nguyn t. Tht vy s F16

Cu hi F17 c phi l hp s hay khng mi c tr li gn y. Nm 1980 G.B.Gostin ch ra


19
n chia ht cho 2 59251857 1. S F18 l hp s. Nm 1903 Western tm c c s nguyn t

220 13 1 ca n. S F19 cng l hp s. Nm 1962 Riesel tm c 221 33629 1 l c s


nguyn t ca n. Ta cha bit F20 , F22 c phi l s nguyn t hay khng. Nm 1963 Wrathall ch
23
ra F21 l hp s v n chia ht cho 2 534689 1 . Nm 1878 Pervouchine ch ra F23 chia ht cho
25
s nguyn t 2 5 1 . Vy ta bit 84 s Fermat l hp s vi

CHNG 10. CC S MERSENNE V CC S FERMAT | 235

n 5, 6, 7, 8, 9,10,11, 12,13,14, 15,16, 1 7, 18,19, 21, 23, 25, 26,27, 29,30, 32, 36,
38, 39, 42, 52, 55, 58, 62, 63, 66, 71, 73, 75, 77, 81,91, 93, 99,117,125, 144,147,150,
201, 205, 207, 215, 226, 228, 250, 255, 267, 268, 275, 284, 287, 298, 316, 329, 334,
398, 416, 452,544,556, 637, 692, 744, 931,1551,1945, 20232089, 2456, 3310, 4724,
6537, 6835,9428, 9448, 23471
(Keller [1], [2], [3]). S Fermat ln nht l hp s c tm ra l F23471 . S ny c c s nguyn
23473
5 1 . S cc ch s ca F23471 l ln hn 107064 . Tnh hung tng t c trnh by
t 2
trong mc 3 v tnh chia ht cng c thc hin theo cch .

kim tra Fn c chia ht cho m hay khng u tin ta k hiu t l s d nhn c khi chia s
nguyn t cho m . Ta nh ngha dy rk k 1, 2,..., bi iu kin

r1 22 ,

rk 1 rk2 ,

k 1, 2,...

D dng chng minh bng quy np m | 22 rk vi k 1, 2,... . H qu l bit Fn c chia ht cho


k

m hay khng ta ch cn tm xem rn 1 c chia ht cho m hay khng.


Ta cha chng minh c tn ti v hn s Fermat l hp s, hoc chng minh c t nht mt s
Fermat F4 l hp s. Cc s kin trn dn ti gi thuyt ni rng mi s Fermat F4 u l
hp s.
m
S dng nh l 5 suy ra cc c s nguyn t ca cc s Fermat u c dng k 2 1 vi k , m l
cc s t nhin. Ta nghin cu vn khi no th cc s dng ny l s nguyn t. Nu k 1 th
2m 1 l s nguyn t khi v ch khi n l s Fermat. H qu l ta bit ch c 5 s nh vy ng vi
m 1, 2, 4,8,16 . S nh nht dng ny m ta cha bit n c phi s nguyn t hay khng l

22 1 . H qu l ta mi ch bit c 4 s c dng 2 2m 1 l s nguyn t ng vi m 1,3,7,15 .


20

m
Tuy nhin ta bit 24 s nguyn t c dng 3 2 1 ng vi

m 1, 2,5, 6, 8,12,18,30, 36, 41, 66,189, 201, 209, 276, 353,


408, 438, 534, 2208, 2816,3168,3189,3912
Ta mi ch bit c 3 s nguyn t c dng 4 2m 1 ng vi m 2,6,14 . Ta bit 17 s nguyn t
m
c dng 5 2 1 ng vi

m 1, 3, 7,13, 15, 25, 39,55, 75,85, 127, 1947,3313, 4687,5947, 13165, 23473
m
Vi mi s t nhin k 3061 ta bit t nht mt s t nhin m m k 2 1 l s nguyn t. Vi
k 3061 th k 2m 1 l hp s vi mi m 17008 (Robinson [2], Cormackand Williams [1],
Baillie, Cormackand Williams [1], Jaeschke [1], Keller [1]). Mt khc c th chng minh tn ti v
m
hn cc s t nhin k m k 2 1 l hp s vi m 1, 2,... (bi tp 3). Vi n 39 v n 207 ta
c 3 2n 2 1| Fn . Vi n 5, 23, 73,125,1945, 23471 ta c 5 2n 2 1| Fn v do 5 2n3 1| Fn vi

n 36 v 3310 . Nu vi cc s c dng k 2m 1 t k m n th ta nhn c cc s Cullen


Cn n 2n 1 (Beeger [2]). A.J.C.Cunningham v H.J.Woodall [1] chng minh rng mi s Cullen
Cn vi 1 n 141 u l hp s v c cc c s nguyn t nh. Tuy nhin C141 l s nguyn t
(Robinson [2]).
Bi tp. 1. Chng minh rng nu m l s t nhin 3 th 2m 1 khng phi ly tha vi s m
ln hn 1 .

236 | c s nguyn t ca cc s Fermat

Chng minh. u tin ta chng minh rng nu m l s t nhin 3 th 2m 1 khng phi bnh
2
phng mt s t nhin. Tht vy nu 2m 1 = n , vi n l s t nhin th n l v > 1 hn na > 3
m
2
v n 3 suy ra m 3 mu thun vi gi thit. Do 2 n 1 n 1 n 1 v vy

n 1 2k , n 1 2mk vi k l s t nhin nm gia 1 v m, k m k . V vy 2mk 2k 2 v l


m
8
v k 1 . Gi s m 3 v 2 1 n vi s l s t nhin 2 . V 2m 1 khng phi bnh phng,
s l. H qu 2m ns 1 n 1 n s 1 n s 2 ... n 1 v l v nhn t th hai l tng ca l s l

nn n cng l s l 1 . iu phi chng minh.


2. Chng minh cc s Fermat m 22 1 n 0,1, 2,..., tha mn m | 2m 2 .
n

n1
2
n
Chng minh. Vi mi s nguyn n 0 ta c n 1 2 suy ra 2 | 2
2n

n1

v h qu l

2n

2
2
22 1| 22 1 v v m 2 1| 2 1 suy ra m | 22 1 vy h qu l m | 2m 2.
n1

Ghi ch. Kt qu trn chng t cc s Fermat l hp s u l s gi nguyn t (Chng 5 mc 7).


k
C th chng minh nu vi s t nhin k s m 2 1 tha mn m | 2m 2 th m l s Fermat
(Jakobczyk [1] trang 122 nh l 10).
n
3. Chng minh rng tn ti v hn s t nhin k m vi mi s k 2 1 u l hp s vi mi s
t nhin n .

Chng minh. Ta bit Fm l cc s nguyn t vi m 0,1, 2,3, 4 . Hn na F5 l tch ca hai s


nguyn t l 641 v p vi p F4 . Theo nh l s d Trung Hoa suy ra tn ti v hn s t nhin
k tha mn

k 1 mod 232 1 641 v k 1 mod p

(18)

Ta s chng minh nu k l s t nhin nh vy v ln hn p th tt c cc s k 2n 1, n 1, 2,...


s
l hp s. Gi s n 2 2t 1 vi s l mt trong cc s 0,1, 2,3, 4 v t l s nguyn ty 0 .
n
Theo (18) ta c k 2 1 2

2s 2t 1

1 mod 232 1 v v Fs | 232 1 v Fs | 22 2t 1 1 suy ra s


s

k 2n 1 chia ht cho Fs v ln hn p Fs , s ny l hp s. t n 25 2t 1 vi t 0,1, 2,...,


25 2t 1

1 mod 641 v v 641| 22 1| 2

25 2t1

1 ta suy ra s k 2n 1
chia ht cho 641 . Nhng s ny ln hn 641 nn n l hp s. By gi xt trng hp n chia ht
6
n
26 t
cho 26 ngha l n 2 t vi t 1, 2,... Theo (18) ta c k 2 1 2 1 mod p . Nhng
n
theo (18) ta c k 2 1 2

p | 22 1| 22 1| 22 t 1 suy ra k 2n 1 chia ht cho p v ln hn p nn n l hp s. Vy


k 2n 1 l hp s vi mi n 1, 2,... (Sierpinski [28] v Aigner [1]).
5

n
4. Tm tt c cc s nguyn t c dng n 1 vi n l s t nhin v khng c qu 300000 ch s.
n
Li gii. Ch c 3 s tha mn. l 11 1 2, 22 1 5, 44 1 257 . Tht vy nu n 1 vi n

k
l s t nhin l s nguyn t th n khng c c s l 1 v do n 2 vi k l s t nhin.
s
Nhng khi nn 1 22 1 suy ra k khng c c s l 1 v do k 2 vi s l s nguyn
0 . V vy nn 1 F2s s . Do vi s 0 ta nhn c F1 5 , vi s 1 l F3 257 , vi s 2 v
k

s 3 th F6 v F11 l hp s, vi s 4 ta nhn c F20 22 210 , nhng s ny c nhiu hn


300000 ch s (Sierpinski [20]).
20

5. Tm tt c cc s nguyn t c dng nn 1 m khng c qu 10


n

616

ch s.

CHNG 10. CC S MERSENNE V CC S FERMAT | 237


1
2
Li gii. Ch c hai s nh vy l 1 1 2, 2 1 17 . Chng minh tng t bi ton trn. u
1

tin ta chng minh nu n 2 v s nn 1 l nguyn t th n 22 vi s l s t nhin. Khi


n

nn 1 F 2s s . Vi s 1, 2 ta nhn c cc hp s F9 , F66 , vi s 3 ta nhn c F2053 c


2
s
n

616
nhiu hn 10 ch s. Vy nu khng tn ti s nguyn t c dng nn 1 vi n 2 th tn ti v
hn s Fermat l hp s.
n

2
6. Chng minh rng trong cc s 2 3, n 1, 2,..., , c v hn hp s.
n

Chng minh. Ta chng minh cc s


vi mi s t nhin

22

2k 1

3 26l 2 3 4 2

k 1

22

3 2l

k 1

22

3 vi k 1, 2,..., u l hp s. Tht vy ta bit

ta u c

22 k 3l 1

vi

l s t nhin. V vy

3 4 3 0 mod 7 . Nhng do vi mi s t nhin k th s

3 7 do n l hp s.

Bi ton hi rng trong cc s c dng 22 3 c v hn s nguyn t vn l cu hi m.


n

2
7. Chng minh rng tt c cc s 2 5, n 1, 2,..., u l hp s.
n

Chng minh. Tt c cc s ny chia ht cho 3.


5. iu kin cn v mt s Fermat l s nguyn t

nh l 5. S Fermat Fn vi n l s t nhin l s nguyn t khi v ch khi Fn | 3

Fn 1 /2

1.

Chng minh. K hiu n l s t nhin.

Gi s Fn | 3

Fn 1 /2

1 th Fn khng chia ht cho 3. Gi s p l c s nguyn t (khc 3) bt k

2
k
ca Fn . K hiu l cp ca 3 mod p . V p | 3Fn 1 1 suy ra | Fn 1 22 . Nu 2 th 2
n

k
2
vi k l s nguyn khng m 2n . H qu l 2 | 2

do p | 3 1, p | 3

Fn 1 /2

1 nn v p | Fn ta c p | 3

n1

Fn 1 /2

Fn 1 / 2 do | Fn 1 / 2 v v vy
1 suy ra p | 2 vy p 2 . iu ny l v l

2
v p | Fn v Fn l. Vy 22 . Nhng ta bit | p 1 suy ra p 2 k 1 vi k l s t nhin.
n

T y suy ra p 2 1 v v p | Fn nn ta c Fn p chng t Fn l s nguyn t. iu kin


c chng minh. chng minh iu kin cn ta chng minh b sau y
2n

p 1 /2

B . Nu p l s nguyn t c dng 12k 5 th p | 3

1 .

Chng minh b . Nu p l s nguyn t v p 12k 5 th theo tnh cht ca k hiu Legendre

3 p
p 1
ta c 1 suy ra theo tnh cht 5 ca k hiu Legendre th 1 v h qu l
3 3
p 3
3
p 1 /2
p 1 /2
1 . iu phi chng minh.
1 mod p . Vy p | 3
1 do 3
p
n
Xt s t nhin n . S Fn 22 1 c dng 12k 5 v vi mi s t nhin n ta c 2 2m v d
n

m
dng chng minh bng quy np 4 4 mod12

vi mi m 1, 2,... v h qu l

Fn 4m 5 5 mod12 ngha l Fn 12k 5 v nu Fn l s nguyn t th theo b


Fn | 3

Fn 1 /2

1 . nh l c chng minh.

238 | iu kin cn v mt s Fermat l s nguyn t

T nh l 5 suy ra nu Fn l s nguyn t th 3 l cn nguyn thy ca Fn . Chng minh iu ny


nhn c vi lu l 3 c cp Fn 1mod Fn c suy trc tip t chng minh nh l 5. p dng
nh l 5 ta c th tc hu hiu kim tra xem mt s Fermat Fn c l s nguyn t hay khng.
K hiu t l phn d nhn c khi chia s nguyn t cho Fn v t

r1 3,

rk 1 rk2 ,

k 1, 2,...
k 1

n
Bng quy np ta chng minh c Fn | 32 rk vi mi k 1, 2,... V vy vi k 2 ta c

Fn | 3

F n 1 /2

r2n . T y suy ra 3

Fn 1 /2

1 ng d vi r2n 1mod Fn .

Bng phng php ny ta chng minh c cc s F7 , F8 , F13 v F14 l hp s. S F7 c 39 ch s


v vy tm c s r27 1 r128 1 bng cch p dng th tc trn ta cn tnh vi trm php
tnh vi ba mi bnh phng cc s t nhin, mi s c t hn 39 ch s. Hn na cc bnh
phng ny cn phi em chia cho F7 (c 39 ch s). S phc tp ca cc php tnh ny khng
phi vn ln i vi cc my tnh din t, nhng vo nm 1905 (ngha l khi Morehead tm
c kt qu ny) th lng tnh ton th cng l v cng ln.
Phng php tng t cng c p dng ch ra F8 , F13 v F14 l hp s.
Phng php trnh by trn khng cho thm thng tin v cc c s nguyn t ca cc s c
xt cng nh cch phn tch chng thnh tch ca hai tha s 1 . y l l do v sao chng ta cha
c nhng phn tch thnh tha s ca F14 .
S Fermat tip theo l F20 c hn 300000 ch s, cc tnh ton m t trn trong trng hp ny
yu cu hn mt triu php chia cc s c hn vi trm nghn ch s, mi s u c nhiu hn
300000 ch s.
Bi tp. Tm c s nguyn t nh nht ca 12

215

1.

1 u c dng 216 k 1 vi k l s t
15
16
nhin. H qu l p 2 1 F4 . V F4 l s nguyn t nn theo nh l 5 ta c F4 | 32 1 . V vy
Li gii. Theo nh l 4 th cc c s nguyn t ca 12

215

32 1 mod F4 . Nhng theo nh l nh Fermat ta c 22 2F4 1 1 mod F4 suy ra


15

16

42 1 mod F4 . V vy 122 32 42 1 mod F4 do F4 |122 1 . Vy ta thy F4 l c s


15

nguyn t nh nht ca 12

15

215

15

15

15

1 , hn na s ny F4 v v vy n l hp s.

2
Ta cha bit c tn ti v hn hp s c dng 12 1 vi n 1, 2,..., hay l trong s c v hn
cc s nguyn t hay khng.
n

CHNG 11
BIU DIN CC S T NHIN THNH TNG CC LY THA BC k KHNG M
1. Tng ca hai bnh phng
nh l 1. S t nhin n l tng ca hai bnh phng khi v ch khi trong phn tch thnh tha s
nguyn t ca n th cc s nguyn t c dng 4k 3 u c s m l.
B . Mi s nguyn t l p l c s ca tng hai bnh phng nguyn t cng nhau u c dng
4k 1 .
Chng minh b . Gi s a, b l 2 s nguyn nguyn t cng nhau v s nguyn t l p tha
p 1/2

2
mn p | a 2 b2 . Th th a 1

a, b 1 ,

b p 1 mod p . Nhng

cc s a, b khng chia ht cho p , suy ra theo nh l Fermat nh ta c

a p 1 b p 1 1 mod p ; h qu l

p 1 / 2

p 1/2

b p 1 mod p ; suy ra a p 1 1

p 1 /2

1 mod p , m p 2 suy ra

p1 /2

1 vy

chn nn p c dng 4k 1 .

Chng minh nh l. Gi s n l tng hai bnh phng

n a 2 b2

(1)

Xt phn tch thnh tha s nguyn t

n q11 q22 ... qss

(2)

Gi s p l c s nguyn t c dng 4k 3 ca n . K hiu d a, b , a da1 , b db1 vi

a1 , b1 1 . Theo (1) th d 2 | n , v do n d 2n1 , vi n1

l s t nhin. Gi s s m ca p trong

phn tch (2) l l th v n d n1 ta c p | n1 a b , mu thun vi b . Ta chng minh


xong iu kin cn.
2

2
1

2
1

1
2
chng minh iu kin ch rng khng gim tng qut c th gi s n 1 . Ta c 1 1 0 .
Gi s (2) l phn tch thnh tha s nguyn t ca n . K hiu m l s t nhin nh nht m bnh
2
phng ca n l c s ca n . Khi n m k , vi k bng 1 hoc l tch ca cc s nguyn t
khc nhau m khng c s no c dng 4k 3 . V 2 12 12 v theo nh l 9 Chng 5 th cc s
nguyn t ny l tng ca hai bnh phng.

2
2
T ng thc a b

d 2 ab cd ad bc suy ra tch ca hai (v do hu hn) s


2

t nhin m mi s l tng hai bnh phng th cng l tng ca hai bnh phng. H qu l k l
2
2
2
tng hai bnh phng. Do k u v , suy ra n m k mu mv .

iu kin c chng minh.


Mt vn c t ra l c bao nhiu cch biu din mt s t nhin thnh tng hai bnh phng
nh vy. Cu tr li c trong Chng 8 mc 9.
H qu. S khng phn tch c thnh tng hai bnh phng nguyn th cng khng phi tng hai
bnh phng hu t.
Chng minh. Nu n l s t nhin khng phi tng hai bnh phng th theo nh l 1 suy ra
trong phn tch thnh tha s nguyn t ca n c s nguyn t p c dng 4k 3 vi s m l. Gi

1 l
s n 1
m m1
2

mm1

vi m, m1 l cc s t nhin v l , l1 , l cc s nguyn. Khi

n lm1 l1m . Nhng p phi c s m l trong phn tch thnh tha s nguyn t ca
2

240 | Tng ca hai bnh phng

v tri ca ng thc v do theo nh l 1 th v tri khng th phn tch thnh tng hai bnh
phng nh trong v phi. Mu thun suy ra h qu c chng minh.
E.Landau [1] chng minh rng nu f x k hiu s cc s t nhin x l tng ca hai bnh
phng th f x :

x
tin ti mt gii hn dng hu hn khi x tng v hn.
log x

Biu din n x 2 y 2 , vi x, y l cc s nguyn 0 x y , v n 10000 , c trnh by bi A.van


Wijngarden [1]. S cch phn tch n thnh tng hai bnh phng vi n 20000 c trnh by
bI H.Gupta [2]. Vi mi s nguyn t p 10007401 c dng 4k 1 th bng cc phn tch nh vy
trnh by bi Kogbetlianz v Krikorian [1].
Bi tp. 1. Tm iu kin cn v s hu t l / m c th biu din thnh tng bnh phng hai
s hu t.
Li gii. iu kin cn v l lm l tng ca hai bnh phng nguyn.
2

l l1 l2
2
2
2
th lm m1m2 mm2l1 mm1l2 .
Lu nu
m m1 m2
2

2
2
Mt khc lm a b suy ra

l a b

m m m

Ghi ch. T bi tp 1 v nh l 1 suy ra phn s ti gin l / m , vi l , m l cc s t nhin, l tng


ca hai bnh phng hu t khi v ch khi mi s l , m u l tng ca hai bnh phng nguyn.
2. Chng minh rng nu s hu t r 0 l tng hai bnh phng hu t th c v hn cch biu
din s nh l tng hai bnh phng hu t dng.
2
2
Chng minh. Gi s r a b , vi a, b l cc s hu t khc 0 . Khng gim tng qut c th gi

k 2 1 a 2kb k 2 1 b 2ka


l
r

s a, b dng v a b . Vi mi s t nhin k ta c

k 2 1
k 2 1

mt biu din ca r thnh tng hai bnh phng cc s hu t. Nu k 3 , ta c


k 2 1 4 b
3k 2 8k 3k k 3 3 3 , suy ra

v do n ak2 bk2 , k 3, 4,..., m 2 ,
2k
3 a
2
k 1 a 2kb 0 . Hn na d dng chng minh dy a tng theo k . V vy cc s a l
ak
k
k
k 2 1
phn bit v vi k 3 th cc s ny dng. Vy vi k 3 ta c th biu din r thnh tng hai
bnh phng hu t dng khc nhau. Ta thy r c v hn cch biu din nh vy. By gi gi s
r a 2 , vi a l s hu t. V r 0 , ta c th gi s a 0 . Vi s t nhin k ta c
2

k 2 1 a 2ka 2
2 . R rng ak k 2 1 a / k 2 1 tng theo k . H qu l tn ti v hn
r 2
k 1 k 1

biu din ca r thnh tng hai bnh phng hu t dng.


2

3. Gi s m l s t nhin. Tm s t nhin n c t nht m biu din thnh tng hai bnh phng
t nhin.

2
2
2
2
Li gii. t n a , vi a 3 1 4 1 ... m 2 1 . S a / k 1 l t nhin vi mi

k 3, 4,..., m 2 . H qu l cc s ak

k 2 1
2ka
a; nk 2
(k 3, 4,..., m 2) cng l s t nhin.
2
k 1
k 1

CHNG 11. BIU DIN CC S T NHIN THNH TNG CC LY THA BC k KHNG M | 241
2

k 2 1 2ka
k 2 1
a 2
a

a,
Nhng ta c ng thc a 2
.
Nu

k
k 2 1
k 1 k 1
2

bk

2ka
k 2 1

ta c

k 2 1 2

k 2 2k 1
2
2
2
a
a 0 vi
n a ak bk vi k 3, 4,..., m 2 . Nhng ta li c ak bk
k 2 1
k 2 1
2a
3, 4,..., m 2 v ak a 2
suy ra a3 a4 ... am2 .
k 1
2

V vy cc biu din n ak2 bk2 , k 3, 4,..., m 2 , u khc nhau v c m biu din tt c. Do n


c tnh cht yu cu. Cng lc ta chng minh c vi mi s t nhin m th tn ti t nht m
tam gic Pythagoras phn bit c tnh cht trong gi thit.
4. Cho trc biu din n thnh tng hai bnh phng. Tm biu din tng t ca 2n .
2
2
Li gii. Nu n a b th 2n a b a b
2

2. S cch biu din thnh tng hai bnh phng


Ta tnh s cch biu din mt s t nhin thnh tng ca hai bnh phng. Nu n c th biu din
thnh tng ca hai bnh phng ngha l

n x2 y 2

(3)

2
th n x v n y 2 , suy ra x n , y n . V vy ch cn thay cc gi tr ca m | x | khng

vt qu

n vo (3) v th xem khi no n x 2 l bnh phng ng. Nu n x 2 l bnh phng

th t y n x 2 v ta nhn c mt biu din ca n thnh tng hai bnh phng. Nu vi


2
mi x th n x u khng phi bnh phng th biu din nh vy l khng tn ti. Ta ch cn xt
2
trng hp x dng v du ca x khng nh hng ti gi tr n x .

Lu rng dy n, n 12 , n 22 , n 32 ,... c hiu lin tip l 1,3,5,..., ngha l dy cc s t nhin l


lin tip. V d vi n 10 ta c dy 10,9,6,1 . Phn t th hai ca dy l bnh phng ng v vy
ta c x 1, y 3, hoc x 3, y 1 . C tt c 8 biu din cn tm. l

Xt n 25 . Ta c dy 25, 24, 21,16,9,0 . y 25,16,9, 0 l cc bnh phng. V vy ta c 12 biu


din tha mn.
K hiu n l s tt c cc biu din ca s t nhin n thnh tng hai bnh phng. Hai biu
din l khc nhau ngay c khi chng sai khc mt hon v. Ta c

Trong mc 5 Chng 5 ta bit mi s nguyn t c dng 4k 1 u c th biu din duy nht


(sai khc mt hon v) thnh tng ca hai bnh phng. Suy ra vi mi s nguyn t p c dng

4k 1 th ta c p 8 .
Lp lun trn suy ra vi mi s t nhin n ta c n 4 n .
Bi tp 3 mc 1 suy ra khng c chn trn ca n .
By gi ta tnh tng

242 | S cch biu din thnh tng hai bnh phng

T n 1 2 ... n

(4)

S k l s nghim nguyn ca phng trnh x 2 y 2 k v vy T n l s nghim ca

0 x2 y 2 n

(5)

Ta chia cc nghim ca (5) thnh cc lp m hai nghim thuc cng mt lp khi v ch khi cc gi
tr x l bng nhau. Ta tnh s nghim trong mi lp. Nu x 0 th theo (5) y c th nhn cc gi
tr nguyn m y 2 n , ngha l y n . C 2 n kh nng nh vy. Nu x k 0 th theo (5)
2
ta phi c k 2 n ; do k n v y 2 n k 2 , suy ra y n k 2 . C 1 2 n k s nh

vy (1 c th thm vo khi y 0 ). V k nhn mi gi tr 1, 2,..., n v cc du khng nh



2
hng ti gi tr ca k , nn
n

2 n 2 1 2 n k 2

k 1

4 n 4 n k 2

k 1

do
n

T n 4 n k 2

k 0

(6)

V vy chng hn vi n 100 ta c

Tng (4) c m t hnh hc kh n gin. Ta bit 1 T n l s


cc cp s nguyn m x 2 y 2 n , s ny bng vi s cc im c
ta nguyn trong mt phng (im nguyn) cha trong hnh trn
C vi tm l 0, 0 v bn knh n . By gi vi mi im nguyn
ta xt hnh vung nhn n l tm v cc cnh song song vi cc trc
ta v din tch l 1. Din tch P b ph bi cc hnh vung nhn
c t cc im nguyn khng nm bn ngoi C bng vi s im
, tc l 1 T n . Hnh trn C1 vi tm 0, 0 v bn knh

1
cha cc im b ph bi hnh vung ng vi cc im
2
ca hnh trn C . V 1/ 2 l khong cch ln nht gia mt im nm trong hnh vung din tch
n

1 ti tm ca n do din tch P nh hn din tch ca hnh trn C1 . V vy P n


.
2

1
Mt khc din tch ca hnh trn C2 vi tm 0, 0 v bn knh n
l nh hn P , nn
2
2

P n
. V vy t ng thc P 1 T n suy ra
2

(7)

1
1

n
1 T n n
1
2
2

CHNG 11. BIU DIN CC S T NHIN THNH TNG CC LY THA BC k KHNG M | 243

Lu 2 5 v vi mi s t nhin n, 0

1
1 1 n . V vy
2

1
1

n
1 n 2 n 2 1 n 6 n
2

1
1

n
1 n 2 n 2 1 n 6 n
2

Theo (7) ta c n 6 n T n n 6 n , suy ra T n n | 6 n vi mi s t nhin n , suy


ra

T n
6

n
n

(8)

1 2 ... n

ngha l gi tr trung bnh ca hm n l .


n
Do l gi tr trung bnh ca s cch biu din mt s t nhin thnh tng hai bnh phng.
Ta c T 100 316 ngha l mt s t nhin khng ln hn 100 c trung bnh 3.16 cch biu
T (8) v (4) suy ra lim
n

din thnh tng hai bnh phng. Tng t theo (6) ta tnh c T 400 1256 suy ra

T 400 / 400 3.14 v T 1000 3148 suy ra T 1000 /1000 3.148 . Theo (6) T n c th tnh
vi mi n (cc tnh ton c th rt di), v t (8) ta c phng php tnh vi sai s cho trc.
T (8) ta c T n n 6 n vi mi s t nhin n .
Nm 1906 ti (Sierpinski) s dng phng php ca Voronoi ch ra tn ti hng s A tha
mn T n n A 3 n (Sierpinski [1]). Kt qu mnh hn c tm ra bi Van der Corput v
nhng ngi khc. Kt qu tt nht ti nay c trong Ivic [1].
trn ta tnh s im nguyn cha trong hnh trn tm 0, 0 . Nm 1957, H.Steinhaus [1] t
ra bi tp sau: chng minh rng vi mi s t nhin n th lun tn ti hnh trn cha ng n im
1

nguyn. Ta s chng minh nu p 2, th vi mi s t nhin n lun tn ti hnh trn Cn vi


3

tm p cha ng n im nguyn bn trong.


Tht vy, gi s hai im x1 , y1 v x2 , y2 phn bit v cch u p .

Th th x1 2

y1 x2 2
3

Suy ra 2 x2 x1 2 x22 y22 x12 y12


V

y2 .
3

2
y1 y2
3

2 l s v t, x1 x2 0 , suy ra y22 y12

2
2

y1 y2 0 , v do y2 y1 y2 y1 3 0 .
3

2
0 v y1 v y2 l cc s nguyn, h qu l y2 y1 0 . Suy ra x1 x2 v y1 y2
3
mu thun vi gi thit cc im ny l phn bit.
Nhng y2 y1

K hiu n l s t nhin ty . R rng mi hnh trn C vi tm p v bn knh ln cha nhiu


hn n im nguyn. Hn na s im nguyn cha trong C l hu hn. trn ta chng minh

244 | S cch biu din thnh tng hai bnh phng

khong cch t p ti cc im nguyn u phn bit, nn ta c th sp xp cc im nguyn nm


trong C theo dy p1 , p2 ,..., pn , pn1 ,... ng vi cc khong cch tng dn t cc im ti p .
K hiu Cn l hnh trn tm p v bn knh bng khong cch t pn 1 ti p . R rng cc im
nguyn cha trong Cn l p1 , p2 ,..., pn . Do hnh trn Cn c tnh cht yu cu. nh l Steinhaus
c chng minh.
C th chng minh khng c im no trong mt phng vi ta hu t c tnh cht: vi mi s
t nhin n th lun tn ti hnh trn nhn im lm tm v cha ng n im nguyn (Sierpinski
[19] trang 26). Mt khc c th chng minh vi mi s t nhin n th lun tn ti hnh trn c tm
c ta hu t v cha ng n im nguyn bn trong. H.Steinhaus chng minh rng vi mi
s t nhin n th lun tn ti hnh trn vi din tch n cha ng n im nguyn bn trong. Tuy
nhin chng minh iu ny kh kh. C th chng minh rng vi mi s t nhin n th lun tn ti
hnh vung cha ng n im nguyn bn trong (Sierpinski [19] trang 28-30).
Vi mi s t nhin n th trong khng gian lun tn ti hnh cu cha ng n im c ta
nguyn. chng minh iu ny ch cn lu rng nu u, v, w l cc s hu t tha mn

u 2 v 3 w 5 th u v w 0 , v hnh cu c tm l

2, 3, 5 vi bn knh 3 cha t

nht mt im nguyn. T hai nhn xt ny chng minh c quy v trng hp hnh trn trn
mt phng. J.Browkin chng minh vi mi s t nhin n th lun tn ti hnh lp phng
(trong khng gian ba chiu) m cha ng n im nguyn.
A.Schinzel [7] chng minh rng vi mi s t nhin n th lun tn ti hnh trn m trn bin ca
n c ng n im nguyn. C th nu n l, ngha l n 2k 1 , vi k l s nguyn khng m th
1
k
hnh trn vi tm , 0 v bn knh 5 / 3 c tnh cht yu cu. Nu n chn, ngha l n 2k , vi
3

1
k l s t nhin th hnh trn vi tm , 0 v bn knh 5 k 1/2 / 2 c tnh cht yu cu.
2
T.Kulikowski [1] chng minh vi mi s t nhin n th lun tn ti hnh cu trong khng gian
ba chiu m trn bin ca n cha ng n im nguyn. ng ta cng m rng nh l ny cho cc
hnh cu chiu cao.
Cc im hu t (ngha l cc im c ta l cc s hu t) trn bin ca hnh trn cng
c nghin cu. Tn ti ng trn m khng cha im hu t no chng hn x 2 y 2 3 . C

cc ng trn i qua ng 1 im hu t chng hn ng trn x 2

y 2
2

4 ch i

qua duy nht mt im hu t l 0, 0 . C ng trn i qua ng hai im hu t chng hn

x2 y 2

3 ch i qua cc im hu t 1, 0 v 1, 0 . Tng qut hn ta chng minh nu

c ba im hu t trn bin mt hnh trn th c v hn im hu t trn . D dng chng minh


rng nu c ba im hu t trn bin hnh trn th hnh trn c tm l im hu t v bnh
phng bn knh ca n cng l hu t. Khng gim tng qut c th gi s tm ca hnh trn l
0, 0 . K hiu hnh trn l C . Khng kh chng minh c nu ng trn C cha t nht mt
im hu t th n cha v hn im hu t. Tht vy nu a, b l cc s hu t tha mn

a 2 b2 r 2 th vi mi s hu t t im x, y vi x

2at b 1 t 2
1 t2

, y

a 1 t 2 2bt
1 t2

l im

hu t v x 2 y 2 r 2 .
Vy vi mt ng trn cho trc th ch c cc kh nng sau: n khng cha im hu t no;
hoc cha ng mt im hu t; hoc cha ng hai im hu t; hoc cha v hn im hu t.
C th chng minh rng trong trng hp cui th tp hp cc im hu t tr mt trn ng
trn, ngha l vi mi cung ca ng trn th u c t nht mt im hu t no .

CHNG 11. BIU DIN CC S T NHIN THNH TNG CC LY THA BC k KHNG M | 245

Sierpinski [21] chng minh nu r 2 l s hu t th ung trn C vi bn knh r cha v hn


im m khong cch gia chng i mt l hu t. T y suy ra vi mi s t nhin n th lun
tn ti ng trn cha n im m khong cch i mt gia chng u l s t nhin.
Mt h qu quan trng khc l vi mi s t nhin n th lun tn ti tp hp n im m khng c
ba im no thuc cng mt ng thng v khong cch i mt gia chng l s t nhin. nh
l ny ln u c chng minh bi N.H.Anning v P.Erdos. Chng minh ca h khng ging
trn (Anning v Erdos [1], Hagwiger [1] trang 85). Cc tc gi ny chng minh rng nu trong
mt tp v hn cc im trn mt phng m khong cch i mt gia cc im l nguyn th
tt c cc im thuc mt ng thng (Erdos [6] v Trost [2]).
Bi tp. Chng minh rng tp hp cc im hu t trn mt phng c th chia thnh hai tp hp,
mt tp c hu hn im chung vi mi ng k ngang v tp kia c hu hn im chung vi
mi ng k dc.

l r
Chng minh. iu kin c tha mn vi tp th nht bao gm tt c cc im c dng , ,
m s
cc phn s l ti gin v t s nguyn, mu s t nhin, chng tha mn l m r s . Tp th
hai l cc im cn li.
C th chng minh tp hp cc im hu t trong khng gian ba chiu c th chia thnh ba phn
m mi phn ch c giao hu hn vi mi ng thng song song vi cc trc ta . Mnh cho
tp hp tt c cc im trong khng gian ba chiu tng ng vi gi thuyt continuum (xem
Sierpinski [13] v [22] trang 397).
3. Tng ca hai bnh phng cc s t nhin
nh l 2. S t nhin n l tng ca hai bnh phng cc s t nhin khi v ch khi mi c s
nguyn t c dng 4k 3 ca n c ly tha chn trong phn tch thnh tha s nguyn t ca n v
hoc l 2 c ly tha l trong phn tch hoc l n c t nht mt c s nguyn t c dng 4k 1 .
Chng minh. Gi s tn ti s t nhin l tng hai bnh phng cc s t nhin v c tnh cht: s
khng c c s nguyn t c dng 4k 1 , v trong phn tch thnh tha s nguyn t ca s
th 2 c ly tha chn 0 . Gi s n l s t nhin nh nht c cc tnh cht ny.
V n l tng hai bnh phng cc s t nhin nn theo nh l 1 tt c cc c s nguyn t ca n
m c dng 4k 3 th c ly tha chn trong phn tch thnh tha s nguyn t ca n . H qu l
n 22k m2 vi m l s t nhin l v s nguyn k 0 . V vy ta c th vit 22k m2 a 2 b2 , vi
a, b l cc s t nhin.
Nu k 0 th v tri ca ng thc ny chia ht cho 4, suy ra a, b cng chn, ngha l
a 2a1, b 2b1, v vy 22( k 1) m2 a12 b12 m, mu thun vi nh ngha ca n . Do k 0 v

n m2 a2 b2 1 . Cc s a, b nguyn t cng nhau v nu a, b d 1 ta c a da2 , b db2 ,

vi a2 , b2 l cc s t nhin suy ra m dm1 v m12 a22 b22 m2 n , mu thun vi nh ngha


ca n . Vy a, b 1 .

Nhng v m l v 1 (khng c c s nguyn t dng 4k 1 ), nn n c c s nguyn t dng


p 4k 3 . Suy ra p a 2 b 2 nn a 2 b2 (mod p) . Ly tha c hai v ln 2k 1 v lu

2(2k 1) p 1 , s dng nh l nh Fermat ta nhn c 1 1

2 k 1

mod p , v l.

Vy ta chng minh c mi s t nhin l tng hai bnh phng cc s t nhin th c tnh


cht l hoc trong phn tch thnh tha s nguyn t ca n s nguyn t 2 c ly tha l hoc n
c c s dng 4k 1 . Kt hp vi nh l 1 suy ra iu kin cn c chng minh.
By gi ta chng minh iu kin . Gi s s t nhin n tha mn cc iu kin trong nh l. Ta
2

2
c hoc n 2m hoc n 2 m l , vi 0 hoc 1 v l l tch cc c s nguyn t c dng

246 | Tng ca hai bnh phng cc s t nhin


2
2
2
4k 1 . Nu n 2m th n m m , v do n l tng hai bnh phng cc s t nhin. Gi s
n 2 m2l vi l l tch cc s nguyn t c dng 4k 1. Theo nh l 9 Chng 5 th mi mt
nhn t u l tng ca hai bnh phng dng. Nhng tch ca hai s l m mi s l tng ca hai
bnh phng dng li l tng ca hai bnh phng dng. Bi v nu n1 a 2 b2 , n2 c 2 d 2 ,

vi n1 , n2 l th mt trong cc s a , b , gi s l a , l l v s kia chn, cng vy vi c, d , gi s c

2
2
2
2
l v d chn. Khi n1n2 a b (c d ) ad bc ac bd vi ac bd l v do
2

0 . V vy n1n2 l tng ca hai bnh phng cc s t nhin. Vy l l tng hai bnh phng cc s
2
2
t nhin, ngha l l a b suy ra m2l ma mb v 2m2l ma mb ma mb v
2

2
2
ma mb 0 (v a khc b do l a b l). Vy mi s n l tng hai bnh phng cc s t
nhin. iu kin c chng minh.

nh l 2 c chng minh.
2
T nh l 2 suy ra cc s n l tng ca hai bnh phng cc s t nhin khi v ch khi n c t
nht mt c s nguyn t c dng 4k 1 . Hoc ni cch khc s t nhin n l cnh huyn ca
mt tam gic Pythagoras khi v ch khi n c t nht mt c s nguyn t c dng 4k 1 . Xem
thm h qu bi tp 3 mc 1.

Bi tp. 1. Chng minh rng s t nhin n l tng hai bnh phng cc s t nhin khc nhau khi
v ch khi (i) cc s nguyn t c dng 4k 3 xut hin trong phn tch thnh tha s nguyn t
ca n u c ly tha chn; (ii) s n c t nht mt c s nguyn t c dng 4k 1 .
Chng minh. S cn thit ca iu kin (i) suy ra t nh l 1. Gi s s t nhin n khng tha
2
2
mn iu kin (ii) ngha l n khng c c s nguyn t dng 4k 1 . Suy ra nu n a b vi
cc s t nhin khc nhau a, b th vi d a, b ta c n d 2 a12 b12 . Vi a da1 , d bd1 v

a1 , b1 l cc s nguyn t cng nhau v phn bit. S a12 b12 khng c c s nguyn t c dng
4k 1 v do v a1 b1 1 nn lp lun tng t trong chng minh nh l 2 ta suy ra s

a12 b12 khng c c s nguyn t c dng 4k 3 . V vy a12 b12 s 2 vi s l s t nhin 1 v

a1 , b1 l cc s t nhin khc nhau. H qu l s a12 b12 chia ht cho 4 v suy ra cc s a1 , b1 u l

chn, mu thun v a1 , b1 1 .

2
2
By gi gi s s t nhin n tha mn cc iu kin (i), (ii) th theo nh l 2 ta c n a b vi
a, b l cc s t nhin. Nu a b th n 2a 2 v v n tha mn (ii) nn n c c s c dng
2
2
2
4k 1 do a l cnh huyn ca tam gic Pythagoras. Ngha l a c d vi c, d l cc s t

2
2
nhin. R rng c d v nu c d th a 2c m

2 l s v t nn suy ra v l. Vy

n 2a c d c d vi c d 0 v c d c d (v d l s t nhin). H qu l n l tng
2

ca hai bnh phng cc s t nhin phn bit. iu kin c chng minh.


2. Chng minh rng s t nhin n l tng hai bnh phng cc s t nhin nguyn t cng nhau
khi v ch khi n khng chia ht cho 4 v cng khng chia ht cho cc s t nhin c dng 4k 3 .
Chng minh. Gi s s t nhin n l tng hai bnh phng cc s t nhin nguyn t cng nhau,
2
2
ngha l n a b . Nu n 4k th cc s a, b cng chn, mu thun vi a, b 1 . Nu n c c
s c dng 4k 3 th n c c s nguyn t cng c dng , m theo chng minh nh l 2 th
c s ny khng phi c s ca tng hai bnh phng cc s t nhin nguyn t cng nhau. V
vy iu kin trn l cn thit.
2
2
Gi s s t nhin n tha mn iu kin c t ra. Nu n 2 th n 1 1 . Nu n 2 th t
iu kin suy ra n l tch ca cc s nguyn t c dng 4k 1 hoc l tch ca 2 vi cc s nguyn
t c dng 4k 1 . Trong trng hp n l v cc c s nguyn t ca n u l tng hai bnh

CHNG 11. BIU DIN CC S T NHIN THNH TNG CC LY THA BC k KHNG M | 247

phng cc s t nhin nguyn t cng nhau th theo b 2 v bi tp 8 mc 5 Chng 5 th


bng quy np n gin ta thy n l tng hai bnh phng cc s t nhin nguyn t cng nhau.
Trong trng hp cn li, ngha l nu n l tch ca 2 v cc s nguyn t c dng 4k 1 th ta c
2
2
n 2 a 2 b2 vi a, b l hai s t nhin nguyn t cng nhau. V a b l nn mt trong hai s

a, b c mt s l v s kia chn. Ta c n a b a b vi a b v a b l cc s t nhin


2

l, hn na chng nguyn t cng nhau v nu d a b v d a b vi d l s t nhin th d 2a v

d 2b ; v d l c s ca s l a b nn n cng l. Ta c d | a v d | b ,m a, b 1 suy ra d 1 .
Vy a b, a b 1 . iu kin c chng minh.
4. Tng ca ba bnh phng
l
nh l 3. S t nhin n l tng ca 3 bnh phng ch khi n khng c dng 4 8k 7 , cc s

nguyn k , l 0 .
l
Chng minh. Gi s tn ti s t nhin c dng 4 8k 7 , cc s nguyn k , l 0 , l tng ba bnh
l
2
2
2
phng. K hiu n l s nh nht nh vy. Khi ta c n 4 8k 7 a b c vi cc s

nguyn a, b, c . Nu trong cc s a, b, c c ng mt s l th tng ca cc bnh phng ny c dng


4t 1 v khc n . Nu hai trong s cc s a, b, c l l th tng cc bnh phng ca chng c dng
4t 2 , do khc n . Nu c ba s l l th tng cc bnh phng ca chng c dng 8t 3 v do
khc n . H qu l cc s a, b, c u chn. t a 2a1, b 2b 1, c 2c1, vi a1 , b1 , c1 l cc s

t 1
2
2
2
nguyn. Vy 4 8k 7 a1 b1 c1 , mu thun vi nh ngha ca n . Do cc s t nhin
l
khng c dng 4 8k 7 , k , l l cc s t nhin khng m, khng th l tng ca 3 bnh phng

cc s nguyn. nh l 3 c chng minh.


C th chng minh iu kin trong nh l 3 cng l iu kin mt s n l tng ca ba bnh
l
phng. Gauss l ngi u tin chng minh rng mi s t nhin khng c dng 4 8k 7 , k
v l l cc s nguyn khng m, l tng ca ba bnh phng cc s nguyn. Li gii gc ca Gauss
c ci tin bi Lejeune Dirichlet v Landau (Landau [2] tp 1 trang 114-125). Gn y
N.C.Ankeny [1] trnh by mt chng minh s cp cho nh l Gauss. Chng minh ny da trn
nh l Minkowski lin quan ti cc im nguyn cha trong mt hnh li v nh l v cp s cng
(Mordell [7], Wojcik [1]). H qu trc tip ca nh l Gauss l ta c th suy ra mi s t nhin c
dng 8k 3 u l tng ca ba bnh phng cc s nguyn, cc s ny l l.
V vy 8k 3 (2a 1) 2 (2b 1) 2 (2 c 1) 2, vi a, b, c l cc s nguyn khng m v do

a(a 1) b(b 1) c(c 1)

t a tb t c ,
2
2
2

nn ta thy nh l Gauss suy ra nh l sau y: mi s t nhin u l tng ca ba (hoc t hn)


cc s tam gic. nh l ny c t ra ln u bi Fermat.
i vi cc s c dng 8k 1 , B.Jones v G.Pall [1] ch ra ch tr 1 v 25 th tt c u l tng ca
10
ba bnh phng cc s t nhin. i vi cc s dng 8k 5 nh hn 5.10 th ch c cc s 5, 13,
37 v 85 l khng phi tng ca ba bnh phng cc s t nhin. Khng c s no c dng 8k 7
l tng ca ba bnh phng cc s nguyn (v do khng l tng ba bnh phng cc s t
nhin).
S c dng 4k l tng ca ba bnh phng cc s t nhin khi v ch khi k cng c tnh cht .
2
2
2
Tht vy nu 4k a b c vi a, b, c l cc s t nhin th a, b, c chn do
a 2a1, b 2b1, c 2c1 vi a1 , b1 , c1 l cc s t nhin. V vy k a12 b12 c12 . Ngc li ng thc

248 | Tng ca ba bnh phng

cui cng suy ra 4k 2a1 2b1 2c1 . Suy ra khng c s no c dng 2n , n 1, 2... l tng
2

ca ba bnh phng cc s t nhin. Nhng 8.3n2 (2n)2 (2n)2 (4n)2 v do c v hn s c


dng 8k l tng bnh phng ca ba s t nhin v c v hn s khng c tnh cht .
i vi cc s c dng 8k 2 , G.Pall [1] t ra gi thuyt ni rng mi s c dng 2 8n 1
ngoi tr 2 u l tng ca ba bnh phng cc s t nhin. A.Schinzel [1] lu rng gi thuyt
ny l sai. S 2 8.8 1 130 khng phi tng ba bnh phng cc s t nhin (bi tp 1). Khng
10
c v d khc nh hn 5.10 trong khi i vi cc s 2 8n 5 th c hai v d l 10 v 58.

Cc s c dng 8k 4 l tng ca ba bnh phng cc s t nhin khi v ch khi cc s 2k 1


tng ng cng c tnh cht . H qu l s 8 4k 3 4 4 8k 7 , k 0,1, 2,.. khng phi tng
ca ba bnh phng cc s t nhin. Mt khc cc s 8(4k 1) 4 4 8k 3 , k 0,1, 2,... u l
tng ca ba bnh phng cc s t nhin. Mi s c dng 8k 6 u l tng ca ba bnh phng
cc s t nhin v theo nh l Gauss th cc s l tng bnh phng ca ba s nguyn, hn na
n khng phi tng bnh phng ca hai s nguyn v 8k 6 2 4k 3 .
T cc kt qu v cc s idonei trong Chng 5 mc 6 vi l thuyt cc dng bc hai suy ra c
nhiu nht mt trong cc s c dng 8k 2 hoc 8k 5 ln hn 130 l tng bnh phng ca ba
s t nhin (Grosswald v Calloway [1], Schinzel [10]).
K hiu 3 n l s cc biu din khc nhau mt s n thnh tng ca ba bnh phng cc s
nguyn. Vi n 10 ta c 3 (1) 6, 3 (2) 12, 3 (3) 8, 3 (4) 6 , 3 (5) 24 , 3 (6) 24, 3 (7) 0 ,

3 (8) 12 , 3 (9) 30 , 3 (10) 24 . nh l 3 suy ra vi v hn cc s n ta c 3 (n) 0 .


i vi cc s T3 n 3 1 3 2 ... 3 n , s dng lp lun hnh hc tng t trong mc 2
i vi tng hai bnh phng (ta thay cc im hu t trong mt phng bi cc im trong khng
gian m c cc ta nguyn v xt cc hnh cu v hnh lp phng thay cho hnh trn v hnh
vung tng ng) ta c bt ng thc
2

4
3
4
3
n
1 T3 n n
1
3
2
3
2
4
T bt ng thc ny suy ra vi mi s t nhin n ta c c lng T3 n n n 10n, suy ra
3

lim

T3 n
1.
4
n n
3

K hiu f ( x) l s cc s t nhin x c th biu din thnh tng ba bnh phng. T nh l Gauss


l
suy ra s x f ( x) l x v khng c dng 4 8k 7 vi cc s nguyn k , l 0 . V vy vi s
l
nguyn khng m l ta c 8 k 1 1 4 x v do k 1

4 l x 1
1 l

.
8 4 x 1 . Ta c x f ( x) 8
l o

Nu l log x / log 4 th 4l x x / 7, suy ra 4l x 1 / 8 1 .

1 l
4 x 1 s cc s k 0 l
8

CHNG 11. BIU DIN CC S T NHIN THNH TNG CC LY THA BC k KHNG M | 249

H qu l x f ( x)

log x /log 4

l 0

Nhng

4 l x 1
8 do x f ( x)

log x /log 4

l 0

log x
4 l x 1
a
1 , 0 a 1 .
8
log 4

4 log x log 41 4 log x /log 4 4


4l .4
.4
.
3
3
3x
t log x /log 41

4 l x x
x f ( x) 1
f ( x) 5
suy ra ta c lim
v do lim
.
T
x

8
6
x
6
x
6
l 0

Cng thc ny c tm ra bi Landau nm 1908.

5
f x x
6 v chng minh c
M.C.Charkrabarti [1] nghin cu hm g x
log x

lim inf g x 0, lim sup g x

Hn na cc gi tr ca g x tr mt trong khong 0,

1
log8

1
.
log 8

Bi tp. 1. Chng minh rng 130 khng phi tng ba bnh phng dng.
2
2
2
Chng minh. Gi s 130 a b c vi a, b, c l cc s t nhin. Khng gim tng qut c th
2
2
2
gi s a b c . Suy ra a 1 1 130 3a . Nn 43 a 128 v do 7 a 11. Nhng
130 72 81 34 ,130 82 66 2.3.11, 130 92 49 72 ,130 102 30 2.3.5,130 112 9 32
v theo cc phn tch thnh tha s nguyn t ca cc s 81,66,49,30,9 th khng c s no tha
mn iu kin nh l 2 nn chng khng l tng ca hai bnh phng cc s t nhin. Vy 130
khng phi tng ba bnh phng cc s t nhin.

Ghi ch. D dng chng minh rng 130 l s t nhin nh nht c dng 2 8k 1 khng phi tng
ba bnh phng cc s t nhin.
2. S dng nh l Gauss chng minh rng s t nhin l tng ba bnh phng cc s hu t khi v
ch khi n l tng ba bnh phng cc s nguyn.
Chng minh. Gi s s t nhin n l tng ba bnh phng cc s hu t th quy ng cc s hu
2
2
2
2
t thnh cc phn s c chung mu s, ta c th vit m n a b c vi a, b, c l cc s
l
r
nguyn. Nu n 4 8k 7 vi k , l nguyn 0 th t m 2 2s 1 , s v r l cc s nguyn
2
k r
khng m ta c m n 4 8t 7 vi k r v t l cc s nguyn khng m. Nhng theo nh l 3
l
2
2
2
2
th iu ny v l v m n a b c . H qu l n khng c dng 4 8k 7 vi cc s nguyn

k , l . V vy theo nh l Gauss n l tng ba bnh phng cc s nguyn. iu kin trong bi ton


cng l iu kin .
3. Chng minh rng khng tn ti cc s hu t x, y, z m x 2 y 2 z 2 x y z 1 .
Chng minh. Phng trnh trn tng ng vi
(9)

2 x 1 2 y 1 2 z 1
2

7.

Trong chng minh bi tp 2 ta chng minh (khng s dng nh l Gauss) khng tn ti s c


l
dng 4 8k 7 vi k v l l cc s nguyn khng m m c th biu din thnh tng ca ba bnh

250 | Tng ca ba bnh phng

phng cc s hu t. V vy 7 khng c biu din nh vy, suy ra x, y, z khng th tha mn


phng trnh (9).
2
2
2
4. S dng nh l Gauss chng minh rng mi s l c dng a b 2c vi k , l l cc s nguyn.

l
Chng minh. Gi s t l s nguyn khng m ty . S 4t 2 khng c dng 4 8k 7 vi k , l l

cc s nguyn khng m. Do theo nh l Gauss th 4t 2 x 2 y 2 z 2 vi x, y, z l cc s


nguyn. Khng th xy ra trng hp tt c cc s u chn v v tri ng thc khng chia ht
cho 4. Tuy nhin s cc s l trong chng l chn v v tri l chn, v vy gi s x, y l v z chn,
ngha l z 2c . Cc s x y v x y chn v do x y 2a , x y 2b suy ra

x a b, y a b . Vy 4t 2 a b a b 4c 2 nn 2t 11 a 2 b2 2c 2 vi a, b, c l cc
s nguyn. iu phi chng minh.
2

2
2
2
2
2
2
5. T nh l Gauss suy ra mi s t nhin s c dng a b c hoc a b 2c vi a, b, c
nguyn.

Chng minh. Nu mt s t nhin khng phi tng ca 3 bnh phng th theo nh l Gauss n c
l
s dng 4 8k 7 vi k , l l cc s nguyn khng m no . Nhng theo bi tp 4 th ta li c

8k 7 x 2 y 2 2 z 2 vi x, y, z l cc s nguyn. V vy 4l 8k 7 2l x 2l y 2 2l z do
2

2
2
2
s ban u c dng a b 2c , a, b, c nguyn.

6. Chng minh mt s 0 l tng ba bnh phng cc s hu t th n c th biu din v hn


cch di dng .
Chng minh. Kt qu ny l h qu trc tip ca nh l c chng minh trong bi tp 2 mc
1: mi s 0 l tng ca hai bnh phng cc s hu t c v hn cch biu din di dng .
7. Chng minh nh l E.Lionnet ni rng mi s t nhin l u l tng cc bnh phng ca bn
s nguyn m hai trong s l cc s lin tip l h qu ca nh l Gauss.
Chng minh. t n 2k 1 vi k 0,1, 2... T nh l Gauss suy ra 4k 1 l tng ca ba bnh
phng, tc l 4k 1 x 2 y 2 z 2 . Lu rng mt trong cc s x, y, z l v cc s cn li l chn.
t x 2a, y 2b , z 2c 1 vi a, b, c l cc s nguyn.
Vy n 2k 1 a b a b c 2 c 1 . iu phi chng minh.
2

2
2
2
8. Chng minh tn ti v hn s nguyn t c dng a b c 1 vi a, b, c l cc s t nhin.

Chng minh. Theo nh l 1 Chng 9 th tn ti v hn s nguyn t c dng 8k 7 . Nu p l


s nguyn t c dng ny th p 1 8k 6. Nhng t nh l Gauss suy ra mi s c dng 8k 6
u l tng cc bnh phng ca ba s t nhin. V vy p 1 a 2 b2 c 2 vi cc s t nhin

a, b, c v vy p a 2 b2 c2 1.
9. Tm v d chng t tch ca hai s m mi s l tng ba bnh phng khng nht thit l tng ba
bnh phng..

Li gii. 63 3.21 12 12 12 12 2 2 4 2 . S 63 khng c dng 8k 7 nn khng l tng ba


bnh phng.
10. T nh l Gauss suy ra mi s t nhin l tng ca 10 (hoc t hn) bnh phng cc s l
(Pollock t ra bi ton ny m khng km theo chng minh [1], S.Turski a ra mt chng
minh trong [1])
Chng minh. T nh l Gauss suy ra mi s t nhin c dng 8k 3 vi k nguyn 0 u l
tng ca ba bnh phng cc s l. Mt khc mi s t nhin n 3 u c dng 8k 3 r vi

CHNG 11. BIU DIN CC S T NHIN THNH TNG CC LY THA BC k KHNG M | 251

r 0,1, 2,3, 4,5,6,7. Ta thy r l tng ca nhiu nht l 7 bnh phng cc s 1, do n l tng
ca nhiu nht l 10 bnh phng cc s t nhin l. C v hn cc s t nhin khng phi tng
ca t hn 10 bnh phng cc s t nhin l (bi tp 12).
Ghi ch. Kt qu m ta va chng minh c gi l nh l T. nh l ny suy ra mi s t nhin
c dng 8k 3 vi k l s nguyn khng m u l tng ca ba bnh phng l. Tht vy theo
nh l T th nu k 0 ta c
8k 3 n12 n22 ... ns2 ,

(*)

2
vi s t nhin s 10, n1 , n2 ,..., ns l. V vy ta c ni 1 mod vi i 1, 2,.., s v theo (*) th

3 s mod8 m 1 s 10 suy ra s 3 . Vy theo (*) th cc s 8k 3 u l tng ca ba bnh


phng l (Sierpinski [8]).
11. Chng minh rng ly tha bc s (vi s l s t nhin) ca mt s nguyn l tng ca ba bnh
phng cc s nguyn cng l tng ca ba bnh phng cc s nguyn.
Chng minh. Nu s l 1 hoc 2 th ta c iu phi chng minh. Ch cn xt trng hp s c dng

2k 3 , k l s nguyn khng m. Ta c n2 k 3 nk

n3 v vy ch cn chng minh cho trng hp

s =3. iu ny suy ra t ng thc Catalan:

y 2 z 2 x 2 3z 2 x 2 y 2 y 2 3z 2 x 2 y 2 z 2 z 2 3x 2 3 y 2 .
3

12. Chng minh rng tn ti v hn cc s t nhin khng th biu din thnh tng ca t hn 10
bnh phng l.
Chng minh. Cc s c dng 72k 42 vi k 0,1, 2,.. c tnh cht . Tht vy gi s
n 72k 42 l s t nhin l tng ca s 10 bnh phng l. Do bnh phng l 1 (mod 8) nn
ta c n s mod8 suy ra v n 72t 42 2 mod8 nn s 2 mod8 . Vy s 2 .
H qu l n l tng ca hai bnh phng. Nhng iu ny l khng th v n 3 24t 14 =

9 8t 4 6 chia ht cho 3 nhng khng chia ht cho 9.


Tng t c th chng minh cc s 72k 66, k 0,1, 2,.., cng c tnh cht ny.
5. Biu din bi tng bn bnh phng
Ta chng minh nh l Lagrange sau y
nh l 4 (Lagrange). Mi s nguyn khng m u l tng ca bn bnh phng.
B 1. Gi s s nguyn t l p l c s ca mt tng bn bnh phng v t nht mt trong cc
bnh phng khng chia ht cho p th p l tng ca bn bnh phng.
Chng minh b 1. Gi s s nguyn t p c tnh cht . Khi tn ti bi s ca p l tng
ca bn bnh phng m c t nht mt s khng chia ht cho p . Gi s n l bi s nh nht nh
vy ca p . Ta c
(10)

n mp

Vi m l s t nhin v
(11)

n a 2 b2 c 2 d 2 ,

vi a, b, c , d l cc s nguyn v t nht mt trong chng khng chia ht cho p , gi s l a .


Gi s a1 , b0 , c0 , d0 l cc s nguyn tha mn

252 | Biu din bi tng bn bnh phng

a0 a mod p , bo b mod p ,

(12)

c0 c mod p , d0 d mod p

a0 p / 2, b0 p / 2, c0 p / 2, d0 p / 2

(13)

tnh a0 ch cn tnh s d r nhn c khi chia a cho p v t a0 r nu r p / 2 , hoc

a0 r p nu r p / 2 . V a khng chia ht cho p nn a0 cng th v t (12),(10) v (11) suy

2
2
2
2
2
2
2
2
ra a0 b0 c0 d0 a b c d 0 mod p . V vy theo nh ngha ca n v (13) suy ra

n a02 b02 c02 d02 4 p / 2 . H qu l n p 2 v theo (10) suy ra mp p 2 t ta c


2

m p

(14)

T (10) v (11) suy ra ta ch cn cn chng minh m 1 .


Gi s m 1 . V m l s t nhin nn theo (14) ta c

1 m p .

(15)

Ta tm cc s t nhin a1 , b1 , c1 , d1 tha mn cc iu kin

a1 a mod m , b1 b mod m , c1 c mod m ;

(16)

c1 c mod m

a1 m / 2, b1 m / 2,

(17)

c1 m / 2, d1 m2 .

T (16) ta c a12 b12 c12 d12 a 2 b2 c2 d 2 (mod m) v suy ra theo (11) v (10) th

m a12 b12 c12 d12 do


a12 b12 c12 d12 ml ,

(18)

Vi s nguyn l 0 . Nu l 0 th theo (18), a1 b1 c1 d1 0 suy ra theo (16) tt c cc s

a, b, c, d u chia ht cho m v suy ra theo (11) th n chia ht cho m 2 v do theo (10) th m p


nhng iu ny mu thun vi (15) v p nguyn t. Do l l s t nhin. Gi s rng

| a1 || b1 || c1 || d1 | m / 2

(19)

iu ny ch c th xy khi m chn, ngha l khi

m 2k ,

(20)

vi k l s t nhin. T ng d thc a1 a mod m suy ra a a1 mt vi t l s nguyn. Do


theo (20) v (19) ta c a k 2kt 2t 1 k k1 k vi k1 l. Tng t ta chng minh c

a k1k , b k2 k , c k3k , d k4 k , vi k1 , k2 , k3 , k4 l.

V vy theo (20), (10) v (11) ta c n 2kp k 2 k12 k22 k32 k42 .

H qu l 2 p k k12 k22 k32 k42 . V mi bnh phng l u ng d vi 1(mod4) suy ra nhn


t th hai trong v phi ca ng thc cui cng l chia ht cho 4 v do 2 p mu thun vi gi
thit. Suy ra (19) l khng th c. H qu l vi t nht mt trong cc bt ng thc (17) th ng
m2
2
thc khng th xy ra. Suy ra a12 b12 c12 d12 4.
v vy theo (18) ta c ml m v do
4
(21)

l m.

CHNG 11. BIU DIN CC S T NHIN THNH TNG CC LY THA BC k KHNG M | 253

So snh vi ng nht thc Euler

Theo (11), (10) v (18) th v tri bng m2lp . Theo (16) ta c

vi a2 , b2 , c2 , d2 l cc s nguyn. Theo (11) v (10) cng thc (23) suy ra

vi t1 , t2 , t3 , t4 l cc s nguyn.

Th cc ng thc ny vo (22) ta c m2lp m2 t12 t22 t32 t42 v t y suy ra


(24)

lp t12 t22 t32 t42 .

2
Nu cc s t1 , t2 , t3 , t4 u chia ht cho p th p lp v do p l , v l v l l s t nhin v theo

(21) v (14) th l p . Cng thc (24) cho ta biu din ca s lp thnh tng ca bn bnh phng
m trong c s khng chia ht cho p . T nh ngha ca n suy ra n lp v do theo (10) th
mp lp suy ra m l mu thun vi (21). V vy t gi thit m 1 dn ti mu thun. Suy ra
m 1 v ta c iu phi chng minh.
B 2. Mi s nguyn t u l tng ca bn bnh phng.
2
2
2
2
Chng minh b 2. Ta c 2 1 1 0 0 do khng gim tng qut gi s p l s nguyn
t l. Theo B 1 th ta ch cn chng minh p l c s ca mt tng bn bnh phng cc s
nguyn m trong c t nht mt s khng chia ht cho p .

S d nhn c khi chia cc s


(25)

p 1
1 0 ,1 1 ,...,1

2
2

p 1
cho p l khc nhau v ta bit theo Chng 5 mc 5 th cc s 02 ,12 ,...,
khi chia cho p
2
s cho cc s d khc nhau. Tng t cc s sau khi chia cho p cng cho cc s d khc nhau
2

(26)

p 1
02 , 12 ,...,

Gi s cc s d nhn c khi chia cc s trong dy (25) cho p v cc s d nhn c khi chia


cc s trong dy (26) cho p l khc nhau th khi tng s cc s d phn bit nhn c t c

p 1

2
hai dy l 2 1
p 1. V l. Vy tn ti phn t 1 x thuc dy (25) c cng s d khi
2

254 | Biu din bi tng bn bnh phng

chia cho p vi phn t y 2 ca dy (26). Khi ta c p 12 x 2 y 2 02 v suy ra p l c s


ca mt tng bn bnh phng cc s nguyn m khng chia ht cho p . iu phi chng minh.
Chng minh nh l 4. Theo ng thc (22) th tch ca hai s m mi s u l tng bn bnh
phng th cng l tng bn bnh phng. S dng mt php quy np n gin suy ra iu ny
ng vi tch hu hn cc s nh vy. Bi v mi s 1 u l tch ca cc s nguyn t m theo
B 2 th tt c cc s nguyn t u l tng bn bnh phng nn bn thn s ban u cng l
2
2
2
2
2
2
2
2
2
tng bn bnh phng. Hn na 0 0 0 0 0 + 0 v 1 1 0 0 0 . nh l c
chng minh.
D.H.Lehmer [4] chng t rng trong cc s t nhin th ch c cc s 1, 2, 5, 7, 11, 15, 23 v cc
h
s c dng 4 m vi h 0,1, 2.., m 2,6 hoc 14 l c biu din duy nht di dng trn (khng
tnh cc hon v). S.Ramanujan [2] nghin cu cc b s t nhin a, b, c, d m mi s t nhin
n u c th biu din dng ax 2 by 2 cz 2 dt 2 vi x, y, z, t l cc s nguyn. ng ta chng
minh rng vi gi thit a b c d th tn ti ng 54 b s nh vy, tt c u c dng 1,1,1, d
vi d 1,2,,7 hoc 1,1,2, d vi d 2,3, ..,14 hoc 1,1,3,d vi d 3,4,5,6 hoc 1,2,2,d vi d
2,3,,7 hoc 1,2,3, d vi d =6,7,,10 (Dickson [6] trang 104 nh l 95).

Ta chng minh nh l Jacobi: mi s t nhin u c dng x2 2 y 2 3z 2 6t 2 vi x, y, z, t nguyn.


Chng minh. Gi s n l s t nhin. Theo nh l 4 th tn ti cc s nguyn a, b, c, d m

n a 2 b2 c 2 d 2 .

(27)

Ta s chng minh vi php i du thch hp thi ta c 3 a b c .


iu ny hin nhin nu t nht ba trong cc s a, b, c, d chia ht cho 3. Gi s ch c hai s c v d
chia ht cho 3 th th a 1 (mod 3) v b 1 mod 3 suy ra vi php i du thch hp ta c

3 a b do 3 a b c . Cui cng nu c ba trong cc s a, b, c, d l a, b, c khng chia ht cho 3


th vi php chn du thch hp ta c 3 a b 3 a b c .
Khng gim tng qut gi s

a b c 3z ,

(28)

vi z l s nguyn. Nhng trong ba s nguyn lun c t nht hai s ng d mod 2 nn c th gi


s a b mod 2 suy ra a b 2k vi k l s nguyn. Ta c ng thc

3 a b c
2

a b c

ab

a b
2
c 6
,
2

2
2

2
2
2
2
Suy ra 3 a b c a b c 2 k c 6 y , m theo (28) suy ra 3 | k c do k c 3t
2

vi t l s nguyn. V vy theo (28) ta c a 2 b2 c 2 3x 2 6t 2 2 y 2 v t (27) ta c

n d 2 2 y 2 3z 2 6t 2 .
Bi tp. 1. T nh l 4 chng minh mi s t nhin chia ht cho 8 l tng ca 8 bnh phng l.
Chng minh, Nu n l s t nhin th theo nh l 4 suy ra tn ti 4 s nguyn a, b, c, d tha mn

n 1 a2 b2 c2 d 2 v

8n (2a 1)2 (2a 1)2 (2b 1)2 (2b 1) 2 (2c 1) 2 (2c 1) 2 (2d 1) 2 (2d 1) 2
2. Chng minh rng khng tn ti s t nhin chia ht cho 8 l tng ca t hn 8 bnh phng l.

CHNG 11. BIU DIN CC S T NHIN THNH TNG CC LY THA BC k KHNG M | 255

Chng minh. Tng ca s bnh phng l c dng 8k s vi k l s nguyn khng m. Do nu


tng ny chia ht cho 8 th 8 s v do s 8 .
6. Tng ca bn bnh phng cc s t nhin
H qu trc tip ca nh l 4 l mi s t nhin u l tng ca bn (hoc t hn) bnh phng
cc s t nhin. S dng nh l Gauss ta s chng minh
nh l 5. S t nhin n l tng bnh phng ca 4 s t nhin khi v ch khi n khng thuc dy
h
h
h
cc s 1,3,5,9,11,17,29,41, 4 .2 , 4 .6 , 4 .14 vi h 0,1, 2,....
nh l ny c Descartes t ra di dng mt gi thuyt (G.Pall [1] trang 11).
Chng minh. Ta ni mt s t nhin l S m nu n l tng bnh phng ca m s t nhin.
D dng chng minh khng c s no trong cc s 1,3,5,9,11,29,41 l S 4 .
2
2
2
2
Ta chng minh tnh cht ny i vi s 41. Gi s ngc li 41 l S 4 ngha l 41 a b c d
2
2
vi a, b, c, d l cc s t nhin v a b c d . V vy a 41 4a v do 4 a 6 . Nu a 6
2
2
2
2
2
2
th 5 b c d v l. Nu a 5 th 16 b c d v l v 16 khng phi S3 . Nu a 4 th

25 b2 c2 d 2 v l v 25 khng phi S3 . Do 41 khng phi S 4 .


By gi k hiu m l mt trong cc s 2,6,14. Khi m c dng 4k 2 . Gi s tn ti s nguyn
h
khng m h tha mn 4 m l S 4 . K hiu h l s nguyn nh nht nh vy. V 2,6,14 khng phi

S4 , h 1. Ta c 4h m a 2 b2 c2 d 2 vi a, b, c, d l cc s t nhin v v tri ca ng thc chia


ht cho 8 v

h 1 v

m 2 2k 1 . T y suy ra a, b, c, d

u chn. V vy

a 2a1 , b 2b1 , c 2c1 , d 2d1 vi a1 , b1 , c1 , d1 l cc s t nhin. Do 4 m a12 b12 c12 d12


h 1

h 1
h
suy ra 4 m l S 4 mu thun vi nh ngha ca h . Vy ta chng minh cc s 4 m vi m

2,6,14 khng phi S 4 vi mi s nguyn khng m h . T y suy ra iu kin cn.


K hiu n l s t nhin l tha mn iu kin trong nh l 5. H qu l n 1,3,5,9,11,17,29,41. V
n l nn n c dng 8k 1,8k 3,8k 5,8k 7 .
Gi s n 8k 1. Ta xt bn trng hp k 4t , k 4t 1 , k 4t 2 , k 4t 3 . Nu k 4t th
n 32t 1 v v n 1 nn ta c t 1 m v do t u 1 vi u l s nguyn khng m. V vy
n 32 u 1 1 4 8u 6 9 . Theo nh l Gauss th s 8u 6 l tng ba bnh phng cc s
nguyn. V 8u 6 = 2 4u 3 khng phi tng bnh phng hai s nguyn nn ta thy 8u 6 l S3
2
2
v suy ra n 2 8u 6 3 l S 4 . Nu k 4t 1 th n 32t 9 . Do v n 9 v n 41 nn ta
2
2
c t 2 do t u 2 vi u l s nguyn 0 . V vy n 32 u 2 9 2 8u 6 7 suy ra n

l S 4 . Nu k 4t 2 th n 32t 17 v v n 17 ta c t 1 v do t u 1 vi u l s nguyn

0. V vy n 32 u 1 17 22 8u 6 52 suy ra n l S 4 . Nu k 4t 3 th n 32t 25
22 8t 6 12 . Vy ta c iu kin trong trng hp n 8k 1 .

Gi s n 8k 3 . Do n 3 v n 11 nn ta c k 2 v do k t 2 vi t l s nguyn khng
2
m. Ta c n 8 t 2 3 8t 3 4 suy ra theo nh l Gauss th 8t 3 l tng bnh phng ba
s l v suy ra n l S 4 . Trng hp n 8k 3 c chng minh.
Tip theo gi s n 8k 5 . Ta xt bn trng hp k 4t , k 4t 1, k 4t 2, k 4t 3 . Nu
k 4t th n 32t 5 v v n 5 ta c t 0, do t u 1 vi u l s nguyn khng m. Do

256 | Tng ca bn bnh phng cc s t nhin

n 32 u 1 5 22 8u 3 52 suy ra n l S 4 . Nu k 4t 1 th n 32t 13 22 8t 3 12 suy


2
2
ra n l S 4 . Nu k 4t 2 th n 32t 21 2 8t 3 3 suy ra n l S 4 . Nu k 4t 3 th

n 32t 29

n 29

nn ta c t 0 v do t u 1 vi u 0 suy ra
n 32 u 1 29 2 8u 3 72 , nn n l S 4 . Trng hp n 8k 5 c chng minh.
v v

Cui cng xt n 8k 7 . Theo nh l 4 th tn ti cc s nguyn a, b, c, d

n a b c d . Mt khc theo nh l 3 th v n 8k 7 nn khng c s no trong cc s


a, b, c, d bng 0 . Do n l S 4 .
2

Vy mt s t nhin l l tng ca bn bnh phng cc s t nhin khi v ch khi n khng phi


1,3,5,9,11,17,29,41. Chng t mi s l 41 u l tng ca bn bnh phng cc s t nhin.
h
h
By gi k hiu n l s t nhin chn khng c dng 4h.2, 42.6 , 4 .14 vi h 0,1, 2 K hiu 4
h
l ly tha cao nht ca 4 l c s ca n . Ta c n 4 m vi m khng chia ht cho 4. H qu l
m 4k 1, m 4k 2, hoc m 4k 3 .

Nu m 4k 1 vi k chn, ngha l k 2t , th m 8t 1, s ny l S 4 , Nu m 1,9 ,17,41 th

n 4h m cng l S 4 . Nhng v n chn v v m khng chia ht cho 4 nn h >0. R rng 4 l S 4 ,

4.17 68 12 32 32 72 , 4.41 164 12 12 92 92 suy ra tt c cc s sau u l S 4


4h.1 4 2h1 , 4h.9 4 2h1.3 , 4h.17 4.17 2h1 , 4h.41 4.41 2h1
2

h
Vy nu m 4k 1 v k chn th n 4 m l S 4 .

Nu m 4k 1 vi k l, ngha l k 2t 1 , th m 8t 5 l S 4 vi m 5 v m 29 . Nhng

4.5 20 12 12 32 32 , 4.29 116 12 32 52 92 suy ra v m l, n chn v h l s t nhin


h
suy ra tt c cc s ny u l S 4 . Vy nu m 4k 1 th n 4 m l S 4 .
h
h
Gi s m 4k 2 th nu k 2t ta c m 8t 2 . V n 4 .2 v n 4 m ta c m 2 v do
2
t 0 ngha l t u 1 vi u l s nguyn khng m. Ta c m 8 u 1 2 8u 6 2 . V 8u 6
h
l S3 suy ra m l S 4 , v do n 4 m cng l S 4 . Trong trng hp k 2t 1 ta c m 8t 6

h
h
v v n 4 .6 v n 4 .14, ta c t 2 do t u 2 vi u l s nguyn khng m. V vy
m 8 u 2 6 8u 6 42 m 8u 6 l S3 suy ra m l S 4 nn n 4h m cng l S 4 . Vy ta
h
chng minh c nu m 4k 2, th n 4 m l S 4 .

Cui cng nu m 4k 3 th nu k 2t ta c m 8t 3 . Nhng vi m 3 v m 11 s


h
m 8t 3 l S 4 . V vy nu m 4k 3 th s n 4 m l S 4 vi n 44.3, n 4h.11 . Nhng

4.3 12 12 12 12 32 v 4.11 44 = 12 32 32 52 . V vy n 4h m vi h 0 l S 4 v n chn


h
v m l. Nu k 2t 1 ta c m 8t 7 v do m l S 4 . Suy ra n 4 m cng l S 4 .

Tng hp cc kt qu ny suy ra nu n chn khng c dng 4h.2, 4h.6, 4h.14 vi h 0,1, 2,.., th n
l S 4 . Kt hp vi kt qu trn ta c nh l 5 c chng minh.
2
H qu. Bnh phng mt s t nhin 1 ngoi tr 3 u l tng bnh phng bn s t nhin.

Bi tp. Khng s dng nh l Gauss hy chng minh rng mi s hu t dng u l tng bnh
phng ca bn hu t dng.
Chng minh. K hiu r l s hu t dng, r m / l , vi l v m l cc s t nhin.

CHNG 11. BIU DIN CC S T NHIN THNH TNG CC LY THA BC k KHNG M | 257

Theo nh l 4 suy ra mi s t nhin u l tng bnh phng ca bn (hoc t hn) cc s t


2
2
2
2
nhin. Nu lm a b c d vi a, b, c, d l cc s t nhin th

r = l / m a / m 2 b / m 2 c / m 2 d / m 2
2
2
2
suy ra r l tng bnh phng ca 4 s t nhin. Nu lm a b c vi a, b, c l cc s t nhin
2
2
th r l / m a / m b / 3m 3c / 5m 4c / 5m . Nu lm a b vi a, b l cc s t
2

2
2
nhin th r lm a / m b / 3m 2b / 3m 2b / 3m . Cui cng nu lm a vi a l s
2

t nhin th r l / m 4 a / 2m .
2

Vy trong mi trng hp r u l tng bnh phng bn s hu t dng.


Ghi ch. C th chng minh rng mi s hu t dng u l tng bnh phng ca bn s hu t
dng khc nhau v vi mi s hu t dng th tn ti v hn cch biu din nh th.
n
Trong mc 4 ta chng minh cc s 2 vi n 1, 2,..., v h qu l cc s 4h.2, h 0,1, 2,..., u

khng phi

S3 . Mt khc 3 12 12 12 , 9 12 22 22 , 11 = 11 11 32 ,17= 22 22 32 ,

29 22 32 42 , 41 12 22 62 , 4h.6 2h 2h 2h12 , 4h.14 2h 2h1 2h.3 vi


2

h 0, 1, 2,... V vy t nh l 5 suy ra
nh l 6. S t nhin n l tng bnh phng ca ba hoc bn s t nhin khi v ch khi n khng
h
phi 1,5 v 4 .2 vi h 0,1, 2...
T y ta c h qu
H qu. S t nhin l n l tng bnh phng ca ba hoc bn s t nhin khi v ch khi n khng
phi 1 hoc 5.
H qu ny c s dng trong chng minh nh l di y
2
nh l 7 (Hurwitz [2]). Tt c cc s t nhin n m n khng phi tng bnh phng ca ba s t
h
h
nhin l cc s n 2 v n 2 .5 vi h 0,1, 2...

Chng minh. Trong mc 4 ta chng minh nu k khng phi S3 th 4k cng khng phi S3 .
h
h
2
Nhng v 1 v 5 khng phi S3 nn cc s 4 v 4 .5 , h =0, 1, 2, cng khng phi S3 . Vy cn
h
h
2
phi chng minh nu n l s t nhin 2 v 2 .5 vi h 0, 1, 2 ..., th n l S3 .
h
h
Gi s n l s t nhin tha mn n 2 v n 2 .5 vi h 0, 1, 2,...
s
Gi s l ly tha ln nht m 2 l c s ca n . Ta c n 2 m vi m l. Hn na theo gi thit
ca n, m phi khc 1 v 5. T h qu ca nh l 6 suy ra m l tng bnh phng ca ba hoc bn

2
2
2
2
s t nhin, m a b c d vi a, b, c l cc s t nhin v d l s nguyn khng m. V vy

m2 a 2 b 2 c 2 d 2 a 2 b 2 c 2 d 2 2 ac bd
2

2 ad bc a
2

b 2 c 2 d 2 2 ad bc 2 ac bd
2

2
2
2
2
V m l nn t m a b c d suy ra trong cc s a, b, c, d c hoc mt hoc ba s l l, cc
2
2
2
2
s cn li chn. V vy a b c d l v khc 0 . V a, b, c l cc s t nhin, ac bd v ad
bc cng l cc s t nhin.

Ta s chng minh c t nht mt trong cc s ad bc, ac bd l khc 0 . Tht vy gi s ad bc


2
2
2
2
2
2
v ac bd th adc dc v acd bd suy ra db bd v do v b 0 , c d . V vy t

258 | Tng ca bn bnh phng cc s t nhin

c 0, a b suy ra m 2 a 2 c 2 v l v m l. Do hoc ad bc 0 hoc ac bd 0 (hoc c


hai). Vy t nht mt trong cc tng trn cho mt biu din m thnh tng bnh phng ca ba
s t nhin. Do ta c m2 x 2 y 2 z 2 vi x, y, z l cc s t nhin. V vy
2

n2 2s x 2s y 2s z chng t n 2 l S3 . nh l 7 c chng minh.


2

T nh l 2 suy ra s t nhin n l ng cho chnh ca hnh hp ch nht vi cc cnh c


h
di l s t nhin khi v ch khi n khng c dng 2h hoc 2 .5 vi h 0, 1, 2... T nh l 7 suy ra
vi mi s t nhin l t khc 1 v 5 tn ti s t nhin x, y, z tha mn t 2 x2 y 2 z 2 .
Cu hi c t ra l c phi vi mi s t nhin l t khc 1 v 5 th u tn ti cc s t nhin
x, y, z tha mn x, y, z =1 v x2 y 2 z 2 t 2 . A.Schinzel ([10] H qu 1) chng minh cu tr
li khng nh cho d on ny (d dng chng minh rng vi s chn t th khng tn ti x, y, z
2
2
2
2
nh vy). F.Ssteiger [1] tm ra 347 h x, y, z nh vy vi t 10 . V d 3 1 2 2 ,

72 22 32 62 ,92 12 42 82 = 42 42 72 , 112 22 62 92 , 132 32 42 122 ,

152 22 52 142 22 102 112 , 172 12 122 122 = 82 92 122 ,


192 12 62 182 62 62 172 62 102 152 .
A.Schinzel ([10] nh l 1) cho iu kin cn v mt s t nhin n c th biu din di
dng x 2 y 2 z 2 vi x, y, z l cc s t nhin tha mn x, y, z 1 . Cc iu kin ny kh phc
tp. Bi ton biu din mt s t nhin thnh tng ca bn bnh phng cc s nguyn phn bit
cng c t ra.
Ta c nh l G.Pall [1]: tt c cc s t nhin khng th biu din thnh tng bnh phng bn s
h
nguyn l cc s 4 a vi h 0, 1, 2,..., a 1, 3, 5, 7, 9, 11, 13, 15, 17, 19, 23, 25, 27, 31, 33, 37, 43, 47,
55, 67, 73, 97, 103, 2, 6, 10, 18, 22, 34, 58, 82.
F.Halter-Koch [1] tnh tt c cc s khng phi tng bnh phng bn s dng phn bit.
7. Tng ca m 5 bnh phng dng
T nh l 5 suy ra mi s t nhin l 41 u l S 4 . V vy vi mi s nh th ta cng thm vi

12 hoc 22 v ta thy mi s chn 42 v mi s l 45 u l S5 . V vy ta ch xt cc s 45 .


T nh l 5 th cc s 4, 7, 10, 12, 15, 16, 18, 19, 20, 21, 22, 23, 25, 26, 27, 28, 30, 31, 33, 34, 35,
36, 37, 38, 39, 40, 42, 43, 44 u l S 4 . Do cng thm 1 hoc 4 vo cc s ta nhn c cc s

S5 . By gi vn cn li cc s 1,2,3,4,6,7,9,10,12,15,18 v 33.
D dng chng minh cc s ny khng phi S5 . Chng hn gi s 33 l S5 ngha l 33=

a 2 b2 c2 d 2 + e 2 vi a, b, c, d , e l cc s t nhin a b c d e . V vy a 2 4 33 5a 2
2
do 6 a 29 suy ra 3 a 5 ngha l a 3 hoc 4 hoc 5. Trong trng hp a 3 s
33 a 2 24 4 .6 l S 4 mu thun vi nh l 5. Nu a 4 th s 33 a 2 17 l S 4 mu thun vi
2
nh l 5. Nu a 5 th 33 a 8 4.2 mu thun vi inh l 5. Ta c

nh l 8. Tt c cc s t nhin khng l tng bnh phng ca 5 s t nhin l cc s 1, 2, 3, 4, 5,


6, 7, 8, 9, 10, 12, 15, 18, 33.
Gi s m l s t nhin 6 . Ta s tm tt c cc s t nhin m 13 l S m . Gi s n l mt s
nh vy. Khi tn ti cc s t nhin a1 , a2 ,...,1m tha mn a1 a2 ....am v n a12 a23 ... am2 .
V vy a12 (m 1) n m 13 suy ra a12 14 tc l a1 3 . Nu a1 1 suy ra a1 a2 ... am =1
v do m n . Gi s a1 2 . Nu t nht bn s trong cc s a2 , a3 ,..., am bng 2 th n 5 .4+

CHNG 11. BIU DIN CC S T NHIN THNH TNG CC LY THA BC k KHNG M | 259

m 5 m 15

mu thun vi gi thit n m 3 . H qu l ch c nhiu nht ba trong cc s

a2 , a3 ,..., am c th bng 2.
V vy c bn kh nng: (1) khng c s no 2 v do n 4 m 1 m 3 ; (2) mt s bng 2
khi n 2.4 m 2 m 6 ; (3) ba s bng 2 do n 3.4 (m 3) m 9 ; (4) ba trong s cc
s a2 , a3 ,..., am bng 2 khi n 4.4 m 4 m 12 . Vy ta ch cn xt trng hp a1 3 . Khi
n 9 a22 a32 ... am2 . Nu a2 3 th n 18 m 2 mu thun vi gi thit n m 13. H
2
qu l a2 2 . Nu a2 1 th a3 a4 ... am 1 do n 3 + m 1 m 8 . Nu a2 2 v trong

2
2
2
cc s a2 , a3 ,.., am c hai hoc nhiu hn cc s bng 2 th n 3 2 2 m 3 = m 14 mu

2
2
thun vi gi thit n m 13 . V vy a3 a4 .. am 1 suy ra m 3 2 m 2 m 11 . Do

ta chng minh c trong cc s t nhin m 13 th ch c cc s m, m 3, 6, m 8, m 9,


m 11, m 12 l S m . By gi gi s n l s t nhin m 13 . Nu n m 28 th v m 6 ta c

n m 28 2.32 4.22 m 6 .12 suy ra n l S m . Gi s n m +28 th n m 5 18 (v

n m 13 ) v n m 5 33 . Theo nh l 8 suy ra s n m 5 l S5 do s

n n m 5 m 5 .12 l S m . Kt hp cc kt qu ny ta nhn c
nh l 9 (Pall [1]). Nu m l s t nhin 6 th tt c cc s nguyn dng khng phi tng bnh
phng ca m s t nhin l cc s 1, 2, 3, , m 1 , m 1 , m 2 , m 4, m 5, m 7, m 10, m 13 .
T cc nh l 8 v 9 suy ra nu m l s t nhin 5 th mi s t nhin ln u l tng bnh
phng ca m s t nhin. iu ny khng ng vi m 1, 2, 3, 4 v tn ti v hn s t nhin m
tt c cc s u
2
1. khng phi bnh phng mt s t nhin (chng hn cc s n 1 vi n 1, 2, ...),
2. khng phi S 2 (chng hn cc s 4k 3 vi k 0, 1, 2...),

3. khng phi S3 (chng hn cc s 8k 7 vi k 0,1, 2.. ),


h
4. khng phi S 4 (chng hn cc s 4 .2 vi k 0, 1, 2...).

Tn ti v hn cc s t nhin khng phi l tng bnh phng ca ba (hoc t hn) cc s t nhin


chng hn cc s c dng 8k 7 vi k 0,1, 2 . Tuy nhin theo nh l Lagrange th mi s t
nhin u l tng bnh phng ca bn (hoc t hn) cc s t nhin.
Bi tp. 1. Chng minh rng vi mi s t nhin m tn ti v hn s t nhin l S i , i 1, 2,.., m , .
Chng minh. Ta chng minh rng mi s c dng 13k ln hn m 3 u c tnh cht yu cu.
2

Tht vy ta c n 13k 5k 12k 3k 4k 12k = 2k 4k 7k + 10k .


2

Vy n l S1 , S2 , S3 v S 4 . Nu i 4 v i m th ta c n 13k 33 v n m 13 do n l S i
2

Ghi ch. S t nhin nh nht ng thi l S1 , S2 v S3 l 169. S ny l S i vi mi i 155 v


trong cc ch s i nm gia 155 v 169 th n l S i ch vi i 157, 158, 160, 161, 163, 166 v 169.
2
2
2
2
2
Chng minh 169 l S100 suy ra t cng thc 169 23.2 77.1 hoc 169 8 2.2 97.1 .

2. Tm s t nhin nh nht n l S i vi mi i 1000 .


2
Li gii. Ta c n 34 . Tht vy v n l S i nn n k 2 , vi k l s t nhin. Ta c n l S1000 ,

k 2 1000 do k 32 . Nhng theo nh l 2 th cc s 322 210 v 332 3.11 khng phi S 2 .


2

260 | Tng ca m 5 bnh phng dng. Hiu ca hai bnh phng


2
2
2
2
2
2
2
2
Tuy nhin 34 16 30 2 24 24 suy ra 34 l S1 , S2 , S3 . Theo nh l 5 ta thy 34 l

Si vi 342 i 13 v i 6 . Do 342 l Si vi mi i 1142 . Mt biu din ca 342 thnh tng


2
2
2
2
ca 1000 bnh phng l 34 2.8 2.4 996.1 .

2
3. Chng minh rng tt c cc s t nhin n m n khng phi S5 l 1,2,3 v tt c cc s t nhin
2
n m n khng phi S 6 l 1,2,4. Chng minh suy trc tip t nh l 8 v 9.

8. Hiu ca hai bnh phng


nh l 10. S nguyn k l hiu ca hai bnh phng khi v ch khi k khng c dng 4t 2 vi t l
s nguyn.
2
2
2
2
Chng minh. Nu a v b l hai s chn th a b chia ht cho 4. Nu a v b cng l th a b
2
2
chia ht cho 8. Nu trong hai s c mt s chn mt s l th a b l. Do ta chng minh
c iu kin cn. Gi s s nguyn k khng c dng 4t 2 . H qu l hoc k l hoc n chia ht
cho 4. Nu k l th k 1 v k 1 u chn v do k 1 / 2 v k 1 / 2 nguyn. Ta c

k 1 k 1
k k
k

. Nu k chia ht cho 4 th k 1 1 . Vy ta chng minh
2 2
4 4
c iu kin . nh l 10 c chng minh.
2

Lp lun s dng trong chng minh trn cho ta kt qu sau


nh l 10 . Mi s t nhin khc 1 v 4 v khng c dng 4t 2 u l hiu ca bnh phng hai
s t nhin.
D dng chng minh khng c s no trong cc s 1 v 4 c th biu din thnh hiu ca hai bnh
phng cc s t nhin. By gi ta s xc nh tt c cc biu din ca mt s t nhin cho trc
thnh hiu hai bnh phng cc s t nhin.
Gi s n l s t nhin khc 1 v 4 v khng c dng 4 z 2 . Gi s n x 2 y 2 vi x, y l cc s
t nhin th ta c n x y x y v nu d x y th d l c s t nhin ca n v nh hn
c s d ' = x y . Hn na cc c s d v d ' c cng tnh chn l v d ' d 2 y . K hiu d l
c s t nhin ty ca n m nh hn c s d ' n / d v tha mn d v d ' c cng tnh chn
2
2
d ' d
d ' d
d ' d d ' d
2
2
l. Th th x
,y
l cc s t nhin v x y
-
= dd ' n . Do
2
2
2 2
n x 2 y 2 . Theo cch ny th ta nhn c mi biu din ca n thnh hiu hai bnh phng

cc s t nhin. V vy s cch biu din nh th l bng vi s c s t nhin ca n v nh hn


i c s tng ng v tha mn c c s v i c s tng ng ca n l c cng tnh
chn l. Vy mi s nguyn t l c ng mt biu din theo cch ny, l

p 1 p 1
p

. Mt h qu khc l nu mt s t nhin l khng phi bnh phng
2 2
2

ng th n c ng d n / 2 cch biu din nh vy. Nu n l bnh phng th n c

d n 1 / 2 cch biu din nh vy ( d n l s c s ca n ). T y chng t khng ch cc s

nguyn t l l ch c ng mt biu din thnh hiu hai bnh phng cc s t nhin m bnh
2

p2 1 p2 1
phng ca cc s cng c tnh cht ny. Ta c biu din p

. Nhng
2 2
mi hp s l khng l bnh phng mt s nguyn t l th c t nht hai biu din nh vy. D
dng chng minh trong s cc s chia ht cho 4 th ch c cc s c dng 4 p hoc 4 p 2 vi p l s
nguyn t 2 l c ng mt biu din dng ny.
2

CHNG 11. BIU DIN CC S T NHIN THNH TNG CC LY THA BC k KHNG M | 261

Bi tp. Chng minh rng vi mi s t nhin m th u tn ti s t nhin n c ng m biu


din thnh hiu hai bnh phng cc s t nhin.
2 m 1
Chng minh. Ta t n 2
. Tht vy s ny c ng m biu din tha mn v d thy cc biu

din l 22 m1 22 mk 2k 1

2
2

2 mk

2k 1 , k 1, 2, ..., m
2

9. Tng ca hai lp phng


D dng chng minh mi s nguyn 0 u c hu hn l 0 cch biu din thnh tng hai lp
phng. R rng ch cn chng minh vi cc s t nhin. S cch biu din mt s thnh tng ca
3
hai lp phng khng m l hu hn. Gi s n x + y 3 vi x, y l cc s nguyn, x 0, y 0 . Ta

c n x y x 2 xy y 2

vi xy 0 . Nhng v x y 0 suy ra x y 1 v ta c

x2 xy y 2 n m xy 0 suy ra x n v 0< y < n . T y suy ra s cp x, y l hu hn.


Mt khc cc lp phng ng d vi 0,1 hoc 8 (mod 9) nn c th chng minh khng c s
nguyn no c dng 9k 4 vi k nguyn li l tng ca ba (hoc t hn) cc lp phng. H qu l
tn ti v hn s t nhin khng phi tng ca hai lp phng.
Cu hi khi no th mt s nguyn t c th biu din thnh tng ca hai lp phng cng tng

i n gin. Gi s p x3 y 3 vi x, y l cc s t nhin th p = x y x y xy suy ra v


2

x y 2 ta phi c p x y v x y xy 1 suy ra x y v xy 1 tc l x y 1 v
p 2 . Vy 2 l s nguyn t duy nht l tng ca hai lp phng cc s t nhin.
2

By gi ta gi s s nguyn t p l tng ca lp phng hai s nguyn m ch c mt trong hai s


l s t nhin. Khi s nguyn t p l hiu ca hai lp phng cc s t nhin. t p a3 b3

ta c p a b a 2 ab b2 suy ra a b =1 v p a 2 ab b2 = 3b b 1 1. T y suy ra nu
s nguyn t p l hiu ca hai lp phng cc s t nhin th p c dng p 3b b 1 1 vi b
l s t nhin. Mt khc nu p c dng th p b 1 b3 . V vy cc s nguyn t c dng
3

3b b 1 +1 l tt c cc s c th biu din thnh hiu ca hai lp phng cc s t nhin. Ta


cha bit c tn ti v hn cc s nguyn t c dng ny hay khng (t gi thuyt H suy ra cu tr
3
3
li khng nh). Tuy nhin ta bit rt nhiu s nguyn t c dng ny. Chng hn 7 2 1 ,19=
33 23 , 37 43 - 33 , 61= 53 43 , 127= 73 63 .
nh l 11. Vi mi s t nhin m th u tn ti s t nhin n c th biu din thnh tng ca hai
lp phng theo t nht l m cch phn bit.
Chng minh. Theo mc 15 Chng 2 ta chng minh tn ti dy v hn cc h
xk , yk , zk k 1, 2, ... cc s nguyn tha mn xk , yk 1, xk3 yk3 7 zk3 v 0 z1 z2 ... i
du nu cn thit cc s xk v yk ta c th gi s zk 0 vi mi k 1, 2,...
t n 7 z13 z23 ...zm3 , ak

z1 z2 ...zm
z z ...z
xk , bk 1 2 m yk vi k 1, 2, ...., m .
zk
zk

Tt c cc s ak v bk u nguyn v hn na ak3 bk3 n . Vi cc ch s i, j no ca dy


1,2,, m ta c ai a j khi v zk 0 vi mi k 1, 2,..m , xi / zi x j / z j suy ra v

xi , zi x j , z j =1 ta c

xi x j v zi z j v l.

Tng t nu ai b j th xi / zi y j / z j m xi , zi y j , z j 1 nn ta c v l.

262 | Tng ca hai lp phng. Phng trnh x y z


3

Vy ta nhn c m cch biu din khc nhau s n thnh tng hai lp phng. nh l 11 c
chng minh.
nh l 12. K hiu n l s t nhin khng phi lp phng mt s t nhin v cng khng phi hai
ln mt lp phng mt s t nhin. Nu n l tng ca hai lp phng cc s hu t th n c v hn
cch biu din nh vy.
3
Chng minh. K hiu r l s nguyn ln nht m r l c s ca n . Th th n r a vi a l s
t nhin khng chia ht cho bt k lp phng mt s t nhin 1 no. Theo gi thit a khng
th bng 1 hoc 2. Gi s n l tng hai lp phng cc s hu t. Ta quy ng cc phn s vi

mu s chung v c th vit n u / t v / t , vi u, v l cc s nguyn v t l s t nhin. V


3

vy u 3 v3 a rt . Cc s u, v l khc 0 v theo gi s n khng phi lp phng mt s t


3

nhin, v do n khng phi lp phng mt s hu t. V vy d u, v l s t nhin. t

u dx , v dy vi x, y l cc s nguyn tha mn x, y 1 . Ta c d 3 a rt suy ra v a khng


3

chia ht cho bt k lp phng mt s t nhin 1 no nn d rt . V do rt dz suy ra z l s


t nhin. Ta thy cc s x, y, z tha mn phng trnh x3 y3 az 3 . V vy theo nh l 10 mc
15 Chng 2 suy ra phng trnh ny c v hn nghim nguyn x, y, z vi x, y x, z y, z
=1 v

z 0 . Vi mi nghim nh vy th ta c

nz 3 a(rz )3 (rx)3 (ry)3 suy ra

n rx / z ry / z . Hn na ta thy cc nghim khc nhau cho tng ng cc biu din khc


3

nhau v cc phn s x / z v y / z u l ti gin. nh l 12 c chng minh.


H qu. Nu r l s hu t khng phi lp phng mt s hu t m cng khng phi hai ln lp
phng mt s hu t th nu r c th biu din thnh tng hai lp phng cc s hu t th s c
v hn cch biu din nh vy
Chng minh. R rng c th gi s r l s hu t dng, ngha l r l / m vi l v m l cc s t
nhin v l , m 1 . Theo gi thit th tn ti cc s nguyn u, v v s t nhin t tha mn
3

l u v
um vm
2
suy ra lm2

. V vy s t nhin lm l tng hai lp phng cc
m t t
t t
s hu t v n khng l lp phng ca s hu t hay l hai ln lp phng mt s hu t bi v
nu ngc li th r l / m s l lp phng mt s hu t hoc hai ln lp phng mt s hu t,
mu thun vi gi thit.
2
Vy theo nh l 12 suy ra s lm c v hn cch biu din thnh tng hai lp phng cc s hu
2
3
t, suy ra s r lm / m cng c tnh cht . H qu c chng minh.

10. Phng trnh x3 y 3 z 3


By gi ta trnh by li gii s cp cho trng hp ring ca nh l cui cng ca Fermat vi s
m 3. Li gii ny c trnh by bi J.Browkin da trn tng c bn ca R.D.Carmichael trong
[4] trang 67-70.
nh l 13. Phng trnh sau y khng c nghim nguyn x, y, z 0.
(29)

x3 y 3 z 3

B . Tt c cc nghim nguyn a, b, s ca phng trnh


(30)

s3 a 2 3b2

m a, b =1, s l, dc cho bi cng thc sau

CHNG 11. BIU DIN CC S T NHIN THNH TNG CC LY THA BC k KHNG M | 263

s 2 3 2 , a 3 9 2 , b 3 2 3 3 ,

(31)

Vi cc s , tha mn iu kin

mod 2 ,

(32)

,3 1

Chng minh b . u tin ta gi s cc s nguyn , tha mn (32). Xt cc s a, b, s xc


nh bi (31). Khi s dng ng thc

(33)

3B 2 A3 9 AB 2 3 3 A2 B 3B3 ,
3

suy ra cc s a, b, s tha mn (30). T (32) suy ra

a, b ( 2 9 2 , 3 a 2 2 ) = 2 9 2 , 2 2 = 8 2 , 2 2 1
v s l. Gi s cc s nguyn a, b, s tha mn (30) v a, b =1 v s l. chng minh b ta
tm cc s nguyn , tha mn (31) v (32).
Lu mi c s nguyn t ca s u c dng 6k 1 . Tht vy nu p s th v s l, p 3 . Nu

p 3 th v (30), 3 a 2 do 3 a , v theo (30), 9 3b 2 suy ra 3 b , mu thun vi (a, b) 1 . Vy


p 3 . V p s v a, b =1 theo (30) suy ra b, p 1 nn 0 a 2 3b2 b2 a 2b p 3 3 (mod p ).

Vy ab p 3/2

mod p . Chng t -3 l thng d bc hai mod p . Suy ra p c dng 6k 1 .

S khc tnh chn l ca , c suy ra bng quy np theo n l s cc c s nguyn t ca s .


3
2
2
Nu n 0 th v s a 3b 0 ta c s 1 . Do a 1, b 0 . Nn 1, 0 . Suy ra (31)
v (32) c tha mn. By gi gi s b c chng minh vi s t nhin n 0 . K hiu s l
s nguyn c n 1 c s nguyn t v hai s nguyn t cng nhau a, b tha mn (30). K hiu p
l c s nguyn t ca s do s tp vi t c n c s nguyn t. V p c dng 6k 1 nn tn
ti cc s nguyn 1 , 1 m p 12 312 vi 1 , 1 tha mn (32). Nu c 13 9112 , d 312 1

- 313 , Theo ng thc (33) suy ra p3 c3 3d 2 v theo (32) th c, d 1 . Ta c

t 3 p6 s3 p3 a 2 3b2 c 2 3d 2 ac 3bd 3 ad bc .
2

(34)

Xt tch

ad bc ad bc ad bc
2

(35)

a 2 3b 2 d 2 b 2 c 2 3d 2

t 3 p 3d 2 b 2 p 3 p 3 t 3d 2 b 2 .

Nu p ad bc v p ad bc th p 2ad v p 2bc suy ra v p l nn p ad v p bc . Nhng

p3 c 2 3d 2 v

c, d 1 .

V vy

p, c p, d 1

a, b 1 . Vy ch c mt trong hai s

v do p a v p b , mu thun vi

ad bc v ab bc l chia ht cho p . Nhng theo (35) s

ny chia ht cho p . H qu l vi la chn du thch hp trong ng thc (34) th s trong du


3

ngoc cui cng v phi l chia ht cho p 3 . Hn na v tri ca (34) chia ht cho p 6 suy ra s cn
li trong du ngoc ca v phi cng thc (34) chia ht cho p 3 .
Do vi la chn du thch hp th
(36)

ac 3bd
ad 3bc
v v
3
p
p3

264 | Tng ca hai lp phng. Phng trnh x y z


3

l cc s nguyn. Cng thc (34) tr thnh


(37)

t 3 u 2 3v2

Ta tm nghim ca phng trnh (36) vi a v b . Ta c a uc 3vd v b ud vc m


a, b 1 suy ra u, v 1 . Theo gi thit quy np v cng thc (37) th tn ti cc s nguyn

2 , 2 tha mn (32) m
t 22 322 , u 23 9 2 22 , v 3 22 2 323.

Ta vit 1 2 312 ,

2 1 21. Khi

i du nu cn thit i vi ta thy , tha mn (31). Do a, b 1 suy ra cc s , tha


mn (32). B c chng minh.
Chng minh nh l 13. Gi s cc s x, y, z tha mn (29) v hn na chng c chn vi

xyz 0 nh nht c th. R rng x, y, z i mt nguyn t cng nhau v nu ngc li trong s


chng c hai s c c s chung l d 1 th c s ny l c s chung ca c ba s v do ta
3
c th chia cc v phng trnh (29) cho d v nhn c mt nghim nh hn. D dng thy cc
s x, y, z khng cng l v c ng mt s trong chng l chn. Gi s z chn v x, y l. Do
cc s x y v x y chn suy ra
(38)

x y 2u,

x y 2w .

V vy
(39)

x u w , y u w.

M x, y 1 , x, y l suy ra u, w 1 v u w (mod2). Th cc gi tr trong (39) vo (29) ta c


(40)

2u u 2 3w2 z 3 .

2
2
Nu u,3 1 th v u w (mod2) ta c 2u, u 3w 1 do

(41)
vi s l s l v

2u t 3 , u 2 3w2 s3 ,

u, v =1.

Theo b th tn ti cc s nguyn , tha mn (32) v

u 3 9 2 . V vy theo (41) th t 3 2u 2 3 3 . D dng kim tra cc s 2 ,

3 , 3 i mt nguyn t cng nhau do 2 = 3 , 3 3 , 3 Q 3 suy ra

3 Q3 3 . Nhng Q t 3 2u x y 0 v x y xyz xyz mu thun vi gi thit


3

v tnh nh nht ca xyz . Nu 3 u ngha l u 3v th (40) tr thnh


(42)

18v 3v 2 w2 z 3 ,

2
2
suy ra v 3v w (mod2) v 3v, w 1 ta c 18v,3v w 1 do

CHNG 11. BIU DIN CC S T NHIN THNH TNG CC LY THA BC k KHNG M | 265

18v t 3 , 3v2 w2 s3 ,

(43)

Vi s l v v, w =1. Theo b tn ti cc s nguyn , tha mn (32) v v 3 2 3 3 .


3
V vy theo (43) th t 18v 27.2 . D dng kim tra cc s 2 , ,

i mt nguyn t cng nhau. Vy 2 3 , 3 , Q3 suy ra 3 3 Q3 . Nhng

| Q |
3

1
3
1 3 2
2
1
t | v | | u | | x y | 0 v x y xyz xyz mu thun vi gi thit v
9
27
3
9
9

tnh nh nht ca xyz . nh l 13 c chng minh.


H qu trc tip ca nh l 13 l
H qu. Phng trnh x3 y 3 z 3 khng c nghim hu t 0 .
Bi tp. 1. Chng minh rng nh l 13 tng ng vi nh l ni rng phng trnh
3x 2 1 4 y3 khng c nghim hu t no ngoi tr x 1, y 1 .
Chng minh. Nu hai s hu t x 1 v y tha mn 3x 2 1 4 y3 th u = 3x 1 / 2 l s hu
2
t, u 1 v u 2 . Hn na t u 2 u 1 3 y 2 suy ra y 0 (v phng trnh u u 1 0

khng c nghim hu t). H qu l 2 u 1 u 3 y mu thun vi h qu ca nh l


3

3
3
13. Gi s nh l 13 sai. Khi tn ti cc s hu t u, v khc 0 tha mn u v 1 v

x u v / u v , y 1/ u v l cc s hu t tha mn 3x2 1 4 y3 . Nu x 1 v y 1 th
u v 1 , u v = 1 , suy ra u 0 hoc v 0 , mu thun.
2. Chng minh rng phng trnh x3 y3 z 3 1 c v hn nghim t nhin x, y, z .
Li gii. Suy trc tip t ng thc Gerardin: 9n4 9n3 1 9n4 3n 1, n 1, 2,...
3

Chng hn vi n 1,93 103 123 1; nu n 2,1443 733 1503 1 .


3
3
3
Ta cng c 64 94 = 103 1 .

3. Tm ba s t nhin phn bit a, b, c m

a , 3 b , 3 c l cc s v t v

a3b 3c.

Li gii. a 2, b 16, c 54 .
11. Tng ca ba lp phng
Trong mc 9 ta lu rng khng c s nguyn c dng 9k 4 m l tng ca ba (hoc t hn)
cc lp phng. Mt khc ta cha bit c phi mi s nguyn khng c dng 9k 4 (vi k
nguyn) u l tng ca ba lp phng hay khng. iu ny ng vi mi s nguyn
n, 30 n 30 , nhng ta cha bit cch no biu din 30 thnh tng ca ba lp phng v cha
bit mt biu din nh th c tn ti hay khng. V.L.Gardiner, R.B.Lazarus v P.R.Stein [1] tm
ra nghim ca phng trnh x3 y3 z 3 k vi 0 k 1000 v cc s nguyn x, y, z, tha
mn 0 x y 216 , 1 . H cng chng minh phng trnh ny khng c nghim vi k=30, 33,
39, 42, 52, 74, 75, 84 v vi k 12, 1 th tn ti duy nht nghim z 11 , y 10 , x 7 . Kt qu
ny khng ch ra rng khi no th phng trnh ny c nghim nguyn x, y, z m c t nht hai s
16

trong chng c gi tr tuyt i ln hn 2 . i vi mt s trng hp ca s nguyn k ta c th


chng minh tn ti v hn cch biu din n thnh tng ca ba lp phng chng hn (Mordell
[4])

0 n3 n 03 , 1 9n4 1 9n3 3n 9n4 , 2 1 6n3 1 6n3 6n2


3

266 | Tng ca ba lp phng

vi mi n 0, 1, 2,... Vi k 1 th c nhng nghim khng nhn c t cng thc trn.


3
3
Chng hn 1 94 + 64 103 .

D.H.Lehmer [7] chng minh rng tn ti v hn biu din nh vy (Godwin [1]).

4 5 9
2 3
3 4 6
3 4
4 2 6
4 9
4 3 6
3 3
Tht vy, ly x 3 t 2 3 t 5 , y 3t 6 t 2 3 t 3 t 1 , z 2 3 t + 2 3 t - 3 t 1 .

Khi d dng kim tra rng vi mi t , x3 y3 z 3 1 . Nu t l s t nhin khng chia ht cho 3


th tt c cc nghim nhn c u khc 9n4 ,1 9n3 ,3n 9n4 bi v kim tra trc tip ta thy
4
3 4
4
khng c gi tr no ca x, y, z bng 9n v y, z khng chia ht cho 9 v nu x 9n th v 3 t x
4 2 6
3 4
suy ra 3t n do n 3ut ( u l s nguyn) suy ra 2 3 t 5 3 u v l.

Thay t 1 ta c x 3753 , y 5262 , z 4528 . Vi t 1 ta c x 3753 , y 2676 , z 3230 .


Vi k 2 ta cha bit biu din no ca k thnh tng ca ba lp phng ngoi cch trnh by
trn. Ta cha bit s nguyn k no khng c dng 9t 4 m c hu hn biu din thnh tng ba
lp phng. Mt khc d dng chng minh tn ti v hn k khng c dng 9t 4 m khng th
biu din thnh tng lp phng ca ba s t nhin. Vi k 3 ta ch bit 4 cch biu din k thnh
tng ba lp phng, l x, y, z = 1,1,1 , 5, 4, 4 , 4, 5, 4 , 4, 4, 5 v ta cha bit c tn ti
cc biu din khc hay khng.
Sau y ta s chng minh s 3 v rt nhiu cc s hu t dng khc c th biu din v hn cch
thnh tng ca ba lp phng cc s hu t dng (nh l 14).
Ta nghin cu cc biu din s nguyn di dng x3 y 3 2 z 3 vi x, y, z l cc s nguyn.
Lai, Russel and Blundon [1] chng minh vi mi s t nhin 1000 ngoi tr 19 s trong
chng (ba trong s ny c tm li bi J.C.Littejohn trong cc cuc trao i vi M.Lai) l c t
3
nht mt biu din nh vy. Chng hn 13 35 62 2 52 , 20 63 3 2 50 ,
3

31 533 + 313 + 2 44 . S 76 l s t nhin nh nht m chng ta cha bit n c dng


3

x3 y3 2 z 3 vi x, y, z l cc s nguyn hay khng. S 2 ngoi tr phn tch tm thng


2 t 3 t 2.13 th c v hn biu din dng trn. iu ny suy ra t ng thc
3

2 1 t t 2 1 t t 2 2 t 2 vi mi s nguyn t . y l h qu ca ng thc cho bi


3

m
B.Segre [1] (trng hp t 2 cho bi Niewiadomski [1]).

nh l 14. Mi s hu t dng u c th biu din v hn cch thnh tng ca ba lp phng cc


s hu t (Hardy v Wright [1] trang 197-199 nh l 34).
Chng minh. Vi r l s hu t dng cho trc k hiu v l s hu t tha mn
3
3r / 2 v 3 3r . t u 3r v3 / 3r v3 , s v 1 u z su , t s / 3 1 u 2 , x s t , y t z .

V v 3 3r suy ra u l s dng v nh hn 1. Cc s u, s, z, t l hu t dng v x, y l cc s

3
1
r suy ra u 6r / 3r v3 1 . H qu l 3 1 u 2 1 ,
2
3
2
s t v 3u 1 u 1 suy ra z 1 . Do x 0 v y 0 . Nhng

hu t. V v 3 3r / 2 nn ta c v3

x3 y 3 z 3 s t t z z 3 s 3 3 s 2 z 2 t 3 s z t 2
3

2
2
2
2
2
2
3,
v 3 s z 3s 1 u suy ra 3 s z t s do

x3 y3 z 3 3 s z t 2 3s 1 u t 2 =

CHNG 11. BIU DIN CC S T NHIN THNH TNG CC LY THA BC k KHNG M | 267

s 3 1 u

3 1 u 2

s3

3 1 u 1 u 2

v3 1 u

3 1 u 2

v3 1 u

r
3 1 u

Do v c th chn l mt s hu t bt k nh hn 3 3r v gn 3 3r nn u (v do su z ) c
th nh ty . Suy ra phng trnh c v hn nghim hu t dng.
nh l 14 c chng minh.
3

2 17 36
Vi r 3, v 1 cng thc ny suy ra 3 + .
15 75 25
nh l 14 c hai h qu sau y
H qu 1. Vi mi s t nhin n th x3 y3 z 3 nt 3 c v hn nghim t nhin x, y, z ,t m

x, y, z, t 1 .
H qu 2. Vi mi s t nhin s 3 mi s hu t dng c v hn cch biu din thnh tng lp
phng ca s s hu t dng.
Nu trong chng minh nh l 14 ta chn v gn v ln hn

3r th u 0, 1 u 0 , 1 u 2 0 ,

2
do s 0, z 0 , t 0 , y 0 , x 0 . T y ta c nh l: mi s hu t dng u c v
3
3
hn cch biu din di dng x3 y 3 z 3 vi x, y, z l cc s hu t dng. p dng vi r t
khi r , t l cc s hu t dng ta c
u2

nh l 15. Mi s hu t u c v hn biu din di dng x3 y3 z 3 t 3 vi x, y, z, t l cc s


hu t dng.
12. Tng ca bn lp phng
Cch y vi nm tc gi t ra gi thuyt C: mi s nguyn g u c v hn biu din di dng

x3 y3 z 3 t 3 vi x, y, z, t l cc s t nhin. Gi thuyt ny c chng minh bi Demyanenko


[2] vi cc s nguyn m -1000 g 1000 v vi mi g 14 (mod 9). Chng minh da trn
3
3
3
3
trn kt qu sau ca L.J.Mordell [13]: nu g= a b c d vi a, b, c, d l cc s nguyn

a b c d 0

v a b hoc c d v hn na nu a b c d khng phi bnh phng

mt s t nhin th gi thuyt C l ng vi g . Vi g=0 gi thuyt C l h qu trc tip ca ng


3
3
3
3
thc 0= n 1 n 1 vi mi n 1, 2,... . Ta trnh by chng minh gi thuyt C vi g=1.

Ch cn chng minh phng trnh t 13 u 14 t 3 u 17 1 c v hn nghim


3

nguyn t , u . Phng trnh ny ng vi t u 0 v nu n ng vi t v u th cc s t1 =

11t 6u 173, u1 20t 11u 315 cng tha mn phng trnh. Chng hn do t 0 v u 0 tha
3
3
3
3
mn phng trnh nn t1 173, u1 315 cng th v hn na 186 329 - 176 - 332 =1.

Do phng trnh x3 y3 z 3 t 3 1 c v hn nghim t nhin x, y, z, t suy ra tn ti v hn s


t nhin n m n v n 1 u l tng hai lp phng dng.
4
3
3
4
Nu g=2 th gi thuyt C c suy ra t ng thc 2 9n 1 9n 1 9n 3n vi mi
3

n 1, 2,... c bit vi n 1 ta c 2 93 13 83 63 .
3
3
3
2
Nu g=3 th gi thuyt C c suy ra t 3 6n 1 1 6n 1 6n vi n 1,2.... .
3

268 | Tng ca bn lp phng. Mt s tng cc lp phng c gi tr bng nhau

Ta cng bit mt s nghim nguyn dng ca phng trnh x3 y3 z 3 t 3 1 . Chng hn

43 43 63 73 = 1.43 383 583 633 = 1.43 373 633 673 1 . Gn y J.A.Gabowicz [1]
chng minh rng phng trnh ny c v hn nghim t nhin.
Mt khc t ng thc 6n3 1 13 6n2 6n3 1 1 vi n 1, 2,... d dng chng minh
3

tn ti v hn nghim t nhin x, y, z, t ca phng trnh x3 y3 z 3 t 3 1 .


A.Makowski ([1] trang 121) chng minh phng trnh x3 y3 z 3 t 3 2 c v hn nghim t

3n

nhin. iu ny suy ra t ng thc

1 3n3 1 3n2 3n2 2 vi n 1, 2,..


3

3
3
3
Phng trnh ny cn c cc nghim khng nhn c t ng thc ny chng hn 235 - 3 - 69 2333 = 2.6833 6503 3533 23 2 .

Bi tp. Chng minh rng tn ti v hn s t nhin g m cc phng trnh g= x3 y3 z 3 t 3 , g=

x3 y3 z 3 t 3 v g= x3 y3 z 3 t 3 u c v hn nghim t nhin x, y, z, t .
3
3
Chng minh. Cc s g = a b vi b a l cc s t nhin ty tha mn tnh cht yu cu v

(Schinzel v Sierpinski [2] trang 26-27). D dng chng minh rng mi s nguyn u c v hn

cch biu din nh l tng ca 5 lp phng. ng thc 6t t 1 t 1 t t chng


3

t mi s nguyn chia ht cho 6 u l tng ca 4 lp phng. chng minh mi s nguyn u


c v hn cch biu din thnh tng ca 5 lp phng ta ch cn chng minh rng vi mi s
nguyn, tn ti s t nhin ln m hiu ca s nguyn ban u v lp phng ca s t nhin
ny l chia ht cho 6. K hiu g l s nguyn ty , r l phn d nhn c khi chia g cho 6. Khi
3
g= 6k r . Vi mi s t nhin n ta c 6k r 6n r r r 0 mod 6 do 6 g- 6n r .

13. Mt s tng cc lp phng c gi tr bng nhau


Trong mi lin h vi nh l 13 mt cu hi th v c t ra l vi cc s t nhin m v n m
no th phng trnh
(44)

x13 x23 .... xm3 y13 y23 ... yn3

c nghim l cc s t nhin phn bit x1, x2 ,..., xm , y1, y2 ,..., yn . R rng phng trnh khng c
nghim nh vy vi n m 1 . nh l 13 suy ra phng trnh cng khng c nghim khi
m 1, n 2 . Ta chng minh nh l sau y
nh l 16. Phng trnh (44) vi n , m l cc s t nhin n m l c nghim t nhin phn bit
x1, x2 ,..., xm , y1, y2 ,..., yn khi v ch khi khng xy ra trng hp m n 1 v m 1, n 2
(Sierpinski [24]).
Ta ch cn chng minh iu kin .
B . Vi mi s t nhin n 2 th lun tn ti s t nhin m lp phng ca n l tng ca n
lp phng dng phn bit.
3
3
3
3
3
3
3
3
3
Chng minh b . Cc cng thc 6 3 4 5 v 13 5 7 9 11 chng minh b
trong trng hp n 3 v n 4 . Gi s b ng vi s t nhin n 2 . Tn ti cc s t nhin
a1 a2 ... an a0 tha mn a03 a13 a23 ... an3 . V vy

6a0

3a1 4a1 5a1 6a2 6a3 ... 6an


3

CHNG 11. BIU DIN CC S T NHIN THNH TNG CC LY THA BC k KHNG M | 269

v hn na 3a1 4a1 5a1 6a2 ... 6an suy ra b ng vi n 2 . Vy nu b ng vi s


t nhin n th n ng vi n 2 . Kt hp vi nhn xt trn suy ra b ng vi mi s t
nhin n 2 .
H qu. nh l 16 ng vi mi s t nhin m, n vi m 3 , n 3 .
Chng minh h qu. Nu m 3 v n 3 th theo b tn ti cc s t nhin
b1 b2 ... bn1 a1 tha mn a13 b13 b23 ... bn31 v cc s a2 a3 ... am bn tha mn
a23 a33 ... am3 bn3 . Hn na c th gi s a2 a1 v nu khng ta c th thay cc s

a2 , a3 ,..., am , bn bi tch ca chng vi a1 1 . Do cc s a1, a2 ,..., am , b1, b2 ,..., bn l phn bit.


Cng cc ng thc theo v suy ra a13 a23 ... am3 = b13 b23 ++ bn3 . y l iu cn chng minh
trong nh l 16. H qu c chng minh.
chng minh nh l 16 trong trng hp tng qut ch cn kim tra hai trng hp m 2 v
m 3 v n m . Nu m 2 , n 2, 3, 4, 5 th nh l 16 suy ra t cng thc

93 103 13 113 ,

73 83 13 53 93 ,

63 363 43 53 273 303 , 263 283 23 33 43 53 343 .


Nu m 2 v n 5 th theo b tn ti cc s t nhin b1 b2 ... bn3 < a1 tha mn
a13 b13 b23 ... bn33 suy ra

a13 6a1 3a13 4a1 5a1 b13 b23 ++ bn33


3

m a1 3a1 4a1 5a1 6a1 suy ra nh l ng vi n v m . Nu m 3, n 3 ,4 th nh l 16 suy


3
3
3
3
3
3
3
3
3
3
3
3
3
ra t cng thc 1 12 15 2 10 16 , 12 13 14 3 9 10 17 . Nu m 3 , n 4
th theo b tn ti cc s t nhin b1 b2 .. bn2 a1 m a13 b13 b23 ... bn32 suy ra

a13 2a1 16a1 9a1 15a1 b13 b23 ++ bn32


3

v do v a1 2a1 9a1 15a1 16a1 suy ra nh l 16 ng vi m, n .


nh l c chng minh hon ton.
14. Tng ca cc trng phng
T nh l cui cng ca Fermat trong trng hp s m 4 (Chng 2 mc 6) ta thy khng c cc
trng phng no l tng ca hai trng phng dng. Gi thuyt ca Euler ni rng khng c cc
trng phng l tng ca ba trng phng dng.
4
4
4
Tuy nhin c cc trng phng l tng ca bn trng phng chng hn 353 30 + 120 +
2724 3154 , 154 44 64 84 94 144 , 914 144 244 344 494 584 844 .

nh l 17. Vi mi s t nhin n 3 tn ti trng phng l tng ca n trng phng dng.


Chng minh. K hiu S l tp tt c cc s t nhin n 1 m tn ti trng phng l tng ca n
trng phng dng. Ta va chng minh 4, 5, 6 thuc S.
By gi ta chng minh nu n, m thuc S th m n 1 cng thuc S. Tht vy nu m v n thuc S
th tn ti cc s t nhin a1 a2 ... am a0 v b1 b2 ... bn b0 tha mn
a04 a14 a24 ... an4 , b04 b14 b24 ... bn4 . V vy

Hn na a1b1 a1b2 ... a1bn a2b0 a3b0 ... amb0. suy ra m n 1 thuc S.

270 | Tng ca cc trng phng. nh l Waring

Ta lu nu tp S cha 4,5 th theo tnh cht trn suy ra S cha mi s t nhin 7 . Tht vy, v
4,5 thuc S nn 4+4-1=7, 5+4-1=8, 5+5-1=9 thuc S, theo quy np suy ra nu m thuc S th
m 3k vi k 1,2... cng thuc S (v m 3k m 3 k 1 4 1 ). H qu l S cha mi s c
dng 7 3k , 8 3k , 9 3k vi k 0, 1, 2,..., ngha l mi s t nhin 7 . Vy S cha mi s t
nhin 3 . nh l 17 c chng minh.
C mt s s t nhin c th biu din thnh tng ca hai trng phng theo hai cch. Chng hn
1334 1344 = 594 1584 . Tuy nhin ta cha bit s t nhin no c nhiu hn hai biu din nh vy
4
4
4
4
4
4
(khng tnh cc hon v). ng thc sau y ng 8 9 17 3 13 16 . Lu rng ng
thc sau y suy ra mi s hu t l tng ca bn trng phng hu t:

442554 x 8 255 2 x 8 255 2 x 32 x 255 32 x 255 .


4

C th chng minh rng vi mi s t nhin n 4 th lun tn ti ly tha bc 5 l tng cc ly


tha bc 5 ca n s t nhin khc nhau. Chng hn (Lander v Parkin [2] v A.S.Bang [1])

1445 275 845 1105 1335 , 125 45 55 65 75 95 115 ,


925 25 95 115 225 515 585 895 , 325 = 35 65 75 + 85
105 115 135 145 155 165 185 315
P.Erdos chng minh vi mi s t nhin m th lun tn ti s t nhin k m m vi mi s t
nhin n km th u tn ti s t nhin ln ,m tha mn mi s t nhin ln hn ln ,m u l tng ca
n s phn bit m mi s l ly tha dng bc m .

15. nh l Waring
Nm 1770 Waring t ra nh l sau m khng km theo chng minh: vi mi ly tha s tn
ti s t nhin k m mi s t nhin n l tng ca k ly tha khng m bc s .
nh l ny c chng minh bi D.Hilbert nm 1909. Chng minh s cp ca nh l Waring
c trnh by bi Yu.V.Linnik [2] v da trn tng ca L.Schnirelman. Chng minh ny c
trong cun sch ca A.Ya.Khinchin [1]. Vi s 1 nh l Waring hin nhin ng. Nu s 2 nh l
4 (Lagrange) cho khng nh vi k 4 . Vi s 3 Waring nhn xt rng c th chn k bng 9,
ngha l mi s t nhin u l tng ca 9 (hoc t hn) cc lp phng dng. Mi ti nm 1909
A.Wieferich mi chng minh iu ny l ng. Vi s 4 Waring ch ra rng k 19 . Kt qu ny
c chng minh gn y (theo cch khng s cp) bi R.Balasubramanian, F.Dress v
J.M.Deshonilles [1]. Ta s trnh by chng minh s cp rng k c th chn l 50 (xem nh l 18).
Vi mi s t nhin s k hiu g s l s t nhin nh nht k m mi s t nhin u l tng ca

k (hoc t hn) cc ly tha bc s . Khi nh l Waring ni rng vi mi s t nhin s th s t


nhin g s tn ti. Ta chng minh
(45)

3 s
s 2 2 ,
2
s

s 1,2,...

t
(46)

3 s
n 2 1 .
2
s

R rng n l s t nhin v v x x ta c
(47)

n 3s .

T nh ngha ca g s suy ra tn ti cc s nguyn khng m xi ( i 1,2,... ,g s ) tha mn


(48)

n x1s x2s ... xgs s .

CHNG 11. BIU DIN CC S T NHIN THNH TNG CC LY THA BC k KHNG M | 271

Theo (47) th mi s xi i 1,2,..., g s phi nh hn 3. Do cc s xi ch nhn cc gi tr 0, 1, 2.


Gi s trong cc s c k s 2, l s 1 v r s 0. Khi k , l , r l cc s nguyn khng m v

g s k l r k l

(49)
Vi

n 2s k l .

(50)

3 s
3 s
V vy n 2s k , v theo cng thc (46) th n 2s ta nhn c k < ngha l
2
2
3 s
k 1 .
2

(51)

s
T (50) ta c l n 2 k v do

k l k n 2s k n 2s 1 k .

(52)

V s l s t nhin nn 2 1 cng l s t nhin. Nhn (51) vi s ny suy ra


3 s
2s 1 k 2s 1 1 . V vy t (49),(52) v (46) ta c
2

9
2
suy ra (45). Nu s 2 t (45) suy ra g 2 2 2 4 2 2 v do g 2 4 . Nhng ta
4
27
3
bit g 2 4 v nu s 3 th t (45) suy ra g s 2 2 9 .
8
Tn ti cc s t nhin (chng hn 23) khng th biu din thnh tng ca 8 lp phng khng
m. Nh cp trn, Wieferich chng minh g 3 9 .

81
4
Nu s 4 th (45) suy ra g 4 2 2 19 .
16
Theo (46) th tn ti s cc t nhin (v d 79) khng th biu din thnh tng ca 18 trng
phng khng m. Blasubramanian, Dress v Deshonilles chng minh g 4 19 .
Nu s 5, t (45) suy ra g 5 37 . J.R.Chen [1] chng minh g 5 37 .

3 s
l.E.Dickson [4],[5] (xem Pillai [3]) chng minh cng thc g s 2s 2 ng vi
2
6 s 400 (tht ra u ny cng ng vi s 5 ). K.Mahler [1] chng minh cng thc trn l
ng vi mi s ln s v R.M.Stemmler [1] kim tra vi 400< s 200000 .
Vi mi s t nhin s k hiu G s l s t nhin nh nht k m mi s t nhin ln (ngha
l tt c cc s tr ra nhiu nht l hu hn trng hp) u biu din c thnh tng ca k ly
tha khng m bc s . Ta chng minh c G(2) 4, G(3) 7, G(4) 16, G(5) 21, G(6) 31
(Davenport [1] v Vaughan [1],[2]). Bt ng thc G(3) 7 c chng minh bi Yu.V.Linnik [1]
nm 1942 v c lm n gin hn bi G.L.Waton [1], xem thm McCurley [1].

272 | Tng ca cc trng phng. nh l Waring

By gi ta trnh by phng php s cp chng minh g 4 50 .


u tin cn nhc li ng thc c E.Lucas tm ra nm 1876 di y

Vi mi s t nhin n chia ht cho 6, ngha l n 6m vi m l s t nhin, th theo nh l 4 ta c


m a 2 b2 c2 d 2 vi a, b, c, d l cc s nguyn khng m. V vy n 6a 2 6b2 6c2 6d 2 .
Nhng theo nh l 4 th tn ti cc s nguyn khng m x1, x2 , x3 , x4 m a x12 x22 x32 x42 . V
vy theo (53) ta c 6a 2 a14 a24 ... a124 vi ai i 1,2,...,12 l cc s nguyn khng m. Ta biu

din cc s 6b2 ,6c 2 ,6d 2 di dng tng t thnh tng ca 12 trng phng. T y suy ra
n 6m l tng ca 48 trng phng. Do ta chng minh c mi s t nhin chia ht cho 6
4
u l tng ca 48 trng phng. Mi s t nhin khng ln hn 95 u c dng 2 k r vi
0 k 5, 0 r 15 v do l tng ca 20 trng phng. Gi s s t nhin n ln hn 95. Khi
n 6m r vi m 15 v 0 r 5 . Cc s m, m 2, m 13 dng v do vi r 0, 1, 2, ..., 5
4
4
4
4
4
4
4
ta c n 6m , n 6m 1 , n 6m 1 1 , n 6 m 13 3 , n 6 m 2 2 , n 6 m 2 1 + 2

tng ng. V vy mi s t nhin u l tng ca 50 trng phng. iu phi chng minh.


nh l 18. Mi s t nhin u l tng ca 50 trng phng.
S dng nh l Gauss c th chng minh g 30 (Dress [1]). Vi mi s t nhin s k hiu v s l
s t nhin nh nht k m mi s t nhin l tng ca k ly tha bc s cc s nguyn. D thy
v 2 3 v 4 v 3 5 tuy nhin ta khng bit v 3 bng 4 hay 5. C th chng minh

9 v 4 10 , 5 v 510. By gi ta chng minh vi mi s t nhin s th v s tn ti. Ta c


ng thc P.Tardy [1] (xem Dickson [7] tp 2 trang 723, 728)

s
Vi s l s t nhin v tng v tri ly trn 2 dy 1, 2 ,..., s cc phn t nhn gi tr 0 v 1. V
s
vy vi x1 x2 ... xs 1 suy ra mi s nguyn chia ht cho s !2 u l tng ca 2 s ly tha bc

s . Mt khc v mi s nguyn u c dng 2!2s k r vi k , r l cc s nguyn v 0 r s!2s1 nn


s
s 1
s
s 1
mi s nguyn u l tng ca 2 s !2 ly tha bc s . Ta c v s 2 s !2 vi mi s 1, 2,...

CHNG 12
MT S BI TON CA L THUYT CNG TNH CA CC S
1. Phn hoch dng tng
Leibmiz, Bernoulli v sau l Euler l nhng ngi u tin nghin cu bi ton tnh s g n tt c
cc biu din s t nhin n cho trc thnh tng ca cc s t nhin khng tng. Bi ton ny
c bit n vi tn gi l bi ton phn hoch dng tng. Sau y l 10 gi tr u tin ca hm
s g : g1 1 , g 2 2 , g3 3 , g 4 5 , g5 7 , g6 11 , g7 15 , g8 22 , g9 30 , g10 42 . Mac
Mahon tnh c g100 1905692292 , g200 3972999029388 . C th chng minh g n chnh l
h s ca khai trin dng chui

g n x n vi x 1 .

n
n 1
n 1 1 x

K hiu hn l s cc biu din ca n thnh tng ca dy tng cc s t nhin. Kh vi x 1,

1 x 1 h x
n

n1

n1

. Cc s g n ( n 1, 2,... ) tha mn cng thc truy hi

ngn (n) g1 (n 1) g2 (n 2) ... gn1 (1)


T cng thc ny ta xy dng c quy tc tnh cc s g n (Vahlen [1]). Sau y l 10 gi tr u
tin ca hm hn : h1 1 , h2 1 , h3 2 , h4 2, h5 3 , h6 4 , h7 =5, h8 6 , h9 8, h10 10 .
K hiu k n l s tt c cc php phn tch n thnh tng ca cc s t nhin m hai phn tch c
coi l khc nhau ngay c khi chng ch sai khc th t cc hng t. Bng quy np ta chng minh
c kn 2n1 vi n 1, 2, ... c bit s 4 c 8 cch phn tch khc nhau thnh tng ca cc s t
nhin. l 4 3 1 1 3 2 2 2 1 1 1 1 2 1 1 1 2 1 1 1 1 .
Cui cng k hiu ln l s cch phn tch s t nhin n thnh tng cc s l khng tng. Vi x 1

ta c

ln x n . Ta c ln hn vi mi n 1, 2, ...

2 n 1
1

x
n 1
n1

K hiu qn l hm (vi i s l s t nhin n ) xc nh bi s cch phn hoch tp hp n phn


t thnh cc tp khng rng ri nhau, hai phn hoch khc nhau ch khi cc tp con trong phn
hoch khng tng ng bng nhau. Cc gi tr u tin ca hm s ny l q1 1, q2 2 ,

q3 5, q4 15, q5 52 . Ta c cng thc tnh qn l ee 1 qn x m / n!. (Whitworth [1] trang 88).


x

n 1

n
k 1 k
n

Ta cng c (Rota [1]) qn1 1 qk . .


Ta tnh s cch khc nhau biu din mt s nguyn thnh tng ca cc ng d khc nhau
modulo m trong dy 1,2,, m 1 . Cc biu din cng c tnh theo ng d modulo m .
M.A.Stern [1] chng minh rng nu p l s nguyn t l th vi mi ng d modulo p c

p1
ng 2 1 / p biu din nh vy vi cc hng t l 1,2,, p 1 . V d vi p 5 ta c

0 1 4 2 3 1 2 3 4 mod 5 ,
1 1 2 4 1 2 3 mod 5 ,
2 2 3 4 1 2 4 mod 5 ,
3 3 1 2 1 3 4 mod 5 ,
4 4 1 3 2 3 4 mod 5 .

274 | Phn hoch dng tng. Biu din thnh tng ca n hng t khng m

2. Biu din thnh tng ca n hng t khng m


Ta s chng minh rng nu n v k l hai s t nhin cho trc th s Fn ,k tt c cc cch biu
din k thnh tng ca n s nguyn khng m m hai biu din l khc nhau ngay c khi th t

n k 1
k
. Tht vy ta c F1,k 1 .
k
k

cc hng t l khc nhau l

n k 1
ng vi mi k 1, 2,.... . Khi ta c
k

Gi s vi n l s t nhin th cng thc Fn ,k

Fn1,k Fn,k Fn ,k 1 F n ,k 2 ... Fn ,1 1


n k 1 n k 2 n k 3
n

.... 1.
k k 1 n 2
1

n k n k 1 n k 1

.
k k k 1

Ta li c

n k n k 1 n k 2
n n

... .
k k k 1
1 0

Suy ra

n k
n k 1
, suy ra cng thc Fn,k
(vi k 1, 2,... ) ng vi mi n .
k
k

T Fn1,k

Mt cch chng minh khc l nh sau: vi mi phn tch k a1 a2 ... an ca s t nhin k


thnh tng ca n s nguyn khng m ta xt dy cc s li a1 a2 ... ai i vi i 1,2,..., n 1 .
R rng dy ny cha cc s t nhin tng m mi s u khng vt qu n k 1 . S cc dy nh

n k 1 n k 1

.
n 1 k

vy ng bng

T.Skolem [3] xt bi ton trong s t nhin n tha mn tnh cht: tp cc s 1, 2, , 2n c


th chia thnh n cp ai , bi i 1,2,..., n m bi ai i vi mi i 1,2,, n . Nu n c tnh cht
ny th

i 1

i 1

bi ai 1 2 ... n n n 1 / 2 . Nhng do cc s a1, b1, a2 , b2 ,..., an , bn ng bng


n

vi cc s 1,2,, 2n nn

a b 1 2 ... 2n n 2n 1 . V vy b 4 n 5n 3
i 1

i 1

i 1

nhng

y khng phi s nguyn v n ng d vi 2 hoc 3 (mod4). Ngc li T.Skolem chng minh


(O.Keefe [1]) nu n ng d vi 0 hoc 1 (mod 4) th phn tch trn l tn ti. V d vi n 4
th cc cp l (6,7), (1,3), (2, 5), (4, 8). Nu n 5 th cc cp l (2,3), (6,8), (7,10), (1,5), (4,9).
3. Ma phng
2
Hnh vung cha cc s 1, 2, , n trong cc ring bit m tng cc s trong hng ngang, ct dc,
ng cho u bng nhau c gi l ma phng bc n . D dng tnh ra gi tr chung ny l

1
n n 2 1 . Trng hp n 1 l tm thng. Vi n 2 th khng tn ti ma phng tng ng.
2
Vi n 3 ta c ma phng
8

CHNG 12. MT S BI TON CA L THUYT CNG TNH CA CC S | 275

Vi n 4 ta c cc ma phng

Di y l cc v d v ma phng vi cp n 5,6,7

Tn ti duy nht (sai khc mt php quay v phn x gng mt ma phng) vi n 3 . Frenicle
ch ra 880 ma phng vi n 4 . Tn ti ma phng cp ty n 3 (Bieberbach [1]). Chng
minh sau y thuc v A.Makowski [8]. u tin ta chng minh rng t hai ma phng Qn v Qm
vi bc n v m ln lt th ta c th nhn c mt ma phng Qnm bc nm . Tht vy, ta ch cn
thay cc cha phn t i ca ma phng Qm bng ma phng nhn c t vic cng thm

n2 i 1 vo tt c cc trong ma phng Qn . Khi ta thu c ma phng bc mn vi tng


1
1
cc hng, ct v ng cho ca Qnm u bng mn n 2 1 n3m m2 1 . T chng minh ny ta
2
2
cng c ngay mt phng php xy dng cc ma phng bc 3k , k 1, 2,..., t ma phng bc 3.
Ma phng bc l c gi l hon ho nu tng ca hai s trong hai i xng vi nhau qua
trung tm ng bng hai ln s trong trung tm. Mi ma phng bc 3 l hon ho (s trung
tm bng 5). Tuy nhin tn ti ma phng bc 5 m khng hon ho. Stifel ch ra mt v d nh
th. Di y l mt ma phng hon ho bc 5

276 | Ma phng

Ma phng bc 7 trnh by trn l hon ho.


2
2
C cc ma phng cha n s nguyn phn bit nhng khng phi 1, 2, , n . V d

Mt v d tng qut hn l vi s 18 ta c ma phng

Ma phng (hiu theo ngha rng) c th ch cha cc s nguyn t phn bit.


V d (xem Moessner [2] v [3])

A.Makowski nhn xt rng nu cc phn t ca cp s cng a b, 2a b,..., n2a b u l s


nguyn t th thay cc s i bi cc s ia b trong hnh vung cha cc s 1, 2,..., n2 ta nhn c
ma phng (theo ngha rng) ch cha cc s nguyn t. T gi thuyt H suy ra tnh tn ti ca x
m tt c cc s x 1, 2 x 1,..., n2 x 1 u nguyn t. V vy t gi thuyt H c th suy ra tnh tn
ti ca ma phng bc b vi mi n 2 m ch cha cc s nguyn t.
A.Moessner xy dng ma phng bc 8 ch cha cc s tam gic t0 ,...., t63 . Ma phng ny c
tng cc hng ngang, ct dc v ng cho ng bng s tam gic t104 (Moessner [1]).
Ma phng (theo ngha rng) gi l cn ma phng nu n to thnh bi cc s s, s 1,..., s n2 .
R rng cc ma phng nh vy s tr thnh ma phng theo ngha thng thng nu tt c cc
s cng c tr i s 1 . L. Bieberbach [1] thng bo vo nm 1544 Michael Stifel a ra
nhn xt rng cn ma phng khi b i hng u tin, hng di cng, ct u tin v ct cui
cng th vn l cn ma phng. C th chng minh tn ti cc ma phng nh vy vi s hng ln
hn 4 ty . y l v d ca Stifel

CHNG 12. MT S BI TON CA L THUYT CNG TNH CA CC S | 277

Ma phng ny (theo ngha thng thng) c to thnh t cc s 1, 2, , 25. Sau khi b i hng
u tin, hng di cng, ct u tin v ct cui cng ta nhn c cn ma phng to thnh bi
9, 10, ... , 17.
Ta cng nghin cu cc hnh vung c to thnh bi cc s t nhin m tch ca cc s trong
mi hng, mi ct v ng cho u bng nhau. Di y l mt s hnh vung nh vy
(Goodstein [1]):

Qu trnh pht trin l thuyt v cc ma phng tnh ti th k 20 c tng kt trong


P.Bachmann [1]. Rt nhiu phng php xy dng ma phng khc c trnh by trong
Postnikov [1].
4. nh l Schur v cc h qu
B . Nu k l s t nhin, N [ek!] nu a0 a1 a2 ... aN l dy cc s nguyn v nu tp
hp cc hiu a j ai vi 0 i j N c chia thnh k lp ri nhau th tn ti t nht mt lp
cha cc hiu am al , an al , an am vi l , m, n no tha mn 0 l m n N .
Chng minh. Gi s phn chng tn ti s t nhin k m b khng ng. t K1 l lp cha
nhiu nht cc hiu c dng a j a0 trong 0 j N v t a j1 a0 , a j2 a0 ,, a jk a l cc
1

phn t ca lp K1 c sp xp theo ln. Khi ta c N k1k. Theo gi thit th k1 1 hiu


(1)

a j2 a j1

a j3 a j1 ,

a jk a j1
1

khng thuc lp K1 . Do chng thuc k 1 lp cn li.


K hiu K 2 l lp cha nhiu nht cc s k 2 cc hiu trong (1). Gi s K 2 cha cc hiu
(2)

a j a j1 ,

a j a j1 ,

a j a j1 ,.,

trong ... . R rng k1 1 k2 k 1 .


Nu s u tin trong (2) l hiu ca cp no trong k2 1 s cn li th ta c cc hiu
(3)

a j a j ,

a j a j , ,

khng thuc K1 v K 2 . Suy ra chng thuc k 2 lp cn li. K hiu K 3 l lp cha nhiu nht

k3 cc s trong (3). Ta c k2 1 k3 k 2 . Tip tc nh vy cho ti khi ta nhn c dy cc s

t nhin k1 , k2 ,...., ks vi s k v
(4)

k 1 ki 1 k i

vi

i 1, 2, ..., s 1 ,

trong ks 1 v nu ks 1 th lp lun trn tip tc v ta thu c ks 1 . Theo (4) suy ra

ki
ki 1
1

,
k i ! k i 1! k i !
Cng cc bt ng thc theo v ta c

i 1, 2, ...., s 1

k1
1
1
1
1

...
e .
k!
k 1! k 1! k 2 !
k s !

278 | nh l Schur v cc h qu

V vy N k1k ek ! 1 , mu thun vi nh ngha ca N. B uc chng minh.


nh l 1 (I.Schur [1], Leveque [2] tp 1 trang 60). Gi s vi s t nhin k th cc s 1, 2, ,
ek ! c chia thnh k lp. Khi tn ti t nht mt lp cha hai phn t ca dy v cha c
hiu ca chng.
Chng minh. Trong b t ai i, i 1, 2,..., ek ! v lu rng gia cc s 1, 2, .., ek ! tt c
cc hiu a j ai vi 0 i j ek ! u xut hin t nht mt ln v hn na an am

an a1 am a1 . Suy ra nh l c chng minh.


Lin quan ti nh l 1 c mt cu hi c t ra nh sau: cho trc mt s t nhin k , tm s t
nhin nh nht N N k c tnh cht ging nh s ek ! , ngha l nu tp hp 1, 2,, N c chia
thnh k lp th t nht mt lp cha hai phn t v c hiu ca chng.
T nh l 1 suy ra rng N k ek ! v vy N 1 2 , N 2 5 , N 3 6 . Mt khc, r rng

N 1 1 nn N 1 2 . Do cc s 1, 2, 3, 4 c th chia thnh hai lp l 1, 4 v 2, 3, m c hai lp


u khng cha hiu cc phn t ca chng suy ra N 2 4 do N 2 5 .

k
I.Schur, chng minh N k 1 3N k 1 (xem bi tp 1). V vy n k 3 1 / 2 , ng thc

xy ra ch trong cc trng hp k 1, 2, 3. L.D.Baumert [1] chng minh N 4 45 .


Gn y c lng Schur c lm cht hn. E.G.Whitehead [1] chng minh


1
N k k ! e , H.L.Abbott v D.Hanson [1] chng minh
24

(5)

N n m 2 N m 1 N n N m 1.

H.Fredericksen [1] chng minh N (5) 158 suy ra t ng thc (5) vi m 5 ta c


(6)

N k 1 315 k 1/5 ,

k 1, 2, 3,....

nh l 2. t 0 a0 a1 ... aN l dy cc s nguyn vi N = ek ! . Nu dy ny khng cha


cp s cng vi t nht ba phn t th mi phn hoch ca tp hp 1, 2, , aN thnh k lp c tnh
cht t nht mt lp cha hai s khc nhau v hiu ca chng.
Chng minh c suy ra t b v ba lu sau y
1) Trong cc s 1, 2, aN cha tt c cc hiu a j ai vi 0 i j N
2) an al an am am al ,
3) an am am al do cc s ny khng to thnh cp s cng.
ek !

H qu 1. Nu k l s t nhin, n 2
v tp hp cc s 1, 2,.., n c chia thnh k lp th t
nht mt lp cha hai s khc nhau v tng ca chng.
i
chng minh h qu ch cn xt ai 2 , i 0, 1, 2, ..., ek ! v p dng nh l 2 vi ch rng
i
dy 2 i 0, 1, 2,... khng cha bt k cp s cng 3 phn t no. H qu ca h qu 1 l

H qu 2. Nu tp tt c cc s t nhin c chia thnh hu hn lp th t nht mt trong cc lp


cha hai s t nhin khc nhau v tng ca chng (xem Rado [1]).

CHNG 12. MT S BI TON CA L THUYT CNG TNH CA CC S | 279

H qu 1 c pht trin tip bi R.W.Irving [1]. Khi 2

ek !

c th thay bng ek ! 2k 1 2 .
2

Lin quan ti nh l 2 ta c cu hi sau y: cho trc s t nhin k , tm s nh nht n n k c


tnh cht nu cc s 1, 2,, n c chia thnh k lp th t nht mt lp cha cc s khc nhau v
tng ca chng. R rng ta c n 1 3 . C th chng minh n 2 9 . Bt ng thc n(2) 9 c
suy ra t vic dy cc s 1, 2, ,8 c th chia thnh hai lp A 1, 2, 4, 8 v B 3, 5, 6, 7 m
khng c lp no cha tng ca hai s cha trong n. T chng minh n 2 9 ta ch cn
ch ra nu tp 1, 2, ,9 c chia thnh hai lp th t nht mt trong chng cha hai phn t v
tng ca chng. Chng minh tnh cht ny c trong Sierpinski [26] trang 427-428.
Ta c n 3 24 . Tht vy, dy 1, 2, ,23 c th chia thnh ba lp A, B, C m khng c lp no
cha tng ca hai phn t cha trong n. Tht vy ta c

A {1, 2, 4, 8, 11, 22} ,

B 3,5,6,7, 19, 21, 23 , C 9,10,12,13,14,15,16,17,18, 20 .


Mt khc G.W.Walker [1] thng bo rng n 3 24 , n 4 67 , n 5 197 nhng khng km
theo chng minh chi tit. Trong ti liu cng a ra bt ng thc 2n k n k 1 3n k
vi mi k 1, 2,... Mt phn ca bt ng thc ny c chng minh trong bi tp 2 v phn cn
li th khng ng theo h qu ca bt ng thc (6) trn.
Mt bi ton khc c lin quan ti ch ny l: cho trc s t nhin N, tm s ln nht r r N
m tn ti dy a1 , a2 ,..., ar cc s t nhin N v khng cha cp s cng no c ba phn t (dy

a1 , a2 ,..., ar gi l dy loi A cp N ).
D dng c r 1 1, r 2 r 3 2, r 4 3 , r 5 r 6 r 7 4 .
P.Erdos v P.Turan [1] chng minh r 8 4, r 9 r 10 5, r 11 r 12 6 , r 13 7 ,

r 14 r 15 r 16 r 17 r 18 r 19 8 , r 21 r 22 r 23 9 v ch ra r (20) 8 ,
nhng iu ny khng ng v A.Makowski [2] chng minh r (20) 9 .
1 k

k
G.Szekeres t ra gi thuyt r 3 1 2 vi mi k 0, 1, 2,... . Tuy nhin gi thuyt ny
2

1c /
khng ng v F.Behrend [1] chng minh r N N

log N

vi c l hng s (Salem v

Spencer [1],[2], Moser [3]). S.S.Wagstaff, Jr [1] tnh r n vi n 53 v tm c

r 24 r 25 10 , r 26 ... r 29 11 , r 30 r 31 12 , r 32 ... r 35 13 ,
r 36 ... r 39 14 , r 40 15 , r 41 ... r 50 16 , r 51 r 52 r 53 17.
Mt khc K.F.Roth [1] chng minh vi C thch hp ta c r n C

lim

r n
n

n
, v vy
log log n

0 . Kt qu cui cng ca Roth v kt qu ca Behrend ln lt c m rng cho dy

khng cha cp s cng k phn t bi Szemeredi [1] v R. A. Rankin [1].


Bi tp. 1. Chng minh t nh l Schur c th suy ra rng N k 1 3N k 1 .

280 | nh l Schur v cc h qu

Chng minh. T nh ngha ca N k suy ra tp hp cc s 1, 2,, N k 1 c th chia thnh k

lp m khng c lp no cha hiu ca hai phn t cha trong n. t Ki x1 , x2 , ..., xsi

i 1, 2,..., k

Li 3x1 1,3x1 1,3x2 ,...,3xsi ,1,3xsi , i 1,2,...,k


1

Lk 1 1, 4, 7, ..., 3N k 2.
D thy tt c cc lp Li i 1,2,....,k 1 u khng cha hiu ca hai phn t no cha trong
chng v tt c cc lp Li i 1, 2, ..., k 1 cha ton b cc s t nhin 1, 2, , 3 N k 2 . T
nh ngha ca N k 1 suy ra N k 1 3N k 2 v do N k 1 3N k 1 .
2. Chng minh rng n k 1 2n k 1 .
Chng minh. T nh ngha ca n k suy ra cc s 1, 2, , n k 1 c th chia thnh k lp m
khng lp no cha tng ca hai phn t cha trong chng. i vi cc lp no ta b sung mt lp
khc cha cc s n k , n k 1 , n k 2 ,, 2n k . Khi ta nhn c phn hoch ca tp cc
s 1, 2, , 2n k thnh k 1 lp c tnh cht nh trn. T nh ngha ca n k 1 suy ra

n k 1 2n k 1 .
1
2

Ghi ch. A.Makowski [3] chng minh bt ng thc mnh hn n k 1 2n k + k k 1 1.


3. Chng minh rng r m n r m r n (Erdos v Turan).
Chng minh c suy ra t nhn xt rng nu a1 a2 ... ar l cc dy A cp N th

a1 k ,...ar k cng l dy A cp N vi mi k a1.


4. Chng minh rng r 2n n vi n 8 (Erdos v Turan).
Chng minh quy np theo n suy ra t cng thc r 2.8 =8, r 2.9 9 , r 2.10 10 ,

r 2.11 11 v nu r 2n n th r 2 n 4 r 2n 8 r 2n r 8 n 4 .

5. Chng minh rng nu n m th r 2n m 1 r m r n .


Kt qu ny suy ra t vic nu a1 a2 ... ar n l dy A cp n v b1 b2 ... br m l dy A cp m
th vi n m , a1 a2 ... ar n 2ar n b1 1 2ar n b2 1 ... ar n 2ar n br m 1 l dy A
cp 2n m 1 cha r n r m phn t.

1 k

k
6. Chng minh rng r 3 1 2 (Erdos v Turan).
2

1 0

0
Chng minh. Chng minh quy np. Ta c r 3 1 r 1 1 2 v nu
2

theo bi tp 5 ta c

1
r 3k 1 1 r 2 3k 1 3k 1 1
2

2
2

1
1

r 3k 1 r 3k 1 2k 1
2
2

r 3k 1 2k th
2

CHNG 12. MT S BI TON CA L THUYT CNG TNH CA CC S | 281

7. Chng minh rng r 51 17.


Chng minh. Chng minh suy trc tip t nhn xt ca S.Maslowski rng dy 1, 2, 5, 6, 12, 14, 15,
17, 21, 35, 38, 39, 42, 44, 47, 48, 51 khng cha cp s cng ba phn t no.
M.Hall Jr. [2] chng minh s tn ti ca tp Z cc s t nhin phn bit m mi s t nhin l
hiu ca ng mt cp trong tp Z . Ta s xy dng dy v hn cc s t nhin Z c tnh cht nh
vy (Browkin [2]). t a1 1, a2 2. Hn na k hiu n l s t nhin v gi s cc s

a1 , a2 ,..., a2 n xc nh. t a2 n1 2a2 n . Gi s rn l s t nhin nh nht khng biu din c


di dng a j ai vi 1 i j 2n 1. Ta xc nh a2 n 2 nh l a2 n1 rn . Ta thy dy a1 , a2 ,...
c nh ngha bng quy np. By phn t u tin l 1, 2, 4, 8, 16, 21, 42. T nh ngha ca rn
suy ra cc s 1, 2, , rn u c dng a j ai vi 1 i j 2n 2 . V vy rn 1 rn vi mi n 1, 2,...
Do mi s t nhin c th biu din di dng a j ai vi cc ch s i, j thch hp.
hon thin chng minh ta ch cn ch ra rng vi mi s t nhin h, k , l , m vi h k v
l m, k m th ak ah am al . Gi s ngc li ak ah am al . Do m k h 1 ta c m 3 .
Nu m l ngha l m 2n 1 vi n l s t nhin th a2n1 al ak 2am1 2a2n a2n1 , v l.
Nu m chn ngha l m 2n 2 trong n l s t nhin th nu l 2n 1 ta c am al =

a2 n2 a2 n1 = rn m do ak ah am al suy ra rn ak ah trong h k m 1 2n 1 , mu
thun vi nh ngha ca rn . Nu l 2n 1 (v l m nn gi tr l 2n 1 b loi b) vi k 2n 1
ta c am ak al ah suy ra v k m, ta c h l 2n v am ak a2 n2 a2 n1 rn do
rn al ah vi h l 2n , mu thun vi nh ngha ca rn . Cui cng nu l 2n 1 v k 2n 1
th a2 n 2 am al ak ah al ak a2 n a2 n = a2 n 1 , v l.
Vy ta thy dy a1 , a2 , ... c tnh cht yu cu.
S dng tin chn th tnh cht tng t c th chng minh cho cc s thc. Ngha l tn ti tp
hp X cc s thc m mi s thc dng c th biu din duy nht di dng hiu hai phn t
ca X (Picard [1], Lindenbaum [1]).
5. Cc s l khng c dng 2k p vi p nguyn t
Nm 1849 A.de Polignac [1] t ra gi thuyt rng mi s l n 1 u c dng 2k p vi k l s
t nhin v p l s nguyn t hoc l 1. Nm Erdos [10] chng minh tn ti v hn s l m gi
thuyt sai (Van de Corput [3]).
nh l 3 (Erdos [10]). Tn ti cp s cng v hn cc s l khng c dng 2k p , k = 0,1,2, p
nguyn t.
B . Mi s t nhin u tha mn t nht mt trong su ng d thc sau

1 k 0 mod 2 , 2 k 0 mod 3 , 3 k 1 mod 4 ,

k 3 mod8 , 5 k 7 mod12 , 6 k 23 mod 24 .

Chng minh b . Nu k khng tha mn (1) hoc (2) th n khng chia ht cho 2 v 3, do n
c dng 24t r vi t l s nguyn v r l mt trong cc s 1, 5, 7, 11, 13, 17, 19, 23. Th trc tip
ta thy k tha mn (3), (3), (5), (4), (3), (3), (4), (6) tng ng.
H qu. Nu k l s nguyn khng m th t nht mt trong cc ng d thc sau ng

7 2k 1 mod 3 ,
9 2k 2 mod 5 ,
11 2k 27 mod13 ,

8 2k 1 mod 7
10 2k 23 mod17
12 2k 223 mod 241 .

282 | Cc s l khng c dng

2k p vi p nguyn t

Chng minh h qu. Kim tra trc tip ta nhn thy 22 1 mod 3 , 23 1 mod 7 , 24 1 mod 5 ,

28 1 mod17 , 212 1 mod13 , 212 1 mod 241 suy ra 224 1 mod 241 . T y suy ra cc ng
d thc (1), (2), (3), (4), (5), (6) suy ra (7), (8), (9), (10), (11), (12) tng ng.
Chng minh nh l. T nh l s d Trung Hoa suy ra tn ti s t nhin a tha mn cc ng
3
7
d thc a 1 mod 2 , a 1 mod3 , a 1 mod 7 , a 2 mod 5 , a 2 mod17 , a 2 mod13 ,

a 223 mod 241 , a 3 mod31 v hn na tn ti cp s cng v hn cc phn t a m mi s


u tha mn cc ng d thc ny. R rng cc phn t ca cp s cng ny l l. Nu a l phn
k
t bt k ca cp s cng th n tha mn cc ng d thc. T h qu ca b suy ra s a 2
chia ht cho t nht mt trong cc s nguyn t 3, 7, 5, 17, 13, 241. Mt khc a 3 mod 31 v vi
k
5
mi k 1, 2, ... s 2 ng d vi mt trong cc s 1, 2, 4, 8, 16 (mod 31) bi v 2 1 mod 31

suy ra a 2 ng d vi mt trong cc s 2, 1, -1, -5, -13(mod31). Nhng khng c s no ng


k
d (mod 31) vi cc s 3, 7, 5, 17, 13, 241. V vy s a 2 khng phi mt trong cc s , nhng
k
mt khc n chia ht cho t nht mt trong cc s . V vy n l hp s. Do a 2 khng phi
s nguyn t vi mi s nguyn khng m k suy ra a khng c dng a 2k p , trong
k 0, 1, 2,..., v p l s nguyn t. Vy ta thy cc phn t ca cp s cng nh ngha trn c
tnh cht cn thit. nh l 3 c chng minh.
k

Chng minh ca nh l 3 suy ra tn ti v hn s t nhin n m vi mi s nguyn khng m k


k
k
th s n 2 (v v vy c s n 2 ) u chia ht cho t nht mt trong cc s 3, 7, 5, 17, 13, 241.
K hiu P l tch ca cc s nguyn t nh vy. Nh trn ta chng minh s n 2
s

nguyn

p P.

Nhng

k P

1 mod P

n2

2 P 1

k P 1

0 mod p

c c

suy

ra

n.2k 1 0 mod p vi n ln (v d n 241 ) t ta c hp s n.2k 1 . Ta c h qu sau


k
H qu. Tn ti v hn s t nhin n m cc s n .2 1 vi k 0, 1, 2,..., u l hp s (xem
Sierpinski [28] v Chng 10 mc 4 bi tp 3).

nh l 4 (R.Crocker). Tn ti v hn s t nhin khng th biu din thnh tng ca hai ly tha


bc nguyn khng m khc nhau ca 2 v mt s nguyn t.
2
Chng minh. Ta chng minh cc s 2 1, n 3, 4,... c tnh cht nh vy.
n

2
k
l
Tht vy gi s vi s t nhin n 2 ta c 2 1 2 2 p vi k , l l cc s nguyn v
n

2
k
k l 0 . Khng th xy ra trng hp l 0 v nu ngc li ta s c p 2 2 2
n

2 22

2k 1 1 v do 2n k , k 1 2n 2 suy ra 22n1 2k 1 22 n1 22 n2 = 22
n

2
nh vy 2

22 2 26
3

2k 1 26 1 >1, mu thun vi vic p l s nguyn t. Vy l 1 v do k 1 .

h
K hiu h l ly tha khng m ln nht m 2 l c s ca k l .

k
l
2
k l
2
l
k l
h
2
S k l / 2 l v 2 1 2 1 . Do p 22 n 2 2 1 2 1 2 2 1 suy ra 2
h

p nguyn t nn p 22 1 . Khi 22 2k 2l 22
h

p m

n
. Do 2 k 1 suy ra s 2l 22 2 chia
k

2 n 2
h
h
2k 2 2l 2 1 , v l v v
ht cho 4. V vy l 1 hoc 2 1 . Nu 2 1 th l 1 v do 2
n

6
2
tri chia ht cho 2 . Vy l 1, 2h 1 do 2

2k 2 22h2 1 m 2n 2 6 suy ra c ng

h
h
h
mt trong hai kh nng k 2 v 2 2 xy ra. Nu k 2 th 2 k l 1 v l v 2 1 , nu

2h 2 th k 3 v 22

2k 3 1 , m n 3 suy ra k 3 . Do n 2 , v l.

CHNG 12. MT S BI TON CA L THUYT CNG TNH CA CC S | 283


2
Vy cc s 2 1 c tnh cht cn thit.
n

H qu (Crocker [1]). Khng c s no trong cc s 22 5 vi n =3, 4, 5, c dng 2k p vi k


0, 1, 2, v p l s nguyn t.
n

2
k
Chng minh. Nu 2 5 2 p vi k l s nguyn khng m v p l s nguyn t th
n

22 1 2k 22 p vi n 3 suy ra k phi bng 2 h qu l 22 1 23 p v do


n

p 22 9 22 1 3 22 n1 3 suy ra 22 1 3 1 mu thun vi gi thit n 3 .


n

Vi nhng lp lun tng t trong chng minh nh l 4, R.Crocker [2] chng minh c s
tn ti v hn cc s t nhin l khng th biu din c thnh tng cc ly tha ca 2 (khng
cn phn bit) v mt s nguyn t.

284 | Cc s l khng c dng

2k p vi p nguyn t

CHNG 13
S NGUYN PHC
1. Chun ca s nguyn phc. Cc s lin kt
S nguyn phc (hoc s nguyn Gauss) l cc s phc a bi vi a, b l cc s nguyn. L thuyt
v cc s nguyn phc l quan trng v my l do. Th nht l v s nguyn phc l mt dng tng
qut ca cc s nguyn thng thng, do s rt c ch khi tm hiu xem cc tnh cht no ca
cc s nguyn l khng m rng c cho lp rng hn. Th hai l t vic tm hiu tnh cht ca
cc s nguyn phc ta c th trc tip suy ra mt s tnh cht ca cc s nguyn thng thng. Cc
tnh cht nh vy thng gp rt nhiu kh khn c th chng minh theo cch khc.
Cc php cng, tr, nhn ca cc s nguyn phc c s dng ging ht nh cc php ton s
hc tng ng ca cc s phc.
Bi tp. 1. Tm tt c cc biu din ca 0 thnh tng bnh phng hai s nguyn phc.
Li gii. Ta c 0 a bi b ai vi a, b l cc s nguyn ty .
2

2. Tm s nguyn phc x yi l tng ca hai bnh phng cc s nguyn phc.


Li gii. S nguyn phc x yi l tng hai bnh phng cc s nguyn phc khi v ch khi y chn
v nu x c dng 4t 2 th y chia ht cho 4. iu kin cn c suy ra v nu

x yi (a bi)2 (c di)2 th x a2 b2 c2 d 2 , y 2(ab cd ) . V vy nu x c dng 4t 2 th


t nht mt trong cc s a v b v t nht mt trong cc s c v d l chn. Nhng khi ab cd
chn suy ra y chia ht cho 4.
iu kin c suy ra v nu x 2t 1 v y 2u th x yi t 1 ui u ti . Nu
2

x 4t 2 v y 4u th x yi t u 1 u t i t u 1 t u i . Nu x 4t v y 4u
2

th x yi t 1 ui u 1 t i , cui cng nu x 4t v y 4u 2 th
2

x yi t u 1 u 1 t i t u t u i .
2

3. Chng minh rng s nguyn phc x yi l tng ca ba bnh phng cc s nguyn phc khi v
ch khi y chn.
2
Gi . S dng bi tp 2 v ng thc 4t 2 2ui 4t 1 2ui 1 .

4. Chng minh rng s nguyn phc a bi 0 l bnh phng mt s nguyn phc khi v ch khi
a 2 b2 c 2 , c a 2 x2 v c a 2 y 2 vi c l s t nhin v x, y l cc s nguyn. Chng minh
rng khi a bi x yi , vi cc du dng nu b 0 v m nu b 0 .
2

Ghi ch. nh l trong bi tp 4 c th xc nh nh l mt php th quyt nh xem mt s


nguyn phc cho trc c l bnh phng mt s nguyn phc no hay khng. ng thi nh
l cng cho bit mt phng php tnh cn bc hai ca mt s nguyn phc (nu tn ti).
Vi s nguyn phc z a bi k hiu z ' l s nguyn phc lin hp ca n, ngha l z ' a bi .
T nh ngha cc php ton s hc cho cc s nguyn phc ta c
(1)
(2)
(3)

nu z t u th z ' t ' u ' ,


nu z t u th z ' t ' u ' ,
nu z tu th z ' t ' u ' .

R rng cc s z v z ' cng l s nguyn phc hoc cng khng phi. S ( z ') ' lin hp vi z '
chnh l z .

286 | Chun ca s nguyn phc. Cc s lin kt

Tch zz ca hai s lin hp gi l chun ca z v k hiu l N z . Ta c N x zz ' do nu


2
2
z a bi (vi a, b l cc s thc) th N z a b suy ra chun ca s nguyn phc lun l s
thc khng m v bng 0 ch khi a b 0 , ngha l z 0 . Hn na chun ca mt s nguyn
phc khc 0 l s t nhin. Cc s lin hp c cng chun.

Ta ni s nguyn phc z chia ht cho s phc t nu tn ti s nguyn phc u tha mn


(4)

z tu.

Khi ta vit t | z .
tnh xem khi no th s nguyn phc a bi chia ht cho s phc c di khc 0 th ta bin i

a bi a bi c di ac bd bc ad

i
c di
c2 d 2
c d 2 c2 d 2
2
2
2
2
Suy ra c di a bi khi v ch khi c d ac bd v c d bc ad .

Chng hn 3 5i 21 i v 34 68 v 34 102, 1 i 2 v 2 2 v 2 2 . Mt khc 1 2i 1 2i v 5 3


T (3) suy ra nu t z th t ' z ' v nu z tu th zz ' tut ' u ' tt ' uu ', suy ra theo nh ngha ca
chun cc s nguyn phc ta c
(5)

N z N t N u .

Vy chun ca tch hai s nguyn phc l tch cc chun ca cc s .


nh l ny ng vi tch hu hn cc s nguyn phc.
Theo (5) ta c N t N z v do nu s nguyn phc t l c s ca z , th chun ca t l c
s ca chun ca z . iu ngc li khng ng, chng hn N 1 2i N 1 2i but 1 2i 1 2i .
Hai s nguyn phc khc 0 m l c s ca nhau th c gi l cc s lin kt.
Ngha l z v t l cc s lin kt khi v ch khi t|z v z|t. Khi ta c N(t)|N(z) v N(z)|N(t) m v
chun ca cc s nguyn phc khc 0 l khc 0 nn N(t) = N(z).
Vy hai s nguyn phc lin kt th lun c cng chun. iu ngc li khng ng, chng hn cc
s 1 2i v 1 2i c cng chun nhng khng phi cc s lin kt v 1 2i |1 2i .
By gi ta tm tt c cc s lin kt vi s nguyn phc z0 cho trc.
Gi s t l s lin kt vi z th vi s nguyn phc u no ta c t = zu v suy ra
(6)

N(t) = N(z) N(u)

Nhng do cc s lin kt th c cng chun nn N ( z ) N (t ) v N ( z ) 0 v z 0 do t (6) suy


ra N (u) 1 . t u a bi suy ra a2 + b2 = 1. Do a = 1 v b = 0 hoc a = 0 v b = 1. T y suy
ra u l mt trong bn s 1, -1, i, -i v do t = zu l mt trong bn s
(7)

z, -z, iz, -iz.

Vy mi s lin kt ca z l mt trong cc s trong (7). Ngc li, d thy cc s trong (7) l lin
kt vi z. V z = (-1)(-z) = (-i)iz = i(-iz). Ta c nh l
nh l 1. Mi s nguyn phc z khc 0 c ng bn s lin kt l cc s trong (7).
R rng bn s lin kt ny l phn bit (v z 0).
Trong cc bi ton lin quan ti tnh chia ht ca cc s nguyn phc th cc s lin kt c th thay
th cho nhau v nu z chia ht cho t th mi s lin kt ca z chia ht cho mi s lin kt ca t.
R rng nu z lin kt vi t th z lin kt vi t.

CHNG 13. S NGUYN PHC | 287

Nu hai s nguyn phc z1 v z2 u chia ht cho t th tng v hiu ca chng cng chia ht cho t
bi v nu z1 tu v z2 tv th z1 z2 t (u v) .
Nu s nguyn phc z chia ht cho t v t chia ht cho u th z chia ht cho u. Tht vy, nu z tw v
t uv th z uvw .
Do nu t l c s chung ca cc s nguyn phc z1 , z2 ,..., zn v nu u1 , u2 ,..., un l cc s
nguyn phc bt k th t | z1u1 z2u2 ... znun .
2. Thut ton Euclid v c s chung ln nht ca cc s nguyn phc
By gi ta chng minh
nh l 1. Nu z v t 0 l cc s nguyn phc th tn ti cc s nguyn phc c v r tha mn
(8)

z = ct + r

v
(9)
T suy ra N(r) N(t).
Chng minh. t
(10)

z|t = x + yi,

vi x, y l cc s hu t. K hiu v l cc s nguyn gn x v y nht. Khi


(11)

x x1 ,

y y1 ,

vi x1 v y1 l cc s hu t tha mn
(12)

x1

1
,
2

y1

1
,
2

t
(13)

c i,

r z ct

R rng c, r l cc s nguyn phc v tha mn (8). Theo (10), (11), (13) ta c

r z ct ( x yi)t ( i)t ( x1 y1i)t


V chun ca mt tch l bng tch cc chun cc nhn t nn theo (12) th

N (r ) N ( x1 y1i) N (t ) ( x12 y12 ) N (t ),

x12 y12

1 1 1

4 4 2

suy ra (9) v ng thi nh l c chng minh.


nh l va chng minh cho ta mt thut ton tng t vi thut ton Euclid i vi cc s
nguyn. Theo tm c s chung ln nht ca hai s nguyn phc cho trc z v t 0 th u
tin theo nh l 2 ta tm cc s c, r. Theo (8) ta suy ra cc s z v t c cng c s chung ging
nh cc s t v r. Hn na t (5) suy ra N(r) < N(t). V vy ta ch cn tm c s chung ca cc s t
v r vi N(r) < N(t).
Nu r = 0 th c s chung ca cc s z v t chnh l cc c s ca t.
Nu r 0 th p dng th tc trn vi cc s t, r thay th cho z, t. V vy tm cc c s chung
ca cc s t, r ta tm cc c s chung ca cc s r, r1 vi N r1 N r .
Nu r1 0 ta tm s r2 v c nh vy.

288 | Thut ton Euclid v c s chung ln nht ca cc s nguyn phc

Dy r, r1, r2, khng th ko di v hn v chun cc phn t ca dy l dy cc s t nhin gim


nghim ngt. Do vi n no ta c rn = 0. Khi cc c s chung ca rn-1 v rn-2 chnh l cc
c s ca rn. Vy ta kt lun rng tn ti s nguyn phc Q m cc c s ca n chnh l cc
c s chung ca cc s z v t.
Chng t hai s nguyn phc khc 0 cho trc c t nht mt c s chung chia ht cho mi c
s chung ca chng. Ta c th gi c s ny l c s chung ln nht ca hai s nguyn phc.
By gi ta tnh s cc c chung ln nht ca hai s nguyn phc. K hiu d v l cc c s
chung ln nht ca cc s nguyn phc z, t. Cc s d, l cc s lin kt. V vy theo nh l 2 ta c
H qu. Hai s nguyn phc khc 0 lun c ng 4 c s chung ln nht l cc s lin kt.
Lu rng cc s nguyn cng c hai c s chung ln nht tri du. Ta khng phn bit cc c
s sai khc du. Tng t trong trng hp s nguyn phc ta c th ch xc nh mt c s
chung ln nht sai khc mt lin kt. Ty trng hp ta s chn cch gi thun tin nht.
V d. 1. Ta tm c s chung ln nht ca 6 - 17i v 18 + I bng thut ton nu trn. p dng cc
bc trong thut ton cho ta cc ng thc

6 17i 6 17i 18 i 91 312i


91 13i

i
2
18 i
18 i
325
325
6 17i = -i(18 + i) + (5 + i),

18 i 18 i 5 i 91 13i
1 i

3
,
2
5i
5 1
26
2
18 + i = 3 (5 + i) + 3 2i,

5 i 3 2i 1 i
5i

3 2i
32 22

Vy c s chung ln nht ca 6 17i v 18 + I l 3 2i v cc s lin kt -3 + 2i, 2 + 3i, -2 3i.


2. Tnh c s chung ln nht ca 2 + 3i v 2 3i. Ta c

2 3i 2 3i
5 12i
5 i
2 2
1
,
2 3i
2 3
13
13
2

2 + 3i = i(2 3i) + i 1,

2 3i 2 3i i 1 5 i
1 i

3
,
i 1
2
2
2

2 3i = -3(i - 1) 1
Do c s chung ln nht ca 2 + 3i v 2 3i l 1 v cc s lin kt - 1, i v i.
3. Tnh c s chung ln nht ca cc s 31 + i v 5 + I. Ta c

31 i 31 i 5 i 156 21i

6i
5i
52 12
26
Do c s chung ln nht ca 31 + i v 5 + i l 5 + i v cc s lin kt -5 i, -1 + 5i v 1 5i.
D thy cc c s chung ln nht c chun ln nht trong cc c s. iu ngc li cng ng.
Do c s chung ln nht c th nh ngha l cc c s chung c chun l ln nht c th. Tuy
nhin khng d chng minh cc c s chung ln nht nh th chia ht cho mi c s khc.

CHNG 13. S NGUYN PHC | 289

L thuyt v cc c s chung ln nht ca hai hoc nhiu hn cc s nguyn phc c th trnh


by da vo cc dng tuyn tnh nh trong trng hp s nguyn. Tht vy, gi s a1, a2 ,..., am l
cc s nguyn phc khc 0 . K hiu Z l tp hp cc s khc 0 c dng a1z1 a2 z2 ... am zm , vi

z1, z2 ,..., zm l cc s nguyn phc. Cui cng k hiu M l tp hp cc gi tr ca chun ca cc s


trong Z . R rng M ch cha cc s t nhin. Gi n l s t nhin nh nht trong M . Khi tn
ti s trong Z m N n, ngha l tn ti cc s nguyn phc 1, 2 ,..., m m
(14)

a1 1 a2 2 ... am m .

By gi ta chng minh cc s trong Z chia ht cho . Tht vy, gi s z l s bt k trong Z . Khi


tn ti cc s nguyn phc z1, z2 ,..., zm tha mn
(15)

z a1z1 a2 z2 ... am zm .

Hn na theo nh l 2 suy ra tn ti cc s nguyn phc c v r tha mn


(16)

z c r v N r N .

Nu r 0 th r thuc Z v t (14),(15) v (12) th


r z c a1 z1 c 1 a2 z2 c 2 ... am zm c m ,

hn na cc s z j c j , j 1, 2,..., m, l cc s nguyn phc. Nhng theo (16) suy ra r l s c


chun nh hn chun n ca , iu ny mu thun vi nh ngha ca n . Do r 0 v suy ra
theo (16) th z c v do | z. R rng cc s a1, a2 ,..., am u thuc Z . V vy s nguyn phc

l c s chung ca cc s a1, a2 ,..., am .

Gi s l c s chung ty ca cc s ny. Khi tn ti cc s nguyn phc t1, t2 ,..., tm tha


mn a j t j vi mi j 1,2,..., m. V vy theo (14) th t1 1 t2 2 ... tm m , chng t | .

T y ta suy ra l c s chung ca cc s a1, a2 ,..., am nhn mi c s chung khc l c s


ca n. Ta cng nhn thy c dng (14) vi 1, 2 ,..., m l cc s nguyn phc.
Hai s nguyn phc bt k a, b lun c t nht bn c s chung l I, 1, i, i.
Nu cc s nguyn phc a, b khng c nhiu hn bn c s chung th chng gi l cc s nguyn
t cng nhau. Ta vit a, b 1 .
D dng thy rng tn ti cc s nguyn phc x, y tha mn
(17)

ax by 1.

Tht vy, nu a, b 1 th xc nh bi (14) vi a a1, b a2 , m 2 s l mt trong cc s 1,

1, i, i . Do mt trong cc s , , i , i bng 1 suy ra vi la chn ph hp th (17) ng.


Mt khc t (17) suy ra mi c s chung ca cc s a, b u l c s ca 1 v do a, b khng
c c s chung no khc 1, 1, i, i , ngha l a, b 1 .
nh l 3. Hai s nguyn phc a, b nguyn t cng nhau khi v ch khi tn ti cc s nguyn phc
x, y m ax by 1.
By gi ta xt ba s nguyn phc a, b, c vi a, b 1 v b | ac . Ta chng minh b | c .
Tht vy, v a, b 1 nn theo nh l 3 th tn ti cc s nguyn phc x, y tha mn (17).

290 | Bi s chung nh nht ca cc s nguyn phc

Nhn c hai v vi vi c, ta c
(18)

acx bcy c .

Theo gi thit b | ac v r rng b | bc vi mi b suy ra (18) ko theo b | c . iu phi chng minh.


nh l 4. Vi mi s nguyn phc a, b, c m (a, b) 1 v b | ac th b | c .
H qu khc ca nh l 3 l
nh l 5. Nu (a, b) 1 v (a, c) 1 th (a, bc) 1 .
Chng minh. Nu (a, b) 1 v

a, c 1

th tn ti cc s nguyn phc x, y, u, v tha mn

ax by 1 v au cv 1. Nhn hai v ta c a x au cv buy bcyv 1 suy ra a, bc 1.


3. Bi s chung nh nht ca cc s nguyn phc
K hiu a1, a2 ,..., am l cc s nguyn phc khc 0 . Khi tn ti bi s chung ca cc s , chng
hn ta c th ly mt bi s chung chnh l tch ca tt c cc s . Trong s cc bi s chung ny
ta chn s c chun nh nht, ngha l chun ca n khng ln hn chun ca bt k bi s chung
no khc ca cc s . K hiu bi s chung ny l v .
Ta s chng minh rng mi bi s chung ca a1, a2 ,..., am u chia ht cho v .
Tht vy, gi s z l bi s chung ty . Theo nh l 2 th tn ti cc s nguyn phc c, r tha
mn z cv r v N r N v . Nu r khc 0 th r khi s l bi s chung ca a1, a2 ,..., am , v
c chun nh hn v , iu ny mu thun vi nh ngha ca v . Ngc li nu r 0 th tn ti t
nht mt bi s chung ca cc s ban u chia ht cho n.
Chun ca mi bi s chung c tnh cht ny th khng ln hn chun ca mi bi s chung khc
0 ca cc cc s a1, a2 ,..., am . Ta gi n l bi s chung nh nht ca cc s a1, a2 ,..., am .
D thy tt c cc bi s chung nh nht ca a1, a2 ,..., am l cc s lin kt v chun ca chng l
nh nht trong cc chun ca cc bi s chung khc 0 ca cc s .
Bi tp. Tnh nghim nguyn phc ca phng trnh x y z xyz 1 .
Li gii. V xyz 1 nn cc s x, y, z l c s ca n v, ngha l cc s 1, 1, i, i . Li v xyz 1
nn suy ra cc s ny khng cng thun o, mt khc v x y z 1 nn nu ba s u l s
thc th c hai s bng 1 v s cn li bng 1 , nhng khi th mu thun vi xyz 1 . V vy t
nht mt trong cc s x, y, z l thun o, nhng khi v x y z 1 nn t nht hai trong ba s
l thun o. V vy ta c th kt lun mt trong cc s x, y, z phi bng i, cc s khc l i v 1. Ta
thy nghim duy nht l x 1, y i, z i v cc hon v. C tt c 6 nghim nh vy.
Ghi ch. J.W.S.Cassels [4] chng minh h phng trnh x y z xyz 1 khng c nghim
hu t (mt chng minh n gin khc c cho bi Sansone v Cassels [1]).
4. Cc s nguyn t phc
V mi s nguyn phc u c t nht bn c s l 1, 1, i, i v hn na mi s nguyn phc z
khng lin kt vi 1 u c bn c s l z, z, iz, iz nn suy ra cc s nguyn phc ny c t nht
tm c s.
Cc s nguyn phc c ng tm c s gi l cc s nguyn t phc.
Ni cch khc, s nguyn phc l nguyn t nu n khng c c s no ngoi tr cc s lin kt
ca n v cc s lin kt ca 1 v hn na n khng lin kt vi 1.

CHNG 13. S NGUYN PHC | 291

R rng nh ngha ny tng ng vi: s nguyn phc l nguyn t nu chun ca n ln hn 1


v n khng biu din c thnh tch ca cc s nguyn phc vi chun ln hn 1.
Tht vy, nu l s nguyn phc, N 1 v v, vi N 1 v N v 1 th s
khng th lin kt vi 1 v nu nh vy th N 1 , v n cng khng lin kt vi v nu th
th N N , suy ra v N N N v ta s c N v 1 , mu thun vi gi thit. Do

c c s khng lin kt vi 1 v , do n khng phi s nguyn t.


Mt khc, nu l s nguyn phc vi N 1 v n khng phi s nguyn t th theo nh
ngha n c c s khng lin kt vi 1 v . Khi ta c v vi v l s nguyn phc.
Nu N 1 th lin kt vi 1, mu thun vi gi thit (tht vy, nu vi s nguyn phc
2
2
a bi ta c N a bi 1 th a b 1 suy ra v a, b nguyn nn a 1 v b 0 hoc a 0 v
b 1 ).

Nu N v 1 th v lin kt vi 1, suy ra v v, s lin kt vi , mu thun.


H qu l N v 1 v N v 1 v do l tch ca hai s nguyn phc vi chun ln hn 1.
R rng mi s nguyn phc lin kt hoc lin hp vi s nguyn t phc cng l s nguyn t phc.
nh l 6. Mi s nguyn phc vi chun ln hn 1 u c th biu din c thnh tch ca hu
hn s nguyn t phc.
Chng minh. Gi s phn chng rng tn ti s nguyn phc vi chun n ln hn 1 m khng
biu din c thnh tch ca hu hn s nguyn t phc. K hiu M l tp hp cc gi tr ca
chun ca cc s nguyn phc vi tnh cht ny. Khi M l tp hp khng rng cc s t nhin.
K hiu m l s nh nht trong M . Khi tn ti s nguyn phc z vi chun m m khng biu
din thnh tch ca hu hn s nguyn t phc. Theo gi thit z khng phi s nguyn t v chun
ca n l m 1 . Do n l tch ca hai s nguyn phc v v vi chun ln hn 1.
Hn na m N z N v N N v , suy ra N m v N v m . T nh ngha ca m

suy ra cc s ,v biu din c thnh tch ca hu hn s nguyn t phc. Nhng t y li suy


ra s z v cng c tnh cht , mu thun vi nh ngha ca z .
nh l c chng minh.
Theo nh ngha th mi s nguyn t phc u c ng tm c s 1, 1, i, i, , , i , i . Do
nu mt s nguyn phc khng chia ht cho s nguyn t phc th ( , ) 1 .
Nu mt s t nhin no l s nguyn t phc th r rng n cng l s nguyn t theo ngha
thng thng. iu ngc li khng ng v c nhng s nguyn t khng phi l s nguyn t
phc. Chng hn 2 1 i 1 i v N 1 i N 1 i 2 1.
Cc s 1 i v 1 i l nguyn t phc v nu 1 i v th N N v N v N 1 i 2; do
(lu chun ca s nguyn phc l s t nhin) ta c N 1 hoc N v 1 , chng t
hoc v l lin kt vi 1.
Cc s 1 i v 1 i l lin kt v 1 i i 1 i . V vy 2 lin kt vi cn ca mt s nguyn phc.
S dng nh l 4 suy ra biu din ca s nguyn phc thnh tch ca cc s nguyn t phc l duy
nht sai khc hon v v cc lin kt.
Ta s m t cc s nguyn t phc trong tp hp tt c cc s nguyn t phc.

292 | Cc s nguyn t phc

Xt cc s t nhin (cng l s nguyn phc) l s nguyn t phc. R rng cc s ny u l s


nguyn t theo ngha thng thng v hn na l s l v 2 khng phi s nguyn t phc. V
vy ta xt cc s nguyn t c dng 4k 1 v 4k 3 vi k l s t nhin.
Gi s p l s nguyn t c dng 4k 1 .
Theo nh l 9 Chng 5 th tn ti cc s t nhin a, b tha mn p a 2 b2 v suy ra

p a bi a bi v hn na N a bi a 2 b2 p 1 . Vy p khng phi s nguyn t phc.


Cc nhn t a bi v a bi l cc s nguyn t phc. Tht vy, nu a bi v vi
(19)

N 1 and N v 1 ,

th p N a bi N N v , v l v p l s nguyn t.
T y suy ra cc c s phc ca s nguyn t c dng 4k 1 vi k l s t nhin l cc s
nguyn t phc. R rng cc c s ny khng lin kt.
Tht vy, ng thc a bi a bi l khng th xy ra v n suy ra b 0 v p a 2 . ng thc

a bi a bi cng khng th v n suy ra a 0, p b2 . Nu a bi i a bi th a b v do


p 2a 2 , v l. Cui cng nu a bi i a bi th a b v do p 2a 2 , v l.
Vi cc s nguyn t c dng 4k 3 vi k l s nguyn khng m ta s chng minh chng u l
cc s nguyn t phc.
Tht vy, nu s nguyn t p 4k 3 l tch ca hai s nguyn phc vi chun ln hn 1 th

p a bi c di , suy ra p 2 a 2 b2 c 2 d 2 , vi a 2 b2 1 v c2 d 2 1 . V p l s

nguyn t nn suy ra p a 2 b2 , nhng iu ny khng th c v p c dng 4k 3 .


Vy trong cc s nguyn t th ch c cc s c dng 4k 3 l s nguyn t phc. Cc s nguyn t
phc khc l 1 i v cc c s lin hp ca cc c s phc ca cc s nguyn t c dng 4k 1 .
trn ta chng minh mi s t nhin 1 u l tch ca s nguyn t phc m ta va lit k
hoc cc s lin kt ca cc s .
R rng khng cn s nguyn t phc no khc v nu l mt s nh vy th v biu din mt s
phc thnh thnh tch ca cc s nguyn t phc l duy nht nn khng phi c s nguyn t
phc ca mi s t nhin. Nhng ' N nn ta c mu thun.
Vy ta chng minh c
nh l 7. Tt c cc s nguyn t phc l cc s thuc v ba lp sau y v cc lin kt ca chng
1. 1 i
2. c s nguyn t phc ca cc s nguyn t c dng 4k 1
3. cc s nguyn t c dng 4k 3
Di y l cc s nguyn t phc (khng tnh cc lin kt) vi chun nh hn 100

Hai s nguyn t phc vi hiu bng 2 c gi l cp s nguyn t sinh i. Chng hn


4 i,6 i,3i,2 3i,3 2i,5 2i,7 i,2 7 i. C cc cp s nguyn t sinh i lp thnh cp s cng ba
phn t. Chng hn 2 i,4 i,6 i v 1 2i,3 2i,5 2i .
T gi thuyt H (Chng 3 mc 8) suy ra tn ti v hn cp s nguyn t phc sinh i.

CHNG 13. S NGUYN PHC | 293

Tht vy, t f1 x x 2 2 x 2, f 2 x x 2 2 x 2 . Cc a thc f1 x v f 2 x khng c nghim


hu t v do bt kh quy. Ta cng c f1 0 f 2 0 4, f1 1 f 2 1 5, chng t iu kin C c
tha mn. Do theo gi thuyt H th tn ti v hn s t nhin x tha mn f1 x v f 2 x u l
s nguyn t. Nhng f1 x x 1 1, f 2 x x 1 1 v x l v nu ngc li th 2 | f 2 x v
2

f 2 x 2, suy ra f 2 x l hp s. H qu l cc s f1 x v f 2 x u c dng 4k 1 v do
cc s x 1 i v x 1 i u l s nguyn t phc v hiu ca chng bng 2. V vy ta nhn c
dy v hn cc cp s nguyn t phc sinh i phn bit. Cc cp nhn c vi
x 3,5,15,25,55,... Tuy nhin c nhng cp s nguyn t phc sinh i khng nhn c theo
cch ny, chng hn 1 2i,3 2i hoc 3 8i v 5 8i .
Cc cp s nguyn t phc sinh i c xc nh bi D.Shanks [1].
5. Phn tch ca s nguyn phc thnh cc c s nguyn t phc
Ta trnh by phng php biu din mt s nguyn phc z thnh tch ca cc s nguyn t phc.
t N z n . Mi c s nguyn t ca z u l c s nguyn t ca chun n zz ' . c s
nguyn t phc ca s t nhin n c th nhn c bng cch tm cc c s nguyn t ca s .
Tht vy, t
(20)

n 2 p11 p22 ... pkk q11 q22 ...qll ,

vi pi l cc s nguyn t c dng 4t 1 v q j l cc s nguyn t c dng 4t 3 . Gi s j v

' j , j 1, 2,..., k , l cc c s nguyn t phc lin hp ca p j . t j a bi v ' j a bi th

p j a 2 b2 . Khi phn tch ca n thnh tha s nguyn t phc l


(21)

z i 1 i 11 '1 '1 22 '2 '2 ... kk 'k 'k q11 q22 ...qll ,

Vi n zz ' ta c
(22)

z i 1 i 11 '1 '1 22 '2 '2 ... kk 'k 'k q11 q22 ...qll ,

vi v l mt trong cc s 1,2,3,4, cc ly tha , 1, '1,..., k , 'k , 1,..., 1 l nguyn khng m.


Ly chun ca (22) v lu N j p j , N q j q 2j ta c

N z 2 p11 '1 p22 '2 ...qkk 'k q12 1 q22 2 ...ql2 l ,


suy ra theo (21) v lu N ( z ) n , so snh cc s m ca cc s nguyn t ta c

, 1 '1 , 2 '2 2 , ... , k 'k k ,


(23)

21 1, 22 2 , ... ,

2l l ,

Cc ng thc (23) chng t tt c cc s m u chn.


V vy nu s t nhin n l chun ca mt s nguyn phc th trong phn tch n thnh tha s
nguyn t cc s nguyn t c dng 4k 3 c s m chn.
Hn na t (23) suy ra , 1

1
1
1
1 , 2 2 , ... , 1 1.
2
2
2

V vy , 1, 2 ,..., l c xc nh duy nht.


tnh j v ' j , vi j 1,2,..., k , ta s dng mt quy tc khc c xc nh nh sau.

294 | Phn tch ca s nguyn phc thnh cc c s nguyn t phc


k

Vi k j l ly tha ln nht m pJ j | z , ngha l k j l ly tha ln nht m pJ j l c s ca c a


v b vi z a bi . Khi

j j k j
'j j k j
k
k
nu p j j | z,
nu p j j | z .
'
j k j
j k j

(24)

Tht vy, t nh ngha ca s m k j suy ra s nguyn phc z / p j j khng chia ht cho j v p ' j
k

v nu nh vy th v j , ' j 1 suy ra n chia ht cho j ' j p j , suy ra p j j | z, mu thun vi


k 1

nh ngha ca k j . H qu l nu j | z / p j j , th s z / p j j khng chia ht cho ' j . V vy t


k

p j j j j j j , theo (22) suy ra ' j k j , nn theo (23) th j j k j . Nu s z / p j j khng chia


ht cho j , th j k j v ' j j k j . Theo (24) suy ra quy tc c chng minh.
k

'k

Cui cng s m v tm c t php chia z cho tch tt c cc c s nguyn t c s m bit.


V d. 1. Vi z 22 7i. Ta c N z 484 49 533 13 41, p1 13 22 32 , p2 41 42 52.
'

' '
H qu l z i 1 11 j 2 2 2 2 , vi 1 2 3i, 1' 2 3i, 2 4 5i, 2' 4 5i. R rng k1 k2 0 .

z / 1 22 7i / 2 3i 22 7i 2 3i /13 5 4i

nguyn

phc

do

1 1 0 1, 1 0. Tng t, thng s z / 2 cng tnh c nhng ch cn lu rng 5 4i l

s nguyn t phc. V vy 22 7i 2 3i 5 4i l biu din cn tnh.


2
2
2. Vi z 19 17i. Ta c N z 361 289 650 2 5 13 2 p1 p2.

H qu l z i 1 i 111 j 22 2' '2 , vi 1 1 2i, 1' 1 2i, 2 2 3i, 2' 2 3i,1 2, 2 1.


'

V ta u khng c 5 | z v 13| z, nn k1 k2 0 . Hn na s 19 17i / 1 2i khng phi s


nguyn phc v do 1 0 v 1' 2. S 19 17i / 2 3i khng phi s nguyn phc. Do

2 0 v 2' 1 . Vy ta c z i 1 i 1 2i 2 3i , vi php chia n gin suy ra 2 . Vy


2

phn tch cn tm l 19 17i 1 i 1 2i 2 3i .


2

3. Vi z 10 100i. Ta c z 10 1 10i 101 v v 10 100i i 1 i 1 2i 1 2i 1 10i


2

nn ch cn tnh biu din ca 1 10i. Ta c N 1 10i 101. y l s nguyn t c dng 4k 1 .


V vy theo nh l 7, 1 10i l s nguyn t phc. Do 10 100i i 1 i 1 2i 1 2i ,
2

Bi tp. Tm phn tch thnh s nguyn t phc ca cc s nguyn phc 1 7i,9 i,7 9i,
107 198i,10 i,7 24i.
Li gii. Ta c 1 7i 1 i 1 2i ,9 i i 1 i 4 5i ,7 9i 1 i 1 2i 3 2i ,
2

107 198i 1 6i ,10 i 10 i,7 24i 1 2i .


3

6. S cc s nguyn phc vi chun cho trc


Ta s tnh xem c bao nhiu s nguyn phc c chun bng mt s t nhin cho trc n . Vn
ny khng ch t n quan trng m cn v n tng ng vi bi ton tm tt c cc cp s
nguyn x, y m x 2 y 2 n . Ni cch khc s n cc s nguyn phc vi chun bng n bng vi
s cch biu din s n thnh tng hai bnh phng cc s nguyn.

CHNG 13. S NGUYN PHC | 295

Do hm n y cng c nghin cu trong Chng 11.


Gi (20) l phn tch thnh tha s nguyn t ca n v (21) l biu din ca s thnh tch cc
s nguyn t phc. Ta chng minh trong mc 5 th N ( z ) n ch khi cc ly tha j , j 1,2,..., l
u chn. Gi s iu kin ny c tha mn, khi s z vi chun n c phn tch thnh tha s
nguyn t phc dng (22), cc ng thc (23) i vi cc s m ca cc ly tha cng c tha
mn v v l mt trong cc s 1,2,3,4. Ngc li nu , 1, 1' , 2 , 2' ..., k , k' , 1, 2 ,..., l l cc b
s nguyn khng m ty tha mn cc ng thc (23) v v l mt trong cc s 1,2,3,4 th s z
xc nh duy nht bi (22) s c chun n . M cc s , 1, 2 ,..., i c xc nh duy nht bi cc
iu kin (23) nn vn tnh s cc s nguyn phc phn bit c chun bng s n tng ng
vi vic tnh s cc b s nguyn khng m phn bit v, 1, 1' , 2 , 2' ..., k , k' tha mn

1 v 4, 1 1' 1, 2 2' 2 , ... , k k' k .


C bn gi tr v c th nhn l 1,2,3,4. Vi 1 , 1' ta c 1 1 kh nng c th xy ra l

0,1;1,1 1;2,1 2;...;1,0 . Tng t ta c 2 1 gi tr ng vi 2 , 2' v c nh th. Chng t

n 4 1 1 2 1... k 1.

(25)

Cng thc ny nhn c vi gi thit l s m ca cc s nguyn t dng 4t 3 trong phn tch


thnh tha s nguyn t ca n u chn. Trong trng hp ngc li th phng trnh N z n
khng c nghim nguyn phc z , do n 0 . Vy ta c nh l
nh l 8. Nu s t nhin n c phn tch thnh tha s nguyn t dng (20) th s n cc cch
biu din n thnh tng hai bnh phng cc s nguyn l bng 4 1 1 2 1 ... k 1 vi gi
thit s m ca cc s nguyn t dng 4t 3 trong phn tch u chn. Trong trng hp ngc li
n 0 .
nh l nhn c t Chng 11 mc 1 (theo mt cch khc) l h qu ca nh l 8.
c bit nu n l s nguyn t c dng 4t 1 th n 8 suy ra nh l 9 Chng 5.
By gi xt hm f h nh ngha nh sau

f (h) 1
1

(26)

khi 2 | k
khi k 4t 1
khi k 4t 3

Vi mi s nguyn a, b th f ab f a f b . Nn nu n h11 h22 ...hkk l phn tch thnh tha s


nguyn t ca n th

f d f 1 f h f h ... f h ... f 1 f h ... f h .


2
1

d |n

Theo (26) ta c f 1 f 2 f 22 ... f 2 1.


Nu h 4t 1, th f 1 f h f h2 ... f h 1.
Nu h 4t 3, th
(27)

1 1
f 1 f h f h ... f h 1 1 1 ... 1
.
2

Theo cng thc ca

f d ta c f d 1
d |n

d |n

hi 1 mod 4

suy ra theo nh l 8 th

296 | S cc s nguyn phc vi chun cho trc

n 4 f d ,

(28)

d |n

nu mi s nguyn t c dng 4t 3 trong phn tch ca n u c s m chn trong phn tch


thnh tha s nguyn t ca n . Trong trng hp ngc li th t (27) ta c f d 0 v theo
d |n

nh l 8 chng t (28) ng. Do n ng vi mi n . Vy ta chng minh nh l Jacobi sau


nh l 9. S cch biu din mt s t nhin n thnh tng bnh phng hai s t nhin l bng bn
ln hiu ca s c s c dng 4t 1 ca n v s cc c s c dng 4t 3 ca n .
Tht vy, trong (28) th cc hng t 1 xut hin vi s ln ng bng s cc c s c dng 4t 1
ca n v cc hng t 1 xut hin vi s ln ng bng s c s c dng 4t 3 ca n .
Theo (28) ta c
x
1 x
x
n f k .

4 n1
k
k 1

(29)

do cc hng t f d xut hin trong tng

f d vi s ln ng bng s cc s n s

n 1 d |n

x
d | n, ngha l ln.
d
x


1 x
Theo cng thc (6) Chng 11 mc 2 ta c n x k 2 , suy ra

4 n 1
k 0
x

xk2

k 0

(30)

f k k ,
k 1

V do x x 12 x 22 ...

x x x x
1 3 5 7 ... .

Cc tng trong ng thc ny u cha hu hn hng t. ng thc ny gi l ng nht thc


Liouville. Vi x = 10 ta c

10 9 6 1 10 10 10 10 10 ,

1 3 5 7 9
Tc l

3 + 3 + 2 + 1 = 10 3 + 2 1 + 1.

ng thc Liouville cng c suy ra t nh l Jacobi theo mt cch khc.


Da vo cc ng thc trong Chng 11 mc 2 th ng nht thc Liouville suy ra khai trin
1 1 1 1 1
Leibniz ca l
... theo mt cch s cp. y l phng php thun ty
4 1 3 5 7 9
s hc m dn ti mt cng thc ca mt hng s hnh hc quan trng nht: t s ca chu vi v
ng knh ca hnh trn. Cng thc ny c dng rt n gin (chui tng nghch o cc s l lin
tip vi cc du cng tr lin tc). Mt cng thc tnh da vo cc s l lin tip khc c cho
2 1
1
1
1
1
bi Euler:
2 2 2 2 2 ... . Cng thc ny cng nhn c theo cch s cp,
8 1
3
5
7
9
Ta c mt s cng thc khc ca c chng minh bng gii tch l
Cng thc Wallis:

1
1
1
1

1 2 1 2 1 2 1 2 ...
4
3
5
7
9

CHNG 13. S NGUYN PHC | 297

Cng thc Euler:

3
32

Cng thc Brouncker:

1
1
1
1
1
3 3 3 3 ...
3
1
3
5
7
9

12

32
2

52

72
2

92

...

7. nh l Jacobi v tng bn bnh phng


By gi ta chng minh nh l Jacobi v vic biu din mt s thnh tng bn bnh phng.
u tin ta xt trng hp khi s t nhin n c dng n = 4u. Gi s
(31)

4u x2 y 2 z 2 t 2

l mt biu din ca 4u thnh tng ca bn bnh phng l.


R rng v x, y, z, t l nn
(32)

x 2 y 2 2u ' v z 2 t 2 2u ''

Vi u ' v u '' l cc s t nhin l. Theo (31) v (32) ta c


(33)

2u u ' u ''.

Mt khc nu w l s l v 2w a 2 b2 nn cc s a,b l v nu a,b u chn th 2w chia ht cho 4,


mu thun vi gi thit w l v nu mt trong cc s a,b l v s kia chn th s 2w l, v l.
V vy tm tt c cc biu din ca 4u thnh tng ca bn bnh phng l ta ch cn tm cc biu
din 2u thnh tng ca hai s l u ' v u '' v sau tm s cch biu din ca cc s u ', u '' thnh
tng hai bnh phng.
K hiu 0(4u) l s cch biu din 4u thnh tng ca bn bnh phng l.
Vi mi cp hai s l c nh u ' v u '' tha mn (33) th theo (28) v 2m m , m 1,2,...,
suy ra t (25) ta c s cch biu din 4u thnh tng bn bnh phng l l

2u ' 2u '' 16 f d ' f d '' .


d '|u '

d ''|u ''

V vy tng s cch biu din l


(34)

0 4u 16

f d '. f d ''

u ' u '' 2 u

d '|u '

d ''|u ''

Trong mi hng t th tng ly trn mi cp s t nhin u ', u '' tha mn (33). V mi c s ca s

f d ' 1

l u l s l nn theo (26) ta c

d '|u '

1
d ' 1
2

v tng t

d '|u '

f d '' 1
d ''|u ''

t s dng (34) suy ra


(35)

1
1

d ' 1
d '' 1
2
2

1
.

u ' u '' 2 u d '|u '


d ''|u ''

0 4u 16

S dng quy tc

a b a a
p

m 1

T (35) suy ra

n 1

... a p b1 b2 ... bq ambn


p

m 1 n 1

d ''|u ''

1
d '' 1
2

298 | nh l Jacobi v tng bn bnh phng

0 4u 16

(36)

1 2

d ' 1

1
d '' 1
2

u ' u '' 2u d '|u ' d ''|u ''

Da vo ng thc
1

l s l, ta c 1 2

1
1
1
d ' 1 d '' 1 d ' d '' d '' 1 v v d '' l c s ca s l nn n
2
2
2
1
2

d ' 1 d '' 1

1 2

d ' d ''

V vy (36) tr thnh
(37)

0 4u 16

1 2

d ' d ''

u ' u '' 2u d '|u ' d ''|u ''

Vi mi cp s t nhin l u ' v u '' tha mn (33) v vi mi cp cc c s d ' v d '' ta k hiu


cc i c s ca chng l ', ''. Ta c
(38)

u ' d ' ',

u '' d '' ''.

Theo (33) suy ra


(39)

2u d ' ' d '' '' ,

vi ' v '' l cc c s ca s l nn chng l cc s l. H qu l mi hng t ca tng (37)


ng vi mt b bn s l duy nht
(40)

d ',

d '',

',

''.

tha mn (39). Ngc li, do hai s u tin trong cc s d ', d '', ', '' c cho trc v hai s
cn li c xc nh bi (38) nn tng (37) c ly trn tt c cc s t nhin tha mn (39).
V vy ta c th vit
(41)

0 4u 16

d ' ' d '' '' 2 u

1 2

d ' d ''

Trong tng v phi ly trn mi b s (40) gm bn s l tha mn (39).


By gi ta chia cc hng t ca (41) thnh hai lp, lp th nht cha cc hng t m d ' d '' v
lp th hai cha cc hng t m d ' d '' .
Vi s t nhin l d cho trc ta tnh tng cc hng t ca (41) vi d ' d '' d .
T (39) ta thy d l c s ca 2u v n l s l nn n l c s ca u. Vy ta c u d , do
theo (39) th 2 ' '' . iu ny chng t s cc hng t trong (41) m d ' d '' d l bng
vi s cch biu din ca 2 thnh tng ca hai s t nhin l, tc l bng . Nhng mi hng t
u bng +1 nn tng ca chng bng u / d .
T y suy ra tng cc hng t thuc lp th nht l

d d u .
d |u

d |u

Cc hng t thuc lp th hai li c chia thnh hai lp, lp th nht cha cc hng t m
d ' d '' v lp th hai gm cc hng t m d '' d ' . Mi hng t xc nh bi cc b s thuc lp
th nht ng vi ng mt hng t thuc lp th hai xc nh bi b s d ', d '', ', '' . Do ch
cn tnh tng cc hng t thuc lp th nht sau nhn vi hai.
t
(42)

d ''

d ' d ''

CHNG 13. S NGUYN PHC | 299

Mi hng t thuc lp th nht xc nh bi (40) u tng ng vi hng t xc nh bi b s


(43)

d1 ,

d2 , 1 ,

2 ,

vi
(44)

d1 ' 1 ' '' ,

d 2 ' ' '' ,

1 d '' d ' d '' ,

2 1 d ' d '' d ''.

u tin ta chng minh h (43) xc nh bi cng thc (44) s tng ng vi hng t thuc lp
th nht. . V l s nguyn v cc s trong (40) l l nn cc s ' '' v d ' d '' chn.
V vy t (44) suy ra cc s trong (43) u l cc s nguyn l.
Theo (42) th s l khng m v vi hng t thuc lp th nht th d ' d '' . H qu l theo (44)
d ''
d ''
th cc s d1 v d 2 dng. Hn na theo (42) th
v nhn vi
1
d ' d ''
d ' d ''

d ' d '' 0 suy ra d '' d ' d '' d ' d '' d ''. M theo (44) suy ra 1 0 v 2 0 . Nhng
s 1 l l v khc 0 do 1 0 . Vy bn s trong (43) l l v dng. Hn na theo (44) ta c
(45)

d1 d2 ' '' .

chng t d1 d 2 . Hn na
(46)

1 2 d ' d '',

Theo (45) v d11 d2 2 d1 1 2 d1 d2 2 , th d11 d2 2 d1 d ' d '' ' '' 2 .


V vy theo (44) ta c d11 d2 2 ' d ' d '' ' '' d ''.
Theo (39) th d11 d2 2 2u .
T y suy ra b s (43) tng ng vi mt hng t trong nhm th nht.
B s (43) khc vi (40) v nu hai b s l trng nhau th theo (45) ta c d ' d '' ' '' suy ra
theo (39) th 2u d ' d '' ' ' '' d '' ' '' ' d '' v do v ' '' v ' d ''
chn nn 2u chia ht cho 4, mu thun vi gi thit u l.
tnh cc s (40) ta gii phng trnh (44) v suy ra theo (45) v (46) ta nhn c

' d1 1 d1 d2 d2 d1 d2 ,

d '' 1 1 2 .

V vy theo (45) v (46) th

'' d1 d 2 ' 1 d1 d 2 d 2
d ' 1 2 d '' 1 11 2
Theo (44) v (42) ta nhn c
(47)

' ' '' '

' ''

' ''

1 2
d1 d 2

V l s nguyn v '/ ' '' l phn s m t s nh hn mu s. Do


(48)

d ' 1 1 11 2 ,

d '' 1 1 1 2 ,

' d 2 1 d1 d 2 ,

'' 1 1 d1 d 2 d 2 .

300 | nh l Jacobi v tng bn bnh phng

So snh (47) v (48) vi (42) v (44) ta kt lun rng cc b s (43) v (40) tng ng vi nhau
theo tng ng xc nh nh trn. Ni cch khc tng ng m ta va xc nh chng t cc hng
t thuc lp th nht c th chia thnh cc cp m mi cp cha hai hng t, mt hng t xc nh
bi (40) v hng t kia xc nh bi (43). Cc hng t ny tng ng vi nhau bi cng thc (44).
By gi ta tnh cc hng t thuc cng mt cp, ngha l tng
(49)

d ' d '' /2

d1 d2 /2

vi d ', d '', d1 v d 2 tng ng xc nh theo (44).


Theo (39) v (45) ta c 2u d ' d '' ' d1 d2 d '' v do
M cc s ', d '' v u l l nn

d ' d '' d1 d 2

u.
2
2

d ' d '' d1 d 2

1 (mod 2)
2
2

Chng t tng (49) bng 0 . Ni cch khc cc hng t trong cng cp trit tiu nhau.
V vy tng ca cc hng t thuc lp th nht bng 0 v do tng tt c cc hng t trong lp
th hai trong phn hoch th nht l bng 0 . M tng cc hng t thuc lp th nht ca phn
hoch th nht l bng u suy ra theo (41) ta c nh l
nh l 10. Nu u l s t nhin l th 4u 16 u .
nh l ny c pht biu ln u tin theo mt cch khc v c chng minh bi Jacobi [1]
(Bachmann [2] trang 349-354).
By gi gi s
(50)

u 2 2 2 2

l biu din mt s t nhin l u thnh tng bn bnh phng v t


(51)

x ' ,
z ' ,

y'
t'

V w2 w (mod 2) vi mi s nguyn w nn theo (50) ta c x ' u (mod 2) m u l suy ra x ' l.


Hn na t cng thc (50) suy ra

y ' x ' 2 ,

z ' x ' 2 ,

t ' x ' 2 ,

vi cc s x ', y ', z ', t ' u l. Theo (50) v (51) ta thy x '2 y '2 z '2 t '2 4u do b s
(52)

x ', y ', z ', t '

xc nh bi (50) cho mt biu din 4u thnh tng bn bnh phng l.


Mt khc t
(53)

x '' ,
z '' ,

y ''
t ''

Cc s
(54)

x '', y '', z '', t ''

u l v x ''2 y ''2 z ''2 t ''2 4u .


C th thy cc b s (52) v (54) l phn bit v theo (51) v (53) ta c
(55)

x ' y ' z ' t ' 4 , x '' y '' z '' t '' 2

CHNG 13. S NGUYN PHC | 301

M l nn tng cc s (52) chia ht cho 4 trong khi tng cc s (54) th khng.


Theo (51) v (53) th mi biu din mt s l u thnh tng bn bnh phng tng ng vi hai
biu din phn bit cc s 4u thnh tng bn bnh phng l.
By gi ta chng minh mi biu din (31) ca s 4u thnh tng bn bnh phng l tng ng vi
biu din duy nht ca s u thnh tng bn bnh phng.
Tht vy, s s x y z t l tng bn s l trong (31) nn n l s chn. Ta xt hai trng hp
(i) Nu s 0 (mod 4) th t cng thc (53) suy ra (55) v do khng tn ti cc s nguyn
, , , tha mn (50) v tha mn cc s x '', y '', z '', t '' xc nh bi chng l tng ng bng
vi x, y, z, t v t s tn ti cc s nguyn nh vy suy ra s khng chia ht cho 4, mu thun vi gi
thit. Mt khc tn ti ng mt b s nguyn , , , tha mn (50) vi
(56)

x ,
z ,

y ,
t

T (56) suy ra

(57)

x y z t
,
4
x y z t
,
4

x y z t
,
4
x y z t
.
4

V suy ra b s x, y, z, t tng ng vi nhiu nht mt b s , , , m tha mn (57). Nu ta


tnh cc s , , , t (57) th cc s nhn c l nguyn v tha mn (56) v theo (31) th
chng tha mn (50) v suy ra b s x, y, z, t tng ng vi t nht mt b s nh vy. V vy
trong trng hp (i) th ta c tng ng mt-mt gia cc biu din (31) v cc biu din (50)
ca u thnh tng ca bn bnh phng.
(ii) Nu s 2(mod 4) th t cng thc (51) suy ra cng thc (55). Khng tn ti cc s nguyn
, , , m cng thc (51) cho h x ' x, y ' y, z ' z, t ' t v nu ngc li th tng s chia ht
cho 4, mu thun vi gi thit. Mt khc tn ti duy nht b s nguyn , , , tha mn (50)
v tha mn
(58)

x ,
z ,

y ,
t

V t (58) suy ra

(59)

x y z t
,
4
x y z t
,
4

x y z t
,
4
x y z t
.
4

Chng t b s x, y, z, t tng ng vi nhiu nht mt b s , , , tha mn (59). Nu cc s


, , , c tnh t cng thc (59) th chng l s nguyn tha mn (58). V (31), (58) suy ra
b s x, y, z, t tng ng vi t nht mt b s , , , .
Do trong trng hp (ii) th tn ti tng ng mt-mt gia cc biu din (31) v cc biu
din (50) ca s u thnh tng ca bn bnh phng.
Vy s cch biu din 4u thnh tng ca bn bnh phng l l hai ln ln hn s 4 u l s cch
biu din ca s u (l) thnh tng bn bnh phng.
V vy t nh l 10 ta suy ra cng thc

302 | nh l Jacobi v tng bn bnh phng

(60)

4 u 8 u

ng vi mi s t nhin l u. Vy ta c nh l
nh l 11. S cch biu din mt s l thnh tng bn bnh phng l bng vi tng cc c s ca
n nhn vi 8.
Do s cc c s ca mt s l 1 t nht l 4 nn theo nh l 11 ta thy mi s t nhin l 1
u c t nht 32 biu din thnh tng bn bnh phng. V mi bnh phng l u c ng 8
biu din thnh tng bn bnh phng m ba trong s l bng 0 nn suy ra mi bnh phng l
ln hn 1 u l tng ca bn bnh phng m t nht hai trong s l khc 0 . V vy t nh l
Lagrange ta suy ra h qu sau y
H qu. Mi s t nhin l ln hn 1 u l tng ca bn bnh phng m t nht hai trong s l
khc 0 .
By gi ta tnh s cch biu din ca 4u (vi u l) thnh tng ca bn bnh phng.
Gi s
(61)

4u x 2 y 2 z 2 t 2

l mt biu din nh vy.


Nu mt trong cc s x, y, z, t l chn v ch c mt s l l (hoc ch c mt s l chn) th khi
ta c tng ca cc bnh phng l l, mu thun vi (61).
Nu hai trong s cc s x, y, z, t l chn v hai s cn li l l th tng cc bnh phng c dng
4k 2 , mu thun vi (61).
Do tt c cc s x, y, z, t cng chn hoc cng l.
Trng hp x, y, z, t cng l c xt trong nh l 10, t suy ra s cch biu din 4u thnh
tng ca bn bnh phng l.
Vy ch cn xt s cch biu din 4u thnh tng bn bnh phng chn.
D thy mi biu din nh vy c dng 4u 2 2 2 2 tng ng vi biu din
2

ca u thnh tng bn bnh phng l u 2 2 2 2 , v c nh vy. T y suy ra s cch


biu din 4u thnh tng bn bnh phng chn bng vi s cch biu din ca u thnh tng ca
bn bnh phng, do theo (60) th s ny bng 8 u .
Vy tng s biu din 4u (vi u l) thnh tng bn bnh phng l 16 u 8 u 24 u .
V vy vi mi s l u ta c
(62)

4 4u 24 u

Cui cng ta tnh s cch biu din 2u thnh tng bn bnh phng.
Ta s chng minh rng
(63)

4 2u 4 4u .

Tht vy, nu (61) l biu din ca 4u (vi u l) thnh tng bn bnh phng th cc s x, y, z, t
cng chn hoc cng l. Trong mi trng hp th
(64)

x y
x y
z t
z t
,
,
,
2
2
2
2

u l cc s t nhin. Ta vit li (61) di dng

CHNG 13. S NGUYN PHC | 303

x y
x y z t
z t
2u



,
2
2 2
2
2

Suy ra ta nhn c biu din


(65)

2u 2 2 2 2

V vy mi biu din (61) ca 4u thnh tng bn bnh phng u tng ng vi biu din (65)
ca 2u thnh tng bn bnh phng. Mt khc r rng mi biu din (65) ca 2u thnh tng bn
bnh phng u tng ng vi ng mt biu din (61) ca 4u thnh tng bn bnh phng.
Chng minh tnh cht ny c suy ra d dng v biu din (65) tng ng vi biu din (61)
theo tng ng xc nh trn, do cng thc (64) ng. Vy ta nhn c

x, y , z, t ,
Cng thc ny xc nh duy nht biu din (64).
Vy tng ng mt-mt gia cc biu din 4u thnh tng bn bnh phng v cc biu din 2u
thnh tng bn bnh phng c xc nh. Cng thc (63) c chng minh. T (62) suy ra
(66)

4 2u 24 u

ng vi mi s l u.
By gi ta tnh s cch biu din ca 2h u ( h 3, 4,... v u l) thnh tng bn bnh phng.
Gi s
(67)

2h u x 2 y 2 z 2 t 2

l mt biu din nh vy. Cc s x, y, z, t khng cng l v nu nh vy th v phi ca (67) ng


d vi 4 modulo 8 trong khi v tri chia ht cho 8. Tng t nu hai trong s chng l chn v cc
s kia l l th v phi ca (67) ng d vi 2 modulo 4, v l. Vy tt c cc s x, y, z, t cng chn.
t x 2 , y 2 , z 2 , t 2 , vi , , , nguyn. Theo (67) ta c
(68)

2h 2 u 2 2 2 2

V vy mi biu din dng (67) ca 2h u thnh tng ca bn bnh phng tng ung vi biu
din (68) ca s 2h 2 u thnh tng bn bnh phng. Mt khc r rng mi biu din ca 2h u u
tng ng vi biu din 2h u 2 2 2 2 . V vy
2

(69)

4 2h u 4 2h 2 u

vi mi h 3 v vi mi s t nhin l u . By gi t s l s t nhin v u l s t nhin l.


Nu s 1 hoc s 2, th theo (66) hoc (62) tng ng ta c
(70)

4 2s u 24 u .

Nu s 2 , ta xt hai trng hp
(i) Nu s 2k th t (69) ta c 4 2s u 4 22 k u 4 22 k 2 u 4 22 k 4 u ... 4 22 u suy ra
cng thc (70) ng.
(ii) Nu s 2k 1 th theo (69) ta c 4 2s u 4 22 k 1u 4 22 k 1u ... 4 23 u 4 2u , suy
ra theo (66) th cng thc (70) ng.
Vy cng thc (70) ng vi mi s t nhin s v s t nhin l u .

304 | nh l Jacobi v tng bn bnh phng

Cng thc (60) v (70) c th kt hp thnh mt nh l: gi s n l s t nhin ty v k hiu


* n n l tng cc c s ca s t nhin n m khng chia ht cho 4.
Nu n u l th khng c c s no ca n chia ht cho 4 do

* n n .

(71)

Nu n chn th t n 2s u vi s l s t nhin v u l s t nhin l. R rng mi c s ca s


2s u m khng chia ht cho 4 cng l c s ca 2u v ngc li mi c s ca 2u u l c s
ca 2s u m khng chia ht cho 4. Do * n * 2s u 2u , m 2, u 1 suy ra

2u 2 u 3 u ,
Ta c

* n 3 u .

(72)

Cc cng thc (71) v (72) kt hp vi (60) v (70) chng t

4 n 8 * n

(73)

vi mi s t nhin n . Vy ta chng minh c nh l


nh l 12. S cch biu din mt s t nhin n thnh tng ca bn bnh phng l bng vi tm
ln tng cc c s khng chia ht cho 4 ca n .
V mi s t nhin u c t nht mt c s khng chia ht cho 4 (chng hn c s bng 1) nn
t nh l 12 suy ra mi s t nhin l tng ca bn bnh phng. nh l ny c chng
minh trong Chng 11 theo mt cch khc. Mt ti liu c lin quan ti s cch biu din mt s
thnh tng cc bnh phng c cho bi E.Grosswald [2].

53 1
Bi tp. Theo (70) ta c 4 100 24 25 24
24.31 744. Do 100 c 744 biu din
5 1
33 1 52 1
thnh tng bn bnh phng. Tng t 4 90 24 45 24
.
24.13.6 1872. y
3 1 5 1
l s 100 c nhiu biu din thnh tng bn bnh phng nht. Ta c 4 7 8 7 8.8 64,

4 6 24 3 24.4 96, 4 96 24 3 24.4 96, 4 1024 4 210 24 1 24.


x

x x



Theo (73) ta c 4 n 8S x 32S . vi S x k 1 . k.
4
k
2
k
k
n1

T y suy ra

n
n1

2
6

2 x2
2

k 1

k 1

100 x x , vi mi s nguyn x v suy ra cng thc Euler

1 1 1 1
... bng phng php thun ty s hc.
12 22 32 42

TI LIU THAM KHO


ABBOTT, H.L v HANSON, D., [1] A problem of Schur amd its generalizations, Acta Arith. 20 (1972) 175-187.
n
AIGNER, A., [1] Folgen der Art ar b , welche nur teilbare Zahlen liefern, Math, Narch 23 (1961) 259-264.
ALAOGLU, L, v ERDOS, P, [1] On highly composite and similar numbers, Trans.Ames.Math.Soc 56 (1944)
448-469. [2] A conjecture in elementary number theory, Bull.Ames.Math.Soc 50 (1944) 881-882.
ANKENY, N.C, [1] Sum of the three squares, Proc.Ames.Math.Soc 8 (1957) 316-319.
ANNING, N.H, v ERDOS, P, [1] Integral distances, Bull.Ames.Math.Soc 51 (1945) 598-600.
AVANESOV, E.T, [1] Solution of a problem on figurate numbers (ting Nga), Acta.Arith. 12 (1966) 409-420
AVANESOV, E.T v GUSEV, V.A, [1] On a problem of Steinhaus (ting Nga), Math.Chasopis 21 (1971) 29-32
[2] On Steinhauss problem (ting Nga), bn tho ti Referativny Zhurnal Math. (1984), No 7A102.
BACHMANN, P, [1] Niedere Zahlentheorie, Encyklopadie der Mathematischenwwissen shafeten mit
Einschlussihrer Anwendungen IC 1, 555-581, (Leipzig 1989 -1904)
[2] Niedere Zahlentheorie II (Leipzig 1901, in li ti New York 1968).
BAILLIE, R.[1] Table of n n 1 , UMT file, Math.Comp. 30 (1976) 289-190.
[2] Solutions of n n 1 For Eulers function, UMT file, cf. Math, Comp. 32 (1978) 1326.
BAILLIE, R, CORMACK, G.V v WILLIAM, H.C, [1] The problem of Sierpinski concerning
k 2n 1, Math.Comp. 37 (1981) 229-231, Corrigendum, ti liu dn 39 (1982) 308.
BAKER, A., [1] Contributions to the theoru of Diophantine equations II. The Diophatine equation
y 2 x3 k , Philos.Trans. Roy. Soc. London A 263 (1968) 193 208.
BAKER, C.L v GRUENBERGER, F.J, [1] The first six million prime numbers. Madison, Wisc. 1959
(Microcards)
BALASUBRAMANIAN,R, DRESS, F. v DESHONILLES, J, M, [1] Problme de Waring pour les bicarrs,
C.R.Acad.Sci.Paris.
BALOG, A,. [1] p a Without large prime factors, Tagungsbericht 44 (1984), Mathematisches
Forschungsbericht, Oberwolfach.
BANG, A.S, [1] U ber Summen von f u nften Potenzen, Neuvie`mecongre`s des math. Scand . 1938,292-296
(Helsinki 1939).
BANG, T, [1] Large prime numbers (ting an Mch), Nordisk Mat.Tidskr. 2 (1954) 157-168
BAUMERT, L.D, [1] Sum-free sts, jet Propulsion Laboratory Res.Summary No 36-10, 1 (1961) 16-18.
BECK, W.E v NAJAR, R.M [1] A lower bound for odd triperfects, Math, Comp. 38 (1982) 249-251
m
BEEGER, N.G.W.H, [1] On even numbers m dividing 2 2, Amer. Math. Monthly 58 (1951) 553-555

[2] Cullen numbers, Math, Tables AidsComp .8(1954) 188


BEHREND, F.A [1] On sets of integers which contain no three terms in arithmetical progression,
Proc. Nat. Acad . Sci.U . S. A. . 32 (1946) 331-332
BELL, E.T [1] Reciprpcal arrays and Diophantine anlaysis, Amer.J . Math 55 (1933) 50-66
BENDZ, T.R, [1] Ofver diophantiskaekvantionen x y z . Diss, (Upsala 1901).
n

BEST, M.R v te RIELE, H.J.J [1] OnaConjectureof ErdosConcerning Sums of

Powers of Integers, Report NW 23/76 Mathematishch Centrum,(Amsterdam 1976)


BEYER, W.A, METROPOLIS, N, v NEUGERARD, J.R [1] Squares roots of integers 2 to 15 in various bases 2 to
10: 88062 binary digits or equivalent, UMT file, cf. Math, Comp. 23 (1969) 679
BIEBERBACJ, L, [1] Uber Stefelsche magische Quadrate I, Arch. Math. 5. (1954) 4-11
BLANUSA, D, [1] Une interprtation gometrique du crible dErathostene (Sebo-Crotian),
Glasnik Mat.Fiz.Astronom.Drustvo Mat.Fiz.Hrvatske (2) 4 (1949) 201-202
BOCHNER, S, [1] Remk on the Euclidean algorithm, J .London Math. Soc. 9(1934) 4.
BORMAN, J, [1] The number of primes less than a given limit, Nordisk Tidskr.

Informationsbehandling (BIT), 12 (1972) 576-577


[2] Some computational results regarding the prime numbers below 2000000000,
Nordisk Tidskr. Informationsbehandling (BIT), 13 (1973) 242-244
BOREL, E [1] Les probabitites denombrables et leurs applications arithmetiques,

306 | TI LIU THAM KHO

Rend .Circ. Mat. Palermo 27 (1909) 247-271


[2] Sur les chiffres decimaux de 2 Et divers problmes de probabit en chaine,
C.R. Acad . Sci. Paris 230 (1950) 591-593
BORHO, W, [1] Befreundete Zahlen: ein zewitausend Jahre altes Thema der elementaren Zahkentheorie,
in Lebendige Zahlen. 5-38 (Basel- Boston- Stuttgart 1981)
BOROZDKIN, K.G [1] On the problem of I.M. Vinogradovs constant (ting Nga),
Trudy tretegovsesojuznogomatematicheskogo s ' ezda, Vol. I, 3, (Moskva 1956)
BOUNIAKOWSKY, V, [1] Notes sur quelques points de lanalyse indeterminee,
Bull. Acad . Sci. St. Petersbourg 6 (1848) 196-199
[2] Sur les diviseurs numerriques invariables des functions rationales entieres,
Acad . Sci. St. Petersbourg Mem. (6), Sci. math. et phys .6 (1857) 305-329
BRAUER, A, [1] Uber einige spezielle diophantische Glechungen, Math. Z . 25 (1926) 499-504
[2] On a property of k consecutive integers, Bull. Amer. Math. Soc. 47 ( 1941) 328-331
BRAUER, A, v REYNOLDS, R.L, [1] On a theorem of AubryThue, Canadian J .Math.3. (1951) 367-374
BREDIHIN, B.M [1] Binary addictive problems of indeterminate type (ting Nga) I, II, I zv .
Akad . Nauk. SSSR, Ser. mat. 27 (1963) 439-462, 577-612
BREMNER, A, [1] Integer points in a special cubric surface, Duke Math. J . 44 (1977) 757-765
BRENT, R.P, [1] The first occurrence of large gaps between successive primes,
Math.Comp 27 (1973) 959-963.
[2] Irregularities of distribution of primes and twin primes, Math.Comp 29 (1975) 43-56.
[3] Tables concerning irregularities in the distribution of primes and twin primes,
UMT file, cf. Math.Comp. 30 (1976) 379.
[4] The first occurrence of certain large prime gaps, Math.Comp. 35 (1980) 1435-1436.
BRILLHART, J., LEHMER, D.H., SELFRIDGE, J.H., TUCKERMAN, B., WAGSTAFF, S.S. Jr.,
n
[1] Factorizationof b 1, b 2,3,5,6,7,10,11,12 upto High Power , (Providence 1983).
BRILLHART, J., TONASCIA, J. v WEINBER GER, P.,[1] On the Fermat quotient, in: A.O.L.
Atkin and B.J Birch (eds.), Computersin NumberTheory, 213-222 (London 1971).
BROMHEAD, T., [1] On square sums of squares, Math.Gaz . 44 (1960) 219-220.
BROWKIN, J., [1] Certain property of triangular numbers (in Polish), Wiadom. Mat. 2 (1957-59) 253-255.
[2] Solution of a certain problem of A.Schinzel (in Polish), Prace Mat. 3 (1959) 205-207.
BROWN, A.L., [1] Multiperfect numbers, Scripta Math. 20 (1954) 103-106.
[2]Multiperfect numbers - Cousins of the perfect numbers - No.1,
Recreational Math. Mag. 14 (1964) 31-39.
BROWN, J.L. Jr., [1] On Lame's Theorem, Fibon.Quart . 5 (1967) 153-160.
[2] Generalization of Richert's theorem, Amer. Math. Monthly 83 (1976) 631-634.
BRUN, V., [1] La srie 1 1 1 1 1 1 1 1 1 1 1 1 ... ou l denominateur
5

7 11 13 17 19

29

31 41 43 59

61

[4] On a Diophantine equation, Acta Arith. 6 (1960) 47-52.


CATTANEO, P., [1] Sui numeri quasiperfetti, Boll. Un. Mat. Ital. (3) 6 (1951) 59-62.
Cel, J., [1 ] On decomposition of a cube into the difference of two biquadrates (in Polish),
Matematyka 36 (1983) 308-310.
CHAKRABARTI, M.C., [1] On the limit points of a function connected with the three-square problem,
Bull. Calcutta Math. Soc. 32 (1940) 1-6.
CHAMPERNOWNE, G.D., [1] Construction of decimals normal in the scale often,
J. London Math. Soc. 8 (1933) 254-260.
CHEIN, E. Z., [1] Some remarks on the exponential Diophantine equation,
Notices Amer. Math. Soc, 26 (1979) A-426.
[2] Remark on the binomial coefficients, Notices Amer. Math. Soc. 26 (1979) A-506.
CHEN, J. R., [1] Waring's problem for g(5), Sc. Sinica 12 (1964), 1547-1568.
[2] On the representation of a large even number as the sum of a prime
and the product of at most two primes, Sc. Sinica 16 (1973) 157-176.
[3] On the least prime in an arithmetical progression and theorems

TI LIU THAM KHO | 307

concerning the zeros of Dirichlet's L functions, Sci. Sinica 22.(1979) 859-889.


CHERNICK, J, [1] On Fermat's simple theorem, Bull. Amer. Math. Soc. 45 (1939) 269-274.
CHIKAWA, K.,ISKI, K. v KUSAKABE, T., [1] Ona problem by H. Steinhaus, Acta Arith. 1 (1962) 251-252.
CHIKAWA, K., ISKI, K., Kusakabe, T. v SHIBAMURA, K., [1] Computation of cyclic parts of Steinhaus
problem for power 5, Acta Arith. 7 (1962) 253- 254 and Corrigendum, ibid. 8 (1963) 259.
CHOI, S.L.G., [1] Covering the set of integers by congruence classes of distinct moduli,
Math. Comp. 25 (1971) 885-895.
CHOJNACKA-PNIEWSKA, M., [1] Sur les congruences aux racines donn6es, Ann. Polon. Math. 3 (1956) 9-12.
CHOWLA, S., [1] An extension of Heilbronn's class-number theorem.
Quart.J. Math. Oxford Ser. 5 (1934), 304-307.
[2] There exists an infinity of 3-combinations of primes in A.P.,
Proc. Lahore Philos. Ser. 6, no 2 (1944) 15-16.
CHOWLA, S. v BRIGGS, W.E., [1 ] On discriminants of binary quadratic forms with a
single class in each genus, Canadian J. Math. 6 (1954) 463-470.
P 1

CIPOLLA, M., [1] Sui numeri composti P, che verificano la congruenza di Fermat a 1mod P , Ann. Mat.
Pura Appl. (3) 9 (1903) 139-160.
CLEMENT, P.A., [1] Congruences for sets of primes, Amer. Math. Monthly 56(1949) 23-25.
[2] Representation of integers in the form: a-th power plus a prime,
Amer. Math. Monthly 56 (1949) 561.
COBLYN, [1] Sur les couples de nombres premiers, Soc. Math, de France, C.R. des Stances 55 (1913), 55-57.
COGHLAN, F.B., v STEPHENS, N. M., [1] The diophantine equation x y k , in: A.O.
L. Atkin and BJ. Birch (eds), Computers in number theory, 199-205 (London 1971).
COHEN, E., [1] Arithmetical notes. V. A divisibility property of the divisor function,
Amer. J. Mart. 83 (1961) 693-697.
3

COHEN, G.L. v HAGIS, P. Jr., [1] On the number of prime factors of n if n | n 1 ,

Nieuw Arch. Wisk. (3) 28 (1980) 177-185.


COHEN, H., [1] On amicable and sociable numbers, Math. Comp. 24 (1970) 423-429.
COLOMBO, M., [1] Tavole numeriche e diagrammi sulla distribuzione delle coppie di numeri primi a
differenza fissa, 1st. Lombardo Sci. Lett. Rend. A 93 (1959) 95-133.
de COMBEROUSSE, C, [1] Algbre suprieure 1 (Paris 1887).
COPELAND, A. v ERDOS, P., [1] Note on normal numbers, Bull. Amer. Math. Soc, 52 (1946) 857-860.
CORMACK, G.V. v WILLIAMS, H.C. [1] Some very large primes of the form k 2 1 ,
Math. Comp. 35 (1980) 1419-1421.
van der CORPUT, J.G., [1] Sur l'hypothese de Goldbach pour presque tous les nombres pairs,
Acta Arith. 2 (1937) 266-290.
[2] ber Summen von Primzahlen und Primzahlquadraten, Math. Ann. 116 (1939) 1-50.
[3] On de Polignac's conjecture (in Dutch), Simon Stevin 27 (1950) 99-105.
n

COUSTAL, R., [1] Calcul de 2 , et reflexion sur une esprance mathrnatique,


C.R. Acad. Sci. Paris 230 (1950) 431-432.
CRAMR, H., [1] On the order of magnitude of the difference between consecutive prime numbers,
Acta Arith. 2 (1936) 396-403.
CROCKER, R., [1] A theorem concerning prime numbers, Math. Mag. 34 (1960/61) 3.16,344.
[2] On the sum of a prime and of two powers of two. Pacific J. Math. 36 (1971) 103-107.

CUNNINGHAM, A.J.C. v Woodall, H.J., [1] Factorization of O 2q q and q

2q

1 .

Messenger Math. 47 (1917) 1-38.


DANILOV, L.V., [1] Letter to the editors (ting Nga) Mat. Zam. 36 (1984) 457-459.
DAVENPORT, H., [1] On Waring's problem for fourth powers, Ann. of Math. (2) 40 (1939) 731-747.
[2] The Higher Arithmetic, An Introduction to the Theory of Numbers
(London and New York 1952, fifth ed. Cambridge 1982).
DEM'YANENKO, VA., [1] On Jemanowicz problem for Pythagorean numbers (ting Nga),
Izv. Vyssh. Uchebn. Zaved. Matematika .1965, no 5 (48) 52-56.
[2] Sums of four cubes (ting Nga), Izv. Vyssh. Uchebn. Zaved. Matematika 1966, no 5 (54) 64-69.
[3] On a conjecture of A. Schinzel (ting Nga),
Izv. Vyssh. Uchebn. Zaved. Matematika 1975, no 8 (159) 33-45.
[4] On a conjecture of A. Mqkowski (ting Nga),
Izv. Vyssh. Uchebn. Zaved. Matematika 1976, no 10 (173) 29-31.

308 | TI LIU THAM KHO

DNES, P, [1] ber die Diophantische Gleichung x y cz Acta Math. 88 (1952) 212-251.
DEPMAN, I.YA., [1] The notable Slavic computers G. Vega and Ya. F. Kulik (ting Nga),
Istor.-Mat. Issled. 6 (1953) 593-604.
DESBOVES, A., [1] Sur un thorme de Legendre et son application la recherche de limites
qui comprennent entre elles des nombres premiers, Nouv. Ann. Math. 14 (1855) 281-295.
DEVITT, J.S. [1] Aliquot sequences, MSc. thesis, The Univ. of Calgary 1976,
cf. Math. Comp. 32 (1978) 942-943.
DICKSON, L.E., [1] A new extension of Dirichlet's theorem on prime numbers,
Messenger Math. 33 (1904) 155-161.
[2] Amicable number triples, Amer. Math. Monthly 20 (1913) 84 91.
[3] Theorems and tables on the sum of the divisors of a number,
Quart. J. Pure Appl. Math. 44 (1913) 264-296.
[4] Proof of the ideal Waring theorem for exponents 7-180, Amer. J. Math. 58 (1936) 521-529.
[5] Solution of Waring's problem, Amer. J. Math. 58 (1936) 530-535.
[6] Modern Elementary Theory of Numbers (Chicago 1939).
[7] History of the Theory of Numbers, 3 vols. (Washington 1919-1923, in li ti New York 1966)
l

DIRICHLET, P.G.L., [1] Sur l'quation t u v w 4m , J. Math. Pures Appl. (2) 1 (1856) 210-214.
DIXON, J.D., [1] The numbers of steps in the Euclidean algorithm. J. Number Theory 2 (1970) 414-422.
[2] A simple estimate for the number of steps in the Euclidean algorithm,
Amer. Math. Monthly 78 (1971) 374-376.
2

DRESS, F., [1] Amlioration de la majoration de g 4 dans le probleme de Waring:

g 4 30 , Acta Arith. 22 (1973) 173-147


DRESSLER, R.E., MAKOWSKI, A. v PARKER, T, [1] Sums of distinct primes from congruence classes modulo
12, Math. Comp. 28 (1974) 651-652.
DUPARC, HJ.A., [1] On Carmichael numbers, Simon Stevin 39 (1952) 21-24.
[2] On Mersenne numbers and Poulet numbers,
Math. Centrum Amsterdam, Rapport ZW 1953 001 (1953).
DUTKA, J., [1] The square root of 2 to 1000000 decimals, Math. Comp. 25 (1971) 927-930.
DYER-BENNET, J., [1] A theorem on partitions of the set of positive integers,
Amer. Math. Monthly 47 (1940) 152-154.
EDITORIAL NOTE, [1] Editorial Note, Math. Comp. 15 (1961) 82.
ERDS, P., [1] Beweis eines Satzes von Tschebyschef, Acta Litt. Sci. Szeged 5 (1932) 194-198.
[2] A theorem of Sylvester and Schur, J. London Math. Soc. 9 (1934) 282-288.
[3] On the normal number of prime factors of p 1 and some related problems concerning
Euler's -function, Quart, J. Math. Oxford Ser. 6 (1935) 205-213.
[4] On the sum and difference of squares of primes,
J. London Math. Soc. 12 (1937) 133-136, 168-171.
[5] Note on products of consecutive integers, J. London Math. Soc. 14 (1939) 194-198.
[6] Integral distances, Bull. Amer. Math. Soc. 51 (1945) 996.
[7] On some applications of Brun's method. Acta Unir. Szeged Sect. Sci. Math. 13(1949) 57-63.
[8] On the converse of Fermat's theorem, Amer. Math. Monthly 56 (1949) 623-624.
[9] On a new method in elementary number theory which leads to an
elementary proof of the prime number theorem, Proc. Nat. Acad. Sci. U.S.A. 35 (1949) 374-384.
k
[10] On integers of the form 2 p and some related problems,
Summa Brasil. Math. 2 (1950)113-123.
[11] On a Diophantine equation, J. London Math. Soc. 26 (1951) 176-178.
[12] On consecutive integers, Nieuw Arch. Wisk. (3) 3 (1955) 124-128.
[13] On amicable numbers, Publ. Math. Debrecen 4 (1955) 108-111.
[14] Some remarks on Euler's function. Acta Arith. 4 (1958) 10-19.
[15] Solution of two problems of Jankowska,
Bull. Acad. Polon Sci. Ser. Sci. Math.Astr.Phys. 6 (1958) 545-547.
[16] Some remarks on the functions <p and a,
Bull. Acad. Polon. Sci. Se'r. Sci. Math. Astr. Phys. 10 (1962) 617-619.
[17] ber die Zahlen der Form n n and n n Elem. Math. 28 (1973) 83-86.

TI LIU THAM KHO | 309

ERDS, P. v MIRSKY, L, [1] The distribution of values of the divisor function d n ,


Proc.London Math. Soc. (3) 2 (1952) 257-271.
ERDS, P. v OBLTH, R., [1] ber diophantische Gleichungen der Form n ! x y and n! m! x ,
Acta Litt. Sci. Szeged 8 (1936) 241-255.
ERDS, P. v RNYI, A., [1] Some problems and results on consecutive primes,
Simon Stevin 27 (1950) 115-125.
ERDS, P. v SCHINZEL, A., [1] Distributions of the values of some arithmetical functions.
Acta Arith. 6 (1961) 473-485.
ERDS, P. v SELFRIDGE, J.L., [1] The product of consecutive integers is never a power.
Illinois J. Math. 19 (1975) 292-301.
ERDS, P. v TURN, P., [1] On some sequences of integers, J. London Math. Soc. 11 (1936) 261-264.
[2] On some new questions on the distribution of prime numbers,
Bull. Amer. Math. Soc. 54 (1948) 371-378.
ESTERMANN, T, [1] Einige Satze ber quadratfreie Zahlen, Math. Ann. 105 (1931) 653-662.
[2] Note on a paper of A- Rotkiewicz, Acta Arith. 8 (1963) 465-467.
p

FABER, G., [1] ber die Abzhlbarkeit der rationalen Zahlen, Math. Ann. 60 (1905)196-203.
FERMAT, P., [1] Oeuvres, vol. II (Paris 1894).
FINSLER, P., [1] ber die Primzahlen zwischen n und 2n ,
Festschrift zum 60. Geburtstag von Prof. Dr Andreas Speiser, 118-122 (Zrich 1945).
FRANQUI, B., v GARCIA, M, [1] Some new multiply perfect numbers,
Amer. Math.Monthly 60 (1953) 459-462.
[2] 57 new multiply perfect numbers, Scripta Math. 20 (1954) 169-171.
FREDERICKSEN, H, [1] Schur numbers and the Ramsey number N 3,3,...,3;2 J.

Combin.Theory, Ser. A. 27 (1979) 376-377.


FROBENIUS, G., [1] ber quadratische Formen, die viele Primzahlen darstellen,
S. Ber. Preuss. Akad. Wss. Phys. Math. Kl. 1912, 966-980.
FRBERG, C.E., [1] Some Computations of Wilson and Fermat Remainders,
Math. Tables Aids Comp. 12 (1958) 281.
[2] Investigation of the Wilson remainders in the interval 3 p 50000 ,
Ark. Mat. 4 (1963) 479-499.
FRCHTL, K., [1] Statistische Untersuchung uber die Verteilung von Primzahl-Zwillin-gen,
Anz. ster. Akad. Wiss. Math. Nat. Kl. 87 (1950) 226-232.
FUETER, R., [1] ber kubische diophantische Gleichungen, Comment. Math. Helv. 2 (1930) 69-89.
GABARD, E., [1] Factorisations et quation de Pell, Mathesi 67 (1958) 218-220.
GABOWICZ, JA., [1] Solutions of the equation x y z t 1 in natural numbers (ting Ba Lan),
Wiadom. Mat. 7 (1963) 63-64.
3
3
3
GARDINER, V.L., Lazarus, R.B. v Stein, P.R., [1] Solutions of the diophantine equation x y z d ,
Math. Comp. 18 (1964) 408-413.
GASPER, R.WJr., [1] Table of simple continued fractions for and the derived decimal approximation,
UMT file, cf. Math. Comp. 31 (1977) 1044.
GELFOND, A.O., [1] A common property of number systems (ting Nga),
Izv. Akad. Nauk SSSR, Ser. Mat. 23 (1959) 809-814.
GEORGIEV, G., [1] On the solution in rational numbers of certain diophantine equations (ting Ba Lan)
Prace Mat. 1 (1955) 201-238.
m
n
GERONO, C.G., [1] Note sur la rsolution en nombres entiers et positife de l'quation x y 1 ,
Nouv. Ann. Math. (2) 9 (1870) 469-471, 10 (1871) 204-206.
GILLOUD, J.v BOURGEr, M., [1] Un million de decimals de (Paris 1974)
GINSBURG, J., [1] The generators of a Pythagorean triangle, Scripta Math. 11 (1945) 188.
GIUGA, G, [1] Su una presumabile propriet caratteristica dei numeri primi,
Ist. Lombardo Sci. Lett. Rend CI. Sci. Mat. Nat. (3) 14 (1950) 511-528.
GLAISHER, J.W.L., [1] Mathematical notes 1. An arithmetical proposition,
Messenger of Mart. (2) 2 (1873)41-43.
[2] Number Divisor Tables (Cambridge 1940).
3
3
3
GODWIN, HJ.,[1] A note x y z 1 , J. London Math. Soc. 32 (1957) 501-503.
3

310 | TI LIU THAM KHO

GOLOMB, S., [1] Sets of primes with intermediate density, Math. Scand. 3 (1955) 264-274.
GOLUBEW, WA., [1] Abzhlung von Vierlingen" von 2000000 bis 3000000 und von Fnflingen"
von 0 bis 2000000 , Anz. sterr. Akad. Wiss. Math. Nat. Kl. 93 (1956)153-157.
[2] Abzhlung von Vierlingen" und ..Fnflingen" bis zu 5000000 und von Sechslingen" von 0 bis 14000000, Anz. sterr. Akad. Wiss. Math Nat. Kl. 94 (1957) 82-87.
[3] Abzhlung von Vierlingen" und Fnflingen" bis zu 10000000 ,
Einige Formeln, Anz. sterr. Akad. Wiss. Math. Nat. Kl. 94 (1957) 274-280.
[4] Abzhlung von Vierlingen" und Fnflingen" bis zu 15000000 ,
Anz. sterr. Wiss. Math. Nat. Kl. 96 (1959) 227-232.
[5] Primzahlen der Form x 1 , Anz. sterr. Akad. Wiss. Math. Nat. Kl. 95 (1958) 9- 13;
96 (1959) 126-129; 97 (1960) 39-44, 312-319; 98 (1961) 59-63; 99 (1962) 33-37.
2

[6] Primzahlen der Form x 7 , Anz. sterr. Akad. Wiss. Math. Nat. Kl.
98 (1961) 165- 169; 100 (1963) 244-251.
GOODSTEIN E., [1] A note on magic squares, Math. Gaz. 24 (1940) 117.
GRAHAM, S., [1] On Linnik's constant, Acta Arith. 39 (1981) 163-179.
GROSSWALD, E., [1] Negative discriminants of binary quadratic forms with one class in each gems,
Acta Arith. 8 (1963) 295-306.
[2] Representation of integers as sums of squares (New York 1985).
GROSSWALD, E., CALLOWAY, A., and CALLOWAY, J., [1] The representation of integers by three positive
squares, Proc. Amer, Math. Soc. 10 (1959) 451-455.
GROSSWALD, E. v HAGIS, P., Jr., [1] Arithmetic progressions consisting only of primes,
Math. Comp. 33 (1979) 1343-1352.
GRUBE, F., [1] Ueber Einige Euler'sche Stze aus der Theorie der quadratischen Formen, Zeitschr.
Math. Phys. 90 (1874) 492-519.
2

GUPTA, H., [1] Congruence properties of n , Math. Student 13 (1945) 25-29.

[2] A table of values of N 2 t , Res. Bull. East Penjab Univ. 1952 no. 20, 13-93.
GUY, R.K. [1] Unsolved Problems in Number Theory (New York-Heidelberg-Berlin 1981).

GUY, R.K. v Shanks, D., [1] A constructed solution of n n 1 , Fibon. Quart. 12 (1974) 299.
HADWIGER, H., [1] Ungelste Probleme Nr 24, Elem. Math. 13 (1958) 85.
HAGIS, P., Jr., [1] A lower bound for the set of odd perfect numbers, Math. Comp. 27 (1973) 951-953.
[2] Outline of a proof that every odd perfect number has at least eight prime factors,
Math. Comp. 34 (1980) 1027-1032.
HAGIS, P., Jr., v COHEN, G.L., [1] Some results concerning quasiperfect numbers,
J. Austral. Math. Soc. 33 (1982) 275-286.
HALL,M., Jr., [1] On the sum and product of continued fractions Ann. of Math. (2) 48 (1947) 966-993.
[2] Cyclic projective planes, Duke Math., J. 4 (1947) 1079-1090.
3
3
[3] The Diophantine equation x y , in: A. O. L. Atkin and B.J. Birek (eds.).
Computer in Number Theory, 173-198 (London 1971).
HALTER-KOCH, F., [1] Darstellung natrlicher Zahlen als Summe von Quadraten,
Acta Arith. 42 (1982) 11-20.
HANLY, V.S., [1] A proposition equivalent to Dirichlet's theorem, Amer. Math. Monthly M 64 (1957) 742.
HARDY, G.H. v WRIGHT, E.M., [1] An Introduction to the Theory of Numbers (Oxford 1954)
HARRIS, V.C, [1] A modification of the sieve of Eratosthenes, Amer. Math. Monthly 60 (1953) 325-326.
HASSE, H., [1] Vorlesungen liber Zahlentheorie (Berlin-Gttingen-Heidelberg 1950)
[2] ber eine diophantische Gleichung von Ramanujan-Nagell und ihre Verall meinerung,
Nagoya Math. J. 21 (1966) 77-102.
HAUSDORFF, F., [1] Grundzllge der Mengenlehre (Leipzig 1914)
HAUSSNER, R.,[1] Uber die Verteilung von LUcken und Primzahlen, J.ReineAngew. Math. 168 (1932) 192.
HEATH-BROWN, D.R., [1] The divisior function at consecutive integers. Mathematika 31 (1984) 141-149.
HECKE, E., [1] Eine neue Art von Zetafunktionen und ihre Beziehungen zur Verteilung der Primzahlen II,
Math. Z. 6 (1920) 11-51.
HEMER, O., [1] On the Diophantine Equation y k x , Diss. (Upsala 1952).
2

[2] Note on the Diophantine equation y k x , Ark. Mat. 3 (1954) 67-77.


2

TI LIU THAM KHO | 311

HENSEL, K., [1] Ueber den grssten gemeinsamen Theiler aller Zahlen, welche durch erne ganze Function
von n Vernderlichen darstellbar sind, J. Reine. Angew. Math. 116 (1896) 350-356.
HENSLEY, D. v RICHARDS, I., [1] Primes in intervals, Acta Arith. 25 (1974) 375-391.
HILL, J.D., [1] Solution of the problem 3449, Amer. Math. Monthly 38 (1931) 298-299.
HORNFECK, B. und WIRSING, E., [1] ber die Hufigkeit vollkommener Zahlen,
Math.Ann. 133 (1957) 431-438.
HOOLEY, C, [1] On the power-free values of polynomials, Mathematika 14 (1967) 21-26.
HUNSUCKER, J.L., NEBB, J. v STARNS, R.E., [1] Computational results concerning some equations involving

n , Math. Student 41 (1973) 285-289.

HURWITZ, A., [1] Uber eine besondere Art der Kettenbruch-Entwicklung reeller Grbssen,
Acta Math. 12 (1889) 367-405.
[2] Somme de trois carrs, Intermdiaire Math. 14 (1907) 106-107.
HYYR, S., [1] On the Catalan problem (in Finnish), Archimedes 1 (1963) 53-54.
IRVING, R.W., [1] An extension of Schur's theorem on sum-free partitions, Acta Arith. 25 (1973) 55-63.
ISKI, K., [1] A problem of number theory, Proc. Japan Acad. 36 (1960) 578-583.
[2] Necessary results for computation of cyclic parts in Steinhaus problem,
Proc. Japan Acad. 36(1960)650-651.
ISEKI, K. v TAKADA, I., [1] On Steinhaus problem in number theory, Computation of cyclic parts of
Steinhaus problem for power 9, Mathem. Seminar Notes Kobe Univ. 8 (1980) 227 231.
IVIC A., [1] The Riemann Zeta-F unction, the Theory of the Riemann Zeta-Function with Applications (New
York-Chichester-Brisbane-Toronto-Singapore 1985).
IWANIEC, H., [1] Almost primes represented by quadratic polynomials, Invent. Math. 47 (1978) 171-188.
JACOBI, C, [1] De compositione numerorum ex quatuor quadratis, J. Reine Angew. Math. 12 (1834) 167-172.
JAESCHKE, G., [1] On the smallest k such that all k 2 1 are composite, Math. Comp. 40 (1983)381-384.
n

JAKOBCZYK, F., [ 1 ] Les applications de la fonction g n l'tude des fractions priodiques et de la


congruence chinoise 2 2 0 mod n , Ann. Univ. Mariae Curie-Sklodowska, Sect. A, 5 (1951) 97-138.
n

JANKOWSKA, S., [1] Les solutions du systeme d'quations

x y et x y pour

x y 10000 , Bull. Acad. Polon. Sci. Ser Sc. Math. Astr. Phys. 6 (1958) 541-543.
JESMANOWICZ, L., [1] Several remarks on Pythagorean triangles (in Polish), Wiadom. Mat. 1 (1956) 196202,de JONCOURT, E., [1] De Natura et Praeclaro Usu Simplicissimae Speciei NumerorumTrigonalium (Hagae
1762).
JONEs, B.W. v PALL, G., [1] Regular and semiregular positive ternary quadratic forms.
Acta Math. 70 (1939) 165-191.
JORDAN, C, [1] Trait des substitutions (Paris 1870).
JOZEFIAK, T., [1] A curiosity concerning triangular numbers (in Polish), Matematyka 13 (1960) 327.
[2] On a hypothesis of L. Jemanowicz concerning Pythagorean numbers (ting Ba Lan),
Prace Mat. 5 (1961) 119-123.
KANOLD, H.J., [1] Untere Schranken fr teilerfremde befreundete Zahlen, Arch. Math. 4 (1953) 399-401.
[2] ber zahlentheoretische Funktionen, J. Reine Angew. Math. 195 (1955) 180-191.
KELLER, W., [1] Factors of Fermat numbers and large primes of the form k 2 1 ,
Math. Comp. 41 (1983)661-673.
[2] New factors of Fermat numbers, Abstracts Amer. Math. Soc. 5 (1984) 391-392.
n
[3] The 17th prime of the form 5 2 1 , Abstracts Amer. Math. Soc. 6 (1985) 31.
KHATRI, M.N., [1] Triangular numbers and Pythagorean triangles, Scripta Math. 21 (1955) 94.
KHINCHIN,'A.Ya., [1] Three Pearls of Number Theory (Rochester 1952)
KILLGROVE, R.B. v RALSTON, K.E., [1] On a conjecture concerning primes,
Math. Tables Aids Comp. 13 (1959) 121-122.
n

KLEE, V.L. Jr., [1] On the equation x 2m , Amer. Math. Monthly 53 (1946) 327-328.

[2] Some remarks on Euler's totient, Amer. Math. Monthly 54 (1947) 332.
[3] A generalization of Euler's function, Amer. Math. Monthly 55 (1948) 358-359.
KNDEL, W., [1] Carmichaelsche Zahlen, Math. Nachr. 9 (1953) 343-350.

312 | TI LIU THAM KHO

KOGBETLIANZ, E v KRIKORIAN, A., [1 ] Handbook of First Complex Prime Numbers, Part 2, Tables of
Decompositions of Real Primes of Type 4 N 1 1 into Sums of two Squares (London-New York-Paris 1971).
KOCHAO, [1] Note on the Diophantine equation x y z , J. Chinese Math. Soc. 2 (1940) 205-207.
[2] Remark on Pythagorean numbers (ting Trung Quc), Acta Sc. Nat. Univ. Szechuan, 1958,73-80.
[3] On a conjecture of Jemanowicz (ting Trung Quc), Acta Sc. Nat. Univ. Szechuan, 1958, 81-90.
x

[4] On a Diophantine equation a b


2

2ab a

2 x

b2 (ting Trung Quc),


z

Acta Sc. Nat. Univ. Szechuan, 1959, 25-34.


KOLESNIK, G., [1] On the method of exponent pairs, Acta Arith. 45 (1985) 115-143.
KOREC, I., [1] Nonexistence of small perfect rational cuboid, II Acta Math. Univ. Comen. 44-45 (1984) 39-48.
2
n
KORHONEN, O., [1] On the diophantine equation Ax 2 B y ,
Acta Univ. Duluensis,Ser. A, Math. No 17 (1979).
KRATCHIK. M., [1] Thorie des nombres II (Paris 1926).
[2] Recherches sur la Thorie des Nombres II, Factorisation (Paris 1929).
[3] Thorie des nombres III. Analyse diophantienne et applications aux cuboides rationnels
(Paris 1947).
[4] Introduction la Thorie des Nombres (Paris 1952).
KRISHNAWAMI, AA, [1] On isoperimetrical Pythagorean triangles, Tohoku Math. J. 27 (1926) 332-348.
KULIK, J.PH., POLETTI, L.et PORTER, R.J., [1 ] Liste des nombres premiers du onzime million (plus
prcisment de 100006741 10999997 ) d'aprs des tables manuscrites (Amsterdam. 1951)
KULIKOWSKI, T., [1] Sur l'existence d'une sphre passant par un nombre donn de points aux coordonnes
entires, Enseignement Math. (2) 5 (1959) 89-90.
LAGARIAS, J.C, MILLER, V.S. v ODLYZKO A.M., [1] Computing x : The Meissel-Lehmer method, Math.
Comp. 44 (1985) 537-560.
LAGRANGE, J., [1] Sets of n squares of which any n 1 have their sum square,
Math. Comp. 41 (1983) 675-681.

LAL, M. v GILLARD, P., [1] On the equation n n k , Math. Comp. 25 (1972) 579-583.
LAL, M., RUSSELL, W. v BLUNDON, W.J., [1] A note on sums of four cubes, Math. Comp. 23 (1969) 423-424.
LAM, G., [1] Note sur la limite du nombre des divisions dans la recherche du plus grand commun diviseur
entre deux nombres entiers, C.R. Acad. Sci. Paris 19 (1844) 867-870.
LANDAU, E., [1] ber die Einteilung der positiven ganzen Zahlen in vier Klassen nach der Mindestzahl der
zu ihrer additiven Zusammensetzung erforderlichen Quadrate, Arch. Math. Phys. (3) 13 (1908) 305-312.
[2] Vorlesungen uber Zahlentheorie, 3 vols (Leipzig 1927, reprint New York 1947).
[3] Handbuch der Lehre von der Verteilung der Primzahlen, 2 vols, 2nd ed. with an
Ph lc bi P.T. Bateman (New York 1953)
LANDER, L.J. v PARKIN, T.R., [1] Equal sums of biquadrates,
Math. Comp. 20 (1966) 450-451; Corrigendum, ibid. 21 (1967) 296.
[2] A counterexample to Euler's sum of powers conjecture, Math. Comp. 21 (1967)101-103.
[3] On first appearance of prime difference, Math. Comp. 21 (1967) 483-488.
[4] Consecutive primes in anthmetic progression, Math. Comp. 21 (1967) 489.
LANDER, L.J., PARKI n, T.R. v SELFRIDGE, J.L., [1] A survey of equal sums of like powers,
Math. Comp. 21 (1967) 446-459.
LANGEVIN, M., [1] Quelques applications de nouveaux rsultats de van der Poorten, Sminaire DelangePhot-Poitou 17 (1975/76) No 12, 1-11.
LEBESGUE, H., [1] Sur certaines dmonstrations d'existence, Bull. Soc. Math. France 45 (1917) 132-144.
LEBESGUE, V.A., [1] Sur l'impossibilit en nombres entiers de lquation x y 1 .
Nouv. Ann. Math. 9 (1850) 178-181.
[2] Note sur quelques quations indtermines, Nouv. Ann. Math. (2) 8 (1869) 452-456,559.
LEE, E.J., MADACHY, J.S, [1] The history and discovery of amicable numbers, J. Recreational Math. 5 (1972)
77-93,155-173; 231-249, Errata, ibid. 6 (1973) 164,229.
LEECH, J., [1] Note on the distribution of prime numbers, J. London Math. Soc. 32 (1957) 56-58.
[2] The rational cuboid revisited, Amer. Math. Monthly 84 (1977) 518-533,
cc chnh sa rong ti lieu dn 85 (1978) 473.
LEGENDRE, A.M., [1] Essai sur la thorie des nombres (Paris 1798).
LEHMER, D.H., [1] On Euler's totient function, Bull. Amer. Math. Soc. 38 (1932) 745-751.
[2] On Lucas's test for the primality of Mersenne's numbers,
m

TI LIU THAM KHO | 313

J. London Math. Soc. 10 (1935) 162-165.


[3] On the converse on Fermat's theorem, Amer. Math. Monthly 43 (1936) 347-354.
[4] On the partition of numbers into squares, Amer. Math. Monthly 55 (1948) 476-481.
[5] On a conjecture of Krishnaswami, Bull. Amer. Math. Soc. 54 (1948) 1185-1190.
[6] On the converse of Fermat's theorem II, Amer. Math. Monthly 56 (1949) 300-309.
3
3
3
[7] On the Diophantine equation x y z 1 , J. London Math. Soc. 31 (1956) 275-282.
[8] On the exact number of primes less than a given limit, Illinois J. Math. 3 (1959) 381-388.
[9] On Fermat's quotient, base 2, Math. Comp. 36 (1981) 289-290.
LEHMER, D.N., [1] Factor Table for the First Ten Millions Containing the Smallest Factor of Every Number not
Divisible by 2, 3, 5 or 7 Between the Limits 0 and 10017000 (Washington 1909, in li ti New York 1956).

a p 1 1
q a , Math. Ann. 60 (1905) 471-490.
LERCH, M., [1] Zur Theorie der Fermatschen Quotienten
p
LESZCZYNSKI, B., [1] On the equation n x n 1 n 2 (in Polish), Wiadom. Mat. 3 (1959-60) 37-39.
y

LE VEQUE, W.J., [1] The distribution of values of multiplicative functions,


Michigan Math. J. 2 (1953-54) 179-192.
[2] Topics in Number Theory, 2 vols. (Reading 1956).
LIETZMANN, W., [1] Lustiges und merkwiirdiges von Zahlen und Formen (Gottingen 1930).
LIGHT, W.A., FORREST, J., HAMMOND, N. v ROE, S., [1] A note on Goldbach's conjecture, Nordisk Tidskr.
Informationsbehandling (BIT) 20 (1980) 525.
LIND, C.E., [1] Untersuchungeniiber die rationalen Punkte derebenen kubischenKurven von Geschlecht Eins,
Diss. (Uppsala 1940)
LINDENBAUM, A, [1] Sur les ensembles dans lesquels toutes les quations d'une famille donne ont un
nombre de solutions fixe d'advance, Fund. Math. 20 (1933) 1-29.
LINNIK, Yu.V., [1] On the representation of large numbers as sums of seven cubes,
Mat. Sb.N.S. 12 (1943) 220-224.
[2] An elementary solution of the problem of Waring by the Schnirelman method (ting Nga),
Mat. Sb. N. S. 12 (1943) 225-230.
LIOUVILLE. J., [1 ] Sur l'quation 1 2 3.... p 1 1 p , J. Math. Pures Appl. (2) 1 (1856) 351-352.
m

LITVER, E. L., YUDINA, G.E., [1] Primitive roots for the first million primes and their powers (ting Nga),
Matematiceskij analiz i ego primienienija, vol. 3, 106-109 (Rostov on Don 1971).
2
4
LIUNGGREN,W., [1] ZurTheoriederGleichung x 1 Dy . Avh.Norske Vid.Akad.Oslo I, 1942, no 5.
[2] ber einige Arcustangensgleichungen die auf interessante unbestimmte Gleichun-gen fhren,
Ark. Mat. Astr. Fys. 39 A no 13 (1943).
[3] On the Diophantine equation x p y
Norske Vid. Selsk. Forh. (Trondheim) 16 (1943) 27-30.
[4] Solution complte de quelques quations du sixime degr deux indtermines,
Arch-Math. Naturvid. 48 (1946) 177-212.
[5] New solution of a problem proposed by E. Lucas, Norsk Mat. Tidsskr. 34 (1952) 65-72.
LOCHS, G., [1] Die ersten 968 Kettenbruchnenner von , Monatsh. Math. 67 (1963) 311-316.
2

LONDON, H. v FINKELSTEIN, R., [1] OnMordeWs Equationy y k x (Bowling Green, Ohio 1973)
LUCAS, E., [1] Question 1180, Nouv. Ann. Math. (2) 14 (1875) 336.
[2] Thorie des nombres, Vol. I (Paris 1891, reprint Paris 1961).
2

LU WEN-TWAN, [1] On Pythagorean numbers 4n 1, 4n 1 (ting Trung Quc),


Acta Sc. Nat. Univ. Szechuan 1959, 39-42.
2

MAHLER, K., [1] On the fractional parts of the powers of a rational number (II),
Mathematika 4 (1957) 122-124.
MAIER, H. v POMERANCE, C, [1] On the number of distinct values of Euler's ( function), Acta Arith. 49.
MAKNIS, M., [1] Density theorems for Mecke Z-functions and the distribution of the prime numbers of an
imaginary quadratic field (ting Nga), Litovsk. Mat. Sb. 16 (1976) no 1, 173-180.
MARGENSTERN, M., [1] Rsultats et conjectures sur les nombres pratiques, C.R. Acad. Sci. Paris, Sr. I Math.
299 (1984) 895-898.
MASAI, P. v VALETTE, A. [1] A lower bound for a counterexample to Carmichaers conjecture, Boll. Unione
Mat. Ital. (6) Al (1982) 313-316.
MASON, Th.E., [1] On amicable numbers and their generalizations, Amer. Math. Monthly 28 (1921) 195-200.

314 | TI LIU THAM KHO

MAYAH, B.H., [1] The second Goldbach conjecture revisited, Nordisk Tidskr. Informationsbehandling (BIT) 8
(1968) 128- 133.
MAKOWSKI, A., [1] Sur quelques problemes concemant lessommes de quatre cubes,
Acta Arith. 5 (1959) 121-123.
[2] Remark on a paper of Erds and Turn, J. London Math. Soc. 34 (1959) 480.
[3] On an arithmetic function (in Polish), Matematyka 10 (1959) 145-147.
[4] On some equations involving functions n and n , Amer. Math. Monthly 67 (1960),
pp. 668-670; Correction, ibidem 68 (1961) 650.

[5] Remarques sur les fonctions n , n et n , Mathesis 69 (1960) 302-303.


[6] Three consecutive integers cannot be powers, Colloq. Math. 9 (1962) 297.
[7] Generalization of Morrow's D numbers, Simon Stevin 36 (1962) 71.
[8] Remarques sur les carrs magiques, Mathesis 70 (1962) 17-19.
[9] Some equations involving the sum of divisors, Elem. Math. 34 (1979) 82.
McCURLEY, K. S., [1] An effective seven cube theorem, J. Number Theory 19 (1984) 176-183.
MELNIKOV, I.G., [1] La dcouverte des "nombres commodes" par Euler (ting Nga) 1st.
Mat. Issled. 13 (1960) 187-216.
MEYL, A, [1] Question 1194, Nouv. Ann. Math. (2) 17 (1878) 464-467.

MIENTKA, W.E. v VOGT, R.L. [1 ] Computational results relating to problems concerning n ,


Mat. Vestnik 7 (1970) 35-36.
MOESSNEr, A., [1] A magic square of triangular numbers, Math. Student 10 (1942-43)95.
[2] Magic squares, Math. Student 19 (1951) 124-126.
[3] All-prime magic squares, SCRIPTA MATH. 18 (1953) 303.
2
3
MORDELL, L.J., [l]The Diophantine equation y k x ,Proc. London Math. Soc. (2)13 (1913) 60-80.
[2] Note on the integer solutions of the equation Ey Ax Bx Cx D ,
Messenger of Math. 51 (1922) 169-171.
[3] On the four integer cubes problem, J. London Math. Soc. 11 (1936) 208-218,
Addendum, ti liu dn 12 (1937) 80, and Corrigendum, ibidem 32 (1957) 383.
[4] On sums of three cubes, J. London Math. Soc. 17 (1942) 139-144.
2

[5] On the integer solutions of the equation x y z 2 xyz n ,


J. London Math. Soc. 28 (1953) 500-510 and Corrigendum, ibidem 32 (1957) 383.
[6] On intervals containing an affinely equivalent set of n integers mod k ,
Proc. Amer. Math. Soc. 5 (1954) 854-859.
[7] On the representation of a number as a sum of three squares,
Rev. Math. Pures Appl. 3 (1958)25-27.
MORET-BLANC, [1] Question 1175, Nouv. Ann. Math. (2) 15 (1876) 44-46.
MORROW, D.C., [1] Some properties of D numbers, Amer. Math. Monthly 58 (1951) 329-330.
Moser, L.,[l] Some equations involving Euler's totient function, Amer. Math. Monthly 56 (1949) 22-23.
2

[2] On the Diophantine equation 1n 2n ... m 1 mn Scripta Math. 19 (1953) 84-88.


n

[3] On non-averaging sets of integers, Canadian J. Math. 5 (1953) 245-252.


[4] On the theorems of Wilson and Fermat, Scripta Math. 22 (1957) 288.
MLLER, M., [1] ber die Approximation reeller Zahlen durch die Nherungsbrche ihres regelmassigen
Kettenbruches, Arch. Math. 6 (1955) 253-258.
NAGELL, T., [1] Zur Arithmetik der Polynome, Abh. Math. Sem. Univ. Hamburg 1 (1922)184-188.
[2] Sur l'impossibilit de quelques quations deux indtermines.
Norsk Mat. Forenings Skrifter 1 Nr 13 (1923).
[3] Einige Gleichungen von der Form ay by c dx ,
Norske Vid. Akad. Skrifter, Oslo I, 1930, no 7.
[4] Solved problems (in Norwegian) Norsk Mat. Tidsskr. 30 (1948) 60-64.
[5] Introduction to the Number Theory (New York and Stockholm 1951, reprint 1964).
[6] Sur un thorme d'Axel Thue, Ark. Mat. 1 (1951) 489-496.
[7] On a special class of Diophantine equations of the second degree, Ark. Mat. 3 (1954) 51-65.
[8] Verallgemeinerung eines Fermatschen Satzes, Arch. Math. 5 (1954) 153-159.
[9] Contributions to the theory of a category of Diophantine equations of the second degree
with two unknowns, Nova Acta Soc. Sci. Upsal. (4) 16 (1954) no 2.
2

TI LIU THAM KHO | 315

[10] On the Diophantine equation x 8D y , Ark. Mat. 3 (1954) 103-112.


[11] Sur une classe d'quations exponentielles, Ark. Mat. 3 (1958) 569-582.
2

[12] The diophantine equation x 7 2 , Ark. Mat. 4 (1961) 182-185.


NIEWIADOMSKL R., [1] Question 4202, Intermdiate Math. 20 (1913) 78.
2

OBLTH, R., [1] Une proprit des puissances parfaites, Mathesis 65 (1956) 356-364.
O'KEEFE, E.S., [1 ] Verification of a conjecture of Th. Skolem., Math. Scand. 9 (1961) 80- 82.
ORE, O., [1] Number Theory and its History (New York 1948)
PALL, G., [1] On sums of squares, Amer. Math. Monthly 40 (1933) 10-18.
Patz, W., [1] Tafel der regelmssigen Kettenbriche und Over vollstndigen Quotienten fiir die
Quadratwurzeln aus den natiirlichen Zahlen von 1-10000 (Berlin 1955)
PAWLAK, Z. v WAKULICZ, A., [1] Use of expansions with a negative basis in the
arithmometer of a digital computer, Bull, Acad. Polon. Sci., CI. Ill, 5 (1957) 233-236.
PEANO, G., [1] Formulaire de Mathmatique (Torino 1901)
PEPIN, T., [1] Sur certains nombres complexes de la forme a b c ,
J. Math. Pures Appl. (3) 1 (1875) 317-372.
PERRON, O., [1] Die Lehre von den Kettenbrchen I (Stuttgart 1954).
Piccard, S., [1] Sur les ensembles de distances des ensembles de points d'un espace euclidien (Paris 1939)
PILLAL S.S., [1] On some empirical theorem of Scherk, J. Indian Math. Soc. 17 (1927- 28) 164-171.
[2] On some functions connected with

n , Bull. Amer. Math. Soc. 35 (1929) 832-836.

[3] On Waring's problem II, J. Indian Math. Soc. (N.S) 2 (1936) 16-44.
[4] On m consecutive integers I, Proc. Indian Acad. Sci., Sect. A 12 (1940) 6-12.
[5] On m consecutive integers III. Proc. Indian Acad. Sci., Sect. A 13 (1941) 530-533.
[6] On m consecutive integers IV, Bull. Calcutta Math. Soc. 36 (1944) 99-101.
[7] On the smallest primitive root of a prime, J, Indian Math. Soc. (N. S) 8 (1944) 14-17.
[8] On the equation 2 3 2 3 , Bull. Calcutta Math. Soc. 37 (1945) 15-20.
PIPPING, N, [1] Neue Tafeln fur das Goldbachsche Gesetz nebst Berichtigungen zu den
Haussnerschen Tafeln, Comment. Phys. Math. 4 (1027-29) no 4.
[2] ber Goldbachsche Spaltungen grosser Zahlen, Comment. Phys. Math.4 (1927-29) no 10.
POCKLINGTON, H.C., [1] Some diophantine impossibilities, Proc. Cambridge Philos. Soc. 17 (1914) 108-121.
PODSYPANIN, V.D., [1] On a property of Pythagorean numbers (ting Nga),
Izv. Vyssh. Uchebn. Zaved. Matematika 1962, no 4 (29) 130-133.
x

van der POL, B., v SPEZIALI, P., [1] The primes in k

, Indag. Math. 13 (1951) 9-15.

de POLIGNAC, A., [1] Six propositions arithmologiques dduites du crible d'Eratosthne,


Nouv. Ann. Math. 8 (1849) 423-429.
POLLOCK, F., [1] On the extension of the principle of Fermat's theorem of the polygonal numbers to the
higher orders of series whose ultimate differences are constant. With a new theorem proposed, applicable
to all the orders, Proc. Roy. Soc. London 5 (1851) 922-924.
POLLACK, R.M., v SHAPIRO, H.N, [1] The next to the last case of a factorial diophantine equation,
Comm. Pure Appl. Math. 26 (1973) 313-325.
Polya G. v SzegSG., [1] Aufgaben und Lehrsatze aus der Analysis, Bd II (Berlin 1925).
POMERANCE, C, [1] On the congruences

n a mod n and n a mod n ,

Acta Arith. 26 (1975) 265-272.


[2] On the distribution of amicable numbers II, J. Reine Angew. Math. 325 (1981) 183-188.
[3] On the distribution of pseudoprimes, Math. Comp. 37 (1981) 587-593.
[4] A new lower bound for the pseudoprime counting function, Illinois J. Math. 26 (1982) 4-9.
9
POMERANCE,C.,SELFRIDGE,J.L.v WAGSTAFF,S.S.Jr, [1] The pseudoprimes to 25 10 ,
Math. Comp. 35 (1980) 1003-1026.
PORGES, A., [1] A set of eight numbers, Amer. Math. Monthly 52 (1945) 379-382.
POSTNIKOV, M.M., [1] Magicheskie kvadraty (MagicSquares, ting Nga) (Moscow 1964).
POULET, P., [1] La chasse aux nombres, Fasc. 1 (Bruxelles 1929)
[2] Table de nombres composes vrifiant le thorme de Fermat pour le
module 2 jusqu' 100000000 , Sphinx 8 (1938) 42-52.
[3] Suites de totalics en dpart de n 2000 , Hectographed copy in possession of

316 | TI LIU THAM KHO

D.H. Lehmer, cf. Math. Tables Aids Comp. 3 (1948) 120.


PRACHAR, K., [1] Primzahlverteilung (Berlin-Gttingen-Heidelberg 1957, reprint 1978).
PRITCHARD, P.A., [1] Long arithmetic progressions of primes: some old, some new,
Math. Comp. 45 (1985) 263-267.
RADO, R., [1] Some solved and unsolved problems in the theory ofnumbers, Math. Gaz. 25 (1941) 72-77.
RAMANUJAN, S., [1] Problem 465, J. Indian Math. Soc. 5 (1913) 120.
[2] On the expression of a number in the form ax by cz du ,
Proc. Cambridge Philos. Soc. 19 (1917) 11-21.
RANKIN, R.A, [1] Sets of integers containing no more than a given number of terms in arithmetical
progression, Proc. Royal Soc. Edinburgh Sect. A 65 (1960/61) 318-331.
2

REICHARDT, H [1] ber die Diophantische Gleichung ax bx y cy ez ,


Math.Ann. 117 (1940) 235-276.
RICCI, G., [1] Sull'andamento della differenza di numeri primi consecutivi,
Riv. Mat. Univ. Parma 5 (1954) 3-54.
4

[2] Sull'insieme dei valori di condensazione de rapporto pn _1 pn / ln pn n 1, 2,3,,...

Riv. Mat. Univ. Parma 6 (1955) 353-361.


RICHERT, H.-E., [1] ber Zerlegungen in paarweise verschiedene Zahlen,
Norsk Mat. Tidssk. 31 (1949) 120-122.
[2] ber Zerfllungen in ungleiche Primzahlen, Math. Z. 52 (1950) 342-343. te RIELE, HJ.J., [1]
New very large amicable pairs, in: Number Theory, Noordwijkerhout 1983, (ed H. Jager)
Lecture Notes in Mathematics 1068 (Berlin-Heidelberg-NewYork-Tokyo 1984).
10

[2] Computation of All Amicable Pairs Below 10 . Report NM-R8503,


Centrum voor Wiskunde en Informatica (Amsterdam 1985).
RIESEl, H. v Vaughan, R.C., [1] On sums of primes, Arkiv. Mat. 21 (1983) 45-74.
ROBINSON, R.M., [1] Mersenne and Fermat numbers, Proc. Amer. Math. Soc. 5 (1954) 842-846.
n
[2] A report on primes of the form k 2 1 and on factors of Fermat numbers,
Proc. Amer. Math. Soc. 9 (1958) 673-681.
de ROCQUIGNY, G., [1] Question 1408, Intermdiate Math. 5 (1898) 268.
RHR [1] Zeitschrift Math. NaturW. Unterricht 50 (1919) 95-96.
ROSE, K., v BRUDNO, S., [1] More about four biquadrates equal one biquadrate,
Math, Comp. 27 (1973) 491-494.
ROSSER, J.B., [1] The n th prime in greater than n log n , Proc. London Math. Soc. (2) 45 (1939) 21-44.
ROSSER, J.B., v SCHOENFELD, L., [1] Approximate formulas for some function of prime numbers,
Illinois J. Math. 6 (1962) 64-89.

[2] Sharper bounds for the Chebyshev functions x and x Math. Comp. 29 (1975) 243-269.

ROTA, G.C., [1] The number of partitions ofa set, Amer. Math. Monthly 71 (1964)498-504.
ROTH, K.F., [1] On certain sets of integers, J. London Math. Soc. 28 (1953) 104-109.
n 1
n 1
ROTKIEWICZ, A., [1] Sur les nombres composs n qui divisent a b .
Rend. Circ. Mat. Palermo (2) 8 (1959) 115-116.
[2] Sur les nombres pairs n pour lesquels les nombres a b ab , respectivement a
sont divisible par n , Rend. Circ. Mat. Palermo (2) 9 (1959) 341-342.
n

n 1

bn1 ,

[3] On the properties of the expression a b (in Polish), Prace Mat. 6 (1961) 1-20.
[4] Demonstration arithmtique d'existence d'une infinit de nombres premiers
de la forme nk 1 , Enseignement Math. (2) 7 (1962) 277-280.
[5] Sur les nombres pseudopremiers de la forme ax b ,
C.R. Acad. Sci. Paris 257 (1963) 2601-2604.
[6] On the pseudoprimes of the form ax b , Proc. Cambridge Philos. Soc. 63 (1967) 389-392.
[7] Un problme sur les nombres pseudopremiers, Indag. Math. 34 (1972) 86-91.
[8] Pseudoprime Numbers and Their Generalizations (Novi Sad 1972).
n

SALEM, R. v SPENCER, D.C, [1] On sets of integers which contain no three terms
in arithmetical progression, Proc. Nat. Acad. U.S.A., 28 (1942) 561-563.
[2] On sets of integers which do not contain a given number of terms in arithmetical progression,
Nieuw Arch. Wisk. (2) 23 (1952) 133-143.

TI LIU THAM KHO | 317

SALZER, H., [1] On numbers expressible as the sum of four tetrahedral numbers,
J. London Math. Soc. 20 (1945) 3-4.
SALZER, H. v LEVINE, NJ., [1] Tables of integers not exceeding 10000000 that are not expressible as the
sum of four tetrahedral numbers, Math. Tables Aids Comp. 12 (1958) 141-144.
SANSONE, G. v CASSELS, J.W.S., [1] Sur le problme de M. Werner Mnich, Acta Arith. 7 (1962)187-190.
SARDI, S. [1] Sulle somme dei divisori dei numeri, Giorn. Mat. Battaglini 7 (1869) 112-116.
SATHE, L.G., [1] On a problem of Hardy on the distribution of integers having a given number of prime
factors, J. Indian Math. Soc. N.S. 17 (1953) 63-141,18 (1954) 27-81.
SCAROWSKY, M. v BOYARSKY, A., [1] A note on the diophantine equation x y z 3 ,
Math. Comp. 41 (1984) 235-237.
SCHERK, H.F., [1] Bemerkungen ber die Bildung der Primzahlen aus einander, J. Relne Angew.
Math. 10 (1833) 201-208.
SCHINZEL, A., [1] Sur la dcomposition des nombres naturels en somme de nombres triangulares distincts,
Bull. Acad. Polon. Sci. CI. Ill, 2 (1954) 409-410.
[2] Sur une proprit du nombre de diviseurs, Publ. Math. Debrecen 2 (1954) 261-262.
n

[3] Generalization of a theorem of B.S.K.R. Somayajulu on the Euler's function n ,


Ganita 5 (1954) 123-128.
k
[4] On the equation x1 x2 ... xn t Bull. Acad. Polon. Sci. CI. Ill, 3 (1955) 17-19.

[5] Sur un probleme concernant la fonction , Czechoslovak Math. J. 6(1956) 164-165.


[6] Sur l'quation x m , Elem. Math. 11 (1956) 75-78.
[7] Sur l'existence d'un cercle passant par un nombre donn de points aux coordonnes entires,
Enseignement Math. (2) 4 (1958) 71-72.
[8] Sur l'quation x k x , Acta Arith. 4 (1958) 181-184.

[9] Sur les nombres.composes nqui divisent a a , Rend. Circ. Mat. Palermo (2) 7 (1958) 1-5.
[10] Sur les sommesde trois carrs, Bull. Acad. Polon, Sci. Sr. Sci. Math. Astr. Phys.7 (1959)307-309.
[11] Sur une consequence de l'hypothse de Goldbach,
Bulgar. Akad. N'auk. Izv. Mat. Inst. 4 (1959) 35-38.
n

[12] Sur l'quation diophantienne

A x

k
k k

k 1

0 (in Polish), Prace Mat. 4 (1960) 45-49.

[13] Remarks on the paper "Sur certaines hypothses concernant les nombres premiers",
Acta Arith. 7 (1961)'1-8.

[14] On the composite integers of the form c ak b ! 1 , Nordisk Mat. Tidskr. 10 (1962) 8-10.
SCHINZEL, A. v SIERPINSKI, W., [1] Sur quelques proprits des fonctions n et n ,
Bull. Acad. Polon. Sci. CI. Ill, 2 (1954) 463-465.
[2] Sur les sommes de quatre cubes, Acta Arith. 4 (1958) 20-30.
[3] Sur certaines hypothses concernant les nombres premiers, Acta Arith. 4 (1958),
185-208, and Corrigendum, ibidem 5 (1960) 259.

[4] Sur les congruences x c mod m et a b mod p Collect. Math. 11 (1959) 153-164.
x

SCHINZEL, A. v TIJDEMAN, R., [1] On the equation y P x Acta Arith. 31 (1976) 199-204.
m

SCHINZEL, A. et WAKULICZ, A., [1] Sur lquation x k x II,Acta Arith.5 (1959) 425-426.
Schmidt, W.M., [1] ber die Normalitat von Zahlen zu verschiedenen Basen, Acta Arith. 7 (1962) 299-309.
SCHNIRELMAN, L., [1] ber additive Eigenschaften von Zahlen, Math. Ann. 107 (1933) 649-690.
SCHOENBERG, I. J., [1] ber asymptotische Verteilung reeller Zahlen mod 1, Math. Z. 28 (1928) 171-200.
SCHOLOMITI, N.C., [1] An expression for the Euler -function, Amer. Math, Monthly 61 (1954) 36-37.
SCHOLZ, A. v SCHOENBERG, B., [1] Einfhrung in die Zahlentheorie (Berlin 1955).
SCHUR, I, [1] ber die Kongruenz x y z
m

mod p .

Jber. Deutsch. Math. Verein. 25 (1916) Abt. 1, 114-117.


[2] Einige Stze ber Primzahlen mit Anwendung auf Irreduzibilittsfragen, S-.B.
Preuss. Akad. Whs. Phys. Math. Kl. 23 (1929) 1-24.

SEGAL, S.L., [1] On x y x y Trans. Amer. Math. Soc. 104 (1962) 523-527.

318 | TI LIU THAM KHO

SEGRE, B., [1] A note on arithmetical properties of cubic surfaces, J. London Math. Soc. 18 (1943) 24-31.
SELBERG, A., [1] An elementary proof of the prime-number theorem, Ann. of Math. (2) 50 (1949) 305-313.
[2] Note on a paper by L.G. Sathe, J. Indian Math. Soc, (N.S.) 18 (1953) 83-87.
Selmer, E.S., [1] The Diophantine equation ax by cz 0 , Acta Math. 85 (1951) 203-362.
3

[2] The Diophantine equation ax by cz 0 . Completion of the tables,


Acta Math. 92 (1954) 191-197.
3

2
3
[3] The rational solutions of the Diophantine equation D for D ,

Math. Scand. 4 (1956) 281-286.


SELMER, E.S. v NESHEIM, G., [1] Tafel der Zwillingsprimzahlen bis 200000, Norske Vid.
Selsk. Fork (Trondheim) 15 (1942) 95-98.
SEXTON, C.R., [1] Counts of twin primes less than 100000 , Math. Tables Aids Comp. 8 (1954) 47-49.
[2] Computo del numero delle coppie di numeri primi gemelli comprese fra 100000 et 1100000,
distinte secondo ciffre terminali, Boll. Un. Mat. Ital. (3) 10 (1955) 99-101.
[3] Abzhlung der Vierlingen von 1000000 bis 2000000,
Am. sterr. Akad. Wiss. Math.-Nat. Kl. 92 (1955) 236-239.
SHANKS, D. [1] A note on Gaussian twin primes, Math. Comp. 14 (1960) 201-203.
SHANKS, D. v WRENCH, W.J., Jr., [1] Calculation of to 100000 decimals, Math. Comp. 16 (1962) 76-99.
[2] Calculation of e to 100000 decimals, deposited in the UMT file, cf. Math. Comp. 23 (1969) 679.
SHOREY, T.N., v TUDEMAN, R. [1] New applications of Diophantine approximations to Diophantine
equations, Math. Scand. 39 (1967) 5-18.
SIERPISINSKI, W., [1] Sur un problne du calcul des fonctions asymptotiques (ting Ba Lan),
Prace Mat. Fiz. 17 (1906) 77-118.
[2] Sur les rapports entre les proprits fondamentales du symbole de Legendre (ting Ba Lan),
C.R. Soc. Sci. Lettr. Varsovie 2 (1909) 260-273.
[3] Sur quelques algorithmes pour dvelopper les nombres rels en sries (ting Ba Lan),
C.R. Soc. Sci. Lettr. Varsovie 4 (1911) 56-77.
[4] Sur un algorithme pour dvelopper les nombres rels en sries rapidement convergentes,
Bull. Acad. Sci. Cracovie, CI. Sci. Math. Se'rie A, 1911, 113-117.
[5] Dmonstration lementaire d'un thorme de M. Borel sur les nombres absolument
normaux et dtermination effective d'un tel nombre, Bull. Soc. Math. France 45 (1917) 125-132.

[6] Remarque sur une hypothse des Chinois concernant les nombres 2n 2 / n ,
Colloq. Math. 1 (1947) 9.
[7] Dzialania nieslconczone (Infinite Operations, in Polish) (Warszawa-Wroclaw 1948).
[8] Remarques sur la dcomposition des nombres en sommes des carrs de nombres impairs,
Colloq. Math. 2 (1949) 52-53.
[9] Contribution l'tude des restes cubiques, Ann. Soc. Polon. Math. 22 (1949) 269-272.
[10] Un thorme sur les nombres premiers, Mathematiche (Catania) 5 (1950) 66-67.
[11] Sur les puissances du nombre 2, Ann. Soc. Polon. Math. 23 (1950) 246-251.
[12] Teoria liczb (Theory of Numbers, in Polish) Warszawa-Wroclaw 1950.
[13] Une proposition de la gomtrie lmentaire quivalente l'hypothse du continu,
C.R. Acad. Sci. Paris 252 (1951) 1046-1047.
[14] Sur une proprit des nombres premiers, Bull. Soc. Roy. Sci. Lige 21 (1952) 537-539.
[15] Remarques sur les racines d'une congruence, Ann. Polon. Math. 1 (1954) 89-90.
[16] Sur une proprit des nombres naturels, Ann. Mat. Pura Appl. (4) 39 (1955) 69-74.
x
y
z
[17] On the equation 3 4 5 (in Polish), Wiadom. Mat. (2) 1 (1955/56) 194-195.
[18] Sur une propriete de la fonction n , Publ. Math. Debrecen 4 (1956) 184-185.
[19] Sur quelques problemes concernant les points aux coordonnes entires,
Einseignement Math. (2) 4 (1958) 25-31.
[20] Sur les nombres premiers de la forme n 1 , Enseignement Math. (2) 4 (1958) 211-212.
[21] Sur les ensembles de points aux distances rationelles sirus sur le cercle,
Elem. Math. 14 (1959) 25-27.
[22] Cardinal and Ordinal Numbers (Warszawa 1959).
[23] Sur l'quivalence de deux hypothses concernant les nombres premiers,
Bulgar, Akad. Nauk. Izv. Mat. Inst. 4 (1959) 3-6.
[24] Sur les sommes gales des cubes distincts de nombres naturels,
Bulgar. Akad. Nauk. Izv. Mat. Inst. 4 (1959) 7-9.
n

TI LIU THAM KHO | 319

[25] Sur les nombres premiers ayant des chiffres initiaux et finals donns.
Acta Arith. 5 (1959) 265-266.
[26] Teoria liczb, Cze II (Theory of Numbers, Part II, in Polish) (Warszawa 1959).
[27] Sur les nombres dont la somme des diviseurs est une puissance du nombre 2.
The Golden Jubilee Commemoration Volume (1958-59) Part I, 7-9 (Calcutta 1963).
n
[28] Sur un problme concernant les nombres k 2 1 ,
Elem. Math. 15 (1960) 73-74, and Corrigendum, ibidem 17 (1962) 85.
[29] Sur les nombres impairs admettant une seule dcomposition en une somme
de deux carrs de nombres naturels premiers entre eux, Elem. Math. 16 (1961) 27-30.
[30] Sur les nombres triangulaires carrs, Bull. Soc. Roy. Sci. Liege 30 (1961) 189-194,
and Univ. Beograd. Publ. Elektrotehn. Fak. Ser. Mat. Fiz. 65 (1961) 1-4.
[31] Dmonstration lmentaire d'un thorme sur les sommes de trois
nombres premiers distincts,
GlasnikMat.-Fiz. Astronom. Drustvo Mat. Fiz. Hrvatske (2) 16(1961) 87-88.
[32] Sur une proprit des nombres triangulaires, Elem. Math. 17 (1962) 28.
[33] Sur une proprit des nombres ttradraux, Elem. Math. 17 (1962) 29-30.
[34] Sur quelques consquences d'une hypothse de MA. Schinzel,
Bull. Soc. Roy. Sci. Liege 31 (1962) 317-320.
[35] Pythagorean triangles (New York 1962).
[36] Sur une proprit des nombres naturels, Elem. Math. 19 (1964) 27-29.
SISPANOV, S, [1] On pseudo-prime numbers (in Spanish), Bol. Mat. 14 (1941) 99-106.
SKOLEM,T., [1] Unlbsbarkeit von Gleichungen deren entsprechende Kongruenzfr jeden Modul lbsbar ist,
Avh. Norske Vid. Akad. Oslo I, no 4 (1942).
[2] Diophantische Gleichungen (Berlin 1938, reprint New York 1950)
[3] On certain distributions of integers in pairs with given differences, Math. Scand. 5 (1957)57-68.
SRINIVASAN, A.K., [1] Practical numbers, Current Sci. 17 (1948) 179-180.
STARK, H.M., [1] A complete determination of the complex quadratic fields of class-number one,
Michigan Math. J. 14 (1967) 1-27.
[2] Effective estimates of solutions of some diophantine equations, Acta Arith. 24 (1973) 251-259.
STEIGER, F, [1] ber die Grundlosung der Gleichung a b c d , Elem. Math. 11 (1956) 105-108,
STEIN, M.L. v STEIN, P.R. [1] New experimental results on the Goldbach conjecture,
Math. Mag. 38 (1965) 72-80.
STEINHAUS, H., [1] Problem 498 (in Polish), Matematyka 10 (1957) No 2, 58.
STEINIG, J., [1] On Euler's idoneal numbers, Elem. Math. 21 (1966) 73-88.
STEMMLER, R.M., [1] The ideal Waring theorem for exponents 401 200000 ,
Math. Comp. 18 (1964)144-146.
STEPHANOS, G., [1] Sur une proprit remarquable des nombres incommesurables,
Bull. Soc. Math. France 7 (1879) 81-83.
STERN, M.A., [1] ber eine der Theilung von Zahlen hnliche Untersuchung und deren Anwendung auf die
Theorie der quadratischen Reste, J. Reine Angew. Math. 61 (1863) 66-94.
2

STEUERWALD,R.,[l]EinSatz ber natrliche Zahlen mit N 3N , Arch. Math. 5 (1954) 449-451.


STEWART, B.M., [1] Sums of functions of digits, Canadian J. Math. 12 (1960) 374-389.
[2] Sums of distinct divisors, Amer. J. Math. 76 (1954) 779-785.
10

mo

mo

mo

STORCHI, E., [1] Alcuni criteri di divisibilit per i numeri di Mersenne e il carattere 6 , 12 , 24 , 48
dell'intero 2, Boll. Un. Mat. Ital. (3) 10 (1955) 363-375.
STRAUSS, E.,[1] Eine Verallgemeinerung der dekadischen Schreibeweise nebst funktionen-Theoretischer
Anwendung, Acta Math. 11 (1887) 13-18.
SUBBA RAO, K., [1] An interesting property of numbers, Math. Student 27 (1959) 57-58.
SWIFT, E., [1] Solution of the problem 213, Amer. Math. Monthly 22 (1915) 70-71.
SYLVESTER, JJ., [1] On arithmetical series, Messenger Math. 21 (1892) 1-19, 87-120.
SZELE, T., [1] Une gnralisation de la congruence de Fermat, Mat. Tidsskr. B, 1948, 57-59.
SZEMEREDI, E., [1] On sets of integers containing no k elements in arithmetic progression,
Acta Arith. 27 (1975) 199-245.
TAKADA, J., [1] Computation of cyclic parts of steinhaus problem for power 8,
Math.Seminar notes kobe univ. 7 (1959) 543-546.
TAMURA, Y. v KANADA, Y., [1] Calculation of to 4196239 decimals based on
Gauss-Legendre algorithm (preprint), cf. Canadian Math. Bull. 27 (1984) 443.

320 | TI LIU THAM KHO

TARDY, P., [ 1 ] Transformazione di un prodotto di n fattori, Ann. Sc. Mat. Fb. 2 (1851) 287-291.
TCHACALOFF, L. et KARANICOLOFF, C, [1] Rsolution de lquation Ax By z
en nombres rationnels, C.R. Acad. Sci. Paris 210 (1940) 281-283.
TEILHET, P.F., [1] Equations indterminees, Intrmediate Math. 12 (1905) 209-210.
TEUFFEL, R., [1] Beweise fur zwei Satze von H.F. Scherk ber Primzahlen,
Jber. Deutsch. Math. Verein. 58 (1955) Abt. 1, 43-44.
THUE, A., [1] Suggestions to a method in number theory (ting Nauy),
Vid. Sekk.Forhandlinger Kristiania 1902 No 7.
m

[2] ber die Unlsbarkeit der Gleichung ax bx c dy in grossen ganzen Zahlen x und y ,
Arch. Math. Naturvid. 34 (1917) No 16.
TIETZE, H., [1] Tafel der Primzahl-Zwillinge unter 300000, S.-B. Bayer.
Akad. Wiss. Math. Nat. Kl. 1947, 57-62.
TUDEMAN, R, [1] On the equation of Catalan, Acta Arith. 29 (1976) 197-207.
TROST, E., [1] Aufgabe 79, Elem. Math. 6 (1951) 18-19.
[2] Bemerkung zu einem Satz ber Mengen von Punkten mit ganzzahligen Entfernungen,
Elem. Math. 6 (1951) 59-60.
[3] Primzahlen (Basel-Stuttgart 1953, 2nd ed. 1968).
TUNNEL, J. B., [1] A classical Diophantine problem and modular forms of weight 3/2,
Invent. Math. 72 (1983) 323-334.
TURAN, P, [1 ] Results of number theory in the Soviet Union (in Hungarian), Mat. Lapok 1 (1950) 243-266.
TURSKL. S., [1] Dcomposition de nombres entiers en sommes de carres de nombres impairs,
Bull. Soc. Roy. Sci. Lige 2 (1933) 70-71.
2

UHLER, H.S., [1] Many figure approximations to 2 and distribution of digits in 2 and 1/ 2 ,
Proc. Nat. Acad. Sci. USA. 37 (1951) 63-67.
[2] A brief history of the investigations on Mersenne numbers and the latest immense primes,
Scripta Math. 18 (1952) 122-131.
[3] On the 16th and 17th perfect numbers, Scripta Math. 19 (1953) 128-131.
USPENSKY, J.V. v HEASLET, MA., [1] Elementary Number Theory (New York and London 1939)
VAHLEN, Th., [1] Beitrge zu einer additiven Zahlentheorie, J. Reine Angew. Math. 112 (1893) 1-36.
VAUGHAN, R.C., [1] On Waring's problem for smaller exponents,
Proc. London Math. Soc. (3) 52 (1986)445-463.
[2] On Waring's problem for sixth powers, J. London Math. Soc. (2) 33 (1986) 227-236.
VEHKA, T, [1] Explicit construction of an admissible set for the conjecture that sometimes

x y x y , Notices Amer. Math. Soc. 26 (1979) A-453.

VIJAYARAGHAVAN, T, [1] The general rational solution of some Diophantine equations of the form
k 1

A X
r 1

n
p

0 , Proc. Indian Acad. Sci., Sect. A, 12 (1940) 284-289. r = l

WAGSTAFF,S.S., Jr., [1] On k -free sequences of integers, Math. Comp. 26 (1972) 767-771.
[2] Greatest of the least primes in arithmetic progressions having a given modulus,
Math. Comp. 33 (1979) 1073-1080.
WAKULICZ, A., [1] On the equation x y 2 z , Colloq. Math. 5 (1957) 11-15.
WALKER, G.W., [1] Solution of the problem E 985, Amer. Math. Monthly 59 (1952) 253.
WALSH, CM, [1] Fermat's Note XIV, Ann. of Math. (2) 29 (1928) 412-432.
WARD, M., [1] A type of multiplicative diophantine systems, Amer. J. Math. 55 (1933) 67-76.
WATSON, G.L., [1] A proof of the seven-cube theorem, J. London Math. Soc. 26 (1951) 153-156.
[2] Sums of eight values of a cubic polynomial, J. London Math. Soc. 27 (1952) 217-224.
WATSON, G.N, [1] The problem of the square pyramid, Messenger Math. 48 (1918) 1-22.
WEINBERGER, P., [1] Exponents of the class groups of complex quadratic fields,
Acta Arith. 22 (1972)117-124.
WEINTRAUB, S., [1] a large prime gap, Math. Comp. 36 (1981) 279.
WERTHEIM, G., [1] Anfangsgrilnde der Zahlenlehre (Braunschweig 1902)
3

WHITEHEAD, E.G., [1] The Ramsey number N 3,3, 3, 3; 2 , Discrete Mathematics 4 (1973) 389-396.

TI LIU THAM KHO | 321

WHITTEN, S, [1] Tables of the totient and reduced totient function, Manuscript deposited in UMT file, cf.
Math. Tables Aids Comp. 4 (1950) 29-31.
WHITWORTH, W.A., [1] Choice and Chance with One Thousand Exercises (Cambridge 1901, New York 1951).
van WUNGARDEN, A., [1] A table of partitions into two squares with an application to rational triangles.
Indag. Math. 12 (1950) 313-325.
WILLEY, M., [1] Solution of the problem E 68, Amer. Math. Monthly 41 (1934) 330.
WILLIAMS, H.C. v DUBNER, H., [1] The primality of R 1031, Math. Comp. 47 (1986) 703-711.
WIRSING, E., [1] Bemerkung zu der Arbeit ber vollkommene Zahlen, Math. Ann. 137 (1959) 316-318.
WOJCIK, J., [1] On sums of three squares, Colloq. Math. 24 (1971) 117-119.
WUNDERLICH, M., [1] Certain properties of pyramidal and figurate numbers,
Math. Comp. 16 (1962) 482-486.
[2] On the Gaussian primes on the line Im X 1 , Math. Comp. 27 (1973) 399-400.

YANNEY, B.F., [1] Another definition of amicable numbers and some of their relations to Dickson's
amicables, Amer. Math. Monthly 30 (1923) 311-315.
YATES, S., [1] Sinkers of titanics, J. Recreational Math. 17 (1984/85) 268-274.
YORINAGA, M., [1] Numerical investigation of some equations involving Euler's function,
Math. J. Okayama Univ. 20 (1978) 51-58.
ZAHLEN, J.P., [1] Sur les nombres premiers une suite d'entiers conscutifs,
Euclides (Madrid) 8 (1948) 115-121.
ZAJTA, AJ., [1] Solutions of the Diophantine equation x y z t Math. Comp. 41 (1983) 635-659.
ZARANKIEWICZ, K., [1] On triangular numbers (in Polish), Matematyka 2 (1949), No 4,1-7 and No 5, 1-8.
4

Ph lc dnh cho cc chng minh


BEDOCCHI. E., [1] Nota ad una congettura sui numeri primi, Riv. Mat. Univ. Parma (4) 11 (1985) 229-236.

DANH SCH TRA CU CC NH TON HC


Abbot, H. L., 278
Aigner, A., 236
Alaoglu, L, 107, 110
Ankeny, N. C, 247
Anning, N. H, 245
Artin, E., 171
Atkin, A. O. L., 75
Avanesov, E. T., 55, 182
Bachmann, P., 218, 277, 300
Baillie, R., 153, 235
Baker, A, 65
Baker, C. L., 74
Balasubramanian, R., All, 270
Balog, A, 155
Bang, A. S., 270
Bang, T., 231
Baumert, L. D., 278
Beck, W. E., 112
Bedocchi, E., 133
Beeger, N.G.W.H. 142, 235
Behrend, F. A., 279
Bell, E. T., 44
Bendz, T. R., 66
Bernoulli, Johannes, 273
Bertrand, J., 85
Best, M. R., 52
Beyer, W. A., 61
Bieberbach, L, 275, 276
Blanusa, D., 73
Blundon, W. J., 266
Bochner, S., 13
Bohman,J., 76, 84
Borel, E., 61, 188
Borho, W., 114
Borozdkin,K. G., 77
Bouniakowsky. V. 66, 82, 83
Bourger, M., 187
Boyarsky, A., 22
Brauer, A., 5, 64
Bredihin, B. M., 81
Bremner, A, 55
Brent, R. P, 95, 234
Briggs, W. E., 141
Brillhart, J, 132, 234
Bromhead, T., 39
Brouwer, L. E. J., 187
Browkin,J. 5, 54, 153, 244,
262, 281
Brown, A. L., 112
Brown, J. L. Jr., 11,94
Brudno, S., 36
Brun, V., 75, 76
Buck, R. C, 80
Buhler, J. P., 131
Buxton, M., 111
Calloway, A., 248
Cantor, G., 189

Carmichael, R. D., 43, 143,


155, 156, 159, 165, 166, 262
Cassels.J.W.S. 50, 65, 290
Catalan, E, 107
Cattaneo, P., 112
Cauchy, A., 143
Cel, J., 48
Champernowne.G.D. 188
Chein, E. Z., 55
Chen, J. R., 76, 95, 271
Chernick, J. 143
Chikawa, K., 182
Choi, S. L. G., 123
Chojnacka Pniewska,M., 125
Chowla,S., 79, 141
Cipolla, M., 141
Clement, P. A, 73, 137
Coblyn, 137
Cohen, G. L., 112, 155
Cohen, H., 108
Colombo, M., 75
de Comberousse, C, 32
Copeland, A. H., 188
Cormack, G. V., 235
van der Corput, J. G., 76, 79,
243, 281
Coustal, R, 61
Coxe, C, 81
Cramer, H. 95
Crandall, R. E., 131
Crocker, R., 282, 283
Cunningham, A. J. C, 235
Danilov, L. V, 65
Davenport, H., 134, 271
Demyanenko, V. A., 67, 68,
267
Denes, P, 55
Depman, I. Ya., 74
Desboves, A., 76
Descartes, R., 112, 113, 255
Deshonilles, J.-M., 270, 271
Devitt, J. S., 107
Dickson, L. E., 39, 40, 77, 78,
82, 107, 114, 131, 132, 141,
271, 272
Diophantus, 21
Dirichlet, P. G. L, 79, 116,
118, 247
Dixon, J. D., 11
Dress, F., 270, 271, 272
Dressier, R., 94
Dubner, H, 73
Duparc, H. J. A., 142
Dyer Bennet, J., 173
Elmore, S., 111
Eratosthenes, 101
Estermann, T., 19, 168

Erds, P., 52, 55, 69, 75, 76,


84, 89, 98, 99, 103, 107, 108,
110, 114, 123, 136, 155, 156,
157, 245, 270, 279, 280, 281
Euclid, 112
Euler, L. 54, 64, 80, 114, 118,
127, 132, 141, 169, 171, 173,
228, 234, 253, 269, 273, 296,
297, 304
Faber, G., 189
Fauquembergue, E., 232
Fermat, P., 30, 32, 34, 36, 48,
56, 64, 66, 67, 112, 114, 128,
132, 134, 140, 144, 151, 158,
162, 166, 167, 169, 172, 173,
174, 178, 227, 230, 233, 234,
235, 236, 237, 238, 239, 245,
247, 262, 269
Finkelstein, R., 64
Finsler, P, 96, 97
Forrest, J., 76
Franqui, B., 112
Fredericksen, H., 278
Frenicle de Bessy, B., 275
Frobenius, G., 81
Frberg, C. E., 131
Fueter, R., 65
Gabard, E., 2
Gabowicz, J. A.. 267
Galgowski, J., 75
Garcia, M. 112
Gardiner, V. L., 265
Gasper, R. W., Jr, 195
Gauss, K. F., 121, 123, 134,
151, 272, 285
Gelfond, A. O., 185
Georgiev, G., 67
Gerono, C. G., 228
Gilbreath, N. L., 96
Gillard, P., 153
Gillies, D. B, 232
Gilloud, J., 187
Ginsburg, J, 24
Giuga, G., 133
Glaisher,J. W. L., 103, 186
Godwin, H. J., 265
Goldbach, Chr., 55, 76
Golomb, S.W., 75
Golubew, W.A., 75, 81, 82
Goodstein, E., 277
Gostin, G. B., 234
Graham, S., 95
Grosswald, E., 79, 141, 248,
304
Grube, F., 141
Gruenberger, F. J., 74
Gupta, H., 108, 239
Gusev, V. A., 182

324 | DANH SCH TRA CU CC NH TON HC

Hadamard, J., 99
Hagis, P. Jr, 111, 112, 154
Halcke, P., 39
Hall, M. Jr, 65, 205, 234, 281
Hallyburton, J. R., 234
Halter-Koch, F., 258
Hammond, N., 76
Hanly, V. S., 80
Hanson, D., 278
Hardy, G. H., 266
Harris, V. C, 101
Hasse, H., 171, 228

Hausdorff, F., 195

Haussner, R., 95
Heaslet, M. A., 41, 64
Heath-Brown, D. R., 103
Hecke, E., 80
Hemer, O, 64
Hensel, K., 3
Hensley, D., 100
Hilbert, D., 270
Hill, J. D., 40, 41
Hooley, C, 19
Hoffman, H., 114
Hornfeck, B., 112
Hunsucker, J. L., 107
Hurwitz, Adolf, 210, 257
Hurwitz, Alexander, 234
Hyyr, S., 50
Ibn AlBanna, 114
Ingham, A. E., 101
Irving, R.W. 279
Iseki, K., 182
Ivic, A., 243
Iwaniec, H., 81
Jacobi, C, 213, 220, 254, 296,
297, 300, 304
Jacobsthal, E., 134
Jaeschke, G., 235
Jakbczyk, F., 236
Jankowska, S., 156
Jemanowicz, L., 25
de Joncourt, E., 53
Jones, B., 247
Jordan, G, 161
Jzefiak, T., 53
Kacperek, L., 75
Kalmar, L, 89
Kanada, Y., 187
Kanold, H. J., 107, 114
Kaprekar, D. R., 181
Karanicoloff, G, 67
Keller, W., 235
Khatri, M. N., 53
Khinchin, A. Ya., 270
Killgrove, R. B., 96
Klee, V. L. Jr, 161

Kndel, W., 143


KoChao, 68
Kogbetlianz, E., 240
Kolesnik, G., 105
Korec, I., 39
Korhonen, O., 66
Kraitchik, M., 39, 72, 202
Krikorian, A., 240
Krishnawami, A. A., 31
Kulik, J. Ph., 74, 136
Kulikowski, T., 136
Kusakabe, T., 182
Lagarias, J. G, 84
Lagrange, Jean, 39
Lagrange, J. L 204, 251, 259,
270, 302
Lam, G., 10
Landau, E., 99, 204, 247, 249
Lander, L. J., 35, 36, 72, 75,
76, 94, 95, 270
Landry, F., 234
Langevin, M., 50
Lazarus, R.B., 265
Lebesgue,H., 188
Lebesgue, V. A, 63, 66
Lee, E. J., 114
Leech, J, 39, 133
Legendre, A. M., 38, 134
Lehmer, D. H., 2, 31, 141,
142, 154, 254, 265
Leibniz, G. F., 131, 141, 296
Lerch, M, 138
LeVeque, W. J., 99, 100, 278
Levine, N. J., 56
Lietzmann, W., 123
Light, W. A., 76
Lind, C. E, 49
Lindenbaum, A., 281
Linnik, Yu. V., 95, 270, 271
Liouville,J. 119, 120, 137,
160, 165
Littlewood, J. E., 133
Litver, E. L., 172
Ljunggren, W., 52, 54, 55, 66
Lochs, G, 195
London, H., 64
Lucas, E, 52, 189, 272
Lu Wen-Twan, 25
Maciag, S., 144
Madachy, J. S., 114
Mac Mahon, P., 273
Mahler, K., 271
Maknis, M., 80
von Mangoldt, H., 118
Margenstern, M., 110
Masai, P., 155
Maslowski, S., 281
Mason, Th. E, 114

Mayah, B. H., 77
Mazur, S, 110
Makowski, A., 50, 107, 112,
114, 144, 154, 169, 219, 268,
275, 276, 279, 280
McCurley, K. S., 271
Melnikov, J. G., 141
Mersenne, M., 112, 113, 114,
144
Meyl, A., 55
Mientka, W. E., 107
Miller, V. S., 84
Mirsky, L., 103
Moessner, A., 276
Mordell, L. J., 62, 63, 64, 247,
265, 267
Morehead, J. C, 234, 238
Moret-Blanc, 68
Morrison, M. J., 234
Morrow, D. C, 143
Moser, L., 52, 153, 154, 279
Mller, M, 181
Mycielski, Jan, 103
Nagell.T. 25, 52, 53, 64, 66,
126, 228
Najar, R. M., 112
Nebb, J., 107
Nickel, E., 232
Niewiadomski, R., 266
Noll, C, 232
Norrie, R., 35
Odlyzko, A.M., 84
OKeefe, E.S., 274
Pall, G., 247, 248, 255, 258,
259
Parker, T., 94
Parkin,T.R., 35, 36, 94, 95,
270
Patterson, J. O., 36
Patz, W., 204
Pawlak, Z., 182
Paxson, G. A, 234
Peano, G., 186
Pell, J., 56, 61
Penk, M. A., 131
Pepin, T., 64
Perron, O., 200, 202
Pervouchine, I. N., 234
Picard, S., 281
Pillai.S.S. 5, 51, 154, 155, 171,
271
Pipping, N., 77
Pisano.L.(Fibonacci). 10, 41
Pocklington, H. C, 48
Podsypanin, V. D., 25
van der Pol, B., 81
Poletti, L., 74

DANH SCH TRA CU CC NH TON HC | 325

de Polignac, A., 281


Pollack, R.M., 69
Pollock, F., 250
Polya, G., 163
Pomerance, C, 112, 141, 143,
154
Porges, A., 181
Postnikov, M. M., 277
Poulet, P., 108, 112, 141, 142,
144
Prachar, K., 99
Pritchard, P. A, 78
Pythagoras, 114
Rado, R., 278
Ralston, K. E., 96
Ramanujan, S, 228, 254
Rankin, R, A, 279
Reichardt, H., 49
Rnyi, A., 75
Reynolds, R. L., 19
Ricci, G, 76
Richards, I, 100
Richert, H. E, 93, 94
Rickert, N. W, 75
te Riele, H. J. J, 52, 114
Riesel, H., 77, 232, 234
Robinson, R. M, 232, 235
de Rocquigny, G, 83
Roe, S., 76
Rose, K., 36
Rosser, J. B, 94, 99
Rota, G.C. 273
Roth, K.F. 279
Rotkiewicz, A, 62, 142, 168
Rohr, 55
Russel, W., 266
Sansone, G., 290
Sardi, C, 128
Sathe, L.G., 100
Scarowsky, M., 22
Scherk, H. F., 91, 92, 93
Schinzel, A, 44, 55, 67, 94, 95,
100, 103, 107, 131, 139, 142,
171, 227, 268
Schmidt, W. M, 188
Schnirelman, L., 270
Schoenberg, B., 18
Schoenberg, I. J., 168
Schoenfeld, L., 94, 99
Scholomiti, N. C, 156
Scholz, A., 18
Schur, I., 89, 277, 278, 279,
280
Segal, S. L., 100
Segre, B, 266
Selberg, A, 99

Selfridge, J, L., 35, 36, 55, 100,


141, 232, 234
Selmer, E. S 53, 65, 75
Serret, J. A., 132, 134
Sexton, C. R, 75
Shanks, D., 107, 293
Shapiro, H.N, 69
Shibamura, K 182
Shorey, T. N, 182
Sierpinski, W 23, 25, 54, 55,
77, 80, 82, 83, 91, 94, 95,
100, 107, 110, 125, 139, 140,
141, 143, 145, 153, 154, 156,
157, 158, 166, 171, 175, 188,
189, 190, 204, 209, 213, 228,
236, 243, 244, 245, 251, 268,
279, 282
Sispanov, S., 143
Skalba, M, 46
Skolem, T., 61, 126, 274
Skula, L, 95
Slowinski, D, 232
Spencer, D. C, 279
Speziali, P, 81
Srinivasan, A. K, 110
Stark, H. M, 65, 81
Steiger, F, 258
Stein, P. R., 265
Steinhaus, H., 100, 243, 244
Steinig, J., 141
Stemmler, R. M., 271
Stephanos, C, 189
Stephens, N.M., 64
Stern, M. A., 273
Steurwald, R., 112
Stewart, B. M., 110, 181
Stifel, M., 275, 276
Storchi, E., 228
Strauss, E., 189
Subba Rao, K., 181
Swift, D., 123
Swift, E., 36
Sylvester, J. J., 89
Szeg, G, 163
Szekeres, G., 279
Szele, T., 132, 162
Szemeredi, E, 279
Takada, J., 182
Tamura, Y., 187
Tardy, P., 272
Tchacaloff, L, 67
Tchebycheff,P.L. 85, 89, 90
Tebay, S, 39
Teilhet, P. F., 54
Teuffel, R., 91
Thue, A, 64
Tietze, H 75
Tijdeman, R., 50, 55, 187

Tonascia, J., 132


Trost, E., 52, 79, 81, 84, 97,
99, 100, 229, 245
Tuckerman, B, 232
Turan, P, 76, 279, 280
Turski, S., 250
Uhler, H. S., 61, 232
Uspensky, J. V., 64
Vahlen, Th., 273
Valette, A., 155
de la Vallee Poussin, Ch., 99
Vaughan, R. C, 77, 271
Vehka, T., 100
Vijayaraghavan, T., 67
Vinogradov, I, 77
Vogt, R. L., 107
Voronoi, G., 105
Wagstaff.S.S. Jr, 95, 141, 279
Wakulicz, Andrzej, 153, 182
Wakulicz, Antoni, 50, 60
Walker, G. W, 279
Walsh, C. M, 36
Ward, M, 44
Waring, E., 270
Watson, G.L, 56
Watson, G.N., 52
Weinberger, P., 132, 141
Weintraub, S., 75
Wertheim, G., 171
Western, A. E., 234
Wheeler, D. J., 232
Whitehead, E. G, 278
Whitten, S., 165
Whitworth, W. A, 273
Wieferich, A., 270, 271
van Wijngarden, A., 240
Willey, M., 26
Williams, H. C, 73, 234, 235
Wirsing, E, 112
Wjcik, J., 247
Woodall, H. J., 235
Wrathall, C. P., 234
Wrench, W. J. Jr, 187
Wright, E. M., 266
Wunderlich, M 55, 81
Yanney, B. F, 114
Yates, S., 75
Yorigana, M., 153
Yudina, G. E., 172
Zahlen, J. P., 91
Zaita, A. J., 36
Zarankiewicz, K., 53, 157
Zeitlin, D., 80

TRA CU NHANH CC CH
Cc s
S t nhin 1
S nguyn t 71
S gi nguyn t 141
S gi nguyn t tuyt i 142
S Mersenne 112
S Pm 112
S siu-Poulet 142
S D 143
S Carmichael 143
S nguyn t Wilson 130
S nguyn t sinh i 101
B ba s nguyn t lin tip 170
S Fermat 233
S Cullen 235
S khng c c s chnh phng 19
S tam gic 23
S t din 55
S iu ha 23
S hon ho 111
S gi hon ho 112
S bn b 114
S phong ph 110
S c s (practical number) 110
S idonei 140
S chun tc 187
S chun tc tuyt i 187
S nguyn phc 285
S nguyn Gauss 285
S lin kt 286
S nguyn t phc 290
S nguyn phc nguyn t cng nhau 289
Gi thuyt
Gi thuyt Artin 171
Gi thuyt Bouniakowsky 82
Gi thuyt Carmichael 155
Gi thuyt Catalan 50
Gi thuyt Catalan-Dickson 107
Gi thuyt C 267
Gi thuyt H 143
Gi thuyt P 94
Gi thuyt Euler 269
Gi thuyt Gilbreath 96
Gi thuyt Goldbach 82
nh l
nh l c bn ca s hc 5
nh l s d Trung Hoa 17
nh l s nguyn t 99
Lut tng h bc hai 217
Lut Eisenstein 222
Lut xp x tt nht 195
nh Bertrand 85
nh l Crocker 282
nh l Erds 281
nh l Euler 161
nh l Fermat nh 128

Cc nh l khc ca Fermat 30, 32, 36, 48, 64, 67,


114, 166, 262
nh l cui cng ca Fermat 34, 262, 269
B Gauss 214
nh l Gauss 247
nh l Hurwitz 257
nh l Jacobi 254, 296, 300, 304
nh l Lagrange 170, 204, 259, 302
nh l Lam 10
nh l Leibniz 131
nh l Lejeune Dirichlet 79
nh l Lucas-Lehmer 220
nh l Lucas 331
nh l Pall 255, 258, 259
nh l Rdei 162
nh l Richert 93
nh l Scherk 91
nh l Schur 277
nh l Tchebycheff 85
nh l Thue 18
nh l Waring 270
nh l Wilson 128
Cc khi nim c bn
Tnh chia ht 1, 286
c s 1, 287
c s chung 1
c s chung ln nht 3, 287
Bi s chung 3
Bi s chung nh nht 3, 290
ng d thc 121
Nghim ca ng d thc 123
Phn tch thnh tha s 71
Biu din thp phn 186
Phn hoch dng tng 273
Ch s thp phn 179
Lin phn s 10, 191
Phn s lin tc 10, 191
Lin phn s n 12, 191
Lin phn s hi t 202
Thng d 173
Cn nguyn thy 165
Thng d bc hai 129
Ch s ca s nguyn 175
C s ca ch s 176
K hiu Jacobi 220
K hiu Legendre 213
Cn ma phng 276
Chun 285
Dy s Fibonacci 10
im nguyn 242
Hng s Euler 105
Ma phng 274
Ma phng hon ho 275
Tam gic Pythagoras 23
Tch Dirichlet 106

328 | TRA CU NHANH CC CH

Thut ton
Thut ton Euclid 10, 287
Thut ton chia 10
Sng Eratosthenes 73
Thut ton lin phn s 10
Cng thc
Cng thc Brouncker 297
Cng thc Dirichlet 104
Cng thc Euler 297, 304
Chui Lambert 105
ng thc Liouville 296
ng thc Lucas 272
Chui Pell 189
Cng thc Wallis 296
Hm s
Hm s Gauss 151
Hm s d (n) 101
Hm s Liouville n 120
Ly tha ph qut nh nht

(n) 84
Hm s (n) 106
Hm s (n) 151
Hm s

Hn ch Euler 151
Hm s Liouville 120
Hm s Mbius 117
Phng trnh
Phng trnh Diophante 21
Phng trnh Pythagoras 22
Phng trnh Fermat 56
Phng trnh Pell 56

n 173

PH LC V B SUNG (THNG 7 NM 1987)


1. Tnh ti nay (7/1987) ngi ta tm ra cp s nguyn t sinh i ln hn cp s nguyn t
26625
1 l 107570463 102250 1 . Xem bi bo ca DUBNER H. V DUBNER R
sinh i 260497545 2
c tiu : v s pht trin trong tnh ton s hc bng my tnh in t ng trn J. Recreational
Math. 18 (1985/6), 92-96.
2. Cc sai s tt nht cho bi cc cng thc c lng i vi cc hm T x trang 144 v T x
trang 337 c trnh by bi Iwaniec v Mozzocchi (xem IWANIEC, H. v MOZZOCCHI, C.J On the
7/22
divisor and circle problems. Preprint). H chng minh cc sai s khng vt qu C X
vi X bng x hoc n tng ng, l s dng ty v C l hng s ch ph thuc vo .
3. Thng tin trong trang 196 ni rng cc nh ton hc Trung Hoa t 25 th k trc cho rng
n | 2n 2 ch khi n nguyn t l khng chnh xc. Thng tin ny da trn trch dn ca J.H.Jeans
trong Dickson [7] tp 1 trang 91 v mt li dch thut. Xem NEEDHAM J., Science and Civilization in
China, tp 3: Mathematics and the Sciences of the Heavens and the Earth, Cambridge 1959 trang 54
ghi ch cui trang. Trch dn ti liu ca Dickson trang 196 ng ra l ti liu [7] tp 1 trang 91.
4. Bi bo [3] ca V.A.Demyanenko c trch dn trong trang 93 cha mt li nghim trng, do
nh l ny cha th coi l c chng minh. Tuy nhin nu x x y y z z th mi c s
nguyn t ca x l c s ca y hoc mi c s nguyn t ca y l c s ca x , xem SCHINZEl
A., Sur l'quation diophantienne x x y y z z , Acta Sc. Nat. Univ. Szechuansis 18 (1958), 81-83.

You might also like